Está en la página 1de 461

BANCO DE PREGUNTAS EXARMED

1. Cual de los siguientes factores contribuye los mayores determinantes en el


incremento de riesgo cardiovascular en los pacientes diabéticos?
a) Ateroesclerosis y neuropatía autonómica
2. Paciente masculino de 13 años que acude a consulta por dolor escrotal que ha
incrementado progresivamente, así como edema de la región. Al interrogatorio el
paciente describe que ha padecido disuria recientemente. A la exploración física de
este paciente se encuentra edematoso y eritematosos. A la palpación se prodice
dolor en la región posterolateral del testículo izquierdo. Cuando se realiza
elevación del testículo izquierdo, el dolor disminuye de intensidad. Cual de las
siguientes opciones representa el diagnostico mas probable en este paciente?
d) epididimitis

3. Cual de las siguientes opciones representa la variante mas frecuente de la


neuropatía diabética?
b) Polineuropatía simétrica distal
4. Cual es el tratamiento del efecto somogyi?
c) Disminuir la dosis de insulina precena
5. Se trata de un paciente masculino de 34 años de edad, se encuentra en el 5 dia de
hospitalización por pancreatitis severa, se le halla desorientado y disneuco, a la
exploración física con taquipnea, uso de musculatura accesoria, acrocianosis,
ambos hemitórax con disminución del murmullo vesicular y estertores de
predominio al final de la inspiración; la saturación por oximetría de pulso esta en
75% con puntas nasales a 3L/min (FiO2 30%) la gasometría mas recuente muestra
hipoxemia con PaO2 de 45 Kpa , por lo que requiere inicio de la ventilación
mecánica, FC 115 lpm, FR 28 rpm T 35.6 C PA 90/50 mmHg peso 89 kh talla 1.70m
5.a. Con los datos del padecimiento actual y la exploración física, usted integra el
diagnostico de:
d) síndrome de insuficiencia respiratoria aguda

5.b Basado en los criterios de la definición de Berlin, publicada en 2012, podemos


clasificar este cuadro como:
c) SIRA moderado

5.c Cuales son las fases del SIRA?


b) exudativa, proliferativa y fibrosa

5.d En el síndrome, estos son los principales mediadores proinflamatorios:


d) IL-6, IL-8 y TNF a

5.e Como se clasifica el SIRa según su etiología?


c) pulmonar y extrapulmonar
6. la principal diferencia en la fisiopatología de estado hiperglucemico hiperosmolar de la
Cetoacidosis diabética es:
c) la menor deficiencia de insulina

7. Paciente mas de 62 años de edad con diagnostico de diabetes mellitus tipo 2 de 13 años
de evolución con hipoglucemiantes orales, ultima glucemia central fue de hace un me en
143. Acude a urgencias por presentar dolor ocular intenso en ojo derecho, disminución de
la agudeza visual de 4 horas de evolución, además de nauseas y vomito. A la exploración
oftalmológica presenta una agudeza visual de movimiento de manos en ojo derecho 20/80
en ojo izquiero. A la biomicroscopia se encuentra en ojo derecho hiperemia conjuntival e
inyección ciliar, cornea edematosa con presencia de microbulas subepiteliales, reacción
inflamatoria en cámara anterior, la cual se observa poco profunda, pupila midriática,
hiporreflecticas, cristalino con catarata. Presión intraocular de 58mmHg. La gonioscopia
muestra un angulo completamente cerrado, el fondo de ojo no es valorable por la
opacidad de medios. Ojo izquiedo cornea transparente, cámara anterior poco
moderadamente abiertos. Cual es el diagnostico mas probable?
c) glaucoma agudo por bloqueo pupilar

8. paciente femenino de 27 años de edad, sin antecedentes relevantes, quien curso un


embarazo gemelar normoevolutivo y a termino, inicio trabajo de parto y fue atendida por
un parto gemelar eutoxico con un sangrado estimado de 450 ml. Una hora después la
paciente presenta taquicardia, taquipnea, palidez y sangrado vaginal 90/60mmHg m peso
65 kg talla 1.62m
8.a Con los datos anteriores cual es el diagnostico etiológico mas frecuente?
d) atonía uterina

8.b la hemorragia posparto es favorecida por varios factores de riesgo, cual de los
siguientes no es una causa intrínseca?
c) uso de tocoliticos

8.c una vez identificada la hemorragia por atonía uterina, cual es el tratamiento
farmacológico de primera línea?
b) oxitocina en infusión
8.d si el sangrado no es controlado con fármacos, que tratamiento farmacológico es el de
primera elección?
b) dilatación y legrado (raspado)
8.e, existen otras causas de sangrado posparto, cual de las siguientes no es traumatica?
d. acretismo placentario

9. cual es la causa mas probable de dolor de rodillas en corredores?


a) síndrome patelofemoral

10. paciente masculino de 65 ños de edad con factores de riesgo cardiovascular se


presenta a la sala de urgencias con hemiplejia derecha, asi como paralisis facial derecha
con preservación de los movimientos de la mitad derecha de la frente. El paciente
también muestra dificultad para la comunicación oral. El paciente se encuentra alerta con
frecuencia cardiaca de 100 latidos por minuto y presión arterial de 175/115mmHg, sin
dificultad ventilatoria y con una cifra de glucosa capilar de 130, los hallazgos a la
exploración del fondo de ojo son normales. Los familiares refieren que el cuadro inicio
hace aproximadamente una hora. Una tomografía computarizada cerebral resulta
normal.cual de los siguientes medicamentos es el tratamiento de elección inicial para esta
paciente?
a) labetalol

11. masculino de 35 años acude a consulta debido a que su esposa le diagnosticaron


gastritis por H. pylorien el chequeo anual de la oficina. El paciente no presenta molestias
al comer, ni toma medicamentos, pero quiere estar seguro de no tener esa enfermedad.
11.a porcentaje de ulceras duodenales que presentan infección por H. pylori:
d) 90%

11.b se considera como la prueba de tamizaje para H. pylori.


c) prueba de aliento

11.c. es el estandarde oro para el diagnostico de infección por H. pylori


b) histología

11.d cual es el tratamiento de primera elección para la enfermedad acido pepticausada


por H. pylori?
c) lansoprazol + claritromicina 500 mg + amoxicilina 500 mg; 2x dia

11.e con el tratamiento adecuado, el índice de erradicación es de alrededor de:


d) 85%

12. el estudio inicial mas valioso en el diagnostico diferencial de hipertiroidismo es:


e) estudio de medicina nuclear, gammagrafía

13. ingresa a la sala de urgencias un paciente masculino de 4 meses de edad, llevado por
sus padres debido a somnolencia excesiva. Al interrogatorio los padres mencnionan que
desde que nacio ha sufrido un bebe inactivo, pero recientemente ha mostrado
disminución en el estado de alerta, anorexia y flacidez. A la exploración física se observan
signos vitales dentro del rango de la normalidad; en cuanto a su desarrollo psicomotor el
paciente es incapaz de mantener su cabeza, y cuando se colora un dedo en la palma de su
mano no hay respuesta; el reflejo de succion es débil; a la exploración neugologica el
paciente no presenta expresión facial o estudio de laboratorios son relevantes
únicamente por acidosis metabolica de brecha anionica elevada, incremento en los niveles
de lactato y piruvaro. Cual es el diagnostico mas probable en este paciente?
c) deficiencia de piruvato deshidrogenasa
14. paciente masculino de 22 años de edad cuya principal molestia es cefalea persistente
con resistencia a los analgésicos convencionales. A la exploración física presetna
incapacidad para dirigir la mirada hacia arriba y dilatación pupilar producida por la
acomodación, pero no por la aplicación de un estimulo luminoso. La realización de un
estudio de imagen revela hidrocefalia de tipo no cominicante. Cual de las siguientes
condiciones causa mas probable de este transtorno?
a) tumor de glandula pineal

15. acude a consulta pediátrica un paciente de 2 años de edad debido a que la madre ha
notado la presencia de heces de color marron. Al interrogatorio, se niega cualquier
antecedente pediátrico y obstétrico de importancia. La exploración física no es relevante
(abdomen blando, no doloroso, no distendido, sin signos de irritación peritoneal). El
pediatra decide realizar un estudio de medicina nuclear con tecnecio 99 el cual demuestra
la presencia de mucosa gástrica ectópica en el cuadrante abdominal inferior derecho. Cual
es el diagnostico mas probable en este paciente?
d) divertículo de Meckel

16. un paciente de 50 años de edad se presenta a la sala de urgencias debido a dolor


intenso a la abducción de su brazo después de caer de una bicicleta. El paciente describe
que cayo con el brazo extendido. El paciente diagnostica con un desgarro del manguito
rotador. Cual de los siguientes es la localización mas frecuente de los desgarros del
manguito rotador?
a) musulo supraespinoso

17. masculino de 75 años, diabético desde hace 15 años en tratamiento con insulina,
acude a consulta por tener dificultad para iniciar la micción, además de sentir que no vacia
completamiente la vejiga y gotea orina al terminar , la molestia empezó hace algunas
semanas y no refiere alguna otra. Al interrogarlo comenta que desde hace un par de
meses presenta nicturia, aumento en la frecuencia urinaria- la cual le atribuye a la
diabetes- y disminución en el calibre del chorro urinario. EF 1.68 mts Ta 140/80 temp 36.5
17. b al realizar el catro rectal usted nota una próstata lisa, firme y elástica. Usted
diagnostica:
b) hipoplasia prostática benigna

17. c fase de la enfermedad que se caracteriza por el vencimiento del destrusor vesical?
c) descompensación

17.e el manejo definitico es:


c) quirúrgico

18. un paciente de 6 años de edad se presenta a la sala de urgencias por nausea, vomito e
incapacidad de pasar gases intestinales. A la EF abdomen se encuentra distenndido con
ausencia de ruidos intestinales. Se decide realizar un estidio de medicina nuclear con
tecnecio 99, el cual demuestra captación ectópica en el ileon distal. Cual de las siguientes
arterias proporciona la irrigación a esta mucosa con captación ectópica de tecnecio 99?
a) arteria mesentérica superior

19. paciente fem de 19 años con antec de hospitalización frecuentes por intoxicación
alcohólica. En esta ocasión es ingresada después de haber estado ingiriendo de manera
constante bebidas alcoholicas desde hace 5 dias. A la EF TA 110/70 mmhg, FC 80, peso 70,
se presenta letárgica y con habla incoherente en la sala de urgencias presenta una crisis
convulsiva tonicoclonica, la cual resuelve posterior a la administración de daizepam
intravenosos. Los estudios de laboratorio son significativos por un niveles serico de sodio
de 110 mEq/L, potasio serico de 3.8 mEq/ L, nivel de alcohol 20, osmolaridad sérica 230 y
concentración de glucosa 92, no hay alteraciones en la función renal. Despies de iniciar
tratamiento con soluciones hipertónicas, el sodio serico se encuentra en 120, la paciente
ahora se encuentra euvolemica y con mejor respuesta a estimulos. Cual de las siguientes
opciones representa el mejor manejo para la paciente en este momento?
a) detener la infusión de liquidos, restringir la ingesta de agua, monitorización del sodio
serico

20. paciente fem de 47 años, acude a urgencias por dolor abdominal. Refiere que el dolor
empezó hace 4 horas después de comer en un puesto de birria de la calle; lo describe
como muy intenso 8/10 de inicio subido en región central del abdomen y se le recorre
hacia la espalda. Desde que llego al servicio ha vomitado en 2 ocasiones. A EF se
encuentra ansiosa, con fascies de dolor y piernas flexionadas sobre el abdomen; se
observa un hematoma en la zona del ombligo, hiperalgesia e hiperbalgesia con ruidos
intestinales disminuidos TA 140/85 FC 100 FR 24 temp 38.5

20.a Cual es el principal sospecha diasnostica?


a) pancreatitis aguda

20. b que alteraciones en los laboratorios confirmarían su diagnostico?


c) aumento de lipasa y amilasas (>3 veces)

20. c En cual de las siguientes opciones se podría observar un aumento o una disminución
del valor de la amilasa?
c) cáncer de páncreas

20. d Para valorar la gravedad, usted utiliza la escada de Ranson y obtiene un valor de 6
puntos, y al utilizar la escala APACHEII obtiene un valor de 8. Como se interpretarían estos
resultados, respectivamente?
a) mortalidad de 40% y pancreatitis grave

21. paciente masculino de 65 años con ant de tabaquismo, refiere padecer dificultar para
respirar al caminar por las mañanas desde hace 3 años por lo que mantiene una vida
sedentaria. Acide a consulta por tos intermitente desde hace 3 meses. Niega dolor
torácico. A la auscultación se detecta disminución del murmullo vesicular. La radiografia
del torax reporta aplanamiento diafragminatico, con disminución del trama vasculas y
silueta cardiaca alargada:
21.a Cual es el punto de corte en la espirometria para el diagnostico de EPOC?
b) FEV1/ FVC <70%
21.B Esquema farmacológico indicaco en este paciente
a) tiotropio + salbutamol/ ipratropio
21. c Cuales son las medidas que han demostrado disminución la mortalidad en la EPOC?
b) Abandono del tabaco- oxigenoterapia
21. d Señale los criterios de Winnipeg que definen exacerbación de la EPOC
d) aumento de disnea´aumento de expectoración+ purulencia del esputo
21.e cual es el tratamiento indicaco ante una exacerbación de la EPOC de causa
infecciosa?
c) amoxicilina + acido clavulanico

22. cual es el tratameinto incial mas adecuado en un paciente que presenta


hiperpotasemia grave asociada con cambios electrocardiográficos ( ondas T acuminadas,
ensanchamiento del complejo QRS)?
c) administración de gluconato de calcio intravenosos

23. paciente masculino de 37 años de edad, diseñador grafico, homosexual, con infección
por VIH diagnosticada hace 2 años acude al servicio de urgencias por presentar fiebre de
15 dias de evolcion, ataque al estado general, perdida de peso, disnea progresiva de
esfuerzo y tos con espectoracion amarillenta. Se realizo tinción de la expectoración, que
reporto bacilos acido -albohol resistente. Su cuenta de CD4 realizada hace un mes fue de
100celulas / ML y una carga viral recientemente de 250 00 copias.
Cual de las siguientes aseveraciones en relación a la tuberculosis en el paciente con sIDA
es correcta?
d) El tratamiento para pacientes VIH positivos con tuberculosos debe ser el
mismo que el que se utiliza para los pacientes no infectados con VIH

24. paci02ente masculino de 59 años de edad que ingresa a la unidad coronaria después
de presentar dolor torácico por 14 horas. En este momento el paciente se encuentra sin
dolor. El electrocardiograma en sala de urgencias presentaba elevación del segmento ST
de 1mm con inversión de la onda T y ondas Q patológicas en las derivaciones II, III y a VF.
Después de 48 horas desde su ingreso se puede percibir un soplo telisistolico en el ápex; a
las 72 horas el soplo ha incrementado en intensidad, describiéndose como 3/6. El paciente
se muestra disneico, y la radiografía de torax muestra redistribución de la vasculatura
pulmonar. Cual es la explicación mas probable para la sintomatología de este paciente?
c) infarto de musculo papilar posterior

25. Neonato masculino de 21 dias de nacido es internado por vomito persistente de 12


dias de evolución. Peso al nacimiento de 295g antecedentes: paciente aparentemente
sano desde el nacimiento hasta el 91 dia de vida; en u principio se alimento solo con seno
materno, pero al 81 dia inicio con formula. El vomito inicio en el 10 º dia de vida y persistio
a pesar de retirar la alimentación con formula. En el dia 21 se le hospitalizo; no se
menciona episodios de diarrea, incluso en días previos al internamiento sus deposiciones
se tornaron de consistencia mas solida e infrecuentes. A la exploración física se palpa
fontanela deprimida, mucosas secas y la madre no recuerda la ultima vez que mojo el
pañal. Acude actualmente con peso de 2850 g FC 152 FR 30 temp 37.5, no se observan
otras anormalidades en la exploración.
25. a con base en los datos proporcionados. Cual es la prueba diagnostica que confirmaría
la sospecha clínica?
c) ultrasonido abdominal

25.b que hallazgos se encontraran en la gasometría arterial?


d) alcalosis metabolica

25.c con los siguientes hallazgos en la exploración física apoyaría el diagnostico de este
paciente?
b) ondas peristálticas tras la toma

25.d con base en las características del paciente. Cual es el diagnostico mas probable?
a) estenosis pilórica

25.e. con base en la sospecha diagnostica. Cual es el tratamiento indicado?


b) pilorotomia

26. paciente masculino de 30 años de edad, fumador, resulto involucrado en un choque


automovilístico hace 2 horas. Los paramédicos descartaron en el lugar de lesiones
mayores. Acude por dolor en la rodilla derecha y comenta que en el choque esta golpeo
con el tablero. EF: presenta dificultad para caminar, dolor y edema en la región de la
rodilla derecha; la tibia se desplaza posteriormente rebasando los cóndilos femorales.
26. a Que prueba elegiría para explorar los ligamentos cruzados?
d) cajón anterior y posterior

26.b que evalua la prueba del cajón posterior?


a) LCP

26.c Cual es la técnica mas sensible para evaluar la integridad de los ligamentos cruzados?
a) RMN
26.d una lesión grado 2(según la prueba del cajón significa que:
c) la tibia puede subluxarse en una línea igual al aspecto anterior de los cóndilos femorales

26. el manejo iniciado de las lesiones de los ligamentos cruzados es:


a) rehabilitación y manejo del dolor
27. Masculino de 65 años con diagnostico de HTA desde hace 15 años (en tratamiento con
captopril) e hipermetropía, acude a urgencias por haber presentado el dia de hoy por la
tarde un episodio de dolor en el ojo izquierdo y cefalea en hemicraneo del mismo lado,
acompañado con blefaroespasmo, lagrimeo e incluso nauseas. El episodio duro cerca de
15 min después cedió paulatinamente; comenta que ya había tenido antes eventos
similares, todos por la tarde, pero con menor intesidad. No refiere perdida de la visión,
solo aclara que las luces las observa con un halo. EF: 1.68, 1.65 cm TA 140/80 mmHg temp
36.5 . a la exploración se encuentra la cornea con un aspecto turbio e inyección mixta.
27.a cual es la principal sospecha diagnostica?
b) glaucoma de angulo cerrado

27. c eventos patológicos que ocurren (en orden) al aumento la PIO:


a) edema corneal a alteración de la visión a dolor intenso a signos vagales a colapso de
arteria retiniana

28. paciente masculino de 22 meses es traido a urgencias por su madre, quien refiere que
esta mañana al cambiar el pañal del paciente lo encontró con sangrado abundante
proveniente del recto. El paciente se observa tranquilo, no manifiesta dolor. A la
exploración física se encuentra sin dolor a la palpación abdominal, sin datos de sangrado
activo y sus signos vitales en parámetros normales.
Cual es el diagnostico mas probable?
e) Divertículo de Meckel

29. un niño de siete años es llevado a consulta por retraso en el crecimiento. Al


interrogarlo se identifican otros síntomas, como cefalea, vomito, disminución de la
agudeza visual, poliuria y polidpsia. En estudio de imagen se demuestra la presencia de
tumor de silla turca.
29. a Cual es el tumor que causa hipopituitarismo con mas frecuencia en niños?
c) craneofaringioma

29.b cual es el manejo mas adecuado?


a) resección quirúrgica

30. paciente masculino de 70 años de edad conocido por el servicio de cardiología porque
acude desde hace tres años por cuadros de angina estable. En ocasiones asiste al servicio
de urgencias por presentar dolor torácico intenso. La enfermera toma un
electocardiograma, el cual revela elevación del segmento ST; las pruebas enzimáticas
rapidas determinan elevación de la creatinicinasa-MB. El paciente es estabilizado y
admitido a la unidad coronaria. Varios días después de deste evento presenta de nuevo
dolor torácico. A la exploración física presenta fiebre, frote pericárdico intenso, estertores
pulmonares y edema de extremidades inferiores. Cual de los siguientes diagnosticos es el
mas probable en este paciente?
d) pericarditis fibrinoide
31. de los métodos diagnosticos presentados a continuación, cual es el mas adecuado para
realizar el diagnostico de embolia pulmonar en un paciente con un proceso neumónico
subyacente?
e) tomografía computarizada espiral constrastada del torax

32. paciente masculino de 44 años de edad que presenta a consulta por intensos dolores
cólicos, constipación, irritabilidad y cefalea. El paciente trabaja en una fabrica de pintura
en aerosol. Presenta unos estudios de laboratorio que el medico laboral le solicito; la
principal característica de estos es una anemia microcitica. Al revisar el expediente medico
el paciente ha presentado episodios depresivos mayores en el pasado. Cual de las
siguientes sustancias es con mayor probabilidad la causante lasintomatologia de este
sujeto?
e) plomo

33. paciente masculino de tres meses de edad que acude a consulta pediátrica de rutina.
En la evaluación es evidente que padece leucocoria. Despies de evaluación oftamologica
se determinara que la leucocoria es debida a un retinoblastoma. Se realiza enucleación del
ojo izquierdo. Cual de las siguientes estructuras del ojo extraido es la que merece especial
atención por el patólogo porque determina el pronostico?
c) nervio óptico

34. paciente masculino de 19 años es llevado al servicio de urgencias por disnea grave y
dolor localizado a la mitad derecha del torax. Los familiares comentan que la
sintomatología inicia hace mas o menos dos horas sin relación con alguna actividad o
evento. Al interrogarosio el paciente comenta que el dolor es mas intenso durante la fase
inspiratoria, y niega la presencia de fiebre o cualquier otro síntoma que puede orientar al
diagnostico. Entre los antecedentes de importancia esta el tabaquismo desde los 17 años
a razón de 15 cigarros al dia. A la exploración física revela disminución de la intensidad de
los sonidos respiratorios en el pulmón derecho con timpanismo a la percusión. La
saturación de oxigeno sin administración de oxigeno suplementario es de 96%. La
radiografia de torax muestra la presencia de neumotórax en 50% del pulmón derecho.
Cual de las siguientes conductas terapéuticas es la mas adecuada en este paciente?
d) aspiración con aguja del neumotórax

35. cual de los siguientes antiarritmicos actua tanto en la fase 0 como en la fase 3 del
potencial de acción?
e) disopiramida

36. paciente masculino de 35 años de edad, bajo tratamiento por depresión mayor, se
presenta al servicio de urgencias por presentar confusión intensa y alucinaciones. A la
exploración física el paciente es poco cooperador, pero se observa que la boca esta seca y
la cara eritematosa. La enfermera toma los signos vitales, resultando en hipotensión
(85/50) y taquicardia 110 latidos. Se realiza un electrocardiograma, el cual revela
prolongación del intervalo QT (0.13s) y contracciones ventriculares prematuras. Cual de
los siguientes medicamentos es el mas adecuado para este paciente?
e) bicarbonato de sodio

37. cual de las siguientes características posee la mola hiatidiforme completa?


a) el genotipo de las células de la mola es 46 xx, y es completamiente de origen materno

38. paciente masculino de 18 años de edad que decide practicar alpinismo. No cuenta con
ningún antecedente de importancia. Al llegar a la cima el paciente inicia con disnea y
estertores difusos bilaterales. Cual es el diagnostico mas probable en este paciente?
a) edema pulmonar de grandes altitudes

39. cual de las siguientes interacciones farmacológicas disminuye el efecto anticoagulante


de la warfarina?
b) fenitoina

40. paciente masculino de 69 años de edad que se presenta a consulta con perdida de
peso no intenciada de aproximadamente 13 kh, disnea durante el ejercicio y tos
productiva acompañada de sangre desde hace 2 meses. Entre los antecedentes de
importancia de este paciente se encuentra el antecedente de tabaquismo intenso desde
los 15 años de edad, a razón de una cajetilla diaria. La radiografia de torax demuestra una
tumoración en el lóbulo pulmonar superior del pulmón derecho, se decide realizar
tomografía computarizada la cual revela una tumoración de 5cm de diámetro en el lóbulo
superior acompañada de adenomegalias mediastinales derechas. La realización de una
tomografía por emisión de positrones revela tres ganglios mediastinales del lado derecho
y uno del izquierdo. Cual es el siguiente paso mas adecuado en el manejo de este
paciente?
a) biopsia de los ganglios mediastinales

41. cual de los siguientes transtornos se presentan con tiempo de sangrado en rangos
normales?
d) purpura de Henoch-Schonlein

42. cual de los siguientes fármacos se asocian con respuestas puramente beta-1 y beta-2
adrenergicas?
d) dobutamina

43. cual de los siguientes medicamentos es una benzodiacepina con bajo riesgo de
producir síndrome de abstinencia?
a) clorodiazepoxido

44. cual es la principal causa de muerte en este pacientes con enfermedad de Kawasaki?
b) secuelas cardiacas
45. la mutacion en el gen de la cistationa beta-sintasa ocasiona gran variabilidad
fenotípica, incluyendo alteraciones musculoesqueleticas, retraso mental y trombosis. Cual
es el nombre de esta propiedad de variabilidad fenotípica?
b) peliotropia

46. cual de los siguientes organismos se asocia con el desarrollo de colangiocarcinoma?


e) clonorchis sinensis

47. cual es el mecasnismo de acción por el que la carbidopa puede mejorar la


sintomatología de la enfermedad de parkison?
c) incremento de la biodisponibilidad de la dopamina por medio de la inhibición de la
enzima dopamina descarboxilasa

48 cual de los siguientes receptores es al que se une con mas afinidad la naloxona?
c) Mu

49. cual de los siguientes medicamentos se asocia con síndrome nefrótico como efecto
adverso?
c) penicilamina

50. cual de los siguientes trastornos se caracteriza por niveles disminuidos de la hormona
folículoestimulante, luteinizante y testosterona en combinación con bajo conteo
espermatozoides?
d) hipogonadismo central

51. paciente femenino de 75 años de edad operada de cirugía de recambio total de


cadera. Después de unos días de hospitalización es egresada a su domicilio con varios
medicamentos, entre ellos ketorolaco. Alrededor de 48 horas después a su egreso percibe
una reducción significativa de su gasto urinario y malestar general, por lo que sus
familiares deciden regresar al hospital. En el servicio de urgencias se le practica estudios
de laboratorio que revelan principalmente incremento en la concentración de nitrógeno
ureico en sangre (66mg/dl) y de la creatinina (6.7 mg/dl). Cual de los siguientes
mecanismos es el responsable de la insuficiencia renal aguda de esta paciente?
a) la administración de ketorolaco ocasiono inhibición de la síntesis de prostaglandinas y
constriccion de las arteriolas aferentes en el riñon

52. Paciente masculino de 27 años de edad que muestra disuria y secreción purulenta
uretral. Es tratado con ceftriaxona intramuscular, dosis única, pero los síntomas no
mejoran. Cual de los siguientes medicamentos es mas probable que resuelvan la
sintomatología de este paciente?
e) azitromicina

53. paciente masculino de 55 años de edad con diagnostico de infección por el virus de la
inamunodeficiencia humana y con síndrome de inmunodeficiencia adquirida. El motivo
principal de consulta es que el paciente ha presentado diarrea, fiebre y perdida de peso
desde hace 2 semanas. Cual de los siguientes estudios diagnosticos determinaría la
presencia de una colitis por citomegalovirus?
a) colonoscopia con toma de biopsia

54. granjero de 75 años casado, con antecedentes de síndrome del seno enfermo tratado
con marcapasos y warfarina, dos episodios de evento vascular cerebral, hipertensión y
gota tratada conc alopurinol. Toma enalapril, verapamil, albuterol y digoxina. La
evaluación de laboratorio revelo hipocalciuria e hiperxaluria extrema. Fue sometido a
extracción uretroscopica de dos cálculos de oxalato de calcio. En relación al citrato
potásico, señale la respuesta correcta:
c) proporciona un aumento de la alcalinidad de la orina

55. se presenta a consulta pediátrico un paciente del sexo masculino de 2 años de edad
A persar de que el paciente había presnetado desarrollo y crecimiento normales en el
primer semestre de vida, en esta ocasión se presenta con un claro retraso del desarrollo. A
la exploración física es evidente que el paciente no responde a estimulos visuales, y
durante la oftalmoscopia se observa una mancha rojo cereza en la macula. Cual de los
siguientes procesos es el que se encuentra alterado en esta paciente?
f) Alteración del metabolismo de los glucolipidos en el cerebro

56. paciente masculino de 77 años de edad ingresa en la madrugada por exacerbación de


insuficiencia cardiaca. A la ef presenta dismnea, taquipnea y estertores basales. Un
residente de medicina interna x poco experimentado admite al paciente y escribe las
indicaciones para el personal de enfermería. Al dia siguiente el paciente empieza a
empeorarado. Durante el paso de visita el medico adscrito revisa las indicaciones del
residente y nota que este ordeno la administración de un diurético que fie responsable del
emperamiento. Cual de los siguientes diuréticos es el que con mayor probabilidad se le
administro a este paciente?
c) manitol

57. cual de las siguientes arterias puede sufrir hemorragia en caso de ulcera perforada en
la porción posterior de la primera parte del duodeno?
a) arteria gastroduodenal

58. cual es la conducta mas adecuada a seguir en el caso de un paciente que después de
un mes de completar adecuadamente un esquema de erradicación de Helicobacter pylori
aun continua con dispepsia y dolor epigástrico después de ingerir alimentos?
a) realizar un estudio de ureasa en aliento

59. paciente masculino de 45 años, sin antecedentes personales patológicos, ingresa a


urgencias a causa de un episodio de dolor agudo en el flanco derecho y hematuria. El
dolor comenzó por la noche e incluso lo despertó. A la ef: Giordano derecho se solicito
laboratorios de rutina que revelan calcio y fosforo en rangos normales, el cultivo de orina
negativo y hematuria. La radiografia simple de abdomen descubre un lito en riñon
derecho. El urograma intravenoso muestra que el calculo no esta obstruyendo en este
momento. Que tipo de lito es mas probable que haya causado esta condición en el
paciente?
a) oxalato de calcio

60. un hombre de 46 años de edad se presenta a consulta por dolor en articulación del
codo izquierdo, el cual se incrementa durante las horas de trabajo ( es plomero). El
paciente acude porque el dolor se ha hecho tan intenso al grado que solo levantar un
tornillo o clavo ocasiona dolor. A la ef refiere dolor en el epicondilo lateral y se resiste a la
extensión de la muleca por el dolor que produce. Cual es el diagnostico mas probable en
este paciente?
b) epicondilitis lateral

61. se presenta a la consulta externa de pediatría un paciente del sexo masculino de 13


años de edad el cual manifiesta hematomas en varias regiones del cuerpo. A la
exploración física se puede observar que presenta piel suave hiperelasticidad, asi como
equimosis en la pared posterior del antebrazo y en las regiones tibiales. Se decide realizar
una bipsia de piel y enviarla a evaluación por microscopia electrónica. El reporte menciona
que existe fibrillas de colágena extracelular anormalmente delegadas e irregulares.
Evaluacion bioquímica posterior revela un numero incrementado de puentes de disulfuro
en las regiones terminales. Cual de las siguientes fases de la síntesis de la colágena se
encuentra afectada en este paciente?
a) segmentación extracelular

62. cual de los siguientes parámetros NO es utilizado como factor pronostico en pacientes
con pancreatitis aguda en los criteriso de Ranson?
b) hiperbilirrubinemia

63. cual de los siguientes medicamentos ocasiona anemia megaloblastica como efecto
secundario?
e) trimetoprim-sulfametoxazol

64. paciente masculino de 36 años que acude a consulta por comenzar con escapes
urinarios al mes de practicarse una cesarea por desproporcion cefalopelvica. Le han
realizado intervension de fistula sacrococcigea. Antecedentes obstétricos se cesarea en
1990 y un parto distócico mediante fórceps por expulsivo prolongado en 1994. Recibió
tratamiento de una infección del tracto urinario con posterior urocultivo negativo. Al
interrogatorio dirigido refiere escapes continuis de orina clara que se desencadenan con la
bipedestación y que aumentan con los esfuerzos, sin sintomatología de urgencia miccional
acompañante,en la ef cuello uterino de multípara bien epitelizado, utero en
anteroversion, zonas axilares libres de masas. La sensibilidad vulvar y los reflejos
perineales (anal y bulvocavernoso) están conservados. Tras realizar prueba de
incontinencia con azul metileno, se observan a través del orificio cervical externo fluir
liquido teñido de azul de forma espontanea, que aumenta con maniobras de valsalva, no
apreciándose escapes a través de uretra. Se solicitan una cistografía en la que se
comprueba, con vejiga en máxima repleción, el paso de contraste a través de un trayecto
fistuloso a cavidad uterina. Diagnostico fistula vesiculouterino, se programa para
fistulorrafia vesicouterina. Con respecto caso de incontinecia urinaria de esfuerzo, todas
las aseveraciones son ciertas, EXCEPTO:
e) es poco frecuente en mujeres posmenopáusicas o multíparas

65. según la clasificación de fracturas del pratillo tibial de schatzker, cual de las siguientes
opciones representan el tipo de fractura del platillo tibial mas frecuente?
b) tipo II

66. Cual de las siguientes deficiencias vitamínicas presentes sintomatologías posterior a


varios años de haber suspendido su ingesta?
d) vitamina B12

67. Niña de 18 meses de edad que presenta dermatitis de la zona del pañal desde hace
tres semanas. La han tratado con talco y quadridem por dos semanas sin observar
mejoría. A la exploración física, aparente de observación eritematosa, erosiones, fisuras y
costras serohematicas, nota presencia de papulas eritematosas satélites que se extienden
hasta la región inguinal y tercio superior de la cara interna de los muslos. Cual es el agente
invasor mas frecuente de estos casos?
b) candida albicans

68. femenino de 62 años llevado a sala de urgencias por experimentar episodios


frecuentes de ptosis, diplopía y fatiga generalizada. EF: se encuentra timo palpable y
paralisis del nervio oculomotor que se corrige transitoriamente al administrar edrofonio.
La entidad nosológica responsable de la sintomatología de este paciente es?
a) miastenia grave

69. un paciente bajo tratamiento antipsicótico por esquizofrenia acude a la consulta por
exceso de rigidez muscular, hipertermia y disminución de la sudoración. A su infreso el
paciente presenta cambios bruscos en las cifras tensionales, acompañados de episodios
de taquicardia y bradicardia. Cual de los siquientes medicamentos se debe administrar
inmediatamente a este paciente?
d) dantroleno

70. paciente masculino de 59 años de edad que acude a evaluación por padecer disnea de
medianos esfuerzos dolor torácico. En el expediente clínico no hay ningún antecedente de
importancia para este padecimiento. A la exploración física los pulsos de las carótidas se
encuentran retrasados respecto al ciclo cardiaco, asi como disminuidos en intensidad. El
punto de máxima intensidad cardiaco muestra hiperdinamico, pero sin desplazamiento. A
la auscultación se encuentra un soplo sistólico en crescendo. Decrescendo a lo largo del
borde esternal izquierdo. El componente aórtico del segundo ruido cardiaco se encuentra
ausente. En el electrocardiograma se encuentra datos en relación a hipertrofia ventricular
izquierda. La radiogradia de torax no muestra incremento del tamaño de la siluieta
cardiaca y campos pulmonares sin alteraciones. Cual es el tratamiento mas adecuado en
este paciente?
b) angiografía coronaria y después cirugía

71. hombre de 32 años, previamente sano invia con fiebre de 38, cefalea y maletar
general, tomando solo tratamiento sintomático. Al dia siguiente por persistencia del
cuadro, acude al servicio de urgencia,donde se encuentra con Glasgow de 14, febril con
rigidez de nuca y signo de brudzinsky postivo
71.a Dado su sospecha dignostica, decide realizar una puncion lumbar. Cual de las
siguientes representa una contraindicación absoluta para realizar este procedimiento?
a) infección de la piel en el sitio de la puncion

71b. el resultado del liquido cefalorraquídeo indica 2700cel con 84% de


polimorfonucleares, glucosa de 36 (sérica de 94) y proteínas de 175 mg/dl. Según su
sospecha clínica y estos resultados, el agente causal mas probable en estos caso es:
b) streptococcus pneumoniae

71.c dicho microorganismo es:


d) diplococo grampositivo

71d. el tratamiento empírico que se debe proporcionar en este caso es:


a) Cefotaxima + vancomicina
71.e el uso de corticoesteroides en este caso:
d) se recomienda administrar antes de la primera dosis de antibióticos y hasta cuatro días
del inicio durante el tratamiento

72. con relación al ataque agudo de gota:


a) hasta 30% de los pacientes puede tener durante el ataque agudo acido urico normal

73. neonato de seis horas de vida cuya madre fue positiva para antígeno de superficie de
hepatitis B (HBsAg). Cual es la conducta mas adecuada en relación a profilaxis con
hepatitis B?
c) administrar la primera dosis de vacuna anti-hepatitis B e inmunoglobulina de hepatitis B
antes de la primeras 12 horas de vida

74. se presenta a la sala de urgencias masculino de 54 años, refiere dolor intenso en la


extremidad pélvica derecha con evolución de 2 horas. Antecedentes: cardiopatía
isquémica hipertensiva, ef: angustiado, con fascies de dolor TA 155/90 FC 120 FR 18 T
36.5, soplo carotideo bilateral, ruidoscardiacos arrítmicos, con buena intensidad,
abdomen se ausculta soplo sistólico en mesogastrio, extremidades toracidas con pulsos
normales hasta distales, extremidad pélvica izquierda con pulso femoral con soplo
sistólico, pulso femoral, poplíteo ni tibiales, palida hipotérmica, con sensibilidad
disminuida, con movimientos normales de los dedos y la articulación del tobillo.
74.a Causa principal de la sintomatología de este paciente:
c) insuficiencia arterial crónica agudizada

74b. La base fisiopatológica de esta enfermedad es:


a) Desarrollo de áreas de acinesia cardiaca secundarias a isquiemia

74c. según las características clínicas del paciente la conclusión diagnostica es:
d) isquemia clase II Rutherford

74d. medida terapéutica inmediata en este paciente es:


a) Heparinizacion y embolectomía femoral

74e. complicación frecuente posterior al tratamiento en este paciente:


d) síndrome de reperfusion

75. paciente femenino de 88 de edad es hospitalizada por una caída en su casa debido a
tropezón. No puede caminar o soportar peso sobre su extremidad inferior derecha. Las
radiografías revelan que tiene una fractura desplazada del cuello del femur, sin demostrar
algún otro hallazgo de importancia. Al interrogarorio niega cualquier otro traumatismo en
la región afectada. Posterior a la estabilización de este paciente. Cual es el tratamiento
mas optimo para su tipo de fractura?
d)hemiartroplastia

76. paciente femenino de 25 años fumadora, gesta 2 para 2 embarazos y parto sin
complicaciones, sexualmente activa ( dos parejas en este ultimo mes) acude por presentar
desde hace 3 dias secrecicon vaginal maloliente con prurito genital level. A la exploración
física muestra FC 78 Fr 18 temp 32.8, abdomen no doloroso, sin datos de irritación
peritoneal, vagina con secreción amarillo verdoso espumosa, maloliente paredes vaginales
eritematosos y cuello uterino con puntilleo rojo-violaceo. Resto de la exploración física sin
alteracicones.
76ª. Cual es la etiología mas probable?
b) Trichomona vaginalis

76b. Cual es el tratamiento mas adecuado?


c) Metronidazol a la paciente y a su pareja

77. paciente femenina de 60 años de edad con diabetes mellitus tipo 2 de larga evolución
e hipertensión. Acude a consulta se seguimiento por haber presentado proteinuria en el
examen de orina hace 12 meses. Sus medicamentos incluyen metformina, metropolol y
IECA. Refiere debilidad muscular desde hace una semana y edema. Filtrado glomerular
estimado 55ML/min/ 1.73 m2.
77ª. Cual de las siguientes afiermaciones es verdadera de acuerdo con la definición de
enfermedad renal crónica?
b) Las alteraciones estructurales o funcionales renales deben estar presentes >3
meses

77c. Respecto al tratamiento de la hiperpotasemia, señale la medida indicada en el


paciente para disminuir el riesgo de arritmia mortal y paro cardiaco:
c) Gluconato de calcio

77d. Señale la causa principal de la anemia en los pacientes con enfermedad renal crónica:
d) Déficit de etritropoyetina

77e. Cual de las siguientes anormalidades del metabolismo de la hormona paratiroidea


provoca la enfermedad renal crónica?
d) Hiperparatiroidismo secundario
78. pacientes masculino de tres años de edad, con antecedente de otalgia y otorrea
derecha, desde los dos años de edad, en cuatro ocasiones las cuales remitieron con
manejo antimicrobiano. Refiere la madre cuadro de vías respiratorias superiores de 48
horas de evolución; acude por otalgia derecha. Ef: temp 38, otoscopia derecha con
membrana timpánica hiperemica, no móvil. Otoscopia izquierda con membrana timpánica
opaca y no móvil. Cavidad oral con amígdalas grado 3m cripticas sin exudados. Cuello sin
adenomegalais

78ª. El diagnostico mas probable es:


b) Otitis media aduga recurrente
78 b. El microorganismo con mas frecuencia implicado en esta patología:
c) streptococcus pneumoniae

78c. El tratamiento cosisten en:


c) amoxicilina

78d. Dos meses después el paciente es levando a consulta porque presenta sensación de
plenitud aural y la madre refiere que escucha el televisor con un volumen mas alto del
habitual. En la otoscopia se observan las membranas timpánicas integras, opacas y no
móviles. Usted solicita una impedanciometria que reporta dosminucion en la complianza
de la membrana timpánica. La conducta siguiente serie:
a) Manejo expectante

79. Paciente masculino de 70 años que acude a consulta por problemas de visión. Refiere
que en los ultimoas meses ha notado visio borrosa ``justo en el frente``de su campo visual,
que no se puede corregir con sus lentes. Este problema le afecta ambos ojos. En sus
antecedentes patológicos menciona hipersion, tratada con amlodipino. Refiere
tabaquismo de 20 paquetes/ años. Al mostrarle la cartilla de Amsler dice nota un área
borrosa y asimétrica cerca del centro. A la oftalmoscopia se obsevan depósitos de drusen
en la macula y áreas de depigmentacion. No hay datos de neovascularizacion. Cual seria el
tratamiento inficado en este paciente?
e) observación

80. Paciente masculino de 19 años con desnutrición, refiere astenia, ataque al estado
general, tos, espectoracion, sudoración nocturna y un episodio de hemoptisis. Los análisis
de laboratorio muestran anemia ferropénica. En la radiografia de torax se aprecian
multiples infiltrados intersticiales en ambos campos pulmonares junto con engrosamiento
pleural aplica. Mantoux de 15 mm de induración. Se realiza fribroncoscopia y en la
baciloscopia del broncoaspirado se aíslan BAAR con cultivo de Lowestwin-Jensen positivo
para Mycobacterium Tuberculosis. Se instaura tratamiento con fármacos
antituberculosos. Que estudios se realizan con el objeto de estudiar esta probable caso de
tuberculosis?
d) todas las anteriores

81. paciente femenino de 25 año acude a urgencias por presentar secreciones vaginal
maloliente asociada con ardor y prurito vulvar, disuria, polaquiuria, dispareunia y
sangrado poscoital. La paciente tiene relaciones sexuales regularmente, con dos parejas
masculinas en los últimos ses meses. A la ef se observa eritema de la valvu y mucosa
vaginal, secreción verde-amarillenta y hemorragias puntiformes en la vagina y el cuello
uterino. El análisis de la secreción con solución salina en el microscopio muestra
tricomonas móviles.
Cual de los siguientes enunciados es falso con respecto al tratamiento de la tricomoniasis?
b) El tratamiento tópico vaginal es igualmente efectivo que el tratamiento via oral
82. Los paramédicos traen a un paciente masculino de 65 años encontrado en un parque
con el siguiente electrocardiograma

-Cual es el diagnostico mas probable de acuerdo con lo hallazgos electrocardiográficos?


a) Hipotermia
83. Una paciente de 20 dias de edad es llevada a la sala de urgencias. La madre menciona
que la paciente ha presentado fiebre y llanto continuo las ultimas 4 horas ; la paciente ha
sido alimentada el dia de hoy en una sola ocasión pero vomito toda la formula ingerida. Se
refiere que la paciente fue producto de un parto vaginal a termino y eutócico de una
madre de 22 años gesta 1 para 1. Ningún otro antecedente de importancia. A la
exploración física la paciente presenta temperatura de 38.3, pulsos 140 latidos, FR 32.
Presenta disminución del tono muscular, sin respuesta a los estimulos visuales o auditivos.
La fontanela anterior se presenta abombada. Mucosa adecuadamente hidratadas y
ausencia de exantemas. Los estudios de laboratorio revelan leucocitosis.
83.a . Se realizan una puncion lumbar, en la que se observan un bacilo grampositivo. Cual
es el agente causal mas probable?
b) listeria monocytogenes

83.b Cual es el tratamiento mas adecuado según el organismo causal?


a) ampicilina y gentamicina

83.c. Cual es la forma mas común de infección del organismo causal del cuadro de
meningitis?
a) ingestión de productos lácteos no pasteurizados

84. Cuantos criterios de Duke se necesitan para hacer el diagnostico de endocarditis


infecciosa?
b) uno mayor y tres menores

85. Lo siguiente es FALSO acerca de la infecion por Taenia Solium:


e) El método mas común de adquirir cisticercosis es al ingerir carne de cerdo mal cocida,
con cisticercos

86. Cual es la fracción de eyección cuando un paciente presenta una frecuentia cardiaca
de 70 latidos/ min, volumen al final de la diasto de 220mL, y volumen final de la sístole de
120mL?
a) 45%

87. Paciente femenino de 7 meses de edad es traída a consulta. La madre refiere que ha
presentado cuadros frecuente de vomito, movimientos no coordinados y crisis
convulsivas; menciona que el cabello se le ha vuelto mas claro en comparación al resto de
sus hermanos. En la exploración física se identifica piel seborreica, microcefalia, maxilar
prominente, diente espaciados y retraso en el crecimiento.
87ª. De acuerdo con las características clínicas, Cual es el trastorno mas probable de esta
paciente?
d) Fenilcetonuria

87b. Con cual de los siguientes exámenes de laboratorio confirmarían el diagnostico?


a) Niveles de fenilalanina en plasma
87c. Con base en su sospecha clínica, ¿Qué tratamiento indicaría?
c)dieta libre de fenilalanina y cofactor tetrahidrobiopterina

87d. Que tipo de transmisión tiene este padecimiento?


b)Rasgo autosómico recesivo

88. Fuentes mas importantes de producción de andrógenos en la mujer:


a) androstenediona y dehifroepiandrosterona

89. Una abuela trae a consulta a su nietp por exantema. Su preocupación es que observan
que su nieto de 3 años amanecio con las mejillas muy rojas esta mañana y que sus brazos
y piernas parecen estar rojos también, además de que lo siente ``caliente``. A la
exploración física usted reconoce de inmediato el signo del ``niño abofeteado``con el
exantema característico.
89ª. Con base en las características del paciente. Cual es el diagnostito mas probable?
b)Eritema infeccioso

89b. De acuerdo con sus sospecha clínica, Cual es el agente etiológico del padecimiento?
a)Parvovirus B19

89c.en relación con su sospecha clínica, que complicación es factible que es presente?
c)crisis aplasica

89d. De acuerdo con su sospecha clínica, Cuales son los hallazgos que esperaría encontrar
en los exámenes complementarios?
b)Leucopenia inicial seguida de leucocitos y linfocitos

89e. Con base en su sospecha clínica, cual es el tratamiento indicado para este paciente?
d)paracetamol

90. Se encuentra realizando la exploración física inicial de un neonato de 12 horas de vida,


producto de primera gestación, peso al nacer 3250 g. Explora intencionadamente la
cadera para descartar la presencia de una luxación congénita.
90ª. Cual de las siguientes NO es una característica de esta enfermedad?
e) es mas frecuente en varones

90b. Cual de los siguientes hallazgos en el examen físico del neonato le hacen sospechar
luxación congénita de cadera?
d)chasquido al abducir la cadera y presionar el trocánter mayor hacia una posición
anterior, con la cadera y rodilla en flexion

90c. Que información aporta la prueba de Barlow?


b)Detecta una cadera que es luxable al momento del examen físico
90d. Ante pruebas positivas de Barlow y Ortolani, usted decide repetir la exploración a las
seis semanas de vida y a los seis meses. Para la evaluación a los seis meses de vida, que
método diagnostico es de elección para diagnosticar la displasia congénita de cadera?
e)radiografia simple

90e. El paciente acude a consulta a los cinco meses de vida y se confirma el diagnostico.
Cual es la opción mas adecuada de manejo de acuerdo con la edad del paciente?
a)arnes de pavlik

91. Paciente femenino de 65 años de edad con larga historia de fibrilación auricular (sin
tratamiento), se presenta en urgencias con antecedentes de inicio súbito de dolor
abdominal intenso. A continuación del inicio del dolor ella vomito y padecio movimientos
intestinales prolongados, no ha expulsado flatulencias desde esa ocasión. La exploración
física revela abdomen distendido moderadamente, el cual se distiende con delicadeza
aunque hay ausencia de signos peritoneales. Hace 15 años fue sometido a histerectomía
abdominal. El diagnostico mas probable en esta paciente podría ser:
b)isquemia mesentérica embolica aguda

92. Paciente masculino de 35 años acude al servico de medicina preventica por presentar
dermatosis diseminada en codos y rodillas con placas erimo-escamosas. El digno clínico
que corrobora el diagnostico de este paciente es:
c) signo de rocio sangrante

93. Se presentan un paciente de 22 años de edad con su primer embarazo. La paciente se


muestra renuente a abstenerse del consumo de alcohol durante el resto de su embarazo.
En el caso de una mujer embarazada que bebe entre 60 y 120 ml de alcohol a 80%. Cual es
el riesgo de presentar un neonato con síndrome alcohólico fetal?
b) 10%

94.Cual de las siguiente translocaciones cromosómicas se asocia con el linfoma folicular?


d)t(14:18)

95. Masculino de 56 años que llega al servicio de urgencias con dolor torácico de mas de
12 horas de evolución, que inicia en forma súbita, mejora al estar sentado, no tiene
antecedentes de importancia. La acción principal en este paciente es descartar:
c. Cardiopatia isquémica

96. Paciente masculino de 30 años de edad acude por poliuria y polidipsia. El paciente
menciona recientemente ha notado un incremento en el tono de la pigmentación de su
piel a pesar de que no se ha expuesto mas tiempo de lo normal a la radiación solar.
Expresa que por las mañanas presenta edema de las extremidades inferiores y necesita
dormir con dos o tres almohadas para evitar la sensacion de disnea nocturna. Cual es el
diagnostico mas probable en este paciente?
d)Hemocromatosis

97. Paciente de 22 años de edad sin antecedentes heredofamiliares de importancia. El


paciente se presenta con cuadro clínico caracterizado por lateropulsion, vértigo, cefalea,
hipoacusia bilateral y catarata. El estudio audiometrico reporto hipoacusia superficial
grave bilateral. Los potenciales evocados auditivos mostraron disrupción de la via auditiva
a nivel del tallo cerebral. En la resonancia magnética de cráneo contrastada con gadolinio,
se observaron lesiones tumorales extraaxiales hiperintensas bilaterales a nivel del angulo
pontocerebeloso, que afectan la emergencia de los complejos VI/VIII, del lado izquierdo
mas prominente, y condicionan compresión del tallo cerebral, en especial del puente. Cual
de los siguientes hallazgos es mas probable probable que se encuentre en este paciente?
a) Nodulos de Lisch

98. Paciente masculino de 3 años y 6 meses de edad llega referido con los siguientes
antecedentes: hijo de primera gestación, sin control prenatal, se observo por parto vaginal
a las 27 semanas de gestación y peso 1400g; presento sangrado de tubo digestivo bajo,
trombocitopenia e infecciones severas a los pocos días de nacido; desde los 4 meses hasta
ahora ha sido hospitalizado en 54 ocasiones por sangrado de tubo digestivo y epistaxis
asociados con choque hemorrágico, gingovorragia, infecciones dentales e infecciones
respiratorias como otitis media crónica, sinusitis y neumonía. En los exámenes resaltan los
siguientes datos: trombocitopenia de 31000, IgM baja, IgA normal e IgG elevada.
98ª.Con base en las características del paciente. Cual es el diagnostico mas probable?
d)Sindrome de Wiskott-aldrich

98b. Que hallazgos apoyan su diagnostico en los estudios complementarios?


c)Trombocitopenia, IgM baja, IgG e IgA elevadas

98c. En relacioncon su sospecha clínica, cual es la alteración genética asociada?


a) proteína WASP

98d.Con base en su sospecha clínica, cual es el tratamiento que estaría indicado en este
paciente?
b) Transplante de medula osea

98e. en relación con su sospecha clínica, cual es la complicación que se podría presentar
en el paciente?
d)muerte por hemorragias

99. Cual de las siguientes neoplasias malignas se asocia con el desarrollo del síndrome de
Lambert-Eaton?
e)Timoma

100. Masculino de 52 años que es derivado al servicio de dermatología por presentar


prurito en manos y antebrazos, eritema y esfacelacion con exacerbación desde hace mas
de 3 meses. Asimismo maculas en cuello y parte alta del torax, agregándose edema ficial.
Antecedentes de trabajar durante 15 años en fabrica de cementos. EF: placa eritematosa
con hiperqueratosis en miembro pelvido izquierdo con huellas de rascado y acompañado
de edema. Cual es el diagnostico mas probable?
a)Dermatitis por contacto

101. Paciente femenino de 32 años que se presenta con alteraciones de la visión central
en el ojo derecho e incontinencia urinaria. Al interrogatorio menciona que hace algunos
años sintió debilidad en la pierna izquierda que cedió espontáneamente; tiempo después
presento paresia en la extremidad superior derecha, la cual también cedió. Cual es el
diagnostico mas probable en esta paciente?
c) esclerosis multiple

102. se presenta a consulta externa una paciente de 25 años de edad en trabajo de parto.
La paciente refiere que nunca acudió a consulta de seguimiento obstétrico. El producto de
esta paciente es un neonato de bajo peso, con filtrum liso, microcefalia y fistula pulmonar.
Cual de las siguientes condiciones es la que mas probablemente ocasiono estos defectos?
a) consumo de etanol durante el embarazo

103. Paciente masculino de 18 años de edad el cual es llevado al servicio de urgencias


después de que sus padres lo encontraron con perdida del estado de alerta en el baño de
su casa. Al ingresos presenta hiperglucemia en una muestra de glucosa capilar; la
gasometría demuestra un PH de 7.2 y el estudio cualitativo de orina presencia de cetonas.
A la exploración física el paciente presenta presión arterial de 100/60mmHg, frecuencia
cardiaca de 130latidos/ minuto, asi como un estado grave de deshidratación. Los padres
mencionar que desde hace algunas semanas su hijo había estado perdiendo peso no
intencionadamente. Despues de iniciar el tratamiento con hidratación, insulina y manejo
de las alteración electrolíticas el paciente recupera el estado de alerta. Horas depues se
presenta fiebre alta y cefalea retroocular. A la exploración de cavidad nal se encuentra
una escara necrótica negra adherida al cornete inferior. Cual de los siguiente métodos
diagnósticos es el mejor para confirmar el diagnostico de este paciente?
c)Bipsia de mucosa

104. Masculino de 40 años acude a cosulta externa refiriendo dolor en el primer ortejo de
pie derecho y ambas rodillas. Antecedentes: diabetes mellitus, por rama materna, y litiasis
renal e hipertensión arterial por su padre. Desde los 30 años de le diagnostico litiasis
renal. Actualmente recibe tratamiento con esomeprazol y procineticos desde hace 6
meses, indicados en el servicio de gastroenterología por padecimiento de ERGE.
104.a El padecimiento causal de la sintomatología de este paciente es:
c) Gota

104b. el examen de laboratorio indicado para confirmar el diagnostico de este paciente


es:
d) cuantificación de acido urico en sangre y orina

104c. Medida terapéutica indicada en este paciente:


c) Etericoxib

105. Cual de las siguientes arterias es mas susceptible de traumatismo durante la


colecistectomía?
b)la arteria cística

106. Mencione cual de los siguientes factores esta mas asociado con el riesgo de padecer
artritis reumatoide:
e)Multifactorial

107. Cual de los siguientes comentarios referentes a los aneurismas de aorta abdominal es
correcto?
a) El seguimiento seriado con métodos de imagen esta indicado en los paciente
asintomáticos con aneurismas menores de 5 cm

108. Cual de las siguientes hormonas gastrointestinales estimula la secreciones del


factores intrínseco?
d)gastrina

109. Cuales son las entidades con las que se debe realizar diagnostico diferencial del
paciente con artritis reumatoide?
e) Todas las opciones

110. Pacientes femenino de 34 años de edad programada por cesarea por desporporcion
cefalopelvica. La paciente había sido diagnosticada con síndrome de Wolffñparkinson-
white a la edad de 18 años, sin embargo se mantenía asintomática desde hace 16 años.
Previa a la anesteria epidural la paciente presentaba frecuencia cardiaca 77 con un ritmo
sinusal. Se inyecta 13 mL de mepivacaina a 2% en el espacio epidural a nivel de T4. La
cesarea se realiza sin eventualidades con una perdida sanguínea de 450mL. Al final de la
cesarea, la paciente presenta taquicardia supraventricular de inicio súbito, incrementando
la FC 190 con disminución de la presión arterial. Se realizo masaje carotideo sin conseguir
estabilización. Cual de los siguientes hallazgos No están presente en la taquicardia
reciproca ortodromica auricoventricular, es decir, la taquicardia paroxitica
supraventricular que se observa en pacientes con el síndrome de Wolff-parkson-white?
b)onda delta

111. Paciente masculino de 65 años de edad, que se presenta al servicio de urgencias por
síntomas de resfriado, nauseas, vomito y cefalea frontal y retroocular. Entre los
antecedentes de importancia se encuentra sedentarismo, obesidad y tabaquismo. A la
exploración física el paciente se presenta alerta y orientada sin alteraciones motoras o
sensoriales; sin embargo, la paciente presenta una hemianopsia homónima derecha con
conservación de la visión. Cual de las siguientes arterial es la que mas probable se
encuentra ocluida en este pacientes?
d) arteria cerebral posterior izquierda

112. Debido a su considerable toxicidad y a la disponibilidad de agentes menos toxicos


( cefalosporinas y quinolonas), los aminoglucosidos se han estado utiliazando menos
desde los últimos años. Estos medicamentos son generalmente utilizados para tratar
organismos gramnegativos, con o sin vancomicina. A pesar de que los aminoglucosidos
demuestran actividad in vitro contra la mayoría de las bacteria grampositivas, estos jamas
deben de usarse aisladamente, debido a que existen alternativas menos toxicas. Cual de
los siguientes enunciados es el que tiene mejor relación con el mecanismo de acción de
los aminoglucosidos?
d) inhibición de la síntesis de proteínas por medio de la inhibición de la subunidad 30s

113. paciente femenino de 15 años de edad se presenta a consulta por presencia de


``bolita``en rodilla derecha. En el interrogarotio los padres mencionan que nosotros la
tumoración porque después de una clase de educación física su hija les menciono que le
dolia su ``bolita``. Refieren que desconocia que la tuviera porque nunca le había dado
molestoas, por lo que desconoce cuanto tiempo lleva con ella. EF nota tumoración en
parte distal del femur derecho; no hay dolor a la palpación ni otros síntomas asociados.
113ª. Con base en las características de la paciente, Cual es el diagnostico mas probabl?
d) Osteocondroma

113b. En relación con su sospecha clínica. Que hallazgos esperarían encontrar en una
radiografia?
c)proyección osea pedunculada

113c. en relación con su sospecha clínica. Cual es la fisiopatología de este padecimiento?


a) defecto en el cartílago de crecimiento

113d. de acuerdo con su sospecha clínica. Cual es el tratamiento que recomendaría?


b) no se indica nungun tratamiento

113e. en relación su sospecha clínica, cual es la complicación mas importante que se


puede presentar?
d) no se presentan complicaciones

114. cual de los siguientes nervios envía fibras hacia la región inferior del canal anal?
e) nervio pudendo

115. paciente masculino, producto de primera gestación de 39semanas de duración,


obtenido por histerotomía indicada por desproporcion cefalopelvica. Se evalua al nacer y
se encuentra con una FC 98, Respirando de forma irregular y lenta, nollora, hay presencia
de algunos movimientos de extremidades con predominio de flexion, cuando se estimula
con sonda nasogástrica se desencadena llanto vigoroso y su piel se observa rosada en el
tronco, pero los dedos de las manos y pues se encuentra cianóticos, de acuerdo con las
observaciones. Cual es el puntaje de Apgar que se le otorga?
d) 6

116. Recien nacido masculino de 39 sdg con adecuado control prenatal presenta cianosis
severa que no mejora con oxigeno suplementario. En la EF se aprecua cianosis
generalizada, no hay dificultad respiratoria y a la auscultación del torax no se identifica
soplo. La radiografia de torax muestra mediastino estrecho y corazón globular.
116ª. Con base en las características del paciente. Cual es el diagnsotico mas probable?
c) Transposición de grandes vasos

116b. de acuerdo con su sospecha clínica, con que problema medico se ha asociado este
diagnostico?
a) Síndrome de DiGeorge

116c. en relación con su sospecha clínica, que hallazgos esperaría encontrar en la


radriografia de torax?
d) Corazón en huevo

116d. de acuerdo con su sospecha clínica, cual es el tratamiento en este paciente?


b) Conmutación arterial

116e. en relación con su sospecha clínica, cual es la complicación que se puede presentar
a largo plazo?
c) Hipertensión pulmonar

117. Paciente femenino decundigesta, con embarazo normoevolutivo y trabajo de parto


sin alteraciones. La exploras y te perctaras que el fondo uterino se encuentra a 35 cm de la
sínfisis del pubis y el producto se ubica en presentación pélvica, y al realizar el tacto
vaginal te das cuenta que esta por completo borrado y dilatado y se encuentra en el tercer
plano de Hodge. No te da tiempo de pasarla a la sala de parto y se produce un parto
precipitado. Alumbramiento sin alteraciones, aunque percibes una hemorragia y un
desgarro que abarca el cuerpo perineal y el esfínter rectal. Que grado de desgarro
presenta?
c) tercer grado

118. paciente femenino de 3 años de edad que acude a consulta de control prenatal.
Tiene 22 semanas de gestación por fecha de ultima menstruación , sin antecedentes de
importancia y al interrogatorio no refiere datos de alarma. Toma acido fólico y fumarato
ferroso. Qu vacunas se le debe administrar para evitar la aparición de problemas
posteriores?
a) vacuna Td
119. paciente femenino de 25años de edad con diagnostico de embarazo de 32 sdg acude
a consulta de urgencias por presentar contracciones en abdomen de 50 segundos de
duración, teniento tres cada 10 min y con hemorragia transvaginal de escasa cantidad, con
salida de tapon mucoso, se encuentra palida, diaforetica, taquicardia, polipneica con
presión arterial 130/80. Usted diagnostica una amenaza de parto pretermino. Que datos
lo orientan hacia un diagnostico diferencial?
c) longitud cervical de 4 cm por ultrasonido

120. niño de 2 años que presento hace 2 dias cuadro de febrícula (hasta 37.5ºC) rinorrea
hialinba; desde hace 8 horas inicio con fiebre 38.7, tos metalica y estridor inspiratorio. En
el examen físico se observa, taquipnea, tiraje subcostal y retracción xifoidea , faringe con
hiperemia sin exudados, en torax murmullo vesicular presente sin ruidos agregados.
120ª. Cual es la causa de la patología que presenta el paciente?
c)virus parainfluenza

120b todas las siguientes son medidas terapéuticas indicadas para esta patología,
EXCEPTO:
b)ceftriaxona

121. se trata de un paciente masculino de 56 años de edad, con antecedentes de etilismo


a base de destilados y fermentados, cada fin de semana hasta llegar al estado de
embriaguez por hasta 2 o 3 dias consecutivos, además tabaquismo a razón de 15
cigarrillos al dia desde hace 16 años (índice tabáquico: 30) acude por presentar dos emesis
en ``posos de café``hoy, además ha presentado durante 1 semana evacuaciones melenicas
2 a 3 veces por dia. A EF con telangiectasias en el territorio de la vena cava superiorm
eritema palmar, hipotrofia tenar e hipotenar con contractura de dupuytren de ambas
manos. FC 95 FR 20, temp 35.7 TA 100/60 Peso 89 talla 1.72

121.a los antecedentes del paciente y los datos clínicos de hematemesis en ``posos de
café``y melena, nos hacen pensar en una hemorragia de tubo digestivo alto; basados en
epidemiologia, el diagnostico mas probable es:
b) varices esofágicas

121b. el estudio de primera elección que ayuda a confirmar el diagnostico es:


c)esofagogastroduodenoscpia

121c. para protocolizar al paciente, se deben hacer estudios bioquímicos que permitan
estratificar el grado de insuficiencia hepática según la clasificación de Child-pgh, el
paciente cursa con ascitis moderada y sin encefalopatía. Con los siguientes laboratorios,
BH Hb 8.5 hto 26.5, leu 13.5, plt 125m BT 4.8 (BD 2.3 BI 2.5) proteínas 5.2, ALB 3.0, Glo 2.2
INR 1.5, que grado tiene esta paciente?
d)child-turcotte-pugh C
121d. para disminuir la frecuencia de sangrados, usted selecciona manejo farmacológico
para largo plazo con:
c)beta-bloqueadores no selectivos

121.e el tratamiento de elección, para pacientes de difícil control con recurrencia es:
b)derivación portosistemica intrahepatica transyugular (TIPS)

122. se trata de paciente femenino que se encuetra en puerperio mediato. Con entuertos
continuios. Sin fiebre, sin taquicardia, presión arterial dentro de parámetros normales. El
utero se encuentra por debajo de la cicatriz umbilical y hay presencia de loquios
serohematicos no fétidos. De donde provienen los loquios?
e) Decidua esponjosa

1.Cual de los siguientes opciones representa correctamente el uso de la hemoglobina


glucosilada?
a)monitoreo y eficacia del apego al tratamiento de diabetes

2.Varon de 50 años de edad transplantado renal, que presenta multiples verrugas


vultares. Una de ellas, en la cara externa del antebrazo, se observan mas queratosica,
eritematosa, ulcerada y sangrante. Acude con usted porque recuerda que le dijeron que
estas lesiones podrían malignizarse. Cual es el cáncer de piel que se relaciona mas a
menudo con infección por VPH en la piel?
a) carcinoma epidermoide

3. Es un análogo de insulina con PH acido:


e) glargina

4. Acude a su consulta un padre con su hijo de 16 años. Lo trae a consulta porque noto
una erupción cutánea en la espalda de su hijo una semana después de que regreso de
campamento escolar hace casi un mes. El exantema, según refiere el padre, se caracterizo
por algunas lesiones rojas circulares con el centro claro que fueron diseminándose. Ha
acudido a varios médicos que le han enviado medicamentos sintomático; sin embargo, no
ha cedido la sintomatología. Durante los últimos 3 dias el joven ha tenido fiebre baja,
cefalea y malestar general. A la ef se notan todavía algunas lesiones eritematosas con
centro claro, como un tiro al blanco y linfadenopatias.
4ª. Cual es el diagnostico mas probable?
d) Enfermedad de lyme

4b. la enfermedad tiene tres etapas. Cual es la complicación cardiaca mas importante en
la segunda etapa?
a)bloqueo cardiaco
4c. quien es el agente responsable de la enfermedad?
a)una espiroqueta, borrelia burgdorferi

4d. Que estudio es el indicado para confirmar el diagnostico?


b)ELISA

4e. Cual es el tratamiento apropiado para este caso?


a)Doxiciclina

5.Un paciente al despertar descubre que no puede observar los objetos a la izquierda con
uno u otro ojo. Los reflejos pupilares a la luz se encuentran sin alteraciones en ambos
ojos. La campimetría revela hemianopsia con involucro de la visión macular. Cual es el
sitio mas probable de la lesión?
b)radicaciones visuales

6.paciente masculino de 55 años con antecedentes de cirrosis alcohólica acude a


urgencias por presentar dolor abdominal y fiebre de cuatro días de evolución. A la
exploración física se encuentra febril, hay caput medusae, distensión abdominal, matidez
cambiante a la percusión y rebote. Los resultados de la paracentesis demuestran >250
polimorfonucleares/mm3 y el cultivo es positivo para un solo organismo.
Cual es el tratamiento de primera elección que deberá administrarse al paciente?
d)cefotaxima

7.Mujer de 42 años de edad acude por una dermatosis diseminada a ambas axilas asi
como cuello, nuca y región sumamaria, la cual se caracteriza por pigmentación café
grisácea, engrosamiento palpable y de aspecto verrugoso. La paciente no recuerda con
exactitud el tiempo de aparición de estas dermatosis pero refiere ser de larga evolución.
Es casi asintomática, aunque en algunas ocasiones le produce prurito. Como antecedente
de importancia tiene diagnostico de diabetes tipo 2 mal controlada, es premenopausica y
tiene hiperandrogenismo ovárico. Presenta obesidad e hirsutismo. El diagnostico de la
dermatosis es:
c)acantosis pigmentaria sindromatica

8. paciente masculino de 31 años de edad con antecedentes de dos semanas de fiebre, a


lo cual se agrego hace una semana distensión abdominal, estreñimiento y dolor
epigástrico. Entre otras molestias también refiere cefalea, escalofríos (calosfríos), mialgias
y malestar general. La ef revela la presenvia de un `exantema rosado``(exantema
maculopapular color salmon que blanquea a la presión), principalmente en el torax asi
como el dolor abdominal generalizado a la palpación y esplenomegalia. Presenta
temperatura de 38.9Cm FC 45 FR 20, TA 100/65. Los estudios de laboratorio básicos son
relevantes por leucopenia y neutropenia. Cual es la prueba diagnostica con mayor
sensibilidad a pesar de tratamiento antibiótico?
c)cultivo de medula osea
9.el tratamiento inicial para cáncer diferencial de tiroides(papilar, folicular) es:
c)quirúrgico

10. se presenta paciente masculino de 78 años a la sala de urgencias por dolor abdominal
punzante de inicio súbito y de 2 horas de evolución acompañado de vomito de contenido
gástrico. El paciente localiza el dolor en la adbominal, lumbar con irradiación hacia ingle.
Refiere que en la mañana de hoy fue tan intenso que le ocasiono sincope. Entre los
antecedentes de importancia el paciente es fumador (20 cigarrillos/dia) desde los 20 años,
enfermedad pulmonar obstructiva crónica de cinco años de diagnostico, hipertensión
arterial sistémica de 10 años de diagnostico y angina estable desde hace ocho meses. Se
registra FC 140, ta 90/50, FR 32, a la Ef es posible palpar una masa abdominal pulsatil por
lo que se sospecha rotura de aneurismática. Se solicita ultrasonido abdominal el cual
reporta liquido en la cavidad peritoneal y aumento del diámetro de la aorta abdominal. El
paciente se somete a cirugía de reparación aneurismática. Posterior a la cirugía el paciente
presenta sintomatología atribuida a la oclusión temporal de las ramas que dan origen a la
arteria espinal anterior. Que sintomatología neurológica presentara este paciente?
d) paresia espástica con hiperreflexia y perdida de la sensación dolorosa y térmica en
ambas extremidades inferiores; conservación de la sensibilidad vibratoria y propiocepción

11. un paciente de 70 años posterior a un infarto cerebral presenta dificultad para


entender el lenguaje hablado, asi como incapacidad para reconocer objetos colocados en
su lado derecho, cual de las siguientes arterias es la que mas probablemente sufrio
trombosis en este paciente?
d)arteria cerebral medial izquierda en su división inferior

12. un paceinte de 40 años de edad acude a consulta por presentar dolor agudo en la
región inguinal posterior a un dia que levanto multiples costales de cemento. El paciente
es diagnosticado con una hernia de disco. Cual de los siguientes nervios espinales es mas
probable que cause el dolor en este paciente?
a)L1

13. acude a la sala de urgencias un paceinte sexo femenino de tres semanas de vida por
presentar vomito en las ultimas 24 horas asi como distensión abdominal. A la ef se
confirma la distensión abdominal, sin palpar ninguna estrucutra sobresaliente. Los
estudios de los electrolitos séricos demuestran una concentración de sodio 131 potasio
2.9 cloro 85, dióxido de carbono 38, cual es el diagnostico mas probable de esta paciente?
a)rotación del intestino delgado alrededor de la arteria mesentérica superior

14. masculino de 40 años con antecedentes de diabetes mellitus tipo 2 e insuficiencia


renal terminal con tratamiento sustitutivo con hemodiálisis, se presenta al servicio de
urgencias debilidad. Menciona que no acudió a su ultima sesión de diálisis, y que desde
hace dos días no se administra insulina debido aque esta esperando recibir su salario. Al
interrogatorio, el único síntoma es la debilidad. A la ef no se encuentra anomalías
relevantes. Los estudios de laboratorio revelan una concentración de sodio 125, potasio
5.2, cloro 104, bicarbonato 18. La glucosa sérica es de 700 y la osmolaridad sérica de 700 y
la osmolalidad sérica de 310. Una radiografia de torax revela campospulmonares normales
y el electrocardiograma es normal. Cual de las siguientes opciones es la mas adecuada
respecto al siguiente paso terapéutico a seguir en este paciente?
b)administración de insulina intravenosa

15. paciente femenino de 64 años refiere historia de tos y fatiga de cinco semanas.
Reporto que su tos era en un principio seca, pero volvió productiva en las ultimas dos
semanas. Refiere escalofríos, sudores nocturnos y perdida de peso de 13kg en los últimos
dos meses. Ha viajado por latinoamerica, tiene antecedentes de consumo de alcohol y
diabetes tipo 2 para lo que toma glibenblamida. A la ef se auscultan disminuidos los ruidos
respiratorios en la zona superior y media de los campos pulmonares. No se encuentra
hepatomegalia ni esplenomegalia. La radiografia de torax mostro infiltrado fibronodular
en lóbulo superior derecho con una cavitación pequeña. Le dieron diagnostico de
tuberculosis pulmonar; mientras los cultivos están pendientes, se inicio tratamiento con
isoniazida, rifampicina, pirazinamida y etambutol de manera simultanea desarrollo
paresterias en ambos miembros inferiores, comenzando por los pies, y termino con un
intenso dolor. Cual es la causa mas probable de la neuropatía?
a) isoniazida

16. se presenta un paciente con diplopía, la cual hace necesario que incline la cabeza para
compensar y evitar este síntoma. La ef revela que la diplopía se ve acentuada cuando el
paciente trata de leer o bajar las escaleras. Cual es el sitio mas probable de la lesión?
a)nervio troclear

17.paciente masculino de 49 años de edad acude a consulta por dolor en las manos. Al
revisar la historia clínica, se descubre que tiene antecedente de hipertensión, insuficiencia
renal crónica y gota. Los medicamentos que toma son amlodipino 10 mg/dia, enalapril 20
mg, furosemide 60 mg dos veces al dia y alopurinol 100 mg. A la ef la presión arterial es de
140/85, FC 80. Las articulaciones de las manos se encuentran edematosas y con datos de
inflamación, asi como con tofos de gota. Se inicia tratamiento con naproxeno 500 mg dos
veces al dia durante 10 dias para tratar el dolor e inflamación. Los estudios de laboratorio
son los siguientes:

Cual de las siguientes opciones representa el mecanismo de acción mas probable


mediante el cual naproxeno induce hiperpotasemia en este paciente?
b)inducción de hipoaldosteronismo hiporrenimemico

18. mujer de 60 años, ama de casa, obesa sin antecedentes de importancia, desde hace
dos años presenta dolor en las muñecas y rodillas de forma ocasional, que se exacerba
con la actividad física, y disminuye con la ingesta de analgésicos como el paracetamol. Ha
tenido rigidez articular matutina de 30 minutos o menos. Asi mismo desde hace tres
meses, después de una caída de su misma altura, presento de forma súbita artritis de
rodilla derecha a lo que se agrego en un periodo de dos semanas artritis de la muleca del
mismo lado, que remitieron artritis de la rodilla derecha desde hace 24 horas, a pesar de
tomar tratamiento antiinflamatorio, a la ef se encuentra con dolor en muñeca derecha,
artritis de rodilla derecha. Sus exámenes de laboratorio muestran una BH con leucositosis
de 13 000 y predominio de neutrófilos, VSG de 40 y acido urico de 5mg. En la radiogradia
de rodilla derecha se observan calcificaciones de aspecto lineal que siguien el contorno del
cartílago.
18ª. Cual es el diagnostico mas probable?
e)artropatía por cristales pirofosfato cálcico

18b. cual es la conducta inmediata a seguir?


e)puncion de rodilla inflamada para búsqueda de cristales e infiltración intra articular de
glucocorticoides

18c. que otros exámenes de laboratorio solicitaría para completar el protocolo de


estudio?
a) B y c son correctas (hormona paratoridea, perfil tiroideo, cinetica de hierro,,,
calcio fosforo, fosfatasa alcalina y magnesio)
18.d que estudios de gabinete se pueden utilizar como parte del protocolo de estudio de
este paciente?
e)todas las anteriores

19. paciente masculino de 27 años acude a urgencias por presentar dolor y deformidad
del antebrazo derecho posterior a sufrir caída sobre su mano derecha extendida. A la ef
notoria la imposibilidad del paciente de hacer una pinza aproximando el pulgar y el índice.
La radiografia muestra fractura transversa distal del radio con desplazamiento de la
articulación radiolunar distal. Usted le informa al paciente que esta fractura requiere
tratamiento quirúrgico en todos los casos.
Cual es el diagnostico?
C)fractura de galeazzi

20. cual de los siguientes enunciados es el mas correcto respecto a los recién nacidos con
ambigüedad de genitales?
c)los genitales ambiguios se asocian en algunas ocasiones con antecedentes de hermanos
con hiperplasia suprarrenal congénita

21. cual de los siguientes dientes son los primeros en aparecer?


a)incisivos centrales mandibulares

22. Paciente masculino de 17 alos de edad, jugador de futbol soccer, sin antecedentes de
enfermerdads cronicodegenerativas; inicia hace seis meses con obstrucción nasal, rinorrea
purulenta y epistaxis recurrente, las cuales son bilaterales,de predominio derecho. Refiere
recibir hace dos días contusion nasal en el futbol. El dia de hoy acude por epistaxis que no
se autolimitan ni cede a la compresión de 8 horas de evolución que inicia de manera
espontnea al estar mirando la televisión.
22ª. Cual es la causa mas probable de la epistaxis en este paciente?
d)nasoangiofibroma juvenil

22b. que hallazgos clínicos esperarías observar?


c)masa lisa, violácea y lobulada en nasofaringe o pared lateral nasal

22c. cual seria el tratamiento de elección con base en tu diagnostico presuncional?


c)resección quirúrgica

23. paciente masculino de 54 años de edad que ingresa a hospitalización por embolismo
pulmonar agudo. Se administra heparina intravenosa. A la admisión, el conteo plaquetario
fue de 223 000 paquetas. Después de cuatro días, el conteo plaquetario desciende a 16
000 plaquetas, pero el paciente se muestra asintomático. Cual de las siguientes oraciones
es la mas correcta respecto al padecimiento de este paciente?
d) los inhibidores directos de la trombina, como la lepirudina, puede ser sustitutos seguros
de la heparina

24. cual de las siguientes enfermedades se caracteriza por presentar dilatación


aneurismática de las arterias subclavias y carótidas?
d)enfermedad de behcet

25. cual de los siguientes esquemas de prevención antibiótica es la mas adecuada para un
paciente al que se le va a realizar extracción de una pieza dental, el cual tiene
antecedentes de colocación de valvula aortica mecánica?
d)amoxicilina 2g por via oral una hora antes de iniciar el procedimiento

26. paciente masculino de 6 años de edad, sin antecedentes médicos de importancia,


incicio su padecimiento 24 h antes de su ingreso. Comenzó con tos metalica, sin otros
síntomas; 12 h después esta se intensifico y apareció dificulta respiratorioa. Fue entonces
que se le llevo a un centro medico en donde se le inicio tratamiento con oxigeno y
micrnebulizaciones con adrenalina, sin que haya habido mejora alguna. Posteriormente
apareció estridor inspiratorio y sialorrea. Al realizar revisión de la via aérea se observa
epiglotitis enrojecida, oscura y edematosa.
26.a con base en las características del paciente, cual es el diagnostico mas probable?
b) epiglotitis
26b. de acuerdo con su sospecha clínica, cual es la causa de este padecimiento?
a)haemophilus ifluenzae

26c. en relación con su sospecha clínica, que hallazgos encontrarías en la radiografia?


d)signo del pulgar

26d. de acuerdo con su sospecha clínica, que tratamiento le indicaría al paciente?


c)intubación endotraqueal y antibióticos

26e. con base en su sospecha clínica, cual es la complicación que se puede presentar en el
paciente?
c)neumonía, linfadenitis cervical o meningitis

27. niño de tres años de edad con cuadro de somlonencia, rubor facial, retención uricnaria
y midriasis con antecedentes de ingesta de antihistamínico sobredosificado. En relación
con el uso de fisostigmina como antídoto en la intoxicación por antihistamínicos, señale lo
verdadero:
a)actua como inhibidor de la acetilcolinesterasa

28. paciente masculino de 56 años es traido con antecedentes de haber ingerido cinco
botellas de cino el dia anterior, acude a urgencias por presentar dolor epigástrico
constante de intensidad 10/10 que se irradia hacia la espalda como si lo ` atravesaran``. El
dolor inicio de manera súbita hace 8 horas seguido de nausea y vomito. A la ef se observa
el paciente recostado en posición de gatillo, deshidratado, se encuentra hipotenso y
taquicardico. Presenta dolor a la palpación del epigastrio y ausencia de ruidos
abdominales. Los estudios de laboratorio muestran hematocrito elevado, leucocitosis
moderada, amilasa sérica 300, lipasa sérica 1000 con una relación amilasa/lipasa >3. Usted
realiza el diagnostico de pancreatitis aguda de etiología alcohólica e inicia tratamiento
medico. Cual de las siguientes condiciones esta asociada con un mal pronostico, en los
controles de 48 horas?
d)elevación del BUN mayor a 5mg/100mL

29. paceinte femenino de 12 años es traída a la sala de urgencias de un hospital en baja


california por su madre una hora después de haber sido mordido por una viuda negra. La
paciente se encuentra anisiosa, refiere dolor intenso en la pierna derecha, sitio en donde
fue morduda por la araña y paresterias en todo e miembro pélvico derecho, sus signos
vitales se encuentran dentro de parámetros normales. De que manera actúa el veneno de
la viuda negra?
a)liberación de adrenalina y acetilcolina en la sinapsis

30. se presenta a consulta un paciente masculino de 8 alos de edad por dolor en pierna
derecha. En el interrogatorio se menciona dolor progresvo de 4 meses de evolución, sin
referir ningún antecedente traumatico, el dolor es intenso y regularmente lo despierta por
la noche. En la ef no hay datos relevantes; la palpación y arcos de movimientos no
producen dolor, pero a la marcha se muestra una cojera casi imperciptible.
30ª. Con base en las características del paciente, cual es el diagnostico mas probable?
b)osteoma osteoide

30b. de acuerdo con su sospecha clínica, que hallazgos esperaría encontrar una
radiografia?
d)zona radiolucida central

30c. en relación con su sospecha clínica, cual es la fisiopatología de este padecimiento?


c)proliferación de osteoblastos inmaduros

30d. en relación con su sospecha clínica, cual es el tratamiento que recomendaría?


a)resección en bloqueo o ablacion percutánea

30e. con base en su sospecha clínica, cual es la complicación mas importante que se
puede presentar?
b)recurrencia

31. paciente masculino de 5 años de edad es traido a consulta para seguimiento de niño
nsano. En el interrogarorio resalta que ha estado muy bien, exepto por el hecho de que
choca contra objetos con frecuencia, en particular por las noches. En la ef se observa que
su estatura y peso se encuentran debajo del percentil 5 y que tiene unos parches de color
gris en la conjuntiva. El resto de la exploración es normal.

31ª. Con base en las características del paciente, cual es el diagnostico mas probable?
a) Déficit de vitamina A
31b. de cuerdo con su sospecha clínica, cual de las siguientes opciones representa una
característica particular del transtorno?
b)manchas de bitot

31c. en relación con su sospecha clínica, cual es la función que tiene en el cuerpo la
vitamina que se encuentra con déficit en el paciente?
d)maduración del epitelio
31d. con base en su sospecha diagnostica, que tratamiento le indicaría al paciente?
c)suplemento de vitamina A

31e. respecto a su sospecha clínica, cual es el mecanismo de prevención que se debio


realizar para evitar este padecimiento?
d)leche materna

32. paciente masculino recién nacido producto de gesta 1 de parto eutócico que presenta
a partir del nacimiento dermatosis diseminada con predominio perioral, espalda, v del
escote, axilas e ingles, caracterizada por erosiones y amplias zonas de denudación que se
acompaña de tejido de granulación exuberante. Presenta peso y talla baja, sin embargo,
no se encuentra enfermo de gravedad. En los estudios posnatales se encontró atresia del
piloro. No tiene otros familiares afectados. El tiempo de epidermólisis ampollosa mas
probable de este neonato es?
c)epidermólisis ampollosa de unión

33. se trata de un paciente masculino de 16 años de edad, quien se encontraba


practicando actividades deportivas, recibe un impacto en la parte media del antebrazo
derecho, inmediatamente presenta edema, equimosis y dolor intenso, con limitación de la
prono-supinacion del antebrazo, sin perdida de la continuidad de la piel, por lo que acude
a valoración FC 95 FR 22 tem 36.5 pa 110/60 peso 65 talla 1.65
33ª. Por los datos de la historia clínica y exploración física, su sospecha diagnostica es:
c)fractura diafisiaria cerrada de cubito

33b. con la sospecha clínica usted solicita:


b)radiografia anteroposterior y lateral de antebrazo

33c. realizo el diagnostico y clasificación de la fractura, cuado no considera que requiere


manejo quirúrgico?

d)fractura aislada de cubito no desplazada

33d. para poder referir a este paciente con un especialista en traumatología y ortopedia,
que tratamiento decide?
b)colocar férula braquipalmar

33e. posterior al manejo quirúrgico y egreso hospitalario, usted recomienda:


c)rehabilitación física

34. estudiante de medicina de sexo masculino de 22 años de edad que acude al


consultorio de su profesor de hematología. El sujeto se siente preocupado porque
recientemente ha presentado fatiga, fiebre recurrente de predominio nocturno, asi como
adenomegalais cervicales. Esta preocupado porque hace poco ha estado rotando por el
servicio de neumología y ha convivido con muchos pacientes con tuberculosis. A pesar de
que el profesor de hematología no parece preocupado por el diagnostico de tuberculosis,
decide tomar una biopsia de ganglio cervical y orgenar estudios de laboratorio. El reporte
de la biopsia menciona que existe expansión de los folículos linfoides con preservación de
la arquitectura normal; existen linfocitos atípicos en las zonas paracorticales. Cual de los
siguientes diagnosticos es el mas probable en este paciente?
d)infección aguda por el virus de Epstein-barr

35. paciente que presenta un estado de acidosis respiratorio crónica. Cual de los
siguientes estudios es la mas adecuada para distinguir entre hipoventilación mediada por
el sistema nervioso central o debido a alteraciones en la via aérea/parénquima pulmonar?
e)medición del gradiente alveoloarterial de oxigeno

36. paciente sexo femenino de 12 años de edad que acude a consulta externa de
pediatría. La madre menicona que la niña ha estado cojeando desde hace varias semanas
y refiere dolor intenso en la rodilla izquierda. Al interrogatorio la madre dice que la
paciente ha encontrado afebril, no recuerda golpes o traumatismos dirigidos a las
extremidades, ni antecedente de enfermedades recientes. A la ef es evidente que la
paciente no puede rotar internamente la pierna afectada, cual de los siquientes
diagnosticos es el mas probable en esta paciente?
a)epifiolisis de cabeza femoral

37, masculino de 76 años que es llevado por sus familiares al neurólogo, pues presenta
alteraciones importantes en la memoria; en las ultimas semanas ya no ha podido salir de
su casa porque no recuerda como regresar. Al interrogatorio dirigido solo refiere
estreñimiento de varios años de evolución. Exploración física; piel seca y fría, tiroides
pequeña pero aumentada de consistencia, ruidos cardiacos rítmicos con frecuencia de 48
y reflejos con la fase de relajación lenta.
37ª. En este paciente el diagnostico mas probable es:
b)hipotiroidismo primario

37b. la alteración metabolica en este paciente es:


a) T3 y T4 bajas, TSH alta

37c. el mecanismo fisiopatológico causal del caudro clínico de este paciente es:
d)autoinmune
38. femenino de 22 años de edad la cual presenta a consulta de evaluación obstétrica por
encontrarse en el segundo trimestre de su primer embarazo, la paciente menciona que
recientemente ha padecido disuria, frecuencia y urgencia urinaria desde hace 48 horas.
Los cultivos de orina son positivos para escherichia coli por lo que se decide tratar a la
paciente con un esquema de nitrofurantoina durante siete días. Dos semanas después se
repite el urocultivo, una vez que la paciente se encuentra asintomática; el resultado de
este es aun positiva para escherichia coli. Cual de las siguientes conductas terapéuticas es
la mas adecuada en esta paciente?
a) iniciar otro esquema de administración de nitrofurantoina y ordenar la realización de
cultivos subscecuentes para monitorizar por la presencia de bacteuria asintomática

39. paciente del sexo femenino de 20 años de edad se presenta a consulta ginecológica
por amenorrea primaria. A la ef tiene una estatura de 145 cm e hipodesarrollo de los
caracteres sexuales secundarios. La paciente trae consigo un ultrasonido que ordeno otro
medico, el cual revela la presencia de ovarios; sin embargo estos son pequeños y
elongados. Cual de los siguientes mecanismos de enfermedad es el que mejor explica el
padecimiento de esta paciente?
d) error en las primeras mitosis del desarrollo

40. cual de las siguientes características puede diferenciar de mejor manera a la


enfermedad de crohb de la colitis ulcerativa?
b) granulomas no caseificantes

41. paceinte femenino de 20 años de edad estudiante de medicina la cual presenta bajo
rendimiento académico. Atribuye este bajo rendimiendo a una sensación de tensión
generalizada la mayor parte del tiempo, la paciente dice que siemore esta preocuoada por
diversas situaciones, la mayoría aquellas que no puede ni siquiera controlar. Durante la
entrevista es evidente que la paciente no sufre depresión, la ef es normal. Se inicia
tratamiento con buspirona; sin embargo una semana después la paciente se presenta sin
consulta diciendo que el medicamento no funciona y que continua con la misma
sintomatología. Cual de las siguientes opciones terapéuticas es la mas adecuada para esta
paciente?
d)solicitar a la paciente que continúe con el tratamiento actual

42. paciente femenino de 71 años de edad que se presenta con hipercolesterolemia que
es controlada con dieta. En esta ocasiónla paciente se presenta con riguidez y dolor
alrededor de las articulaciones de los hombros y caderas. La paciente menciona que al
despertar siempre le cuesta trabajo salir de la cama. A la ef de las articulaciones no se
encuentra ninguna anormalidad, solo disminución en la fuerza muscular. Cual de las
siguientes opciones es el mejor estudio que se le debe realizar a esta paciente?
d)medición de la velocidad de sedimentación eritrocitaria

43. paceinte femenino de 40 años, indígena, que se presenta a la consulta del centro de
salud y lleva a un neonato de 20 dias de vida. La paciente menciona que su hijo ha estado
flácido desde hace varios días y no acepta el alimento. El recién nadico fue producto de un
parto asistido por partera en la comunidad indigena de donde proviente esta paciente. A
la ef el paciente presenta ictericia leve, crecimiento de la lengua, hipotonía generalizada y
hernia umbilical. Cual de los siguientes es el diagnostico mas probable en este paciente?
a) hipotiroidismo

44. cual de los siguientes antiarritmicos prolonga el potencial de acción y disminución del
flujo de salida celular de potasio, sin afectar los canales de sodio ni de potasio?
a) ibutilide

45. cual de los siguientes medicamentos se asocia con hipopotasemia como efecto
adverso?
d)furosemida

46. una resonancia magnética cerebral revela la presencia de tumoración bilaterales en el


lóbulo occipital de un paciente de 21 años con perdida progresiva de la visión. La biopsia
de musculo esquelético revela la presencia de fibras musculares rasgadas. Entre los
antecedentes de importancia se encuentra que el paceinte tiene una madre de 50 años de
edad la cual presenta de manera interminente debilidad muscular y niveles elevados de
lactato serico. Uno de los hermanos de su mama desarrollo hemiplejia a la edad de 30
años. Si todos estos miembros de la familia, incluyendo el paciente presentan la misma
mutacion genética, cual es el termino adecuado para expresar la variablididad en la
presentación clínica?

a)heteroplasmia

47. cual de los siguientes mecanismos de acción corresponde al de la colestiramina y


colestipol?
A)unión y excrecionde las secreciones biliares

48. paciente recién nadico con cianodis inmediata al parto, la realización de un


ecocardiograma revela la presencia de persistencia del tronco arterioso, cual de los
siguientes defectos es probable que también se encuentre?
b)comunicación interventricular de tipo membranosa

49. paciente masculino de 30 años de edad que se presenta al servicio de urgencias pro
presentar fiebre elevada, escalofríos, dolor en espalda torácica y lumbar, asi como
malestar general en el pasado este paciente había sido hospitalizado debido al uso de
drogas intravenosas; en esta ocasión el paciente reporta que ha consumido este tipi de
drigas en cuatro ocasiones en el ultimo mes, la ultima hace una semana. A la ef el paciente
presenta fiebre de 38.9 fc 109 ta 120/75 y sat 98% con administración de oxigeno de 2L
por canula nasal. Una radiografia de torax demuestra la presencia de un gran numer de
infiltrados nodulares periféricos; estos hallazgos son corroborados por una tomografía
computarizada. Se toman hemocultivos, cuyo resultados todavía no se encuentran
disponibles. Cuantos criterios menores de duke para endocarditis presenta este paciente?
d) 3

50. un bebe de cinco meses de edad es llevado por sus padres a revisión por antecedente
de 48 horas con tos y secreción nasal. A la ef el paceinte presenta temp 37.2 fc 120 y fr 35.
La frecuencia cardiada y el ritmo son regulares, sin presencia de soplos. El pacinte
presenta leve aleteo nasal y ausencia de retracciones intercostales. Los sonidos
respiratorios demuestran esfuerzo respiratorio y en ocasiones se puede percibir
sibilancias. Las membranas timpánicas son normales. De la siguiente conductas, cual
describe el siguiente paso?
d)evaluar el nivel de oxigenación y capacidad para tolerar la via oral

51. paciente masculino de 5 meses de edad el cual presenta anemia hemolítica. Al frotis
de sangre periférica se observan eritrocitos con agregados insolubles de subunidades de
hemoglobina. Al interrogatorio el paceinte fue producto de un embarazo a termino y
parto eutócico; abuelos paternos y maternos son de origen griego. Cual de las siguientes
moléculas fue la principal responsable del adecuado desarrollo intrauterino de este
lactante?
c)Gamma globina

52. paciente masculino de 50 años de edad, acude a consulta por disminución súbita de la
agudeza visual; al interrogatorio refiere que cuenta con el diagnostico de miopía hace 15
años, se realizo cirugía refractiva corneal hace 1 año, niega haber recibido traumatismos
recientes en la región facial; la sintomatología inicio de forma aguda con fotopsias y un
escotoma temporal superior del ojo derecho. FC 75, Fr 18, Pa 130/70, peso 71 talla 1.72

52ª. Con los datos obtenidos en el interrogatorio, usted sospecha de:


c)desprendimiento de retina regmatogeno

52b. para confirmar su diagnostico usted realiza una exploración de fondo de ojo
esperando encontrar:
d)velo grisáceo suspendido en el vítreo

52c. el tratamiento de elección en este paciente es:


c)vitrectomia

52d. para la vigilancia del paciente, cual de los siguientes es la complicación mas común
del manejo?
b)membrana epirretiniana

52e. este paciente puede presentar desprendimiento de retina en el otro ojo, cual es el
tratamiento profiláctico?
b)fotocoagulación con laser
53. paciente masculino de 55 años de edad es llevado por su esposa al servicio de
urgencias por presentar melena desde hace 3 dias. El paciente niega la presencia de dolor
abdominal. A la ef se encuentra fc110 e hipotensión ortostatica significativa. A la
inspección se encuentra atrofia bilateral temporal, conjuntvas palidas, angiomas en araña
en la parte superior del torax, atrofia muscular, hepatoesplenomegalia, a la auscultación
se encuentra hiperactividad de los movimientos intestinales; a la palpación el abdomen no
es doloroso. Durante la exploración física el paciente evacuo heces de características
melenicas. Se decide colocar una sonda nasogástrica la cual aspira material gástrico en
posos de café. Los estudios de laboratorio son caracteristicos por un hematocrito de 0.26,
cual de las siguientes conductas es la mas adecuada de manera inmediata para este
paciente después de su estabilización hemodinámica?
a)realizacionde panendoscopia

54. cual de los siguientes fármacos antibióticos ejerce su efecto mediante la inhibición de
la síntesis de proteínas en bacterias alterando la función de la subunidad 50S ribosomal?
a)clindamicina

55. paciente femenino de 22 años de edad que se presenta al consultorio universitario por
haber presentado desde hace 24 horas fiebre de 38.5C, ardor faríngeo, disfagia, asi como
lesiones papulovesiculares grisáceas en el paladar blando, uvula y tonsilas sin presencia de
gingivitis. Cual es el agente infeccioso que con mayor probabilidad ocasiona estas
lesiones?
d)coxsackievirus

56. paciente masculino de 60 años de edad con discapacidad neurologicam tiene


síndrome de von hippel.lindau es parapléjico, con vejiga e intestino neurogenico. Se
detecto como hallazgo en una radiografia simple de abdomen un calculo grande bilateral
en astas de venado. Cual es la composición mas probable del calculo?
b)estruvita

57. cual de los siguientes enunciados es el que mas adecuadamente representa las
características de la fractura de monteggia?
a) fractura proximal del cubito y luxación de la cabeza del radio

58. paciente masculino de dos semanas de edad que se presenta a la consulta pediátrica
por presentar vomito y letargo. A la ef el paciente presenta hipertonicidad y rigidez
muscular. Al interrogatorio se revela que la madre ha notado un olor a azúcar quemada
cada vez que cambia los pañales del paciente. Cual de los siguientes aminoácidos debe ser
restringido en la alimentación de este paciente?
b)leucina

59. cual de los siguientes métodos de diagnostico temprano es el mas adecuado para
detectar el carcinoma hepatocelular en pacientes con cirrosis?
a)combinación de ultrasonido y medición de alfa-fetoproteina

60. cual de las siguientes infecciones NO incluye en su tratamiento la administración de


esteroides?
a)neumonía por aspergillus fumigatus

61. un niño de 12 alos de edad es llevado al consultorio por sus padres por presentar dolor
durante deambulación y dolor localizado en la ingle y rodilla izquierdas. Al interrogatorio
el paciente menciona que el dolor es constante, sin alteraciones en la sensibilidad. Al
parecer no existio ningún evento que se haya relacionado con el inicio de esta
sintomatología, sin embargo el paciente menicona que el dolor se incrementa durante los
entrenamientos de futbol. Cuando se rota externamente la articulación de la cadera del
paciente, este flexiona la cadera. Cual es el diagnostico mas probable en este paciente?
a)epifisiolisis de cabeza femoral

62. paciente masculino de 4 horas de vida, el cual presenta letargo, vomito y taquipnea.
Los estudios de laboratorio revelan la presencia de acidosis metabolica con brecha
anionica incrementada, cetosis e hipoglucemia, asi como elevación del acido
metilmalonico. Cual de las siguientes reacciones se encuentra deficiente en este paciente?
c)isomerización

63. cual de los siguientes mecanismos de acción productora de diarrea corresponde al de


guardia lamblia?
a)disminución de la capacidad para absorber del intestino debido a la ocupación de la
superficie por este organismo

64. paciente masculino de 66 años de edad acude al consultorio por cefalea de


predominio izquierdo, dolor mandibular y dolor en el hombro desde hace
aproximadamente una semana. No hay antecedentes de importancia revelados durante el
interrogatorio. A la ef el paciente muestra pulsaciones prominentes en la arteria temporal
izquierda, y dolor en esta a la palpación, es posible que tenga una arteritis temporal o de
células gigantes y se requiera biopsia para realizar el diagnostico. Cual de las siguientes
conductas terapéuticas es la mas adecuada en lo que se reciben los resultados de la
biopsia?
a)administrar prednisona

65. cual de los siguientes organismos se asocian con heces de aves y murciélagos, y por lo
tanto se encuentra en cavernas?
c)histoplasma capsulatum

66. cual de las siguientes alteraciones es la única que se asocia a deficiencia de


argininosuccinato sintasa?
e)citrulinemia
67. paceinte femenino de 28 años de edad en tratamiento para embarazarse. En sus
antecedentes no existe problemas relacionados con la fertilidad, a pesar de esto ya ha
sufrido tres abortos espontaneos en el pasado. Los estudioss dirigidos a evaluar la
anatomía de la paciente no han revelado ninguna anormalidad que justifique este cuadro,
entre los antecedentes de importancia se encuentran hematomas de frecuente aparición,
en especial durante la adolescencia. Los estudios de coagulación, como el tiempo de
protombina, el tiempo de tromboplastina parcial y el conteo plaquetario, se encuentra en
rangos normales. En cual de los siguientes factores de la coagulación es mas probable que
haya deficiencia?
e)factor XIII

68. paciente masculino de 41 años de edad acude al consultorio por presentar cefalea
generalizada, intensa y progresiva que inicio hace tres meses. Actualmente la cefalea se
acompaña de vomito en proyectil el paciente ha perdido la capacidad para dirigir la
mirada hacia arriba y en la exploración física se nota que el ojo esta desviado con la
mirada hacia abajo. Cual de los siguientes diagnosticos es el mas probable?
a)tumoración en la región de la glandula pineal

69. cual es el anticonvulsivante de elección en caso de crisis convulsivas simples parciales?


a) carbamacepina

70. paciente masculino de 40 años de edad que se presenta a consulta por presentar
desde hace aproximadamente tres meses prurito generalizado, exantemas eritematosos
súbitos y dolor abdominal tipo colico. A la ef el paciente muestra sibilancias en ambos
pulmones. Se realiza una biopsia de intestino delgado la cual demuestra nidos de
infiltración de mastocitos en la mucosa. Cual de los siguientes hallazgos es mas probable
que este paciente también presente?
C) incremento de la acidez basal del estomago

71. paciente masculino de 55 años de edad que acude a consulta de rutina. En la consulta
se discuten los recientes resultados de laboratorio del paciente que revelan elevación de
colesterol-LDL, asi como triglicéridos. Se le receta gemfibrozil, y se le cita de nuevo en un
mes. Cual de los siquientes resultados de laboratorio es el que se esperaría encontrar en
este paciente en caso de que tenga adecuado apego respuesta al gemfibrozil?
b) disminución intensa en la concentración de trligliceridos, ligera disminución en la
concentración de colesterol -LDL y un ligero incremento del colesterol HDL

72. se trata de una paciente de 75 años de edad, que vive en un medio rural e ingresa al
srevicio de urgencias, fue encontrada por sus familiares en su casa somnolienta y refieren
que durante el trayecto presento crisis convulsivas tónico-clonico generalizadas, ensu
habitación tiene un calentador por combustión encendido. A su ingreso a urgencias con
escala de coma Glasgow 12 puntos (M6 O6 V2), se le observa disneica, con rubicundez
facial y presenta nueva crisis convulsiva, que se mitiga con benzodiacepinas intravenosas.
FC 110 FR 24 tem 35.6 PA 90/50 peso 68 talla 1.52
72ª. Con los datos obtenidos del interrogatorio y exploración, que diagnostico sospecha?
b)intoxicación por monóxido de carbono

72b. para confirmar el diagnostico usted solicida uno de los siguientes estudios?
c)gasometría hemática

72c. basado en los datos clínicos de la paciente, que porcentaje de COHb% espera
encontrar?
c)de 40 a 50%

72d establecido el diagnostico, usted decide iniciar el siguiente tratamiento?


a)oxigeno por mascarilla con reservorio

72e. cual de las siguientes medidas no esta indicada en el tratamiento secundario de este
paciente?
d)anticonvulsivos

73. paciente masculino de 29 años de edad que acude al servicop de urgencia por
presentar disnea en reposo. Menciona qie necesita utilizar tres o cuatro almohadas al
dormir para evitar disnea. Recientemente el paciente ha notado que se le hinchan los pies.
Entre los antecedentes el paciente niega el consulmo de alcoho, hipertensión o
enfermedad arterial coronaria. Hace una semana tuvo un cuadro de infección de vías
respiratorias. A la ef la presión es de 115/76 y FC 105. La presión venosa yugular es de 14
mmHg. Existen estertores en ambos campos pulmonares que se encuentran hasta las
escapulas. A la auscultación cardiaca no se encuentra soplosm pero es posible identificar
el tercer y cuarto ruido cardiaco. En las extremides nferiores tiene edema 2++. El
electrocardiograma no muestra cambios en el segmento ST o en las ondas T. la radiografia
de torax muestra datos en relación a edema pulmonar y aumento del tamaño de la silueta
cardiaca. Existe elevación leve la fracción MB de la creatinina cinasa. Cual de las siguientes
opciones representa la principal causa de los síntomas de este paciente?
d) miocardiopatía dilatada idiopática

74. paciente masculino de 63 años de edad con los siguientes antecedentes de


importancia: hipertensión arterial de 22 años de diagnostico en tratamiento con losartan
50 mg cada 24 horas, diabetes mellitus tipo 2 de 25 años de diagnostico en tratamiento
con metformina, tabaquismo intenso por 10 años, suspendido hace 6 años y dislipidemia.
Acude al servicio de urgencias por presentar dolor torácico de tipo opresivo 4 horas
previas , mientras se encontraba en reposo, con irradiación hacia brazo izquierdo y región
mandibular inferior, de 30 minutos de duración acompañada de nausea y vomito de
características gastroalimentarias por una ocasión. Ha persistido con el dolor. Glasgow 12
puntos. FC 92, FR 22 Temp 35.6 PA 180/100 peso 97 talla 1.68
74ª. Su diagnostico presuntivo es:
c)infarto agudo de miocardio
74b. el abordaje del paciente requiere de un ECG de 12 derivaciones para identificar el
tipo de lesión, aquí se presenta un ECG de 12 derivaciones. Señale el tipo de lesión que se
observa en el trazo:
c)infarto agudo de miocardio con elevación del segmento ST

74c. se recicben los resultados de laboratorio en donde se reporta: troponina I 14.3 CK


128 CK-MB 45, mioglobina 76. En relación con la clínica del paciente y los cambios en ECG
se confirma el diagnostico de IAMCEST. Usted se encuentra en un centro de segundo
nivel, por lo que el manejo inicial es:
a)nitroglicerina sublingual

74d. al obtener mejoría en las cifras de tensión arterial y disminución del dolor, usted
decide que el paciente requiere tratamiento de reperfusion inmediato. Usted se
encuentra en una unidad de segundo nivel y tiempo hasta el siguiente nivel de atención
son mas de 6 horas. El mejor manejo en este momento es:
c)fibrinólisis inmediata y posterior angioplastia en tercer nivel

74e. los criterios de reperfusion son, excepto:


c)desarrollo de bloqueo de rama izquierda

75. femenino de 25 años de edad se presenta a consulta con historia de diarrea de mas de
6 semanas de evolución, la cual se acompaña de distensión abdominal. Refiere haber
observado en algunas ocasiones sangre en las heces. Además, tiene historia de anemia ya
resuelta y comenta que los cereales le causan diarrea desde que era niña. A la exploración
física usted observa a la paciente palida, con apariencia de cansancio y en el área de los
codos multiples ronchas y ampollas que parecen muy pruriginosas.

75ª. Cual es el síndrome clínico principal en esta paciente?


d)diarreico crónico

75b. su principal sospecha diagnostica es:


a)enfermedad celiaca
75c. según su sospecha diagnostica, cual seria el mecanismo fisiopatológico?
c)no digestión de gliadina e inmunológico

75e. el tratamiento para esta paciente es?


c)dieta libre de gluten

76. usted se encuentra informando a un paciente, masculino de 45 años de edad, que


requiere transfusión de concentrados eritrocitarios debido a una hemorragia de tubo
digestivo alta que ya fue controlada mediante endoscopia. El paciente niega antecedentes
de alergias, transfusiones anteriores u otro dato patológico previo, cuando ha
administrado la mitad del concentrado eritrocitario el paciente presenta fiebre, escalofrio,
dolor en el sitio de puncion e hipotensión, sin eritema.
76ª. Que tipo de reacción transfusional sospecha?
b)reacción hemolítica inmune

76b. para confirmar el diagnostico de reacción hemolítica inmune, usted solicita


laboratorios encontrados?
d)coombs directo positivo, DHL y bilirrubina indirecta elevada

76c. en caso de que usted sospecha de toxicidad por citrato. Que datos clínicos esperaría?
a) Signos de chvostek y de trousseau

76d. en relación al daño pulmonar vinculado con transfusión (TRALI), cual es la respuesta
correcta?
d)Se debe a una reacción de anticuerpos contra antígenos de leucocitos

76e. en relación al riesgo de transmisión de enfermedades por transfucion, cual de las


siguientes aseveraciones es correcta?
a) VIH afecta a uno de 1,400,000 a uno de 2,400,00 casos
77. paciente masculino de 25 años, floricultor, originario y residente de Cuautla morelos,
cuenta con antecedente de rinitis alérgica detectada durante la infancia, asi como con
alergia a la penicilina. PA: inicia en la infancia, con cuadros intermitentes de congestion
nasal, que se han intensificado desde hace 2 años y consisten en obstrucción nasal
bilateral de predominio izquierdo, rinorrea anterior y posterior de color amarillento,
hiposmia, sensación de oídos tapados, prurito nadal, estornudo en salva y pesantez de la
cara.
77ª. Cual seria su diagnostico?
c)rinosinusitis crónica

77b. la transiluminacion sirve para valorar:


c)senos maxilares y frontales

77c. agentes etiológicos mas frecuentes de esta enfermedad:


b)s, aureus, estafilococos coagulasa negativos y anaerobios
77d. agente principal de la sinusitis no bacteriana, no viral:
c)aspergillus fumigatus

77e. complicaciones que se observa principalmente en diabeticos?


d)mucormicosis

78. masculine de 40 años de edad, trabajador de la contruccion, fumador y bebedor social,


sin antecedentes de hipertension ni diabetes, acude a consulta por presentar desde hace
un par de semanas adormecimiento de la cara dorsal del dedo medio y de la parte externa
del antebrazo derecho,ademas de notar que le cuesta mas trabajo cargar obtejos con este
mismo brazo. Al realizar la exploracion fisica se encuentra perdida de la sensibilidad en al
region afectada, asi como perdida del reflejo tricipital.
78ª, a que nivel se integra el reflejo bicipital?
b)C5-C6

78c. la prueba de imagen con mayor utilidad es:


c)resonancia magnetica

78e. la causa mas comun de la radiculopatia en mayores de 55 años:


c)espondilosis

79. mujer de 58 años, acude por cuadro de poliartritis simetrica que afecta hombros,
codos, muñeca, IFP,IFD, rodillas,tarso y tobillos, de cuatro años de evolucion; los cuadros
se presentan una vez al mes, y desde hace 4 meses de forma continua tiene gistoria de
uso cronico de glucocorticoides desde hace cuatro años que inicio la enfermedad, en la
actualidad se encuentra en clase funcional IV, poliartritis, multiples nodulos en
interfalangico-proximales y bursas oleocraneanas, con obesidad, hiperglucemia e
hipertension arterial, sus examens de laboratorio con glucosa 120, acido urico de 9,
trigliceridos 300, cr 1.4, BH 10,000 leucocitos, diferencial normal, VSG 40. Las radiografias
de manos y pies muestran erosiones en sacabocado en 2ª IFP derecha y 1ª MTF, el espacio
articular esta conservado. El diagnostico que sospecha es:
a)gota

80. una mujer de 72 años acude a consutla por fatiga y falta de apetido desde hace tres
meses, que ha empeorado ultimamente. En su histotia medica se refiere enfermedad
coronaria, habiendo padecido un infarto de miocardio hace 4 años, tratado con
trombolisis. Ademas refiere osteoartritis, hipertension e hiperlipidemia. Menciona que
tambien ha padecido fiebre intermitente, llegando a 38.5. ha perdido 6k desde el inicio del
padecimiento, sus SV FC 101, FR 19, T 38.2, PA 131/77. A la Ef se observan petequias en
las extremidades y nodulos dolorosos en las falanges terminales de las manos.a la
auscultacion cardiaca se escucha un soplo sistolico grado IV/VI en el segundo espacio
intercostal derecho que se irradia a la clavicula. Al interrogarla refiere que hace cuatro
meses tuvo que someterse a una extraccion dental.
80ª. Cual es el organismo causal mas probable de este padecimiento?
e) Streptococus viridians
80b. que procedimiento diagnostico llevarias a cabo en esta paciente?
e)A,B y C

80c. el hemocultivo da como resultado streptocucus viridans con concentracion inhibitoria


media 0.5 mcg/mL. Que regimen antibiotico se recomendaria?
d)penicilina G 18 millones U/d + Gentamicina 1 mg/kg

80d. cual de las siguientes no es una indicacion de tratamiento quirurgico de endocarditis


infecciosa?
d)desarrollo de glomerulonefritis

81.usted se encuentra haciendo su servicio social en un centro de salud rural. Llega a la


sala de espera un señor con su hija de 3 años de edad en brazos. El padre le comenta que
su hija habia estado perfectamente bien, pero despues de jugar por varias horas en la
granja de su abuelo inicio con dolor abdominal, tos, salivacion y lagrimeo; la lleva con
usted porque no mejora su estado. Usted explora rapidamente a la paciente y nota
dificultad respiratoria e incontinencia de esfinteres.
81ª. Cual es el primer paso que realiza?
b)ABC

81b. despues de realizar la valoracion inicial, que medicamento elige administrar?


d)atropina

81c. cual es la fisiopatologia de este padecimiento, la cual sustenta su decision de


administrar el medicamento de la pregunta anterior?
b)exceso de acetilcolina

81d. con los antecedentes que se mencionan en el caso, que otra medida terapeutica
especifica se debe realizar?
c)remover ropa

82. masculino de 12 años de edad es traido por su mama, la cual señana que el niño
presenta dolores durante la noche en la parte superior externa de la pierna izqiierda. Los
sintomas iniciaron hace una semana, y el dolor cede o dosminuye de intensidad con la
toma de paracetamol o aspirina. A la ef se observa un ligero edema en la aprte
superexterna de la pierna izquierda, y al palpar el trocanter mayor del femur el paciente
refiere dolor intenso. Temp 36.5 ta 125/75 fc 70 . no hay antecedentes recientes de
infecciones ni traumatismos
82ª. El diagnostico mas probable es:
b)tumor oseo
82b. en frecuencia, cual es el segundo tumor maligno oseao?
c)osteosarcoma

82c. es el metodo diagnostico de primera linea


b)radiografias

82d. caracteristicas radiologicas del osteoma osteoide:


c)nido central redondeado radiolucido y rodeado por una zona esclerotica

83. paciente femenino de 35 años acude a urgencias por haber sufrido un accidente
mientras realizaba labores del hogar, durante lo cual agarro un bote de plastico con un
liquido desconocido, que se le resbalo y le salpico dentro de los ojos. Inmediatamente
despues de esto presento un cuado de disminucion de la agudeza visual, fotofobia intensa
y dolor importante. Al realizar el examen del ojo se observan lesiones con bordes bien
delimitados, edema corneal y conjuntiva hiperemica.
83ª. El diagnostico probable es:
c)quemadura por acido

83b. la primera medida que se toma en un accidente de este tipo es:


c)lavado con agua o suero inmediato

83d. el tratamiento medico debe ser:


c)pilocarpina topica a irrigacion corneal y fondos de saco con solucion salina durante 30
min o 1L minimo a ocluir el ojo

83e. que agentes provocan coagulacion de proteinas:


c)fuego directo y acidos

84. paciente masculino de 42 años de edad actualmente hospitalizado por evento vastular
cerebral, se interconsulta por presentar rinorrea purulenta, fiebre y halitosis. A la ef se
encuentra bajo intubacion orotraqueal y con una sonda nasogastrica en fosa nasal
derecha; presenta una deflexion septal izquierda contactante y salida de material
purulento por ambos meatos medios.
84ª. Cual es el germen mas frecuente que ocasiona el cuadro del paciente?
a)proteus

84b. cuatro dias despues el paciente presenta proptosis y quemosis izuqierda, limitacion
en la aduccion del ojo izquierdo y disminucion en la agudeza visual izquierda, cual es el
diagnostico del paciente?
e)celulitis orbitaria

84c. cual es el tratamiento para esta complicacion?


a) Cirugia endoscopica funcional en caso de no mejorar con antibioticos, irrigacion
nasal y descongestionantes
85. cual de los siguientes enunciados es correcto respecto de los pacientes
homocistinuria?
a)los pacientes se pueden beneficiar mediante una dieta baja en metionina y alta en
cisteina

86. ante un neonato de 20 dias de edad, con antecedentes de tapon meconial, que ha
cursado con vomitos fecaloides, distension abdominal, rechazo al alimento y retraso del
crecimiento, se solicita analisis histopatologico de biopsia rectal. Que hallazgos espera
esncontrar?
a)ausencia de celulas ganglionares en plexos mucoso y submucoso

87. lactante de 6 meses de edad, con antecedentes de dos cuadros de urticaria y


dermatitis atopica, que presenta vomito de contenido alimentario y diarrea acuosa, sin
retraso del crecimiento. Cual es el tratamiento mas adecuado para sus sintomas
digestivos?
d)sustituir la formula de leche por hidrolizado de proteinas

88.paciente masculino de 55 años con DM2 diagnosticada hace un año. Acude al servicio
de urgencias con historia de compromiso del estado general progresivo en los ultimos tres
meses, perdida de peso, y sudoracion nocturna. La semana previa al ingreso se agrego
dolor en flanco y fosa iliaca izquierda, fiebre de 38.5 y vomito escaso. Al examen fisico se
observa caquectico, palido, febril, taquicardico, destacando muget en la cavidad bucal. Se
realiza tomografia de abdomen, que revela multiples adenopatias hipodensas
retroperitoneales y a nivel de la raiz del mesenterio. Se confirma inmunodepresion
asociada a VIH. Despues se realiza puncion de adenopatias obteniendose baciloscopia +
para tuberculosis. La biopsia mostro abundantes bacilos intracelulares.
Respecto a las distindas formas clinicas de tuberculosis, señale la aseveracion falsa:
e)la afeccion intestinal mas frecuente es duodeno-yeyunal

89. paciente masculino de 63 años acude a consulta por perdida de 18 kg de peso (>20%
peso habitual) y fiebre vespertina de cinco meses de evolucion. Dentro de sus
antededentes tiene un indice tabaquico de cunco. A la ef se observa febril, adelgazado,
disneico, FC 100, Ta100/70, sin ortostatismo, hepatomegalia. Sin otras lesiones. El
diagnostico de tuberculosis se hace por:
d)estudio bacterologico

90. paciente femenino de 55 años con antecedemtes de diaetes mellitus descontrolada es


traida a urgencia en ambulancia por presentar disuria y polaquiuria asociada con dolor en
el flanco izquierdo de cuatro dias de evolucion, agregandose malestar general, fiebre y
escalofrios desde hace dos dias. Sus SV temp 39, FC 100, TA90/60 FR18. BH leucocitos de
10,000 con 20% de bandas, plaquetas 130,00, usted diagnostico un sindrome de respuesta
inflamatoria sistemica y mantiene una alta sospecha clinica de pielonefritis con urosepsis.
Los siguierntes son criterios del sindrome de respuesta inflamatoria sistemica, excepto:
c)presion arterial <110/60mmHg
91. la siguiente es una enfermedad definitoria de SIDA:
a)carcinoma cervical invasor

92. la forma de entamoeba histolytica que infecta al ser humano al ser ingerida es:
c)quistes

93. paciente masculino de 59 años de edad que se ha sometido a un transplante


ortotopico de corazon hace seis años miocardiopatia isquemica. Ha tenido tres episodios
de rechazo, los cuales han sido tratados con dosis intravenosas de metilprednisolona.
Cuales de las siguientes opciones representan las complicciones que puede sufir este
paciente?
e)todas las opciones anteriores representan posibles complicaciones en esta paciente

94. el sindrome de stein-leventhal se presenta con hirsutismo por:


a)transtorno funcional ovarico, con relacion LH:FSH mayor a 2

95. la madre de un recien nacido de una semana de vida, con antecedentes de parto
vaginal instrumentado, le pregunta sobre una depresion que sintio en el craneo del
neonato. Al explorarlo, usted palpa la fontanela anterior. De acuerdo con su impresión
diagnostica, que informacion le indicaria a la madre del neonato?
d) es normal y desaparecera entre los nueve y 18 meses de edad cuando se cierren las
suturas craneales

96. un hombre de 75 años con larga historia de disfuncion cardiaca de varios años que
ocurrio posterior a un infarto lateral del miocardio. Como tratamiento en aquel entonces
se le realizo un bypass con arteria mamaria. El resto de sus antecedentes patologicos
incluye diabetes, IRC, HTA e hiperlipidemia. Sus medicamentos incluye metropolol,
lisinopril, aspirina, roxuvastatina, furosemida, nitroglicerina en caso de angina e insulina.
En los ultimos meses ha sentido un deterioro de su estado al referir mas fatiga. Menciona
que ha dejado de trabajar en el jardin como lo hacia antes y con solo unos pasos tiene
disnea. Refiere de vez en cuando despertarse en las noches con disnea requiriendo
levantarse para que mejore, y usa dos almohadas para dormir. Niega dolor toracico y no
ha requerido utilizar nitroglicerina sublingual. A la ef se observa edema periderico + y sus
SV FC88 FR 18 T 37.4 pa 123/78, se auscultan S1 y S2 sin agregados y leves estertores
pulmonares. Se observan distension venosa yugular, auqneu no se palpan
organomegalias. Su creatinina y nitrogeno ureico estan estables desde la visita anterior. Se
realizan un ecocardiograma que revela una fraccion de eyeccion de 20% comparado con
uno hace un año que era de 35%, cual de los siguientes medicamentos estaria indicado
agregar a este paciente para disminuir sus sintomas y mortalidad?
d)espironolactona

97. una mujer de 67 años fue sometida a reemplazo de cadera hace 6 dias posterior a una
fractura. Como medicamentos ha recibido fentanilo, morfina, atorvastatina, metropolol y
heparina subcutanea. Llaman porque la paciente ha desarrollado dolor de la extremidad
inferior izquierda. A la ef paciente concompresas neumaticas y la extremidad izquierda se
observa con edema. el signo de homan es positivo. Se realiza ultrasonido doppler,
encontrando trombosis venosa. La cuenta de plaquetas es de 63,000. No hay evidencia de
sangrado o hematomas. Cual de las siguientes medidas preventivas seria la mjor para esta
paciente?
b)descontinuar la heparina

98. femenino de 35 años, acude al servicio de urgencias por referir fiebre desde hace mas
de tres dias, dolor en epigastrio asi como cansancio leve. Antecedentes: trasnplante renal
hace mes y medio, asi mismo trae consiglo los siguientes resultados de laboratorios:
leucopenia 2400, transaminasas ALT 75, AST 89. En esta paciente el diagnostico mas
probable es:
b)infeccion por citomegalovirus

99. femenino de 54 años de edad acude al servicio de urgencias por presentar fiebre,
ictericia, acolia y coluria ademas de dolor tipo colico en hipocondrio derecho. El
diagnostico mas probable en esta paciente es?
a)colangitis por obstruccion

100. masculino que se presenta a consulta externa, refiere que su pareja ha sido
diagnosticada con infeccion de virus del papiloma. La forma mas frecuente de la infeccion
por papilomavirus en varon es:
c)subclinica en genitales

101. masculino de 73 años llevado a servicio de urgencias. Antecedente: acude a una cena
familiar de celebracion y pasa una gran noche. Al dejar el restaurante sufre un colapso. La
entidad causal de la sintomatologia de este paciente es:
d) sincope posprandial

102. paciente femenino de 25 años de edad la cual acude a consulta por padecer
dispareunia, fiebre, dolor abdominal en hipogastrio principalmente y escasa secrecion
vaginal maloliente y sanguinolenta. Entre los antecedentes de importancia la paciente
menciona que es sexualmente activa desde los 14 años de edad, con multiples parejas
sexuales, y no siempre utiliza prorteccion contra ETS. A la ef se presenta dolor a la
movilizacion crevical. Se sospecha enfermedad pelvica inflamatoria. Cual de los siguientes
organismos se relaciona con esta enfermedad:
a)neisseria gonorrhoeae

103. cual de los siguientes estudios de laboratorio corresponde a un paciente con


osteoporosis?
a)calcio normal, fosfato normal, fosfatasa alcalina normal
104. cual o cuales de las siguientes aseveraciones en relacion con la miocardiopatia
hipertrofica son las mas correctas?
a)hipertrofia asimetrica, de mayor intensidad en al region septal

105. paciente masculino de 20 años de edad que hace cuatro semanas tuvo una relacion
sexual homosexual sin proteccion. El dia anterior el paciente recibio la noticia de que
dicho compañero sexual era portador del virus de la inmunodeficiencia humana. Cual de
las siguientes opciones representa mas correctamente los resultados de laboratorio en
este caso de que este individuo se encuentr infectado en este momento?
a)disminucion en el numero de linfocitos T CD4+ e incremento en los niveles del antigeno
P24

106. paciente masculino de 22 años de edad se presenta a la consulta externa, desde hace
cuatro años se le diagnostico asma; en esta ocasión acude debido a que al parecer la
presencia de sibilancias y disnea se ha hecho cada vez mas frecuente. El paciente informa
que presenta sintomatologia alrededor de tres veces por semana y que durante el ultimo
mes se desperto tres noches debido a ataques de disnea. Refiere que los sintomas
mejoran cada vez que inhala el agonista beta adrenergico. Cual de las siguientes
aseveraciones describe la mejor conducta terapeutica en este paciente?
c)iniciar tratamiento con dosis bajas de corticoesteroides inhalados

107. cual de los siguientes tipos de foliculos se caracteriza por localizacion estromal y la
presencia de un antro?
b) foliculo secundario

108. cual de las siguientes estructuras podria resultar dañada en caso de una lesion
penetrante por arma punzocortante en el quinto espacio intercostal al nivel de la linea
media axilar del lado derecho?
a) higado

109. cual de las siguientes afiermaciones NO es correcta respecto a la intoxicacion con


monoxido de carbono?
c)el uso de oxigeno hiperbarico se recomienda cuando los niveles de carboxiheemoglobina
se encuentran por arriba del 15%

110. paciente de 5 años de edad que acude a consulta pediatrica por presentar desde
hace una semana una tumoracion cervical. La madre refiere que la aparicion de esta
tumoracion coincide con una infeccion respiratoria de vias aereas superiores. A la ef se
encuentra una tumoracion en la linea media del cuello de aprox 2.5 cm por encima del
cartilago tiroideo. Cuando se le solicita al paciente que protruya la lengua, la masa se
eleva. Cual es el diagnostico mas probable en este paciente?
c)quiste del conducto tirogloso
111. paciente masculino de 55 años de edad el cual se presenta al servicio de urgencias. El
paciente refiere que desde hace dos dias ha presentado fiebre, dolor abdominal de leve
intensidad y cefalea. El paciente menciona que tiene malestar general similar al de un
resfriado un poco mas fuerte de lo habitual. El paciente refiere que los sintomas iniciaron
en el momento que viajaba en un avion de regreso a un viaje que hizo con su esposa al
africa. cual es la conducta mas adecuada a seguir en este paciente?
a)realizar un frotis de sangre periferica

112. cual de las siguientes NO es una causa de masa mediastinal posterior?


b)timoma

113. cual es el tratamiento de eleccion en el caso de portadores asintomaticos de str


ngyloides stercoralis?
c)ivermectina

114. cual de los siguientes hallazgos se puede identificar en un paciente con mordedura de
la araña vuida negra?
b)espasmos musculares dolorosos

115. cual es el principal efecto secundario de la administracion de tenofovir en pacientes


con infeccion del virus de la inmunodeficiencia humana?
a)sindrome de fanconi

116. se presenta a consulta pediatrica un neonato de 18 dias de vida con cataratas


bilaterales sublenticulares. Se solicitan estudios de funcion hepatica, los cuales resultan
normales. A la ef hay ausencia de hepatoesplenomegalia. Cual es la enxima cuya ausencia
es mas probable en este paciente?
a)galactocinasa

117. se trata de paciente femenino pretermino que al nacer manifesto cianosis y


freciuencia cardiaca de 45. Se inicio reanimacion neonatal procediendose a ventilacion
con presion positiva y se reevaluo despues de 30 segundos encontrandose con frecuencia
cardiaca 59 x min y coloracion rosada generalizada. Señale cual de los siguientes es el
siguiente paso a seguir de acuerdo al manejo durante la reanimacion neonatal:
e) dar compresiones toracicas externas

118. paciente masculino de 63 años con antecedentes de alcoholismo cronico es traido or


un vecino al encontrarlo incosciente en el parque. A la ef se observa paciente caquectico,
incosciente, sin pulsos perifericos palpables. El monitor muestra taquicardia ventricular
polimorfica con frecuencia de 240 lpm. Usted comienza la reanimacion pulmonar,
administra tres descargas, adrenalina IV seguido de otras tres descargas, sin éxito.Cual es
la siguiente medida terapeutica que usted realizaria en este momento?
d)sulfato de magnesio
119. paciente masculino de 51 años de edad que es traido al servicio de urgencias por una
ambulancia. El reporte inform que el paciente se encontraba en clase de spinning en el
gym, cuando sufrio un sincome con lesion subsecuente al area fronta; el reporte tambien
meniona que no ocurrienron movimientos tipo crisis convulsivas y la duracion de la
perdida del estado de alerta fue de aproximadamente 30 s. al interrogatorio sobre el
evento el paciente niega sintomas presincopales. Sin embargo, refiere que en los ultimoas
meses ha padecido disnea leve sin llegar al sincope o inclusive presincope. Cuando se le
interrogo sobre un farmaco o uso de drogas ilegales, el paciente menciona que
recientemente se sometio a una intervencion dental y que tuvo que tomar antibiotico
profilactico debido a una condicion cardiaca, sin poder especificar cual. A la ef normales, a
excepcion de hipotension arterial leve 110/55; pulso radial se describe como de lenta
elevacion. A la ex cardiada se encuentra incremento de la intensidad del componente
aortico del segundo ruido cardiaco; en el segundo se encuentra incremento de la
intensidad del componente aortico del segundo ruido, el segundo espacio intercostal
derecho se puede percibir un clic de eyeccion seguido de un soplo de eyeccion sistolico. A
la palpacion toracica se encuentra incremento en la intensidad del punto maximo impulso
cardiaco, sin desplazamiento de este hacia la izquierda o derecha. Utilizando unicamente
la informacion previamente descrita, responda las siguientes preguntas?
119ª. Cual de las siguientes opciones es el diagnostico mas probable en este paciente?
c)estenosis aortica

119b. cual de los siguientes hallazgos electrocardiograficos es el que se presentaria con


mayor probabilidad en este paciente?
c)desviacion del eje electrico cardiaco hacia la izquierda

119c. cual de los siguientes hallazgos ecocardiograficos se encontrarian con mayor


probabilidad en este paciente=
b)incremento del gradiente entre el ventriculo izquierdo y la aorta

119d. cual de las siguientes opciones representan mejor la situacion de este paceinte?
b)una vez establecida la insuficiencia cardiaca, angina o sincope, la mortalidad sin cirugia
es del 50% a 36 meses

120, llega una mujer de 31 años da urgencias ginecoobstetricia con embarazo de 37 sdg
G3 C2 el lapso entre los 3 embarazos fue de 10 a 12 meses. Acude porque hace 2 horas
inicio con contracciones 3 cada 10 min, con duracion 50 seg. Pero que es subitamente
presento dolor suprapubico intenso. Refiere salida de tapon mucoso y despues
hemorragia transvaginal leve. A la ef: FC 98, FR 19, temp 37, pa 125/80. Se observa
angustiada, palida y diaforetica, cardiopulmonar sin compromiso. Abdomen con utero
getante de 33 cm de longitud desde sinficis del pubis, situacion longitudinal, presentacion
pelvica, posicion izquierda se palpal con facilidad las partes fetales, FCF 130 lpm, y no se
pervibe actividad uterina, cual es el factor mas simportante para desarrollar la patologia
que tiene esta paciente?
a)cicatriz uterina previa
121. paciente femenino de 18 años de edad acude a consutla acompanada de su madre,
esta refiere que desde hace 2 semanas su hija ha presentado nausea y en 2 ocasiones ha
llegado al vomito de contenido alimentario. Al interrogatorio sin la madre la paciente
refiere astenia, adinamia, disuria, polaquiuria, urgencia miccional, con retraso en la
mentruacion de 4 semanas. Refiere actividad sexual. El abdomen se encuentra blando,
depresible, no doloroso y a la exploracion vaginal se observa la mucosa vaginal de color
rojo violaceo. Cual de los siguientes NO es undato de probabilidad de embarazo?
b)red venosa de haller

122. acude a consulta paciente femenino de 37 semanas de gestacion G3 P2, no presenta


datos de vasoespasmo y el embarazo ha sido normoevolutivo. A la ef: FC, FR Temp, TA sin
alteraciones. Paraconocer la situacion fetal, que maniobra de leopold realizaria?
b)segunda maniobra de leopold

123. llega a consulta paciente primigesta de 24 años de edad con 34 semanas de


embarazo, por iniciar con contracciones hace 2 horas. No refiere salida de tapon mucoso
pero si sangrado transvaginal de moderada cantidad. Sin datos de vasoespasmo FU 29 cm,
FCF 150lpm, contracciones cada 5 min con una duracion 40 seg. Cuello uterino dehiscente
central, firme, borramiento 20%. Usted piensa que la administracion de un supositorio de
indometacina no es adecuada en el paceinte. Cual de los siguientes es un efecto adverso
de la indometacina?
c)hemorragia intraventricular, cierre del conducto arterioso y enterocolitis necrosante

124. neonato en su segunda semana de vida extrauterina, producto de primera gestacion


de 30 sdg, nacimiento por parto vaginal eutocico, con rotura prematura de membranas
(de 24 horas), que peso 2,200 gramos. Entre sus comorbilidades presenta onfalitis desde
la primera semana de vida. Actualmente presenta FCF 190 lpm, FR 55 y temp 39, con
42,000 leucocitos en la BH. Cual de los siguientes agentes etiologicos es la causa mas
frecuente de sepsis neonatal de inicio tardio?
d)staphykiciccus epidermidis

125. esta en urgencias con una paciente con diagnostico de preeclampsia grave. Se
encuentra recibiendo hidralacina y sulfato de magnecio. Al explorarla presenta
hiporrelfexia, con ocho respiraciones por minuto y oliguria. Que medicamento utilizaria?
c)gluconato de calcio

126. una mujer de 71 años se presenta a su medico de primer concacto con un a historia
de un mes de evolucion de cefalea, fiebre y malestar general. Las cefaleas las describe
como dolor pulsatil, localizado en la region temporal bilateral. Niega traumatismo o
infecciones recientes. A la ef registra una temperatura normal FC 82, TA 130/80 mmHg,,
dolor a la palpitacion y abombamiento de arterias temporales bilaterales.
126ª. Cual es el diagnostico mas probable?
b)enfermedad de horton
126b. cual es el tratamiento de inicio?
c)prednisona oral

126c. cual no es un criterio de clasificacion de las arteritis del temporal?


d)claudicacion/dolor de mandibula

126d. la paciente ademas tiene posibilidad de presentar:


b)polimialgia reumatica

126e. el tratamiento no se debe de atrasar para evitar la siguiente complicacion:


a)ceguera

127. mujer de 30 años de edad con amenorrea de tres meses de evolucion. Refiere
menstruaciones regulares previas. Con antecedentes de legrado uterino por aborto.
Prueba de embarazo negativa, niveles de TSH y prolactina normales, no menstrua despues
de administrar progestagenos y tampoco tras la administracion de estrogenos y
progestagenos, cual es la posible causa de la amenorrea secundaria?
a) sindrome de asherman

128. cual es el valor del sodio corregido en un paciente que llega al servicio de urgencias
con Na en 120 y una glucosa en 853?
c)132

129. cual de los siguientes marcadores tumorales es el mas adecuado para detectar
enfermedad recurrente posterior al tratamiento del cancer testicular no seminomatoso?
d)alfa fetoproteina

1. Paciente masculino de 22 años de edad acude al servicio de urgencias por exacerbación


de asma. Se conocía asmático desde hace ocho años. A pesar de que se mantenía en un
nivel de asma leve intermitente, con administración de broncodilatador por razón
necesaria, el día de hoy experimentó mayor obstrucción de la vía aérea, esto lo llevó a
utilizar en varias ocasiones el medicamento inhalado. Al no observar mejoría decidió
acudir al servicio de urgencias. A su ingreso es colocado bajo tratamiento con
nebulizadores. A pesar de que mejora en cuanto a su obstrucción de la vía aérea, presenta
náusea, vómito y debilidad generalizada. El electrocardiograma muestra la presencia de
ondas U. ¿Cuál de los siguientes medicamentos es el responsable de estos últimos signos y
síntomas?
a) Albuterol

2. ¿Cuál de los siguientes medicamentos se asocia con la disminución de la excreción de


agua libre como complicación?
a)Clorpropamida
3. ¿Cuál de los siguientes trastornos se caracteriza por disminución de los niveles de la
hormona luteinizante, normalidad en los niveles de la hormona foliculoestimulante e
incremento de la testosterona en sangre, en combinación con un bajo conteo de
espermatozoides?"
a) Administración de andrógenos

4. Paciente masculino de 39 años de edad se presenta a consulta por antecedente de


disnea en reposo desde hace cinco días, así como tos no productiva. Se decide tomar una
radiografía de tórax, la cual revela infiltrados difusos bilaterales; la citometría hemática
reporta un valor de conteo leucocitario de 2 000 células/mm3. Al ver los resultados de
laboratorio se interroga al paciente sobre la posibilidad de infección por el virus de la
inmunodeficiencia humana; el paciente acepta, y menciona que no lo reportó desde el
principio porque creía que no era necesario para que atendieran su padecimiento actual.
Se realiza una tinción de metenamina de plata de esputo, el cual revela al organismo
causal, y se inicia tratamiento específico dirigido de primera elección. ¿Cuál de los
siguientes mecanismos de acción es el del medicamento administrado?
e) Bloqueo de la formación de nucleótidos

5. ¿Cuál de los siguientes antiarrítmicos actúa con mayor selectividad en células de


despolarización de alta frecuencia?
c) Lidocaína

6. Paciente masculino de 45 años, con antecedentes personales de cólicos nefríticos de


repetición. Con antecedentes heredofamiliares de cáncer de próstata en su abuelo, su
padre y seis de sus ocho hermanos, a edades tempranas. Comienza hace tres meses con
disuria, hematuria, polaquiuria y dolores óseos. Consulta y es tratado con antibióticos. Al
tacto rectal se detecta agrandamiento prostático, se realiza biopsia que muestra
carcinoma prostático y se decide operarlo realizándose prostatectomía parcial con
vaciamiento testicular bilateral, antígeno prostático específico mayor de 100 ng/ml.
Continúa tratamiento médico con flutamida; no mejora su disuria ni polaquiuria.
Señale la afirmación correcta:
b) Cuarenta por ciento de los cánceres de comienzo temprano es hereditario

7. Paciente masculino de 18 años de edad que se presenta con una tumoración en la


región del tendón de Aquiles. El médico tratante decide obtener una biopsia de la región,
la cual revela la presencia de macrófagos con alto contenido de colesterol (células
espumosas), los cuales se localizan entre las fibras de colágena. Al interrogatorio de los
antecedentes heredofamiliares el paciente menciona que tanto la madre como el padre
han desarrollado artritis grave en el pasado asociada con la producción de xantomas; sin
embargo, sus padres presentaron este cuadro después de los 40 años de edad. ¿Cuál de
las siguientes partículas en sangre es la que con mayor probabilidad se encuentra más
elevada en este paciente?
e) Lipoproteínas de baja densidad (LDL}
8. ¿Cuál de los siguientes mecanismos de acción corresponde al de los fibratos?
d) Incremento de la actividad de la lipoproteína lipasa

9. Paciente femenino de 69 años de edad se somete a cirugía para resección de una


tumoración solitaria no dolorosa en la región anterior al trago de su oreja izquierda. A la
evaluación histopatológica, se observa un tumor blanco amarillento bien delimitado, con
proliferación de células epiteliales y mioepiteliales. En el reporte quirúrgico se menciona
que durante el procedimiento se lesionó la división inferior del nervio facial. ¿Cuál de los
siguientes hallazgos podría estar presente con mayor probabilidad en esta paciente?
e) Debilidad para mover la mitad izquierda del labio inferior

1O. ¿Cuál de los siguientes enunciados referentes a la infección por Giardia lamblia es
correcto?
d) Tan sólo laingestión de 10 quistes de Giardia lamblia puede producir infección

11. Sujeto masculino de 30 años que se presenta en urgencias por intenso dolor tipo
cólico de inicio súbito, localizado en flanco izquierdo que se irradia del abdomen bajo
hacia el escroto. Sin antecedentes heredofamiliares de importancia. Al interrogatorio
dirigido refiere polaquiuria y disuria. A la exploración física se encuentra afebril,
hiperestesia en el ángulo izquierdo costovertebral. Examen general de orina que muestra
hematuria microscópica. Se estudia el sedimento que confirma la presencia de hematuria
y muestra la presencia de cristales bipiramidales tetragonadales al observarlos con lupa
binocular. ¿Cuál de los siguientes es la anormalidad metabólica más comúnmente
asociada con la enfermedad de este paciente?
a) Hipercalciuria

12. Una niña de nueve años de edad es llevada a consulta por sus padres por presentar
fiebre desde hace 48 horas y malestar faríngeo, con intensa odinofagia a sólidos, cefalea
leve y dolor abdominal difuso. Al interrogatorio se niega la presencia de tos o dificultad
respiratoria. Se decide realizar una prueba rápida de detección de antígeno
estreptocócico, la cual resulta positiva. La paciente no presenta alergias medicamentosas
¿Cuál de los siguientes medicamentos es el más adecuado para la paciente?
b) Penicilina V oral a dosis de 250 mg/tres veces al día durante 10 días

13. Paciente masculino de 39 años de edad que desde hace cuatro meses manifiesta
episodios de dolor en hipocondrio con fiebre, los cuales son autolimitados. Entre los
antecedentes de importancia está el diagnóstico de colitis ulcerativa a la edad de 31 años,
ahora en etapa de remisión y bajo tratamiento con sulfasalazina, 2 g/día. A la exploración
física se encuentra ictérico y con hepatoesplenomegal ia. Los estudios de laboratorio
muestran moderada elevación de las transaminasas hepáticas y bilirrubina, pero una
marcada elevación de los niveles de fosfatasa alcalina. ¿Cuál de los siguientes diagnósticos
es el más probable en el paciente?"
e)Colangitis esclerosante primaria
14. Paciente femenino de 84 años con demencia vascular de varios años de evolución,con
pérdida total de la independencia funcional, postrada en cama desde hace
aproximadamente tres años, padece miocardiopatía dilatada, en tratamiento con
digoxina, furosemida, enalapril, ácido acetilsalicílico. Paciente en mal estado general,con
lenguaje incomprensible, con desnutrición extensa, piel pálida,seca e inelástica, con
pliegue persistente, hipotrofia muscular generalizada, mucosas secas, halitosis, dentadura
en mal estado, presenta principios de escaras sacras y bitrocantéreas. ¿Cuáles son
factores de riesgo para úlceras por presión?
d)Incontinencia, inmovilidad y desnutrición

15. ¿Cuál de los siguientes enunciados es el que más adecuadamente representa las
características de la fractura de Barton?
d) Fracturas con desprendimiento delborde dorsal o posterior de porción distal delradio

16. ¿Cuál de las siguientes deficiencias enzimáticas se presenta en la fructosuria esencial?


e) Fructocinasa

17. Niño de cuatro años que presenta en cuello, fosas antecubitales y poplíteas múltiples
pápulas eritematosas, milimétricas, aisladas y confluentes que forman placas en cuya
superficie se observan costras hemáticas. La madre refiere una evolución de 15 días con
presencia de prurito intenso durante el día y la noche. Tiene antecedente de rinitis
alérgica desde hace un año y su padre sufrió de asma bronquial en la infancia. Los
exámenes de laboratorio se reportan dentro de valores normales. ¿Cuál es el diagnóstico
más probable?
a)Dermatitis atópica

18. Paciente masculino de ocho años de edad, cuenta con el diagnóstico de asma desde la
infancia. Entre sus antecedentes destaca un episodio de asma previo que requirió de
intubación endotraquealy ventilación mecánica, además de visitas frecuentes al servicio
de urgencias por cuadros de exacerbación del asma. En esta ocasión acude al servicio de
urgencias por presentar tos, sibilancias, disnea en reposo y dolor torácico de 30 minutos
de evolución,a la exploración física se encuentra con frecuencia respiratoria de 30 rpm,
frecuencia cardiaca de 120 lpm, saturación de 88% al aire ambiente, agitado, con uso de
músculos accesorios de la respiración, retracción supraesternal y sibilancias audibles
a distancia,en su gasometría arterial con Pa02 de 55 mm Hg y PaC02 de 46 mm Hg. Su
placa de tórax es normal. En este tipo de pacientes."
18a. ¿Qué factores se consideran como de alto riesgo para presentar muerte relacionada
al asma?
e)Todas las anteriores

18b. De acuerdo a los signos y síntomas del paciente, su exacerbación se clasifica como:
c) Grave
18c. El manejo inicial de este paciente debe incluir lo siguiente:
e) Oxígeno suplementario, agonista 82 de acción rápida inhalado y esteroides sistémicos"

19. Femenino de 36 años acude a consulta externa refiriendo antecedentes: 2 años de


relaciones sexuales sin anticoncepción, hace 1 año presenta dismenorrea, dispareunia y
sangrado vaginal intermenstrual. Con los datos anteriores el diagnóstico de esta paciente
es:
d) Endometriosis

20. Paciente femenino gesta 1, partos O, abortos O, cesáreas O, acude para evaluación
obstétrica. En un ultrasonido de control se reporta un peso fetal estimado por debajo del
décimo percentil para la edad gestacional. Debido a que la paciente no acudió a control
obstétrico desde el inicio del embarazo, no es posible obtener una edad gestacional con
precisión. La paciente menciona que desde hace más de cinco meses notó el crecimiento
del perímetro abdominal. Se realiza una amniocentesis que revela un índice
lecitina/esfingomielina > 2. ¿Cuál es la conducta terapéutica a seguir?
e) Interrumpir elembarazo

21. Mujer de 54 años de edad quien presenta una dermatosis bilateral y con tendencia a
la simetría que se disemina a ambas caras pretibiales. Se caracteriza por 3 placas
infiltradas amarillo-violáceas y esclerodermiformes de forma irregular y mal delimitadas,
que miden entre 5 y 7 cm. La paciente refiere que sus lesiones iniciaron hace un mes
como granitos de color rojo-violáceo y otros como nódulos de forma redondeada, de
bordes no bien definidos y a veces elevados. Fueron creciendo de forma lenta y
excéntrica, tornándose de color amarillo, con telangiectasias en la superficie. Una de las
lesiones se ulceró, dejando cicatriz y atrofia posterior.A la exploración física la paciente
presenta en las lesiones sensibilidad disminuida al pinchazo y al tacto leve, hipohidrosis y
alopecia parcial. La paciente es diabética de larga evolución y al parecer lleva un buen
control. Niega antecedentes de traumatismos o de infecciones previas. El diagnóstico es:
b) Necrobiosis lipoídica

22a. De acuerdo con los antecedentes, ¿cuál es el diagnóstico probable?


a) Ictericia fisiológica

22c. ¿Cuál de los siguientes mecanismos NO predispone a los neonatos a la


hiperbilirrubinemia?
b) Aumento de la unión y transporte

22d. Si el paciente descrito continúa con la elevación de niveles de bilirrubina y se cree


que puede entrar en rangos tóxicos, ¿qué medida
terapéutica está indicada?
b) Fototerapia
23. Con relación a la excreción renal del ácido úrico:
e) Los procesos implicados en la excreción de uratos se realizan en el túbulo proximal

24. Paciente masculino de 13 años de edad, acude por dolor escrotal derecho de 12 h de
evolucion. Ha sido progresivo, pero ahora permanece constante y, según menciiona la
madre, hubo un episodio de fiebre. En la exploracion fisica se aprecia dificultad a la
deambulacion, aumento de volumen en la region escrotal y eritema, especialmente del
lado derecho; reflejo cremasteriano presente y signo de Prehn positivo.

24a. Con base en las características del paciente, ¿cuál es su sospecha diagnóstica?
d) Epididimitis

24b. De acuerdo con su sospecha clínica,¿qué hallazgos espera encontrar durante la


exploración física con transiluminación?
b) Líquido libre

24c. Usted decide realizar una ecografía doppler.De acuerdo con su sospecha clínica, ¿qué
hallazgos espera encontrar?
c)Flujo arterial normal

24d. Si la exploración física y la ecografía concuerdan con los hallazgos esperados de su


sospecha clínica, ¿cuál es el tratamiento indicado?
c) Antiinflamatorios, antibióticos y reposo

24e. En conclusión, ¿cuál de las siguientes opciones menciona las características que se
presentan en el padecimiento de este paciente?
c)Todas las edades, dolor leve a moderado, posición normal del testículo, reflejo
cremastérico presente, signo Prehn positivo

25. Masculino de 38 años quien es llevado al servicio de urgencias en ambulancia después


de ser agredido por terceras personas, quejándose de dolor intenso en tórax. A la
exploración física, signos vitales: PA 100/60 mm Hg, FR 24 respiraciones/min, FC 104
latidos/min; dolor intenso a la palpación en hemitórax izquierdo a nivel del cuarto y
quinto arcos costales; pulmón izquierdo hipoventilado en la base, a la percusión en misma
región con zona de matidez. Se administran soluciones cristaloides intravenosas y
analgésicos.
¿Cuál es el tratamiento inmediato en este paciente?
a) Colocación de una sonda pleural izquierda

26. Masculino de 2 años llevado al servicio de consulta externa por presentar otorrea
bilateral mucopurulenta de una semana de evolución.Antecedente: ha presentado
cuadros repetitivos de infecciones de vías respiratorias altas, en todos ellos ha recibido
diferentes tratamientos. Exploración física: ambas membranas timpánicas opacas e
íntegras, a la rinoscopia presenta mucosa pálida y violácea con moco hialino en forma
abundante, orofaringe sin alteraciones. Cuello sin adenomegalias.
26a. La patología responsable de la sintomatología de este paciente es:
a} Otitis media bilateral

26b. El estudio de gabinete indicado para corroborar el diagnóstico en este paciente es:
a) Rx. lateral de cuello

27. Paciente femenino de seis años, con esquema de vacunación incompleto, acude por
fiebre leve, dolor faríngeo, tos y masas en cuello. En el examen se encuentra temperatura
de 38.5 ºC, hiperemia de mucosa conjuntiva!, exantema maculopapular coalescente en
cara, tronco y extremidades. Se palpan linfadenopatías posoccipitales y retroauriculares
bilaterales.
Respecto a la enfermedad de la paciente, todos los enunciados siguientes son verdaderos,
EXCEPTO:
e} Se debe iniciar tratamiento antibiótico

28. Un paciente de cinco años previamente con desarrollo normal acude a consulta
pediátrica debido a que ha presentado náusea,vértigo y alteraciones en la marcha. A la
exploración física el paciente muestra datos de hipertensión endocraneana por lo que se
obtiene una tomografía axial computarizada la cual demuestra una lesión tumoral en el
vermis del cerebelo. ¿Cuál es el tipo de tumor más probable en este paciente?
a} Meduloblastoma

29. Se presenta a consulta paciente femenino de 1O años de edad con antecedentes


patológicos personales de rinitis alérgica y antecedentes familiares de atopia (tíos
paternos con asma bronquial). Tiene historia de infecciones respiratorias altas de
repetición (aprox. 6 al año) desde el año de edad, las cuales son de evolución prolongada.
Se le solicitaron exámenes complementarios en donde se reporta lgG elevada, lgM
normal, lgA mínima y complemento normal.
29a. Con base en las características dela paciente, ¿cuál es el diagnóstico más probable?
c} Deficiencia selectiva de lgA

29b. ¿Qué hallazgos confirman su sospecha en los estudios complementarios?


a) Ausencia de lgA

29c. En relacióncon su sospecha clínica,¿cuál es la alteración genética asociada?


b) No se haidentificado

29d. Con base en su sospecha clínica, ¿cuál es el tratamiento que estaría indicado en esta
paciente?
a) Tratamiento deinfecciones, NOinmunoglobulina intravenosa"
29e. De acuerdo con su sospecha clínica, ¿cuál es la complicación que se podría presentar
en la paciente?
c) Reacción anafiláctica tras transfusión

30. ¿En qué paciente se debe de sospechar un angiofibroma nasofaríngeo juveni l?


e) Paciente masculino de 18 años con epistaxis frecuentes y masa lobulada en
nasofaringe

32. Paciente masculino de 11 años de edad acude a consulta por dolor intenso en la rodilla
izquierda posterior a una lesión en un partido de fútbol hace ocho semanas. Refiere
edema a lo largo de la porción posterior de la rodilla afectada y en toda la pierna del lado
izquierdo. El dolor se incrementa cuando se involucra en actividades que incluyen
levantamiento de peso. Al interrogatorio niega fiebre, sudación nocturna o pérdida de
peso reciente. Al revisar el expediente, se encuentra que en el pasado padeció un
retinoblastoma, el cual fue tratado con defecto visual residual mínimo. Se solicita un
escaneo óseo con tecnecio, el cual muestra una lesión localizada en la epífisis distal del
fémur izquierdo. Después se solicita una resonancia magnética que demuestra que las
estructuras neurovasculares no están involucradas. ¿Cuál de los siguientes diagnósticos es
el más probable en este paciente?
a) Osteosarcoma

33. ¿Cuál de los siguientes bloqueadores beta adrenérgicos oftálmicos es el más adecuado
para un paciente con antecedente de asma de leve intensidad?
b) Betaxolol

34. Un recién nacido aparentemente sano es referido a su consulta por un sangrado


excesivo que requirió transfusión tras una circuncisión de rutina. Los padres traen consigo
estudios de laboratorio, los cuales reportan: tiempo de protrombina normal,tiempo de
tromboplastina prolongado y actividad del factor VIII disminuida.
34a. Con base en las características del paciente, ¿cuál es el diagnóstico más probable?
b) Hemofilia A

34b. ¿Qué hallazgos en los exámenes de laboratorio apoyan su sospecha diagnóstica?


d) PT normal,PTT prolongado, actividad de vWF normal,actividad de factor VIIIdisminuida

34c. En relación con su sospecha clínica,¿cuál es la complicación que se puede presentar a


largo plazo?
a) Destrucción articular

34d. De acuerdo con su sospecha clínica, ¿cuál es el tratamiento indicado?


e) Factor VIII
34e. ¿Qué aspecto del cuadro clínico del paciente es relevante mencionar a los padres?
b) lnhibidores de factor

35. Durante una intervención quirúrgica en un paciente inestable, el uretero izquierdo es


lacerado en 50% de su circunferencia. Debido a que la condición de base del paciente es
demasiado seria y no permite la reparación definitiva de este defecto, ¿cuál de las
siguientes opciones representa la alternativa terapéutica más efectiva?
a) Ligadura del uretero dañado y nefrostomía ipsolateral

36. Paciente masculino de 10 meses de edad es llevado al departamento de Urgencias por


su madre de 14 años de edad debido a que está muy irritable. En el interrogatorio la
madre refiere que hace 48 hse encontraba perfectamente bien y que ha sido alimentado
con leche evaporada desde el nacimiento porque su familia no tiene dinero para comprar
fórmula. Todavía no inicia con alimentación complementaria. En la exploración física se
observa al paciente muy irritable, con algunas petequias en el cuerpo; además, toma una
posición con las caderas y rodillas flexionadas y llora cuando es movido. El resto de la
exploración es normal.
36a. Con base en las características del paciente, ¿cuál es el diagnóstico más probable?
d) Deficiencia de vitamina C

36b. En relación con su sospecha clínica,¿cuál de las siguientes opciones representa una
característica particular del padecimiento?
d) Apariencia de vidrio deslustrado en la radiografía

36c. Respecto a su sospecha clínica, ¿qué función tiene en el cuerpo la vitamina que se
encuentra con deficiencia en el paciente?
a) Mantenimiento del material intracelular

36d. Con base en su sospecha diagnóstica, ¿qué tratamiento le indicaría al paciente?


b) Consumo de jugo de naranja

36e. De acuerdo con su sospecha clínica, ¿cuál es el mecanismo de prevención que se


debió realizar para evitar esta patología?
c) Jugos de cítricos

"37. Paciente masculino de 53 años de edad, con antecedentes de hipertensión arterial


sistémica en tratamiento con IECA, con buen apego a tratamiento, y etilismo a base de
fermentados (vino tinto), acude por presentar dolor articular de repetición a nivel de la
articulación metatarsofalángica del primer dedo del pie derecho, la sintomatología se
manifiesta con predominio nocturno con aumento de volumen y dolor punzante 10/1O,
que no remite al reposo, que en otras tres ocasiones ha remitido de 3 a 1O días de forma
espontánea; a la exploración física se encuentra un nódulo de Bouchard en la articulación
referida, caliente, eritematosa y dolorosa a la palpación, que limita la movilidad; no se
encuentran datos en otras articulaciones ni datos patológicos agregados. FC 95 lpm, FR 18
rpm, temp. 36.5ºC, PA 130/70 mm Hg, peso 95 kg, talla 1.70 kg.
37a. Con los datos del interrogatorio y la exploración física usted diagnostica:
c) Artritis inducida por cristales

37b. Al realizar una punción articular,usted solicita una cristalografía esperando


encontrar:
b} Urato monosódico (MSU}

37c. Para el abordaje diagnóstico adicional, el estudio que permite iniciar el tratamiento
adecuado es:"
d} Depuración de ácido úrico en orina de 24 horas

37d. Ya que el paciente se encuentra en una crisis gotosa, usted decide iniciar tratamiento
con:
d} Cualsea delos anteriores

37e. Adicional al manejo farmacológico, usted sugiere cambios en la dieta y estilo de vida,
¿cuáles son los alimentos que usted recomienda evitar?
a} Vísceras (hígado, riñón,estómago}, refresco y alcohol

38. Paciente femenino de 78 años de edad que acude a consulta por disuria. Refiere que
inició hace dos días. En sus antecedentes se refiere hipertensión. Sus medicamentos
incluyen hidroclorotiazida y aspirina. En el IPAS menciona también tener que ir más
frecuente al baño, a veces debiendo apresurarse por el peligro de no llegar. Niega dolor
abdominal o hematuria y no ser alérgica a algún medicamento. A la exploración física sus
signos vitales incluyen FC 85/min, FR 15/min, T 37.3 C y PA 134/82 mm Hg. Refiere que es
la primera vez que tiene estos síntomas. Se realiza una tirilla y análisis al microscopio de
una muestra de orina. El espécimen se manda para cultivo. A continuación se muestran
los resultados:

¿Cuál sería el siguiente paso en el manejo de esta paciente?


c} Iniciar tratamiento con trimetoprim-sulfametoxazol
39. ¿Cuál de las siguientes aseveraciones es más correcta respecto del ejercicio
anaeróbico en pacientes sanos de la tercera edad?
d} La disminución enla respuesta ala estimulación beta adrenérgica contribuye a la caída
enla frecuencia cardiaca máxima en individuos ancianos

40. Paciente femenino de 36 años emigrada del sureste asiático, comenzó a presentar tres
semanas posteriores a su llegada, astenia,adinamia, malestar general,episodios febriles,
pérdida de 4 kg de peso en dos semanas, anorexia; poco después refiere aumento de
volumen de la región lateral izquierda del cuello, ligeramente dolorosa al tacto por lo que
acude a la consulta. Niega presencia de tos, expectoración o falta de aire. La exploración
física reveló: Adenomegalia única, localizada en la cadena lateral izquierda del cuello,
ligeramente dolorosa al tacto, 3 a 5 cm de diámetro, de consistencia elástica, no adherida
a planos profundos. La radiografía de tórax no muestra alteraciones. Prueba de Mantoux:
29 mm, el resultado de patología de la biopsia del ganglio linfático: cultivo compatible con
Mycobacteríum tuberculosis. NO es una indicación de profilaxis para la tuberculosis
utilizando isoniazida:
c} Contactos de edad avanzada Mantoux negativos

41. Paciente masculino de 50 años, inmigrante mexicano en Estados Unidos, se presenta


con tos productiva con esputo sanguinolento acompañada de sudoración nocturna,
pérdida de peso y fatiga de tres meses de duración. Signos vitales: temperatura: 38 ºC,
presión arterial: 130/85 mm Hg, frecuencia cardiaca: 90 latidos por minuto, frecuencia
respiratoria: 22 respiraciones por minuto. A la exploración física se observa pálido, hay
disminución de ruidos respiratorios en los ápices de ambos pulmones.
La duración del tratamiento de tuberculosis en la población general:
e} Tiene duración fija

42. Paciente masculino de 58 años con antecedente de carcinoma epidermoide de pulmón


es traído a urgencias por su esposa tras notar que desde hace una semana presenta
poliuria, vómito y confusión progresiva hasta aparecer letárgico esta mañana. Su
electrocardiograma muestra ritmo sinusal con FC de 11O latidos por minuto e intervalo QT
de 0.20 segundos y tiene un calcio sérico de 13 mg/dL."
¿Cuál es el tratamiento inicial en el manejo de este paciente?
a} Infusión intravenosa de solución salina a 0.9%

43. En el siguiente ECG se observa prolongación del intervalo QT (424 ms) y QT corregido
(513 ms). ¿Cuál es la alteración responsable de estos hallazgos?
d) Hipocalcemia
44. Paciente masculino de 67 años de edad, cursa con antecedentes de etilismo crónico
intenso desde la juventud, hace 5 días inició con fiebre >37.8ºC, disnea, tos, expectoración
verdosa y palpitaciones; a la exploración física se encuentran estertores ydisminución del
murmullo vesicular en la región apical derecha, sin sibilancias, se integra un síndrome
pleuropulmonar de consolidación apical derecha, la saturación por pulsooximetría es de
84% respirando al aire ambiente. Vive en su domicilio, no ha estado hospitalizado ni
ingerido antibióticos recientemente, niega otros antecedentes relevantes. FC 105 lpm, FR
24 rpm,temp.38ºC, PA 110/70 mm Hg, peso 84 kg, talla 1.68.

44a. Con los datos obtenidos del interrogatorio y la exploración física usted diagnostica:
b} Probable neumonía adquirida en la comunidad

44b. Usted realiza una radiografía de tórax para confirmar el diagnóstico, la cual evidencía
una radioopacidad en el lóbulo superior del pulmón derecho, con los antecedentes y éstos
infiltrados apicales; ¿qué agente etiológico es el más probable?"
e} Klebsiella pneumoniae

44c. Según la clasificación CRB-65, usted determina que el manejo debe ser:
a} Ambulatorio

44d. El tratamiento de primera línea, según las guías de la IDSA, es:


b} Betalactámico + macrólido

44e. Usted recomienda al paciente que se aplique un esquema de vacunación bacteriano,


¿cuál es el de elección?
d} Vacuna contra neumococo

45. Masculino de 75 años con diagnóstico de ERGE desde hace 15 años, en tratamiento
con IBP,acude a consulta por tener dificultad para ingerir alimentos. Al interrogarlo
comenta que desde hace un par de años ya notaba dolor altragar alimentos sólidos, por lo
cualfue reduciendo el tamaño de los bocados, pero esto ha empeorado en los últimos días
y ha llegado a ser muy difícil para él tragar incluso líquidos. Usted nota que el paciente
carraspea y tose mucho durante el interrogatorio. EF: 1.68m, 1.65cm, TA: 140/80, Temp:
36.5ºC. A la exploración se encuentra pérdida de piezas dentales, halitosis importante,
amígdalas de tamaño normal, con ligera hiperemia de mucosa faríngea.
45a. ¿Cuál de los siguientes estudios sería de mayor utilidad diagnóstica?
d} Endoscopia + toma de biopsia

45b. ¿Qué estudio se requeriría para realizar la estadificación de la enfermedad?


b} TC

45c. Se considera como el factor de riesgo más importante para el Ca de esófago.


c} Metaplasia intestinal
45d. ¿Cuál es el tipo de cáncer de esófago más frecuente?
b} Adenocarcinoma

45e. Para los tumores limitados (T1-2, N0-1, MO), el tratamiento de elección es:"
e} Quirúrgico

46. Paciente masculino de 40 años de edad, fumador, acude a Urgencias por dolor
precordial súbito e intenso irradiado a hombro, el cual se intensifica a la inspiración. TA:
130/80, FC: 75 lpm. Ruidos cardiacos ligeramente disminuidos, sin soplos. Frote
pericárdico. Electrocardiograma: elevación cóncava superior del ST en Dll, Dlll,aVF,V2 a V6.
Se solicitaron troponinas cuyos resultados aún se desconocen."

46a. ¿Cuál es el diagnóstico más probable en este paciente?


c} Pericarditis aguda

46b. ¿Cuál es la causa más frecuente de pericarditis?


a} ldiopática

46c. ¿En qué fase electrocardiográfica de la pericarditis aguda se encuentra el paciente?


a} Fase 1

46d. Menciona el estudio de gabinete indicado para continuar con el estudio del paciente.
b} Ecocardiografía

46e. Tratamiento indicado en este paciente:


c} lbuprofeno + colchicina

47. Paciente femenino de 67 años que acude a consulta por sangrado transvaginal. Refiere
que éste inició hace tres semanas, aunque anteriormente sí había notado que manchaba
ligeramente su ropa interior. Su última menstruación fue a los 48 años, y en la actualidad
es sexualmente activa con su esposo con el que lleva casada 17 años (sin asociar el
sangrado con la relación sexuaQ. Tiene dos hijos, de su esposo anterior. Refiere además
tabaquismo de 32 paquetes/año. En sus antecedentes patológicos habla de hipertensión,
diabetes mellitus tipo 2 e hiperlipidemia. Sus medicamentos incluyen hidroclorotiazida,
metformina, atorvastatina y tamoxifeno (para prevención de cáncer de mama). A la
exploración física se observa una paciente obesa. Sus signos vitales incluyen FC 89/min, FR
16/min,T 36.6 C y PA 129/82 mm Hg. Al examen pélvico se observa una mucosa atrófica,
sin evidencia de sangrado. No hay dolor a la palpación bimanual y no hay masas.
¿Cuál de los siguientes es un factor de riesgo de cáncer endometrial?"
e} Uso de tamoxifeno

48. Paciente masculino de 73 años que acude a consulta por disnea y tos de varios meses
de evolución. Refiere que ha sentido que no puede realizar la misma actividad física que
antes, sintiendo dificultad para respirar después de caminar dos cuadras. En sus
antecedentes se reporta hipertensión e hiperlipidemias. Menciona tabaquismo de 40
paquetes/año, y toma 3 a 4 cervezas por semana. Al IPAS refiere expectoración de una
flema amarillenta, negando hemoptisis. Niega pérdida de peso, dolor torácico, disnea
paroxística nocturna, ortopnea y edema periférico. A la exploración física se observa un
hombre delgado, sus signos vitales incluyen FC 79/min, FR 19/min, T 37.0 C y PA 134/78
mm Hg. A la exploración torácica se observa un diámetro anteroposterior aumentado. Se
ausculta S1 y S2 sin agregados. A la auscultación pulmonar hay roncus leves. No hay
organomegal ias a la exploración abdominal ni distensión venosa yugular. Sin cianosis
periférica ni edema. Se toma una radiografía de tórax, observando pulmones hiperinflados
y diafragmas aplanados, con atenuación en las marcas vasculares y un mediastino
estrecho. ¿Cuál de los siguientes patrones sería el más probable de encontrar en las
pruebas pulmonares?"
Letra VEF1 CVF VEF1/FVC "Difusión
co" Respuesta a oroncodilatadores
A 58% 91% 63% 62% 5%
B 60% 66% 90% 59% 1%
e 61% 90% 66% 80% 15%
D 82% 80% 100% 83% 1%
E 89% 67% 100% 84% 2%

a} Renglón A

49. ¿Cuál de las siguientes relaciones entre fármacos antituberculosos y sus efectos
adversos es incorrecta?
d} Etambutol:hepatotoxicidad

50. ¿Cuál es el vaso más afectado en la arteritis de Takayasu y que ocasiona claudicación
de los brazos y fenómeno de Raynaud?
e} Arteria subclavia

51. Masculino de 24 años de edad, sin antecedentes patologicos y multiples parejas


sexuales; ultima relacion no protegida hace 10 dias. Consulta por dolor al orinar, que
empezo hace 2 dias y lo describe como si fueran navajas. Asimismo, refiere que siente
comezon en el pene y aumento en la frecuencia urinaria. A la exploracion fisica se observa
una secrecion verde-amarillenta que sale de la uretra.
51a. ¿Cuál es su sospecha diagnóstica?
a} Uretritis

51b. ¿Cuáles son los agentes causales más comunes?


a} Chlamydia trachomatis, Ureaplasma urealyticum, Neisseria gonorrhoeae"
51c. Estudio de elección para el diagnóstico y clasificación
e} Tinción de Gram

51d. ¿Cuál es el tratamiento indicado para este paciente?


a} Ceftriaxona

52. Paciente masculino de 28 añosacude aUrgencias por presentar dolor intenso en el


abdomen, además de náusea y vómito. Refiere que el dolor comenzó hace 8 h y que
aunque en un inicio fue difuso, al paso del tiempo se fue localizando hacia el lado inferior
derecho del abdomen; asimismo, comenta que desde el inicio fue tipo cólico. A la
exploración física, usted observa en este paciente facies de dolor en posición antiálgica,
hiperalgesia e hiperbaralgesia, y ruidos intestinales disminuidos de intensidad; en los
primeros laboratorios llaman la atención los leucocitos (12 000) y los neutrófilos de
80%.EF: 1.80m; 85kg; TA: 135/85; Temp: 38.5ºC."

52a. ¿Cuál es la principal sospecha diagnóstica?


c} Apendicitis aguda

52b. ¿Cuál es la causa más común de abdomen agudo quirúrgico?


b} Apendicitis

52c. Son signos negativos al realizar una TC para apendicitis aguda:


a} Aire intraluminal o contraste oral

52d. El tratamiento de elección es:


b} Quirúrgico

53. Lo siguiente es cierto acerca del absceso hepático amibiano.

e} Las complicaciones pleuropulmonares sonlas más frecuentes

54. Paciente masculino de 48 años de edad, con obesidad grado 1, hipertensión y diabetes
mellitus 11, inicia padecimiento con gonartralgia intensa súbita derecha y aumento del
volumen, con disminución paulatina de dolor hasta la remisión a los 5 días. Al mes
presenta por la noche, tras comida copiosa, dolor e inflamación de la primera articulación
metatarsofalángica derecha, por lo que acude a urgencias. TA: 140/90 mm Hg, Temp: 38
ºC, articulación con edema,eritema, calor y dolor. Laboratorios: glucosa: 130
mg/dl,triglicéridos: 200 mg/dl,ácido úrico: 7 mg/dl, Hb: 17 mg/dl, leucocitos: 14000,
neutrófilos: 80%.
54a. Señale el diagnóstico compatible con el cuadro clínico y los resultados de laboratorio
del paciente:
d} Gota
54b. Estudio indicado para confirmar el diagnóstico:
a} Microscopia delíquido sinovial

54c. ¿Cómo se observan los cristales de urato monosódico en la microscopia?


c} Birrefringencia fuertemente negativa en forma de aguja

54d. ¿Cuál es la base fisiopatológica más frecuente de la hiperuricemia?


b} Excreción disminuida de ácido úrico

54e. Tratamiento indicado en este paciente:


a} AINE

55. Paciente femenino de 41 años de edad que acude al servicio de urgencias por dolor
torácico. Menciona que siempre ha tenido buen estado de salud; sin embargo, desde hace
una semana presenta secreción nasal blanquecina, ardor faríngeo, tos y malestar general.
Describe que el dolor inició hace 5 horas y es de tipo ""desgarrador"", empeora durante la
inspiración profunda, y mejora cuando se inclina hacia delante. A la exploración física
presenta una presión arterial de 110/60 mm Hg y una frecuencia cardiaca de 90
latidos/minuto. En la exploración física se puede encontrar un frote trifásico. La
radiografía de tórax no muestra ninguna alteración. El electrocardiograma muestra
elevación del segmento ST en la mayoría de las derivaciones.
¿Cuál es el tratamiento más adecuado para esta paciente?"

d} Reposo y aspirina a altas dosis

56. Paciente femenino de 50 años de edad acude a consulta debido a que desde hace
algunas semanas presenta torpeza al manejar objetos con las manos y dolor en ambas
muñecas con intensidad 5 de 1O, además de adormecimiento y a veces sensación de
ardor en los dedos 2° y 3°. La paciente se dedica al hogar y refiere que desde pequeña le
gusta lavar la ropa a mano. Tiene antecedentes de artritis reumatoide en la familia y hace
3 años se le diagnosticóDM2, para la cual no recibe tratamiento alguno por el momento. A
la exploración física no se observan cambios de tamaño ni de temperatura en las
articulaciones de la muñeca y falanges; sólo se aprecia mala calidad"
muscular en la zona tenar.
56a. ¿Cuál es su sospecha diagnóstica?
d} Neuropatía por atrapamiento

56b. Es la prueba clínica másespecífica para este diagnóstico.


a} Tinel
56c. ¿Cuál es la estructura comprometida en este síndrome?
b} N. mediano

56d. El tratamiento de primera elección es:


d} Conservador

56e. En la cara dorsal, ¿qué sensibilidad cutánea es transmitida por el nervio mediano?"
e} Índice y medio

57. Ante un recién nacido de 38 semanas de gestación, obtenido por cesárea, que inicia de
forma precoz con taquipnea, dificultad respiratoria, Prueba de Silverman de 4, exploración
pulmonar normaly mejoría con administración de oxígeno; ¿qué hallazgos radiológicos
serían consistentes con el cuadro clínico que presenta?
d} Líquido presente enlas cisuras y prominencia vascular pulmonar

58. Ante un recién nacido que presenta salida de material líquido amarillo transparente
por el sitio del cordón umbilical sin otro dato en la exploración física, ¿qué estudio le sería
de mayor utilidad para diferenciar entre los probables diagnósticos?
a} Determinación del pH del líquido

59. ¿Cuál es la célula que sigue en diferenciación del metamielocito?


b} Banda

60. Recibe usted los resultados histopatológicos de una paciente de 24 años de edad que
acudió a revisión rutinaria, los resultados reportan imagen histológica de coilocitos lo cual
sugiere infección por:

Lesión intraepitelial de bajo grado. Alteraciones celulares compatibles con infección


(coilocitos).
e} Virus del papiloma humano

61. Paciente femenino de 59 años de edad acude a consulta para evaluación de rutina
anual. Durante la exploración física se encuentran linfadenopatías cervicales y axilares sin
hepatomegal ia o esplenomegal ia. Los estudios de laboratorio de rutina revelan
hemoglobina de 12.5 g/dl, hematocrito de 37%, conteo leucocitario de 12 000
células/mm3, y conteo plaquetario de 300 000 plaquetas/mm3. En estudios adicionales se
reporta positividad para el receptor CDS de células B que producen expresión clonal de
cadenas ligeras lambda. ¿Cuál es el tratamiento más adecuado para esta paciente?
d} Observación

62. Neonato pretérmino de 5 horas de vida,en el examen fisico se detectó soplo continuo
en maquinaria, más intenso en focos aórtico y pulmonar,con desplazamiento del impulso
apical hacia la línea anterior axilar izquierda y pulsos periféricos saltones y amplios. Ha
desarrollado un síndrome de distrés respiratorio.
62a. ¿Qué diagnóstico sospecha?
c} Persistencia delconducto arterioso

62b. ¿Qué tratamiento está indicado en este momento?


a}lndometacinaintravenosa

63. Masculino de 68 años, acude a consulta externa para control metabólico de diabetes
mellitus,a la exploración de fondo de ojo usted observa lo siguiente:
La complicación temprana en este paciente es:

a} Microaneurismas

64. Paciente de cinco años de edad con antecedente de reflujo gastroesofágico y


enfermedad acidopéptica que se somete a una esofagogastroduodenoscopia debido a
enfermedad refractaria. La biopsia de estómago revela la presencia de gastritis con
linfoma asociado a tejido linfoide de mucosas (MALToma). ¿Cuál de las siguientes
opciones es el tratamiento más adecuado para este paciente?
b} Tratamiento de erradicación para Helicobacter pylori

65. Un paciente de 68 años de edad es llevado a consulta psiquiátrica por su hijo debido a
la presencia de sintomatología de demencia. En lo que se refiere a los antecedentes de
importancia, el paciente no había presentado ningún comportamiento extraño, sino hasta
que fue forzado a retirarse de su trabajo hace algunos meses debido a que su movilidad se
vio afectada por su osteoartritis. El paciente enviudó hace un año y en este momento se
encuentra viviendo en la casa de su hijo. ¿Cuál de las siguientes opciones puede ser con
mayor probabilidad la causa de la sintomatología de este paciente?
c} Depresión

66. Masculino de 3 años de edad es traído a consulta por sus padres debido a que en los
últimos días han notado una constante desviación del ojo derecho. Hasta el momento el
desarrollo del niño ha sido normal.

66a. El cuadro más probable de este paciente es:


c} Estrabismo

66b. ¿Cuál es la causamás prevalente de este padecimiento?


b} Alteración neuromuscular idiopática

66d. La prioridad en el tratamiento de este paciente es:


a} Prevenir o evitar la ambliopía

66e. ¿Cuál de los siguientes NO inerva el III par craneal?


a} Oblicuo superior

67. ¿Cuál de las siguientes neoplasias del sistema nervioso central en adultos se presenta
principalmente en pacientes con neurofibromatosis?
c} Schwannoma

68. Masculino de 72 años, se le diagnostica enfermedad de Parkinson idiopática.


El dato clínico que corrobora el diagnóstico de este paciente es:
d} Pérdida de movimientos asociados en la marcha

69. ¿Cuál de los siguientes hallazgos se asocia con el adenocarcinoma de estómago?


d} Signo de Leser-Trélat

70. ¿En cuál de las siguientes enfermedades la tirosina se convierte en un aminoácido


esencial?
a} Fenilcetonuria

71. ¿Cuál de las siguientes neoplasias malignas se asocia con síndrome de Cushing como
síndrome paraneoplásic
a} Carcinoma celular de células pequeñas

72. Se trata de paciente masculino de 5 años de edad que cursa con diagnóstico de
varicela. ¿Cuál de los siguientes medicamentos aumenta el riesgo de producir síndrome de
Reye?
c} Ácido acetilsalicílico

73. ¿Cuál de los siguientes efectos adversos se relaciona con la administración de


prazosina?
d} Aumento en el riesgo de padecer eventos de síncope

74. ¿Cuál de las siguientes aseveraciones es la más correcta respecto del bloqueo de rama
izquierda en el sistema de conducción del corazón?
c} La presencia de bloqueo de rama izquierda se puede asociar con disminución
significativa dela supervivencia alargo plazo

75. Paciente femenino de 49 años de edad que acude a la consulta externa de medicina
interna. El motivo principal de la consulta es porque la paciente manifiesta fatiga, disnea y
palpitaciones, y recientemente se agregó a su sintomatología alteraciones del balance. El
esposo de la paciente la acompaña y refiere que el estado de ánimo de ella ha cambiado,
ya que ahora es más irritable y olvidadiza. A la exploración física la paciente se presenta
pálida con escleras ictéricas y taquipnea; la auscultación cardiaca revela la presencia de un
soplo sistólico; la exploración neurológica es relevante por alteraciones en el balance y la
propiocepción de las extremidades inferiores. Se solicitan estudios de laboratorio los
cuales muestran un conteo eritrocitario de 3 300 células/mm3, leucocitos de 4 000
células/mm3, volumen corpuscular medio de 11O fl y concentración de hemoglobina
corpuscular de 25 pg/célula.

¿Cuál de las siguientes reacciones bioquímicas se vería afectada en esta paciente y es


responsable de las alteraciones neurológicas?
e} Metilmalonil coenzima A ---+ succinil coenzima

76. ¿Cuál de los siguientes componentes del virus de la hepatitis B se correlaciona con la
infectividad del paciente?
c} HBeAg

77. Masculino de 34 años es llevado al servicio de urgencias por referir visión de cuerpos
volantes (miodesopsias), destellos luminosos (fotopsias) y disminución de la visión
periférica en alguna zona del campo visual. Antecedentes: recibir golpe en ojo cuando
jugaba frontón de mano.
Los datos clínicos anteriores confirman el diagnóstico de:
d} Desprendimiento de retina

78. ¿Cuál de las siguientes alteraciones electrolíticas se asocia con aleteo (flutter)
ventricular?
d} Hiperpotasemia

79. ¿Cuál es el mecanismo de acción de las tetraciclinas?


a}Inhibición dela subunidad ribosomal 308

80. Paciente masculino de 65 años que acude a la consulta por presentar dolor dorsal y
dificultad para iniciar la micción. Dentro de los antecedentes heredofamiliares comenta
que de sus cuatro hermanos, dos han sido diagnosticados con hiperplasia prostática
benigna por lo que fueron sometidos a resección transuretral prostática. Es por ello que
anualmente acude a su consulta periódica para realización de tacto rectal y antígeno
prostático específico. Pero debido a problemas personales no había venido a consulta
desde hace cuatro años. En esta ocasión al examen rectal se encuentra próstata dura e
irregularmente crecida. Se reporta antígeno prostático específico de 15 ng/ml. Ante esta
situación se decide realizar ecografía transrectal con toma de biopsia por sextantes. El
reporte histopatológico es adenocarcinoma bien diferenciado. Dentro de los estudios
complementarios se tomaron radiografías de columna donde se observaron lesiones
osteoblásticas en columna vertebral y en los huesos de la pelvis.
¿Cuál es el tratamiento de elección?
d} Tratamiento hormonal

81. ¿Cuál de los siguientes estudios de laboratorio son esperados en un paciente con
insuficiencia renal crónica?
d} Calcio disminuido, fosfato incrementado, fosfatasa alcalina normal"

82. ¿Cuál es el cáncer más frecuente asociado a artritis reumatoide?


c} Linfomas Hodgkiny no Hodgkiny leucemia linfocítica aguda

83. Paciente femenino de 87 años de edad con antecedente de hipertensión arterial,


hiperuricemia, dislipidemia, cardiopatía isquémica con infarto inferior antiguo, anemia de
enfermedad crónica, isquemia crónica grado IV en miembros inferiores, prótesis aorto-
aórtica por aneurisma de aorta abdominal intervenido hace ocho años. Ingresa al servicio
de urgencias por dolor torácico retroesternal opresivo, irradiado a región de la espalda. A
la exploración física se encuentra una paciente inquieta por el dolor, presión arterial de
110/80 mm Hg, frecuencia cardiaca 11O latidos/min;soplo sistólico IVNI en el foco aórtico.
En la radiografía de tórax se observa una imagen de aumento de densidad de borde bien
definida, periférica al borde cardiaco izquierda, sin borrar la línea media ventricular (por lo
tanto, extracardiaco), sugestivo de aorta descendente.
¿Cuál de los siguientes síntomases menos común en los pacientes con aneurisma de aorta
torácica en expansión?
e} Disnea

84. ¿Cuál de los siguientes hallazgos se puede observar en un paciente con lesión del
nervio peroneo común?
a} Alteración dela sensibilidad en el dorso del pie

85. El primer fármaco que demostró ser efectivo en el tratamiento de la infección por el
virus de la inmunodeficiencia humana fue la zidovudina. ¿Cuál es el mecanismo de acción
de este fármaco?
a}Inhibición dela transcriptasa reversa
86. Entre los factores protectores para presentar leucemia mieloide aguda se encuentra
uno de los siguientes:
e}Infecciones virales

87. ¿Cuál de los siguientes nervios se encarga de la sensibilidad de la piel del escroto y la
porción medial del muslo?
a} Nervio inguinal

88. Un paciente de 15 años acude a la consulta de urgencias con vómito y dolor abdominal
difuso. Los estudios de laboratorio muestran los siguientes hallazgos: sodio de 130 mEq/L,
potasio 5.3 mEq/L, cloro 95 mEq/L,dióxido de carbono 9 mEq/L, nitrógeno ureico 35
mg/dL,creatinina 1.2 mg/dL, glucosa 389 mg/dL, pH arterial 7.2. ¿Cuál de las siguientes
sustancias deberá encontrarse más probablemente elevada en este paciente?
d} Cetonas

89. Mujer de 70 años de edad que acude a consulta por un episodio de amaurosis fugaz en
ojo derecho hace cinco días. Hace tres días notó una disminución intensa de la agudeza
visual en ese mismo ojo. Al interrogatorio dirigido refiere claudicación mandibular y
cefalea ocasional,así como hipersensibilidad de la piel cabelluda desde hace dos meses. A
la exploración física muestra induración de la región temporal derecha. La agudeza visual
de ojo derecho es de 5/200 y ojo izquierdo 20/30. Presión intraocular de 13 mm Hg en ojo
derecho y 15 mm Hg en ojo izquierdo. A la oftalmoscopia de ojo derecho se encuentra
papila pálida++, edematosa, sobreelevada, con bordes borrados, arterias"
"retinianas discretamente atenuadas, mácula sin alteraciones. El fondo de ojo del ojo
izquierdo es normal."
89a. ¿Cuál es el diagnóstico más probable?
b} Neuropatía óptica isquémica anterior arterítica

89b. ¿Cuáles son los estudios que se debe indicar?


b} Velocidad de sedimentación globular,proteína C reactiva y biopsia de la arteria
temporal

89c. ¿Cuál es el tratamiento más adecuado?


b} Administrar corticoesteroides sistémicos

90. Se trata de paciente femenino de 29 años de edad con diagnóstico de embarazo de 32


semanas de gestación. En su familia existe antecedente de gemelos monocigotos. Refiere
que durante el primer trimestre presentó hiperemesis y después tuvo diagnóstico de
preeclampsia. Se encuentra bajo tratamiento con alfa metildopa. A la exploración física
usted percibe un fondo uterino de mayor tamaño para la edad gestacional y al realizar la
segunda maniobra de Leopold siente dos dorsos, uno a la derecha y otro a la izquierda; a
la auscultación se perciben dos latidos cardiacos con frecuencias distintas entre sí,en un
embarazo gemelar monocigoto, ¿cómo se encontrará la placenta y el amnioscon mayor
frecuencia? b} Monocorial-biamniótica
91.Ante un recién nacido, hijo de madre que presentó infección por citomegalovirus
durante el primer trimestre de gestación, usted esperaría todas las siguientes
manifestaciones clínicas EXCEPTO:
b} Hipoacusia neurosensorial bilateral

92. Paciente masculino de 65 años, hipertenso, acudió a consulta hace unos meses por
dolor abdominal vago y diarrea de varios meses de evolución. A la exploración física sus
signos vitales eran normales y no se encontró nada que llamara la atención. En los
resultados de la biometría hemática se encontró una anemia probablemente ferropénica
(microcítica hipocrómica) y un guayaco positivo. Se decide realizar una colonoscopia, por
medio de la cual se encuentra una masa en el colon,que lo más probable es que se trate
de un cáncer colorrectal.

92a. ¿En qué parte del colon es lo más probable que se encuentre el cáncer del paciente?
a} Colon proximal

92b. ¿La mutación de la cuál de los siguientes genes se asocia con mayor frecuencia a
cáncer de colon?
a} APC

92c. ¿Cuál de los siguientes métodos de escrutinio para cáncer de colon se debe de utilizar
y con qué frecuencia?
d} A partir de los 50 años, colonoscopia cada 1O años

92d. El cáncer de colon se asocia a:


b} Bacteriemia por Streptococcus gallolyticus

92e. ¿Cómo se realiza el seguimiento de los pacientes con cáncer colorrectal?


b} Colonoscopia anual y antígeno carcionoembrionario (ACE} periódicamente

93. ¿Cuál es el germen más común que causa faringoamigdalitis bacteriana?


e} Estreptococo beta hemolítico del grupo A

94. Prevalencia de diabetes mellitus tipo 2 en México de acuerdo con ENSANUT 2006:
a} 7.5%

95. Niño de ocho años de edad que presenta una dermatosis diseminada, bilateral y
simétrica que afecta ambas palmas, plantas y cavidad oral constituida por máculas en
forma de balón de fútbol,pápulas eritematosas, vesículas y erosiones. El diagnóstico es
enfermedad mano-pie-boca. ¿Cuál es el tratamiento y el pronóstico?
d} La enfermedad no es grave y requiere tratamiento sintomático

96. Masculino de 5 años, llevado a servicio de urgencias por la madre quien refiere que el
día de ayer el menor presentó malestar general, no quiso comer y refirió dolor de cabeza
con temperatura de 37.6 ºC. El día de hoy se le encontraron en la cara anterior del tórax
cinco vesículas aperladas, redondeadas por un halo eritematoso, con líquido transparente
en su interior. La familia refiere preocupación porque hay perro y gato en la casa y temen
que sea un contagio la causa del padecimiento. Exploración física:se le encuentran aún
más vesículas umbilicadas en el tronco y algunas vesículas en el paladar, se aprecia que el
menor presenta escozor,T 37.7 ºC.
96a. El cuadro clínico de este paciente es producido por:
d} Varicela

96b. Se solicita biometría hemática, esperando encontrar:


a} Leucopenia

96c. El menor tiene una hermana de 21 días de nacimiento, la principal recomendación es:
c} Administrar gammaglobulina al neonato

97. ¿En qué momento se observa el efecto máximo de la insulina glargina?


e} No tiene

98. Paciente femenino de 38 años de edad, con embarazo de 17 semanas de gestación


acude a consulta de urgencias porque desde hace varios días inició con náusea, la cual ha
ido incrementando en intensidad hasta llegar al vómito. También refiere sangrado
transvaginal intermitente semejante a uvas, con poco dolor en hipogastrio. A la
exploración física se observa fondo uterino de 8 cm de longitud desde la sínfisis del pubis,
sin presencia de actividad uterina, no se palpan partes fetales y no se logra encontrar el
latido cardiaco fetal. Se realiza ultrasonografía y se observa imagen de copos de nieve y el
laboratorio reporta una elevación de la hormona gonadotropina coriónica humana
fracción beta. De acuerdo al diagnóstico más probable de la paciente ¿Qué conducta
terapéutica NO estaría indicada?
b} Colocación de dispositivo intrauterino

99. Paciente femenino de 50 años de edad y con sobrepeso acude a consulta externa
debido a que ha presentado en los últimos meses dolor en hipocondrio derecho,
generalmente después de ingerir alimentos (en particular en la noche y después de
consumir alimentos grasos), los cuales han llegado a interferir con su estilo de vida. El
dolor tiene duración entre 30 minutos y una horas. En ocasiones ha presentado náusea y
vómito acudiendo una vez al servicio de urgencias, donde la dieron de alta después de
algunas horas de hospitalización. La paciente trae los resultados de laboratorio de su corta
estancia en la sala de urgencias, los cuales no demuestran anormalidad alguna (biometría
hemática, electrólitos séricos y pruebas de función hepática). También presenta
ultrasonido de hígado y vías biliares con litos biliares, pero sin engrosamiento de la pared
ni calcificaciones en la vesícula biliar. ¿Cuál es el manejo más
adecuado en esta paciente?
b} Colecistectomía laparoscópica
1OO. Paciente masculino de 39 años de edad con dolor en el brazo derecho desde hace
varios años. El paciente menciona que debido a su trabajo, recientemente ha cargado
cajas pesadas y desde entonces el dolor ha empeorado, con presencia ocasional de
parestesias en el quinto y tercer dedos de la mano. La región dolorosa se localiza en la
porción medial del brazo y antebrazo, así como en la porción cubital de la mano. En
general, la fuerza muscular en la extremidad derecha se encuentra reducida en
comparación con la del lado izquierdo. Además, existe debilidad a los movimientos de
oposición y aducción del primer dedo. A la inspección es evidente la atrofia de las
eminencias tenar e hipotenar. ¿Cuál es el sitio donde más probablemente se encuentra la
lesión nerviosa?
a} Tronco inferior del plexo braquial

101. Un paciente de 40 años presenta hemianopsia homónima contralateral con


conservación de la visión macular posterior a la resección de un tumor en el lóbulo
occipital. ¿Cuál es el sitio más frecuente en el que se encuentra la lesión del paciente?
e} Fisura calcarina

102. Un atleta de alto rendimiento se presenta a consulta por debilidad en la extremidad


inferior derecha e inestabilidad de la marcha desde hace dos meses. El paciente refiere
que su sintomatología se ve aumentada durante el ejercicio. A la inspección, el paciente
muestra atrofia de la porción medial del muslo y debilidad para realizar la aducción de
éste. ¿Cuál es el nervio más probablemente afectado?
d} Nervio obturador

103. Un paciente de 22 años de edad acude al servicio de urgencias debido a que ha


presentado dificultad para abrir y cerrar su mandíbula posterior a una intervención dental
hace tres días. Existe sospecha de que durante la intervención dental se lesionó la rama
mandibular del nervio trigémino. ¿Cuál de los siguientes músculos se encuentra inervado
por la rama mandibular del nervio trigémino y es responsable de abrir la mandíbula?
a} Pterigoideo lateral

104. Paciente femenino de 85 años de edad con antecedente de depresión acude al


servicio de urgencias por debilidad y mareo. Al revisar el expediente clínico se observa que
toma furosemida 20 mg/día para reducir el edema de miembros inferiores. Menciona que
hace poco agregó a su lista de medicamentos la hidroclorotiazida para el manejo de la
presión arterial. Al interrogatorio niega fiebre, escalofríos, náusea, o vómito; menciona
que a últimas fechas ha tenido mucha sed, por lo que ha ingerido grandes cantidades de
jugo de manzana. A la exploración física la presión arterial en decúbito es de 100/50 mm
Hg, y sentada de 80/44 mm Hg, con un peso de 60 kg. La exploración pulmonar es normal.
Los estudios de laboratorio revelan osmolalidad sérica de 260 mOsm/kg, sodio sérico de
125 mEq/L, potasio sérico de 3.4 mEq/L, creatinina sérica de 0.8 mg/dl. El examen general
de orina revela una concentración de sodio urinario de 50 mEq/L,y la osmolalidad urinaria
de 200 mOsm/kg. Cuál de los siguientes"
diagnósticos es el más probable en esta paciente?
c} Hiponatremia secundaria a la administración de hidroclorotiazida

105. Masculino de 66 años con diagnóstico de DM tipo 2, gota, hipertensión,


hiperlipidemia e insuficiencia renal crónica, con creatinina de 2.8 mg/dL. Acude a
Urgencias por dolor torácico; el EKG muestra elevación del segmento ST y es llevado de
inmediato a la sala de hemodinámica para angioplastia primaria. Antes de iniciar se le
administra solución salina para prevenir la nefropatía por medio de contraste. Al finalizar
el procedimiento, el registro de enfermería indica que se administraron alrededor de 140
ml de medio de contraste no yodado de baja osmolaridad, así como hipertensión
transitoria. En los siguientes días desarrolla hipertensión, coloración púrpura en los dedos
del pie derecho, y hemorragia gastrointestinal. La creatinina sérica incremento a 6 mg/dl,
requirió hemodiálisis. ¿Cuál de las siguientes opciones es la causa de la agudización de la
insuficiencia renal en este caso?
a} Embolización de colesteroldesde la aorta

106. Paciente masculino de 45 años de edad, consulta por edema maleolar de 2 meses de
evolución. Antecedentes heredofamiliares negativos. No refiere haber padecido
infecciones, diabetes o enfermedades autoinmunes, y niega uso de medicamentos y
drogas. A la exploración se registra TA: 135/85, IMC: 22 kg/m2, edema maleolar con
fóvea. EGO: proteinuria ++++, hematuria 5 ertirocitos x campo, cilindros grasos.
Recolección de orina de 24 h con 5 g de proteína. Creatinina sérica 1.0 mg/dL, albúmina
2.3 g/dL,colesterol 450 mg/dL. Ultrasonido renal normal.

106a. Señale la causa más probable del cuadro que cursa el paciente:
b} Glomerulonefritis membranosa

106b. Estudio a realizar para confirmar el diagnóstico de glomerulonefritis membranosa.


d} Biopsia renal

106c. Señale los hallazgos que describen mejor el síndrome nefrótico:


d} Proteinuria, hipoalbuminemia, edema

106d. ¿A partir de qué nivel de proteinuria se considera rango nefrótico?


c} > 3.5 g/24 h

106e. Señale cuál es una de las medidas generales para disminuir la proteinuria en el
síndrome nefrótico:
a} IECA

107. ¿Cuál debe ser la duración de la anticoagulación en el contexto de síndrome de


anticuerpos antifosfolípidos y eventos vasculares cerebrales de repetición?
a} Permanente
108. Niña de cuatro años con masa abdominal de reciente diagnóstico, con antecedentes
de doble sistema colector,aniridia y retraso mental. En relación con la enfermedad que se
sospecha, señale lo INCORRECTO.
e} La hematuria está presente enla mayoría delos pacientes

109. ¿Cuál de las siguientes formas de porfiria se caracteriza por su aparición esporádica,
sin antecedentes familiares relacionados?
d} Porfiria cutánea tarda

11O. Paciente del sexo femenino que se diagnosticó con esclerodermia hace cinco años.
En su actual consulta de seguimiento la paciente menciona que presenta disfagia
principalmente a alimentos sólidos. ¿Cuál de los siguientes enunciados explica de mejor
forma la causa de la disfagia de la paciente?
c} Atrofia delmúsculo liso enla porción inferior del esófago

111. ¿Cuál de los siguientes cambios endometriales se observaría en caso de que se


realice una biopsia de endometrio a una paciente al final de la fase proliferativa del ciclo
menstrual?
c} Crecimiento de las arterias espirales

112. Paciente del sexo femenino de 55 años de edad que ha sido diagnosticada
recientemente con diabetes mellitus tipo 2. En este momento el índice de masa corporal
de la paciente es de 31.5 kg/mm2, y no consume bebidas alcohólicas. La paciente
suspendió el tabaquismo desde hace 1O años. Se decide iniciar tratamiento con
metformina. ¿Cuál de los siguientes estudios es recomendable realizar antes de iniciar el
tratamiento con metformina?
c} Concentración plasmática de creatinina

113. ¿Cuál de los siguientes efectos adversos corresponde a la administración de


doxorrubicina?
c} Edema de miembrosinferiores y ortopnea

114. ¿Cuál de los siguientes factores de coagulación ocasiona diátesis hemorrágica en


pacientes con fibrosis quística?
d} Factor VII

115. ¿Cuál de las siguientes aseveraciones es correcta respecto a la infección por


Helicobacter pylorí?
d} Después de la erradicación exitosa de lainfección por Helicobacter py/ori, la
recurrencia es menor de 5%"

116. ¿Cuál de los siguientes efectos adversos se relaciona principalmente con la


administración de foscarnet?
d} Hipofosfatemia
117. ¿Cuál de los siguientes medicamentos se utiliza en el tratamiento de pacientes que
padecen crisis convulsivas generalizadas tónico-clónicas en combinación con crisis de
ausencia?
b} Ácido valproico

118. Paciente femenino de 49 años de edad la cual se presenta por dolor en hipocondrio
derecho. La paciente tiene obesidad con un índice de masa corporal de 41 kg/m2 y
antecedente de gesta 4, para 4. A la exploración física la paciente presenta dolor intenso a
la palpación en el hipocondrio derecho, principalmente durante la inspiración,así como
dolor referido hacia el hombro. ¿Cuál de las siguientes estructuras nerviosas se encarga de
la sensación dolorosa en el hombro?
b} Nervio frénico

119. ¿Cuál de los siguientes antiarrítmicos prolonga la duración de la fase 3 del potencial
de acción mediante el bloqueo de los canales del potasio?
a} Amiodarona

120. ¿Cuál de los siguientes efectos adversos se asocia con el consumo de tamoxifeno?
c} Hiperplasia endometrial

121. ¿Cuál de las siguientes enfermedades se presenta con elevación del periostio,
secuestro e involucro óseo?"
c} Osteomielitis

122. ¿Cuál de las siguientes condiciones se asocia con normalidad de los parámetros de
función respiratoria?
b} Hipertensión pulmonar familiar

123. ¿Cuálde los siguientes fármacos se utiliza en eltratamiento de la eyaculación precoz?


e) Fluoxetina

124. Paciente masculino de 71 años en estudio de anemia microcítica. El equipo médico


del paciente decidió que era necesario realizar una colonoscopia para descubrir el origen
de la anemia. En la colonoscopia se encuentra una tumoración friable en el colon
descendente, sin embargo debido a que el paciente no realizó una preparación colónica
adecuada,el estudio no puede ser concluido. Los resultados de la biopsia demuestran que
padece un adenocarcinoma. El paciente no muestra datos en relación a obstrucción
intestinal. ¿Cuál de las siguientes conductas terapéuticas es la más adecuada en este
paciente inmediatamente?
a} Repetir la colonoscopia para lograr evaluar todo el colon
125. ¿Cuál de los siguientes mecanismos de supresión de expresión génica es el que
corresponde al que sucede en la enfermedad de Huntington?
c} Hipermetilación de histonas

126. ¿Cuál de los siguientes trastornos de la hemostasia se caracteriza por una deficiente
adhesión plaquetaria al subendotelio que no responde a la administración de
crioprecipitados?
a} Síndrome de Bernard-Soulier

127. ¿Cuál de los siguientes fármacos se asocia con una respuesta puramente alfa
adrenérgica ocasionando vasoconstricción?
a} Fenilefrina

128. Paciente masculino de 35 años de edad que es llevado al servicio de urgencias


después de que se encontró con pérdida total del estado de alerta en su casa hace
aproximadamente 3 horas. A la llegada al servicio de urgencias el paciente responde a
estímulos dolorosos. La presión arterial es de 100/65 mm Hg; frecuencia cardiaca de 112
latidos/minuto; frecuencia respiratoria de 1O respiraciones por minuto. La oximetría de
pulso se presenta normal. La enfermera menciona que ha notado un aliento característico,
a lo cual los familiares responden que al encontrarlo desmayado junto a él se encontraba
una botella de alcohol a 70%. Los estudios de laboratorio muestran la presencia de una
brecha osmolar,cetonemia, pero sin acidosis metabólica. La principal sospecha en este
momento es de una intoxicación por ingesta de isopropanol. ¿Cuál de los siguientes
tratamientos es el más adecuado para este paciente?"
b} Tratamiento de soporte únicamente

129. ¿Cuál de las siguientes translocaciones cromosómicas se presenta en la leucemia


aguda mieloblástica tipo M2?
c} t(8;21}

1. Paciente masculino de 65 años de edad con factores de riesgo cardiovascular se


presenta a la sala de urgencias con hemiplejía derecha, así como parálisis facial derecha
con preservación de los movimientos de la mitad derecha de la frente. El paciente
también muestra dificultad para la comunicación oral. El paciente se encuentra alerta con
frecuencia cardiaca de 100 latidos por minuto y presión arterial de 175/115 mm Hg, sin
dificultad ventilatoria y con una cifra de glucosa capilar de 130 mg/dL. Los hallazgos a la
exploración del fondo de ojo son normales. Los familiares refieren que el cuadro inició
hace aproximadamente una hora. Una tomografía computarizada cerebral resulta normal.
¿Cuál de los siguientes medicamentos es el tratamiento de elección inicial para este
paciente?
a} Labetalol
2. Masculino de 35 años acude a consulta debido a que a su esposa le diagnosticaron
gastritis por H. pylorien el chequeo anual de la oficina. El paciente no presenta molestias
al comer,ni toma medicamentos, pero quiere estar seguro de no tener esa enfermedad.
2a. Porcentaje de úlceras duodenales que presentan infección por H. pylori.
d} 90%

2b. Se considera como la prueba de tamizaje para H. pyilori.


e} Prueba del aliento

2c. Es el estándarde oro para el diagnóstico de infección por H. pylori.


b} Histología

2d. ¿Cuál es el tratamiento de primera elección para la enfermedad ácido pépticacausada


por H. pylori?
e} Lansoprazol + claritromicina 500mg + amoxicilina 1000 mg; 2xdía

2e. Con el tratamiento adecuado, el índice de erradicación es de alrededor de:


d} 85%

3. Paciente masculino de 25 años de edad, con antecedentes familiares por línea materna
de hemorragias. Acude a consultar por hematomas que aparecieron en ambos brazos tras
golpes mínimos desde la infancia. Ha recibido anteriormente tratamiento con
desmopresina por hemofilia tipo A sin resultados.
3a. Señale el tratamiento indicado en este paciente:
a} Factor VIII recombinante

3b. Tipo de herencia de la enfermedad con la que cursa el paciente.


c} Ligada al cromosoma X

3c. ¿Cuál de las siguientes manifestaciones hemorrágicas no forma parte de la clínica en


los pacientes con hemofilia?:
b} Petequias

3d. ¿Cuál de las siguientes alteraciones en el laboratorio se encuentran en los pacientes


con hemofilia?
b} Tiempo tromboplastina parcial alargado-tiempo de protrombina normal

3e. Señale el enunciado correcto respecto a la terapia anticoagulante:


d} El control dela medicación anticoagulante oral se realiza con elTP

4. El estudio inicial más valioso en el diagnóstico diferencial de hipertiroidismo es:


e} Estudio de medicina nuclear,gammagrafía
5. Ingresa a la sala de urgencias un paciente masculino de cuatro meses de edad, llevado
por sus padres debido a somnolencia excesiva. Al interrogatorio los padres mencionan
que desde que nació ha sido un bebé inactivo, pero que recientemente ha mostrado
disminución en el estado de alerta, anorexia y ""flacidez"". A la exploración física se
observan signos vitales dentro del rango de la normalidad; en cuanto a su desarrollo
psicomotor el paciente es incapaz de mantener su cabeza, y cuando se coloca un dedo en
la palma de su mano no hay respuesta; el reflejo de succión es débil; a la exploración
neurológica el paciente no presenta expresión facial o seguimiento con la mirada de
objetos brillantes, así como movimientos tonicoclónicos de la pierna izquierda. Los
estudios de laboratorio son relevantes únicamente por acidosis metabólica de brecha
aniónica elevada, incremento en los niveles de lactato y piruvato. ¿Cuál es el diagnóstico
más probable en este paciente?"
c} Deficiencia de piruvato deshidrogenasa

6. Paciente masculino de 22 años de edad cuya principal molestia es cefalea persistente


con resistencia a los analgésicos convencionales. A la exploración física presenta
incapacidad para dirigir la mirada hacia arriba y dilatación pupilar producida por la
acomodación, pero no por la aplicación de un estímulo luminoso. La realización de un
estudio de imagen revela hidrocefalia de tipo no comunicante. ¿Cuál de las siguientes
condiciones es la causa más probable de este trastorno?
a} Tumor de glándula pineal

7. Acude a consulta pediátrica un paciente de dos años de edad debido a que la madre ha
notado la presencia de heces de color marrón. Al interrogatorio, se niega cualquier
antecedente pediátrico u obstétrico de importancia. La exploración física no es relevante
(abdomen blando, no doloroso, no distendido, sin signos de irritación peritoneal). El
pediatra decide realizar un estudio de medicina nuclear con tecnecio 99 el cual demuestra
la presencia de mucosa gástrica ectópica en el cuadrante abdominal inferior derecho.
¿Cuál es el diagnóstico más probable en este paciente?
e} Divertículo de Meckel

8. Un paciente de 50 años de edad se presenta a la sala de urgencias debido a dolor


intenso a la abducción de su brazo después de caer de una bicicleta. El paciente describe
que cayó con el brazo extendido. El paciente se diagnostica con un desgarro del manguito
retador. ¿Cuál de los siguientes es la localización más frecuente de los desgarros del
manguito retador?
a} Músculo supraespinoso

9. Masculino de 75 años, diabético desde hace 15 años, en tratamiento con insulina,acude


a consulta por tener dificultad para iniciar la micción, además de sentir que no vacía
completamente la vejiga y gotea orina al terminar.La molestia empezó hace algunas
semanas, y no refiere alguna otra. Al interrogarlo comenta que desde hace un par de
meses presenta nicturia, aumento en la frecuencia urinaria -la cual le atribuye a la
diabetes-y disminución en el calibre del chorro urinario. EF: 1.68m, 1.65cm, TA: 140/80,
Temp: 36.5ºC.
9b. Al realizar el tacto rectal usted nota una próstata lisa, firme y elástica.Usted
diagnostica:
b} Hiperplasia prostática benigna

9c. Fase de la enfermedad que se caracteriza por el vencimiento del detrusor vesical.
c} Descompensación

9e. El manejo definitivo es:


c} Quirúrgico

1O. Paciente femenino de 19 años de edad con antecedente de hospitalizaciones


frecuentes por intoxicación alcohólica. En esta ocasión es ingresada después de haber
estado ingiriendo de manera constante bebidas alcohólicas desde hace cinco días. A la
exploración física, la presión arterial es de 110/70 mm Hg, frecuencia cardiaca de 80
latidos/min, peso de 70 kg. Se presenta letárgica y con habla incoherente. En la sala de
urgencias, presenta una crisis convulsiva tonicoclónica, la cual resuelve posterior a la
administración de diazepam intravenoso. Los estudios de laboratorio son significativos por
un niveles sérico de sodio de 11O mEq/L, potasio sérico de 3.8 mEq/L, nivel de alcohol de
250 mg/dl, osmolalidad sérica de 230 mOsm/kg, y concentración de glucosa de 92 mg/dl.
No hay alteración en la función renal. Después de iniciar tratamiento con soluciones
hipertónicas, el sodio sérico se encuentra en 120 mEq/L; la paciente ahora se encuentra
euvolémica y con mejor respuesta a estímulos. ¿Cuál de las siguientes opciones
representa el mejor manejo para la paciente en este momento?
a} Detener la infusión de líquidos, restringir la ingesta de agua, monitorización delsodio
sérico

11. Paciente femenino de 47 años de edad, acude a Urgencias, por dolor abdominal.
Refiere que el dolor empezó hace 4 h después de comer en un puesto de birria de la calle;
lo describe como muy intenso (8/1O), de inicio súbito en región central del abdomen y se
le recorre hacia la espalda. Desde que llegó al servicio ha vomitado en dos ocasiones. A la
exploración física se encuentra ansiosa, con facies de dolor y las piernas flexionadas sobre
el abdomen;se observa un hematoma en la zona del ombligo, hiperalgesia e
hiperbaralgesia con ruidos intestinales disminuidos. TA: 140/85, FC: 100 lpm, FR: 24x ',
Temp: 38.SºC."
11a. ¿Cuál es la principal sospecha diagnóstica?
a} Pancreatitis aguda

11b. ¿Qué alteraciones en los laboratorios confirmarían su diagnóstico?


c} Aumento de lipasa y amilasa (>3 veces}

11c. ¿En cuál de las siguientes opciones se podría observar un aumento o una disminución
del valor de la amilasa?
c} Cáncer de páncreas

11d. Para valorar la gravedad, usted utiliza la Escala de Ranson y obtiene un valor de 6
puntos, y al utilizar la Escala APACHEll obtiene un valor de 8.¿Cómo se interpretarían estos
resultados, respectivamente?:
a} Mortalidad de 40% y pancreatitis grave

12. Paciente masculino de 65 años de edad, con antecedente de tabaquismo, refiere


padecer dificultad para respirar al caminar por las mañanas desde hace 3 años, por lo que
mantiene una vida sedentaria. Acude a consulta por tos intermitente desde hace 3 meses.
Niega dolor torácico. A la auscultación se detecta disminución del murmullo vesicular. La
radiografía de tórax reporta aplanamiento diafragmático, disminución del trama vascular y
silueta cardiaca alargada.
12a. ¿Cuál es el punto de corte en la espirometría para el diagnóstico de EPOC?
b} FEV1/FVC < 70%

12b. Esquema farmacológico indicado en este paciente.


a} Tiotropio + salbutamol/ipratropio

12c. ¿Cuáles son las medidas que han demostrado disminuir la mortalidad en la EPOC?
b} Abandono deltabaco-oxigenoterapia

12d. Señale los criterios de Winnipeg que definen exacerbación de la EPOC:


d} Aumento de disnea + aumento de expectoración + purulencia delesputo

12e. ¿Cuál es el tratamiento indicado ante una exacerbación de la EPOC de causa


infecciosa?
e} Amoxicilina + ácido clavulánico

13. Paciente masculino de 37 años de edad, diseñador gráfico, homosexual,con infección


por VIH diagnosticada hace dos años, acude al servicio de urgencias por presentar fiebre
de 15 días de evolución,ataque al estado general,pérdida de peso, disnea progresiva de
esfuerzos y tos con expectoración amarillenta. Se realizó tinción de la expectoración, que
reportó bacilos ácido-alcohol resistente. Su cuenta de CD4 realizada hace un mes fue de
100 células/ml y una carga viral reciente de 250 000 copias.
¿Cuál de las siguientes aseveraciones en relación a la tuberculosis en el paciente con SIDA
es correcta?
d} El tratamientoinicial para pacientes VIH positivos con tuberculosis debe ser elmismo
que el que se utiliza para los pacientes no infectados con VIH

14. Paciente masculino de 30 años de edad, fumador,resultó involucrado en un choque


automovilístico hace 2 h. Los paramédicos descartaron en el lugar lesiones mayores.
Acude por dolor en la rodilla derecha y comenta que en el choque ésta golpeó con el
tablero. EF: presenta dificultad para caminar, dolor y edema en la región de la rodilla
derecha; la tibia se desplaza posteriormente rebasando los cóndilos femorales.
14a. ¿Qué prueba elegiría para explorar los ligamentos cruzados?
d} Cajón anterior y posterior

14b. ¿Qué evalúa la prueba del cajón posterior?


a} LCP

14c. ¿Cuál es la técnicamás sensible para evaluar la integridad de los ligamentos cruzados?
a} RMN

14d. Una lesión grado 2 (según la prueba del cajón) significa que:
c} La tibia puede subluxarse en unalíneaigual al aspecto anterior delos cóndilos femorales

14e. El manejo inicial de las lesiones de los ligamentos cruzados es:


a} Rehabilitación y manejo deldolor

15. Masculino de 65 años con diagnóstico de HTA desde hace 15 años (en tratamiento con
captopril) e hipermetropía,acude a Urgencias por haber presentado el día de hoy por la
tarde un episodio de dolor en el ojo izquierdo y cefalea en hemicráneo del mismo lado,
acompañado con blefaroespasmo, lagrimeo e incluso náusea. El episodio duró cerca de 15
min y después cedió paulatinamente; comenta que ya había tenido antes eventos
similares, todos por la tarde, pero con mucha menor intensidad. No refiere pérdida de la
visión, sólo aclara que las luces las observaba con un halo. EF: 1.68m, 1.65cm, TA: 140/80
mm Hg, Temp: 36.5ºC. A la exploración se encuentra la córnea con un aspecto turbio e
inyección mixta.
15a. ¿Cuál es la principal sospecha diagnóstica?
b} Glaucoma de ángulo cerrado

15c. Eventos patológicos que ocurren (en orden) al aumentar la PIO.


a} Edema cornea! a alteración de la visión a dolor intenso a signos vagales a colapso de
arteria retiniana

16. Paciente masculino de 22 meses es traído a urgencias por su madre, quien refiere que
esta mañana al cambiar el pañal del paciente lo encontró con sangrado abundante
proveniente del recto. El paciente se observa tranquilo, no manifiesta dolor.A la
exploración física se encuentra sin dolor a la palpación abdominal,sin datos de sangrado
activo y sus signos vitales en parámetros normales.
¿Cuál es el diagnóstico más probable?
d} Divertículo de Meckel
17. Paciente masculino de 13 meses de edad con historia de fiebre de tres días de
evolución (temperatura máxima 39.8 C) que se resolvió y después apareció rash en
tronco, diseminándose después a brazos, cuello y cara. En la exploración se encuentran
lesiones maculopapulares rosadas, no confluentes, en cara, cuello, brazos y tronco.
¿Cuál es el agente etiológico más probable?
e} Herpes virus tipo 6

18. Un niño de siete años es llevado a consulta por retraso en el crecimiento. Al


interrogarlo se identifican otros síntomas, como cefalea, vómito, disminución de la
agudeza visual, poliuria y polidipsia. En estudio de imagen se desmuestra la presencia de
tumor en silla turca.
18a. ¿Cuál es el tumor que causa hipopituitarismo con más frecuencia en niños?
e} Craneofaringioma

1Bb. ¿Cuál es el manejo más adecuado?


a} Resección quirúrgica

19. Paciente masculino de 45 años de edad el cual se presenta con fatiga, palidez de piel y
mucosas, y soplo sistólico. En los estudios de laboratorio se presenta un hematocrito de
27%, volumen corpuscular medio de 80 fl,concentración corpuscular media de
hemoglobina de 25 pg/célula y reticulocitos de 4.5%, así como una concentración de
ferritina de 340 µg/L. El frotis de sangre periférica demuestra macrocitos policromatófilos.
¿Cuál es el diagnóstico más probable en este paciente?
a} Disfunción de médula ósea

20. Paciente femenino de 45 años de edad con antecedente de cáncer de glándula


mamaria. La paciente fue diagnosticada hace dos meses y se le realizó una mastectomía
radical con extirpación de ganglios cervicales. Posteriormente fue programada para
administración de quimioterapia. Posterior a la administración de quimioterapia la
paciente presenta náusea y vómito incontrolable por lo que se administra un fármaco
para disminuir esta sintomatología, Aunque el medicamento disminuyó la náusea y el
vómito, también ocasionó constipación. ¿Cuál de los siguientes fármacos es el que más
probablemente se le administró a esta paciente?
a} Un antagonista de los receptores de serotonina 5HT3

21. Paciente masculino de 70 años con dolor en hipocondrio derecho desde hace 1O
horas. A la exploración física se presenta ictérico y con fiebre. Los estudios de laboratorio
revelan elevación leve de las transaminasas, hiperbilirrubinemia (6 mg/dL) y leucocitosis
(16 000 células/mm3). ¿Cuál de las siguientes conductas es la más adecuada en este
paciente?
c} Solicitar una colangiopancreatografía endoscópica retrógrada
22. ¿Cuál de los siguientes fármacos antihipertensivos es adecuado para el manejo de
hipertensión arterial sistólica aislada en pacientes no diabéticos?
a} Amlodipino

23.¿Cuál de las siguientes opciones representa la lesión glomerular intrínseca primaria


más frecuente?
b) Nefropatía porlgA

24. Paciente masculino de 66 años de edad que acude a consulta por presentar desde
hace varias semanas febrícula, poliartralgias, y exantema eritematoso localizado
principalmente en la cara, aunque hace poco tiempo se ha observado también en la
región superior del tórax. Al interrogatorio el paciente menciona que recientemente sufrió
un infarto agudo al miocardio, así como una arritmia (no especificada en este momento).
En los exámenes de laboratorio se revela positividad para el anticuerpo antinuclear. ¿Cuál
de los siguientes medicamentos es el que está tomando este paciente con mayor
probabilidad?"
b} Procainamida

25. Paciente masculino de 70 años de edad conocido por el servicio de cardiología porque
acude desde hace tres años por cuadros de angina estable. En esta ocasión asiste al
servicio de urgencias por presentar dolor torácico intenso. La enfermera toma un
electrocardiograma, el cual revela elevación del segmento ST; las pruebas enzimáticas
rápidas determinan elevación de la creatinincinasa-MB. El paciente es estabilizado y
admitido a la unidad coronaria. Varios días después de este evento presenta de nuevo
dolor torácico. A la exploración física presenta fiebre, frote pericárdico intenso, estertores
pulmonares y edema de extremidades inferiores. ¿Cuál de los siguientes diagnósticos es el
más probable en este paciente?
d} Pericarditis fibrinoide

26. ¿De los métodos diagnósticos presentados a continuación, cuál es el más adecuado
para realizar el diagnóstico de embolia pulmonar en un paciente con un proceso
neumónico subyacente?
e} Tomografía computarizada espiral contrastada deltórax

27. Paciente masculino de 44 años de edad que se presenta a consulta por intensos
dolores cólicos, constipación, irritabilidad y cefalea. El paciente trabaja en una fábrica de
pintura en aerosol. Presenta unos estudios de laboratorio que el médico laboral le solicitó;
la principal característica de éstos es una anemia microcítica. Al revisar el expediente
médico el paciente ha presentado episodios depresivos mayores en el pasado. ¿Cuál de
las siguientes sustancias es con mayor probabilidad la causante la sintomatología de este
sujeto?
e} Plomo
28. Se requiere de su presencia en la sala de parto debido a que se espera el nacimiento
de un masculino de 37 semanas de gestación. Observa los procedimientos del obstetra, no
se sucitan complicaciones y recibe un recién nacido masculino. Inicia la exploración física
de éste y observa movimiento activo, frecuencia cardiaca de 139 lpm, buen tono
muscular, flexión de las cuatro extremidades, acrocianosis y llanto vigoroso.
28a. De acuerdo con las características del recién nacido, ¿qué puntaje de Apgar obtiene?"
e} 8 puntos

28b. Al recibir a un recién nacido vigoroso como en el caso descrito, ¿cuál de los siguientes
pasos se debe realizar primero?
a} Verificar que la vía aérea esté permeable, secar,y estimular al recién nacido

28c. En caso de no contar con la información de la edad gestacional del recién nacido
descrito anteriormente, ¿cuál de los siguientes signos clínicos le indicaríamejorque se
trata de un recién nacido a término?
c} Tejido mamario palpable

28d. El neonato pasa a cuidados en cuneros y se comienza el protocolo de recién nacido


por las enfermeras, el cual incluye escrutinio de hipoglucemia. En el neonato a término
asintomático, ¿a qué nivel de glucosa debe iniciarse la evaluación y tratamiento de
hipoglucemia?
e} 45 mg/dl

28e. Después de una semana de nacido, el paciente es llevado a consulta de revisión.


¿Cuál de las siguientes opciones es la correcta respecto al peso de un recién nacido en la
primera semana de vida?
d} Perder aproximadamente 5 a 10% delpeso al nacimiento

29. Paciente masculino de 50 años de edad con esófago de Barrett que se presenta con
gastroenterólogo para endoscopia de seguimiento. Se
toman múltiples biopsias y se envían a estudio histopatológico. El reporte revela la
presencia de displasia de bajo grado. ¿Cuál de las siguientes conductas clínicas es la más
adecuada?
a} Repetir endoscopia en 6 y 12 meses

30. Paciente masculino de tres meses de edad que acude a consulta pediátrica de rutina.
En la evaluación es evidente que padece leucocoria. Después de evaluación oftalmológica
se determina que la leucocoria es debida a un retinoblastoma. Se realiza enucleación del
ojo izquierdo.
¿Cuál de las siguientes estructuras del ojo extraído es la que merece especial atención por
el patólogo porque determina el pronóstico?
c} Nervio óptico
31. Paciente masculino de 19 años es llevado al servicio de urgencias por disnea grave y
dolor localizado a la mitad derecha del tórax. Los familiares comentan que la
sintomatología inició hace más o menos dos horas sin relación con alguna actividad o
evento. Al interrogatorio el paciente comenta que el dolor es más intenso durante la fase
inspiratoria, y niega la presencia de fiebre o cualquier otro síntoma que pueda orientar al
diagnóstico. Entre los antecedentes de importancia está el de tabaquismo desde los 17
años a razón de 15 cigarros al día. A la exploración física se encuentra una frecuencia
respiratoria levemente incrementada (25 respiraciones/min); la auscultación revela
disminución de la intensidad de los sonidos respiratorios en el pulmón derecho con
timpanismo a la percusión. La saturación de oxígeno sin administración de oxígeno
suplementario es de 96%. x La radiografía de tórax muestra la presencia de neumotórax
en 50% del pulmón derecho. ¿Cuál de las siguientes conductas terapéuticas es la más
adecuada en este paciente?
d} Aspiración con aguja del neumotórax

32. ¿Cuál de los siguientes antiarrítmicos actúa tanto en la fase O como en la fase 3 del
potencial de acción?
e} Disopiramida

33. Paciente masculino de 35 años de edad, bajo tratamiento por depresión mayor, se
presenta al servicio de urgencias por presentar confusión intensa y alucinaciones. A la
exploración física el paciente es poco cooperador, pero se observa que la boca está seca y
la cara eritematosa. La enfermera toma los signos vitales, resultando en hipotensión
(85/50 mm Hg) y taquicardia (11O latidos/minuto). Se realiza un electrocardiograma, el
cual revela prolongación del intervalo QT (0.13 mseg) y contracciones ventriculares
prematuras. ¿Cuál de los siguientes medicamentos es el más adecuado para este
paciente?
e} Bicarbonato de sodio

34. ¿Cuál de las siguientes características posee la mola hidatidiforme completa?


a} El genotipo de las células de la mola es 46,XX y es completamente de origen paterno

35. Paciente masculino de 18 años de edad que decide practicar alpinismo. No cuenta con
ningún antecedente de importancia. Al llegar a la cima el paciente inicia con disnea y
estertores difusos bilaterales. ¿Cuál es el diagnóstico más probable en este paciente?
a} Edema pulmonar de grandes altitudes

36. ¿Cuál de las siguientes interacciones farmacológicas disminuye el efecto


anticoagulante de la warfarina?
b} Fenitoína
37. Paciente masculino de 69 años de edad que se presenta a consulta con pérdida de
peso no intencionada de aproximadamente 13 kg, disnea durante el ejercicio y tos
productiva acompañada de sangre desde hace dos meses. Entre los antecedentes de
importancia de este paciente se encuentra el antecedente de tabaquismo intenso desde
los 15 años de edad, a razón de una cajetilla diaria. La radiografía de tórax demuestra una
tumoración en el lóbulo pulmonar superior del pulmón derecho. Se decide realizar
tomografía computarizada la cual revela una tumoración de 5 cm de diámetro en el lóbulo
superior acompañada de adenomegalias mediastinales derechas. La realización de una
tomografía por emisión de positrones revela tres ganglios mediastinales del lado derecho
y uno del izquierdo. ¿Cuál es el siguiente paso más adecuado en el manejo de este
paciente?
a} Biopsia de los ganglios mediastinales

38. ¿Cuál de los siguientes trastornos se presenta con tiempo de sangrado en rangos
normales?
d} Púrpura de Henoch-Schonlein

39. ¿Cuál de los siguientes fármacos se asocia con respuestas puramente beta-1 y beta-2
adrenérgicas?
d} Dobutamina

40. ¿Cuál de los siguientes medicamentos es una benzodiacepina con bajo riesgo de
producir síndrome de abstinencia?
a} Clorodiazepóxido

41. ¿Cuál es la principal causa de muerte en pacientes con enfermedad de Kawasaki?


b} Secuelas cardiacas

42. La mutación en el gen de la cistationina beta-sintasa ocasiona gran variabilidad


fenotípica, incluyendo alteraciones musculoesqueléticas, retraso mental y trombosis.
¿Cuál es el nombre de esta propiedad de variabilidad fenotípica?
b} Peliotropía

43. ¿Cuál de los siguientes organismos se asocia con el desarrollo de colangiocarcinoma?


e} Clonorchis sinensis

44. ¿Cuál es el mecanismo de acción por el que la carbidopa puede mejorar la


sintomatología de la enfermedad de Parkinson?
c}Incremento de la biodisponibilidad de la dopamina por medio dela inhibición dela
enzima dopamina descarboxilasa

45. ¿Cuál de los siguientes receptores es al que se une con más afinidad la naloxona?
e} Mu
46. ¿Cuál de los siguientes medicamentos se asocia con síndrome nefrótico como efecto
adverso?
e} Penicilamina

47. ¿Cuál de los siguientes trastornos se caracteriza por niveles disminuidos de la hormona
foliculoestimulante, luteinizante y testosterona en combinación con un bajo conteo de
espermatozoides?
d} Hipogonadismo central

48. Paciente femenino de 75 años de edad operada de cirugía de recambio total de


cadera. Después de unos días de hospitalización es egresada a su domicilio con varios
medicamentos, entre ellos ketorolaco. Alrededor de 48 horas después a su egreso percibe
una reducción significativa de su gasto urinario y malestar general, por lo que sus
familiares deciden regresar al hospital. En el servicio de urgencias se le practican estudios
de laboratorio que revelan principalmente incremento en la concentración de nitrógeno
ureico en sangre (66 mg/dl) y de la creatinina (6.7 mg/dl). ¿Cuál de los siguientes
mecanismos es el responsable de la insuficiencia renal aguda de esta paciente?
a} La administración de ketorolaco ocasionó inhibición dela síntesis de prostaglandinas
y constricción delas arteriolas aferentes en el riñón

49. Paciente masculino de 27 años de edad que muestra disuria y secreción purulenta
uretral. Es tratado con ceftriaxona intramuscular, dosis única, pero los síntomas no
mejoran. ¿Cuál de los siguientes medicamentos es más probable que resuelva la
sintomatología de este paciente?
e} Azitromicina

50. Paciente masculino de 55 años de edad con diagnóstico de infección por el virus de la
inmunodeficiencia humana y con síndrome de inmunodeficiencia adquirida. El motivo
principal de consulta es que el paciente ha presentado diarrea, fiebre y pérdida de peso
desde hace dos semanas. ¿Cuál de los siguientes estudios diagnósticos determinaría la
presencia de una colitis por citomegalovirus?
a} Colonoscopia con toma de biopsia

51. Granjero de 75 años casado, con antecedente de síndrome del seno enfermo tratado
con marcapasos y warfarina, dos episodios de evento vascular cerebral, hipertensión y
gota tratada con alopurinol. Toma enalapril, verapami l,albuterol y digoxina. La evaluación
de laboratorio reveló hipocalciuria e hiperoxaluria extrema. Fue sometido a extracción
uretroscópica de dos cálculos de oxalato de calcio. En relación al citrato potásico, señale la
respuesta correcta:
c} Proporciona un aumento de la alcalinidad dela orina
52. Se presenta a consulta pediátrica un paciente del sexo masculino de dos años de edad.
A pesar de que el paciente había presentado desarrollo y crecimiento normales en el
primer semestre de vida, en esta ocasión se presenta con un claro retraso del desarrollo. A
la exploración física es evidente que el paciente no responde a estímulos visuales, y
durante la oftalmoscopia se observa una mancha rojo cereza en la mácula. ¿Cuál de los
siguientes procesos es el que se encuentra alterado en este paciente?
e} Alteración del metabolismo delos glucolípidos en elcerebro

53. Paciente masculino de 77 años de edad ingresa en la madrugada por exacerbación de


insuficiencia cardiaca. A la exploración física presenta disnea,taquipnea y estertores
basales. Un residente de Medicina Interna x poco experimentado admite al paciente y
escribe las indicaciones para el personal de enfermería. Al día siguiente el paciente parece
que ha empeorado. Durante el paso de visita el médico adscrito revisa las indicaciones del
residente y nota que éste ordenó la administración de un diurético que fue el responsable
del empeoramiento. ¿Cuál de los siguientes diuréticos es el que con mayor probabilidad
se le administró al paciente?
e} Manitol

54. ¿Cuál de las siguientes arterias puede sufrir hemorragia en caso de úlcera perforada
en la porción posterior de la primera parte del duodeno?
a} Arteria gastroduodenal

55. ¿Cuál es la conducta más adecuada a seguir en el caso de un paciente que después de
un mes de completar adecuadamente un esquema de erradicación de Helicobacter pylorí
aún continúa con dispepsia y dolor epigástrico después de ingerir alimentos?
a} Realizar un estudio de ureasa en aliento

56. Paciente masculino de 45 años, sin antecedentes personales patológicos, ingresa a


urgencias a causa de un episodio de dolor agudo en el flanco derecho y hematuria. El
dolor comenzó por la noche e incluso lo despertó. A la exploración física: Giordano
derecho. Se solicitan laboratorios de rutina que revelan calcio y fósforo en rangos
normales, cultivo de orina negativo y hematuria. La radiografía simple de abdomen
descubre un lito en el riñón derecho. El urograma intravenoso muestra que el cálculo no
esta obstruyendo en este momento. ¿Qué tipo de lito es más probable que haya causado
esta condición en el paciente?
a} Oxalato de calcio

57. Un hombre de 46 años de edad se presenta a consulta por dolor en la articulación del
codo izquierdo, el cual se incrementa durante las horas de trabajo (es plomero). El
paciente acude porque el dolor se ha hecho tan intenso al grado que sólo levantar un
tornillo o clavo ocasiona dolor.A la exploración física refiere el dolor en el epicóndilo
lateral,y se resiste a la extensión de la muñeca por el dolor que produce. ¿Cuál es el
diagnóstico más probable en este paciente?
b} Epicondilitis lateral
58. Se presenta a la consulta externa de pediatría un paciente del sexo masculino de 13
años de edad el cual manifiesta hematomas en varias regiones del cuerpo. A la
exploración física se puede observar que presenta piel suave e hiperelasticidad, así como
equimosis en la parte posterior del antebrazo y en las regiones tibiales. Se decide realizar
una biopsia de piel y enviarla a evaluación por microscopia electrónica. El reporte
menciona que existen fibrillas de colágena extracelular anormalmente delgadas e
irregulares. Evaluación bioquímica posterior revela un número incrementado de puentes
de disulfuro en las regiones terminales. ¿Cuál de las siguientes fases de la síntesis de la
colágena se encuentra afectada en este paciente?
a} Segmentación extracelular

59. Paciente femenino de 36 años que acude a consulta por comenzar con escapes
urinarios al mes de practicársele una cesárea por desproporción cefalopélvica. Le han
realizado intervención de fístula sacrococcígea. Antecedentes obstétricos de cesárea en
1990 y un parto distócico mediante fórceps por expulsivo prolongado en 1994. Recibió
tratamiento de una infección del tracto urinario con posterior urocultivo negativo. Al
interrogatorio dirigido refiere escapes continuos de orina clara que se desencadenan con
la bipedestación y que aumentan con los esfuerzos, sin sintomatología de urgencia
miccional acompañante. En la exploración física se aprecia cuello uterino de multípara
bien epitelizado, útero en anteversión, zonas anexiales libres de masas. La sensibilidad
vulvar y los reflejos perineales (anal y bulbocavernoso) están conservados. Tras realizar
prueba de incontinencia con azul de metileno, se observa a través del orificio cervical
externo fluir líquido teñido de azul de forma espontánea, que aumenta con maniobras de
Valsalva, no apreciándose escapes a través de uretra. Se solicita una cistografía en la que
se comprueba,con vejiga en máxima repleción,el paso de contraste a través de un
trayecto fistuloso a cavidad uterina. Diagnosticando fístula vesicouteri na, se programa
para fistulorrafia vesicouterina. Con respecto al caso de incontinencia urinaria de
esfuerzo, todas las aseveraciones son ciertas, EXCEPTO:
e} Es poco frecuente en mujeres posmenopáusicas o multíparas

60. Niña de 18 meses de edad que presenta dermatitis de la zona del pañal desde hace
tres semanas. La han tratado con talco y quadríderm por dos semanas sin observar
mejoría. A la exploración física, aparte de observar eritema, erosiones, fisuras y costras
serohemáticas, nota presencia de pápulas eritematosas satélites que se extienden hasta la
región inguinal y tercio superior de la cara interna de los muslos. ¿Cuál es el agente
invasor más frecuente en estos casos?
b} Candida albicans

61. Femenino de 62 años llevado a sala de urgencias por experimentar episodios


frecuentes de ptosis, diplopía y fatiga generalizada. EF: se encuentra timo palpable y
parálisis del nervio oculomotor que se corrige transitoriamente al administrar edrofonio.
La entidad nosológica responsable de la sintomatología de este paciente es:
a} Miastenia grave
62. Un paciente bajo tratamiento antipsicótico por esquizofrenia acude a la consulta por
exceso de rigidez muscular, hipertermia y disminución de la sudación. A su ingreso el
paciente presenta cambios bruscos en las cifras tensionales, acompañados de episodios
de taquicardia y bradicardia. ¿Cuál de los siguientes medicamentos se debe administrar
inmediatamente a este paciente?
d} Dantroleno

63. Paciente masculino de 59 años de edad que acude a evaluación por padecer disnea de
medianos esfuerzos y dolor torácico. En el expediente clínico no hay ningún antecedente
de importancia para este padecimiento. A la exploración física los pulsos de las carótidas
se encuentran retrasados respecto al ciclo cardiaco, así como disminuidos en intensidad.
El punto de máxima intensidad cardiaco se muestra hiperdinámico, pero sin
desplazamiento. A la auscultación se encuentra un soplo sistólico en crescendo-
decrescendo a lo largo del borde esternal izquierdo. El componente aórtico del segundo
ruido cardiaco se encuentra ausente. En el electrocardiograma se encuentran datos en
relación a hipertrofia ventricular izquierda. La radiografía de tórax no muestra incremento
del tamaño de la silueta cardiaca y campos pulmonares sin alteraciones. ¿Cuál es el
tratamiento más adecuado en este paciente?
b} Angiografía coronaria y después cirugía

64. Hombre de 32 años, previamente sano, inicia con fiebre de 38 C, cefalea y malestar
general,tomando sólo tratamiento sintomático. Al día siguiente, por persistencia del
cuadro, acude al servicio de urgencias, donde se encuentra con Glasgow de 14, febril, con
rigidez de nuca y signo de Brudzinsky positivo.
64a. Dado su sospecha diagnóstica, decide realizar una punción lumbar. ¿Cuál de las
siguientes representa una contraindicación absoluta para realizar este procedimiento?
a} Infección de la piel en elsitio de la punción

64b. El resultado del líquido cefalorraquídeo indica 2 700 cel/µL, con 84% de
polimorfonucleares, glucosa de 36 (sérica de 94) y proteínas de 175 mg/dl. Según su
sospecha clínica y estos resultados, el agente causal más probable en este caso es:
b} Streptococcus pneumoniae

64c. Dicho microorganismo es:


d} Diplococo grampositivo

64d. El tratamiento empírico que se debe proporcionar en este caso es:


a} Cefotaxima + vancomicina

64e. El uso de corticoesteroides en este caso:


d} Se recomienda administrar antes de la primera dosis de antibióticos y hasta cuatro días
delinicio de tratamiento
65. Con relación al ataque agudo de gota:
a} Hasta 30% de los pacientes puede tener durante el ataque agudo ácido úrico normal

66. Neonato de seis horas de vida cuya madre fue positiva para antígeno de superficie de
hepatitits B (HBsAg). ¿Cuál es la conducta más adecuada en relación a profilaxis contra
hepatitis B?
c} Administrar la primera dosis de vacuna anti-hepatitis B e inmunoglobulina de hepatitis
B antes de las primeras 12 horas de vida

67. Paciente masculino de 72 años de edad acude a consulta por mareo y ojo rojo crónico
con disminución de la agudeza visual. Se realiza un angiograma, el cual demuestra retraso
en la visualización de los vasos retinianos con hemorragias en el polo posterior y periferia.
¿Cuál es la condición con más probabilidad de ser responsable de este cuadro?
a} Estenosis carotídea

68. Paciente femenino de 88 años de edad es hospitalizada por una caída en su casa
debido a tropezón. No puede caminar o soportar peso sobre su extremidad inferior
derecha. Las radiografías revelan que tiene una fractura desplazada del cuello del
fémur,sin demostrar algún otro hallazgo de importancia. Al interrogario niega cualquier
otro traumatismo en la región afectada. Posterior a la estabilización de esta paciente,
¿cuál es el tratamiento más óptimo para su tipo de fractura?
d} Hemiartroplastia

69. Paciente femenino de 25 años de edad, fumadora, gesta 2, para 2 embarazos y parto
sin complicaciones, sexualmente activa (dos parejas en el último mes) acude por presentar
desde hace tres días secreción vaginal, maloliente, con prurito genital leve. A la
exploración física muestra FC 78 lpm, FR 18 resp/min,temperatura 36.8 C, abdomen no
doloroso, sin datos de irritación peritoneal, vagina con secreción amarillo-verdosa
espumosa, maloliente, paredes vaginales eritematosas y cuello uterino con puntilleo rojo-
violáceo. Resto de la exploración sin alteraciones.
69a. ¿Cuál es la etiología más probable?
e} Trichomona vaginalis

9b. ¿Cuál es el tratamiento más adecuado?


d} Metronidazol a la paciente y su pareja

70. Paciente femenino de 33 años de edad que acude a consulta por presentar debilidad y
parestesias en los primeros tres dedos de la mano derecha. La paciente informa también
dolor localizado a la articulación de la muñeca. Durante el interrogatorio acepta que los
síntomas son peores cuando escribe en la computadora. Lo más probable es que la
paciente presente un síndrome del túnel del carpo. ¿Cuál de las siguientes pruebas es la
más sensible para realizar este diagnóstico?
b} Prueba de compresión
71. Paciente femenina de 60 años de edad con diabetes mellitus tipo 2 de larga evolución
e hipertensión. Acude a consulta de seguimiento por haber presentado proteinuria en el
examen de orina hace 12 meses. Sus medicamentos incluyen metformina, metoprolol y
IECA. Refiere debilidad muscular desde hace una semana y edema. Filtrado glomerular
estimado: 55 mUmin/1.73 m2.
71a. ¿Cuál de las siguientes afirmaciones es verdadera de acuerdo con la definición de
enfermedad renal crónica?
b} Las alteraciones estructurales o funcionales renales deben estar presentes >3 meses

71c. Respecto al tratamiento de la hiperpotasiemia,señale la medida indicada en la


paciente para disminuir el riesgo de arritmia mortal y paro cardiaco:
c} Gluconato de calcio

71d. Señale la causa principal de la anemia en los pacientes con enfermedad renal crónica:
d} Déficit de eritropoyetina

71e. ¿Cuál de las siguientes anormalidades del metabolismo de la hormona paratiroidea


provoca la enfermedad renal crónica?
c} Hiperparatiroidismo secundario

72. Paciente masculino de tres años de edad, con antecedente de otalgia y otorrea
derecha, desde los dos años de edad, en cuatro ocasiones las cuales remitieron con
manejo antimicrobiano. Refiere la madre cuadro de vías respiratorias superiores de 48 h
de evolución; acude por otalgia derecha. Exploración física. Temperatura 38 ºC. Otoscopia
derecha con membrana timpánica hiperémica, no móvil. Otoscopia izquierda con
membrana timpánica opaca y no móvil. Cavidad oral con amígdalas grado 3, crípticas sin
exudados. Cuello sin adenomegalias.
72a. El diagnóstico más probable es:
e} Otitis media aguda recurrente

72b. El microorganismo con más frecuencia implicado en esta patología es


e} Streptococcus pneumoniae

72c. El tratamiento consiste en:


e} Amoxicilina

72d. Dos meses después el paciente es llevado a consulta porque presenta sensación de
plenitud aural y la madre refiere que escucha el televisor con un volumen más alto del
habitual. En la otoscopia se observan las membranas timpánicas íntegras, opacas y no
móviles. Usted solicita una impedanciometría que reporta disminución en la complianza
de la membrana timpánica. La conducta siguiente sería:
a} Manejo expectante
73. Paciente masculino de 70 años que acude a consulta por problemas de visión. Refiere
que en los últimos meses ha notado visión borros justo en el frente de su campo
visual,que no se puede corregir con sus lentes. Este problema le afecta ambos ojos. En sus
antecedentes patológicos menciona hipertensión,tratada con amlodipino. Refiere
tabaquismo de 20 paquetes/año. Al mostrarle la cartilla de Amsler dice notar un área
borrosa y asimétrica cerca del centro. A la oftalmoscopia se observan depósitos de drusen
en la mácula y áreas de depigmentación. No hay datos de neovascularización. ¿Cuál sería
el tratamiento indicado en este paciente?
e} Observación

74. Paciente masculino de 19 años con desnutrición, refiere astenia,ataque al estado


general,tos, expectoración,sudoración nocturna y un episodio de hemoptisis. Los análisis
de laboratorio muestran anemia ferropénica. En la radiografía de tórax se aprecian
múltiples infiltrados intersticiales en ambos campos pulmonares junto con engrosamiento
pleural apical. Mantoux de 15 mm de induración. Se realiza fibrobroncoscopia y en la
baciloscopia del broncoaspirado se aíslan BAAR con cultivo de Lowestwin-Jensen positivo
para Mycobacteríum tuberculosis. Se instaura tratamiento con fármacos antituberculosos.
¿Que estudio(s) se realizan(n) con el objeto de estudiar esta probable caso de
tuberculosis?
d} Todos los anteriores

75. Paciente femenino de 25 años acude a urgencias por presentar secreción vaginal
maloliente asociada con ardor y prurito vulvar,disuria, polaquiuria, dispareunia y sangrado
poscoital. La paciente tiene relaciones sexuales regularmente, con dos parejas masculinas
en los últimos seis meses. A la exploración física se observa eritema de la vulva y mucosa
vaginal, secreción verde-amarillenta y hemorragias puntiformes en la vagina y el cuello
uterino. El análisis de la secreción con solución salina en el microscopio muestra
tricomonas móviles.
¿Cuál de los siguientes enunciados es falso con respecto al tratamiento de la
tricomoniasis?
b} El tratamiento tópico vaginal esigualmente efectivo que eltratamiento por vía oral

76. Una paciente de 20 días de edad es llevada a la sala de urgencias. La madre menciona
que la paciente ha presentado fiebre y llanto continuo las últimas 4 horas; la paciente ha
sido alimentada el día de hoy en una sola ocasión pero vomitó toda la fórmula ingerida. Se
refiere que la paciente fue producto de un parto vaginal a término y eutócico de una
madre de 22 años gesta 1, para 1. Ningún otro antecedente de importancia. A la
exploración física la paciente presenta temperatura de 38.3 ºC, pulso de 140 latidos por
minuto y frecuencia respiratoria de 32 respiraciones por minuto. Presenta disminución del
tono muscular,sin respuesta a los estímulos visuales o auditivos. La fontanela anterior se
presenta abombada. Mucosas adecuadamente hidratadas y ausencia de exantemas. Los
estudios de laboratorio revelan leucocitosis.
76a. Se realiza una punción lumbar,en la que se observa un bacilo grampositivo. ¿Cuál es
el agente causal más probable?
b} Listeria monocytogenes

76b. ¿Cuál es el tratamiento más adecuado según el organismo causal?


a} Ampicilina y gentamicina

76c. ¿Cuál es la forma más común de infección del organismo causal del cuadro de
meningitis?
a}Ingestión de productos lácteos no pasteurizados

77. Paciente masculino de 67 años que acude a consulta por fiebre y disnea. Refiere que
inició hace dos días, con fiebre y tos, pero actualmente tiene dificultad para respirar.
También cuenta que expectora flema amarillenta. Cuenta con tabaquismo de 24
paquetes/año e ingerir tres cervezas diarias. Niega antecedentes médicos de importancia.
El único medicamento que toma es aspirina. A la exploración física sus signos vitales son
FC 101/min, FR 21/min,T 38.6 C, PA 133/80 mm Hg. A la exploración física se auscultan
estertores crepitantes. En una placa de tórax se muestra una consolidación en el lóbulo
medio del pulmón derecho. ¿Cuál es la etiología más común de este padecimiento en
pacientes de este grupo de edad?
d} Streptococcus pneumoniae

78. Paciente femenino de 73 años que acude a urgencias por cefalea. Inició hace unas
horas, pero ha ido aumentando en intensidad y no mejoró con dos tabletas de
paracetamol de 500 mg. El dolor es unilateral,y cuando señala donde coloca su mano
frente a su ojo del lado derecho. También refiere algo de visión borrosa y náusea. Niega
que el dolor hubiera ocurrido antes de esa manera. Al IPAS niega vómito, fotofobia y
sonofobia. Al examen de agudeza visual no puede identificar números ni letras, sólo
movimientos de la mano. Se observa inyección conjuntiva! ocular y una pupila dilatada, no
reactiva a la luz. Sus signos vitales incluyen FC 103/min, FR 18/min, T 37.1 C, PA 134/79
mm Hg. El resto de los pares craneales, incluyendo los movimientos oculares, está intacto.
No hay anormalidades en el resto de la exploración neurológica. ¿Cuál sería el siguiente
paso en el manejo de este paciente?
d} Medición de la presión intraocular

79. ¿Cuántos criterios de Duke se necesitan para hacer el diagnóstico de endocarditis


infecciosa?
b} Uno mayor y tres menores

80. Lo siguiente es FALSO acerca de la infección por Taenia solium:


e} El método más común de adquirir cisticercosis es al ingerir carne de cerdo mal cocida,
con cisticercos
81. ¿Cuál es la fracción de eyección cuando un paciente presenta una frecuencia cardiaca
de 70 latidos/min, volumen al final de la diástole de 220 ml, y volumen al final de la sístole
de 120 ml?
a} 45%

82. Paciente femenino de 7 meses de edad es traída a consulta. La madre refiere que ha
presentado cuadros frecuentes de vómito, movimientos no coordinados y crisis
convulsivas; menciona que el cabello se le ha vuelto más claro en comparación al resto de
sus hermanos. En la exploración física se identifica piel seborreica, microcefalia, maxilar
prominente, dientes espaciados y retraso en el crecimiento.
82a. De acuerdo con las características clínicas, ¿cuál es el trastorno más probable de esta
paciente?
d} Fenilcetonuria

82b. ¿Con cuál de los siguientes exámenes de laboratorio confirmaría el diagnóstico?


a} Niveles de fenilalanina en plasma

82c. Con base en su sospecha clínica, ¿qué tratamiento indicaría?


c} Dieta libre de fenilalanina y cofactor tetrahidrobiopterina

82d. ¿Qué tipo de transmisión tiene este padecimiento?


b} Rasgo autosómico recesivo

83. Una abuela trae a consulta a su nieto por exantema. Su preocupación es que observa
que su nieto de 3 años amaneció con las mejillas muy rojas esta mañana y que sus brazos
y piernas parecen estar rojos también, además de que lo siente ""caliente"". A la
exploración física usted reconoce de inmediato el signo del ""niño abofeteado"" con el
exantema característico.
83a. Con base en las características del paciente, ¿cuál es el diagnóstico más probable?
b} Eritema infeccioso

83b. De acuerdo con su sospecha clínica, ¿cuál es el agente etiológico del padecimiento?
a} Parvovirus 819

83c. En relacióncon su sospecha clínica, ¿qué complicación es factible que se presente?


c} Crisis aplásica

83d. De acuerdo con su sospecha clínica, ¿cuáles son los hallazgos que esperaría encontrar
en los exámenes complementarios?
b} Leucopenia inicial seguida de leucocitosis y linfocitosis

83e. Con base en su sospecha clínica, ¿cuál es el tratamiento indicado para este paciente?
d} Paracetamol

84. Se encuentra realizando la exploración física inicial de un neonato de 12 horas de vida,


producto de primera gestación, peso al nacer 3 250g. Explora intencionadamente la
cadera para descartar la presencia de una luxación congénita.
84a. ¿Cuál de las siguientes NO es una característica de esta enfermedad?
e} Es más frecuente en varones

84b. ¿Cuál de los siguientes hallazgos en el examen físico del neonato le hacen sospechar
luxación congénita de cadera?
d} Chasquido al abducirla cadera y presionar eltrocánter mayor hacia una posición
anterior,conla cadera y rodilla en flexión

84c. ¿Qué información aporta la prueba de Barlow?


b} Detecta una cadera que esluxable al momento del examen físico

84d. Ante pruebas positivas de Barlow y Ortolani, usted decide repetir la exploración a las
seis semanas de vida y a los seis meses. Para la evaluación a los sies meses de vida, ¿qué
método diagnóstico es de elección para diagnosticar la displasia congénita de cadera?
e} Radiografía simple

84e. El paciente acude a consulta a los cinco meses de vida y se confirma el diagnóstico.
¿Cuál es la opción más adecuada de manejo de acuerdo con la edad del paciente?
a} Arnés de Pavlik

85. ¿Cuál de los siguientes anticonvulsivantes produce sedación e inducción del citocromo
P450?
e} Fenobarbital

86. Paciente femenino de 65 años de edad con larga historia de fibrilación auricular (sin
tratamiento), se presenta en Urgencias con antecedente de inicio súbito de dolor
abdominal intenso. A continuación del inicio del dolor ella vomitó y padeció movimientos
intestinales prolongados, no ha expulsado flatulencias desde esa ocasión.La exploración
física revela abdomen distendido moderadamente, el cual se distiende con delicadeza
aunque hay ausencia de signos peritoneales. Hace 15 años fue sometida a histerectomía
abdominal.
El diagnóstico más probable en esta paciente podría ser:
b} Isquemia mesentérica embólica aguda

87. ¿Cuál de los siguientes fármacos puede producir neuropatía periférica?


e} Paclitaxel
88. Acude a consulta de urgencias un paciente del sexo femenino de 38 años de edad sin
antecedentes de importancia. La paciente refiere que tuvo una riña con uno de sus
hermanos; posteriormente cuando trató de salir de inmediato de la casa, ella fue incapaz
de abrir la puerta por un ""ataque"" de debilidad en el brazo derecho. A la exploración
física la paciente presenta una fuerza muscular en el brazo derecho 0/5 sin alteraciones de
los reflejos de estiramiento en la misma extremidad. ¿Cuál es el diagnóstico más probable
en esta paciente?"
a} Trastorno conversivo

89. ¿Cuál de las siguientes neoplasias del sistema nervioso central en pacientes
pediátricos se asocia con el síndrome de von Hippel-Lindau?
b} Hemangioblastoma

90. Paciente masculino de 35 años acude al servicio de medicina preventiva por presentar
dermatosis diseminada en codos y rodillas con placas eritemo-escamosas.
El signo clínico que corrobora el diagnóstico de este paciente es:
e} Signo del rocío sangrante

91. ¿Cuál de los siguientes marcadores tumorales se eleva en el carcinoma colorrectal?


b} Antígeno carcinoembrionario

92. Se presenta una paciente de 22 años de edad con su primer embarazo. La paciente se
muestra renuente a abstenerse del consumo de alcohol durante el resto de su embarazo.
En el caso de una mujer embarazada que bebe entre 60 y 120 mlde alcohol a 80%, ¿cuál
es el riesgo de presentar un neonato con síndrome alcohólico fetal?"
b} 10 %

93. ¿Cuál de las siguientes translocaciones cromosómicas se asocia con el linfoma


folicular?
d} t(14:18}

94. Masculino de 56 años que llega al servicio de urgencias con dolor torácico de más de
12 horas de evolución,que inicia en forma súbita, mejora al estar sentado, no tiene
antecedentes de importancia. La acción principal en este paciente es descartar:
e} Cardiopatía isquémica

95. Paciente masculino de 30 años de edad acude por poliuria y polidipsia. El paciente
menciona que recientemente ha notado un incremento en el tono de la pigmentación de
su piel a pesar de que no se ha expuesto más tiempo del normal a la radiación solar.
Expresa que por las mañanas presenta edema de las extremidades inferiores y necesita
dormir con dos o tres almohadas para evitar la sensación de disnea nocturna. ¿Cuál es el
diagnóstico más probable en este paciente?
d} Hemocromatosis
96. Paciente de 22 años de edad sin antecedentes heredofamiliares de importancia. El
paciente se presenta con cuadro clínico caracterizado por lateropulsión, vértigo, cefalea,
hipoacusia bilateral y catarata. El estudio audiométrico reportó hipoacusia superficial
grave bilateral. Los potenciales evocados auditivos mostraron disrupción de la vía auditiva
a nivel del tallo cerebral. En la resonancia magnética de cráneo contrastada con gadolinio,
se observaron lesiones tumorales extraaxiales hiperintensas bilaterales a nivel del ángulo
pontocerebeloso, que afectan la emergencia de los complejos VINlll, del lado izquierdo
más prominente, y condicionan compresión del tallo cerebral,en especial del puente.
¿Cuál de los siguientes hallazgos es más probable que se encuentre en este paciente?
a} Nódulos de Lisch

97. Paciente masculino de 3 años y 6 meses de edad llega referido con los siguientes
antecedentes:hijo de primera gestación, sin control prenatal,se obtuvo por parto vaginal a
las 27 semanas de gestación y pesó 1400 g;presentó sangrado de tubo digestivo bajo,
trombocitopenia e infecciones severas a los pocos días de nacido;desde los 4 meses hasta
ahora ha sido hospitalizado en 54 ocasiones por sangrado de tubo digestivo y epistaxis
asociados con choque hemorrágico, gingivorragia, infecciones dentales e infecciones
respiratorias como otitis media crónica,sinusitis y neumonía. En los exámenes resaltan los
siguientes datos: trombocitopenia de 31000/µL, lgM baja, lgA normal e lgG elevada.
97a. Con base en las características del paciente, ¿cuál es el diagnóstico más probable?
d} Síndrome de Wiskott-Aldrich

97b. ¿Qué hallazgos apoyan su diagnóstico en los estudios complementarios?


c} Trombocitopenia,lgM baja,lgG e lgA elevadas"

97c. En relación con su sospecha clínica,¿cuál es la alteración genética asociada?


a} Proteína WASP

97d. Con base en su sospecha clínica, ¿cuál es el tratamiento que estaría indicado en este
paciente?
b} Trasplante de médula ósea

97e. En relación con su sospecha clínica, ¿cuál es la complicación que se podría presentar
en el paciente?
d} Muerte por hemorragias

98. ¿Cuál de las siguientes neoplasias malignas se asocia con el desarrollo del síndrome de
Lambert-Eaton?
e} Timoma
99. Masculino de 52 años que es derivado al servicio de dermatología por presentar
prurito en manos y antebrazos, eritema y esfacelación con exacerbación desde hace más
de 3 meses. Asimismo máculas en cuello yparte alta del tórax, agregándose edema facial.
Antecedentes de trabajar durante 15 años en fábrica de cementos. Exploración física:
placa eritematosa con hiperqueratosis en miembro pélvico izquierdo con huellas de
rascado y acompañado de edema.
¿Cuál es el diagnóstico más probable?
a} Dermatitis por contacto

1OO. Paciente femenino de 32 años que se presenta con alteraciones de la visión central
en el ojo derecho e incontinencia urinaria. Al interrogatorio menciona que hace algunos
años sintió debilidad en la pierna izquierda que cedió espontáneamente; tiempo después
presentó paresia en la extremidad superior derecha, la cual también cedió. ¿Cuál es el
diagnóstico más probable en esta paciente?

c} Esclerosis múltiple

101. ¿Cuál de las siguientes alteraciones electrolíticas se asocia con la aparición de la onda
de Osborn en el electrocardiograma?
b} Hipercalcemia

102. Se presenta a consulta externa una paciente de 25 años de edad en trabajo de parto.
La paciente refiere que nunca acudió a consulta de seguimiento obstétrico. El producto de
esta paciente es un neonato de bajo peso, con filtrum liso, microcefalia y fístula pulmonar.
¿Cuál de las siguientes condiciones es la que más probablemente ocasionó estos
defectos?
O a} Consumo de etanol durante elembarazo

103. Paciente masculino de 18 años de edad el cual es llevado al servicio de urgencias


después de que sus padres lo encontraron con pérdida del estado de alerta en el baño de
su casa. Al i ngreso presenta hiperglucemia en una muestra de glucosa capilar; la
gasometría demuestra un pH de 7.2 y el estudio cualitativo de orina presencia de cetonas.
A la exploración física el paciente presenta presión arterial de 100/60 mm Hg, frecuencia
cardiaca de 130 latidos/minuto, así como un estado grave de deshidratación. Los padres
mencionan que desde hace algunas semanas su hijo había estado perdiendo peso no
intencionadamente. Después de iniciar el tratamiento con hidratación, insulina,y manejo
de las alteraciones electrolíticas el paciente recupera el estado de alerta. Horas después se
presenta fiebre alta y cefalea retroocular. A la exploración de la cavidad nasal se
encuentra una escara necrótica negra adherida al cornete inferior. ¿Cuál de los siguientes
métodos diagnósticos es el mejor para confirmar el diagnóstico de este paciente?
c} Biopsia de mucosa
104. Masculino de 40 años acude a consulta externa refiriendo dolor en el primer ortejo
de pie derecho y ambas rodillas. Antecedentes: diabetes mellitus, por rama materna, y
litiasis renal e hipertensión arterial por su padre. Desde los 30 años se le diagnosticó
litiasis renal. Actualmente recibe tratamiento con esomeprazol y procinéticos desde hace
6 meses, indicados en el servicio de gastroenterología por padecimiento de ERGE.
104a. El padecimiento causal de la sintomatología de este paciente es:
e} Gota

104b. El examen de laboratorio indicado para confirmar el diagnóstico de este paciente


es:

O d} Cuantificación de ácido úrico en sangre y orina

104c. Medida terapéutica indicada en este paciente:

c} Etericoxib

105. ¿Cuál de las siguientes arterias es más susceptible de traumatismo durante la


colecistectomía?
b} La arteria cística

106. Mencione cuál de los siguientes factores está más asociado con el riesgo de padecer
artritis reumatoide:
e} Multifactorial

107. ¿Cuál de los siguientes comentarios referentes a los aneurismas de aorta abdominal
es correcto?
a} El seguimiento seriado con métodos deimagen está indicado en los pacientes
asintomáticos con aneurismas menores de 5 cm

108. ¿Cuál de las siguientes hormonas gastrointestinales estimula la secreción del factor
intrínseco?
d} Gastrina

109. ¿Cuáles son las entidades con las que se debe realizar diagnóstico diferencial del
paciente con artritis reumatoide?
e} Todas las opciones

11O. Paciente femenino de 34 años de edad programada para cesárea por desproporción
cefalopélvica. La paciente había sido diagnosticada con síndrome de Wolff-Parkinson-
White a la edad de 18 años, sin embargo se mantenía asintomática desde hace 16 años.
Previa a la anestesia epidural la paciente presentaba frecuencia cardiaca de 77 latidos/min
con un ritmo sinusal. Se inyecta 13 mlde mepivacaína a 2% en el espacio epidural a nivel
de T4. La cesárea se realiza sin eventualidades con una pérdida sanguínea de 450 ml. Al
final de la cesárea, la paciente presenta taquicardia supraventricular de inicio súbito,
incrementando la frecuencia cardiaca hasta 190 latidos/min con disminución de la presión
arterial. Se realizó masaje carotídeo sin conseguir estabilización. ¿Cuál de los siguientes
hallazgos NO está presente en la taquicardia recíproca ortodrómica auriculoventricular, es
decir, la taquicardia paroxística supraventricular que se observa en pacientes con el
síndrome de Wolff Parkinson-White?
b} Ondas delta

111. Paciente masculino de 65 años de edad, que se presenta al servicio de urgencias por
síntomas de resfriado, náusea, vómito, y cefalea frontal y retroocular. Entre los
antecedentes de importancia se encuentra sedentarismo, obesidad y tabaquismo. A la
exploración física el paciente se presenta alerta y orientada sin alteraciones motoras o
sensoriales; sin embargo, la paciente presenta una hemianopsia homónima derecha con
conservación de la visión. ¿Cuál de las siguientes arterias es la que más probablemente se
encuentra ocluida en este paciente?"
d} Arteria cerebral posterior izquierda

112. Debido a su considerable toxicidad y a la disponibilidad de agentes menos tóxicos (p.


ej., cefalosporinas y quinolonas), los aminoglucósidos se han estado utilizando menos
desde los últimos años. Estos medicamentos son generalmente utilizados para tratar
organismos gramnegativos, con o sin vancomicina. A pesar de que los aminoglucósidos
demuestran actividad in vitro contra la mayoría de las bacterias grampositivas, éstos
jamás deben de usarse aisladamente, debido a que existen alternativas menos tóxicas.
¿Cuál de los siguientes enunciados es el que tiene mejor relación con el mecanismo de
acción de los aminoglucósidos?
d} Inhibición de la síntesis de proteínas por medio de la inhibición de la subunidad 30S

113. Paciente femenino de 15 años de edad se presenta a consulta por presencia de


""bolita"" cerca de la rodilla derecha. En el interrogatorio los padres mencionan que
notaron la tumoración porque después de una clase de educación física su hija les
mencionó que le dolía su ""bolita"". Refieren que desconocían que la tuviera porque
nunca le había dado molestias, por lo que desconocen cuánto tiempo lleva con ella. En la
exploración física se nota tumoración en la parte distal del fémur derecho; no hay dolor a
la palpación ni otros síntomas asociados.
113a. Con base en las características de la paciente, ¿cuál es el diagnóstico más probable?
d} Osteocondroma

113b. En relación con su sospecha clínica,¿qué hallazgos esperaría encontraren una


radiografía?
c} Proyección ósea pedunculada

113c. En relación con su sospecha clínica, ¿cuál es la fisiopatología de este padecimiento?


a} Defecto en el cartílago de crecimiento
113d. De acuerdo con su sospecha clínica, ¿cuál es el tratamiento que recomendaría?
b} No se indica ningún tratamiento

113e. En relacióncon su sospecha clínica, ¿cuál es la complicación más importante que se


puede presentar?
d} No se presentan complicaciones

114. Paciente masculino, producto de primera gestación de 39 semanas de


duración,obtenido por histerotomía indicada por desproporción cefalopélvica. Se evalúa al
nacer y se encuentra con una frecuencia cardiaca de 98 latidos por minuto, respirando de
forma irregular y lenta, no llora, hay presencia de algunos movimientos de extremidades
con predominio de flexión, cuando se estimula con sonda nasogástrica se desencadena
llanto vigoroso y su piel se observa rosada en el tronco, pero los dedos de manos y pies se
encuentran cianóticos. De acuerdo con estas observaciones, ¿cuál es el puntaje de Apgar
que se le otorgaría?"
d} 6

115. Recién nacido masculino de 39 semanas de gestación con adecuado control prenatal
presenta cianosis severa que no mejora con oxígeno suplementario. En la exploración
física se aprecia cianosis generalizada, no hay dificultad respiratoria y a la auscultación del
tórax no se identifica soplo. La radiografía de tórax muestra mediastino estrecho y corazón
globular.
115a. Con base en las características del paciente, ¿cuál es el diagnóstico más probable?
c} Transposición de grandes vasos

115b. De acuerdo con su sospecha clínica, ¿con qué problema médico se ha asociado este
diagnóstico?
a} Síndrome de DiGeorge

11Sc. En relación con su sospecha clínica, ¿qué hallazgos esperaría encontrar en la


radiografía de tórax?
d} Corazón en huevo

115d. De acuerdo con su sospecha clínica, ¿cuál es el tratamiento indicado en este


paciente?
b} Conmutación arterial

115e.Se. En relación con su sospecha clínica, ¿cuál es la complicación que se puede


presentar a largo plazo?
c} Hipertensión pulmonar

116. Paciente femenino secundigesta, con embarazo normoevolutivo y trabajo de parto


sin alteraciones. La exploras y te percatas que el fondo uterino se encuentra a 35 cm de la
sínfisis del pubis y el producto se ubica en presentación pélvica, y al realizar el tacto
vaginal te das cuenta que está por completo borrado y dilatado y se encuentra en el tercer
plano de Hodge. No te da tiempo de pasarla a la sala de parto y se produce un parto
precipitado. Alumbramiento sin alteraciones, aunque percibes una hemorragia y un
desgarro que abarca el cuerpo perineal y el esfínter rectal. ¿Qué grado de desgarro
presenta?
c} Tercer grado

117. Paciente femenino de 30 años de edad que acude a consulta de control prenatal.
Tiene 22 semanas de gestación por fecha de última menstruación, sin antecedentes de
importancia y al interrogatorio no refiere datos de alarma. Toma ácido fálico y fumarato
ferroso. ¿Qué vacunas se le debe administrar para evitar la aparición de problemas
posteriores?
a} Vacuna Td

118. Paciente femenino de 25 años de edad con diagnóstico de embarazo de 32 semanas


de gestación acude a consulta de urgencias por presentar contracciones en abdomen de
50 segundos de duración, teniendo tres cada 1O minutos y con hemorragia transvaginal
de escasa cantidad, con salida de tapón mucoso; se encuentra pálida, diaforética,
taquicárdica, polipneica con presión arterial de 130/80. Usted diagnostica una amenza de
parto pretérmino ¿Qué datos lo orientarían hacia un diagnóstico diferencial?
c} Longitud cervical de 4 cm por ultrasonido

119. Se trata de un paciente masculino de 56 años de edad, con antecedentes de etilismo


a base de destilados y fermentados, cada fin de semana hasta llegar al estado de
embriaguez por hasta 2 o 3 días consecutivos, además tabaquismo a razón de 15 cigarros
al día desde los 16 años de edad (índice tabáquico: 30). Acude por presentar dos emesis
en ""posos de café"" hoy,además ha presentado durante 1 semana evacuaciones
melénicas 2 a 3 veces por día. A la exploración encuentra telangiectasias en el territorio de
la vena cava superior,eritema palmar, hipotrofia tenar e hipotenar con contractura de
Dupuytren de ambas manos. FC 95 lpm, FR 20 rpm,temp. 35.?ºC, TA 100/60 mm Hg, peso
89 kg, talla 1.72 m. 119a. Los antecedentes del paciente y los datos clínicos de
hematemesis en ""posos de café"" y melena, nos hacen pensar en una hemorragia
de tubo digestivo alto; basados en epidemiología, el diagnóstico más probable es:
b} Várices esofágicas

119b. El estudio de primera elección que ayuda a confirmar el diagnóstico es:


e} Esofagogastroduodenoscopia

119c. Para protocolizar al paciente, se deben hacer estudios bioquímicos que permitan
estratificar el grado de insuficiencia hepática según la clasificación de Child-Pugh, el
paciente cursa con ascitis moderada y sin encefalopatía. Con los siguientes laboratorios,
Biometría hemática: Hb 8.5, hto 26.5, Leu 13.5, Plt 125, Bilirrubina total 4.8 (BD 2.3, 81
2.5), Proteínas 5.2, Alb 3.0, Glo 2.2, INR 1.5 ¿Qué grado tiene este paciente?
d} Child-Turcotte-Pugh C
119d. Para disminuir la frecuencia de sangrados, usted selecciona manejo farmacológico
para largo plazo con:
c} Beta-bloqueadores no selectivos

119e. El tratamiento de elección, para pacientes de difícil control con recurrencia es:
b} Derivación portosistémica intrahepática transyugular (TIPS}

120. ¿Cuál de los siguientes factores constituyen los mayores determinantes en el


incremento de riesgo cardiovascular en los pacientes diabéticos?
a} Ateroesclerosis y neuropatía autonómica

121. Paciente masculino de 13 años de edad que acude a consulta por dolor escrotal que
ha incrementado progresivamente, así como edema de la región. Al interrogatorio el
paciente describe que ha padecido disuria recientemente. A la exploración física, el
escroto de este paciente se encuentra edematoso y eritematoso. A la palpación, se
produce dolor en la región posterolateral del testículo izquierdo. Cuando se realiza
elevación del testículo izquierdo, el dolor disminuye de intensidad. ¿Cuál de las siguientes
opciones representa el diagnóstico más probable en este paciente?
d} Epididimitis

122. ¿Cuál de las siguientes opciones representa la variante más frecuente de la


neuropatía diabética?
c} Polineuropatía simétrica distal

123. ¿Cuál es el tratamiento del efecto Somogyi?


b} Disminuirla dosis de insulina precena

124. Se trata de un paciente masculino de 34 años de edad, se encuentra en el 5° día de


hospitalización por pancreatitis severa, se le halla desorientado y disneico, a la exploración
física con taquipnea, uso de musculatura accesoria, acrocianosis, ambos hemitórax con
disminución del murmullo vesicular y estertores de predominio al final de la inspiración; la
saturación por oximetría de pulso está en 75% con puntas nasales a 3 Umin (Fi02 30%). La
gasometría más reciente muestra hipoxemia, con Pa02 de 45 kPa, por lo que requiere
inicio de la ventilación mecánica. FC 115 lpm, FR 28 rpm, T 35.6ºC, PA 90/50 mm Hg, peso
89 kg, talla 1.70 m."
124a. Con los datos del padecimiento actual y la exploración física, usted integra el
diagnóstico de:
d} Síndrome de insuficiencia respiratoria aguda

124b. Basado en los criterios de la definición de Berlín, publicada en 2012, podemos


clasificar este cuadro como:
e} SIRA moderado
124c. ¿Cuáles son las fases del SIRA?
b} Exudativa, proliferativa y fibrosa

124d. En el síndrome, estos son los principales mediadores proinflamatorios:


d} IL-6, IL-8 y TNF-a

124e. ¿Cómo se clasifica el SIRA según su etiología?


c} Pulmonar y extrapulmonar

125. La principal diferencia en la fisiopatogenia del estado hiperglucémico hiperosmolar


de la cetoacidosis diabética es:
c} La menor deficiencia de insulina

126. Paciente masculino de 62 años de edad con diagnóstico de diabetes mellitus tipo 2 de
13 años de evolución, controlada con hipoglucemiantes orales; última glucemia central
fue de hace un mes en 143 mg/dL. Acude a consulta de urgencias por presentar dolor
ocular intenso en ojo derecho, disminución de la agudeza visual de 4 horas de evolución,
además de náusea y vómito. A la exploración oftalmológica presenta una agudeza visual
de movimiento de manos en ojo derecho y 20/80 en ojo izquierdo. A la biomicroscopia se
encuentra en ojo derecho hiperemia conjuntiva! e inyección ciliar,córnea edematosa con
presencia de microbulas subepiteliales, reacción inflamatoria en cámara anterior, la cual
se observa poco profunda, pupila midriática, hiporrefléctica, cristalino con catarata.
Presión intraocular de 58 mm Hg. La gonioscopia muestra un ángulo completamente
cerrado, el fondo de ojo no es valorable por la opacidad de medios. Ojo izquierdo córnea
transparente, cámara anterior poco profunda, pupila fotorreactiva, iris sin alteraciones,
cristalino transparente y a la gonioscopia ángulos moderadamente abiertos. ¿Cuál es el
diagnóstico más probable?
c} Glaucoma agudo por bloqueo pupilar

127. ¿Cuál es la causa más probable del dolor de rodillas en corredores?


a} Síndrome patelofemoral

3d. ¿Cuál de los siguientes hallazgos en la exploración orienta más hacia el diagnóstico de
estenosis valvular aórtica?
b) Aumento de la intensidad del soplo al sentar al paciente e inclinar el tórax hacia
adelante

5. Paciente masculino de 18 años de edad. Acude a consultar por síndrome febril. Al


interrogatorio refiere artralgia en "ambas rodillas y codos de carácter migratorio. A la
exploración física se registra TA: 100/60 mm Hg, FC: 70 lpm, Temp: 38 ºC, articulaciones
con rango de movimiento normal, sin signos de inflamación; se ausculta un soplo leve
sistólico en ápex irradiado a axila. Se sospecha fiebre reumática."
5a. ¿Cuál es el microorganismo causante de esta enfermedad?
b) Streptococcus pyogenes
5e. ¿Cuál es la valvulopatía más frecuente asociada a la fiebre reumática?
b) Estenosis mitral

9e. ¿Cómo se diferencia un sangrado por defecto plaquetario de uno por defecto de la
coagulación?
c) En los defectos plaquetarios hay equimosis y petequias

11. Paciente masculino de 1O años de edad que es llevado al Servicio de Urgencias


posteriormente a que ingirió material cáustico. Se inician maniobras de reanimación y
estabilización. ¿Cuál de las siguientes opciones representa una complicación a largo plazo
que pueda sufrir este paciente?
d) Carcinoma esofágico

12. ¿Cuál de las siguientes opciones representa el tratamiento curativo más apropiado
para un paciente de 65 años de edad con bocio tóxico multinodular y síntomas
compresivos de la vía aérea?
e) Tiroidectomía

25. ¿Cuál de las siguientes bacterias es una causa de meningitis y epiglotitis en pacientes
pediátricos?
e) Haemophilus influenzae

41. ¿Cuál de las siguientes condiciones se relaciona con la presencia de cuerpos de


Howell-Jolly en el frotis de sangre periférica?
d) Esplenomegalia

50. ¿Cuál de los siguientes efectos adversos se relaciona principalmente con la


administración de didanósido?
a) Pancreatitis

"60. Neonato de tres días de edad, no ha logrado alimentarse del seno materno por
problemas con la succión. En el" examen físico se demostró presencia de paladar hendido
unilateral.
60a. ¿En qué momento está indicada la reparación quirúrgica de este defecto?
b) A los 1O meses de vida

60b. Desde las primeras cirugías de reparación de labio o paladar hendido se recomienda
también efectuar un procedimiento para evitar una de las principales complicaciones
secundarias al efecto anatómico. ¿De qué procedimiento se trata?
a) Colocación de tubos de timpanostomía

74. ¿Cuál de los siguientes tipos de catarata se asocia con debilidad muscular?
a) Catarata policromática múltiple (catarata en "árbol de navidad")
76. Se presenta al servicio de urgencias una paciente de 29 años de edad que se encuentra
en trabajo de parto. La "paciente no acudió a ninguna consulta de manejo prenatal.
Durante el trabajo de parto se rompen artificialmente las membranas y es evidente que
existía oligohidramnios. El recién nacido pesa 1.8 kg, y presenta dificultad respiratoria
significativa inmediatamente después de nacer. La radiografía de tórax evidencia la
presencia de pulmones 'hipoplásicos. ¿Cuál de los siguientes estudios es el más adecuado
de realizar en este paciente?"
b) Ultrasonido renal

82. Paciente femenino de 27 años de edad, que acude a consulta de planificación familiar.
Refiere paridad no satisfecha, no embarazada, sin datos de enfermedad pélvica
inflamatoria. Se decide colocarle un dispositivo intrauterino. Histerometría de 7 cm. Se
observa escéptica en el uso, ya que le comenta, que ella ha escuchado que es un abortivo.
¿Cuál es el mecanismo de acción del dispositivo intrauterino?
d) Crea una respuesta inflamatoria

85. Paciente masculino de 7 años de edad, previamente sano, acude a consulta por dolor
en testículo derecho. El dolor que tiene 1O h de evolución, comenzó de manera súbita la
noche previa sin antecedente traumático. Es de intensidad moderada y se ha mantenido
constante, aunque cede parcialmente con ibuprofeno. No hay irradiación, problemas de
micción, fiebre o vómito. En la exploración se observa piel eritematosa, dolor en el polo
superior del testículo, cierto aumento de volumen y reflejo cremastérico presente. No
tiene episodios previos similares.
85a. Con base en las características del paciente, ¿cuál es su sospecha diagnóstica?
b) Torsión del apéndice testicular

85c. Usted decide realizar una ecografía doppler. De acuerdo con su sospecha clínica,
¿qué hallazgos espera encontrar?
b) Flujo arterial aumentando

85d. Si los hallazgos dela exploración física y la ecografía concuerdan con su sospecha
clínica, ¿cuál es el tratamiento" indicado?
a) Sintomático con antiinflamatorios y reposo

92b. Ante un paciente de estas características, edad, género, escolaridad, origen y cultura,
el médico debe considerar:
a) Informar en forma detallada los procedimientos y resultados y aceptar las
decisiones delos pacientes

94. ¿Cuál de las siguientes vasculitis tiene el mejor pronóstico de vida?


a) Púrpura de Henoch-Schonlein
102. Acude a consulta externa un hombre de 28 años de edad por presentar desde hace
15 días fatiga, malestar general y disnea. El paciente es originario de un área rural, y por el
momento se encuentra laborando como constructor; ha estado viviendo debajo de un
puente. Al interrogatorio el paciente refiere que sus síntomas se han acompañado de tos
sin flema, la cual en ocasiones lo lleva al vómito; también refiere anorexia en los últimos
días. El paciente niega cualquier pérdida de peso o sudoración nocturna, sin embargo,
menciona que es posible que haya sufrido de fiebre (subjetiva, no cuantificada). A la edad
de 26 años, un médico le diagnosticó síndrome de inmunodeficiencia humana con un
conteo de linfocitos CD4+ de 134 (el paciente trae reporte con los resultados).
Actualmente no se encuentra consumiendo medicamentos y confiesa ser adicto al uso de
cocaína y etanol. A la exploración física se encuentra una presión arterial de 120/75 mm
Hg, taquicardia de 95 latidos/minuto; la saturación de oxígeno se encuentra reducida; la
Pa02 es de 65 mm Hg. La inspección general muestra un paciente con dificultad
respiratoria y hallazgos auscultatorios de neumonía. La radiografía torácica demuestra un
infiltrado difuso bilateral, con predominio basal.
Utilizando únicamente la información previamente descrita, responda las siguientes
preguntas:
102a. ¿Cuál de los siguientes agentes patógenos es el que con mayor probabilidad está
casionando los síntomas de este paciente?
a) Pneumocystis jiroveci

104. Paciente de 18 años de edad que acude a consulta por malestar general de dos
semanas y malestar faríngeo. A la exploración física se revela faringitis,
hepatoesplenomegalia y linfadenopatías axilares. Los resultados de estudios de
laboratorio son: hemoglobina 13 g/dL, hematocrito 43%, volumen corpuscular medio 95
fL, conteo plaquetario 300 000 plaquetas/mm3 y leucocitos de 9 000 células/mm3. El
frotis de sangre periférica demuestra linfocitos atípicos.
¿Cuál fue el principal origen por el que esta paciente adquirió esta enfermedad?
d) Por su novio
105. Paciente de sexo femenino de 18 meses de edad es llevada al departamento de
urgencias por alteraciones mentales y retraso mental progresivo. A la exploración física se
encuentra hepatoesplenomegalia, linfadenopatías, sin alteraciones a la exploración del
fondo de ojo. Se realiza biopsia hepática, esplénica y de ganglios linfáticos, los cuales
demuestran macrófagos aumentados de tamaño con lisosomas alargados y distendidos.
¿Cuál es la deficiencia enzimática que presenta esta paciente?
b) Glucocerebrosidasa

113. ¿Cuál de las siguientes neoplasias del sistema nervioso central en adultos se localiza
principalmente en las convexidades de los hemisferios cerebrales?
b) Meningioma

115. Paciente femenino de 12 años de edad que acude a consulta externa de pediatría. La
madre menciona que la niña ha estado cojeando desde hace varias semanas y refiere
dolor intenso en la rodilla izquierda. Al interrogatorio la madre dice que la paciente ha
estado afebril, no recuerda golpes o traumatismos dirigidos a las extremidades, ni
antecedente de enfermedades recientes. A la exploración física es evidente que la
paciente no puede rotar internamente la pierna afectada. ¿Cuál de los siguientes
diagnósticos es el más probable en esta paciente?"
a) Epifisiólisis de cabeza femoral

3e. ¿Cuál de las siguientes afirmaciones es correcta?:


b) El fenómeno dela disociación acústica, que consiste en una mayor propagación del
soplo sistólico aórtico al ápex que al cuello con carácter musical, es característico dela
estenosis aórtica.

4. Son ototóxícos no farmacológicos:


c) Mercurio y plomo

6e. Tratamiento indicado en este paciente:


d) AINE
7b. ¿Qué se observa en el frotis de sangre periférica?
a) Linfocitos atípicos

13. ¿Cuál de los siguientes medicamentos se caracteriza por presentar miocardiopatía


dilatada como complicación?
b) Doxorrubicina

17c. ¿Qué estudio serviría para corroborar el origen de la enfermedad?


c) Antígenos urinarios

21c. ¿En cuál de los siguientes genes esperaría encontrar una mutación?
c) KRAS

26. ¿Cuál de las siguientes opciones representa la causa más frecuente de derrame
pleural?
e) Insuficiencia cardiaca

28. Paciente masculino de 12 años de edad que acude a consulta con su madre debido a
que presenta sacudidas súbitas periódicas. Al parecer las contracciones musculares
ocurren en las mañanas antes de irse a la escuela, y se ven aumentadas cuando se
desvela. El paciente no pierde el estado de alerta durante estos episodios. Al
interrogatorio, la madre comenta que un hermano de ella presenta episodios similares.
¿Cuál es el fármaco de elección en este paciente?
e) Ácido valproico

40. Se solicita interconsulta por el personal de psiquiatría de manejo de adicciones al


servicio de Cardiología. Se trata de un paciente del sexo masculino de 51 años de edad con
antecedente de alcoholismo intenso desde los 15 años. A pesar de que ha tenido que ser
hospitalizado en varias ocasiones por intoxicación alcohólica, éste sólo ha disminuido un
poco el consumo. El paciente presenta disnea paroxística nocturna. A la exploración física
es posible detectar un tercer ruido cardiaco así como un soplo diastólico; el paciente
también presenta estertores pulmonares y edema periférico. Se solicita un
ecocardiograma. ¿Cuál de los siguientes hallazgos es el más probablemente esperado en
el ecocardiograma?
b) Dilatación ventricular izquierda y derecha con disminución de la contracción en todo el
miocardio

48. ¿Cuál de los siguientes efectos adversos corresponde a la administración de


ciclofosfamida?
b) Disuria y urgencia urinaria

61. Un paciente de un mes de edad acude al consultorio para consulta rutinaria. La madre
expresa preocupación porque ha notado una "tumoración" en la región escrotal derecha.
A la exploración física se encuentra una tumoración en la región escrotal de consistencia
elástica que impide la palpación del testículo y presenta translucidez; es posible localizar
mediante palpación el cordón espermático en la parte superior de esta tumoración. Al
parecer la tumoración no es dolorosa a la palpación. El paciente fue resultado de un parto
prematuro. ¿Cuál es el diagnóstico más probable?
a) Hidrocele

64. Paciente de 54 años de edad, recientemente deportado de la zona Este de Estados


Unidos. Acude a consultar por fiebre de varios días de evolución, mialgia, astenia,
adinamia y artralgia migratoria. TA: 120/80, FC: 85 lpm. Se observa en el muslo eritema
circular de 1O cm de diámetro con aclaramiento parcial, borde externo rojo y centro en
diana; el paciente refiere que inició como una pápula roja que se extendió poco a poco
hasta formar la lesión anular actual.
64a. Según las características clínicas, ¿cuál es el diagnóstico más probable?
b) Enfermedad de Lyme

64e. En caso que el paciente presente afectación neurológica por la enfermedad, ¿cuál es
el tratamiento recomendado?
a) Ceftriaxona

70. Femenino de 25 años acude a consulta externa refiriendo tenesmo vesical, disuria y
escalofrío. Antecedente: cursa su 14ª semana de gestación,
La medida terapéutica de primera intención en esta paciente es:
a) Ampicilina

77. Paciente masculino de 70 años de edad con diagnóstico de diabetes mellitus tipo 2
desde hace 25 años, como complicaciones crónicas presenta retinopatía, neuropatía y
nefropatía incipiente. No existe evidencia de aterosclerosis. En este paciente debemos
alcanzar las siguientes metas, con el fin de disminuir la progresión del daño renal:
b) Presión arterial< 140/90 mm Hg, HbA1c < 7% y sin metas LDL específicas

83. Paciente masculino de ocho días de vida extrauterina, con antecedente de haber sido
producto de G1 P1, con embarazo normoevolutivo, de 38 SDG, con rotura prematura de
membranas 5 horas previas al inicio del trabajo de parto, obtenido por parto eutócico y
Apgar 8/9. Acude a revisión oftalmológica porque la madre refiere secreción oftálmica
amarillenta e hiperemia conjuntiva! intensa en ambos ojos de dos días de evolución,
asociado a febrícula e irritabilidad. A la exploración se observa secreción amarillenta en
moderada cantidad, quemosis conjuntiva!, córnea clara intacta y el resto de la exploración
oftalmológica, sin alteraciones.
83a. El diagnóstico más probable del caso clínico previo es:
c) Oftalmia neonatorum

85. Paciente masculino de 7 años de edad, previamente sano, acude a consulta por dolor
en testículo derecho. El dolor, que tiene 1O h de evolución, comenzó de manera súbita la
noche previa sin antecedente traumático. Es de intensidad moderada y se ha mantenido
constante, aunque cede parcialmente con ibuprofeno. No hay irradiación, problemas de
micción, fiebre o vómito. En la exploración se observa piel eritematosa, dolor en el polo
superior del testículo, cierto aumento de volumen y reflejo cremastérico presente. No
tiene episodios previos similares.
85e. En conclusión, ¿cuál de las siguientes opciones mencionalas características que se
presentan en el padecimiento de este paciente?
b) Edad prepúberes, dolor leve a moderado, posición normal deltestículo, reflejo
cremastérico presente, signo del punto azul.

93. Niño de cuatro años de edad con antecedente de infección de vías respiratorias
superiores hace tres días que recibió sólo manejo sintomático con paracetamol
(acetaminofén). El día de ayer inició con anorexia, náusea y vómito. A partir de hoy se
queja de dolor abdominal intenso en hipocondrio derecho. En la exploración se encuentra
pálido, diaforético, con mucosa oral seca, dolor a la palpación en cuadrante superior
derecho del abdomen y hepatomegalia.
Los estudios de laboratorio reportaron hemoglobina de 13, hematocrito de 45%,
leucocitos 9 700, plaquetas 225 000, sodio 140, potasio 3.8, creatinina 0.7, AST 120, ALT
135 y bilirrubina total 1.7.
93a. En el contexto de la intoxicación que sufre el paciente, ¿qué metabolito es
responsable del cuadro clínico?
b) N-acetil-p-benzoquinoneimina

102b. ¿Cuál de las siguientes opciones es la mejor conducta terapéutica en este paciente?
d) Administración de trimetoprim-sulfametoxazol y prednisona
108. Paciente femenino de 19 años de edad que se presenta a consulta por varias
molestias. La paciente menciona que debido a su sobrepeso decidió iniciar una dieta,
consumo de suplementos alimenticios y ejercicio. La paciente menciona que su
padecimiento actual inició hace aproximadamente 20 días con cefalea, fatiga, dolores
óseo y articular; sin embargo, al interrogatorio se reveló que la paciente ya presentaba
disminución del apetito y cabello quebradizo desde hace un mes. A la exploración física la
paciente presenta labios secos y con fisuras, sin ninguna otra anormalidad evidente. Los
estudios de laboratorio revelan incremento de las enzimas hepáticas e hiperlipidemia.
¿Cuál de las siguientes vitaminas se asocia con el cuadro clínico que presenta esta
paciente?
a) Vitamina A

2. La presencia de neumoconiosis, nódulos en el parénquima pulmonar y artritis


reumatoide, se conoce con el nombre de:
c) Síndrome de Caplan

3. Paciente masculino de 75 años de edad acude a consulta tras haber sufrido un síncope.
Al interrogatorio menciona que en las últimas semanas ha tenido dificultad para respirar
al llevar a cabo actividades que antes efectuaba sin problemas, y que ello va acompañado
de dolor precordial que cede con el reposo. Es portador de un stent coronario. Entre sus
antecedentes está el hecho de que fumó hasta hace 1O años, padece de hipertensión y es
dislipidémico.
Sus medicamentos incluyen aspirina, ramipril, simvastatina, metoprolol. En la exploración
física se registra TA: 150/90 mm Hg; FC: 60 lpm; FR: 20; Temp: 36.5 ºC; pulso rítmico y
débil;un soplo sistólico romboidal con intensidad 2/6, que predomina en el 2do espacio
intercostal paraesternal derecho; S4 presente; enzimas cardiacas normales. ECG: ritmo
sinusal, sin datos de isquemia.
3a. La causa más probable del síncope en este paciente es:
c) Estenosis valvular aórtica

3c. Acerca del tratamiento en este paciente:


a) Referir a cirugía para reemplazo valvular

6. Paciente masculino de 45 años de edad, con antecedente de diarrea infecciosa hace 2


semanas que remitió sin complicaciones, presenta de forma aguda artralgia intensa en
rodilla derecha, y transcurridas unas horas inicia con dolor lumbar, talalgia y dolor en
articulaciones metatarsofalángicas e interfalángicas del pie izquierdo, así como con dolor
monoocular y diarrea. A la exploración física se registra TA: 120/80, FC: 90 lpm, Temp: 38
ºC, dolor a la compresión de la pelvis y a la palpación de articulación sacroilíaca, rodilla
derecha con signos de inflamación, dactilitis en 2do dedo de pie izquierdo, ojo izquierdo
con hiperemia ciliar y lagrimeo.
6a. Señale el diagnóstico compatible con la clínica del paciente:
d) Artritis reactiva
7c. ¿Cuál de las siguientes opciones sirve para confirmar el diagnóstico?
a) Anticuerpos heterófilos

7e. ¿Qué complicación mortal puede presentar la paciente?


a) Rotura esplénica

8. Un paciente masculino de 23 años de edad acude al servicio de urgencias después de


haber mantenido una relación homosexual con penetración anal con un hombre con
infección por el virus de la inmunodeficiencia humana. Durante su visita al servicio de
urgencias se obtiene una muestra de sangre, la cual resulta negativa para anticuerpos
contra el virus de la inmunodeficiencia humana. ¿Cuándo es el menor tiempo posible en
que debe citarse de nuevo al paciente para detectar seroconversión en caso de que éste
se encuentre infectado?
c) Seis a 12 semanas

9. Paciente femenino de 74 años ha estado hospitalizada 5 días por una cirugía de prótesis
de cadera que se complicó. Recibe heparina no fraccionada para prevenir trombosis por la
cirugía y el reposo prolongado, además de utilizar medias compresivas. Refiere que tiene
dolor en la pantorrilla izquierda, y a la exploración física presenta signos de trombosis
venosa profunda, lo cual se confirma al realizarle un ultrasonido. También se le hace una
biometría hemática, de cuyos resultados llaman la atención las plaquetas de 56
OOO/mm3.
9a. ¿Cuál es el paso a seguir con esta paciente?
b) Suspender la heparina

9b. ¿A partir de cuántas plaquetas se tiene el riesgo de sangrado espontáneo?


c) menor a 20 000

9c. ¿Cuál es el mecanismo de acción de la heparina no fraccionada y cuál es su principal


efecto secundario?
b) Acelera la acción dela antitrombina III y provoca trombocitopenia

14. Paciente femenino de 27 años, primigesta, con embarazo de 32 semanas de gestación.


El embarazo no ha tenido complicaciones. Acude a consulta por presentar un dolor
precordial agudo que se exacerba con la inspiración profunda,así como por falta de aire de
una hora de evolución. Niega dolor en pantorrillas. Sus signos vitales son Temp: 37.9 ºC,
TA: 130/80, FC: 110, FR: 24 y satura a 90%. A la EF se registran estertores basales derechos
y signo de Homans negativo bilateral. Se toma una radiografía de tórax, la cual es
normal;en el análisis de sangre hay dímero D elevado, y en el electrocardiograma se
muestra taquicardia sinusal con desviación del eje a la derecha.
14a. ¿Cuál es el siguiente paso diagnóstico?
d) Gammagrama ventilación/perfusión

14b. ¿Cuál es la tríada de Virchow?


b) Trauma vascular, estasis venosa e hipercoagulabilidad

14c. ¿Cuál de los siguientes no es un criterio modificado de Wells?


d) Electrocardiograma con S2Q3T3

14d. ¿Cuál es la alteración más frecuente que se espera encontrar en el


electrocardiograma?
b) Taquicardia sinusal

15. ¿Cuál de las siguientes opciones corresponde a una indicación para iniciar tratamiento
renoprotector con inhibidores de la enzima convertidora de angiotensina en pacientes
diabéticos?
a) Presencia de microalbuminuria

19. Paciente femenino de 8 años es traída por sus padres a consulta, pues les comentan
en la escuela que la niña no pone atención en clase y pierde la concentración muy
fácilmente. Los padres la han estado observando y se han dado cuenta de que durante el
día tiene varios episodios, que duran entre 5 y 1O s aprox., en los que “se pierde en el
espacio" y sus ojos hacen cierto movimiento. La exploración física es normal.
19a. ¿Cuál es el tratamiento de primera elección para este padecimiento?
b) Etosuximida

19b. ¿Cuál de los siguientes es un efecto secundario de la carbamazepina?:


b) Anemia aplásica

19c. ¿Cuál de los siguientes medicamentos causa hiperplasia gingival?:


d) Fenitoína

19d. ¿Cuál es el primer tratamiento de urgencia en el status epílectícus?


b) Diazepam

20. Paciente masculino de 15 años de edad, refiere traumatismo nasal de 4 h de evolución


al caerse de su propia altura. Presentó epistaxis autolimitada. A la exploración física se
observa edema leve en dorso nasal, sin crepitación de huesos propios; rinoscopia con
septum funcionaly costras hemáticas. La siguiente conducta sería:
b) Indicar antiinflamatorios y hielo local

21. Paciente masculino de 79 años con historia de alcoholismo, pancreatitis crónica


(última exacerbación hace un año) e hipertensión (bajo control con bloqueador beta),
acude a consulta acompañado de su hija, quien comenta que ha perdido mucho peso en
los últimos 3 meses, aprox. 1O kg, aparte nota sus ojos amarillos y se queja de prurito y
dolor abdominal.
21a. ¿Qué esperaría encontrar en la exploración física abdominal que le ayude con el
diagnóstico?
e) Signo de Courvoisier

21e. Los resultados de imagen del paciente muestran un tumor localmente avanzado e
irresecable, ¿cuál es su pronóstico?
b) Una media de supervivencia de 8 a 12 meses

22b. El documento escrito firmado por el paciente, su representante legal o el familiar


más cercano en vínculo, mediante el cual acepta un procedimiento quirúrgico con fines
diagnósticos o terapéuticos una vez que ha recibido la información de los riesgos y
beneficios para el paciente, es:
d) La carta de consentimiento informado que puede ser preelaborada

23. ¿Cuál de las siguientes opciones es la más adecuada en un paciente con antecedente
de sinusitis estacional y con sintomatología típica de sinusitis aguda de tres días de inicio?
d) Fluticasona nasal

24. Paciente masculino de 55 años de edad con fiebre intensa, pérdida de peso,
infecciones oportunistas, ausencia de adenomegalias, pancitopenia y esplenomegalia.
¿Cuál es el diagnóstico más probable en este paciente?
b) Leucemia de células vellosas

31. ¿Cuál de los siguientes virus que ocasionan hepatitis provoca alta mortalidad en
pacientes embarazadas?
e) Virus de la hepatitis E

33. Paciente femenino de 35 años de edad la cual inicia tratamiento con altas dosis de
prednisona por diagnóstico de nefritis lúpica grave. ¿Cuál de los siguientes cambios en la
biometría hemática son resultado del tratamiento con glucocorticoides?
a) Incremento en el conteo de neutrófilos en sangre

35. ¿Cuál de los siguientes resultados de laboratorio es el que se esperaría encontrar en


un paciente dislipidémico en caso de que éste tenga adecuado apego y respuesta a la
colestiramina?
d) Disminución del colesterol LDL, sin observar efecto significativo en elnivel de colesterol-
HDL o triglicéridos

36. Paciente masculino de 29 años de edad que se presenta a consulta por haberse
detectado una tumoración en el lado derecho de su cuello. Al interrogatorio el paciente se
muestra preocupado pues no intencionadamente ha perdido alrededor de 8.5 kg en el
último mes; entre otros hallazgos realizados durante el interrogatorio se descubre que el
paciente ha padecido sudación nocturna. Debido al cuadro clínico tan sugestivo, se decide
realizar una biopsia de la adenomegalia cervical, la cual revela presencia de células de
Reed-Sternberg. Durante el resto de la evaluación se descubre que el involucro ganglionar
se limita a la zona del cuello y la axila. ¿Cuál es el estadio en el que se encuentra este
paciente?
d) IIB

44. Paciente masculino de 71 años de edad que trabajó para la industria de la


construcción durante muchos años se presenta a una revisión médica de rutina por
insistencia de uno de sus hijos. A la exploración física el paciente parece sano y en buen
estado. Sin embargo, durante el interrogatorio claramente menciona que tuvo exposición
crónica al asbesto. ¿Cuál de las siguientes neoplasias malignas se encuentra con mayor
riesgo de desarrollar este paciente?
a) Carcinoma broncogénico

46. Paciente femenino de 45 años de edad que inicia tratamiento para artritis reumatoide.
Al parecer el medicamento ha resultado efectivo ya que la paciente presenta disminución
de la artritis; sin embargo, acude de nuevo al centro hospitalario por ataques de tos con
hemoptisis. Se realiza una tinción de esputo la cual revela bacilos ácido alcohol
resistentes. ¿Cuál de la siguiente lista de fármacos es el que más probablemente se le
administró a la paciente para su tratamiento de artritis reumatoide?
a) lnfliximab

51. Paciente masculino de 77 años de edad con hiperplasia prostática benigna, la cual le
ocasionó insuficiencia renal aguda posrenal. En este momento su función renal se ha
estabilizado; los resultados de los estudios de laboratorio más recientes en sangre son los
siguientes: sodio 138 mEq/L, potasio 5.9 mEq/L, cloro 108 mEq/L, bicarbonato 18 mEq/L,
nitrógeno ureico en sangre 28 mg/dl, creatinina 2.3 mg/dl, pH 7.3, PaC02 35 mm Hg, Pa02
82 mm Hg."
¿Cuál de los siguientes enunciados es el más correcto respecto a la condición de este
paciente?
c) Hay disminución en la producción o el efecto de la aldosterona

52. ¿Cuál de los siguientes anticuerpos es más probable que se presente en pacientes con
hemorragia alveolar y glomerulonefritis?
e) Anticuerppos contra la membrana basal glomerular

53. ¿Cuál de los siguientes antiarrítmicos tiene gran afinidad por los canales del sodio en
estados activo e inactivado, pero no en reposo, además de presentar rápida disociación de
los canales del sodio que no se encuentran en reposo?
d) Lidocaína

54. Paciente masculino de 29 años de edad que se presenta a consulta de rutina. Parece
sano, sólo reporta calambres ocasionales en las piernas. El paciente no tiene antecedentes
de importancia, no se encuentra tomando medicamentos, no fuma, no es usuario de
drogas, y ocasionalmente consume bebidas alcohólicas. A la exploración física se
encuentra una presión arterial de 120/75 mm Hg, frecuencia cardiaca de 65
latidos/minuto. Los estudios de laboratorio en sangre son los siguientes: sodio 138 mEq/L,
potasio 3 mEq/L, bicarbonato 34 mEq/L,cloro 91 mEq/L, nitrógeno ureico 14
mg/dL,creatinina 1 mg/dL,glucosa 88 mg/dL. En orina los niveles de cloro se encuentran
elevados y el calcio urinario está disminuido. ¿Cuál de los siguientes diagnósticos es el más
probable en este paciente?
c) Síndrome de Gitelman

57. ¿Cuál de los siguientes mecanismos de acción corresponde al de la etosuximida?


d) Disminución delas corrientes de calcio en las neuronas talámicas

62. Paciente femenina de 23 años, acude a consultar tumefacción en cuello. A la


exploración física se evidencia un nódulo tiroideo fijo, sin adenomegalias. El ultrasonido
reporta nódulo tiroideo de 2 cm con microcalcificaciones en su interior. La gammagrafía
con yodo marcado muestra únicamente un nódulo ""frío"" en la tiroides. Se realiza una
BAAF que reporta papilas recubiertas de células atípicas, calcificaciones en grano de arena
(cuerpos de psamoma) y núcleos escindidos con aspecto de "Anita la huerfanita".
62a. Por epidemiología, ¿cuál es el tumor más probable en esta paciente?
d) Carcinoma papilar de tiroides

62b. ¿Cuál es el tratamiento indicado en esta paciente?


b) Cirugía

62c. Señale la estrategia recomendada para el seguimiento de esta paciente luego de


haberle brindado tratamiento:
a) Medición de tiroglobulina

62d. Según los datos mencionados en el caso, el pronóstico para la vida de esta paciente
es:
c) Bueno

63. Paciente femenino de 23 años de edad acude a reumatología proveniente del servicio
de infectología con el diagnóstico de coriorretinitis por citomegalovirus en base a lgG
elevada. La paciente presenta desde hace un mes pérdida de agudeza visual en gran
medida en ojo izquierdo y del mismo tiempo de evolución crisis convulsivas tonicoclónicas
generalizadas; signos vitales tomados por pasante de enfermería TA 100/60, FC 90, FR 22,
temperatura 37. Se identifica disminución de intensidad de pulsos carotídeos, carotidinea,
soplo subclavio izquierdo y ausencia de pulsos radiales y cubitales.
63a. ¿Cuál es el diagnóstico más probable?
d) Probable vasculitis sistémica

63b. Por el grupo etáreo y las características clínicas. ¿Cuál es el diagnóstico específico que
puede tener la paciente?
a) Arteritis de Takayasu

63c. Durante la evaluación de la paciente en la consulta externa, qué determinación de


parámetro o signo vital está en duda?
d) Presión arterial

66. Luisa es una paciente femenina de 65 años de edad con antecedente de


hipotiroidismo hace 1O años en buen apego a levotiroxina. Acude a consulta por
presentar desde hace 7 días visión borrosa, fotofobia, cefalea de moderada intensidad de
tipo punzante sin predominio de horario, así como náusea y vómito. Al interrogatorio
dirigido refiere escotomas. Dentro del exámen físico se indentifica congestión conjuntiva!
y dolor a la presión de globos oculares. Resto de la exploración sin alteraciones. FC 98 lpm,
FR 18 rpm,temp 36.8 ºC, PA 130/85 mm Hg, peso 76 kg, talla 1.56 m.
66a. Con base en el interrogatorio de la paciente, el diagnóstico presuntivo es:
b) Glaucoma

66c. ¿Cuál es el tratamiento para el glaucoma de ángulo cerrado?


b) Acetazolamida

66d. En el cierre angular intermitente se puede observar al examen oftalmológico:


d) Sinequias anteriores periféricas

66e. La córnea en el cierre angular crónico se encuentra:


b) Transparente

67d. Tratamiento indicado en esta paciente.


c) Piridostigmina

71. Paciente femenino de siete años que inició con cefalea, malestar general, fiebre y
dolor faríngeo. Acude a consulta y se encuentra febril (39.9 ºC), faringe hiperémica, con
exudado amigdalino blanco, papilas linguales hipertróficas y coloración hiperémica de
lengua. Se palpan linfadenomegal ias cervicales bilaterales dolorosas. Se observan lesiones
papuloeritematosas en cara, cuello, tórax, regiones axilar e inguinal; al palparlas se
sienten ásperas.
En relación a la enfermedad que presenta la paciente, ¿cuál de los siguientes enunciados
es FALSO?
d) Las manchas de Koplik y conjuntivitis no exudativa son manifestaciones típicas

73. Se trata de Juan Carlos, un paciente masculino de 23 años detenido en el aeropuerto


de la Ciudad de Mexico por la Policía Federal por sospecha de transporte ilícito de
sustancias. En el examen con rayos X se evidencia la presencia de aproximadamente 22
cápsulas en cámara gastrica. Al interrogatorio el paciente desconoce el contenido de
dichas cápsulas. Duarante su estancia con los peritos presenta de manera subita, cefalea,
deterioro neurológico, taquicardia, midriasis, las pupilas se encuentran midriáticas ,
covulsiones y datos de focalización. FC 130 lpm, FR 27 rpm, temp 35.8 ºC, PA 180/100 mm
Hg, peso 85 kg, talla 1.56 m.
73a. Con base en el interrogatorio y los datos clínicos del paciente, el diagnóstico
presuntivo es:"
a) intoxicación por cocaína

78b. Con respecto a esta patología, ¿cuál es la respuesta correcta?


a) Eleccema dishidrótico coexiste con dermatitis atópica

78c. El tratamiento de elección en esta enfermedad es:


b) Corticoesteroides tópicos de alta potencia

79. Masculino de 5 años no inmunizado es traído a consulta con una historia de 2 semanas
de evolución de tos paroxística, fiebre de bajo grado, emesis después de toser y descarga
nasal viscosa. En la exploración física se observa otitis media bilateral y conjuntivitis
hemorrágica. En la auscultación se detectan estertores inspiratorios de forma bilateral.
Los exámenes de laboratorio reportan biometría hemática con 45000 leucocitos, con
diferencial de 95% linfocitos.
79a. Con base en las características del paciente, ¿cuál es el diagnóstico más probable?
b) Tosferina

79c. En relación con su sospecha clínica, ¿cuál es el tratamiento que indicaría?


a) Eritromicina

"102. Acude a consulta externa un hombre de 28 años de edad por presentar desde hace
15 días fatiga, malestar"
"general y disnea. El paciente es originario de un área rural, y por el momento se
encuentra laborando como constructor; ha estado viviendo debajo de un puente. Al
interrogatorio el paciente refiere que sus síntomas se han acompañado de tos sin flema, la
cual en ocasiones lo lleva al vómito; también refiere anorexia en los últimos días. El
paciente niega cualquier pérdida de peso o sudoración nocturna, sin embargo, menciona
que es posible que haya sufrido de fiebre (subjetiva, no cuantificada). A la edad de 26
años, un médico le diagnosticó síndrome de inmunodeficiencia humana con un conteo de
linfocitos CD4+ de 134 (el paciente trae reporte con los resultados)."
"Actualmente no se encuentra consumiendo medicamentos y confiesa ser adicto al uso de
cocaína y etanol. A la exploración física se encuentra una presión arterial de 120/75 mm
Hg, taquicardia de 95 latidos/minuto; la saturación de oxígeno se encuentra reducida; la
Pa02 es de 65 mm Hg. La inspección general muestra un paciente con dificultad
respiratoria y hallazgos auscultatorios de neumonía. La radiografía torácica demuestra un
infiltrado difuso bilateral," con predominio basal.

"Utilizando únicamente la información previamente descrita, responda las siguientes


preguntas:"
102a. ¿Cuál de los siguientes agentes patógenos es el que con mayor probabilidad está
ocasionando los síntomas de
este paciente?

 a Pneumocystis jiroveci

102b. ¿Cuál de las siguientes opciones es la mejor conducta terapéutica en este paciente?
 d} Administración de trimetoprim-sulfametoxazol y prednisona
103. ¿Cuál de los siguientes fármacos inhibe la enzima escualeno epoxidasa?

 d} Terbinafina

104. Paciente de 18 años de edad que acude a consulta por malestar general de dos
semanas y malestar faríngeo. A
"la exploración física se revela faringitis, hepatoesplenomegal ia y linfadenopatías axilares.
Los resultados de estudios de laboratorio son: hemoglobina 13 g/dL, hematocrito 43%,
volumen corpuscular medio 95 fL, conteo plaquetario 300 000 plaquetas/mm3 y
leucocitos de 9 000 células/mm3. El frotis de sangre periférica demuestra linfocitos
atípicos."
¿Cuál fue el principal origen por el que esta paciente adquirió esta enfermedad?
 d} Por su novio

105. Paciente de sexo femenino de 18 meses de edad es llevada al departamento de


urgencias por alteraciones
"mentales y retraso mental progresivo. A la exploración física se encuentra
hepatoesplenomegalia, linfadenopatías, sin alteraciones a la exploración del fondo de ojo.
Se realiza biopsia hepática, esplénica y de ganglios linfáticos, los cuales demuestran
macrófagos aumentados de tamaño con lisosomas alargados y distendidos. ¿Cuál es la
deficiencia enzimática que presenta esta paciente?"

 b} Glucocerebrosidasa

106. ¿Cuál de las siguientes hormonas gastrointestinales estimula la secreción de


bicarbonato del páncreas?

 b} Secretina

107. Paciente femenino de 22 años de edad se diagnosticó con un trastorno hemorrágico


desde la adolescencia.
"Entre los principales síntomas, la paciente informa hemorragias al cepillado de los
dientes, episodios de menorragia y anemia grave. El mismo cuadro presentaron la madre
y abuela de esta paciente. Los estudios de laboratorio revelan un conteo plaquetario de
250 OOO/mm3, incremento en el tiempo de sangrado, pero con normalidad del tiempo de
protrombina y el tiempo de tromboplastina parcial. ¿Cuál es el diagnóstico más probable
en esta paciente?"
O e} Trombastenia de Glanzmann

108. Paciente femenino de 19 años de edad que se presenta a consulta por varias
molestias. La paciente menciona
"que debido a su sobrepeso decidió iniciar una dieta, consumo de suplementos
alimenticios y ejercicio. La paciente menciona que su padecimiento actual inició hace
aproximadamente 20 días con cefalea, fatiga, dolores óseo y articular; sin embargo, al
interrogatorio se reveló que la paciente ya presentaba disminución del apetito y cabello
quebradizo desde hace un mes. A la exploración física la paciente presenta labios secos y
con fisuras, sin ni nguna otra anormalidad evidente. Los estudios de laboratorio revelan
incremento de las enzimas hepáticas e hiperlipidemia."
¿Cuál de las siguientes vitaminas se asocia con el cuadro clínico que presenta esta
paciente?

 a} Vitamina A

"109. Paciente de 45 años de edad que se presenta con mama derecha aumentada de
tamaño, con induraciones y"
eritematosa. La mastografía revela la presencia de una región con distorsión de la
arquitectura. La biopsia de glándula mamaria y piel indican un carcinoma ductal
infiltrante. ¿Cuál es el tratamiento más adecuado para esta paciente?

 c} Quimioterapia y posteriormente realizar mastectomía

110. Paciente masculino de 61 años de edad que acude a consulta por dolor en el flanco
izquierdo desde hace un
mes. A pesar de que ha tratado con diferentes analgésicos y reposo el dolor no
desaparece o disminuye. Al interrogatorio niega disuria o hematuria. Se solicita un
examen general de orina el cual revela la presencia de hematuria microscópica. Debido a
estos hallazgos se decide solicitar una tomografía computarizada contrastada
abdominopélvica la cual revela una tumoración proveniente de la corteza renal con
presencia de reforzamiento y componentes sólidos. ¿Cuál de los siguientes diagnósticos es
el más probable en este paciente?

 b} Carcinoma renal

111. ¿Cuál es la neoplasia maligna que se caracteriza por la presencia de células de Reed-
Sternberg?

 b} Linfoma Hodgkin (' c} Linfoma folicular


112. Paciente de 45 años de edad que ha sido tratado desde los últimos seis meses con
litio. ¿Cuál de las siguientes
es la enfermedad de base más probable?

 b} Trastorno bipolar

113. ¿Cuál de las siguientes neoplasias del sistema nervioso central en adultos se localiza
principalmente en las
convexidades de los hemisferios cerebrales?

 b} Meningioma

114. ¿Cuál de los siguientes hallazgos se asocia con carcinoma renal?

 c} Policitemia y/o hipercalcemia con hipofosfatemia

115. Paciente femenino de 12 años de edad que acude a consulta externa de pediatría. La
madre menciona que la
"niña ha estado cojeando desde hace varias semanas y refiere dolor intenso en la rodilla
izquierda. Al interrogatorio la madre dice que la paciente ha estado afebril, no recuerda
golpes o traumatismos dirigidos a las extremidades, ni antecedente de enfermedades
recientes. A la exploración física es evidente que la paciente no puede rotar internamente
la pierna afectada. ¿Cuál de los siguientes diagnósticos es el más probable en esta
paciente?"
 a} Epifisiólisis de cabeza femoral

116. ¿Cuál de las siguientes enfermedades produce acumulación de ácido homogentísico


en sangre?

 e} Alcaptonuria

117. ¿Cuál de las siguientes translocaciones cromosómicas se asocia con el sarcoma de


Ewi ng?

 e} t(11:22}

118. Paciente de 35 años de edad acude a consulta por un exantema difuso que involucra
a los folículos pilosos con
pústulas. La paciente informa que recientemente ingresó a un centro deportivo y ha
utilizado el baño sauna. ¿Cuál de las siguientes bacterias es más probablemente
responsable de la condición de esta paciente?

 a} Pseudomonas aeruginosa
119. Paciente de 75 años de edad que ha sido llevada a la sala de cateterismo por infarto
miocárdico agudo. La
"paciente expresa que había presentado dolor torácico opresivo con irradiación hacia el
cuello y el brazo izquierdo. A la exploración física la paciente se presenta diaforética, con
frecuencia cardiaca de 50 latidos/minuto, presión arterial de 85/50 mm Hg, distensión
venosa yugular y estertores pulmonares bilaterales. La angiografía coronaria demuestra
oclusión del ostium de la arteria coronaria derecha la cual es dominante, sin enfermedad
de la circulación coronaria izquierda. ¿Cuál de las siguientes opciones se presentaría con
menor probabilidad en esta paciente?"

 e} Depresión del segmento PR

"120. Un paciente masculino de 60 años acude a consulta por presentar fatiga,anorexia y


diaforesis. El paciente"
muestra los resultados de una biometría hemática y un frotis de sangre periférica la cual
reporta neutrófilos y metamielocitos. Se realiza un estudio citogenético que demuestra
una translocación 9:22. ¿Cuál de las siguientes opciones describe la función de la proteína
producida en este paciente?

 a} Una tirosincinasa constitutivamente activa

86. Un hombre de 69 años acude a consulta para revisión general. En sus antecedentes
patológicos se refiere hipertensión, hiperlipidemia y diabetes mellitus tipo 2. Sus
medicamentos incluyen amlodipino, rosuvastatina y gliburida. Refiere ser fumador con
tabaquismo de 22 paquetes/año. Al examen físico se presenta un paciente obeso. Sus
signos vitales incluyen FC 82/min, FR 15/min, T 37.3C y PA 155/81 mm Hg. Su glucosa
capilar aleatoria es de"
143. ¿Cuál de los siguientes cambios disminuiría más el riesgo de EVC en este paciente?

 b} Disminuir su presión arterial

87. Un hombre de 72 años acude a revisión general. En sus antecedentes patológicos se


refiere hiperplasia prostática
"benigna, hipertensión, diabetes, hiperlipidemia y gota. Sus medicamentos incluyen
metoprolol,gliburida, atorvastatina y alopurinol. Refiere tabaquismo de 1O paquetes/año,
y tomar una copa de vino al día. Niega síntomas al IPAS, refiriendo que nunca se había
sentido mejor. De los estudios que se le realizaron antes de ir a consulta, lo único anormal
es un aumento en el antígeno prostático de 2.2 a 4.3 ng/ml. Al tacto rectal se aprecia un
nódulo aproximadamente del tamaño de la yema del dedo índice en el lóbulo derecho.
¿Cuál es el siguiente paso en el manejo de este paciente?"

 a} Biopsia prostática
"88. Una mujer de 65 años acude a consulta para revisión general. Es una persona sana,
refiriendo sólo dolores"
"articulares intermitentes e hipotiroidismo. Sus medicamentos incluyen levotiroxina y
ocasionalmente paracetamol. Refiere tabaquismo de 20 paquetes/año y consume bebidas
alcohólicas de forma regular.Al interrogatorio niega otros síntomas. Sus signos vitales
incluyen FC 82, FR 16, T 37.3 C y PA 120/67. Su preocupación principal es la"
osteoporosis. ¿Cuál de los siguientes son factores de riesgo para la osteoporosis en esta
paciente?
 a} Tabaquismo y alcoholismo

89. Paciente femenino de 74 años que acude a urgencias por dolor abdominal. Describe al
dolor como tipo cólico, de intensidad 8/10, presente en el cuadrante inferior izquierdo. En
sus antecedentes lo único que refiere es hipertensión, controlada con hidroclorotiazida.
Menciona acostumbrar tomar una copa de vino al día. Al IPAS refiere haber tenido diarrea
y sensación de distensión abdominal los días anteriores, y el dolor ya había ocurrido en
ocasiones anteriores, aunque nunca en este grado. Niega esteatorrea o hematoquezia. A
la exploración física sus signos vitales incluyen FC 106/min, FR 18/min, T 38.2 C, PA 136/83
mm Hg. En la palpación abdominal se describe dolor en el cuadrante inferior izquierdo. No
hay signo de rebote presente. Se indica biometría hemática que muestra una leucocitosis
de 19 OOO/mm3. Se realiza una tomografía computarizada, que muestra absceso
pericólico de 2.5 cm de diámetro. ¿Qué tratamiento le daría a esta paciente?"

 a} Ciprofloxacina + metronidazol

90. Paciente femenino de 67 años que acude a consulta por sangrado transvaginal. Refiere
que éste inició hace tres
"semanas, aunque anteriormente sí había notado que manchaba ligeramente su ropa
interior. Su última menstruación fue a los 48 años, y en la actualidad es sexualmente
activa con su esposo con el que lleva casada 17 años (sin asociar el sangrado con la
relación sexual). Tiene dos hijos, de su esposo anterior. En sus antecedentes patológicos
refiere hipertensión, diabetes mellitus tipo 2 e hiperlipidemia. A la exploración física se
observa una paciente obesa. Sus signos vitales incluyen FC 89/min, FR 16/min, T 36.6 C y
PA 129/82 mm Hg. Al examen pélvico se observa una mucosa atrófica, sin evidencia de
sangrado. No hay dolor a la palpación bimanual y no hay masas. ¿Cuál sería el siguiente
paso en el manejo de esta paciente?"

 b} Biopsia endometrial

91. Paciente masculino de 74 años que es traído por su hija para ser evaluado por
depresión. Refiere que el paciente
"ya no muestra interés por actividades que disfrutaba antes, como leer o pintar. La mayor
parte del día se la pasa frente al televisor. Sus antecedentes incluyen hipertensión,
colelitiasis y osteoartritis. La hija cuenta también que ha tenido comportamientos
extraños, haciendo comentarios eróticos a otras mujeres e incluso ha llegado a pellizcar o
dar una nalgada a algunas. También ha llegado a comportarse agresivo sin una razón
particular detonante. Al hablar con el paciente se nota un afecto plano, negando cambios
de humor, insomnio, pérdida de apetito o disminución en la capacidad de concentración.
También niega cualquier ideación o plan suicida. El minimental resulta en 21/30, no
teniendo problemas de orientación. Se realiza una resonancia magnética, mostrando
atrofia de los lóbulos frontales."
¿Cuál es la causa más probable de los síntomas de este paciente?

 e} Enfermedad de Pick

"92. Se trata de una paciente de 55 años, con escolaridad 3° de primaria, procedente de


Guerrero, quien tiene un problema de gonartrosis de ambas rodillas y es vista en consulta
de ortopedia. Se le ofrece tratamiento quirúrgico para poner implante. Se le explica que
no recuperará del todo la función y la paciente rechaza la cirugía.

92a. ¿Cuál debe ser la conducta del médico en este caso?

- d} Aceptar la voluntad de la paciente y enviar a rehabilitación


92b. Ante un paciente de estas características, edad, género, escolaridad, origen y cultura,
el médico debe considerar:

 a}Informar en forma detalladalos procedimientos y resultados y aceptar las


decisiones delos pacientes

93. Niño de cuatro años de edad con antecedente de infección de vías respiratorias
superiores hace tres días que recibió sólo manejo sintomático con paracetamol
(acetaminofén). El día de ayer inició con anorexia, náusea y vómito. A partir de hoy se
queja de dolor abdominal intenso en hipocondrio derecho. En la exploración se encuentra
pálido, diaforético, con mucosa oral seca, dolor a la palpación en cuadrante superior
derecho del abdomen y hepatomegalia."
"Los estudios de laboratorio reportaron hemoglobina de 13, hematocrito de 45%,
leucocitos 9 700, plaquetas 225 000,"
"sodio 140, potasio 3.8, creatinina 0.7, AST 120, ALT 135 y bilirrubina total 1.7."
"93a. En el contexto de la intoxicación que sufre el paciente, ¿qué metabolito es
responsable del cuadro clínico?"

 b} N-acetil-p-benzoquinoneimina

93b. ¿Cuál es el antídoto indicado para la intoxicación del paciente?

 d} N-acetilcisteína

94. ¿Cuál de las siguientes vasculitis tiene el mejor pronóstico de vida?


 a} Púrpura de Henoch-Schonlein

"95. Varón de 25 años, de oficio herrero, sin antecedentes de importancia, que acude a
consulta por dolor,fotofobia y disminución de agudeza visual posterior a trabajar aluminio
con su martillo, 8 horas previas a la consulta. A la exploración oftalmológica se encuentra
con hiperemia conjuntiva!, inyección ciliar, córnea con leve edema, anillo de óxido
alrededor de un cuerpo opaco, cámara anterior vacía, pupila normorrefléctica, cristalino
transparente, vítreo transparente, retina aplicada."
95a. ¿Cuál es el tratamiento inmediato del trauma superficial con cuerpo extraño
metálico?

 d} Retiro delcuerpo extraño

95b. ¿Cuál es la primera exploración oftalmológica en un paciente con trauma?

 c} Agudeza visual

"95c. Basados en la agudeza visual posterior altrauma, ¿qué se le puede ofrecer al


paciente?"
 d} Pronóstico funcional

96. ¿Cuál de las siguientes condiciones tiene mayor influencia en la progresión de


infección inicial por el virus de la inmunodeficiencia humana hacia el síndrome de
inmunodeficiencia adquirida?

 b} La carga viral seis meses posteriores alainfección inicial

97. Una paciente de 60 años se presenta con diagnóstico reciente de cáncer de mama en
actual tratamiento con ciclofosfamida, metotrexato y 5-fluorouracilo. El motivo de
consulta es por náusea y vómito posterior a la quimioterapia, así como disnea de reciente
inicio, úlceras orales y fatiga. A la exploración física la paciente presenta derrame pleural
del lado derecho y mucositis. Los resultados de laboratorio son los siguientes: creatinina
de 2.4 mg/dL, nitrógeno ureico de 25 mg/dL, leucocitos de 1 OOO/mm3, hemoglobina de
9.2 mg/dL y plaquetas de 91 OOO/mm3. ¿Cuál es el origen de la mayoría de sus actuales
síntomas?"

 a} Toxicidad por metotrexato

98. ¿Cuál de los siguientes procesos fisiopatológicos producen principalmente una


hipertrofia concéntrica ventricular?

 c} Estenosis aórtica
99. Paciente femenino de 29 años de edad que se presenta con poliartralgias y rigidez
matutina en las manos y rodillas. Hace una semana la paciente presentó un episodio
febrilacompañado de exantema eritematoso facial y reticular en las extremidades,
características que ya no se encuentran presentes. A la exploración física presenta ligero
edema y dolor a la presión de las articulaciones del carpo, metacarpofalá ngicas e interfalá
ngicas proximales bilateralmente. ¿Cuál es el diagnóstico más probable en esta paciente?"

 d} Infección por el parvovirus 819

100. ¿Cuál de las siguientes hormonas se encarga de la conversión de testosterona en 17-


beta estradiol en las células
de la granulosa?

 d} Hormona foliculoestimulante

"101. Se presenta una paciente de 15 años de edad, con preocupación porque no ha


comenzado a menstruar. Al interrogatorio no se revela ningún antecedente de
importancia. La exploración física es significativa por un adecuado
"desarrollo mamario, estatura dentro de rangos normales, vagina corta y ausencia de
cervix y útero a la exploración pélvica bimanual. ¿Cuál es el diagnóstico más probable en
esta paciente?"

 e} Síndrome de feminización testicular

"78. Adolescente de 16 años que presenta una dermatosis diseminada, bilateral, con
tendencia a la simetría que afecta"
"palmas y plantas, caracterizada por ampollas milimétricas con contenido seroso, algunas
zonas con descamación en forma de collarete. Inicia desde la infancia con periodos de
remisiones y exacerbaciones. Tiene antecedente de dermatitis atópica desde la infancia
con buen control. Refiere hiperhidrosis palmoplantar y exacerbaciones en verano."
78a. ¿Cuál es la etiología más probable de esta hiperhidrosis?

 e} Desconocida

"78b. Con respecto a esta patología, ¿cuál es la respuesta correcta?"

 a} Eleccema dishidrótico coexiste con dermatitis atópica

78c. El tratamiento de elección en esta enfermedad es:

 b} Corticoesteroides tópicos de alta potencia

79. Masculino de 5 años no inmunizado es traído a consulta con una historia de 2 semanas
de evolución de tos paroxística, fiebre de bajo grado, emesis después de toser y descarga
nasal viscosa. En la exploración física se observa otitis media bilateral y conjuntivitis
hemorrágica. En la auscultación se detectan estertores inspiratorios de forma bilateral.
Los exámenes de laboratorio reportan biometría hemática con 45000 leucocitos, con
diferencial de 95% linfocitos."

79a. Con base en las características del paciente, ¿cuál es el diagnóstico más probable?

 b) Tosferina

79b. ¿Qué método es el más adecuado para confirmar la sospecha diagnóstica?

 C) Identificación de B pertussispor cultivo o PCR

"79c. En relacióncon su sospecha clínica, ¿cuál es el tratamiento que indicaría?"

o a} Eritromicina

79d. ¿Cuál es la complicación más común y grave de la tosferina?

 d} Bronconeumonía

80. Paciente femenino de 72 años que acude a consulta por incontinencia urinaria. Refiere
que la evolución ha sido progresiva, hasta hay que ha tenido que usar toallas absorbentes
en caso de tener salida de orina de manera involuntaria. En sus antecedentes manifiesta
hipertensión e hiperlipidemias, que controla con amlodipino y atorvastatina. Tiene cinco
hijos, que tuvo por parto natural. Niega tabaquismo y consumo de alcohol o de productos
con cafeína. Refiere que la salida de orina puede ocurrir cuando se ríe o estornuda,
aunque a veces también simplemente siente ganas de manera urgente y tiene que
apresurarse al baño. Niega prolapso de tejidos, disuria, fiebre o cambios de la coloración
de la orina. Actualmente no quiere someterse a ningún procedimiento quirúrgico, por lo
que usted decide recomendar ejercicios pélvicos e iniciar oxibutinina. ¿Cómo funciona
este medicamento?

 b} Antagonista de receptores muscarínicos

81. ¿Cuál de las siguientes opciones representa adecuadamente la posición en el tiempo


más común de los soplos inocentes?

 c} Mesosistólico

"82. Paciente femenino de 27 años de edad, que acude a consulta de planificación
familiar. Refiere paridad no satisfecha, no embarazada, sin datos de enfermedad pélvica
inflamatoria. Se decide colocarle un dispositivo intrauterino. Histerometría de 7 cm. Se
observa escéptica en el uso, ya que le comenta,que ella ha escuchado que es"
un abortivo. ¿Cuál es el mecanismo de acción del dispositivo intrauterino?

 d} Crea una respuesta inflamatoria

"83. Paciente masculino de ocho días de vida extrauterina,con antecedente de haber sido
producto de G1 P1, con embarazo normoevolutivo, de 38 SDG, con rotura prematura de
membranas 5 horas previas al inicio del trabajo de parto, obtenido por parto eutócico y
Apgar 8/9. Acude a revisión oftalmológica porque la madre refiere secreción oftálmica
amarillenta e hiperemia conjuntiva! intensa en ambos ojos de dos días de evolución,
asociado a febrícula e irritabilidad. A la exploración se observa secreción amarillenta en
moderada cantidad, quemosis conjuntiva!,córnea clara intacta y el resto de la exploración
oftalmológica, sin alteraciones."
83a. El diagnóstico más probable del caso clínico previo es:

 c} Oftalmia neonatorum

83b. ¿Cuál de los siguientes agentes es el que más frecuentemente ocasiona esta
condición?
r a} Staphy/ococcus aureus
 e} Chlamydia trachomatis

83c. En el caso clínico anterior cual es el tratamiento más adecuado:


 b} Tetraciclina tópica

84. Paciente femenino de 55 años de edad que acude por presentar metrorragia desde
hace cinco meses, sin agregados. Refiere menarquía precoz. Es diabética tipo 2 de larga
evolución con tratamiento a base de metformina y glibenclamida, hipertensa con
tratamiento a base de lisinopril y metoprolol. Obesa, TA 140/90.

84a. Se sabe que existen factores pronóstico para esta patología. ¿Cuál de los siguientes
factores, significa un mejor"
pronóstico?

 d} Jóvenes

84b. ¿Cuál es el mejor tratamiento para la paciente?

 b} Histerectomía radical con linfadenectomía pélvica y radioterapia

85. Paciente masculino de 7 años de edad, previamente sano, acude a consulta por dolor
en testículo derecho. El dolor,que tiene 1O h de evolución, comenzó de manera súbita la
noche previa sin antecedente traumático. Es de intensidad moderada y se ha mantenido
constante, aunque cede parcialmente con ibuprofeno. No hay irradiación, problemas de
micción, fiebre o vómito. En la exploración se observa piel eritematosa, dolor en el polo
superior del testículo, cierto aumento de volumen y reflejo cremastérico presente. No
tiene episodios previos similares."

85a. Con base en las características del paciente, ¿cuál es su sospecha diagnóstica?"

 b} Torsión del apéndice testicular

"85b. Con base en su sospecha clínica, ¿qué hallazgos espera encontrar durante la
exploración física con"
transiluminación?
 a} Signo de punto azul
"85c. Usted decide realizar una ecografía doppler.De acuerdo con su sospecha clínica,
¿qué hallazgos espera"
encontrar?
 b} Flujo arterial aumentando

"85d. Si los hallazgos dela exploración física y la ecografía concuerdan con su sospecha
clínica, ¿cuál es el tratamiento"indicado?
 a} Sintomático con antiinflamatorios y reposo
85. Paciente masculino de 7 años de edad, previamente sano, acude a consulta por dolor
en testículo derecho. El dolor,que tiene 1O h de evolución, comenzó de manera súbita la
noche previa sin antecedente traumático. Es de intensidad moderada y se ha mantenido
constante, aunque cede parcialmente con ibuprofeno. No hay irradiación, problemas de
micción, fiebre o vómito. En la exploración se observa piel eritematosa, dolor en el polo
superior del testículo, cierto aumento de volumen y reflejo cremastérico presente. No
tiene episodios previos similares."

85e. En conclusión, ¿cuál de las siguientes opciones mencionalas características que se


presentan en el padecimientode este paciente?

 b} Edad prepúberes, dolor leve a moderado, posición normal deltestículo, reflejo


cremastérico presente, signo del punto azul"

67. Mujer de 20 años, sin antecedentes patológicos, inicia padecimiento con diplopía
intermitente hace un mes, y recientemente se acompaña de ptosis unilateral. A la
exploración física se observa fuerza muscular y sensibilidad conservada en todas las
extremidades. Pares craneales sin alteraciones. No refiere cefalea. Se sospecha de
miastenia gravis.
67a. ¿Cuál se las siguientes pruebas es la más específica para el diagnóstico de miastenia
gravis?
 d} Anticuerpos antirreceptor de acetilcolina

67b. En relación con la clínica de la miastenia gravis, señale la opción correcta:


 c} La debilidad muscular en los miembros es proximal y asimétrica
67c. Origen fisiopatológico de la enfermedad con la que cursa la paciente.
 a} Bloqueo de receptores nicotínicos de acetilcolina en la membrana postsináptica
dela placa motora

67d. Tratamiento indicado en esta paciente.


 e} Piridostigmina

67e. ¿Con qué enfermedad neoplásica se relaciona la miastenia gravis?


 b} Timoma

68. Paciente femenina de 77 años de edad, con hipertensión arterial e insuficiencia


cardiaca. Ingresa a urgencias por descompensación de la IC. Al interrogatorio el familiar
menciona que ha referido palpitaciones desde hace una semana. A la exploración se
registra TA: 80/55 mm Hg, Sa02: 88%, estertores crepitates bibasales, precordio arrítmico
sin soplos, pulso radial arrítmico de intensidad variable, ingurgitación yugular grado 11 sin
hepatomegalia, pares craneales sin alteraciones, fuerza y sensibilidad en extremidades
conservada. ECG: frecuencia ventricular 160 x min, irregular, QRS angosto y ondas f."

68a. ¿Cuál es la actitud terapéutica a seguir en esta paciente?


 a} Cardioversión eléctrica

68c. ¿Cuál es el mecanismo fisiopatológico de la fibrilación auricular?


 c} Anomal ía en la propagación delimpulso por reentrada

68d. ¿Cuál es la recomendación para la profilaxis tromboembólica en esta paciente?


 b} Anticoagulación oral crónica manteniendo INR entre 2 y 3

69. Paciente de 79 años que consulta por presentar una mácula pigmentada heterocroma,
de contorno irregular, de aproximadamente 2 a 3 cm de diámetro, localizada en mejilla
derecha, que ha experimentando un crecimiento lento en los últimos años."
¿Cuál es el diagnóstico más probable en este paciente?

 d} Lentigo maligno
70. Femenino de 25 años acude a consulta externa refiriendo tenesmo vesical,disuria y
escalofrío. Antecedente: cursa su 14ª semana de gestación, La medida terapéutica de
primera intención en esta paciente es:
 a} Ampicilina

"71. Paciente femenino de siete años que inició con cefalea, malestar general, fiebre y
dolor faríngeo. Acude a consultan y se encuentra febril (39.9 ºC), faringe hiperémica, con
exudado amigdalino blanco, papilas linguales hipertróficas y coloración hiperémica de
lengua. Se palpan linfadenomegal ias cervicales bilaterales dolorosas. Se observan lesiones
papuloeritematosas en cara,cuello, tórax, regiones axilar e inguinal;al palparlas se sienten
ásperas. En relación a la enfermedad que presenta la paciente, ¿cuál de los siguientes
enunciados es FALSO?"

 d} Las manchas de Koplik y conjuntivitis no exudativa son manifestaciones típicas

72. Se presenta a la sala de urgencias un paciente masculino de 60 años de edad. La


principal molestia del paciente son palpitaciones, palidez y diaforesis. A la exploración
física se encuentra una temperatura de 39 ºC, frecuencia cardiaca de 145 latidos por
minuto y presión arterial de 180/11O mm Hg. Los familiares mencionan que el paciente
enviudó hace un año y recientemente se encuentra desempleado. A pesar de que no era
un consumidor frecuente de alcohol, desde la muerte de su esposa el paciente ha
incrementado el consumo de bebidas alcohólicas. Hace dos semanas se le diagnosticó
depresión mayor y se inició tratamiento farmacológico. ¿Cuál es el medicamento que
más"
probablemente está tomando el paciente?
 d} Fenelzina

"73. Se trata de Juan Carlos, un paciente masculino de 23 años detenido en el aeropuerto


de la Ciudad de Mexico por la Policía Federal por sospecha de transporte ilícito de
sustancias. En el examen con rayos X se evidencia la presencia de aproximadamente 22
cápsulas en cámara gastrica. Al interrogatorio el paciente desconoce el contenido de
dichas cápsulas. Duarante su estancia con los peritos presenta de manera subita, cefalea,
deterioro neurológico, taquicardia, midriasis, las pupilas se encuentran midriáticas ,
covulsiones y datos de focalización. FC 130 lpm, FR 27 rpm, temp 35.8 ºC, PA 180/100 mm
Hg, peso 85 kg, talla 1.56 m. 73a. Con base en el interrogatorio y los datos clínicos del
paciente, el diagnóstico presuntivo es:

 a} intoxicación por cocaína

"73. Se trata de Juan Carlos, un paciente masculino de 23 años detenido en el aeropuerto


de la Ciudad de Mexico por la Policía Federal por sospecha de transporte ilícito de
sustancias. En el examen con rayos X se evidencia la presencia de aproximadamente 22
cápsulas en cámara gastrica. Al interrogatorio el paciente desconoce el contenido de
dichas cápsulas. Duarante su estancia con los peritos presenta de manera subita, cefalea,
deterioro neurológico, taquicardia, midriasis, las pupilas se encuentran midriáticas ,
covulsiones y datos de focalización. FC 130 lpm, FR 27 rpm, temp 35.8 ºC, PA 180/100 mm
Hg, peso 85 kg, talla 1.56 m."

73b. Con base en lo expuesto en el caso clínico. ¿usted qué tipo de toxíndrome integra en
este paciente
 e} Simpaticomimético

73c. ¿Cuál es el abordaje inicial para intoxicación por cocaína?


 d} Permeabilidad de vía aérea, monitoreo hemodinámico, hidratación"
73d. Una vez estabilizado el paciente se identifica que durante las últimas tres horas los
flujos urinarios han
disminuido y en los paraclínicos se encuentra elevación importante de CK y CK-MB. ¿Cuál
es su sospecha diagnóstica?
 c} Rabdomiolisis

"73e. Una vez controlado el evento agudo en urgencias, ¿cuál es la recomendación


estándar que indicaría a su paciente?
 c} Vigilar datos de hipertensión crónica

74. ¿Cuál de los siguientes tipos de catarata se asocia con debilidad muscular?
 "O a} Catarata policromática múltiple (catarata en ""árbol de navidad'1"

75. Paciente femenino de nueve años de edad con antecedentes de pubertad y menarca
precoz acude a consulta porpresentar desde hace cuatro meses dolor en las articulaciones
de la cadera derecha y en los tobillos. A la exploración física presenta lesiones
hiperpigmentadas (manchas café con leche), con bordes irregulares y dolor a la palpación
de las articulaciones antes mencionadas. Se solicitan estudios de rayos X, los cuales
revelan lesiones poliostósicas en forma de vidrio despulido en la porciones proximal del
fémur y distal de la tibia. ¿Cuál es el diagnóstico más probable de esta paciente?
 b} Síndrome de McCune-Albright r

76. Se presenta al servicio de urgencias una paciente de 29 años de edad que se encuentra
en trabajo de parto. La paciente no acudió a ninguna consulta de manejo prenatal.
Durante el trabajo de parto se rompen artificialmente las membranas y es evidente que
existía oligohidramnios. El recién nacido pesa 1.8 kg, y presenta dificultad respiratoria
significativa inmediatamente después de nacer. La radiografía de tórax evidencia la
presencia de pulmones 'hipoplásicos. ¿Cuál de los siguientes estudios es el más adecuado
de realizar en este paciente?"
 b} Ultrasonido renal

67b. En relación con la clínica de la miastenia gravis, señale la opción correcta:&quot;

 c} La debilidad muscular en los miembros es proximal y asimétrica

67c. Origen fisiopatológico de la enfermedad con la que cursa la paciente.


 a} Bloqueo de receptores nicotínicos de acetilcolina en la membrana postsináptica
dela placa motora

67d. Tratamiento indicado en esta paciente.


 e} Piridostigmina

67e. ¿Con qué enfermedad neoplásica se relaciona la miastenia gravis?


 b} Timoma

68. Paciente femenina de 77 años de edad, con hipertensión arterial e insuficiencia


cardiaca. Ingresa a urgencias por&quot; &quot;descompensación de la IC. Al
interrogatorio el familiar menciona que ha referido palpitaciones desde hace una semana.
A la exploración se registra TA: 80/55 mm Hg, Sa02: 88%, estertores crepitates bibasales,
precordio arrítmico sin soplos, pulso radial arrítmico de intensidad variable, ingurgitación
yugular grado 11 sin hepatomegalia, pares craneales sin alteraciones, fuerza y sensibilidad
en extremidades conservada. ECG: frecuencia ventricular 160 x min, irregular, QRS
angosto y ondas f.&quot;

68a. ¿Cuál es la actitud terapéutica a seguir en esta paciente?


 a} Cardioversión eléctrica

68c. ¿Cuál es el mecanismo fisiopatológico de la fibrilación auricular?

 c} Anomal ía en la propagación delimpulso por reentrada

68d. ¿Cuál es la recomendación para la profilaxis tromboembólica en esta paciente?


 b} Anticoagulación oral crónica manteniendo INR entre 2 y 3

69. Paciente de 79 años que consulta por presentar una mácula pigmentada heterocroma,
de contorno irregular, de aproximadamente 2 a 3 cm de diámetro, localizada en mejilla
derecha, que ha experimentando un crecimiento lento en los últimos años
¿Cuál es el diagnóstico más probable en este paciente?

 d} Lentigo maligno

70. Femenino de 25 años acude a consulta externa refiriendo tenesmo vesical,disuria y


escalofrío. Antecedente: cursa su 14ª semana de gestación, La medida terapéutica de
primera intención en esta paciente es:
 a} Ampicilina

71. Paciente femenino de siete años que inició con cefalea, malestar general, fiebre y
dolor faríngeo. Acude a consulta y se encuentra febril (39.9 ºC), faringe hiperémica, con
exudado amigdalino blanco, papilas linguales hipertróficas y coloración hiperémica de
lengua. Se palpan linfadenomegal ias cervicales bilaterales dolorosas. Se observan lesiones
papuloeritematosas en cara,cuello, tórax, regiones axilar e inguinal;al palparlas se sienten
ásperas En relación a la enfermedad que presenta la paciente, ¿cuál de los siguientes
enunciados es FALSO;
 d} Las manchas de Koplik y conjuntivitis no exudativa son manifestaciones típicas

72. Se presenta a la sala de urgencias un paciente masculino de 60 años de edad. La


principal molestia del paciente;son palpitaciones, palidez y diaforesis. A la exploración
física se encuentra una temperatura de 39 ºC, frecuencia cardiaca de 145 latidos por
minuto y presión arterial de 180/11O mm Hg. Los familiares mencionan que el paciente
enviudó hace un año y recientemente se encuentra desempleado. A pesar de que no era
un consumidor
frecuente de alcohol, desde la muerte de su esposa el paciente ha incrementado el
consumo de bebidas alcohólicas. Hace dos semanas se le diagnosticó depresión mayor y
se inició tratamiento farmacológico. ¿Cuál es el medicamento que más&quot;
probablemente está tomando el paciente?
 d} Fenelzina

73. Se trata de Juan Carlos, un paciente masculino de 23 años detenido en el aeropuerto


de la Ciudad de Mexico por la Policía Federal por sospecha de transporte ilícito de
sustancias. En el examen con rayos X se evidencia la presencia de aproximadamente 22
cápsulas en cámara gastrica. Al interrogatorio el paciente desconoce el contenido de
dichas cápsulas. Duarante su estancia con los peritos presenta de manera subita, cefalea,
deterioro neurológico, taquicardia, midriasis, las pupilas se encuentran midriáticas ,
covulsiones y datos de focalización. FC 130 lpm, FR 27 rpm, temp ;35.8 ºC, PA 180/100
mm Hg, peso 85 kg, talla 1.56 m. ;73a. Con base en el interrogatorio y los datos clínicos del
paciente, el diagnóstico presuntivo es

 a} intoxicación por cocaína

73. Se trata de Juan Carlos, un paciente masculino de 23 años detenido en el aeropuerto


de la Ciudad de Mexico por Policía Federal por sospecha de transporte ilícito de
sustancias. En el examen con rayos X se evidencia la presencia de aproximadamente 22
cápsulas en cámara gastrica. Al interrogatorio el paciente desconoce el contenido de
dichas cápsulas. Duarante su estancia con los peritos presenta de manera subita, cefalea,
deterioro neurológico, taquicardia, midriasis, las pupilas se encuentran midriáticas ,
covulsiones y datos de focalización. FC 130 lpm, FR 27 rpm, temp ;35.8 ºC, PA 180/100
mm Hg, peso 85 kg, talla 1.56 m.

73b. Con base en lo expuesto en el caso clínico. ¿usted qué tipo de toxíndrome integra en
este paciente

 e} Simpaticomimético

73c. ¿Cuál es el abordaje inicial para intoxicación por cocaína?


 d} Permeabilidad de vía aérea, monitoreo hemodinámico, hidratación
;
73d. Una vez estabilizado el paciente se identifica que durante las últimas tres horas los
flujos urinarios han disminuido y en los paraclínicos se encuentra elevación importante de
CK y CK-MB. ¿Cuál es su sospecha diagnóstica?
 c} Rabdomiolisis

73e. Una vez controlado el evento agudo en urgencias, ¿cuál es la recomendación


estándar que indicaría a su paciente?
 c} Vigilar datos de hipertensión crónica

74. ¿Cuál de los siguientes tipos de catarata se asocia con debilidad muscular?
 a} Catarata policromática múltiple (catarata en árbol de

75. Paciente femenino de nueve años de edad con antecedentes de pubertad y menarca
precoz acude a consulta por presentar desde hace cuatro meses dolor en las
articulaciones de la cadera derecha y en los tobillos. A la exploración física presenta
lesiones hiperpigmentadas (manchas café con leche), con bordes irregulares y dolor a la
palpación de las articulaciones antes mencionadas. Se solicitan estudios de rayos X, los
cuales revelan lesiones poliostósicas en forma de vidrio despulido en la porciones
proximal del fémur y distal de la tibia. ¿Cuál es el diagnóstico más probable&quot;
de esta paciente?

 b} Síndrome de McCune-Albright r

76. Se presenta al servicio de urgencias una paciente de 29 años de edad que se encuentra
en trabajo de parto. La paciente no acudió a ninguna consulta de manejo prenatal.
Durante el trabajo de parto se rompen artificialmente las membranas y es evidente que
existía oligohidramnios. El recién nacido pesa 1.8 kg, y presenta dificultad respiratoria
significativa inmediatamente después de nacer. La radiografía de tórax evidencia la
presencia de pulmones hipoplásicos. ¿Cuál de los siguientes estudios es el más adecuado
de realizar en este paciente?&quot;
 b} Ultrasonido renal

77. Paciente masculino de 70 años de edad con diagnóstico de diabetes mellitus tipo 2
desde hace 25 años, como complicaciones crónicas presenta retinopatía, neuropatía y
nefropatía incipiente. No existe evidencia de aterosclerosis. En este paciente debemos
alcanzar las siguientes metas, con el fin de disminuir la progresión del daño&quot;
renal:
 b} Presión arterial < 140/90 mm Hg, HbA1c < 7% y sin metas LDL especificas
62. Paciente femenina de 23 años, acude a consultar tumefacción en cuello. A la
exploración física se evidencia un&quot;
&quot;nódulo tiroideo fijo, sin adenomegalias. El ultrasonido reporta nódulo tiroideo de 2
cm
con microcalcificaciones en su interior. La gammagrafía con yodo marcado muestra
únicamente un nódulo &quot;&quot;frío&quot;&quot; en la tiroides. Se realiza una BAAF
que reporta papilas
recubiertas de células atípicas, calcificaciones en grano de arena (cuerpos de psamoma) y
núcleos escindidos con aspecto de &quot;&quot;Anita la huerfanita&quot;&quot;.&quot;
&quot;62a. Por epidemiología, ¿cuál es el tumor más probable en esta paciente?&quot;
 d} Carcinoma papilar de tiroides
62b. ¿Cuál es el tratamiento indicado en esta paciente?
 b} Cirugía

62c. Señale la estrategia recomendada para el seguimiento de esta paciente luego de


haberle brindado tratamiento:
 a} Medición de tiroglobulina

62d. Según los datos mencionados en el caso, el pronóstico para la vida de esta paciente
es

 c} Bueno

63. Paciente femenino de 23 años de edad acude a reumatología proveniente del servicio
de infectología con el diagnóstico de coriorretinitis por citomegalovirus en base a lgG
elevada. La paciente presenta desde hace un mes pérdida de agudeza visual en gran
medida en ojo izquierdo y del mismo tiempo de evolución crisis convulsivas tonicoclónicas
generalizadas; signos vitales tomados por pasante de enfermería TA 100/60, FC 90, FR 22,
temperatura

37. Se identifica disminución de intensidad de pulsos carotídeos, carotidinea, soplo


subclavio izquierdo y ausencia de pulsos radiales y cubitales.
63a. ¿Cuál es el diagnóstico más probable?
 d} Probable vasculitis sistémica

63b. Por el grupo etáreo y las características clínicas. ¿Cuál es el diagnóstico específico que
puede tener la paciente?
 a} Arteritis de Takayasu

63c. Durante la evaluación de la paciente en la consulta externa, qué determinación de


parámetro o signo vital está en duda?
 d} Presión arterial

63d. ¿Cuál es el estándar de oro para el diagnóstico de la patología que presenta la


paciente?

 c} Arteriografía

64. Paciente de 54 años de edad, recientemente deportado de la zona Este de Estados


Unidos. Acude a consultar por fiebre de varios días de evolución, mialgia, astenia,adinamia
y artralgia migratoria. TA: 120/80, FC: 85 lpm. Se observa en el muslo eritema circular de
1O cm de diámetro con aclaramiento parcial, borde externo rojo y centro en diana; el
paciente refiere que inició como una pápula roja que se extendió poco a poco hasta
formar la lesión anular actual 64a. Según las características clínicas, ¿cuál es el diagnóstico
más probable

 b} Enfermedad de Lyme

64b. ¿Cuál es el agente causal de la enfermedad?

 a} Borrelia burgdorferi

64c. ¿Cuál es el estudio diagnóstico indicado en este paciente?


 d} Serología
64d. ¿Cuál es el tratamiento recomendado en este paciente?
 b} Doxiciclina

64e. En caso que el paciente presente afectación neurológica por la enfermedad, ¿cuál es
el tratamiento recomendado?
 a} Ceftriaxona

65. Hombre de 55 años que acude por referir pérdida súbita de la visión en ojo derecho al
despertarse en la mañana el día de ayer, refiere pérdida de la visión inferior que se ha
mantenido sin cambios, niega sintomatología agregada. Es hipertenso de ocho años de
evolución,controlado con enalapril 1 x 2, refiriendo buen control. Cursa con dislipidemia
controlada con Lipitor, refiriendo buen control,tabaquismo positivo a razón de 15
cigarrillos/día, niega antecedentes heredofamiliares de otras enfermedades sistémicas. Es
valorado por el servicio de oftalmología reportando: agudeza visual ojo derecho (OD)
20/400 que no mejora con refracción y en 01 20/40 refracción de -0.50 -O.75 a 180
mejora a 20120. Defecto pupilar aferente (DPA) + en OD y reflejos pupilares normales en
01. Se realiza prueba de sensibilidad a color ishihara reportando alterado en OD 0/8 y
normal 01 8/8. El segmento anterior está sin alteraciones. En fondo de OD se observa
papila hiperémica con pérdida de los bordes, hemorragias en astilla, con presencia de
brillo foveolar, relación AV 1/3, con cruces AV, sin lesiones en periferia. 01 con papila
naranja, bordes bien definidos, excavación puntiforme, mácula sin alteraciones. Se envía
campos visuales de Goldman donde se observa un defecto actitudinal OD y en el ojo
izquierdo CV completo. Estudios de laboratorio reportan Hg 15, Hct 46, triglicéridos 300
mg, colesterol 350 mg, HDL 25 mg/dl, VSG normal,su evolución de la AV es tórpida; cuatro
semanas después la AV continúa 20/400 y en el fondo de ojo se observa palidez de la
papila ++ OD.
¿Cuál es el diagnóstico más probable?
 c} Neuropatía óptica isquémica anterior

66. Luisa es una paciente femenina de 65 años de edad con antecedente de


hipotiroidismo hace 1O años en buen apego a levotiroxina. Acude a consulta por
presentar desde hace 7 días visión borrosa, fotofobia, cefalea de moderada intensidad de
tipo punzante sin predominio de horario, así como náusea y vómito. Al interrogatorio
dirigido refiere escotomas. Dentro del exámen físico se indentifica congestión conjuntiva!
y dolor a la presión de globos oculares. Resto de la exploración sin alteraciones. FC 98 lpm,
FR 18 rpm,temp 36.8 ºC, PA 130/85 mm Hg, peso 76 kg, talla 1.56 m

66a. Con base en el interrogatorio de la paciente, el diagnóstico presuntivo es:&quot;


 b} Glaucoma

66b. Al momento de abordar al paciente se realiza tonometría ocular. ¿Cómo se


diagnostica glaucoma de ángulo
cerrado?
 c} Presiónintraocular 50- 80mmHg más celularidad y fiare en cámara anterior

66c. ¿Cuál es el tratamiento para el glaucoma de ángulo cerrado?


 b} Acetazolamida

66d. En el cierre angular intermitente se puede observar al examen oftalmológico:


 d} Sinequias anteriores periféricas
66e. La córnea en el cierre angular crónico se encuentra:
 b} Transparente
67. Mujer de 20 años, sin antecedentes patológicos, inicia padecimiento con diplopía
intermitente hace un mes, y recientemente se acompaña de ptosis unilateral. A la
exploración física se observa fuerza muscular y sensibilidad conservada en todas las
extremidades. Pares craneales sin alteraciones. No refiere cefalea. Se sospecha de
miastenia gravis.

67a. ¿Cuál se las siguientes pruebas es la más específica para el diagnóstico de miastenia
gravis?
 d} Anticuerpos antirreceptor de acetilcolina

1. una paciente de 50 años de edad con diabetes mellitus tipo 2 y antecedente de


sindrome de ovario poliquistico se somete a histerectomia total abdominal para
tratamiento y estadificacion de carcinoma endometrial. Cual de las siguientes
estructuras estan en riesgo de ser dañada cuando las arterias uterinas son ligadas?
e)ureteros

2. cual de los siguientes antiarritmicos prolonga el potencial de accion y disminucion


del flujo de salida celulcar de potasio, sin afectar los canales de sodio ni de
potasio?
a)Ibutilide

3. cual de los siguientes mecanismos de accion corresponde al de la colestiramina y el


colestipol?
a) Union y excrecion de las secreciones biliares
4. Cual de las siguients enzimas es responsable de la activacion de la ciclofosfamida
en el cuerpo humano?
a) Citocromo P450

5. En cual de los siguientes organelos celulares el etanol es metabilizado a


acetaldehido?
c)peroxisomas

6. Cual de las siguientes hormonas gastrointestinales se libera posterior a la


distension gastrica?
e)gastrina

7. Cual de los siguientes receptores es al que se une con mas afinidad la ketamina?
d)NMDA

8. Cual de las siguientes estructuras forma el limite lateral y medial del triangulo
femoral?
d)musculo aductor largo y sartorio

9. Cual de las siguientes enzimas se encuentra involucrada en el produccion de ribosa


a partir de la fuctosa-6-fosfato, pero no en la produccion de NADPH?
b)Transcetolasa

10. Cual de las siguientes arterias se encarga de la irrigacion del colon ascendente?
a)arteria mesenterica superior

11. Cual de las siguientes hormonas gastrointestinales funciona principalmente como


reguladora de los complejos mioelectricos migratorios en el tubo digestivo?
b)motilina

12. Desde el punto de vista embriologico, ¿Cómo se forma la aurícula?


b)mesodermo

13. ¿Cómo está formado el conducto auditivo externo?


a)el canal cartilaginoso constituye un tercio del canal, dos tercios es óseo

14. Cual de los siguientes antiarritmicos actua con mayor selectividad en celulas de
desporalizacion de alta frecuencia?
c)Lidocaína

15. Cuál de los siguientes mecanismos de acción corresponde al de los fibratos?


d)incremento de la actividad de la lipoproteína lipasa

16. Cuál de las siguientes sustancias es capaz de estimular la actividad de la fosfolipasa


C?
c)calcio

17. Cual de las siguientes sustancias favorece la via de la gluconeogénesis por medio
de estimulación alostérica en el hígado?
c)Acetil-coenzima A

18. Cual de los siguientes antiarritmicos tiene su principal mecanismo de accion en


celulas de despolarizacion automatica y de lenta conduccion?
e) Esmolol

19. Cual de los siguientes mecanismos de accion corresponde al de las estatinas?


b) Inhibicion de la enzima hidroxi-metil-glutaril-coenzima A

20. Cual de las siguientes enzimas se inhibe irreversiblemente por el farmaco


pentostatina?
e)Adenosina deaminasa

21. Cual de las siguintes hormonas gastrointestinales funciona como un modulador de


la transmision de los impulsos dolorosos al sistema nervioso?
e)sustancia P

22. Cual es el gastro cardiaco de un paciente que tiene un consumo de oxigeno en


reposo de 200mL/min, contenido arterial de oxigeno de 0.20mL O2 x min,
contenido venoso de oxigeno de 0.15 mL O2 x/mL en sangre?
c) 4L/min

23. Cuál de los siguientes farmacos con actividad analgesica tiene la menor actividad
antiplaquetaria?
b)Rofecoxib

24. Cual de las siguientes hormonas gastrointestinales se libera posterior a la


distension intestinal?
d)Péptido intestinal vasoactivo

25. Cual de los siguientes receptores es al que se une con mas afinidad la naloxona?
c)Mu

26. Cual de las siguientes hormonas gastrointestinales se libera por estimulacion de la


presencia intraluminal de glucosa?
a)Péptido inhibitorio gástrico

27. ¿Desde dónde se extiende el coducto auditivo externo?


b) De la aurícula a la porción externa de la membrana timpánica

28. Cuál de los siguientes antiarrítmicos ejerce su efecto farmacologico únicamente


durante la fase 0 del potencial de acción?
d)Flecainamida

29. Cuál de las siguiente enzimas requiere de la vitamina K como coenzima?


d)Gammaglutamil carboxilasa

30. Cuál de los siguientes mecanismo de acción corresponde al de la niacina?


e)Disminución de la lipólisis perifércia

31. Cual de los siguientes nervios inverca la piel que se encuentra sobre el musculo
deltoides?
a)Nervio axilar

32. Cual de los siguientes mecanisos es la forma principal de trasnporte del dioxigo de
carbono hacia los pulmones?
d)En forma de bicabonato en el suero

33. Cual de los siguientes antiarritmicos actúa tanto en la fase 0 como en la dase 3 del
potencial de acción?
b)Quinidina

34. Cuál de los siguientes mecanismos de acción corresponde al de la ezetimiba?


c)Inhibición de la absorcion de colesterol en el intestino delgado

35. Cuál de los siguientes hallazgos se presenta en caso de lesión del nervio hipogloso
izquierdo?

a)Desviación de la lengua hacia la izquierda durante la protusión


1. ¿Cuál de los siguientes fármacos se utiliza en el tratamiento de la
eyaculación precoz?
e) Fluoxetina.
2. ¿Cuál de los siguientes trastornos de la hemostasia se caracteriza por una
deficiente adhesión plaquetaria al subendotelio que no responde a la
administración de crioprecipitados?
a) Síndrome de Bernand-Soulier.
3. ¿Cuál de los siguientes fármacos se asocia con una respuesta puramente
alfa adrenérgica ocasionando vasoconstricción?
a) Fenilefrina.
4. Paciente masculino de 35 años de edad que es llevado al servicio de
urgencia después de que se encontró con perdida total del estado de altera
en su casa aproximadamente 3 horas. A la llegada al servicio de urgencias
el paciente responde a estímulos dolorosos. La presión arterial es de
100/65 mm Hg; frecuencia cardiaca de 112 latidos/minuto; frecuencia
respiratoria de 10 respiraciones por minuto. La oximetría de pulso se
presenta normal. La enfermera menciona que ha notado un aliento
característico, a lo cual los familiares responden que al encontrarlo
desmayado junto a el se encontraba una botella de alcohol a 70%. Los
estudios de laboratorio muestran la presencia de una brecha osmolar,
cetonemia, pero sin acidosis metabólica. La principal sospecha en este
momento es de una intoxicación por ingesta de isopropanol. ¿Cuál de los
siguientes tratamientos es el mas adecuado para este paciente?
b) Tratamiento de soporte únicamente.

5. Paciente femenino de 63 años con antecedentes de diabetes mellitus tipo 2


de 7 años de evolución en tratamiento con insulina, e hipertensión arterial
sistémica en tratamiento con enalapril, acude a consulta a valoración por
cuadro de tos productiva, diaforesis y fiebre nocturna de 5 semanas de
evolución. A la EF usted nota paciente con edad mayor a la cronológica,
caquéctica, con signos vitales dentro de parámetros normales. Con base en
ellos usted sospecha tuberculosis pulmonar.

5ª. ¿Qué prueba solicitaría primero para realizar el diagnostico?


c) Radiografía de tórax.
5b. ¿A partir de qué cifra se considera positivo el PPD en esta paciente?
b) 10 mm
5c. La forma más comúnmente asociada al desarrollo de la enfermedad es:
c) Reactivación.
5d. El tratamiento de elección es:
c) Rifampicina/isoniazida/pirazinamida/etambutol por 6 meses.
5e. ¿Qué medicamento que se utiliza para la tuberculosis se asocia a
neuritis óptica?
d) Etambutol.

6. ¿Cuál de las siguientes translocaciones cromosómicas se presenta en la


leucemia aguda mieloblástica tipo M2?
e) t(8;21)
7. Paciente masculino de 22 años de edad acude al servicio de urgencias por
exacerbación de asma. Se conocía asmático desde hace ocho años. A
pesar de que se mantenía en un nivel de asma leve intermitente, con
administración de broncodilatador por razón necesaria, el día de hoy
experimento mayor obstrucción de la vía aérea, esto lo llevo a utilizar en
varias ocasiones el medicamento inhalado. Al no observar mejoría decidió
acudir al servicio de urgencias. A su ingreso es colocado bajo tratamiento
con nebulizadores. A pesar de que mejora en cuanto a su obstrucción de la
vía área, presenta nauseas, vomito y debilidad generalizada. El
electrocardiograma muestra la presencia de ondas U. ¿Cuál de los
siguientes medicamentos es el responsable de estos últimos signos y
síntomas?
a) Albuterol.
8. ¿Cuál de los siguientes medicamentos se asocia con la disminución de la
excreción de agua libre como complicación?
a) Clorpropamida.
9. ¿Cuál de los siguientes trastornos se caracteriza por disminución de los
niveles de la hormona luteinizante, normalidad en los niveles de la hormona
foliculoestimulante e incremento de la testosterona en sangre, en
combinación con un bajo conteo de espermatozoides?
a) Administración de andrógenos.
10. Paciente masculino de 39 años de edad se presenta a consulta por
antecedentes de disnea en reposo desde hace cinco días, así como tos no
productiva. Se decide tomar una radiografía de tórax, la cual revela
infiltrados difusos bilaterales; la citometría hemática reporta un valor de
conteo leucocitario de 2000 células/mm3. Al ver los resultados de laboratorio
se interroga al paciente sobre la posibilidad de infección por el virus de la
inmunodeficiencia humana; el paciente acepta, y menciona que no lo
reporto desde el principio porque creía que no era necesario para que
atendieran su padecimiento actual. Se realiza una tinción de metenamina
de plata de esputo, el cual revela al organismo casual, y se inicia
tratamiento especifico dirigido de primera elección. ¿Cuál de los siguient4es
mecanismo de acción es el del medicamento administrado?
e) Bloqueo de la formación de nucleótidos.
11. ¿Cuál de los siguientes antiarrítmicos actúa con mayor selectividad en
células de despolarización de alta frecuencia?
c) Lidocaína.

12. Paciente masculino de 45 años, con antecedentes personales de cólicos


nefríticos de repetición. Con antecedentes heredofamiliares de cáncer de
próstata en su abuelo, su padre y seis de sus ocho hermanos, a edades
tempranas. Comienza hace tres meses con disuria, hematuria, polaquiuria y
dolores óseos. Consulta y es tratado con antibióticos. Al tacto rectal se
detecta agrandamiento prostático, se realiza biopsia que muestra
carcinoma prostático y se decide operarlo realizándose prostatectomía
parcial con vaciamiento testicular bilateral, antígeno prostático especifico
mayor de 100 ng/mL. Continúa tratamiento medico con flutamida; no mejora
su disuria ni polaquiuria. Señale la afirmación correcta.
b) Cuarenta por ciento de los cánceres de comienzo temprano es
hereditario.
13. Paciente masculino de 18 años de edad que se presenta con una
tumoración en la región del tendón de Aquiles. El medico tratante decide
obtener una biopsia de la región, la cual revela la presencia de macrófagos
con alto contenido de colesterol (células espumosas), los cuales se
localizan entre las fibras de colágena. Al interrogatorio de los antecedentes
heredofamiliares el paciente menciona que tanto la madre como el padre
han desarrollado artritis grave en el pasado asociada con la producción de
xantomas; sin embargo, sus padres presentaron este cuadro después de
los 40 años de edad. ¿Cuál de las siguientes partículas en sangre es la que
con mayor probabilidad se encuentra mas elevado en este paciente?
e) Lipoproteinas de baja densidad (LDL).
14. ¿Cuál de los siguientes mecanismos de acción corresponde al de los
fibratos?
d) Incremento de la actividad de la lipoproteína lipasa.
15. Paciente femenino de 69 años de edad se somete a cirugía para resección
de una tumoración solitaria no dolorosa en la región anterior al trago de su
oreja izquierda. A la evaluación histopatológica, se observa un tumor blanco
amarillento bien delimitado, con proliferación de células epiteliales y
mioepiteliales. En el reporte quirúrgico se menciona que durante el
procedimiento se lesiono la división inferior del nervio facial. ¿Cuál de los
siguientes hallazgos podría estar presente con mayor probabilidad en esta
paciente?
e) Debilidad para mover la mitad izquierda del labio inferior.
16. ¿Cuál de los siguientes enunciados referentes a la infección por Giardia
lamblia es correcto?
d)Tan solo la ingestión de 10 quistes de Giardia lamblia puede producir
infección.
17. Sujeto masculino de 30 años que se presenta en urgencias por intenso
dolor tipo cólico de inicio súbito, localizado en flanco izquierdo que se
irradia del abdomen bajo hacia el escroto. Sin antecedentes
heredofamiliares de importancia. Al interrogatorio dirigido refiere polaquiuria
y disuria. A la exploración física se encuentra afebril, hiperestesia en el
ángulo izquierdo costovertebral. Examen general de orina que muestra
hematuria microscópica. Se estudia el sedimento que confirma la presencia
de hematuria y muestra la presencia de cristales bipiramidales
tetragonadales al observaros con lupa binocular. ¿Cuál de los siguientes es
la anormalidad metabólica mas comúnmente asociada con la enfermedad
de este paciente?
a) Hipercalciuria.
18. Una niña de nueve años de edad es llevada a consulta por sus padres por
presentar fiebre desde hace 48 años y malestar faríngeo, con intensa
odinofagia a sólidos, cefalea leve y dolor abdominal difuso. Al interrogatorio
se niega la presencia de tos o dificultad respiratoria. Se decide realizar una
prueba rápida de detección de antígeno estreptocócico, la cual resulta
positiva. La paciente no presenta alergias medicamentosas ¿Cuál de los
siguientes medicamentos es el más adecuado para la paciente?
b) Penicilina V oral a dosis de 250 mg/tres veces al día durante 10 días.
19. Paciente masculino de 39 años de edad que de hace cuatros meses
manifiesta episodios de dolor en hipocondrio con fiebre, los cuales son
autolimitados. Entre los antecedentes de importancia esta el diagnostico de
colitis ulcerativa a la edad de 31 años, ahora en etapa de remisión y bajo
tratamiento con sulfasalazina, 2 g/dia. A la exploración física se encuentra
ictérico y con hepatoesplenomegalia. Los estudios de laboratorio muestran
moderada elevación de las transaminasas hepáticas y bilirrubina, pero una
marcada elevación de los niveles de fosfatasa alcalina. ¿cuál de los
siguientes diagnósticos es el mas probable en el paciente?
e)Colangitis esclerosante primaria.
20. Paciente masculino de 50 años de edad con diagnostico de insuficiencia
cardiaca secundaria a cardiopatía isquémica. Recientemente sufrió una
exacerbación que requirió hospitalización. Posterior a su alta, fue iniciado
con digoxina y dosis elevadas de espironolactona; cinco días después de
su egreso, se presenta con alteración del estado de alerta, así como con
ondas T acuminadas y ensanchamiento del complejo QRS. ¿Cuál de los
siguientes hallazgos podría estar presente en la evaluación de electrolitos
de la orina?
d)Disminución de potasio, elevación de sodio, elevación del volumen
urinario.
21. ¿Cuál de las siguientes características de las vegetaciones valvulares de la
endocarditis infecciosa hace más probable la embolización sistémica?
b)Localizacion mitral.
22. Paciente femenino de 84 años con demencia vascular de varios años de
evolución, con perdida total de la independencia funcional, postrada en
cama desde hace aproximadamente tres años, padece miocardiopatía
dilatada, en tratamiento con digoxina, furosemida, enalapril, acido
acetilsalicílico. Paciente en mal estado general, con lenguaje
incomprensible, con desnutrición extensa, piel pálida, seca e inelástica, con
pliegue persistente, hipotrofia muscular generalizada, mucosas secas,
halitosis, dentadura en mal estado, presenta principios de escaras sacras y
bitrocantereas. ¿Cuáles son factores de riesgo para ulceras por presión?
d)Incontinencia, inmovilidad y desnutrición.
23. ¿Cuál de los siguientes enunciados es el que más adecuadamente
representa las características de la fractura de Barton?
d)Fracturas con desprendimiento del borde dorsal o posterior de porción
distal del radio.
24. ¿Cuál de las siguientes deficiencias enzimáticas se presenta en la
fructosuria esencial?
e)Fructocinasa.
25. ¿Cuál de los siguientes síndromes se asocia con la aparición de
meduloblastoma?
c) Síndrome de Turcot
26. ¿Cuál de los siguientes hallazgos podría estar presente en pacientes con
administración prolongada de amiodarona?
a) Disminución de peso, temblor y dolor torácico.
27. ¿Cuál de las siguientes condiciones no forma parte de los criterios
definitorios del síndrome de inmunodeficiencia adquirida en un paciente con
infección por el virus de inmunodeficiencia humana?
e)Herpes zoster de aparición en más de un dermatoma.
28. Paciente femenina de 65 años, con antecedentes de hipertensión arterial
sistémica controlada con captopril, presenta un trastorno de la marcha y
parestesias en extremidades inferiores. A la exploración física se registra
FC: 68, FR: 15, Temp: 36°C, y el resto sin alteraciones. Usted sospecha
deficiencia de vitamina B12; sin embargo, en los resultados de laboratorio
los resultados son normales, de 250 pg/mL.
28a. ¿Qué estudio es el indicado para confirmar el diagnostico?
d) Cuantificar el ácido metilmalónico y la homocisteína total plasmáticos.
28b. ¿Cuál de las siguientes afirmaciones en cuanto al déficit de vitamina
B12 NO es correcta?
d)Los niveles de ácido metilmalonico o los de homocisteína estarán
disminuidos.
28c. ¿En dónde se absorbe la vitamina B12?
d)Iteon terminal.
28d. ¿Cuál de las siguientes afirmaciones en cuanto al déficit de vitamina
B12 es correcta?
d)En ocasiones hay síntomas neurológicos incluso sin anemia.
28e. ¿Cuál es el tratamiento de la anemia por déficit de vitamina B12?
a) 1 mg IM diario por 7 días, luego semanalmente por 4 o 8 semanas y
después de manera mensual de por vida
29. Paciente masculino de 45 años de edad acude a consulta por presentar
cefalea intensa y continua desde hace 72 horas. Menciona que el dolor es
de localización hemifacial del lado izquierdo. Entre otros síntomas
menciona secreción nasal amarillenta. El único antecedente de importancia
es el de rinitis alérgica estacional. A la exploración física muestra
temperatura de 38°C y dolor a la palpación del seno maxilar. En la
orofaringe no se pueden observar exudados ni adenomegalias. ¿Cuál de
las siguientes opciones es la mejor conducta a seguir en este momento?
d)Fluticasona nasal.
30. ¿Cuál de los siguientes enunciados es el más correcto respecto al cáncer
de ovario?
c)Aquellos pacientes con resección quirúrgica completa tiene mejor
pronóstico de supervivencia.
31. ¿Cuál de las siguientes bacterias representa la segunda causa más
frecuente de meningitis en adultos, y es posible encontrarla colonizado la
faringe de hasta 5% de la población?
b)Neisseria meningitidis.
32. Niño de cuatro años que presenta en cuello, fosas antecubitales y poplíteas
múltiples pápulas eritematosas, milimétricas, aisladas y confluentes que
forman placas en cuya superficie se observan costras hemáticas. La madre
refiere una evolución de 15 días con presencia de prurito intenso durante el
día y la noche. Tiene antecedente de rinitis alérgica desde hace un año y su
padre sufrió de asma bronquial en la infancia. Los exámenes de laboratorio
se reportan dentro de valores normales. ¿Cuál es el diagnostico más
probable?
a) Dermatitis atópica.
33. Paciente del sexo femenino de 22 años de edad con diagnostico de
fenilcetonuria se presenta en trabajo de parto al Servicio de Urgencias
Obstetricas. Debido a que fue diagnosticada a temprana edad, no ha
sufrido signicativas consecuencias por este trastorno genético. Al
interrogatorio la paciencia menciona que su esposo no padece
fenilcetonuria, ni ninguna de los parientes de este. Menciona que nunca
asistió a algún tipo de control prenatal durante su embarazo. El parto
progresa sin eventualidades, sin embargo, la enfermera nota que el recién
nacido presenta microcefalia. Los estudios de laboratorio del recién nacido
demuestran elevación de las concentraciones de fenilalanina. ¿Cuál de los
siguientes enunciados es el que explica de mejor forma la condición del
recién nacido?
b) El trastorno del recién nacido se debe a deficiente control dietético
materno durante el embarazo.
34. Paciente femenino de 77 años de edad cae al bajar las escaleras de su
casa. Al llegar al hospital se observa que tiene la extremidad inferior
izquierda aparentemente mas corta que la derecha y con rotación externa.
¿Cuál de las siguientes opciones es el sitio más probable de fractura?
b)Cuello del fémur.
35. ¿Cuál es el anticonvulsivante de elección en caso de crisis convulsivas
generalizadas de ausencia?
d)Etosuximida.
36. ¿Cuál de los siguientes hallazgos histopatológicos óseos son
característicos de la enfermedad de Paget?
b)Estructuras de huesos lamelar que son parecidas a un mosaico.
37. ¿Cuál es el porcentaje de pacientes con epilepsia de inicio después de los
25 años de edad cuya etiología es la neurocisticercosis o secuelas de esta?
d)50%
38. Mujer de 54 años de edad quien presenta una dermatosis bilateral y con
tendencia a la simetría que se disemina a ambas caras pretibiales. Se
caracteriza por 3 placas infiltradas amarillo-violáceas y esclerodermiformes
de forma irregular y mal delimitadas, que miden entre 5 y 7 cm. La paciente
refiere que sus lesiones iniciaron hace un mes como granitos de color rojo-
violáceo y otros como nódulos de forma redondeada, de bordes no bien
definidos y a veces elevados. Fueron creciendo de forma lenta y excéntrica,
tornándose de color amarillo, con telangiectasias en la superficie. Una de
las lesiones se ulcero, dejando cicatriz y atrofia posterior. A la exploración
física la paciente presenta en las lesiones sensibilidad disminuida al
pinchazo y al tacto leve, hipohidrosis y alopecia parcial. La paciente es
diabética de larga evolución y al parecer lleva un buen control. Niega
antecedentes de traumatismos o de infecciones previas. El diagnóstico es:
b)Necrobiosis lipoídica.
39. Con relación a la excreción renal de ácido úrico:
e)Los procesos implicados en la excreción de uratos se realizan en el
túbulo proximal.
40. ¿En qué paciente se debe de sospechar un angiofibroma nasofaríngeo
juvenil?
e) Paciente masculino de 18 años con epistaxis frecuentes y masa
lobulada en nasofaringe.
41. Paciente masculino de 7 años de edad es llevado a consulta por ojos rojos.
En el interrogatorio resalta la presencia de síntomas estacionales
(primavera y otoño), entre ellos congestión vascular, prurito ocular intenso,
estornudos y lagrimeo ocasional. La madre refiere administración de
antibiótico tópico intermitente, el cual no mejora la sintomatología del
paciente. A la exploración física se observa conjuntiva hiperémica, poco
exudado mucoso, papilas gigantes en la conjuntiva palpebral, y con la
lampara de hendidura bajo fluoresceína no se visualizan ulceraciones en la
córnea. El frotis de exudado reporta eosinófilos.

41a. Con base en las características del paciente. ¿Cuál es el diagnostico


más probable?
a) Conjuntivitis alérgica.
41b. De acuerdo con su sospecha clínica. ¿Cuál es el tratamiento que
indicaría?
c)Antihistamínicos y estabilizadores de mastocitos.
41c. En relación con su sospecha clínica. ¿Cuál es la causa más frecuente?
b) Pólenes estacionales
41d. De acuerdo con su sospecha clínica. ¿Cuál es la complicación que se
puede presentar en este paciente?
b)No hay complicación.
41e. En conclusión. ¿Cuál de los siguientes conjuntos de signos y síntomas
orientó su sospecha diagnóstica?
b)Prurito severo, eosinófilos en frotis y epífora moderada.
42. Paciente masculino de un año de edad es llevado a consulta por sus padres
por presentar desviación y mancha blanca en el ojo derecho de dos meses
de evolución. Producto de la gesta de 2 de 38 SDG obtenido vía vaginal,
Apgar 7/9. No incubadora, convulsiones o administración de oxígeno.
Esquema de inmunizaciones completo. A la exploración oftalmológica se
encuentra exotropia, proptosis, hiperemia conjuntival++, pupila
hiporrefléctica y fondo de ojo con presencia de reflejo blanco.
42a. ¿Cuál es el principal diagnostico a descartar en este paciente?
b) Retinoblastoma.
42b. Los hallazgos iniciales mas frecuentes en esta enfermedad son:
- Leucocoria, estrabismo, glaucoma y uveítis.
42c. ¿Qué estudios complementarios solicitaría para confirmar su
diagnóstico?
- Ecografía y resonancia magnética.
42d. ¿En qué cromosoma se encuentra la mutación genética relacionada
con esta enfermedad?
- Cromosoma 13q14
42e.
43. Un recién nacido aparentemente sano es referido a su consulta por un
sangrado excesivo que requirió transfusión tras una circuncisión de rutina.
Los padres traen consigo estudios de laboratorio, los cuales reportan:
tiempo de protrombina normal, tiempo de tromboplastina prologado y
actividad del factor VIII disminuida.
43a. Con base en las características del paciente, ¿Cuál es el diagnostico
más probable?
-Hemofilia A
43b. ¿Qué hallazgos en los exámenes de laboratorio apoyan su sospecha
diagnóstica?
-PT normal, PTT prolongado, actividad de vWF normal, actividad de factor
VIII disminuida.
43c. En relación con su sospecha clínica, ¿Cuál es la complicación que se
puede presentar a larga plazo?
- Destrucción articular.
43d. De acuerdo con su sospecha clínica, ¿Cuál es el tratamiento indicado?
- Factor VIII
43e. ¿Qué aspecto del cuadro clínico del paciente es relevante mencionar a
los padres?
- Inhibidores de factor.
44. Paciente masculino de 10 meses de edad es llevado del departamento de
urgencias por su madre de 14 años de edad debido a que está muy
irritable. En el interrogatorio la madre refiere que hace 48 hse encontraba
perfectamente bien y que ha sido alimentado con lecha evaporada desde el
nacimiento porque su familia no tiene dinero para comprar formula. Todavía
no inicia con alimentación complementaria. En la exploración física se
observa al paciente muy irritable, con algunas petequias en el cuerpo;
además, toma una posición con las caderas y rodillas flexionadas y llora
cuando es movido. El resto de la exploración es normal.
44a. Con base en las características del paciente, ¿Cuál es el diagnostico
más probable?
- Deficiencia de vitamina C
44b. En relación con su sospecha clínica, ¿Cuál de las siguientes opciones
representa una característica particular del padecimiento?
- Apariencia de vidrio deslustrado en la radiografía.
44c. Respecto a su sospecha clínica, ¿Qué función tiene el cuerpo la
vitamina que se encuentra con deficiencia en el paciente?
- Mantenimiento del material intracelular.
44d. Con base en su sospecha diagnostica, ¿Qué tratamiento le indicaría al
paciente?
- Consumo de jugo de naranja.
44e. De acuerdo con su sospecha clínica, ¿Cuál es el mecanismo de
prevención que se debió realizar para evitar esta patología?
- Jugos de cítricos.
45. ¿Cuál de los siguientes enunciados es correcto respecto de la
ophthalmianeonatorum?
a) Uno de los diagnósticos se puede realizar la observación
46. Niño de un año ocho meses de edad, sin antecedentes de importancia, es
llevado a consulta por historia de tres evacuaciones con hematoquezia. No
presenta dolor abdominal, diarrea o estreñimiento. En la biometría hemática
no se encuentran alteraciones. En relación con la enfermedad que
presenta, ¿Cuál de las siguientes afirmaciones es correcta?
b) Es la anomalía congénita gastrointestinal mas frecuente.
47. Niña de dos años que presenta desde hace seis meses hiporexia,
distensión abdominal, con evacuaciones pastosas y abundantes,
irritabilidad y aparición de equimosis ante el menor traumatismo. Hace un
mes presento fractura tibial patológica. En el examen físico se observa que
la curva de talla para la edad se ha enlentecido, presenta palidez de
tegumentos, aftas en cavidad oral, hipoplasia de esmalte dental, abdomen
muy distendido, hipotrofia muscular en extremidades a nivel proximal y
equimosis en miembros inferiores.
47a. ¿Cuál es el diagnóstico de la paciente?
- Enfermedad celiaca.
47b. ¿De qué forma establecería el diagnostico?
- Anticuerpos antitransglutaminasa.
47c. ¿Cuál de las siguientes condiciones está asociada a la patología que
presenta la paciente?
- Síndrome de Down
47d. ¿Qué trastorno fisiopatológico explica el cuadro clínico de la paciente?
- Linfocitos de la mucosa intestinal sensibilizados al gluten y producción
de anticuerpos.
47f. En relación con la enfermedad de la paciente, ¿Cuál es el mejor
tratamiento?
- Restringir consumo de trigo, avena, cebada y centeno.
48. Paciente masculino de 53 años de edad, con antecedentes de hipertensión
arterial sistémica en tratamiento con IECA, con buen apego a tratamiento, y
etilismo a base de fermentados (vino tinto), acude por presentar dolor
articular de repetición a nivel de la articulación metatarsofalángica del
primer dedo del pie derecho, la sintomatología se manifiesta con
predominio nocturno con aumento de volumen y dolor punzante 10/10, que
no remite al reposo, que en otras tres ocasiones ha remitido de 3 a 10 días
de forma espontánea; a la exploración física se encuentra un nódulo de
Bouchard en la articulación referida, caliente, eritematosa y dolorosa a la
palpación, que limita la movilidad; no se encuentran datos en otras
articulaciones ni datos patológicos agregados. FC 95 lpm, FR 18 rpm, temp.
36.5°C, PA 130/70 mm Hg, peso 95 kg, talla 1.70 kg.
48a. Con los datos del interrogatorio y la exploración física usted
diagnostica:
- Artritis inducida por cristales.
48b. Al realizar una punción articular, usted solicita una cristalografía
esperando encontrar:
- Urato monosódico (MSU)
48c. Para el abordaje diagnostico adicional, el estudio que permite iniciar el
tratamiento adecuado es:
- Depuración de acido úrico en orina de 24 horas.
48d. Ya que el paciente se encuentra en una crisis gotosa, usted decide
iniciar tratamiento con:
- Cual sea de los anteriores.
48e. Adicional al manejo farmacológico, usted sugiere cambios en la dieta y
estilo de vida, ¿Cuáles son los alimentos que usted recomienda evitar?
- Vísceras (hígado, riñón, estomago), refresco y alcohol.
49. Paciente femenino de 36 años emigrada del sureste asiático, comenzó a
presentar tres semanas posteriores a su llegada, astenia, adinamia,
malestar general, episodios febriles, perdida de 4 kg de peso en dos
semanas, anorexia: poco después refiere aumento de volumen de la región
lateral izquierdo del cuello, ligeramente dolorosa al tacto por lo que acude a
la consulta. Niega presencia de tos, expectoración o falta de aire. La
exploración física revelo: Adenomegalia única, localizada en la cadena
lateral izquierda del cuello, ligeramente dolora al tacto, 3 a 5 cm de
diámetro, de consistencia elástica, no adherida a planos profundos. La
radiografía de tórax no muestra alteraciones. Prueba de mantoux: 29 mm,
el resultado de patología de la biopsia del ganglio linfático: cultivo
compatible con Mycobacterium tuberculosis. No es una indicación de
profilaxis para la tuberculosis utilizando isoniazida:
c) Contactos de edad avanzada Mantoux negativos.

50. Paciente masculino de 50 años, inmigrante mexicano en Estados Unidos,


se presenta con tos productiva con esputo sanguinolento acompañada de
sudoración nocturna, pérdida de peso y fatiga de tres meses de duración.
Signos vitales: temperatura: 38°C, presión arterial: 130/85 mm Hg,
frecuencia cardiaca: 90 latidos por minuto, frecuencia respiratoria: 22
respiraciones por minuto. A la exploración física se observa pálido, hay
disminución de ruidos respiratorios en los ápices de ambos pulmones.

La duración del tratamiento de tuberculosis en la población general:


d) Tiene duración Fija

51. Paciente masculino de 58 años con antecedente de carcinoma epidermoide


de pulmón es traído a urgencias por su esposa tras notar que desde hace
una semana presenta poliuria, vómito y confusión progresiva hasta
aparecer letárgico esta mañana. Su electrocardiograma muestra ritmo
sinusal con FC de 110 latidos por minuto e intervalo QT de 0.20 segundos y
tiene un calcio sérico de 13 mg/dL.

¿Cuál es el tratamiento en el manejo de este paciente?


a) infusión intravenosa de solución salina a 0.9%
52. En el siguiente EGC se observa prolongación del intervalo QT (424 ms) y
QT corregido (513 ms).

¿Cuál es la alteración responsable de estos hallazgos?


d) Hipocalcemia

53. Paciente masculino de 67 años de edad, cursa con antecedentes de


etilismo crónico desde la juventud, hace 5 días inició con fiebre > 37°C,
disnea, tos, expectoración verdosa y palpitaciones; a la exploración física
se encuentran estertores y disminución del murmullo vesicular en la región
apical derecha, sin sibilancias, se integra un síndrome pleuropulmonar de
consolidación apical derecha, la saturación por pulsooximetría es de 84%
respirando al aire ambiente. Vive en su domicilio, no ha estado
hospitalizado ni ingerido antibióticos recientemente, niega otros
antecedentes relevantes, FC 105 lpm, FR 24 rpm, temp. 38°C, PA 110/70
mm Hg, peso 84 kg, talla 1.68.

53a.- Con los datos obtenidos del interrogatorio y la exploración física usted
diagnostica:
b) probable derrame pleural derecho

53b.- usted realiza una radiografía de tórax para confirmar el diagnostico, la


cual evidencia una radioopacidad en el lóbulo superior del pulmón derecho,
con los antecedentes y éstos infiltrados apicales, ¿Qué agente etiológico es
el más probable?
c) Klebsiella pneumoniae

53c.- Según la clasificación CRB-65, usted determina que el manejo debe


ser:
a) Ambulatorio

53d.- El tratamiento de primera línea, según las guías de la IDSA, es:


b) Betalactámico + macrólido
53e.- Usted recomienda al paciente que se aplique un esquema de
vacunación bacteriano. ¿Cuál es el de elección?
d) Vacuna contra neumococo

54. Varón de 25 años con dermatosis diseminada que afecta codos, caras
extensoras de antebrazos, glúteos y rodillas, caracterizada por pápulas,
vesículas y placas urticarianas. Las lesiones inician hace alrededor de un
año. Refiere prurito intenso y diarrea crónica. Al interrogatorio dirigido
refiere que lo relaciona con la ingesta de algunos alimentos, en especial
pan, harinas, productos integrales y cerveza. Acudió con un dermatólogo
quien prescribe dapsona con respuesta dramática al tratamiento.

¿Cuáles de los siguientes hallazgos hacen sospechar el diagnóstico de


dermatitis?
a) Pápulas pruriginosas en superficies extensoras

55. Rodrigo es un paciente masculino de 67 años de edad con antecedente de


diabetes mellitus tipo 2, hiperplasia benigna e hipertrigliceridemia en regular
control- Acude al servicio de urgencias por presentar cuadro de fiebre
cuantificada 38°C y dolor en región lumbar derecha de tipo punzante de
aproximadamente 5 días de evolución acompañado de astenia y adinamia.
Al interrogatorio dirigido refiere disuria y tenesmo vesical hace 48 horas,
automedicado con AINE y quinolonas. Al examen físico se encuentra
deshidratado, taquicárdico, abdomen blando depresible, dolor a la
palpación de puntos ureterales superiores, sin datos de irritación peritoneal,
región lumbar derecha tumefacta, discretamente eritematosa, caliente, con
dolor a la palpación, no tolera búsqueda del signo de Giordano por
intensidad de dolor. Extremidades simétricas sin edema, llenado capilar de
2 segundos. FC 110 lpm. FR 22 rpm, temp. 38°C, PA 90/60 mm Hg, peso
97 kg, talla 1.70 m.

55a.- Con base en el cuadro clínico del paciente, ¿Cuál es su problema


diagnóstico?
c) Absceso renal

55b.- Una vez que se sospecha el cuadro de absceso renal, ¿Cuál es el


abordaje inicial del paciente?
b) inicio antibiótico inmediato

55c.- ¿Cuál es el estudio de imagen de elección en el paciente con absceso


perirrenal?
c) Tomografía simple y contrastada de abdomen

55d.- ¿Qué antibiótico es el de primera elección en caso de absceso


perirrenal?
a) Carbapenémico + aminoglucósido + metronidazol
55e.- ¿Cuáles son las secuelas más importantes luego de haber
presentado absceso renal?
c) Esclerosis del riñón afectado

56. ¿Cuál de las siguientes relaciones entre fármacos antituberculosos y sus


efectos adversos es incorrecta?
d) Etambutol: hepatotoxicidad

57. Masculino de 24 años de edad, sin antecedentes patológicos, y múltiples


parejas sexuales; última relación no protegida hace 10 días. Consulta por
dolor orinal, que empezó hace 2 días y lo describe como si fueran navajas.
Así mismo, refiere que siente comezón en el pene y aumento en la
frecuencia urinaria. A la exploración física se observa una secreción verde-
amarilla que sale de la uretra.

57a.- ¿Cuál es su sospecha diagnóstica?


a) Uretritis

57b.- ¿Cuáles son los agentes causales más comunes?


a) Chlamydia trachomatis, Ureaplasma urealyticum, Neisseria
gonorrhoeae

57c.- Estudio de elección para el diagnóstico y clasificación


c) Tinción de Gram

57d.- ¿Cuál es el tratamiento indicado para este paciente?


a) Ceftriaxona

58. Paciente masculino de 28 años acude a urgencias por presentar dolor


intenso en el abdomen, además de nausea y vomito, Refiere que el dolor
comenzó hace 8 h y que, aunque en un inicio fue difuso, al paso del tiempo
se fue localizado hacia el lado inferior derecho del abdomen; asimismo,
comenta que desde el inicio fue tipo cólico. A la exploración física, usted
observa en este paciente facies de dolor en posición antiálgica, hiperalgesia
e hiperbaralgesia, y ruidos intestinales disminuidos de intensidad; en los
primeros laboratorios llaman la atención los leucocitos (12 000) y los
neutrófilos de 80%. EF: 1.80m; 85kg; TA 135/85; Temp: 38°C.

58a.- ¿Cuál es la principal sospecha diagnóstica?


c) Apendicitis aguda

58b.- ¿Cuál es la causa más común de abdomen agudo quirúrgico?


b) Apendicitis

58c.- Son signos negativos al realizar una TC para apendicitis aguda:


a) Aire intraluminal o contraste oral
58d.- El tratamiento de elección es:
b) Quirúrgico

59. Lo siguiente es cierto acerca del absceso hepático amibiano.


e) Las complicaciones pleuropulmonares son las más frecuentes

60. Paciente femenino de 41 años de edad que acude al servicio de urgencias


por dolor torácico, menciona que siempre ha tenido buen estado de salud;
sin embargo, desde hace una semana presenta secreción nasal
blanquecina, ardor faríngeo, tos, y malestar general. Describe que el dolor
inició hace 5 horas y es tipo “desgarrador”, empeora durante la inspiración
profunda, y mejora cuando se inclina hacia delante. A la exploración física
presenta una presión arterial de 110/60 mm Hg y una frecuencia cardiaca
de 90 latidos/minuto. En la exploración física se puede encontrar un frote
trifásico. La radiografía de tórax no muestra ninguna alteración. El
electrocardiograma muestra elevación del segmento ST en la mayoría de
las derivaciones.
¿Cuál es el tratamiento más adecuado para esta paciente?
d) Reposo y aspirina a altas dosis

61. Se trata de José, masculino de 21 años de edad, presentado por sus


padres a Urgencias pues presenta alucinaciones durante la última semana;
al interrogatorio dirigido niegan otros antecedentes relevantes, sin embargo,
las últimas 2 a 3 semanas notaron que estaba retraído, silencioso, poco
interactivo con la familia, comportamiento que no es normal en él. Durante
la entrevista el paciente manifiesta tener indicaciones de un ser “supremo” y
el pensamiento es desorganizado, retraído, ensimismado y su estado
general es descuidado, sucio con la misma ropa durante varios días y no ha
ido a la universidad en la última semana. FC 75 lpm, FR 18 rpm, temp.
36°C, PA 130/70 mm Hg, peso 85 kg, talla 1.74m

61a.- Por los datos obtenidos usted sospecha en:


b) trastorno psicótico breve

61b.- Para el diagnóstico diferencial usted debe solicitar:


a) Punción lumbar y electroencefalograma

61c.- ¿Cuál es la mejor opción terapéutica para este paciente?


d) Antipsicócitos

61d.- ¿Cuál es el pronóstico de este trastorno?


c) Retorno a su nivel de funcionamiento en la mayoría de los casos

61e.- En cuanto a las mujeres afectadas por este trastorno, ¿Cuál de las
siguientes aseveraciones es correcta?
b) El rango de edad más común es de los 25 a 35 años
62. ¿Cuál es el primer punto en el tratamiento de pacientes con hipercalcemia
en casos de amenaza de vida?
a) 200 a 300 mL/h de solución salina junto con furosemida 20 a 40 mg/6 h o
ácido etacrínico 40 mg cada 12 horas

63. Masculino recién nacido, de 4 h de vida, persiste con dificultad respiratoria,


Antecedentes: prematuro de 34 semanas de gestación, peso de 2150g, se
desconoce control prenatal de la madre. A la exploración se encuentra
cianótico, con quejido intenso, retracciones intercostales y subcostales con
aleteo nasal. A la auscultación se detectan estertores finos en ambas
bases. De signos vitales se registra FC: 135 lmp, FR: 70 rpm, Temp: 37°C.
La cianosis no responde con la administración de oxígeno.

63a.- De acuerdo con las características del paciente, ¿Cuál es la


fisiopatología con mayor probabilidad de haber ocasionado el
padecimiento?
c) Deficiencia de surfactante

63b.- De las siguientes manifestaciones clínicas, ¿Cuál es la menos


probable como primera manifestación del síndrome de dificultad
respiratoria?
d) Sibilancias

63c.- ¿Cuál de los siguientes antecedentes es de mucha utilidad para


apoyar la sospecha diagnóstica?
a) Edad gestacional

63e.- ¿Cuál es el tratamiento específico que se debe instaurar en este


paciente?
b) Surfactante endotraqueal

64. Paciente femenino de 50 años de edad acude a consulta debido a que


desde hace algunas semanas presenta torpeza al manejar objetos con las
manos y dolor en ambas muñecas con intensidad de 5 de 10, además de
adormecimiento y a veces sensación de ardor de los dedos 2° y 3°. La
paciente se dedica al hogar y refiere que desde pequeña le gusta lavar la
ropa a mano. Tiene antecedentes de artritis reumatoide en la familia y hace
3 años se le diagnosticó DM2, para la cual no recibe tratamiento alguno por
el momento. A la exploración física no se observan cambios de tamaño ni
de temperatura en las articulaciones de la muñeca y falanges; solo se
aprecia mala calidad muscular en la zona tenar.

64a.- ¿Cuál es su sospecha diagnóstica?


d) Neuropatía por atrapamiento

64b.- Es la prueba clínica más específica para este diagnóstico


a) Tinel

64c.- ¿Cuál es la estructura comprometida en este síndrome?


b) N. mediano

64d.- El tratamiento de primera elección es:


d) Conservador

64e.- En la cara dorsal, ¿Qué sensibilidad cutánea es transmitida por el


nervio mediano?
c) índice y medio

65. Ante un recién nacido de 38 semanas de gestación, obtenido por cesárea,


que inicia de forma precoz con taquipnea, dificultad respiratoria, Prueba de
silverman de 4, exploración pulmonar normal y mejoría con administración
de oxígeno;

¿Qué hallazgos radiológicos serían consistentes con el cuadro clínico que


presenta?
d) Líquido presente en las cisuras y prominencia vascular pulmonar

66. Ante un recién nacido que presenta salida de material liquido amarillo
transparente por el sitio del cordón umbilical sin otro dato en la exploración
física.

¿Qué estudio le sería de mayor utilidad para diferenciar entre los probables
diagnósticos?
a) Determinación del pH del líquido

67. Una mujer de 69 años es llevada por su hija al departamento de urgencias


quejándose de mareo y desorientación. En su historia médica se reporta
hipertensión, hiperlipidemia y diabetes mellitus tipo. Al IPAS se encuentra
orientada en persona, pero no en lugar o fecha. A la exploración física sus
signos vitales son FC 48/min, FR 13/min, T 36.8°C y Pa 86/62 mm Hg
¿Cuál es la terapia inicial más apropiada para este paciente?
c) Atropina

68. Una mujer de 67 años acude a urgencias por disnea. En sus antecedentes
patológicos se refiere hipertensión, diabetes, hiperlipidemia y enfermedad
coronaria, sufriendo un infarto tratado con bypass coronario hace tres años.
Sus medicamentos incluyen metoprolol, lisinopril, aspirina, rosuvastatina e
insulina, aunque refiere no ser muy constante con los medicamentos. Su
ultimo ecocardiograma revela una fracción de eyección de 45%. A la
exploración física se observa una paciente obesa, sus signos vitales
incluyen FC 99/min, FR 21/min, T 37.2°C y PA 193/94 mm Hg. A la
auscultación cardiaca se escuchan S1, S2 y S3. La auscultación pulmonar
revela estertores crepitantes. Se observa distensión venosa yugular y
edema (++) de miembros inferiores. Se realiza electrocardiograma y
enzimas cardiacas, las cuales resultan normales. La placa de tórax revela
edema pulmonar.

¿Cuál de las siguientes afirmaciones es cierta?


c) La presión arterial debe disminuirse inmediatamente con
medicamentos intravenosos, aunque la presión arterial media no
debe disminuirse de 20% de manera inmediata.

69. Una mujer de 68 años acude a consulta por prurito genital desde hace un
mes. En sus antecedentes, se refiere historia obstétrica de G3P2A1 y haber
tenido su último período hace 15 años. En sus antecedentes patológicos,
cuando era joven padeció de condilomas y miomatosis uterina (tratada con
miomectomía hace muchos años). Refiere tener relaciones sexuales
únicamente con su esposo desde hace 32 años. Niega cambios en el flujo
vaginal o sangrado. Como medicamentos sólo toma aspirina. A la
exploración física sus signos vitales incluyen FC 80/min, FR 16/min, T
37.1°C y PA 112/78 mm Hg. Al examen pelvico se observa una lesión
blanquecina ulcerativa no dolorosa de aproximadamente un cm de diámetro
en los labios mayores.

¿Cuál sería el siguiente paso en el manejo de esta paciente?


d) Biopsia

70. Masculino de 68 años, acude a consulta externa para control metabólico de


diabetes mellitus, a la exploración de fondo de ojo usted observa lo
siguiente.

a) Microaneurismas

71. Masculino de 3 años de edad es traído a consulta por sus padres debido a
que en los últimos días han notado una constante desviación del ojo
derecho. Hasta el momento el desarrollo del niño ha sido normal.
71a.- El cuadro más probable de este paciente es:
c) Estrabismo

71b.- ¿Cuál es la causa más prevalente de este paciente?


b) Alteración neuromuscular idiopática

71d.- La prioridad en el tratamiento de este paciente es:


a) Prevenir o evitar la ambliopía

71e.- ¿Cuál de los siguientes No inerva el III par craneal?


a) Oblicuo superior

72. Masculino de 21 años, se le diagnostica enfermedad de Parkinson


idiopática.
El dato clínico que corrobora el diagnóstico de este paciente es:
d) Pérdida de movimientos asociados en la marcha
73. Se trata de paciente masculino de 5 años de edad que cura con diagnostico
de varicela. ¿Cuál de los siguientes medicamentos aumenta el riesgo de
producir síndrome de Reye?
c) Acido de acetilsalicílico.
74. ¿Cuál de los siguientes componentes del virus de la hepatitis B se
correlaciona con la infectividad del paciente?
c) HBeAg

75. Masculino de 34 años es llevado al servicio de urgencias por referir visión


de cuerpos volantes (miodesopsias), destellos luminosos (fotopsias) y
disminución de la visión periférica en alguna zona del campo visual.
Antecedentes: recibir golpe en ojo cuando jugaba frontón de mano
.
Los datos clínicos anteriores confirman el diagnóstico de:
d) Desprendimiento de retina

76. ¿Cuál es el mecanismo de acción de las tetraciclinas?


a) Inhibición de la subunidad ribosomal 30S

77. Paciente masculino de 65 años que acude a la consulta por presentar dolor
dorsal y dificultad para iniciar la micción. Dentro de los antecedentes
heredofamiliares comenta que, de sus cuatro hermanos, dos han
diagnosticado con hiperplasia prostática benigna por lo que fueron
sometidos a resección transuretral prostática. Es por ello que anualmente
acude a su consulta periódica para realización de tacto rectal y antígeno
prostático específico. Pero debido a problemas personales no había venido
a consulta desde hace cuatro años. En esta ocasión al examen rectal se
encuentra próstata dura e irregularmente crecida. Se reporta antígeno
prostático especifico de 15 ng/mL. Ante esta situación se decide realizar
ecografía transrectal con toma de biopsia por sextantes. El reporte
histopatológico es adenocarcinoma bien diferenciado. Dentro de los
estudios complementarios se tomaron radiografías de columna donde se
observaron lesiones osteoblásticas en columna vertebral y en los huesos de
la pelvis.

¿Cuál es el tratamiento de elección?


d) Tratamiento citotóxica
78. ¿Cuál de los siguientes estudios son esperados en un paciente con
insuficiencia renal crónica?
d) Calcio disminuido, fosfato incrementado, fosfato alcalino normal
79. Paciente femenino de 87 años de edad con antecedentes de hipertensión
arterial, hiperuricemia, dislipidemia, cardiopatía isquémica con infarto
inferior antiguo, anemia de enfermedad crónica, isquemia crónica grado IV
en miembros inferiores, prótesis aorto-aórtica por aneurisma de aorta
abdominal intervenido hace ocho años. Ingresa al servicio de urgencias por
dolor torácico retroesternal opresivo, irradiado a región de la espalda. A la
exploración física se encuentra una paciente inquieta por el dolor, presión
arterial de 110/80 mm Hg, frecuencia cardiaca de 110 latidos/min; soplo
sistólico IV/VI en el foco aórtico. En la radiografía de tórax se observa una
imagen de aumento de densidad de borde bien definida, periférica al borde
cardiaco izquierdo, sin borrar la línea media ventricular (por lo tanto,
extracardiaco), sugestivo de aorta descendente.

¿Cuál de los siguientes síntomas es menos común en los pacientes con


aneurisma de aorta torácica en expansión?
e) Disnea

80. Paciente femenino de 28 años de edad, con IMC de 30, TA de 130/85,


glucosa en ayunas de 115, con acné e hirsutismo, triglicéridos de 200, LDL
de 150, HDL de 30. Con oligomenorrea y sin antecedentes de embarazo.
LH incrementada y FSH disminuida. Además de la información previa

¿Qué otros datos NO podría tener esta paciente?


d) Relación LH/FSH < 2.5

81. ¿Cuál de los siguientes hallazgos se puede observar en un paciente con


lesión del nervio peroneo común?
a) Alteración de la sensibilidad en el dorso del pie

82. El primer fármaco que demostró ser efectivo en el tratamiento de la


infección por el virus de la inmunodeficiencia humana fue la zidovudina.

¿Cuál es el mecanismo de acción de este fármaco?


a) Inhibición de la transcriptasa reversa

83. ¿Cuál de los siguientes nervios se encarga de la sensibilidad de la piel del


escroto y la porción medial del muslo?
a) Nervio inguinal
84. Un paciente de 15 años acude a la consulta de urgencias con vomito y
dolor abdominal difuso. Los estudios de laboratorio muestran los siguientes
hallazgos: sodio de 13 mEq/L, potasio 5.3 mEq/L, cloro 95 mEq/L, dióxido
de carbono 9 mEq/L, nitrógeno ureico 35 mg/dL, creatinina 1.2 mg/dL,
glucosa 389 mg/dL, pH arterial 7.2. ¿Cuál de las siguientes sustancias
deberá encontrarse mas probablemente elevada en este paciente?
- Cetonas
85. Paciente de 28 años de edad que se presenta con dolor en cuadrante
abdominal inferior, fiebre y leucocitosis con predominio de neutrófilos. Se
intenta realizar una apendicetomía, sin embargo, el apéndice se encuentra
normal sin datos de enfermedad. Por otra parte, se encuentra que los
ganglios linfáticos que rodean al apéndice están aumentados de tamaño y
con datos de inflamación. ¿Cuál de los siguientes organismos es con mayor
probabilidad el responsable de este cuadro?
- Yersinia enterocolitica
86. Recién nacido producto de un embarazo a termino y parto eutócico. Tanto
uno como 5 minutos después del nacimiento el paciente presenta un
puntaje de Apgar de 9 puntos. Al momento de realizar la exploración física
de rutina se observa fusión de pliegues escrotales y una prominencia
parecida a un pene, pero de menor tamaño. El paciente presenta
hipertensión e hipernatremia. Se solicita un estudio de cariotipo que revela
un genotipo 46,XX. ¿Cuál de las siguientes deficiencias enzimáticas es la
más probable en este paciente?
a) Deficiencia de 11-beta hidroxilasa.

87. Paciente del sexo masculino de 65 años de edad con diagnóstico de


insuficiencia renal crónica con tratamiento sustitutivo. El día de hoy durante
su sesión de hemodiálisis a través de un catéter en su hombro el paciente
se queja de dolor lumbar y fiebre. A la exploración física la fiebre se
confirma, pero con normalidad del resto de los signos vitales. La paciente
se presenta diaforética y con un soplo sistólico 2/4. El sitio de inserción del
catéter se presenta eritoedematoso con elevación local de la temperatura,
pero sin evidencia de exudados. La exploración neurológica es normal. El
resto de la exploración física es irrelevante. La biometría hemática presenta
un conteo leucocitario de 19,000 células/mm³ con 18% de bandas.

¿Cuál de las siguientes conductas NO se encuentra indicada de manera


urgente en esta paciente?
c) Ecocardiograma transtorácico

88. Paciente de 35 años de edad, actor de cine, con pérdida del estado de
alerta, es traído por su novia a la sala de urgencias. Los signos que reporta
el personal paramédico son: frecuencia cardiaca de 90 latidos/min; presión
arterial de 120/70 mm Hg; temperatura de 37°C; frecuencia respiratoria de
10 respiraciones/min. Las pupilas del paciente son de 2 mm
aproximadamente y reactivas, sin alguna otra alteración significativa a la
exploración física. Se toma una gasometría arterial que revela un pH de
7.18 PaCO2 de 63 mm Hg y PaO2 de 75 mm Hg.

¿Cuál es la explicación más probable de la hipoxemia que sufre este


paciente?
a) Hipoventilación
89. Paciente femenino de 40 años de edad que se presenta con diagnóstico de
infección por el virus de la hepatitis B con presencia de antígeno E.

¿Cuál de los siguientes medicamentos debe estar incluido en el tratamiento


de esta paciente?
c) Lamivudina

90. Paciente de sexo femenino de 45 años de edad acude a consulta por malestar
general y edema desde hace semanas. Se reporta previamente sana. Hace un
mes acudió al médico para un examen de rutina, donde no se reportó ninguna
anormalidad. Menciona que ha notado incremento en el número de ocasiones que
micciona durante la noche. A la exploración física presenta leve hipertensión y
edema en la región inferior de las extremidades pélvicas. Los estudios de
laboratorio solicitado reportan:

Entre otros estudios de laboratorio se solicitaron anticuerpos antinucleares,


complemento hemolítico total antiestreptolisina O y factor reumatoide, los
cuales resultaron normales. La biopsia renal revela que alrededor de 80% de
los glomérulos están afectados con imágenes de media luna. La
inmunofluorescencia muestra ausencia de IgG, IgA y complemento en el riñón.
¿Cuál de los siguientes síndromes es probable que presente este paciente?
a. Glomerulonefritis rápidamente progresiva de tipo idiopático

91.- Paciente masculino de 79 años de edad con antecedente de hipertensión


arterial de 12 años de diagnóstico, angina estable y diabetes, se presenta al
servicio de urgencias por disnea de inicio súbito, sin dolor torácico. El paciente
presenta edema periférico desde hace varias semanas, así como ortopnea, dolor
en el cuadrante superior derecho y fatiga progresiva. La exploración física revela
presión arterial de 160/60 mm Hg, frecuencia cardiaca de 90 latidos/min,
frecuencia respiratoria de 26 respiraciones/min y temperatura de 37°C. El paciente
se presenta taquipneico, pulsos reducidos en volumen e intensidad, con una
presión venosa yugular de 12 cm H2O y estertores en ambos campos pulmonares.
A la exploración cardiaca el corazón se encuentra desplazado lateralmente, con
presencia de tercer y cuarto ruidos cardiacos. A la palpación abdominal, el
paciente muestra dolor leve en hipocondrio derecho con hepatomegalia. Las
extremidades se presentan con edema desde los pies hasta las rodillas (3+). Los
estudios de laboratorio son relevantes por creatinina de 1.6 mg/dL, nitrógeno
ureico en sangre de 24 mg/dL, troponina I > 0.05 mg/dL. El electrocardiograma
muestra ritmo sinusal con cambios inespecíficos del segmento ST. Este paciente
se diagnostica con descompensación aguda de insuficiencia cardiaca.

¿Cuál es el fármaco de mato efectividad para disminuir de forma aguda la


precarga del ventrículo izquierdo?
d) Nitratos intravenosos

92.- La colonización por Helicobacter pylori es un factor protector contra:


a) Reflujo gastroesofágico

93.- Paciente masculino en etapa neonatal producto de un embarazo y parto


eutócico. A la exploración física el paciente se presenta con un pequeño defecto
en la porción ventral de la parte distal del pene. El escroto es normal y ambos
testículos se encuentran descendidos y aparecen normales a la palpación

¿Cuál de los siguientes enunciados es correcto con respecto a la


malformación congénita que presenta este paciente?
d) Las lesiones distales tienen mejor pronóstico que las lesiones
proximales

94.- Paciente del género femenino de 33 años de edad que acude al servicio de
urgencias por presentar fiebre y malestar general. Hace aproximadamente 15 días
la paciente acudió a consulta externa por presentar disuria. En esa ocasión decidió
realizar un urocultivo, cuyo reporte demuestra la presencia de > 100,000 unidades
formadores de colonias de Escherichia coli. El antibiograma presentó sensibilidad
a varios antibióticos, y uno de estos fue recetado. La paciente refiere que con el
tratamiento antibiótico los síntomas de disuria desaparecieron; sin embargo desde
hace 48 horas ha sufrido de fiebre no cuantificada y mal estado general. Al
interrogatorio de antecedentes familiares y personales, lo único relevante es que
sufrió una laceración esplénica secundaria a una caída de moto. Al interrogatorio
del padecimiento, la paciente niega cefalea, otalgia, rigidez de cuello, tos, disnea,
odinofagia, dolor abdominal/lumbar, secreción vaginal, náusea, vómito, diarrea,
disuria, hematuria o cualquier otro síntoma a parte de la fiebre y malestar general.
A la exploración física, la paciente se encuentra febril, con una temperatura
corporal de 39°C, sin ningún hallazgo. Utilizando únicamente la información
previamente descrita, responda las siguientes preguntas

94a.- ¿Cuál de las siguientes opciones representa el diagnostico mas


probable en esta paciente?
c) infección bacteriana postesplenectomía

94b.- ¿Cuál de los siguientes conductos sería el más adecuado?


a) Hospitalizar inmediatamente al paciente en el piso de medicina
interna

95.- Una paciente de 30 años antecede tres embarazos, dos partos y una cesárea,
acude al servicio de urgencias dos días después de haber dado a luz por
presentar dolor hipogástrico de intensidad progresiva asociado a fiebre, diaforesis
y escalofríos. A la exploración física se encuentra taquicárdica, taquipneica, con
temperatura de 40°C, presión arterial de 100/6 mm Hg, se palpa útero suave a la
altura de la cicatriz umbilical doloroso a la palpación, asi como dolor a la
movilización del cuello uterino. Usted realiza el diagnóstico de endometritis.

95a.- ¿Cuál de las siguientes condiciones se asocia con más frecuencia al


desarrollo de sepsis puerperal?
d) cesárea

95b.- ¿Cuál es el tratamiento adecuado para la endometritis posparto?


e) Todas las anteriores (Cefoxitina, Gentamicina + piperacilina,
CLindamicina + aminoglucósido, ceftriaxona)

96.- En el servicio de urgencias se presenta un paciente del género masculino de


19 años de edad con antecedentes de diabetes mellitus tipo 1. El paciente fue
llevado por los familiares por presentar incremento importante en la diuresis (los
padres del paciente ya tienen experiencia en el manejo de la enfermedad), Usted y
los familiares del paciente sospecha la presencia de cetoacidosis diabética. A la
exploración física los signos vitales son temperatura 36°C, presión arterial 100/60
mm Hg, frecuencia cardiaca 105 latidos/min. A la inspección general el paciente
presenta evidentemente sequedad de mucosas. El reporte de laboratorio es el
siguiente:
● Glucosa, 899 mg/dL

● Creatinina, 1.25 mg/dL


● Nitrógeno ureico en sangre, 33 mg/dL
● Bicarbonato 15 mEq/L
● Cloro 97 mEq/L
● Potasio 4.5 mEq/L
● Sodio 128 mEq/L
Utilizando únicamente la información previamente descrita, responda las
siguientes preguntas:

96a.- ¿Cuál de los siguientes enunciados respecto a la concentración del


sodio del paciente es el más adecuado?
a) El nivel de sodio es normal

96b.- ¿Cuál de las siguientes conductas terapéuticas es la más adecuada


en este paciente?
c) iniciar infusión de insulina

96c.- Una gasometría arterial demuestra que el paciente presenta un pH de


7.0 ¿Cuál de las siguientes opciones describe la conducta más adecuada?
a) Administración de una ámpula (44 mEq) de bicarbonato de sodio
al 7.5% en 200 mL de solución estéril y administrar a 200 mL/h + 15
mEq/ de KCI

96d.- Eventualmente, la concentración de glucosa sérica disminuye a 240


mg/dL, ¿Cuál de las siguientes es la más adecuada?
a) Cambiar la hidratación con solución salina a solución glucosada al
5% y mantener la infusión de insulina

97.- Se trata de paciente femenino de 29 años de edad con diagnóstico de


embarazo de 32 semanas de gestación. En su familia existe un antecedente de
gemelos monocigotos. Refiere que durante el primer trimestre presentó
hiperémesis y después tuvo diagnóstico de preeclampsia. Se encuentra bajo
tratamiento con alfa metildopa. A la exploración física usted percibe un fondo
uterino de mayor tamaño para la edad gestacional y al realizar la segunda
maniobra de Leopold siente dos dorsos, uno a la derecha y otro a la izquierda; a la
auscultación se perciben dos latidos cardiacos con frecuencias distintas entre sim
en un embarazo gemelar monocigótico.
¿Cómo se encontrará la placenta y el amnios con mayor frecuencia?
b) Monocorial-biamniótica

98.- Ante un recién nacido, hijo de madre que presentó infección por
citomegalovirus durante el primer trimestre de gestación, usted esperaría todas las
siguientes manifestaciones clínicas Excepto:

b) Hipoacusia neurosensorial bilateral

99.- Mujer de 18 años de edad, que presenta ciclos menstruales regulares, de 28


días, eumenorreica. Usted sabe que el ciclo ovárico es muy importante y consta
de tres fases: folicular, ovulación y lútea.
¿Qué dato es cierto sobre la capa granulosa?
a) Tiene receptores de FSH

100.- Prevalencia de diabetes mellitus tipo 2 en México de acuerdo con ENSANUT


2006:
a) 7.5%

101. Niño de ocho años de edad que presenta demantosis diseminada, bilateral y
simétrica que afecta ambas palmas, plantas y cavidad oral constituida por maculas
en forma de balón de futbol, pápulas eritematosas, vesículas y erosiones. El
diagnóstico es enfermedad mano-pie-boca. ¿Cuál es el tratamiento y el
pronóstico?
d) La enfermedad no es grave y requiere tratamiento sintomático.
102. Masculino de 5 años, llevado a servicios de urgencias por la madre quien
refiere que el dia de ayer el menor presento malestar general, no quiso comer y
refirió dolor de cabeza con temperatura de 37.6 °C. El día de hoy se le
encontraron en la cara anterior del tórax cinco vesículas aperladas, redondeadas
por un halo eritematoso, con liquido transparente en su interior. La familia refiere
preocupación porque hay perro y gato en la casa y temen que sea un contagio la
causa del padecimiento. Exploración física: se le encuentran aún más vesículas
umbilicadas en el tronco y algunas vesículas en el paladar, se aprecia que el
menor presenta escozor, T 37.7 °C.
102a. el cuadro clínico de este paciente es producido por:
- Varicela
102b. Se solicita biometría hemática, esperando encontrar:
- Leucopenia
102c. El menor tiene una hermana de 21 dias de nacimiento, la principal
recomendación es:
- Administrar gammaglobulina al neonato.
103. ¿En qué momento se observa el efecto máximo de la insulina glargina?
e) No tiene
104. Acude a consulta externa un paciente de 80 años de edad quien ha
presentado disminución de la percepción auditiva. Al momento de la exploración
física el paciente presente mayor incapacidad para detectar sonidos de alta
frecuencia. ¿Cuál de los siguientes diagnósticos es el más probable?
a) Presbiacusia.
105. Paciente masculino de tres años de edad es traído a urgencias por presentar
fiebre desde hace siete dias de evolución acompañada de conjuntivitis no
purulenta, la madre refiere que hace dos días se agregó edema doloroso de las
manos y pies. A la exploración física se observa preescolar irritable con labios
eritematosos, secos y agrietados, adenopatías no dolorosas y exantema
maculopapular morbiliforme, edema de las manos y los pies con descamación de
palmas y plantas.

105a. ¿Cuál es la complicación más grave que puede presentar el


paciente?
b)aneurismas coronarios
105b. ¿Cuál es el tratamiento más adecuado?
c)Gammaglobulina por vía intravenosa.
106. Paciente femenino de 38 años de edad, con embarazo de 17 semanas de
gestación acude a consulta de urgencias porque desde hace varios dias inicio con
nauseas, la cual h ido incrementado en intensidad hasta llegar al vomito. También
refiere sangrado transvaginal intermitente semejante a uvas, con poco dolor en
hipogastrio. A la exploración física se observa fondo uterino de 8cm de longitud
desde la sínfisis del pubis, sin presencia de actividad uterina, no se palpan partes
fetales y no se logra encontrar el latido cardiaco fetal. Se realiza ultrasonografía y
se observa imagen de copos de nieve y el laboratorio reporta una elevación de la
hormona gonadotropina coriónica humana fracción beta. De acuerdo al
diagnostico mas probable de la paciente. ¿Qué conducta terapéutica NO estaría
indicada?
b) Colocación de dispositivo intrauterino.
107. Paciente masculino de 54 años que se sometió hace poco tiempo a un
trasplante de córnea. Doce semanas después de la cirugía, exhibe alteraciones
neurológicas, principalmente caracterizadas por demencia y movimientos
mioclónicos. Es evidente que muestra alteraciones de la marcha, caracterizadas
por ataxia. Entre otros hallazgos se observa nistagmo, disartria, ataxia natural y
ataxia de extremidades. Todos estos datos son indicativos de:
d) Enfermedad de Creutzfeldt-Jakob
108. Paciente masculino de 39 años de edad con dolor en el brazo desde varios
años. El paciente menciona que debido a su trabajo, recientemente ha cargado
cajas pesadas y desde entonces el dolor ha empeorado, con su presencia
ocasional de parestesias en el quinto y tercer dedo de la mano. La región dolorosa
se localiza en la porción medial del brazo y antebrazo, así como en la porción
cubital de la mano. En general, la fuerza muscular en la extremidad derecha se
encuentra reducida en comparación con la del lado izquierdo. Además, existe
debilidad a los movimientos de oposición y aducción del primer dedo. A la
inspección es evidente la atrofia de las eminencias tenar e hipotenar. ¿Cuál es el
sitio donde más probablemente se encuentra la lesión nerviosa?
a) Tronco inferior del plexo braquial.
109. Un paciente de 40 años presenta hemianopsia homónima contralateral con
conservación de la visión macular posterior a la resección de un tumor en el lóbulo
occipital. ¿Cuál es el sitio mas frecuente en el que se encuentra la lesión del
paciente?
e) Fisura calcarina
110. Un atleta de alto rendimiento se presenta a consulta por debilidad en la
extremidad inferior derecha e inestabilidad de la marcha desde hace dos meses.
El paciente refiere que su sintomatología se ve aumentada durante el ejercicio. A
la inspección, el paciente muestra atrofia de la porción medial del muslo y
debilidad para realizar la aducción de este. ¿Cuál es el nervio más probablemente
afectado?
d) Nervio obturador
111. Un paciente de 22 años de edad acude al servicio de urgencias debido a que
ha presentado dificultad para abrir y cerrar su mandíbula posteríos a una
intervención dental hace tres días. Existe sospecha de que durante a la
intervención dental se lesiono la rama mandibular del nervio trigémino. ¿Cuál de
los siguientes músculos se encuentra inervado por la rama mandibular del nervio
trigémino y es responsable de abrir la mandíbula?
a) Pterigoideo lateral.
112. Paciente femenino de 85 años de edad con antecedente depresión acude al
servicio de urgencias por debilidad y mareo. Al revisar el expediente clínico se
observa que toma furosemida 20 mg/día para reducir el edema de miembros
inferiores. Menciona que hace poco agrego a su lista de medicamentos la
hidroclorotiazida para el manejo de la presión arterial. Al interrogatorio niega
fiebre, escalofríos, nausea, o vomito; menciona que a ultimas fechas ha tenido
mucha sed, por lo que ha ingerido grandes cantidades de jugo de manzana. A la
exploración física la presión arterial en decúbito es de 100/50 mm Hg, y sentada
de 80/44 mm Hg, con un peso de 60 kg. La exploración pulmonar es normal. Los
estudios de laboratorio revelan osmolalidad sérica de 260 mOsm/kg, sodio sérico
de 125 mEq/L, potasio sérico de 3.4 mEq/L, creatinina sérica de 0.8 mg/dL. El
examen general de orina revela una concentración de sodio urinario de 50 mEq/L,
y la osmolalidad urinaria de 200 mOsm/kg. ¿Cuál de los siguientes diagnósticos es
el más probable en esta paciente?
c) Hiponatremia secundaria a la administración de hidroclorotiazida
113. Se presenta un paciente que, al mirar hacia el lado izquierdo, el ojo derecho
no puede aducirse, y el ojo izquierdo presenta nistagmo horizontal. Al realizar una
prueba de convergencia, esta se encuentra conservada. ¿Cuál es el sitio más
probable de la lesión?
a) Fascículo longitudinal medial derecho
114. Masculino de 66 años con diagnostico de DM tipo 2 gota, hipertensión,
hiperlipidemia e insuficiencia renal crónica, con creatinina de 2.8 mg/dL. Acude a
urgencias por dolor torácico; el EKG muestra elevación del segmento ST y es
llevado de inmediato a la sala de hemodinámica para angioplastia primaria. Antes
de iniciar se le administra solución salina para prevenir la nefropatía por medio de
contraste. Al finalizar el procedimiento, el registro de enfermería indica que se
administraron alrededor de 140 mL de medio de contraste no yodado de baja
osmolaridad, así como hipertensión transitoria. En los siguientes días desarrolla,
coloración purpura en los dedos del pie derecho, y hemorragia gastrointestinal. La
creatinina sérica incremento a 6 mg/dL, requirió hemodiálisis. ¿Cuál de las
siguientes opciones es la causa de agudización de la insuficiencia renal en este
caso?
a) embolización de colesterol desde la aorta
115. Paciente masculino de 44 años de edad que acude al servicio de urgencias
con cólico renal. La radiografía de abdomen revela un lito de 1.5 cm. ¿Cuál de los
siguientes enunciados es el más correcto respecto al padecimientos del paciente?
e) El manejo conservativo mediante hidratación y analgésicos no proporciona el
mejor pronostico en este paciente.
116. Paciente masculino de seis años acude a urgencias por presentar fiebre de
bajo grado, edema y dolor de rodillas, dolor abdominal tipo cólico y hematuria. A la
exploración física se observa un exantema purpúrico y prurítico que cubre de los
pies a la cintura. Los exámenes de laboratorios muestran sangre oculta en heces,
leucocitosis y plaquetas de 200 000/mm3 , el urianalisis revela cilindros
eritrocitarios y proteinuria leve.
c) Vasculitis de Henoch-Schönlein.
117. Paciente masculino de 63 años de edad acude a consulta desde hace una
semana presenta disnea durante el ejercicio, tos y dolor de torácico de
características pleuríticas. La exploración física revela distensión venosa yugular,
ruidos cardiacos velados y edema leve bimaleolar. La radiografía de tórax muestra
crecimiento de la silueta cardiaca. El reporte del ecocardiograma la presencia de
derrame pericárdico circunferencial. Se realiza una pericardiocentesis guiada por
ecocardiografía, la cual remueve casi la totalidad del derrame; la evaluación
citológica revela celular de adenocarcinoma. ¿Cuál de las siguientes oraciones NO
es verdadera respecto al padecimiento de este paciente?
b) Se debe realizar con urgencia una pericardiectomía quirúrgica total.
118. ¿Cuál de los siguientes agentes fibrinolíticos presenta actividad en su forma
uní y bicatenaria?
b) Activador tisular del plasminógeno.
119. ¿Cuál de los siguientes medicamentos se caracteriza por presentar aumento
de la permeabilidad capilar como complicación?
a) Interleucina 2
120.- ¿Cuál de los siguientes esquemas profilácticos es el más adecuado en
paciente que se someterá a una colecistectomía electiva en un paciente con
estenosis mitral?
a) No se requiere ningún tipo de profilaxis antibiótica

121.- Un niño de cuatro años es llevado a urgencias por un cuadro clínico


desencadenado al entrar en contacto con un insecticida derivado del ácido
fosfórico.

¿Qué manifestaciones constituyen el cuadro clínico de intoxicación aguda


por organofosforados?
a) Miosis, broncoespasmo, sudación, lagrimeo, vómito, diarrea e
incontinencia urinaria

122.- Un varón de 68 años de edad acude a consulta por palpitaciones y disnea


que iniciaron hace tres días. Refiere que las palpitaciones ya habían ocurrido en
varias ocasiones en las últimas semanas, pero como duraban sólo unos segundos
y no le daban molestias, no le dio importancia. Esta semana ha sentido dificultad
para respirar al caminar hacia su trabajo, que se resuelve al reposo. En su historia
médica refiere hipertensión e hiperlipidemia. Sus medicamentos incluyen
hidroclorotiazida y rosuvastatina. A la auscultación cardiaca se escucha un ritmo
rápido e irregular, y el resto de sus signos vitales consisten en frecuencia
respiratoria de 19/min, presión arterial de 126/83 mm Hg, temperatura de 37.2 C y
saturación de oxígeno de 90%. No se observa distensión venosa yugular o edema
periférico. Las enzimas cardiacas se encuentran en límites normales. En los
campos pulmonares se auscultan ligeros estertores. Se le solicita un
electrocardiograma, que muestra:

122a.- ¿Cuál es el diagnóstico de este paciente?


c) Fibrilación auricular

122b.- ¿Cuál es la incidencia del evento vascular cerebral isquémico por


año en pacientes ancianos con fibrilación auricular sin tratamiento
anticoagulante?
a. 5%
122c.- Todas las siguientes son causas de fibrilación auricular, excepto
e) Hipotiroidismo

122d.- ¿Cuál sería el manejo inicial de este paciente?


c) Diltiazem y heparina

123.- ¿Cuál debe ser la duración de la anticoagulación en el contexto de síndrome


de anticuerpos antifosfolípidos y eventos vasculares cerebrales de repetición?
a) Permanente

124.- Paciente masculino de 53 años de edad que refiere inestabilidad de cuatro


años de evolución, acompañado con pérdida auditiva progresiva unilateral
derecha, con problemas para la discriminación del habla. Refiere cefaleas que se
han intensificado. El paciente acude con el ORL quien revela una hipoacusia
neurosensorial derecha y durante la realización de las pruebas calóricas, una
hiporreflexia del oído derecho. ¿Cuál es tu diagnóstico presuncional?
c) Schwannoma vestibular

125. Acude a consulta paciente masculino de 55 años por dolor en el primer dedo
del pie izquierdo. El dolor inició ayer por la noche y ha ido empeorando con el
paso de las horas. Refiere que le ha ocurrido un cuadro similar cuatro veces en el
último año. En las ocasiones anteriores acudió al servicio de Urgencias, en donde
le dieron un medicamento para el dolor, pero no recuerda cuál. Niega
antecedentes de importancia. A la exploración física tiene fiebre de 38 ºC, FC: 90,
TA: 130/80. El primer dedo del pie izquierdo se encuentra eritematoso, doloroso a
la palpación, caliente y con ligero edema. Con el cuadro clínico mencionado se le
diagnostica un ataque agudo de gota.

125a.- ¿Cuál sería el medicamento de elección en este momento para el


paciente?
d) AINE

125b.- ¿Cuál es el estándar de oro para el diagnóstico de gota?


b) Identificación de cristales de urato monosódico en líquido sinovial

125c.- Se decide practicar una artrocentesis del paciente. ¿Qué se observa


en el líquido sinovial?
b) Cristales con birrefringencia negativa, en aguja, amarillos cuando
están paralelos al eje y azules si están perpendiculares

125d.- ¿Cuál es la relación hombre: mujer en cuanto a la epidemiología de


gota?
d) 9:1

125e.- La siguiente es una indicación para iniciar profilaxis con terapia


antihiperuricémica crónica:
C) Tofos
126. Recién nacido de 48 horas de vida, producto de primera gestación, con
diagnóstico de macrosomía y polihidramnios. Durante la atención inicial no fue
posible pasar la sonda nasogástrica y se ha observado que tiene sialorrea y con
las tomas de comida presenta ahogamiento. De acuerdo con su sospecha
diagnóstica, señale cuál de los siguientes defectos estructurales es el más
frecuente en estos pacientes:
b) Atresia proximal esofágica y fístula distal traqueoesofágica

--------------------------------------------------------------------------------------------------------------
--------------------------------------------------------------------------------------------------------------
1. Se presenta paciente gesta 1, para 0, aborto 0, cesárea 0, en trabajo de
parto desde hace 4 horas con aproximadamente tres contracciones cada 10
minutos con duración de 30 segundos. Se puede palpar un cuello casi
borrado en su totalidad, con cerca de 4 cm de dilatación con membranas
rotas y presentación cefálica. La paciente acude con pulso regular y
frecuencia de 80 latidos por minuto sin dolor abdominal, ha acudido a
control obstétrico desde el principio y recuerda con precisión la fecha de su
ultima menstruación, determinándose que se trata de un embarazo de 38
semanas de gestación. El examen general de orina y la biometría hemática
son normales y amniocentesis demuestra una índice lecitina/esfingomielina
> 2. ¿Cuál es la conducta a seguir?
b) Manejo expectante y espera por dilatación completa.

2. Un recién nacido de sexo masculino, con antecedentes de síndrome de


Down, presenta datos de obstrucción intestinal al nacimiento, con imagen
de dientes de sierra a nivel de sigmoides en estudio baritado. Respecto a
este caso clínico, señale la opción FALSA.
c) La enterocolitis es infrecuente como complicación de esta enfermedad.
3. Niña de un año ocho meses de edad con antecedentes de gastroenteritis
viral de dos semanas, resuelta sin complicaciones, que inicio hace dos días
con flatulencia y evacuaciones diarreicas acuosas y abundantes. Se
encuentra en el examen físico con aumento de los ruidos peristálticos y
eritema perianal. En relación con la enfermedad que presenta, ¿Cuál es el
tratamiento indicado?
E) Retirar alimentos con lactosa de la dieta.
4. Paciente masculino de 86 años, con antecedentes de fibrilación auricular,
acude a urgencias por presentar disnea progresiva de dos meses de
evolución, acompañada de astenia, adinamia y perdida de peso no
cuantificada. A la exploración física se encuentra afebril y se sospecha
derrame pleural derecho. La radiografía de tórax confirma el derrame
pleural y un ensanchamiento mediastínico sin afectación de parénquima
pulmonar, los análisis de sangre no muestran alteraciones. La
toracocentesis mostro un liquido espeso de aspecto lechoso con 1 400
leucocitos/mm3 con 10% de neutrófilos y 90% de linfocitos, proteínas 49 g/L,
LDH 262 U/L, glucosa 10.2 mmol/L, adenosina deaminasa 16 U/L,
colesterol 1.75 mmol/L y triglicéridos 12.8 mmol/L. la citología y el cultivo
fueron negativos. La intradermorreacción de Mantoux fue negativa. La
tomografía toracoabdominal mostro múltiples masas adenopáticas en el
mediastino y derrame pleural bilateral de predominio derecho. Se realizo
una puncion-aspiracion con aguja fina de la masa mediastínica. Los
resultados de patología mostraron lesiones granulomatosas crónicas
compatibles con tuberculosis tanto en pleura como en pulmón.
En relación a la tuberculosis miliar, señale la aseveración falsa:
B) Los tubérculos coroideos son frecuentes
5. Paciente masculino de 46 años acude a consulta por haber presentado
esputo hemoptoico que se autolimito en cuatro días. Dentro de sus
antecedentes destaca el diagnostico en tuberculosis pulmonar a los 26
años que se trato de manera correcta, quedando lesión cavitaria residual de
pared fina.
En la tuberculosis pulmonar la radiografía de tórax es útil porque:
B) Complementa el diagnostico de tuberculosis.
6. Paciente masculino de 45 años con antecedentes de hipertensión arterial
en tratamiento con captopril, hidroclorotiazida y nifedipina acude a
urgencias tras una semana de no haber tomado ninguno de sus
medicamentos por razones económicas. El paciente refiere que desde hace
tres días ha tenido cafelea progresiva, fosfenos, nausea e irritabilidad. A la
exploración física se encuentra agitado y desorientado, hipertenso con
presión arterial de 210/130 mm Hg y el examen de fondo de ojo revela
papiledema y hemorragias, no hay déficit neurológicos focales.
¿Cuál de las siguientes aseveraciones es falsa con respecto al tratamiento
de la emergencia hipertensiva?
B) El diazoxido cruza la barrera hematoencefálica ejerciendo un efecto
directo sobre la circulación cerebral.
7. Un paciente de 32 años con antecedentes de lupus eritematoso sistémico
acude a su ginecólogo por abortos espontáneos recurrentes. Ha tenido tres
embarazos, los cuales terminaron en abortos antes de la semana 10 de
gestación. Los padres no tienen ninguna alteración cromosómica y a la
madre se le realizo un histerosalpingograma y USG vaginal, que resultaron
normales, además de cultivos vaginales que fueron negativos. Presenta un
VDRL positivo.
7a. ¿Cuál de los siguientes estudios hubiera servido para sospechar el
riesgo de abortos espontáneos?
a) Anticoagulante lúpico
7b. ¿Cuál de las siguientes manifestaciones clínicas es la más común en el
síndrome de anticuerpos antifosfolipídicos (SAAF)?
b) Trombosis venosa profunda
7c. En cuanto a las manifestaciones cardiacas en el SAAF, ¿Cuál es la más
común?
a) Lesiones valvulares
7d. ¿Cuál es el tratamiento en pacientes con SAAF asintomático?
a) Sin tratamiento o aspirina a dosis bajas.
7e. ¿Cuál de los siguientes no es un criterio de SAAF catastrófico?
b) Confirmación de trombosis venosa recurrente
8. Se agrega ampicilina al tratamiento empírico de una meningitis cuando se
sospecha que el agente causal puede ser:
B) Listeria monocytogenes
9. El siguientes es un método adecuado para el diagnostico de la fiebre
tifoidea:
B) Mielocultivo en la segunda semana.
10. Paciente masculino de 55 años de edad que sufrió un infarto con elevación
del segmento ST tratado mediante fibrinolíticos y heparina de bajo peso
molecular. Después del tratamiento menciona que el dolor torácico ha
desaparecido, y el electrocardiograma muestra resolución de la elevación
del segmento ST. Tres días después del infarto inicia con dolor torácico
grave, descrito como “desgarrante”. ¿Cuál de las siguientes opciones
explica el cuadro del paciente a tres días del infarto?
E) Todas las opciones anteriores pueden ocasionar el cuadro que ahora
muestra el paciente
11. Es el mecanismo fisiopatológico por medio del cual el hipercortisolismo
induce hipertensión arterial:
a) Aumento de volumen circulante por la retención de Na
12. Un hombre de 91 años que acude a urgencias por disnea de dos días de
evolución. En su historia clínica se refiere a hipertensión, hiperlipidemias,
hiperplasia prostática benigna y enfermedad coronaria, habiéndose
sometido a un bypass coronario. Refiere historia de tabaquismo de 12
paquetes/año, habiendo dejando de fumar hace 30 años. Sus signos vitales
incluyen FC 98/min, FR 23/min, T 36.7 C y PA de 140/88 mm Hg. A la
exploración física se observa distensión venosa yugular, hepatomegalia y
edema 3+ de extremidades inferiores. Los campos pulmonares se
auscultan con murmullo vesicular. Electrocardiograma muestra datos
hipertrofia ventricular izquierda. Se realiza un ecocardiograma. Se realiza
una radiografía de tórax que muestra de atrapamiento de aire. No se
observa enfermedad valvular. ¿Cuál de las siguientes patologías es la
causa de estos síntomas?
a) Falla cardiaca diastólica
13. Una mujer de 72 años de edad acude al departamento de urgencias por
disnea y palpitaciones. En sus antecedentes patológicos se refiere EPOC
hipertensión, hiperlipidemia y enfermedad coronaria. Habiendo sufrido un
infarto hace seis años tratado con stent. Sus medicamentos incluyen
bromuro de ipatropio, metoprolol, captopril, simvastatina y oxigeno en casa,
el cual mantiene a 3 L/min. A la exploración física sus signos vitales
muestran pulsos rápidos e irregulares, FR 31/min, T 38.2 C, PA 143/88 MM
Hg y saturación de oxigeno de 82%. A la auscultación pulmonar se
escuchan roncantes espiratorios. Se realiza un electrocardiograma, el cual
se presenta a continuación:

¿Cuál es el diagnostico más probable de esta paciente?


E) Taquicardia multifocal paroxística.
14. Femeninode 17 años es traída al servicio de urgencias por sus familiares,
quienes que la encontraron excitada y se les informo que fue expuesta a
una sustancia toxica. Exploración física: taquicardia, hipertensión arterial,
taquipnea y midriasis.
La sintomatología de esta paciente es consecuencia de la administración
de:
D) Cocaína.
15. Es llevado al servicio de urgencias lactante menor que muestra una
fotografía.
La complicación mas común en esta paciente es:

B) Candidosis
16. ¿Cuál de las siguientes translocaciones cromosómicas se asocia con el
linfoma de celular del manto?
E) t(11:14)
17. Femenino de 24 años, cursa con embarazo de 37 semanas atendida en
sala de urgencias por presentar perdida del estado de alerta posterior a
crisis convulsivas tonico-clónicas. Antecedentes: G-2, signos vitales con TA
170/120 mm Hg, Fc 95x´, reflejos osteotendinosos aumentados, FCF de
132x´y edema importante de miembros inferiores.
La entidad patológica causal de la sintomatológica de esta paciente es:
a) Eclampsia
18. Ingresa al servicio de urgencias paciente femenino de 76 años de edad con
antecedente de hipertensión de 23 años de diagnóstico, diabetes de 15
años de diagnostico y dislipidemia de reciente diagnóstico. Al momento del
ingreso la paciente presenta un episodio de emesis, sindroma de Horner del
lado izquierdo y perdida de la sensibilidad facial del lado izquierdo, sin
alteración de la sensibilidad del lado derecho del cuerpo. ¿Cuál de las
siguientes arterias es la más que probablemente esta ocluida?
a) Arteria cerebelar posteroinferior izquierda
19. Se presenta a consulta pediátrica paciente masculino de cinco años de
edad. Este es el hijo único de padres sanos, producto de un embarazo a
termino y parto autocico, la madre refiere que desde las primera semanas
de haber nacido presento un exantema eritmatoso en las regiones
expuestas al sol, las cuales después se volvieron color oscuro y se
generalizaron, con la aparición posterior de lesiones malignas tumorales. A
la exploración física el paciente muestra un exantema generalizado que
respeta axilas y genitales externos, caracterizada por aumento de la
pigmentación, entremezclada con áreas de hipopigmentación y regiones
con atrofia. Se observa también escamas y telangiectasis generalizadas.
Existen múltiples cicatrices en antebrazo, las cuales son secuelas de
extirpación e carcinomas epidermoides. ¿Cuál de los siguientes hallazgos
es más probable que se ubiquen en este paciente?
E) Dímero de timina en el DNA.
20. ¿Cuál es el mecanismo por el que se produce la diarrea en infecciones
intestinales por Escherichia coli enterotoxigénica?
d) Incremento de las concentraciones intracelulares de AMP cíclico.
21. Paciente femenino de 6 años de edad que presenta lesiones dérmicas,
maculas y papular, que evolucionan a ampollas en horas. La madre informa que
su hija padeció infección de vías respiratorias superiores hace 2 semanas.
¿Cuál es el diagnostico en este caso?
c) Varicela
22. Paciente de 65 años de edad que se presenta al servicio de urgencias con
fiebre alta, mareo y disnea. A la exploración física el paciente se presenta
taquicárdico, taquipneico y con insuficiencia mitral. Se realiza un ecocardiograma
el cual confirma la presencia de insuficiencia mitral. Se decide tomar hemocultivos
e iniciar tratamiento con vancomicina y gentamicina mientras se esperan los
resultados del cultivo. Dos días después el cultivo resulta positivo para
Streptococcusbovis. ¿Cuál de las siguientes opciones es la más adecuada en el
manejo de este paciente?
c) Solicitar una colonoscopia.
23. Paciente masculino de 29 años de edad que sufre una caída con lesión en la
pierna izquierda durante un partido de futbol americano. En el servicio de
urgencias se le toma una radiografía de la pierna izquierda la cual demuestra
fractura del cuello del peroné. ¿Cuál de los siguientes hallazgos puede ser el más
probable que se presente en este paciente?
24. Paciente femenino de 28 años de edad con neumonía atípica por Mycoplasma
pneumoniae. Al momento mas agudo de su enfermedad la paciente presenta
hemoglobina de 10 mg/dL y elevación de los niveles de lactato deshidrogenasa.
La paciente se trata con macrólidos y regresa a consulta tres meses después. Los
estudios de laboratorio de control demuestran ausencia de la anemia o de la
elevación de lactato deshidrogenasa. ¿Cuál es la mejor explicación a este
fenómeno?
c) normalización de la respuesta inmune
25. ¿Cuál de los siguientes estudios de laboratorio corresponde a un paciente con
intoxicación por vitamina D?
b) Calcio elevado, fosfato elevado, fosfatasa alcalina normal.
26. ¿Cuál de las siguientes condiciones se asocia con elevación de la presión
capilar pulmonar de enclavamiento?
b) insuficiencia ventricular izquierda con hipertensión pulmonar secundaria.
27. Paciente masculino de 80 años de edad que acude externa llevada por su hija.
El paciente enviudo hace cuatro años y desde entonces vive con su hija. Refiere
que ha presentado frecuencia y urgencia urinarias, así como nocturia desde la
muerte de su esposa, pero en los ultimo meses el problema ha empeorado. El
paciente refiere que presenta dificultad para iniciar la micción. Comunica que la
aproxima semana acudirá con el ortopedista porque desde hace varias semanas
ha presentado dolor lumbar que no mejora con reposo. El paciente también refiere
perdida de peso en los últimos cuatro meses. ¿Cuál es el medicamento mas
adecuado para el padecimiento principal de este paciente?
d) Flutamida
28. Mujer de 26 años de edad presenta desde hace 20 días artritis de las
articulaciones del hombro derecho, ambas rodillas y 2ª y 3ª metacarpofalángicas
izquierdas, con incapacidad para realizar sus actividades cotidianas; por 36 horas
requirió apoyo por parte de sus familiares para realizar actividades cotidianas,
como aseo personal. Comenta una rigidez articular matutina de 120 minutos,
niega cualquier otra sintomatología. A la exploración física: se corrobora artritis, en
clase funcional 111, sin gran afección sistémica. Un médico le recetó diclofenaco
100 mg cada 12 h con mejoría parcial. Usted decide realizar artrocentesis de
rodilla derecha y encuentra un líquido sinovial turbio con viscosidad baja,
leucocitos de 5000/µL con predominio de polimorfonucleares, aumento de
proteínas y ausencia de cristales y de bacterias. Una semana después, la paciente
regresa con la misma sintomatología. Le presenta una BH con leucocitos de 1O
000, trombocitosis 678 000, hemoglobina 11 g, Hto 33%, PCR 1O y VSG 45
3/h.
28a. Su primera sospecha diagnostica seria:
C) Artritis reumatoide.
28b. ¿Qué estudio solicitaría para corroborar su diagnóstico?
D) Factor reumatoide y Anti-CCP (antipéptidos cíclicos citrulinados)
28c. ¿Qué tratamiento iniciaría usted?
D) Iniciar con un esquema corto y a dosis bajas de prednisona mas
metotrexato semanal e hidroxicloroquina.
28d. ¿Qué vigilaría durante la administración de metotrexato?
a) La medula ósea y la función hepática
28e. El paciente tiene muchas dudas. Ha escuchado sobre el riesgo de
enfermedades neoplásicas asociadas a su enfermedad, y le cuestiona a
usted sobre ese supuesto riesgo. Usted:
b) Le explica sobre el riesgo de cáncer hematológico.
29. ¿Cuál de los siguientes eventos es el menos probable que se presente durante
un ataque agudo de asma?
b) La presión pleural durante la inspiración se vuelve mas negativo
30. ¿Cuál de los siguientes vasos venosos se encarga de conducir sangre de la
circulación prota en el sistema cava a pesar de la obstrucción de la vena porta (p.
ej., durante cirrosis hepática)?
e) Venas ácigos y hemiácigos
31. Los pacientes con cáncer de colon no polipósico hereditario presentan genes
con inestabilidad de microsatélites, lo que significa que contienen múltiples
regiones con fragmentos de DN de cadena única y de estructura anormal. ¿Cuál
de las siguientes es el tipo de proteína anormal en estos pacientes?
a) Enzimas de reparación de apareamiento incorrecto
32. Paciente masculino de 68 años de edad con diagnostico de mieloma múltiple
desde hace seis meses, acude al servicio de urgencias por presentar tos con
expectoración purulenta y fiebre, a la exploración física se encuentran con
frecuencia respiratorio de 24 rpm, saturación de 88% al aire ambiental, en los
campos pulmonares se auscultan estertores en base derecha, la placa de tórax
muestra una zona de infiltrado, este tipo de pacientes suele infectarse por los
siguientes microorganismos:
a) Streptococcus pneumoniae, Staphylococcus aureus y Klebsiella pneumoniae
33. Paciente masculino de 45 años de edad con antecedentes de infección por el
virus de la inmunodeficiencia humana se presenta con eritema de 12 mm e
induración de 7 mm a la prueba cutánea de la tuberculina utilizando 5 unidades de
PPD. La radiografía torácica no muestra anormalidades. El paciente se encuentra
bajo tratamiento antirretroviral altamente activo y no existe sintomatología. ¿Cuál
de las siguientes aseveraciones representa la conducta más adecuada?
c) Iniciar tratamiento con isoniazida por 12 meses
34. ¿Cuál de las siguientes condiciones es un factor protector para el desarrollo de
neumonía por Legionella?
d) Neutropenia
35. Paciente masculino de 29 años de edad sufre de accidentes por colisión de su
motocicleta contra un automóvil. El paciente se presenta con fractura de la diáfisis
derecha de fémur, sin trauma craneoencefálico ni perdida del estado de alerta
debido a que utilizaba casco protector; 24 horas posteriores a su admisión el
paciente se presenta con infiltrados pulmonares bilaterales y difusos. ¿Cuál es le
etiología mas probable de los infiltrados pulmonares?
c) Embolia grasa
36. Paciente femenino de 78 años de edad con diagnóstico de diabetes mellitus
tipo 2 desde hace 22 años. El paciente carece de apego al tratamiento, y debido a
que le han comentado que la insulina es muy ""dañina"", ha preferido continuar
esporádicamente con tratamiento a base de bebidas de medicina alternativa y
metformina. Desde hace varios meses el paciente notó la presencia de una úlcera
sobre la cabeza del tercer metatarso del lado derecho. La úlcera ha crecido, y
recientemente el paciente inició con dolor en la zona afectada por lo que acude a
consulta. A la exploración física se encuentra una úlcera de aproximadamente 4
cm de diámetro, con olor y secreción pútrida. Al parecer la úlcera tiene una
profundidad de 3 cm y afecta estructuras óseas. Se toma una muestra de la base
y se envía a tinción de Gram la cual reporta cocos grampositivos en cadenas,
bacilos gramnegativos, diplococos gramnegativos, gramnegativos pleomórficos y
exceso de neutrófilos. ¿En este momento, cuál es el tratamiento más adecuado en
lo que es posible obtener mientras se esperan los resultados de hemocultivos y
cultivos de la secreción de la herida?
e) Ampicilina con sulbactam
37. un paciente se presenta con botulismo posterior a la ingesta de miel
contaminada. ¿Cuál de los siguientes hallazgos es factible en esta paciente?
b) Diplopía.
38. Paciente masculino de 54 años de edad que acude al departamento de
urgencias por dolor torácico de tipo """desgarrante"" que inició hace 1 hora. El
paciente refiere que el dolor tiene irradiación hacia la región interescapular, y que
sufrió un episodio sincopal de 2 minutos que coincidió con el inicio del dolor
torácico. En el expediente clínico menciona que es conocido por padecer
síndrome de Marfan y dilatación de la raíz aórtica. En sus dos consultas de
seguimiento previas muestra cifras tensiónales elevadas (155/100 y 160/99 mm
Hg). A la exploración física presenta presión arterial de 180/95 mm Hg en la
extremidad escapular derecha y de 100/60 mm Hg en el miembro escapular
izquierdo, así como taquicardia de 105 latidos/minuto; la temperatura y frecuencia
respiratoria se encuentran dentro de los rangos de la normalidad. A la auscultación
cardíaca se puede percibir un soplo diastólico 11/IV en el borde esternal inferior
izquierdo. El ecocardiograma demuestra disección de la aorta torácica con
involucro de la aorta torácica ascendente y descendente.
Utilizando únicamente la información previamente descrita, responda las
siguientes preguntas:
38a. ¿Cuál de los siguientes fármacos es el más adecuado para este
paciente en este momento?
a) Labetalol
38b. ¿Cuál de las siguientes opciones corresponde al tipo de disección
aortica de este paciente?
a) DeBakey tipo I
38c. ¿Cuál de las siguientes opciones es el tratamiento definitivo mas
adecuado para este paciente?
a) Cirugía urgente.
39. Ante el caso de un recién nacido postérmino, con presencia de liquido
amniótico con meconio, que manifestó apnea y cianosis al nacimiento, ¿Qué
situación de las siguientes justificaría el uso de adrenalina para reanimarlo?
d) Frecuencia cardiaca menor a 60 latidos por minuto posterior a ventilación con
presión positiva y compresión torácicas externas.
40. Lactante masculino de diez meses es traído a urgencias por su madre por
presentar diarrea acuosa de dos días de evolución asociada a sed intensa e
irritabilidad. Se encuentra taquipneico, taquicárdico, con los ojos hundidos, llanto
sin lágrimas, lengua seca, saliva espesa, signos del lienzo húmedo y llenado
capilar de 4 segundos.
¿Cuál es el tratamiento adecuado para tratar este caso de deshidratación?
e) Administrar Vida Suero Oral 100 mL por kilogramo de peso, cada 30 minutos
durante cuatro horas en la unidad salud.
41. Paciente femenino de 40 años de edad con diagnóstico de embarazo de 14
semanas de gestación. Como antecedentes de importancia tiene los siguientes: 2
embarazos, en los cuales no hubo complicaciones, los dos fueron partos. Los
productos sin alteraciones. Acude por estar preocupada, ya que sabe que los
embarazos a edades mayores se relacionan con alteraciones en los niños. ¿En
qué momentos de la meiosis se realizan las dos detenciones en la ovogénesis?
b) Diplotena y metafase II
42. Te encuentras explorando a una paciente embarazada con 33 semanas de
gestación, con fondo uterino de 26 cm desde sínfisis de pubis, situación transversa
y decides auscultar foco fetal. ¿A partir de que semana de gestación se puede
percibir el ladito fetal a través del Pinard?
d) 20 semanas de gestación
43. Paciente masculino de 2 años y medio es llevado a consulta por irritabilidad
persistente. Al interrogara los padres, se conoce presencia de irritabilidad, vomito
bilioso y cólicos de 24 h de evolución, junto con distensión abdominal. A la
exploración física se observa paciente irritable, deshidratación leve, abdomen
ligeramente distendido con molestias a la palpación y ruidos peristálticos de lucha.
Signos vitales dentro de los limites normales.
43a. Con base en las características del paciente, ¿Cuál sería su principal
sospecha clínica?
a) Malrotación intestinal
43b. ¿Qué método diagnóstico es el más adecuado para confirmas su
sospecha clínica?
D) Serie gastroduodenal
43c. De acuerdo con su sospecha clínica, ¿Cuál es la razón por la cual se
desarrolló el padecimiento?
D) Alteración durante el desarrollo fetal.
43d. Si su sospecha clínica se confirma, ¿Cuál es el tratamiento indicado?
C) Reparación quirúrgica.
43e. De acuerdo con su sospecha clínica, ¿Cuál es la complicación que
podría presentar este paciente?
a) Síndrome de intestino corto
44. Neonato de 12 días de vida, que fue dado de alta en condiciones estables, y a
partir del décimo día de vida presento episodios de fiebre, con temperaturas entre
38 y 39.7 °C, hiporreactividad, rechazo al alimento, ¡DATOS DE MENINGITIS! De
los siguientes agentes etiológicos: 1) Sthaphylococcus epidermidis, 2)
Staphylococcus aurerus, 3) L. monocytogenes, 4) H. influenzae. ¿Cuáles dos
agentes se relacionan mas con sepsis tardía?
e) 1 y 2
45. Acude a urgencias primigesta de 19 años de edad en segundos periodo de
trabajo de parto. El producto se encuentra en el tercer plano de Hodge con una
variedad de posición occipitopsterior. Lleva una hora en el segundo periodo e
inicio con datos sufrimiento fetal. Sabiendo que no hay anestesiólogo. ¿Qué
acción se realizará en este momento?
c) Uso de fórceps.
46. Llega a urgencias paciente femenino de 38 años de edad primigesta, por
haber presentado crisis tonicoclónica generalizada hace 30 minutos. Refiere el
marido que previamente había tenido una cefalea muy intensa, fosfenos y dolor en
el epigastrio. Se encuentra en la semana 24 de gestación. TA de 180/110 mm Hg,
estuporosa, en anasarca, ROT incrementados. ¿Qué medicamento
anticonvulsivante es el de elección?
d) Sulfato de magnesio
47. Paciente masculino de 4 años de edad ingresa al departamento de Urgencia
por presentar accesos de tos y cianosis con periodos de apnea. Los padres
mencionan que previo dicho cuadro el niño se encontraba perfectamente bien. A la
exploración física se encuentra paciente masculino muy ansioso, con paroxismos
de tos, cianosis peribucal, babeo y estridor inspiratorio. Se aspira secreciones
abundantes y se oxigena adecuadamente.
47a. Con base en las características del paciente, ¿Cuál es el diagnostico
mas probable?
D) Cuerpo extraño
47b. De acuerdo con su sospecha clínica, ¿Cuál es la causa de este
padecimiento?
c) Cuerpo extraño en vía aérea.
47c. En relación con su sospecha clínica, ¿Qué hallazgos encontraría en la
radiografía?
c) Enfisema obstructivo
47d. Con base en su sospecha clínica, ¿Qué tratamiento le indicaría al
paciente?
a) Broncoscopia
47e. De acuerdo con su sospecha clínica, ¿Cuál es la complicación que se
puede presentar en el paciente?
b) Estenosis bronquial o neumonías recidivantes
48. De acuerdo con la clasificación de la Universidad de Texas, una ulcera
superficial, infectada y no isquémica en el pie de un paciente diabético
corresponde a:
a) 1B
49. Es el primer fármaco de elección en el tratamiento de diabetes tipo 2 y
obesidad.
d) Metformina
50. Prepuber de 12 años de edad con verrugas plantares
50a. ¿Cuál es el agente causal de esta infección?
a) Virus del papiloma humano
50b. ¿Cuál es el serotipo más prevalente en este tipo de verrugas?
a) 1
50c. ¿Qué le explicarías al paciente con respecto a la evolución y respuesta
al tratamiento?
e) Las verrugas pueden llegar a involucrar espontáneamente
50d. El paciente decide tratarse ya que le molesta mucho y no le gusta. De
los siguientes, ¿Cuál es una opción de tratamiento indicado?
b) Crioterapia.
51. Hombre de 31 años, con historia familiar de diabetes tipo 2 en padre y abuelo
paterno, hipertensión arterial sistémica en madre, infarto agudo de miocardio en
abuelo paterno; con diagnóstico de hipertensión hace tres años tratado con
enalapril. Desde hace aproximadamente 15 años ha presentado aumento
progresivo de peso (15 kg). A la exploración física: peso 140 kg, talla 1.85 m,
índice de masa corporal 41.22 kg/m2, tensión arterial 110/80 mm Hg, pulso 78x,
con obesidad de predominio central, acrocordones y acantosis pigmentaria en
región posterior del cuello y en ambas axilas, tiroides palpable y normal. Abdomen
globoso por panículo adiposo sin estrías. Fuerza muscular conservada. Resto sin
alteraciones. En sus exámenes de laboratorio destacan: glucosa 99 mg/dl,
creatinina 0.61 mg/dl, urea 37.1,ácido úrico 9.73 mg/dl, AST 54 Ul/L,ALT 65 Ul/L,
bilirrubinas y fosfatasa alcalina dentro de parámetros normales, colesterol total 202
mg/dl, triglicéridos 247 mg/dl.
51a. el diagnostico sindromático del paciente es:
b) Síndrome metabólico.
51b. La presencia de acrocordones y acantosis pigmentaria se debe a:
b) Resistencia a la insulina
51c. ¿Qué examen paraclínico es el mas indicado en este caso?
b) Curva de tolerancia a la glucosa oral
51d. El perfil viral de hepatitis es negativo y en un ultrasonido de hígado y
vías biliares no se reporta obstrucción de la vía biliar ni neoplasia hepática.
El diagnostico mas probable es:
c) Esteatohepatitis no alcohólica

52. Paciente masculino de 4 años de edad se presenta al departamento de


Urgencias por dolor abdominal. En el interrogatorio se refiere cuadro viral hace 2
semanas, dolor abdominal de pocas horas de evolución, vomito, oliguria y
erupción cutánea extensa. En la exploración física se encuentra un exantema
maculopapular en las nalgas y extremidades inferiores que no blanquea con la
digitopresión. La biometría hemática es normal, pero el examen general de orina
muestra hematuria.
52ª. Con base en las características del paciente, ¿Cuál es el diagnostico
mas probable?
d) Purpura de Henoch-Schönlein.
52b. ¿Qué hallazgos en los exámenes de laboratorio apoyan su sospecha
diagnostica?
b) Biometría hemática normal, sangre oculta en heces.
52c. De acuerdo con su sospecha clínica, ¿Cuál es la complicación que se
puede presentar a largo plazo?
c) Invaginación intestinal
52d. Con base en su sospecha clínica, ¿Cuál es el tratamiento indicado?
a) AINE
52e. ¿Qué aspecto del cuadro clínico del paciente es relevante mencionar a
los padres?
d) Hematuria microscópica

53. Paciente masculino de 88 años con múltiples condiciones médicas (angina


estable, enfermedad pulmonar obstructiva crónica, osteoartritis, diabetes mellitus
tipo 2,enfermedad acidopéptica) ingresa a la sala de urgencias por presentar dolor
abdominal de inicio abrupto de 6 horas de evolución. El paciente informa dolor
abdominal difuso, con leve localización en la región periumbilical. A la exploración
física se registran frecuencia cardiaca de 120 latidos por minuto, presión arterial
de 100/60 mm Hg, frecuencia respiratoria de 30 respiraciones por minuto e
hipotermia. A la palpación abdominal se encuentran signos de irritación peritoneal.
Las radiografías de tórax y abdomen demuestran la presencia de aire
subdiafragmático. ¿Cuál es el vaso sanguíneo más frecuentemente afectado en
casos de perforación de úlcera duodenal en la pared posterior del duodeno
(primera porción)?
b) Arteria gastroduodenal.
54. Paciente femenino de cinco años que es traída por sus padres tras ingerir
accidentalmente limpiador de drenaje guardado en una botella de refresco. La
paciente esta quejumbrosa, con dolor de torácico intenso, náuseas, disfagia y
sialorrea profunsa. A la exploración física se observa quemaduras de los labios,
edema y eritema de lengua y paladar. ¿Cuál de las siguientes está contraindicada
en el manejo de este caso?
d) Lavado nasogástrico.
55. Paciente femenino de 19 años de edad, con embarazo de 12 semanas de
gestación por fecha de última menstruación, acude a urgencias por presentar
hemorragia genital persistente, con dolor en hipogastrio tipo cólico, sin
irradiaciones que ha ido aumentando en intensidad y frecuencia. A la exploración
física: frecuencia cardiaca 105 latidos por minuto, frecuencia respiratoria 20
respiraciones por minuto, temperatura 37 C. Abdomen blando, depresible,
doloroso a la palpación del hipogastrio, sin datos de irritación peritoneal. Se
observa hemorragia genital y se realiza tacto vaginal encontrando cuello uterino
central, con 4 cm de dilatación y borramiento de 30%. ¿Cuál es su impresión
diagnóstica?
c) Aborto en evolución.
56. Paciente masculino de 2 años de edad es llevado a consulta de rutina. Se
mencionan como antecedentes que nació a las 37 semanas de gestación, fue
alimentado de manera exclusiva mediante seno materno durante el primer año de
vida y que actualmente no es bueno comiendo. Su peso y estatura se encuentran
en el percentil 5. A la exploración física se observa que sus piernas están algo
arqueadas y el hueso frontal es muy prominente. No tiene dientes y todavía no
camina por sí solo.
56ª. Con base en las características del paciente, ¿Cuál es el diagnostico
más probable?
b) Deficiencia de vitamina D
56b. En relación con su sospecha clínica, ¿cuál de las siguientes opciones
representa una característica particular del padecimiento?
a) Extremos distales ensanchados con forma de copa y con desgaste en la
radiografía
56c. Respecto a su sospecha clínica, ¿Qué función tiene en el cuerpo la
vitamina que se encuentra con deficiencia en el paciente?
c) Deposito de calcio
56d. De acuerdo con su sospecha diagnóstica, ¿Qué tratamiento le
indicaría al paciente?
d) Suplemento de vitamina D
56e. Referente a su sospecha clínica, ¿Cuál es el mecanismo de
prevención que se debió realizar para evitar esta patología?
b) Exposición al sol
57. Mujer de 54 años de edad con dermatosis bilateral y con tendencia a la
simetría que se disemina a ambas caras pretibiales. Se caracteriza por tres placas
infiltradas amarillo violáceas y esclerodermiformes, irregulares en la forma y mal
delimitadas, que miden entre 5 y 7 cm. La paciente refiere que sus lesiones
iniciaron hace un mes como ""granitos"" de color rojo violáceo y otras como
nódulos de forma redondeada, de bordes no bien definidos y a veces elevados.
Fueron creciendo de forma lenta y excéntrica, tornándose de color amarillo, con
telangiectasias en la superficie. Una de las lesiones se ulceró, dejando cicatriz y
atrofia posterior. A la exploración física presenta en las lesiones sensibilidad
disminuida al pinchazo y al tacto leve, hipohidrosis y alopecia parcial. La paciente
es diabética de larga evolución y al parecer lleva un buen control. Niega
antecedentes de traumatismos o de infecciones previas. El diagnóstico es:
b) Necrobiosis lipoídica.
58. Una paciente de 35 años sin antecedentes previos de importancia ingresa a la
sala de urgencia. El Motivo es por dolor abdominal de apropiadamente 24 horas
de evolución, el cual inicio en la región periumbilical pero recientemente ha
migrado hacia el hipocondrio derecho. La paciente refiere nausea y vomito en tres
ocasiones. A la exploración física, se detectan mucosas deshidratadas, así como
dolor abdominal localizada al hipocondrio derecho con signo de rebote positivo.
Los estudios de laboratorio son relevantes por leucocitosis a base de neutrófilos.
¿Cuáles son los nervios que condicen la sensación dolora periumbilical en este
paciente?
b) Nervio esplácnico menor
59. Paciente masculino de 31 años de edad con antecedente de dos semanas de
fiebre, a lo cual se agregó hace una semana distensión abdominal, estreñimiento y
dolor epigástrico. Entre otras molestias también refiere cefalea, escalofríos,
mialgias y malestar general. La exploración física revela la presencia de un
""exantema rosado (exantema maculopapular color salmón que blanquea a la
presión), principalmente en el tórax, así como dolor abdominal generalizado a la
palpación y esplenomegalia. Presenta temperatura de 38.9 ºC, frecuencia cardiaca
de 45 latidos por minuto, frecuencia respiratoria de 20 respiraciones por minuto,
presión arterial de 100/65 mm Hg. Los estudios de laboratorio básicos son
relevantes por leucopenia y neutropenia. Se realiza un hemocultivo el cual resulta
positivo para Sa/mone//a typhi. ¿Cuál es el tratamiento antibiótico más efectivo en
cepas no provenientes de brotes resistentes?
a) Fluoroquinolonas
60. Un paciente de 21 años se presenta a la sala de urgencias posterior a una
lesión en su hombro durante un partido de futbol. A la exploración física el hombro
parece deprimido y plano y no es capaz de abducir su brazo. Una radiografía de la
región demuestra que el paciente tiene fractura del cuello quirúrgico del humero.
¿De dónde proviene la inervación del muslo afectado en este paciente?
b) C5 a C6
61. Un paciente de 45 años con sobrepeso es llevado a la sala de urgencias con
perdida del estado de alerta después de un accidente automovilístico. Luego de
recuperar la conciencia, la exploración física revela debilidad de ambas
extremidades derechas con signos de Babinski positivo en el pie derecho. El
paciente muestra la lengua desviada hacia la izquierda cuando la protruye. A la
evaluación de la sensibilidad corporal, el paciente ha perdido la sensibilidad
vibratoria en el lado derecho del cuerpo. ¿Cuál es el sitio más probable del lesión
en este paciente?
a) Lesión de la región medial de la medula oblongada
62. Paciente femenino de 60 años de edad con diagnósticos previos de
hipertensión, diabetes tipo 2, miocardiopatía isquémica, e insuficiencia renal
crónica; reporta dolor en la rodilla derecha. La presión arterial es de 140/80 mm
Hg, con una frecuencia cardiaca de 66 latidos/min. Debido al edema y al dolor en
la rodilla, se le receta celecoxib 200 mg/día. Después de dos semanas bajo este
tratamiento, la paciente refiere disnea, edema de extremidades inferiores y fatiga.
La presión arterial en esta ocasión es de 180/105 mm Hg, nitrógeno ureico de 67
mg/dL, creatinina sérica de 3.9 mg/dl. ¿Cuál de los siguientes diagnósticos es el
mecanismo de acción más probable por el cual el analgésico administrado
ocasionó insuficiencia renal?
A) Perdida del balance entre prostaglandinas, ocasionada por inhibición de la
ciclooxigenasa tipo 2
63. Paciente masculino de 36 años, arqueólogo, diabético. Estuvo en contacto con
pacientes con tuberculosis en su último viaje a Zimbabwe. Comenzó con fiebre,
tos productiva con estrías sanguinolentas y ataque al estado general. Durante su
traslado presentó sudores nocturnos. La radiografía de tórax revela infiltrados
nodulares en lóbulo superior, con una lesión cavitada que está drenando a través
de un bronquio. PPD negativo. Se inicia tratamiento antituberculoso. Acude a
consulta a ajuste de hipoglucemiantes orales debido a que su glucosa sérica se
elevó casi el doble de lo que manejaba hace tres meses. Refiere que ha respetado
su dieta y ha estado tomando su medicación. Pocas semanas después del inicio
del tratamiento antituberculoso el paciente reporto disminución de tos e
incremento de energía.
¿Cuál es la explicación de la hiperglucemia en este paciente?
b) Rifampicina
64. Una paciente de 64 años es llevada al hospital por sensación de vértigo y
sordera del oído izquierdo. A la exploración física la paciente presenta sordera
neurogénica del lado izquierdo; analgesia y anestesia térmica del lado derecho del
cuerpo y del lado izquierdo de la cara; depresión del paladar con dificultad para la
deglución; síndrome de Horner izquierdo; tendencia a caerse hacia el lado
izquierdo; nistagmo horizontal.
64a. ¿Cuál de las siguientes estructuras es la responsable de la depresión
del paladar y la dificultad para la deglución?
a) Núcleo ambiguo
64b. ¿Cuál de los siguientes enunciados es correcto en relación con el
nistagmo del paciente?
a) El componente rápido se dirige hacia la derecha
64c. ¿Cuál de las siguientes opciones representa la etiología más probable
del cuadro clínico de este paciente?
d) Oclusión de la arteria cerebelar posteroinferior izquierdo.

65. Paciente masculino de 3 años de edad, quien 24 horas previo a la consulta


presento rinorrea, fiebre y tos no traqueal, el día de hoy se agrega ronquera, tos
“perruna” y estridor laríngeo en reposo; a la exploración física se encuentra con
adecuado estado mental, ligera palidez y tiraje intercostal bajo, la SpO2 es de
88%. Se niegan antecedentes relevantes. FC 110 lpm, FR 24 rpm, temp. 38.3°C,
PA 90/60 mm Hg, peso 14kg, talla 96 cm.
65a. El diagnostico de este paciente es:
c) Laringotranqueítis.
65b. ¿Cuál de los siguientes es un signo de alarma al ingreso?
d) Estridor laríngeo
65c. Según los datos obtenidos hasta ahora, usted clasifica este cuadro
como:
b) Moderado
65d. El tratamiento farmacológico que usted selecciona es:
d) Glucocorticoides
65e. ¿Cuáles son los datos de alarma para la familia del paciente?
a) Cianosis
66. Paciente masculino de 18 años con antecedentes de anemia de células
falciformes e hipertensión portal es traído a urgencias por presentar hematemesis
masiva. El paciente se encuentra ictérico, taquicárdico e hipotenso. Usted indica
oxigeno por puntas nasales, canalizar dos vías periféricas con catéter de calibre
16, bolos de solución salina y obtiene muestras de sangre para biometría
hemática, estudios de coagulación y cruzar sangre.
¿Cuál es el tratamiento de elección del sangrado de carices esofágicas?
c) Endoscopia con ligadura de várices
67. Paciente masculino de 70 años de edad acude al servicio de urgencias por
dolor torácico. El paciente niega disnea, palpitaciones, nausea, vomito o mareo. El
expediente clínico es relevante por hipertensión, manejada en la actualidad con
diuréticos. La presión arterial del paciente es de 180/100 mm Hg, con una
frecuencia cardiaca de 110 latidos/minuto. La exploración del cuello evidencia un
desplacamiento inferior del cartílago cricoides durante cada contracción, asi como
positividad para el signo de Carderelli. ¿Cuál de los siguientes estudios
diagnósticos es el más apropiado en este paciente?
e) Ecocardiograma transesofágico
68. Paciente femenino de 23 años de edad que ingresa a hospitalización por
presentar desde hace 30 días fatiga y disnea al ejercicio. Entre los antecedentes,
se encuentra una infección de vías respiratorias superiores hace un mes. No hay
antecedentes heredofamiliares de importancia. A la exploración física muestra una
presión arterial de 95/55 mm Hg, y un ritmo cardiaco de 115 latidos/minuto. Las
venas yugulares se encuentran distendidas y la auscultación pulmonar revela
estertores bilaterales; existe tercer ruido cardiaco. Se realiza un ecocardiograma
que demuestra dilatación de ambos ventrículos con hipocinesia difusa y fracción
de eyección del ventrículo izquierdo de 18%. ¿Cuál de los siguientes enunciados
NO es correcto respecto al cuadro de esta paciente?"
c) La administración de corticoesteroides puede disminuir la progresión de la
enfermedad
69. Paciente masculino de 70 años de edad que es llevado al servicio de
urgencias por confusión en las últimas 12 horas. Dos semanas previas el paciente
había sufrido un infarto miocárdico, con colocación de stent. Entre los
medicamentos del paciente se encuentra ticlopidina, aspirina, atorvastatina,
metoprolol y captopril. A la exploración física tiene fiebre de 39 ºC y exantema
petequial difuso. El resto de la exploración física no es relevante. Los estudios de
laboratorio presentan elevación de la creatinina (3.5 mg/dL) y caída en el
hematocrito (22%), con un conteo plaquetario de 45 000 plaquetas/mm3. ¿Cuál de
los siguientes medicamentos es el responsable por el cuadro de este paciente?
b) Ticlopidina
70. Mujer de 25 años de edad, acude a consulta por presente dolor en el pie
derecho con aumento de volumen y tumefacción del primer ortejo, limitación para
la movilidad de las articulaciones que componen ese dedo del pie; al interrogatorio
dirigido niega antecedentes crónico-degenerativos, ovolactovegetariana, sin
toxicomanías ni alergias referidas, refiere haber cursado con cuadro infeccioso
gastrointestinal hace 3 semanas completamente remitido y niega antecedentes
traumático; a la exploración se encuentra aumento de volumen del dorso del pie,
sin eritema ni hipertermia. FC 75 lpm, FR 18 rpm, temp. 36.5°C, PA 110/70 mm
Hg, peso 55 kg, talla 1.62 m.
70a. ¿Cuál es la causa más probable de la artropatía?
c) Artritis reactiva
70b. ¿Cuál de los siguientes agentes está asociado con artritis reactiva?
a) Salmonella
70c. Con el diagnostico y cuadro clínico, usted decide iniciar el
tratamiento de primera línea.
d) Antiinflamatorios no esteroides
70d. ¿Cuál de los siguientes criterios no es diagnóstico de artritis
reactiva?
b) Afección simétrica
70e. Además de la artritis reactiva, los pacientes pueden presentar otras
manifestaciones ¿Cuál de las siguientes no está asociada con artritis
reactiva?
d) Eritroderma
71. ¿Cuál de los siguientes medicamentos se caracteriza por presentar infarto
miocárdico como complicación?
c) 5-fluorouracilo
72. Masculino de 40 años acude a consulta por presentar desde hace 5 meses
dolor retroesternal molestias al ingerir alimento. Dichas molestias son de carácter
intermitente y el dolor a veces se irradia a la mandíbula. No refiere pirosis ni
episodios de regurgitación.
72ª.¿cuál es su sospecha diagnostica?
d) Espasmo esofágico difuso
72b. usted solicita una manometría. ¿Qué resultado apoyaría su sospecha
diagnostica?
a) >20% de las contracciones de los dos tercios inferiores son
peristálticas, de amplitud variable (amplitud media >30 mm Hg)
72c. La imagen característica en el trago de bario es:
b) Esófago en sacacorchos
72d. ¿Cuál de las siguientes opciones no es parte del manejo de
estetrastorno?
d) Funduplicatura
72e. En caso de que el tratamiento conservador fracasara, ¿Cuál otro no
estaría indicado?
d) Funduplicatura
73. Femenino de 45 años de edad es traída por los servicios de emergencia al
hospital donde usted esta de guardia. La paciente ha sufrido un accidente con un
coche, se encuentra inconsciente y se observan lesiones graves en todo el macizo
facial y zona del ojo derecho. TA: 130/80, FC: 95x ‘, FR: 23x ‘, Temp: 37.5°C.
73a. La fractura mas característica de la órbita es:
b) Polo inferior.
73b. La fractura mas frecuente de la órbita, que además puede lesionar la
tróclea del oblicuo superior, es:
c) Medial
73c. ¿Qué huesos conforman la pared inferior de la órbita?
b) Maxilar superior, malar y palatino
73d. Ante un trauma ocular, ¿Qué estudio de gabinete se debe solicitar?
d) TC
74. Hombre de 35 años de edad, fumador y bebedor social desde los 17 años,
consulta por haber observado sangre fresca en el retrete y en el papel higiénico
tras obrar; no refiere dolor ni cambios en sus hábitos intestinales, así como
tampoco sensaciones de masas o tumoración. A la exploración física se observa
región anal sin sangre ni tumoraciones. TA: 120/80 mm Hg, FC: 70x ‘, Temp:
36.5°C.
74ª. Su principal sospecha diagnostica es:
c) Hemorroides
74b. ¿Cuál es la causa de este padecimiento?
b) Dilatación anormal de las anastomosis arteriovenosas
74c. Es la referencia para la clasificación de esta enfermedad.
b) Línea dentada.
74d. El primer paso dentro del tratamiento es:
a) Fibra en la dieta y ablandadores de heces.
74e. La escleroterapia y la ligadura con banda elástica están
contraindicadas en:
d) Hemorroides externas
75. Masculino de 30 años acude a consulta por presentar un cuadro diarreico de 3
dias de evolución, entre seis y ocho evacuaciones al día, abundantes, liquidas, sin
presencia de moco o sangre. Presento vomito en dos ocasiones al inicio, y en las
últimas horas ha mostrado mareo. Niega fiebre. Antecedente: haber asistido a
competencia de triatlón a Nueva Delhi la semana previa al padecimiento actual
75ª. El germen causal de la sintomatología de este paciente es:
d) Vibrio cholerae
75b. Tratamiento de elección para este paciente:
D) Doxiclina
75c. El cuadro clínico de este paciente es causado por aumento en :
d) cAMP
76. Paciente femenino de 50 años es llevada al servicio de urgencias por
presentar disnea de una semana de evolución que ha progresado hasta ser de
pequeños esfuerzos y se acompaña en ocasiones de palpitaciones y dolor torácico
que dura pocos minutos. La disnea fue máxima esta mañana, asociada con dolor
retroesternal 7/10 que aumenta con la respiración. La paciente no tiene historia de
tabaquismo, niega cirugías recientes. Su esposo refiere que la ha visto cansada y
que ya casi no se levanta de la cama desde que le colocaron una férula en el
tobillo izquierdo debido a que sufrió un esguince hace dos semanas. A la
exploración física es evidente la dificultad respiratoria con uso de músculos
accesorios, signos vitales con frecuencia respiratoria de 26, frecuencia cardiaca
105 lpm, presión arterial 85/50 mm Hg, temperatura 37.5 ºC. Se aprecia
ingurgitación yugular de 8 cm, taquicardia, S1 y S2 sin soplos ni ruidos accesorios,
campos pulmonares bien ventilados y edema de miembros pélvicos,
especialmente del lado izquierdo, con dolor a la palpación de la pantorrilla
izquierda."
La sospecha clínica de que esta paciente presente una embolia pulmonar es:
d) Alta
77. Paciente femenino de cinco años es traída a urgencias por sus padres quienes
la encontraron jugando en un campo que había sido regado con pesticidas. La
paciente presento inicialmente abundante salivación y lagrimeo, vomito, diarrea e
incontinencia urinaria seguida de confusión y contracciones musculares
generalizadas. A la exploración física se encuentra letárgica, diaforética, con
pupilas mioticas y fasciculaciones. Usted de inmediato sospecha intoxicación por
organofosforados.
¿Cuál es el medicamente adecuado para tratar los efectos nicotínicos en esta
paciente?
e) Pralidoxima
78. Paciente masculino de 35 años, albañil, acude a consulta por presentar prurito
intenso en las manos, así como fisuras y costras entre los dedos de 15 días de
evolución. Se ha aplicado Vitacilina sin mejoría. A la EF se observa dermatosis
diseminada a palmas de manos y a región interdigital, caracterizada por múltiples
placas eritematoescamosas ,algunas fisuras y costras hemorrágicas. Niega
ingesta de medicamentos. Comenta que se encuentra bastante preocupado, ya
que no le es posible laborar en la obra ni lavarse las manos porque nota mucho
ardor al contacto con acetona.
78a. ¿Cuál es el diagnóstico de este paciente?
b) Dermatitis por contacto
78c. Dentro de las enfermedades ocupacionales, ¿Qué porcentaje
representa la dermatitis por contacto de manos?
a) 25 a 50%
78d. ¿Qué tipo de reacción es la dermatitis de contacto por irritante
primario?
d) No inmunitaria.
78e. ¿Qué tipo de reacción inmunitaria es la dermatitis por contacto con
sensibilización o alérgica?
d) IV
79. Masculino de 14 años, sin antecedentes patológicos.es traído al hospital por
sus padres tras haber sufrido una lesión mientras jugaba futbol americano. El
paciente comenta que al ser tacleado cayó sobre el hombro derecho e
inmediatamente sintió dolor en la región (EVA 10/1O); dicho dolor ha ido en
aumentoy le impide mover la extremidad superior derecha. A la exploración usted
comprueba que a la movilización de dicha extremidad el dolor es insoportable y
observa que el hombro derecho está descendido con respecto al izquierdo; al
palpar la zona superior derecha del tórax, usted nota edema, crepitación y
movilidad anormal en la región.
79ª. ¿Cuál sería su sospecha diagnostica?
a) Fractura clavicular
79b. ¿Qué estudio solicitaría para confirmar su diagnóstico?
a) Radiografía de tórax anteroposterior
79c. ¿Cómo se clasifica esta lesión?
c) Tercio lateral, tercio medio o tercio medial
79d. el tratamiento clásico de esta lesión es:
a) Vendaje o dispositivo en “8”
79e. Fractura mas frecuente durante el trabajo de parto
c) Clavicular
80. Paciente femenino de 45 años acude a consulta por “hinchazón” de manos
desde hace 5 meses; también ha notado un cambio de textura en su piel, que
ahora esta mas firme y dura. Tiene antecedentes de enfermedad por reflujo
gastroesofágico y disfagia a sólidos. A la exploración física se nota inflamación en
las articulaciones de sus dedos y presenta nódulos subcutáneos en estos; así
mismo, se percibe leve edema e induración en ambas manos.
80ª. ¿Cuál sería el diagnostico más probable?
d) Esclerosis sistémica cutánea limitada
80b. De acuerdo con el diagnóstico clínico, ¿Cuáles de los siguient4es anticuerpos
ayudaría a tener un diagnostico más certero?
a) Anticuerpos anticentrómero.
80c. ¿Cuál es la fisiopatología de la enfermedad?
d) Proliferación de fibroblastos con aumento en los depósitos de colágeno.
80d. En cuanto a las alteraciones renales de esta enfermedad, ¿Cuál es la más
características?
a) No hay afección renal
81. Masculino de 11 años de edad es traído a consulta por su madre, la cual
comenta que la maestra del niño le ha reportado que lo nota distraído en las
clases y que, por otra parte, en la casa ve la TV muy de cerca; además de que por
la tarde trae los ojos muy rojos. Durante la entrevista usted se da cuenta de que el
niño entrecierra mucho los ojos para observar los objetos en la pared. Al interrogar
al niño, este comenta que le cuesta trabajo ver de lejos, por lo que se acerca para
ver bien, y que después de ver mucho tiempo la TV le da dolor de cabeza
81ª. ¿Cuál sería su diagnóstico para este paciente?
b) Miopía
81c. ¿Cómo se define un dioptrio?
c) Toda superficie que separa dos medios con distinto índice de refracción
81d. Del sistema óptico del ojo, ¿Cuál estructura es la mas potente?
b) Córnea
81e. El tratamiento de primera elección para esta paciente seria:
c) Lentes divergentes
82. Paciente femenino de 50 años con dermatosis diseminada que afecta tronco y
extremidades superiores e inferiores caracterizada por ampollas flácidas,
erosiones y costras, algunas sanguíneas y otras melicéricas, y se acompaña de
estomatosis caracterizada por erosiones. No ha recibido tratamiento, refiere fiebre
de hasta 38.5 C. Inició hace alrededor de dos semanas con ampollas que han ido
evolucionando a costras. Las lesiones en boca iniciaron un mes antes y recibió
tratamiento con aciclovir por probable estomatitis herpética, sin mejoría. A la
exploración física muestra además mal estado de hidratación de mucosas y
taquicardia. Los hallazgos histológicos de la biopsia por sacabocado de una de las
ampollas son:
b) Ampolla suprabasal con células acantolíticas y células basales unidas a la
membrana basal
83. Paciente masculino de 12 años de edad que es llevado por sus padres a
consulta pediátrica. El paciente se unió hace un mes a un equipo de fútbol, y
desde entonces ha padecido dolor de rodillas. Debido a que se le realizó un
examen físico antes de ingresar al equipo la madre puede comentar con confianza
que el hijo ha perdido exactamente 4 kg desde entonces. Por indicaciones del
entrenador del equipo se le tomó al paciente una radiografía de rodilla, la cual
presentan los padres ahora al médico. En la radiografía se observa una lesión
densamente esclerótica en la porción distal del fémur la cual se extiende desde el
disco de crecimiento hacia la diáfisis. Se observa que el periostio se encuentra
elevado, formando una angulación con la corteza ósea. Los tejidos blandos
alrededor de la lesión se observan como ""rayos de sol"". ¿Cuál de Jos siguientes
diagnósticos es el más probable en este paciente?
b) Osteosarcoma
84. De acuerdo con los criterios CURB-65, ¿Cuál de los siguientes pacientes debe
ser ingresado para tratamiento de neumonía adquirida en la comunidad?
c) Paciente masculino de 45 años de edad sin alteraciones del estado de alerta,
nitrógeno ureico en sangre de 25 mg/dL, frecuencia respiratoria de 31
respiraciones/min, y presión arterial de 110/75 mm Hg.
85. ¿Cuál de los siguientes fármacos se utiliza en el tratamiento del tabaquismo?
a) Bupropiona
86. Paciente del sexo femenino de 40 años de edad que es llevada al servicio de
urgencias por presentar parálisis de la mitad izquierda de la cara. La paciente notó
la evidente simetría al despertar esta mañana. Al interrogatorio la paciente y
familiares mencionan que es una persona aparentemente sana sin ningún
antecedente de infecciones virales o exantemas recientes. A la exploración física
se puede corroborar la asimetría facial, con parálisis del lado izquierdo; es incapaz
de cerrar el ojo izquierdo, y cuando trata de hacerlo el globo ocular desvía la
mirada hacia arriba. Al continuar con la exploración física se corrobora
incapacidad para fruncir la frente del lado izquierdo y mostrar los dientes. ¿Cuál es
el manejo más adecuado en esta paciente?
c) Administración de aciclovir y corticoesteroides
87. ¿Cuál de los siguientes diuréticos es capaz de ocasionar edema pulmonar?
c) Manitol
88. ¿Cuál de los siguientes diagnósticos se presenta con un reporte de biopsia de
ganglios linfáticos que menciona expansión de folículos linfoides, preservación de
la arquitectura ganglionar con linfocitos atípicos en la zona paracortical?
d) Infección aguda por el virus de Epstein-Bar
89. Paciente femenino de 42 años de edad que se presenta con adenomegalia
cervical única sin fiebre, sudación nocturna y pérdida de peso. Se realiza una
biopsia del ganglio linfático la cual revela presencia de linfocitos pequeños y pocas
células de Reed-Sternberg. Se efectúa una resonancia magnética para etapificar a
esta paciente; el estudio de imagen indica enfermedad limitada a sólo un grupo
cervical. ¿Cuál sería la clasificación adecuada de esta paciente?
a) Linfoma de Hodgkin etapa IA
90. Paciente femenino de 36 meses de edad que se presenta con fiebre de 38 ºC
y dolor en ambos oídos. Los padres mencionan que esto ha ocurrido en
varias ocasiones, y que casi siempre le recetan dos medicamentos, los cuales no
recuerdan en este momento. A la exploración física la paciente se encuentra
alerta, con agitación secundaria al dolor, congestión nasal y tos. Las membranas
timpánicas se observan eritematosas, con movimiento positivo a la insuflación. El
timpanograma demuestra una curva ""picuda"". ¿Cuál es el tratamiento más
adecuado en esta paciente?
c) Administración de paracetamol.
91. Femenino de 40 años que se encuentra en evaluación por infecciones de las
vías urinarias recurrentes desde hace varios años, junto con hematuria
microscópica. A la exploración física no se encuentran datos de utilidad para este
padecimiento. Los estudios de laboratorio muestran normalidad de los niveles de
creatinina y nitrógeno ureico en sangre. Se solicita un ultrasonido el cual muestra
crecimiento irregular de los conductos papilares de ambos riñones, así como
múltiples quistes medulares; el tamaño de ambos riñones es normal. ¿Cuál de los
siguientes diagnósticos es el más probable en este paciente?
b) Riñón esponjoso medular
92. Paciente masculino de 40 años de edad que se presenta a hospitalización por
fiebre elevada y persistente, así como hemorragia de encías y fatiga. En un frotis
de sangre periférica se observan abundantes mieloblastos con bastones de Auer.
El paciente desarrolla trastornos de la coagulación correspondientes a la
coagulación intravascular diseminada. ¿Cuál de las siguientes alteraciones
cromosómicas es la que presentan con mayor probabilidad las células anormales
de este paciente? c) t(15;17)
93. ¿Cuál de los siguientes trastornos del sueño es el más común en la población
general?
d) Insomnio
94. Paciente masculino de 59 años de edad con antecedentes de enfermedad
acidopeptica y reflujo gastroesofágico. Debido a que presenta insuficiencia
cardiaca y fibrilación se encuentra bajo tratamiento con digoxina. El paciente
decide iniciar por si mismo tratamiento con antiácidos de hidróxido de aluminio.
¿Cuál de las siguientes alteraciones electrolíticas puede ser ocasionada por el
antiácido y con alta morbilidad en este paciente?
d) Hipopotasemia
95. Paciente femenino de 85 años de edad que se presenta con alteración del
estado de alerta, desorientación, combativa, con pupilas midriáticas no reactivas a
la luz, taquicardia sinusal, piel seca, hipertermia, constipación y retención urinaria.
Todos estos síntomas aparecieron después que por error de una estudiante de
enfermería se le administró una dosis excesivamente alta de atropina. ¿Cuál de
los siguientes medicamentos es el mejor antídoto en esta paciente?
a) Fisostigmina
96. Paciente masculino de 24 horas de vida producto de una paciente de 42 años
de edad. A la exploración física el neonato se presenta con microcefalia,
implantación baja de orejas, occipital prominente y micrognatia. ¿Cuál de las
siguientes anormalidades en el cariotipo es la que se puede encontrar con más
probabilidad en este paciente?
b) Trisomía 18
97. Paciente masculino de 65 años se presenta en la madrugada al servicio de
urgencias por presentar disnea en reposo y edema remitente de miembros
inferiores pero que va en ascenso. Cuenta con antecedente de hipertensión y
diabetes mellitus tipo 2, así como de tres infartos miocárdicos, de los cuales dos
de ellos no pudieron ser trombolizados porque el paciente llegó fuera de ventana
terapéutica. Hace dos días presentó un cuadro de infección de vías aéreas
superiores, que ha progresado hasta convertirse en una neumonía incipiente.
¿Cuál de las siguientes combinaciones de fármacos, a pesar de ser efectiva en
este paciente, también puede resultar peligrosa?"
a) Digoxina + furosemida
98. ¿Cuál de los siguientes antiarrítmicos tiene su principal mecanismo de acción
en células de despolarización automática y de lenta conducción?
e) Esmolol
99. ¿Cuál de los siguientes medicamentes se asocia con hiperpotasemia como
efecto adverso?
e) Ciclosporina
100. Paciente masculino de dos días de vida el cual presenta acidosis, vomito,
hipotonía y alteraciones neurológicas. Se solicitan estudios de laboratorio los
cuales revelan niveles elevados de lactato y alanina. ¿Cuál de las siguientes
enzimas es la que con mayor probabilidad se encuentra deficiente en este
paciente?
b) Piruvato deshidrogenasa
101. ¿Cuál de los siguientes mecanismos de acción corresponder al de las
estatinas?
b) Inhibición de la inzima hidroxi-metil-glutaril-coenzima A
102. ¿Cuál de los siguientes defectos es ocasionado por ausencia de la formación
del septo aorticopulmonar?
b) Comunicación interventricular tipo membranosa
103. Sujeto femenino de 60 años que llega a la consulta por episodio de litiasis.
Sin antecedentes heredofamiliares de importancia. Refiere historia de varios
episodios de pielonefritis aguda que han requerido hospitalización. Trae consigo
una urografía excretora, que revela captación del medio de contraste únicamente
por el riñón derecho en fase tardía, captación por el riñón izquierdo normal.
Asimismo, cuenta con un renograma que cuantifica la función renal del riñón
derecho en 15%, frente a 85% del izquierdo. ¿Cuál es la actitud terapéutica que se
debe tomar?
d) Nefrectomía
104. Una mujer de 18 años de edad se presenta a consulta externa de Obstetricia
en su 18 semana de gestación. Hace tres años, la paciente fue diagnosticada con
depresión moderada y se le recetó paroxetina 20 mg/día, la cual continuó tomando
hasta hace aproximadamente seis meses, debido a que consideró que no
necesitaba el medicamento. La paciente informa que recientemente no ha dormido
""en semanas"", y lo atribuye al embarazo. Al interrogatorio menciona que también
se ha mostrado recientemente letárgica, sin ideación suicida y que desea
alimentar a su futuro hijo con leche de su propio seno. ¿Cuál de los siguientes
esquemas de tratamiento es el más adecuado para el manejo de depresión de
esta paciente?
a) Sertralina 50 mg al día, con consulta psiquiátrica de seguimiento en un mes
105. ¿Cuál de las siguientes cadenas de globinas se produce exclusivamente
durante la etapa embrionaria?
e) Zeta globina
106. Paciente femenino de 22 años de edad se presenta a consulta por ictericia y
malestar general desde hace 20 días. No había acudido antes porque su novio
había tenido un tipo de hepatitis algunos meses atrás, y éste le recomendó que no
acudiera por tratamiento médico, pues seguramente era un cuadro autolimitado.
Los estudios de laboratorio revelan una concentración de alanina transaminasa de
250 U/L, aspartato transaminasa de 201 U/L, bilirrubina de 7 mg/dL, positividad
para el antígeno de superficie del virus de la hepatitis B y anticuerpo lgM contra el
antígeno central (core) del virus de la hepatitis B. La paciente muestra un claro
cuadro de hepatitis aguda por el virus de la hepatitis B. ¿Cuál de los siguientes
enunciados respecto de la hepatitis tipo B es incorrecto?
b) La administración de interferón en la etapa aguda de la infección evita el
desarrollo del estado de portador crónico del virus de la hepatitis B
107. ¿Cuál de los siguientes fármacos antibióticos ejerce mediante la inhibición de
la enzima DNA topoisomerasa en bacterias?
c) quinolonas
108. Paciente masculino de 31 años de edad que acude a consulta de rutina.
Entre los antecedentes de importancia el paciente refiere que a la edad de seis
años sufrió un cuadro de fiebre reumática aguda, lo cual lo confinó a la cama por
varios meses. Desde entonces ha estado bajo tratamiento antibiótico con
penicilina V oral a dosis de 500 mg al día. Cuatro años después del episodio de
fiebre reumática aguda, es decir, a los 1O años, tuvo un cuadro de exacerbación
relacionada a la suspensión del medicamento antibiótico. Actualmente el paciente
trabaja como enfermero en un hospital pediátrico. A la exploración física presenta
un soplo holosistólico grado 3/4, el cual tiene su punto de mayor intensidad en el
ápex. ¿Cuál de los siguientes enunciados es el más adecuado en relación a la
conducta a seguir en este paciente?
e) El paciente debe continuar tomando penicilina por tiempo indefinido
109. Paciente femenino de 82 años presenta historia de frecuencia urinaria y
urgencia con restos de moderadas cantidades de orina, la cual comienza después
de un evento vascular cerebral tres meses antes. Sin antecedentes familiares de
importancia. A la exploración física nada relevante. Orina residual 35 ml. La
biometría hemática y el examen general de orina son normales. El tratamiento que
más probablemente mejore los síntomas de la paciente es:
a) Oxibutina
110. ¿Cuál de los siguientes enunciados es el que más adecuadamente
representa las características de la fractura de Colles?
b) Fractura de la metáfisis distal del radio en la que existe desplazamiento dorsal
del fragmento distal.
111. ¿Cuál de las siguientes hormonas contribuye a mantener niveles normales de
glucosa en la sangre durante periodos prolongados de ayuno y actúa mediante un
receptor intracelular?
e) Cortisol
112. ¿Cuál de los siguientes síndromes se presenta con tumores desmoides y
osteomas mandibulares?
a) Sindrome de Gardner
113. ¿Cuál de los siguient4es medicamentes representa un alto riesgo de hepatitis
colestásica?
c) Eritromicina
114. ¿Cuál de los siguientes hallazgos incrementa la probabilidad del diagnóstico
de epididimitis bacteriana?
c) Secreción uretral
115. Paciente femenino de 34 años de edad. Inicia su padecimiento tres días
previos a su ingreso con dolor enregión dorsal, progresivo, de tipo punzante,
intenso, de predominio nocturno con irradiación hacia región anterior del tórax, se
administran analgésicos y antiinflamatorios sin mejoría, después se agrega
retención urinaria aguda, debilidad muscular de los miembros inferiores,
parestesias de los miembros pélvicos y en región torácica anterior. Evolucionando
con pérdida progresiva de la fuerza en miembros inferiores y disminución de la
sensibilidad hasta la región torácica. Presenta dolor a la palpación en región dorsal
a nivel de T3-T4. Extremidades superiores con fuerza 5/5, inferior derecha 3/5,
inferior izquierda 1/5, reflejos osteotendinosos de +++ en extremidades inferiores y
++ en extremidades superiores, con respuesta plantar extensora bilateral. Nivel
sensitivo de hipoestesia en T4, incontinencia urinaria y pérdida en el tono del
esfínter anal. Se realizaron exámenes de laboratorio de rutina en rangos normales.
Se programa para tomografía computarizada simple y contrastada de columna
dorsal con foco en T3-T4, sin encontrar lesiones espinales ni lesiones
intrarraquídeas o intramedulares. Se realiza IRM de columna dorsal donde se
encuentra una lesión quística a nivel de T1 con reforzamiento anular y una imagen
hiperdensa dentro de la lesión con edema perilesional compatible con cisticerco
intramedular. En relación a la fisiología de la micción y la incontinencia urinaria
todo es característico EXCEPTO:
d) La incontinencia urinaria aguda tiene un inicio brusco, generalmente en relación
con una enfermedad aguda o problema iatrogénico y no remite tras resolverse la
enfermedad o el problema de medicación. Se cura aproximadamente 10%
116. ¿Cuál de los siguientes mecanismos de acción productora de diarrea
corresponde al de Tropheryma whipplei?
b) colonización intracelular del organismo de las células epiteliales intestinales
117. Paciente masculino de 67 años de edad al cual se le realizó una
colonoscopia como método de detección temprana de cáncer colónico. Se
encontró un adenoma velloso sésil de 1.2 cm en el colon ascendente. La
tumoración fue extirpada. ¿Cuál de las siguientes conductas terapéuticas es la
más adecuada?
b) Repetir la colonoscopia en tres años
118. ¿Cuál de los siguientes organismos se asocia con levaduras de base ancha
en la biopsia de los tejidos infectados?
d) Blastomyces dermatitidis
119. Paciente de cuatro días de edad con deficiencia de la enzima galactora-1-
fostato uridiltransferasa. ¿Cuál es la evolución más probable de este paciente en
caso de que su enfermedad no sea trata oportunamente?
e) El paciente sufrirá insuficiencia hepática.
120. ¿Cuál de los siguientes esquemas profilácticos de tromboembolia venosa es
el más adecuado en pacientes de riesgo moderado para desarrollar esta?
e) la combinación de A y C es correcta
121. ¿Cuál de los siguientes hallazgos histopatológicos óseos son característicos
del hiperparatiroidismo primario?
c) Resorción de huesos subperióstico con degeneracion quística
122. Niño de tres años de edad sin antecedentes de importancia. Desde hace
varias semanas los padres notaron edema facial que después se generalizó. El
niño ha estado hipoactivo con malestar general. Signos vitales normales. El
examen físico confirma la presencia de edema generalizado. Los estudios de
laboratorio revelan albúmina 2.3 g por decilitro y el examen general de orina
muestra proteinuria. Ante la sospecha de un síndrome nefrótico en este paciente,
¿cuál es la enfermedad que con mayor frecuencia produce este cuadro clínico en
niños?
b) Enfermedad de cambios mínimos
123. Ante un niño de 2 días de vida con antecedente diagnostico fibrosis quística,
¿Qué manifestaciones de patología digestiva se pueden encontrar?
a) Íleo meconial y prolapso rectal
124. Paciente masculino de 29 años de edad que acude al servicio de urgencia
por presentar disnea en reposo. Menciona que necesita utilizar tres o cuatro
almohadas al dormir para evitar la disnea. Recientemente el paciente ha notado
que se le hinchan los pies. Entre los antecedentes el paciente niega el consumo
de alcohol, hipertensión o enfermedad arterial coronaria. Hace una semana tuvo
un cuadro de infección de vías respiratorias. A la exploración física la presión es
de 115/76 mm Hg y la frecuencia cardiaca de 105 latidos/minuto. La presión
venosa yugular es de 14 mm Hg. Existen estertores en ambos campos
pulmonares que se encuentran hasta las escápulas. A la auscultación cardiaca no
se encuentran soplos, pero es posible identificar el tercer y cuarto ruido cardiaco.
En las extremidades inferiores tiene edema 2++. El electrocardiograma no muestra
cambios en el segmento ST o en las ondas T. La radiografía de tórax muestra
datos en relación a edema pulmonar y aumento del tamaño de la silueta cardiaca.
Existe elevación leve la fracción MB de la creatinina cinasa. ¿Cuál es la
probabilidad de que este paciente se recupere de este evento?
e) 50%
125. ¿Cuál de las siguientes articulaciones es la más afectada en la enfermedad
por depósito de cristales de fosfato cálcico básico?
a) Hombro
126. Paciente femenino de 29 años de edad, con lateralidad manual derecha, que
se presenta al servicio de urgencias por pérdida súbita de fuerza en hemicuerpo
izquierdo, desorientada y con lenguaje farfulleante. Tiene el antecedente de dos
cuadros similares: el primero hace un año, que provocó hemiparesia faciocorporal
derecha y afasia motora, parcialmente reversible tras 24 horas, dejó como secuela
paresia de ambos miembros derechos; el segundo hace dos semanas, con
desorientación súbita y parálisis de miembro pélvico derecho. Éste último fue
reversible tras 30 minutos. Al interrogatorio a familiares ha cursado con úlceras
crónicas en miembros pélvicos y piel ""amoratada"". En la exploración física existe
tensión arterial 160/110 mm Hg, frecuencia cardiaca 90 latidos por minuto,
frecuencia respiratoria 28 respiraciones por minuto, temperatura 38 ºC. Tiene
puntaje por escala de Glasgow de 13 puntos. Se corrobora hemiparálisis
faciocorporal derecha,signo de babinsky ipsolateral, sin alteración en pulsos
periféricos y los ruidos cardiacos son normales.
126ª. ¿Cuál es el diagnostico mas probable?
d) Evento vascular cerebral isquémico
126b. Además del estudio de tomografía axial de cráneo simple, los estudios de
mayor utilidad para el abordaje diagnostico de esta entidad en un paciente joven
son:
e) A y c son correctos
126d. El aspecto violáceo o amoratado en los miembros pélvicos de la paciencia
en el contexto de su probabilidad diagnostica causal de la trombofilia es:
d) Livedo reticularis
127. Recién nacido de término, con antecedente de distrés respiratorio después
del nacimiento, resultó sin complicaciones. A los 13 días de vida inicia con
distensión abdominal importante, intolerancia a los alimentos con vómito y
evacuaciones hemorrágicas abundantes. Se solicita una radiografía abdominal, en
la que se encuentra neumatosis intestinal.

127ª. En relación con el diagnóstico que sospecha, todas las siguientes


condiciones se asocian a esta patología, EXCEPTO una, indique cuál:
e) Fibrosis quística
127b. Se decide iniciar manejo médico. Todas las opciones que se indican son
correctas EXCEPTO una, indique cuál:
b) Alimentación enteral
127c. ¿Cuál de las siguientes situaciones justificaría el manejo quirúrgico de este
paciente?
b) Neumoperitoneo
127d. Posterior a manejo quirúrgico el paciente egresa y continúa sin
complicaciones los primeros meses de vida. A los cuatro meses, inicia con
distensión abdominal y vómitos persistentes. De acuerdo con los antecedentes,
¿qué complicación puede esperarse en este paciente?
a) Estenosis colónica

--------------------------------------------------------------------------------------------------------------
--------------------------------------------------------------------------------------------------------------
1. Recién nacido de término, con antecedente de distrés respiratorio después del
nacimiento, resultó sin complicaciones. A los 13 días de vida inicia con distensión
abdominal importante, intolerancia a los alimentos con vómito y evacuaciones
hemorrágicas abundantes. Se solicita una radiografía abdominal, en la que se
encuentra neumatosis intestinal.
1a. En relación con el diagnóstico que sospecha, todas las siguientes condiciones
se asocian a esta patología, EXCEPTO una, indique cuál:"
e) Fibrosis quística
1b. Se decide iniciar manejo médico. Todas las opciones que se indican son
correctas EXCEPTO una, indique cuál:
b) Alimentación entera!
1c. ¿Cuál de las siguientes situaciones justificaría el manejo quirúrgico de este
paciente?
b} Neumoperitoneo
1d. Posterior a manejo quirúrgico el paciente egresa y continúa sin complicaciones
los primeros meses de vida. A los cuatro meses, inicia con distensión abdominal y
vómitos persistentes. De acuerdo con los antecedentes, ¿qué complicación puede
esperarse en este paciente?
a} Estenosis colónica
2. ¿Cuál de los siguientes enunciados respecto de la galactosemia es correcto?
b} La galactosemia es el resultado de la deficiencia de galactocinasa o galactosa-
1-fosfato uridiltransferasa

3. Un recién nacido de sexo masculino, con antecedente de síndrome de Down,


presenta datos de obstrucción intestinal al nacimiento, con imagen de dientes de
sierra a nivel de sigmoides en estudio baritado. Respecto a este caso clínico,
señale la opción FALSA.
c} La enterocolitis esinfrecuente como complicación de esta enfermedad

4. Paciente masculino recién nacido producto de un embarazo pretérmino de 29


semanas de gestación y peso de 930 g ha ingresado a la unidad de cuidados
intensivos neonatales. ¿Cuándo se deberá solicitar una consulta oftalmológica
para detectar de manera oportuna retinopatía del prematuro en este paciente?
c} Entre cuatro y seis semanas posteriores al nacimiento

5. Niña de un año ocho meses de edad con antecedente de gastroenteritis viral


hace dos semanas, resuelta sin complicaciones, que inició hace dos días con
flatulencia y evacuaciones diarreicas acuosas y abundantes. Se encuentra en el
examen físico con aumento de los ruidos peristálticos y eritema perianal. En
relación con la enfermedad que presenta, ¿cuál es el tratamiento indicado?

e) Retirar alimentos con lactosa de la dieta


6. Niño de nueve años que inicia con dolor escrotal moderado hace 6 horas que
ha progresado en intensidad hasta ser intenso. Niega traumatismo en la región
escrotal. En el examen físico se encuentra paciente afebril, con signos vitales
estables, escroto de apariencia normal, dolor a la palpación más intenso en el polo
superior del testículo izquierdo, con reflejo cremastérico ausente.
6a. ¿Cuál de los siguientes hallazgos es patognomónica de la patología que
presenta el paciente?
c} Nódulo para testicular en polo superior, azul y puntiforme"
6b. Se solicitó una ecografía Doppler testicular,¿qué hallazgos espera encontrar
de acuerdo con el diagnóstico?"
a} Lesión hipoecogénica en polo superior testicular izquierdo con flujo ausente

6c. ¿Qué tratamiento está indicado para este paciente?


c} Reposo en cama y antiinflamatorios

7. Paciente masculino de 45 años con antecedentes de hipertensión arterial en


tratamiento con captopril, hidroclorotiazida y nifedipina acude a urgencias tras una
semana de no haber tomado ninguno de sus medicamentos por razones
económicas. El paciente refiere que desde hace tres días ha tenido cefalea
progresiva, fosfenos, náusea e irritabilidad. A la exploración física se encuentra
agitado y desorientado, hipertenso con presión arterial de 210/130 mm Hg y el
examen de fondo de ojo revela papiledema y hemorragias, no hay déficit
neurológicos focales."
¿Cuál de las siguientes aseveraciones es falsa con respecto al tratamiento de la
emergencia hipertensiva?
b} El diazóxido cruzala barrera hematoencefálica ejerciendo un efecto directo
sobre la circulación cerebral
8. Una paciente de 32 años con antecedente de lupus eritematoso sistémico
acude a su ginecólogo por abortos espontáneos recurrentes. Ha tenido tres
embarazos, los cuales terminaron en aborto antes de la semana 1O de gestación.
Los padres no tienen ninguna alteración cromosómica y a la madre se le realizó un
histerosalpingograma y USG vaginal, que resultaron normales, además de cultivos
vaginales que fueron negativos. Presenta un VDRL positivo."

8a. ¿Cuál de los siguientes estudios hubiera servido para sospechar el riesgo de
abortos espontáneos?
a} Anticoagulante lúpico
8b. ¿Cuál de las siguientes manifestaciones clínicas es la más común en el
síndrome de anticuerpos antifosfolipídicos (SAAF)?
b} Trombosis venosa profunda
"8c. En cuanto a las manifestaciones cardiacas en el SAAF, ¿cuál es la más
común?"
a} Lesiones valvulares

8d. ¿Cuál es el tratamiento en pacientes con SAAF asintomático?


O a} Sin tratamiento o aspirina a dosis bajas
8e. ¿Cuál de los siguientes no es un criterio de SAAF catastrófico?

b} Confirmación de trombosis venosa recurrente


9. Paciente femenino que se encuentra en puerperio mediato y solicita la
colocación de un dispositivo intrauterino. No tiene contraindicaciones para su
empleo. Usted decide realizar la técnica con insertar. ¿En qué otra situación se
debe utilizar la técnica con insertar?
a} lntergenésico
1O. El siguiente es un método adecuado para el diagnóstico de la fiebre tifoidea:
b} Mielocultivo en la segunda semana
11. Paciente masculino de 55 años de edad que sufrió un infarto con elevación del
segmento ST tratado mediante fibrinolíticos y heparina de bajo peso molecular.
Después del tratamiento menciona que el dolor torácico ha desaparecido, y el
electrocardiograma muestra resolución de la elevación del segmento ST. Tres días
después del infarto inicia con dolor torácico grave, descrito como ""desgarrante"".
¿Cuál de las siguientes opciones explica el cuadro del paciente a tres días del
infarto?"
e} Todas las opciones anteriores pueden ocasionar el cuadro que ahora muestra
el paciente
12. Paciente femenino de 56 años con antecedente de tabaquismo (cuatro
cigarrillos diarios desde hace 15 años) y bronquitis crónica sin tratamiento
específico, inicia hace 24 h con fiebre de 39 ºC, escalofríos, cefalea, mialgias,
artralgias, tos y ardor faríngeo. Actualmente es invierno, por lo que usted
sospecha influenza estacional."
12a. ¿A qué familia pertenece el virus de la influenza?
b} Orthomyxoviridae

12b. ¿Cuál es la complicación más importante de la infección por influenza?


a} Neumonía

12c. ¿Cuál es la prueba más específica y sensible para el diagnóstico?


c} PCR en tiempo real

12d. ¿Cuál de los siguientes medicamentos está contraindicado en pacientes con


influenza menores de 18 años?

c} Aspirina
12e. ¿Qué tratamiento le administraría a la paciente del caso clínico?
c} Oseltamivir

13. Es el mecanismo fisiopatológico por medio del cual el hipercortisolismo induce


hipertensión arterial:
a} Aumento de volumen circulante por la retención de Na
14. Un hombre de 91 años que acude a urgencias por disnea de dos días de
evolución. En su historia clínica se refiere hipertensión, hiperlipidemias, hiperplasia
prostática benigna y enfermedad coronaria, habiéndose sometido a un bypass
coronario. Refiere historia de tabaquismo de 12 paquetes/año, habiendo dejado de
fumar hace 30 años. Sus signos vitales incluyen FC 98/min, FR 23/min, T 36.7 C y
PA de 140/88 mm Hg. A la exploración física se observa distensión venosa
yugular, hepatomegalia y edema 3+ de extremidades inferiores. Los campos
pulmonares se auscultan con murmullo vesicular. El electrocardiograma muestra
datos de hipertrofia ventricular izquierda. Se realiza un ecocardiograma, el cual
muestra una fracción de eyección de 60% con distensión auricular izquierda e
hipertrofia ventricular. Se realiza una radiografía de tórax que muestra datos de
atrapamiento de aire. No se observa enfermedad valvular. ¿Cuál de las siguientes
patologías es la"
causa de estos síntomas?
a} Falla cardiaca diastólica

15. Una mujer de 72 años de edad acude al departamento de urgencias por


disnea y palpitaciones. En sus antecedentes patológicos se refiere EPOC,
hipertensión, hiperlipidemia y enfermedad coronaria, habiendo sufrido un infarto
hace seis años tratado con stent. Sus medicamentos incluyen bromuro de
ipatropio, metoprolol, captopril, simvastatina y oxígeno en casa, el cual mantiene a
3 Umin. A la exploración física sus signos vitales muestran pulso rápido e irregular,
FR 31/min,T 38.2 C, PA 143/88 mm Hg y saturación de oxígeno de 82%. A la
auscultación pulmonar se escuchan roncantes espiratorios. Se realiza un
electrocardiograma, el cual se presenta a continuación:

¿Cuál es el diagnóstico más probable de esta paciente?


e} Taquicardia multifocal paroxística

16. Paciente femenino de 56 años, sin antecedentes de importancia, acude a


consulta por fatiga e intolerancia al ejercicio de una semana de evolución. Refiere
orina oscura en la micción el día anterior. A la exploración física se encuentra
pálida, levemente ictérica, bazo palpable. FC: 11O, FR: 26, Temp: 36 ºC. Se le
realizan estudios de laboratorio, en donde se reporta lo siguiente:"

• Hemoglobina 6.2 g/dL


• Leucocitos 7 OOO/mm3
• Plaquetas 255 000/mL
• Reticulocitos 7%
• Cr .6 mg/dL
• Bilirrubina total 6.4 mg/dL a expensas de la indirecta

16a. ¿Cuál es el diagnóstico más probable?


c} Anemia hemolítica autoinmune
16b. ¿Qué anticuerpos están implicados en la fisiopatología de la enfermedad?
b} lgG

16c. ¿Qué prueba guiaría con certeza al diagnóstico del trastorno?


d} Coombs positivo y esferocitos
16d. ¿Cuál es el tratamiento ideal para esta paciente?
a} Esteroides o rituximab
16e. ¿Cuál de las siguientes anemias hemolíticas no se considera congénita sino
adquirida?
c} Hemoglobinuria paroxística nocturna
17. Un hombre de 73 años acude a consulta por hemoptisis de dos meses de
evolución. Refiere además que ha perdido 8 kg de peso, sin cambiar sus hábitos
alimentarios. Refiere tabaquismo de 40 paquetes/año. En sus antecedentes
patológicos refiere hipertensión y osteoartritis, tratadas con amlodipino y
paracetamol en caso de dolor. A la exploración física sus signos vitales son FC
81/min, FR 18, T 37.2 C y PA 133/82. La auscultación cardiaca revela S1 y S2 sin
agregados. No hay distensión venosa yugular.A la auscultación pulmonar se
escuchan estertores crepitantes en el pulmón inferior derecho. Se realiza placa de
tórax, que revela una masa de 3.5 cm. No se palpa hepatomegal ia o
esplenomegalia. Se solicitan estudios de laboratorio, que se muestran a
continuación:
Examen Resultado
Leucocitos 8 500/mm;j
Eritrocitos 4 200 000/mm;J
Hb 1 16.0 g/dL
HCT 1 48.0%
VCM 1 79 mm3
MHC 1 27 pg
CMHC 1 34 g/dL
RDW 14%
Plaquetas 220 OOO/mm3
Glucosa 123 mg/dL
"Creatinina 1.1 mg/dL
1"
BUN 1 10 mg/dL
Albúmina 4.0 mg/dL
Proteínas totales 6.2 mg/dL
Na 128 mEq/L
K 4.2 mEq/L
CI 104 mEq/L
HC03 24 mEq/L
Na urinario 32 mEq/L
¿Cuál de las siguientes sería la histología más probable de esta paciente?
c} Carcinoma de células pequeñas
18. Una mujer de 80 años acude a consulta por problemas de memoria. Refiere
que los últimos meses ha tenido mayor dificultad de recordar

"personas que conoce, y de eventos que ocurrieron recientemente. Su hija, que la


acompaña, confirma estos acontecimientos. Niega cambios en la personalidad,
desinhibición, temblores, falta de fuerza o parestesias. Aún puede hacer la
mayoría de las actividades básicas, aunque su hija le ayuda a controlar sus
medicamentos. Los antecedentes patológicos de la paciente incluyen hipertensión,
enfermedad coronaria (habiendo sufrido un infarto de miocardio hace tres años,
tratado con stent), e hiperlipidemia. Al IPAS refiere fatiga, sensación de
escalofríos, estreñimiento, resequedad de cabello y de piel. A la exploración física
se observa una paciente obesa. Sus signos vitales incluyen FC 58/min, FR
14/min,T 36.0 C, PA 112/68 mm Hg y saturación de oxígeno de 96%. No se
detectan anormalidades en la exploración neurológica. Al examen de Mini-Mental
califica 27/30, fallando principalmente en su capacidad de recordar objetos. ¿Cuál
de los siguientes estudios daría el diagnóstico de la paciente?

a} TSH

19. Femenino de 24 años, cursa con embarazo de 37 semanas atendida en sala


de urgencias por presentar pérdida del estado de alerta posterior a crisis
convulsivas tónico-clónicas. Antecedentes: G-2, signos vitales con TA 170/120mm
Hg, Fe 95x', reflejos osteotendinosos aumentados, FCF de 132x' y edema
importante de miembros inferiores. La entidad patológica causal de la
sintomatología de esta paciente es:
a} Eclampsia
20. Paciente femenino de 6 años de edad que presenta lesiones dérmicas,
máculas y pápulas, que evolucionan a ampollas en horas. La madre informa que
su hija padeció infección de vías respiratorias superiores hace 2 semanas.

¿Cuál es el diagnóstico en este caso?


c} Varicela
21. Paciente femenino de 32 años, presentó un cuadro gripal hace 2 meses
acompañado de lesiones cutáneas eritematosas nodulares en cara anterior de
miembros inferiores. Un mes después se presenta en consulta por poliartralgias,
fiebre y empeoramiento de las lesiones cutáneas, y una semana después vuelve
con parálisis facial. Usted decide medir la VSG y la proteína C reactiva por
sospecha de sarcoidosis.
21a. ¿Cuál es el síntoma respiratorio más frecuente en la sarcoidosis?
b} Tos seca crónica

21b. ¿Cuáles son los tres principales sistemas orgánicos afectados en la


sarcoidosis?
d} Pulmonar, cutáneo y ocular"

21c. ¿Qué es el síndrome de Lofgren?


a} Eritema nodoso, adenopatías hiliares y artritis"

21d. ¿Qué reporta el estudio histológico de la biopsia de ganglio en la sarcoidosis?


b} Granulomas no caseificantes

21e. ¿Cuál es el tratamiento de primera línea en la sarcoidosis sintomática?


b} Prednisona
22. ¿Cuál de los siguientes eventos es el menos probable que se presente durante
un ataque agudo de asma?
b} La presión pleural durante la inspiración se vuelve más negativa

23. ¿Cuál de los siguientes vasos venosos se encarga de conducir sangre de la


circulación porta en el sistema cava a pesar de la obstrucción de la vena porta (p.
ej.,durante cirrosis hepática)?
e} Venas ácigos y hemiácigos
24. Los pacientes con cáncer de colon no polipósico hereditario presentan genes
con inestabilidad de microsatélites, lo que significa que contienen múltiples
regiones con fragmentos de DNA de cadena única y de estructura anormal. ¿Cuál
de las siguientes es el tipo de proteína anormal en estos pacientes?

a} Enzimas de reparación de apareamiento incorrecto

25. Paciente masculino de 15 años de edad el cual sufre una caída desde su
patineta. A la exploración física presenta una lesión sobre la porción lateral de la
rodilla, la cual permite observar parte de la cabeza del peroné. Se solicita una
radiografía de la región la cual es negativa para fractura. Cuando el paciente trata
de levantarse de la silla de ruedas en la que estuvo todo este tiempo es notable la
presencia de ""pie caído"" al intentar caminar. A la exploración intencionada tiene
incapacidad para la dorsiflexión o eversión del pie. ¿Cuál de las siguientes
estructuras es la que más probablemente se encuentra lesionada?
b} Nervio peroneo común
26. Paciente masculino de 45 años de edad con antecedente de infección por el
virus de la inmunodeficiencia humana se presenta con eritema de 12 mm e
induración de 7 mm a la prueba cutánea de la tuberculina utilizando 5 unidades de
PPD. La radiografía torácica no muestra anormalidades. El paciente se encuentra
bajo tratamiento antirretroviral altamente activo y no existe sintomatología. ¿Cuál
de las siguientes aseveraciones representa la conducta más adecuada?

c}Iniciar tratamiento con isoniazida por 12 meses

27. ¿Cuál de las siguientes condiciones es un factor protector para el desarrollo de


neumonía por Legionella?
c} Neutropenia
28. Paciente masculino de 29 años de edad sufre un accidente por colisión de su
motocicleta contra un automóvil. El paciente se presenta con fractura de la diáfisis
derecha de fémur, sin trauma craneoencefálico ni pérdida del estado de alerta
debido a que utilizaba casco protector; 24 horas posteriores a su admisión el
paciente se presenta con infiltrados pulmonares bilaterales y difusos. ¿Cuál es la
etiología más probable de los infiltrados pulmonares?
c} Embolia grasa

29. Paciente femenino de 78 años de edad con diagnóstico de diabetes mellitus


tipo 2 desde hace 22 años. El paciente carece de apego al tratamiento, y debido a
que le han comentado que la insulina es muy ""dañina"", ha preferido continuar
esporádicamente con tratamiento a base de bebidas de medicina alternativa y
metformina. Desde hace varios meses el paciente notó la presencia de una úlcera
sobre la cabeza del tercer metatarso del lado derecho. La úlcera ha crecido, y
recientemente el paciente inició con dolor en la zona afectada por lo que acude a
consulta. A la exploración física se encuentra una úlcera de aproximadamente 4
cm de diámetro, con olor y secreción pútrida. Al parecer la úlcera tiene una
profundidad de 3 cm y afecta estructuras óseas. Se toma una muestra de la base
y se envía a tinción de Gram la cual reporta cocos grampositivos en cadenas,
bacilos gramnegativos, diplococos gramnegativos, gramnegativos pleomórficos y
exceso de neutrófilos. ¿En este momento, cuál es el tratamiento más adecuado en
lo que es posible obtener mientras se esperan los resultados de hemocultivos y
cultivos de la secreción de la herida?
e} Ampicilina con sulbactam

30. Un paciente se presenta con botulismo posterior a la ingesta de miel


contaminada. ¿Cuál de los siguientes hallazgos es factible en este paciente?

b} Diplopía
31. ¿Cuál es el principal efecto secundario de la administración de zidovudina en
pacientes con infección del virus de la inmunodeficiencia humana?
d} Neutropenia
32. Paciente de ocho años de edad acude a la consulta pediátrica oftalmológica
debido a problemas de visión. A la exploración ocular, se revela una subluxación
del cristalino. Entre otros hallazgos el paciente presenta extremidades largas y
disminución generalizada del tono muscular. Los padres también mencionan que
su hijo presentó retraso en el desarrollo con leve retraso mental. Los estudios de
laboratorio indican elevación de los niveles de homocisteína en sangre y orina.
¿Cuál de las siguientes condiciones es la responsable de los signos y síntomas de
este paciente?
a} Deficiencia de cistationina sintasa

"33. Paciente masculino de 54 años de edad que acude al departamento de


urgencias por dolor torácico de tipo ""desgarrante"" que inició hace 1 hora. El
paciente refiere que el dolor tiene irradiación hacia la región interescapular,y que
sufrió un episodio sincopal de 2 minutos que coincidió con el inicio del dolor
torácico. En el expediente clínico menciona que es conocido por padecer
síndrome de Marfan y dilatación de la raíz aórtica. En sus dos consultas de
seguimiento previas muestra cifras tensiónales elevadas (155/100 y 160/99 mm
Hg). A la exploración física presenta presión arterial de 180/95 mm Hg en la
extremidad escapular derecha y de 100/60 mm Hg en el miembro escapular
izquierdo, así como taquicardia de 105 latidos/minuto; la temperatura y frecuencia
respiratoria se encuentran dentro de los rangos de la normalidad. A la auscultación
cardíaca se puede percibir un soplo diastólico 11/IV en el borde esternal inferior
izquierdo. El ecocardiograma demuestra disección de la aorta torácica con
involucro de la aorta torácica ascendente y descendente."

"Utilizando únicamente la información previamente descrita, responda las


siguientes preguntas:"
33a. ¿Cuál de los siguientes fármacos es el más adecuado para este paciente en
este momento?
a} Labetalol
33b. ¿Cuál de las siguientes opciones corresponde al tipo de de disección aórtica
de este paciente?
a} DeBakey tipo 1
33c. ¿Cuál de las siguientes opciones es el tratamiento definitivo más adecuado
para este paciente?
a} Cirugía urgente.
34. ¿Cuál de los siguientes fármacos actúa mediante la inhibición de la síntesis del
DNA en infecciones fúngicas?
b} Flucitosina
35. Acude a consulta de urgencias paciente masculino de cuatro años de edad
debido a que la madre ha notado que los labios de su hijo se tornaron color azul
después de realizar actividad física, así como dificultad para respirar. La madre
revela que éste no es el primer ataque y habitualmente la coloración azulada labial
desaparece cuando el niño adopta la posición de cuclillas. Se realiza un
ecocardiograma que revela un corazón con morfología de ""bota"" o ""zueco"", lo
cual es característico de la hipertrofia ventricular derecha. ¿Cuál de los siguientes
diagnósticos es el más probable en este niño?
d} Desplazamiento anterosuperior del septo aorticopulmonar

36. Lactante masculino de diez meses es traído a urgencias por su madre por
presentar diarrea acuosa de dos días de evolución asociada a sed intensa e
irritabilidad. Se encuentra taquipneico, taquicárdico, con los ojos hundidos, llanto
sin lágrimas, lengua seca, saliva espesa, signo del lienzo húmedo y llenado capilar
de 4 segundos.
¿Cuál es el tratamiento adecuado para tratar este caso de deshidratación?
e} Administrar Vida Suero Oral 100 ml por kilogramo de peso, cada 30 minutos
durante cuatro horas en la unidad de salud"

37. Paciente masculino de 55 años de edad que es ingresado al servicio de


Cardiología por presentar endocarditis infecciosa. En los cultivos se ha identificado
cepas de Staphy/ococcus aureus; los resultados del antibiograma se encuentran
pendientes. El paciente refiere antecedente de choque anafiláctico a la
administración de penicilina. Utilizando únicamente la información previamente
descrita, responda las siguientes preguntas:
37a. ¿Cuál de los siguientes tratamientos es el más adecuado para este paciente
en este momento?
a} Administración de vancomicina intravenosamente

37b. Posterior a la conducta realizada en la respuesta anterior el paciente


presenta eritema en varias partes del cuerpo, pero particularmente en el cuello, sin
otro tipo de sintomatología. ¿Cuál es la conducta terapéutica más adecuada en
este momento?
b} Disminuir la velocidad de infusión del medicamento

37c. Después de tratamiento adecuado, los síntomas del paciente se resuelven y


es dado de alta. Seis meses después, el paciente acude a consulta y pregunta si
es necesario que tome profilaxis antibiótica pues se someterá a una intervención
dental. ¿Cuál de las siguientes respuestas es la más adecuada para este
paciente?
a} Sí debe someterse a profilaxis previa al procedimiento dental, pues claramente
existe una indicación por sus antecedentes.

37d. En caso de que sea necesario utilizar profilaxis contra endocarditis infecciosa
previamente al procedimiento dental ¿Cuál de las siguientes opciones es la más
adecuada para este paciente?
b} Clindamicina
"38. Te encuentras explorando a una paciente embarazada con 33 semanas de
gestación, con fondo uterino de 26 cm desde sínfisis de pubis, situación transversa
y decides auscultar foco fetal. ¿A partir de qué semana de gestación se puede
percibir el latido fetal a través del Pinard?
d} 20 semanas de gestación

39. Neonato de 12 días de vida, que fue dado de alta en condiciones estables, y a
partir del décimo día de vida presentó episodios de fiebre, con temperaturas entre
38 y 39.7 ºC, hiporreactividad, rechazo al alimento, ¡DATOS DE MENINGITIS! De
los siguientes agentes etiológicos: 1) Staphylococcus epidermidis, 2)
Staphylococcus aureus, 3) L. monocytogenes, 4) H. influenzae. ¿Cuáles dos
agentes se relacionan más con sepsis tardía?
e} 1 y 2
40. Acude a urgencias primigesta de 19 años de edad en segundo periodo de
trabajo de parto. El producto se encuentra en el tercer plano de Hodge con una
variedad de posición occipitoposterior. Lleva una hora en el segundo periodo e
inició con datos de sufrimiento fetal. Sabiendo que no hay anestesiólogo. ¿Qué
acción se realizaría en este momento?
c} Uso de fórceps
41. Llega a urgencias paciente femenino de 38 años de edad primigesta, por
haber presentado crisis tonicoclónica generalizada hace 30 minutos. Refiere el
marido que previamente había tenido una cefalea muy intensa, fosfenos y dolor en
el epigastrio. Se encuentra en la semana 24 de gestación. TA de 180/11O mm Hg,
estuporosa, en anasarca, ROT incrementados. ¿Qué medicamento
anticonvulsivante es el de elección?
d} Sulfato de magnesio
42. Acude a consulta una mujer con amenorrea secundaria, que no menstrúa con
progesterona, pero sí con la combinación de estrógeno y progesterona, con TSH y
prolactina normales, pero con FSH y LH disminuidas y prueba de embarazo
negativa. Con los datos previos. ¿Qué patología podría tener esta paciente?

c} Tumor hipofisiario
43. Es el primer fármaco de elección en el tratamiento de la diabetes tipo 2 y
obesidad.
d} Metformina
44. Prepúber de 12 años de edad con verrugas plantares.

44a. ¿Cuál es el agente causal de esta infección?


a} Virus del papiloma humano
44b. ¿Cuál es el serotipo más prevalente en este tipo de verrugas?
a} 1
44c. ¿Qué le explicarías al paciente con respecto a la evolución y respuesta al
tratamiento?
e} Las verrugas pueden llegar a involucionar espontáneamente
44d. El paciente decide tratarse ya que le molesta mucho y no le gusta. De los
siguientes, ¿cuál es una opción de tratamiento indicada?
b} Crioterapia
45. Hombre de 31 años, con historia familiar de diabetes tipo 2 en padre y abuelo
paterno, hipertensión arterial sistémica en madre, infarto agudo de miocardio en
abuelo paterno; con diagnóstico de hipertensión hace tres años tratado con
enalapril. Desde hace aproximadamente 15 años ha presentado aumento
progresivo de peso (15 kg). A la exploración física: peso 140 kg, talla 1.85 m,
índice de masa corporal 41.22 kg/m2, tensión arterial 110/80 mm Hg, pulso 78x,
con obesidad de predominio central, acrocordones y acantosis pigmentaria en
región posterior del cuello y en ambas axilas, tiroides palpable y normal. Abdomen
globoso por panículo adiposo sin estrías. Fuerza muscular conservada. Resto sin
alteraciones. En sus exámenes de laboratorio destacan: glucosa 99 mg/dl,
creatinina 0.61 mg/dl, urea 37.1, ácido úrico 9.73 mg/dl, AST 54 Ul/L, ALT 65
Ul/L,bilirrubinas y fosfatasa alcalina dentro de parámetros normales, colesterol
total 202 mg/dl, triglicéridos 247 mg/dl.
45a. El diagnóstico sindromático del paciente es:
b} Síndrome metabólico
45b. La presencia de acrocordones y acantosis pigmentaria se debe a:
b} Resistencia a la insulina
45c. ¿Qué examen paraclínico es el más indicado en este caso?
b} Curva de tolerancia a la glucosa oral
45d. El perfilviral de hepatitis es negativo y en un ultrasonido de hígado y vías
biliares no se reporta obstrucción de la vía biliar ni neoplasia hepática. El
diagnóstico más probable es:
c} Esteatohepatitis no alcohólica
46. Paciente masculino de 88 años con múltiples condiciones médicas (angina
estable, enfermedad pulmonar obstructiva crónica, osteoartritis, diabetes mellitus
tipo 2, enfermedad acidopéptica) ingresa a la sala de urgencias por presentar
dolor abdominal de inicio abrupto de 6 horas de evolución. El paciente informa
dolor abdominal difuso, con leve localización en la región periumbilical. A la
exploración física se registran frecuencia cardiaca de 120 latidos por minuto,
presión arterial de 100/60 mm Hg, frecuencia respiratoria de 30 respiraciones por
minuto e hipotermia. A la palpación abdominal se encuentran signos de irritación
peritoneal. Las radiografías de tórax y abdomen demuestran la presencia de aire
subdiafragmático. ¿Cuál es el vaso sanguíneo más frecuentemente afectado en
casos de perforación de úlcera duodenal en la pared posterior del duodeno
(primera porción)?"
b} Arteria gastroduodenal
47. Paciente femenino de cinco años que es traída por sus padres tras ingerir
accidentalmente limpiador de drenaje guardado en una botella de refresco. La
paciente está quejumbrosa, con dolor torácico intenso, náusea, disfagia y sialorrea
profusa. A la exploración física se observan quemaduras de los labios, edema y
eritema de lengua y paladar. ¿Cuál de las siguientes está contraindicada en el
manejo de este caso?
d} Lavado nasogástrico
48. Paciente femenino de 67 años de edad con diagnóstico de hipertensión arterial
sistémica, bajo tratamiento con captopril 25 mg cada 12 horas, actualmente
descontrolada, inicia hace 24 horas con epistaxis profusa que no se autolimita ni
cede a la compresión por lo que acude a urgencias hace 6 horas donde se le
coloca taponamiento nasal anterior y es dada de alta. Tres horas después regresa
por goteo hemático a través del tapón nasal y sensación de descarga hemática
posterior, diagnosticándose epistaxis posterior. ¿Cuál es el origen del sangrado?
b} Arteria esfenopalatina
49. Paciente masculino de 2 años de edad es llevado a consulta de rutina. Se
mencionan como antecedentes que nació a las 37 semanas de gestación, fue
alimentado de manera exclusiva mediante seno materno durante el primer año de
vida y que actualmente no es bueno comiendo. Su peso y estatura se encuentran
en el percentil 5. A la exploración física se observa que sus piernas están algo
arqueadas y el hueso frontal es muy prominente. No tiene dientes y todavía no
camina por sí solo.
49a. Con base en las características del paciente, ¿cuál es el diagnóstico más
probable?
b} Deficiencia de vitamina D C
49b. En relación con su sospecha clínica, ¿cuál de las siguientes opciones
representa una característica particular del padecimiento?
a} Extremos distales ensanchados con forma de copa y con desgaste en la
radiografía
49c. Respecto a su sospecha clínica, ¿qué función tiene en el cuerpo la vitamina
que se encuentra con deficiencia en el paciente?
c} Depósito de calcio

49d. De acuerdo con su sospecha diagnóstica, ¿qué tratamiento le indicaría al


paciente?"
d} Suplemento de vitamina O

49e. Referente a su sospecha clínica, ¿cuál es el mecanismo de prevención que


se debió realizar para evitar esta patología?
b} Exposición al sol
50. Mujer de 54 años de edad con dermatosis bilateral y con tendencia a la
simetría que se disemina a ambas caras pretibiales. Se caracteriza por tres placas
infiltradas amarillo violáceas y esclerodermiformes, irregulares en la forma y mal
delimitadas, que miden entre 5 y 7 cm. La paciente refiere que sus lesiones
iniciaron hace un mes como ""granitos"" de color rojo violáceo y otras como
nódulos de forma redondeada, de bordes no bien definidos y a veces elevados.
Fueron creciendo de forma lenta y excéntrica, tornándose de color amarillo, con
telangiectasias en la superficie. Una de las lesiones se ulceró, dejando cicatriz y
atrofia posterior. A la exploración física presenta en las lesiones sensibilidad
disminuida al pinchazo y al tacto leve, hipohidrosis y alopecia parcial. La paciente
es diabética de larga evolución y al parecer lleva un buen control. Niega
antecedentes de traumatismos o de infecciones previas. El diagnóstico es:
b} Necrobiosis lipoídica
51. Paciente femenino de 32 años de edad, quien tiene como antecedentes
relevantes crisis convulsivas por cisticercosis, previamente tratada con valproato;
hace 4 semanas se hizo cambio por carbamazepina. Hace 3 días inicia con
malestar general, astenia, adinamia, hiporexia y hepatalgia, el día de hoy inicia
con eritema morbiliforme generalizado, motivo por el cual acude a consulta. FC 95
lpm, FR 18 rpm, temp. 38.6ºC, PA 110/70, peso 58 kg, talla 1.68 m.

"51a. Con los datos clínicos anteriores, usted realiza el diagnóstico de:"
b} Síndrome de DRESS
51b. Estadísticamente, ¿qué órgano se afecta con mayor frecuencia en el
síndrome de DRESS?
b} Hígado C
51c. Usted solicita una biometría hemática que reporta eosinofilia > 1.5 x 109 L,
los laboratorios que solicita reportan elevación de TGO, TGP y DHL, con tiempos
de coagulación prolongados. Usted solicita una biopsia de piel, ¿qué hallazgos
espera que se reporten?"
a} Paraqueratosis, acantosis local e infiltrado linfocitario con epidermotropismo

51d. Se debe iniciar el tratamiento, empezando por retirar el fármaco que provocó
el síndrome. Usted inicia el tratamiento con:"
d} Esteroides
51e. Se han documentado diversos datos de severidad del síndrome de DRESS,
¿cuál se asocia con mayor mortalidad?"
b} Necrosis hepática

52. Paciente masculino de 31 años de edad con antecedente de dos semanas de


fiebre, a lo cual se agregó hace una semana distensión abdominal, estreñimiento y
dolor epigástrico. Entre otras molestias también refiere cefalea, escalofríos,
mialgias y malestar general. La exploración física revela la presencia de un
""exantema rosado"" (exantema maculopapular color salmón que blanquea a la
presión), principalmente en el tórax, así como dolor abdominal generalizado a la
palpación y esplenomegalia. Presenta temperatura de 38.9 ºC, frecuencia cardiaca
de 45 latidos por minuto, frecuencia respiratoria de 20 respiraciones por minuto,
presión arterial de 100/65 mm Hg. Los estudios de laboratorio básicos son
relevantes por leucopenia y neutropenia. Se realiza un hemocultivo el cual resulta
positivo para Salmonella typhi. ¿Cuál es el tratamiento antibiótico más efectivo en
cepas no provenientes de brotes resistentes?
a} Fluoroquinolonas
53. Un paciente de 21 años se presenta a la sala de urgencias posterior a una
lesión en su hombro durante un partido de fútbol. A la exploración física el hombro
parece deprimido y plano y no es capaz de abducir su brazo. Una radiografía de la
región demuestra que el paciente tiene fractura del cuello quirúrgico del húmero.
¿De dónde proviene la inervación del músculo afectado en este paciente?

b} C5 a C6
54. Un paciente de 45 años con sobrepeso es llevado a la sala de urgencias con
pérdida del estado de alerta después de un accidente automovilístico. Luego de
recuperar la conciencia, la exploración física revela debilidad de ambas
extremidades derechas con signo de Babinski positivo en el pie derecho. El
paciente muestra la lengua desviada hacia la izquierda cuando la protruye. A la
evaluación de la sensibilidad corporal, el paciente ha perdido la sensibilidad
vibratoria en el lado derecho del cuerpo. ¿Cuál es el sitio más probable de lesión
en este paciente?"
a} Lesión de la región medial de la médula oblongada

55. Paciente femenino de 60 años de edad con diagnósticos previos de


hipertensión, diabetes tipo 2, miocardiopatía isquémica, e insuficiencia renal
crónica; reporta dolor en la rodilla derecha. La presión arterial es de 140/80 mm
Hg, con una frecuencia cardiaca de 66 latidos/min. Debido al edema y al dolor en
la rodilla, se le receta celecoxib 200 mg/día. Después de dos semanas bajo este
tratamiento, la paciente refiere disnea, edema de extremidades inferiores y fatiga.
La presión arterial en esta ocasión es de 180/105 mm Hg, nitrógeno ureico de 67
mg/dL, creatinina sérica de 3.9 mg/dL. ¿Cuál de los siguientes diagnósticos es el
mecanismo de acción más probable por el cual el analgésico administrado
ocasionó insuficiencia renal?"
a} Pérdida del balance entre prostaglandinas, ocasionada por la inhibición de la
ciclooxigenasa tipo 2"
56. Paciente masculino de 36 años, arqueólogo, diabético. Estuvo en contacto con
pacientes con tuberculosis en su último viaje a Zimbabwe. Comenzó con fiebre,
tos productiva con estrías sanguinolentas y ataque al estado general. Durante su
traslado presentó sudores nocturnos. La radiografía de tórax revela infiltrados
nodulares en lóbulo superior, con una lesión cavitada que está drenando a través
de un bronquio. PPD negativo. Se inicia tratamiento antituberculoso. Acude a
consulta a ajuste de hipoglucemiantes orales debido a que su glucosa sérica se
elevo casi el doble de lo que manejaba hace tres meses. Refiere que ha respetado
su dieta y ha estado tomando su medicación. Pocas semanas después del inicio
del tratamiento antituberculoso el paciente reporto disminución de tos e
incremento de energía."
¿Cuál es la explicación de la hiperglucemia en este paciente?
b} Rifampicina
57. Una paciente de 64 años es llevada al hospital por sensación de vértigo y
sordera del oído izquierdo. A la exploración física la paciente presenta sordera
neurogénica del lado izquierdo; analgesia y anestesia térmica del lado derecho del
cuerpo y del lado izquierdo de la cara; depresión del paladar con dificultad para la
deglución; síndrome de Horner izquierdo; tendencia a caerse hacia el lado
izquierdo; nistagmo horizontal.
57a. ¿Cuál de las siguientes estructuras es la responsable de la depresión del
paladar y la dificultad para la deglución?
a} Núcleo ambiguo
57b. ¿Cuál de los siguientes enunciados es correcto en relación con el nistagmo
del paciente?
a} El componente rápido se dirige hacia la derecha

57c. ¿Cuál de las siguientes opciones representa la etiología más probable del
cuadro clínico de esta paciente?
d} Oclusión de la arteria cerebelar posteroinferior izquierda

58. Paciente masculino de 3 años de edad, quien 24 horas previo a la consulta


presentó rinorrea, fiebre y tos no traqueal, el día de hoy se agrega ronquera, tos
""perruna"" y estridor laríngeo en reposo; a la exploración física se encuentra con
adecuado estado mental, ligera palidez y tiraje intercostal bajo, la Sp02 es de
88%. Se niegan antecedentes relevantes. FC 110 lpm, FR 24 rpm, temp. 38.3ºC,
PA 90/60 mm Hg, peso 14 kg, talla 96 cm.
58a. El diagnóstico de este paciente es:
c} Laringotraqueítis
58b. ¿Cuál de los siguientes es un signo de alarma al ingreso?
d} Estridorlaríngeo
"58c. Según los datos obtenidos hasta ahora, usted clasifica este cuadro como:
b} Moderado
58d. El tratamiento farmacológico que usted selecciona es:
d} Glucocortidoides
58e. ¿Cuáles son los datos de alarma para la familia del paciente?
a} Cianosis
59. Paciente masculino de 18 años con antecedentes de anemia de células
falciformes e hipertensión portal es traído a urgencias por presentar hematemesis
masiva. El paciente se encuentra ictérico, taquicárdico e hipotenso. Usted indica
oxígeno por puntas nasales, canalizar dos vías periféricas con catéter de calibre
16, bolos de solución salina y obtiene muestras de sangre para biometría
hemática, estudios de coagulación y cruzar sangre.
¿Cuál es el tratamiento de elección del sangrado de várices esofágicas?
c} Endoscopia con ligadura de várices
60. Paciente masculino de 70 años de edad acude al servicio de urgencias por
dolor torácico. El paciente niega disnea, palpitaciones, náusea, vómito o mareo. El
expediente clínico es relevante por hipertensión, manejada en la actualidad con
diuréticos. La presión arterial del paciente es de 180/100 mm Hg, con una
frecuencia cardiaca de 11O latidos/minuto. La exploración del cuello evidencia un
desplazamiento inferior del cartílago cricoides durante cada contracción, así como
positividad para el signo de Carderelli. ¿Cuál de los siguientes estudios
diagnósticos es el más apropiado en este paciente?
e} Ecocardiograma transesofágico

61. ¿Cuál de las siguientes neoplasias óseas es más probable de presentar


histológicamente osteoclastos?
b} Tumor de células gigantes
62. Paciente femenino de 23 años de edad que ingresa a hospitalización por
presentar desde hace 30 días fatiga y disnea al ejercicio. Entre los antecedentes,
se encuentra una infección de vías respiratorias superiores hace un mes. No hay
antecedentes heredofamiliares de importancia. A la exploración física muestra una
presión arterial de 95/55 mm Hg, y un ritmo cardiaco de 115 latidos/minuto. Las
venas yugulares se encuentran distendidas y la auscultación pulmonar revela
estertores bilaterales; existe tercer ruido cardiaco. Se realiza un ecocardiograma
que demuestra dilatación de ambos ventrículos con hipocinesia difusa y fracción
de eyección del ventrículo izquierdo de 18%. ¿Cuál de los siguientes enunciados
NO es correcto respecto al cuadro de esta paciente?
c} La administración de corticoesteroides puede disminuir la progresión de la
enfermedad
63. ¿Cuál de los siguientes calendarios de administración de la vacuna
neumocócica conjugada (7 valente) es el más correcto según el esquema de
vacunación de México?
e} A los dos y cuatro meses de edad y un año de edad

64. Paciente masculino de 70 años de edad que es llevado al servicio de


urgencias por confusión en las últimas 12 horas. Dos semanas previas el paciente
había sufrido un infarto miocárdico, con colocación de stent. Entre los
medicamentos del paciente se encuentra ticlopidina, aspirina, atorvastatina,
metoprolol y captopril. A la exploración física tiene fiebre de 39 ºC y exantema
petequial difuso. El resto de la exploración física no es relevante. Los estudios de
laboratorio presentan elevación de la creatinina (3.5 mg/dL) y caída en el
hematocrito (22%), con un conteo plaquetario de 45 000 plaquetas/mm3. ¿Cuál de
los siguientes medicamentos es el responsable por el cuadro de este paciente?"

b} Ticlopidina
65. Mujer de 25 años de edad, acude a consulta por presentar dolor en el pie
derecho, con aumento de volumen y tumefacción del primer ortejo, limitación para
la movilidad de las articulaciones que componen ese dedo del pie; al interrogatorio
dirigido niega antecedentes crónico degenerativos, ovolactovegetariana, sin
toxicomanías ni alergias referidas, refiere haber cursado con cuadro infeccioso
gastrointestinal hace 3 semanas completamente remitido y niega antecedente
traumático; a la exploración se encuentra aumento de volumen del dorso del pie,
sin eritema ni hipertermia. FC 75 lpm, FR 18 rpm,temp. 36.5 ºC, PA 110/70 mm
Hg, peso 55 kg, talla 1.65 m.
65a. ¿Cuál es la causa más probable de la artropatía?
c} Artritis reactiva
65b. ¿Cuál de los siguientes agentes está asociado con artritis reactiva?
a} Salmonella
"65c. Con el diagnóstico y cuadro clínico, usted decide iniciar el tratamiento de
primera línea"
d} Antiinflamatorios no esteroides
65d. ¿Cuál de los siguientes criterios no es diagnóstico de artritis reactiva?
b} Afección simétrica
"65e. Además de la artritis reactiva, los pacientes pueden presentar otras
manifestaciones ¿Cuál de las siguientes no está asociada con artritis reactiva?

d} Eritroderma
66. Mujer de 35 años de edad, consulta por malestar general, palpitaciones,
diarrea escasa, fiebre y dolor unilateral en mandíbula y oído derecho. Refiere
haber presentado una infección de vía aérea superior hace un par de semanas.
TA: 130/70, FC: 90 lpm, Temp: 37.0 ºC. A la exploración se encuentra bocio leve y
dolor a la palpación de tiroides de característica nodular. Perfil tiroideo: TSH
suprimida, T4 y T3 elevadas, VSG elevada. Usted sospecha de tiroiditis
subaguda."
"66a. Al realizarle una gammagrafía a esta paciente, ¿qué hallazgos esperaría
encontrar?"
d} Muy baja captación a causa de la lesión de las células foliculares y la supresión
de TSH
66b. ¿Cuál es la base fisiopatológica de la enfermedad de esta paciente?
d} Infección viral
66c. ¿Cuál es el tratamiento indicado en esta paciente?
c} AINE + bloqueador beta
66d. ¿Cuál es el pronóstico de la función glandular en esta paciente?
a} Recuperación completa
67. ¿Cuál de los siguientes medicamentos se caracteriza por presentar infarto
miocárdico como complicación?
c} 5-fluorouracilo
68. ¿Cuál de los siguientes esquemas profilácticos se recomienda para un
paciente con miocardiopatía hipertrófica obstructiva que se va a someter a una
cistoscopia?
O a} No se requiere ningún tipo de profilaxis antibiótica

69. Javier es un paciente masculino de 42 años de edad con antecedente de


diabetes mellitus tipo 2 desde hace 5 años en tratamiento con insulina NPH,
tabaquismo positivo durante 12 años a razón de 5 unidades al día. Hace 2
semanas cursó cuadro de neumonía adquirida en la comunidad recibiendo
tratamiento ambulatorio a base de amoxicilina y ácido clavulánico durante 1O días
con aparente buena respuesta, sin embargo, hace 48 horas presenta fiebre
intermitente cuantificada en 38.5 ºC, disnea de moderados esfuerzos, tos sin
expectoración y dolor en punta de costado en hemitórax derecho que se
incrementa con movimientos respiratorios. A su ingreso a Urgencias diaforético,
con evidencia de disnea, taquicárdico, a la auscultación pulmonar con disminución
del murmullo vesicular subescapular derecho, disminución de transmisión de
vibraciones vocales, percusión mate en la misma región,resto sin alteraciones. Fe
11O lpm, FR 25 rpm, temp. 38.5 ºC, PA 100/60 mm Hg, peso 72 kg, talla 1.72 m.

69a. El diagnóstico más probable es:


c} Empiema
69b. Una vez que se evidencia la ocupación del espacio pleural el cual
compromete la mecánica ventilatoria se debe realizar lo siguiente:

d} Realización de toracocentesis

69c. Una vez que se realiza el estudio citoquímico de líquido pleural se reporta:
proteínas de líquido pleural / proteínas séricas 1.1. Usted considera lo siguiente:

b} Líquido pleural corresponde a exudado


69d. Dentro del estudio macroscópico del líquido pleural se determina la presencia
de pus por lo que el siguiente paso consiste en:
a} Drenaje costal cerrado
70. Paciente masculino de 38 años de edad, miembro de un reclusorio, inicia su
padecimiento con malestar general, fiebre y pérdida de peso y diaforesis;
posteriormente se agrega tos con esputo purulento y con estrías sanguinolentas,
por lo que acude a consultar. A la exploración se registra TA: 135/75, FC: 95 lpm,
Temp: 38 ºC. Estertores inspiratorios en ápice derecho y soplo anfórico en ápice
izquierdo. Laboratorios: hemoglobina 1O g/dL, leucocitos totales 17000, plaquetas
450 000. La radiografía reporta infiltrados en lóbulo superior derecho y cavidades
con niveles hidroaéreos."
70a. Ante la sospecha de tuberculosis, ¿cuál es el estudio recomendado para el
diagnóstico?
c} Basciloscopia seriada

70b. ¿A partir de cuál de los siguientes resultados se considera un PPD positivo


en los familiares con los que vive el paciente?:
b} <!: 5 mm
70c. Según la historia natural de la enfermedad, ¿en qué fase se encuentra el
paciente?"
b} Reactivación tuberculosa

70d. ¿Cuál es el tratamiento indicado en este paciente?


b} lsoniazida-rifampicina-pirazinamida-etambutol por 2 meses y
posteriormentelsoniazida-rifampicina por 4 meses

70c. Sobre los efectos adversos y secundarios de los fármacos antituberculosos,


¿cuál de las siguientes correlaciones es correcta?
a} Etambutol-neuritis óptica

71. Femenino de 45 años de edad es traída por los servicios de emergencia al


hospital donde usted está de guardia. La paciente ha sufrido un accidente con un
coche, se encuentra inconsciente y se observan lesiones graves en todo el macizo
facial y zona del ojo derecho. TA: 130/80, FC: 95x ', FR: 23x ', Temp: 37.SºC.
71a. La fractura más característica de la órbita es:
(' a} Polo superior
o b} Polo inferior
71b. La fractura más frecuente de la órbita, que además puede lesionar la tróclea
del oblicuo superior, es:
c} Medial
71c. ¿Qué huesos conforman la pared inferior de la órbita?
b} Maxilar superior, malar y palatino
71d. Ante un trauma ocular, ¿qué estudio de gabinete se debe solicitar?
d} TC
72. Masculino de 30 años acude a consulta por presentar un cuadro diarreico de 3
días de evolución, entre seis y ocho evacuaciones al día, abundantes, líquidas, sin
presencia de moco o sangre. Presentó vómito en dos ocasiones al inicio, y en las
últimas horas ha mostrado mareo. Niega fiebre. Antecedente: haber asistido a
competencia de triatlón a Nueva Delhi la semana previa al padecimiento actual.

72a. El germen causal de la sintomatología de este paciente es:


d} Vibrio cholerae
72b. Tratamiento de elección para este paciente:
d} Doxiciclina
72c. El cuadro clínico de este paciente es causado por aumento en :
d} cAMP

73. Paciente masculino de seis años de edad con dolor abdominal difuso y
estreñimiento crónicos. Recientemente se ha mostrado muy distraído, con
problemas en su desempeño escolar. El padre del paciente trabaja en una fábrica
de vidrio. Se sospecha de una posible intoxicación por plomo. Señale, cuál es el
mecanismo de acción de este metal.
b} Unión a ligandos sulfhidrilo con inhibición de múltiples enzimas
74. Paciente femenino de 50 años es llevada al servicio de urgencias por
presentar disnea de una semana de evolución que ha progresado hasta ser de
pequeños esfuerzos y se acompaña en ocasiones de palpitaciones y dolor torácico
que dura pocos minutos. La disnea fue máxima esta mañana, asociada con dolor
retroesternal 7/10 que aumenta con la respiración. La paciente no tiene historia de
tabaquismo, niega cirugías recientes. Su esposo refiere que la ha visto cansada y
que ya casi no se levanta de la cama desde que le colocaron una férula en el
tobillo izquierdo debido a que sufrió un esguince hace dos semanas. A la
exploración física es evidente la dificultad respiratoria con uso de músculos
accesorios, signos vitales con frecuencia respiratoria de 26, frecuencia cardiaca
105 lpm, presión arterial 85/50 mm Hg, temperatura 37.5 ºC. Se aprecia
ingurgitación yugular de 8 cm, taquicardia, S1 y S2 sin soplos ni ruidos accesorios,
campos pulmonares bien ventilados y edema de miembros pélvicos,
especialmente del lado izquierdo, con dolor a la palpación de la pantorrilla
izquierda.
La sospecha clínica de que esta paciente presente una embolia pulmonar es:
d} Alta
75. Paciente femenino de cinco años es traída a urgencias por sus padres quienes
la encontraron jugando en un campo que había sido regado con pesticidas. La
paciente presentó inicialmente abundante salivación y lagrimeo, vómito, diarrea e
incontinencia urinaria seguida de confusión y contracciones musculares
generalizadas. A la exploración física se encuentra letárgica, diaforética, con
pupilas mióticas y fasciculaciones.
Usted de inmediato sospecha intoxicación por organofosforados.
¿Cuál es el medicamento adecuado para tratar los efectos nicotínicos en esta
paciente?
e} Pralidoxima
76. Paciente masculino de 35 años, albañil, acude a consulta por presentar prurito
intenso en las manos, así como fisuras y costras entre los dedos de 15 días de
evolución. Se ha aplicado Vitacilina sin mejoría. A la EF se observa dermatosis
diseminada a palmas de manos y a región interdigital, caracterizada por múltiples
placas eritematoescamosas, algunas fisuras y costras hemorrágicas. Niega
ingesta de medicamentos. Comenta que se encuentra bastante preocupado, ya
que no le es posible laborar en la obra ni lavarse las manos porque nota mucho
ardor al contacto con acetona.
76a. ¿Cuál es el diagnóstico de este paciente?
b} Dermatitis por contacto
76c. Dentro de las enfermedades ocupacionales, ¿qué porcentaje representa la
dermatitis por contacto de manos?
a} 25 a 50%
76d. ¿Qué tipo de reacción es la dermatitis de contacto por irritante primario?
d} No inmunitaria
76e. ¿Qué tipo de reacción inmunitaria es la dermatitis por contacto con
sensibilización o alérgica?
d} IV
77. Se trata de Vanesa, una paciente femenino de 27 años de edad previamente
sana, que cursa la semana 24 de gestación, ha acudido a todas la consultas de
control prenatal hasta el momento, en esta ocasión corresponde realizar tamizaje
en búsqueda de alteraciones de la glucosa. La exploración física solo resalta un
abdomen globoso por estado de gravidez, sin edema periférico, sin otros datos
relevantes. FC 90 lpm, FR 20 rpm,temp 36.7 ºC, PA 130/70 mm Hg, peso 69 kg,
talla 1.62 m."
77a. El diagnóstico de diabetes gestacional se define como:
c} Intolerancia a carbohidratos reconocida durante el embarazo
77b. Usted realiza una prueba de carga de glucosa de 75 g en ayuno y curva de 2
horas ¿cuáles son los criterios para considerar una prueba positiva?
d} Niveles de glucosa > 153 mg/dl en 2 horas
77c. Si esta paciente obtuviera un resultado positivo en ambas pruebas de
tolerancia a la glucosa ¿qué modificaciones haría al seguimiento?
b} Realizar 1 cita cada 15 días de la semana 28 a 35 y una semanal de la 36 en
adelante
77d. Se detecta que en ayuno la glucemia es > 95 mg/dL, ¿cuál de las siguientes
terapéuticas será la ideal?
c} Hipoglucemiantes orales
77e. ¿Cuáles son las complicaciones fetales más comunes durante el segudo y
tercer trimestre del embarazo?
d} Macrosomía
78. Paciente femenino de 45 años acude a consulta por ""hinchazón"" de manos
desde hace 5 meses; también ha notado un cambio de textura en su piel, que
ahora está más firme y dura. Tiene antecedentes de enfermedad por reflujo
gastroesofágico y disfagia a sólidos. A la exploración física se nota inflamación en
las articulaciones de sus dedos y presenta nódulos subcutáneos en éstos;
asimismo, se percibe leve edema e induración en ambas manos.
78a. ¿Cuál sería el diagnóstico más probable?
d} Esclerosis sistémica cutánea limitada
78b. De acuerdo con el diagnóstico clínico, ¿cuáles de los siguientes anticuerpos
ayudarían a tener un diagnóstico más certero?:
a} Anticuerpos anticentrómero
78c. ¿Cuál es la fisiopatología de la enfermedad?
d} Proliferación de fibroblastos con aumento en los depósitos de colágeno

78d. En cuanto a las alteraciones renales de esta enfermedad, ¿cuál es la más


característica?"
a} No hay afección renal
79. Masculino de 11 años de edad es traído a consulta por su madre, la cual
comenta que la maestra del niño le ha reportado que lo nota distraído en las
clases y que, por otra parte, en la casa ve la TV muy de cerca; además de que por
la tarde trae los ojos muy rojos. Durante la entrevista usted se da cuenta de que el
niño entrecierra mucho los ojos para observar los objetos en la pared. Al interrogar
al niño, éste comenta que le cuesta trabajo ver de lejos, por lo que se acerca para
ver bien, y que después de ver mucho tiempo la TV le da dolor de cabeza.
79a. ¿Cuál sería su diagnóstico para este paciente?
b} Miopía
79c. ¿Cómo se define un dioptrio?
c} Toda superficie que separa dos medios con distinto índice de refracción
79d. Del sistema óptico del ojo, ¿cuál estructura es la más potente?
o b} Córnea
79e. El tratamiento de primera elección para este paciente sería:
c} Lentes divergentes
80. Paciente femenino de 50 años con dermatosis diseminada que afecta tronco y
extremidades superiores e inferiores caracterizada por ampollas flácidas,
erosiones y costras, algunas sanguíneas y otras melicéricas, y se acompaña de
estomatosis caracterizada por erosiones. No ha recibido tratamiento, refiere fiebre
de hasta 38.5 C. Inició hace alrededor de dos semanas con ampollas que han ido
evolucionando a costras. Las lesiones en boca iniciaron un mes antes y recibió
tratamiento con aciclovir por probable estomatitis herpética, sin mejoría. A la
exploración física muestra además mal estado de hidratación de mucosas y
taquicardia. Los hallazgos histológicos de la biopsia por sacabocado de una de las
ampollas son:
b} Ampolla suprabasal con células acantolíticas y células basales unidas a la
membrana basal

81. Ramiro es un paciente de 62 años de edad, quien inicia desde hace 12 meses
con disminución de la agudeza visual, se ha dado cuenta pues constantemente se
tropieza con objetos cercanos al caminar, tiene miopía pero su última visita al
optometrista no reveló necesidad de nueva graduación a sus lentes; refiere que su
papá tuvo ceguera después de los 75 años de edad; la exploración clínica es
normal. FC 75 lpm, FR 18 rpm, temp. 36.5 ºC, PA 125/65 mm Hg, peso 67 kg, talla
1.72 m."
81a. ¿Cuál es su diagnóstico presuntivo?
a} Glaucoma de ángulo abierto
81b. ¿Cuáles son los mecanismos fisiopatológicos que explican esta enfermedad?
b} Mecánicos, vasculares y apoptóticos
81c. ¿Cuál de los siguientes valores representa hipertensión intraocular?
c} > 21 mm Hg
81d. ¿Cuál de los siguientes fármacos se utiliza para control de presión
intraocular?
c} lnhibidores de la anhidrasa carbónica

81e. En relación al pronóstico ¿cuál de los siguientes es verdadero?


c} La progresión disminuye con el control
82. Paciente masculino de 65 años se presenta en la madrugada al servicio de
urgencias por presentar disnea en reposo y edema remitente de miembros
inferiores pero que va en ascenso. Cuenta con antecedente de hipertensión y
diabetes mellitus tipo 2, así como de tres infartos miocárdicos, de los cuales dos
de ellos no pudieron ser trombolizados porque el paciente llegó fuera de ventana
terapéutica. Hace dos días presentó un cuadro de infección de vías aéreas
superiores, que ha progresado hasta convertirse en una neumonía incipiente.
¿Cuál de las siguientes combinaciones de fármacos, a pesar de ser efectiva en
este paciente, también puede resultar peligrosa?
a} Digoxina + furosemida

83. ¿Cuál de los siguientes antiarrítmicos tiene su principal mecanismo de acción


en células de despolarización automática y de lenta conducción?
e} Esmolol
84. Paciente masculino de dos días de vida el cual presenta acidosis, vómito,
hipotonía y alteraciones neurológicas. Se solicitan estudios de laboratorio los
cuales revelan niveles elevados de lactato y alanina. ¿Cuál de las siguientes
enzimas es la que con mayor probabilidad se encuentra deficiente en este
paciente?
b} Piruvato deshidrogenasa
85. ¿Cuál de los siguientes mecanismos de acción corresponde al de las
estatinas?
b} Inhibición de la enzima hidroxi-metil-glutaril-coenzima A

86. ¿Cuál de los siguientes defectos es ocasionado por ausencia de la formación


del septo aorticopulmonar?
b} Comunicación interventricular de tipo membranosa

87. Una mujer de 18 años de edad se presenta a consulta externa de Obstetricia


en su 18 semana de gestación. Hace tres años, la paciente fue diagnosticada con
depresión moderada y se le recetó paroxetina 20 mg/día, la cual continuó tomando
hasta hace aproximadamente seis meses, debido a que consideró que no
necesitaba el medicamento. La paciente informa que recientemente no ha dormido
""en semanas"", y lo atribuye al embarazo. Al interrogatorio menciona que también
se ha mostrado recientemente letárgica, sin ideación suicida y que desea
alimentar a su futuro hijo con leche de su propio seno. ¿Cuál de los siguientes
esquemas de tratamiento es el más adecuado para el manejo de depresión de
esta paciente?
a} Sertralina 50 mg al día, con consulta psiquiátrica de seguimiento en un mes

88. ¿Cuál de las siguientes cadenas de globinas se produce exclusivamente


durante la etapa embrionaria?
e} Zeta globina

88. ¿Cuál de las siguientes cadenas de globinas se produce exclusivamente


durante la etapa embrionaria?
e} Zeta globina

89. Paciente femenino de 22 años de edad se presenta a consulta por ictericia y


malestar general desde hace 20 días. No había acudido antes porque su novio
había tenido un tipo de hepatitis algunos meses atrás, y éste le recomendó que no
acudiera por tratamiento médico, pues seguramente era un cuadro autolimitado.
Los estudios de laboratorio revelan una concentración de alanina transaminasa de
250 U/L, aspartato transaminasa de 201 U/L, bilirrubina de 7 mg/dL, positividad
para el antígeno de superficie del virus de la hepatitis B y anticuerpo lgM contra el
antígeno central (core) del virus de la hepatitis B. La paciente muestra un claro
cuadro de hepatitis aguda por el virus de la hepatitis B. ¿Cuál de los siguientes
enunciados respecto de la hepatitis tipo B es incorrecto?

O b} La administración de interferón en la etapa aguda de la infección evita el


desarrollo del estado de portador crónico del virus de la hepatitis B

90. ¿Cuál de los siguientes fármacos antibióticos ejerce su efecto mediante la


inhibición de la enzima DNA topoisomerasa en bacterias?
c} Quinolonas

91. Paciente masculino de 31 años de edad que acude a consulta de rutina. Entre
los antecedentes de importancia el paciente refiere que a la edad de seis años
sufrió un cuadro de fiebre reumática aguda, lo cual lo confinó a la cama por varios
meses. Desde entonces ha estado bajo tratamiento antibiótico con penicilina V
oral a dosis de 500 mg al día. Cuatro años después del episodio de fiebre
reumática aguda, es decir, a los 1O años, tuvo un cuadro de exacerbación
relacionada a la suspensión del medicamento antibiótico. Actualmente el paciente
trabaja como enfermero en un hospital pediátrico. A la exploración física presenta
un soplo holosistólico grado 3/4, el cual tiene su punto de mayor intensidad en el
ápex. ¿Cuál de los siguientes enunciados es el más adecuado en relación a la
conducta a seguir en este paciente?
e} El paciente debe continuar tomando penicilina por tiempo indefinido

92. ¿Cuál de los siguientes enunciados es el que más adecuadamente representa


las características de la fractura de Colles?
b} Fractura de la metáfisis distal del radio en la que existe desplazamiento dorsal
del fragmento distal
93. ¿Cuál de las siguientes hormonas contribuye a mantener niveles normales de
glucosa en la sangre durante periodos prolongados de ayuno y actúa mediante un
receptor intracelular?
e} Cortisol

94. ¿Cuál de los siguientes síndromes se presenta con tumores desmoides y


osteomas mandibulares?
a} Síndrome de Gardner

95. ¿Cuál de los siguientes medicamentos representa un alto riesgo de hepatitis


colestásica?
c) Eritromicina

96. ¿Cuál de los siguientes hallazgos incrementa la probabilidad del diagnóstico


de epididimitis bacteriana?
c} Secreción uretral

97. ¿Cuál de los siguientes mecanismos de acción productora de diarrea


corresponde al de Tropheryma whipplei?
b} Colonización intracelular del organismo de las células epiteliales intestinales

98. Paciente masculino de 67 años de edad al cual se le realizó una colonoscopia


como método de detección temprana de cáncer colónico. Se encontró un
adenoma velloso sésil de 1.2 cm en el colon ascendente. La tumoración fue
extirpada. ¿Cuál de las siguientes conductas terapéuticas es la más adecuada?
b} Repetir la colonoscopia en tres años

99. ¿Cuál de los siguientes organismos se asocia con levaduras de base ancha en
la biopsia de los tejidos infectados?
d} 8/astomyces dermatitidis

99. ¿Cuál de los siguientes organismos se asocia con levaduras de base ancha en
la biopsia de los tejidos infectados?
d} 8/astomyces dermatitidis
100. Paciente de cuatro días de edad con deficiencia de la enzima galactosa-1-
fosfato uridiltransferasa. ¿Cuál es la evolución más probable de este paciente en
caso de que su enfermedad no sea tratada oportunamente?
e} El paciente sufrirá insuficiencia hepática

101. ¿Cuál de los siguientes esquemas profilácticos de tromboembol ia venosa es


el más adecuado en pacientes de riesgo moderado para desarrollar ésta?

(' a} Medias de compresión


(' b} No se requiere profilaxis
C c} Administración subcutánea de heparina de bajo peso molecular
(' d} Administración de warfarina para obtener un índice de normalización
internacional (INR} de 1.5 a 2
O e} La combinación de A y C es correcta

102. ¿Cuál es el nervio que se daña con mayor frecuencia durante las inyecciones
intramusculares administradas con técnica deficiente en la región glútea?

d} Nervio ciático

103. ¿Cuál es el anticonvulsivante de elección en caso de crisis convulsivas


parciales complejas?
a} Carbamacepina
104. ¿Cuál de los siguientes hallazgos histopatológicos óseos son característicos
del hiperparatiroidismo primario?
c} Resorción de hueso subperióstico con degeneración quística

105. ¿Cuál de las siguientes opciones representa de mejor forma el significado de


una diferencia entre la concentración de albúmina en suero y líquido de ascitis
mayor a 1?
d} Cirrosis
106. Paciente femenino de 44 años de edad que se sometió a una resección de
cáncer medular tiroideo. Posterior a la cirugía presenta voz de tono nasal. ¿Cuál
de las siguientes estructuras fue la que con mayor probabilidad se lesionó?

e} Nervio laríngeo recurrente

107. Paciente masculino de cinco años de edad que se presenta con debilidad y
calambres musculares después del ejercicio. A la exploración física el paciente no
presenta ninguna anormalidad aparente, ni retraso mental o alteraciones en el
desarrollo. ¿Cuál de las siguientes enzimas es la que más probablemente se
encuentra deficiente en este paciente?
d} Glucógeno fosforilasa

108. ¿Cuál de los siguientes resultados de laboratorio es el que se esperaría


encontrar en un paciente dislipidémico en caso de que éste tenga adecuado
apego y respuesta al gemfibrozil?
b} Disminución intensa enla concentración de triglicéridos, ligera disminución en la
concentración de colesterol-LDL, y un ligero incremento del colesterol-HDL"
109. Paciente masculino de 44 años de edad propietario de una granja. El
paciente acude a consulta porque padece ataques agudos de fiebre, disnea, tos y
leucocitosis cada vez que se encuentra cerca de heno, principalmente cuando
éste está húmedo. ¿Cuál de los siguientes hallazgos en la biopsia pulmonar es
más probable de encontrar?
e} Neumonitis intersticial con infiltración por linfocitos, células plasmáticas y
macrófagos"

11O. Paciente masculino de 70 años de edad con antecedente de hipertensión


arterial, diabetes mellitus y sobrepeso. Es sometido a una endarterectomía
carotídea en la que el nervio hipogloso derecho es lesionado. ¿Cuál de los
siguientes hallazgos es más probable que se pueda observar en este caso?

a} Desviación de la lengua hacia la derecha durante la protrusión


111. ¿Cuál de las siguientes medidas es capaz de disminuir la proteinuria en
pacientes con síndrome nefrótico?
d} Diltiazem
112. ¿Cuál de las siguientes complicaciones se puede presentar en pacientes con
administración crónica de mestranol?
d} Adenoma hepático
113. Ante un niño de 2 días de vida con antecedente diagnostico fibrosis quística,
¿qué manifestaciones de patología digestiva se pueden encontrar?
a} Íleo meconial y prolapso rectal
114. ¿Cuál de las siguientes articulaciones es la más afectada en la enfermedad
por depósito de cristales de fosfato cálcico básico?
a} Hombro
115. Paciente femenino de 29 años de edad, con lateralidad manual derecha, que
se presenta al servicio de urgencias por pérdida súbita de fuerza en hemicuerpo
izquierdo, desorientada y con lenguaje farfulleante. Tiene el antecedente de dos
cuadros similares: el primero hace un año, que provocó hemiparesia faciocorporal
derecha y afasia motora, parcialmente reversible tras 24 horas, dejó como secuela
paresia de ambos miembros derechos; el segundo hace dos semanas, con
desorientación súbita y parálisis de miembro pélvico derecho. Éste último fue
reversible tras 30 minutos. Al interrogatorio a familiares ha cursado con úlceras
crónicas en miembros pélvicos y piel ""amoratada"". En la exploración física existe
tensión arterial 160/11O mm Hg, frecuencia cardiaca 90 latidos por minuto,
frecuencia respiratoria 28 respiraciones por minuto, temperatura 38 ºC. Tiene
puntaje por escala de Glasgow de 13 puntos. Se corrobora hemiparálisis
faciocorporal derecha, signo de babinsky ipsolateral, sin alteración en pulsos
periféricos y los ruidos cardiacos son normales."
115a. ¿Cuál es el diagnóstico más probable?
e} Evento vascular cerebral isquémico
115b. Además del estudio de tomografía axial de cráneo simple, los estudios de
mayor utilidad para el abordaje diagnóstico de esta entidad en un paciente joven
son:
e} a y c son correctas
115d. El aspecto violáceo o amoratado en los miembros pélvicos de la paciente en
el contexto de su probabilidad diagnóstica causal de la trombofilia es:
d} Livedo reticularis
--------------------------------------------------------------------------------------------------------------
--------------------------------------------------------------------------------------------------------------
1. Paciente masculino de 67 años que acude a consulta por fiebre y disnea.
Refiere que inició hace dos días, con fiebre y tos, pero actualmente tiene dificultad
para respirar. También cuenta que expectora flema amarillenta. Cuenta con
tabaquismo de 24 paquetes/año e ingerir tres cervezas diarias. Niega
antecedentes médicos de importancia. El único medicamento que toma es
aspirina. A la exploración física sus signos vitales son FC 101/min, FR 21/min, T
38.6 C, PA 133/80 mm Hg. A la exploración física se auscultan estertores
crepitantes. En una placa de tórax se muestra una consolidación en el lóbulo
medio del pulmón derecho. ¿Cuál es la etiología más común de este padecimiento
en pacientes de este grupo de edad?
d} Streptococcus pneumoniae
2. ¿Cuál de los siguientes anticonvulsivantes produce sedación e inducción del
citocromo P450?
e} Fenobarbital
3. ¿Cuál de los siguientes fármacos puede producir neuropatía periférica?
e} Paclitaxel
4. Acude a consulta de urgencias un paciente del sexo femenino de 38 años de
edad sin antecedentes de importancia. La paciente refiere que tuvo una riña con
uno de sus hermanos; posteriormente cuando trató de salir de inmediato de la
casa, ella fue incapaz de abrir la puerta por un ""ataque"" de debilidad en el brazo
derecho. A la exploración física la paciente presenta una fuerza muscular en el
brazo derecho 0/5 sin alteraciones de los reflejos de estiramiento en la misma
extremidad. ¿Cuál es el diagnóstico más probable en esta paciente?
a} Trastorno conversivo
5. ¿Cuál de las siguientes alteraciones electrolíticas se asocia con la aparición de
la onda de Osborn en el electrocardiograma?
b} Hipercalcemia
6. Paciente femenino de 45 años de edad con antecedente de cáncer de glándula
mamaria. La paciente fue diagnosticada hace dos meses y se
le realizó una mastectomía radical con extirpación de ganglios cervicales.
Posteriormente fue programada para administración de quimioterapia. Posterior a
la administración de quimioterapia la paciente presenta náusea y vómito
incontrolable por lo que se administra un fármaco para disminuir esta
sintomatología, Aunque el medicamento disminuyó la náusea y el vómito, también
ocasionó constipación. ¿Cuál de los siguientes fármacos es el que más
probablemente se le administró a esta paciente?
a} Un antagonista de los receptores de serotonina 5HT3
7. ¿Cuál de las siguientes opciones representa la lesión glomerular intrínseca
primaria más frecuente?
b) Nefropatía porlgA
8. Paciente masculino de 66 años de edad que acude a consulta por presentar
desde hace varias semanas febrícula, poliartralgias, y exantema eritematoso
localizado principalmente en la cara, aunque hace poco tiempo se ha observado
también en la región superior del tórax. Al interrogatorio el paciente menciona que
recientemente sufrió un infarto agudo al miocardio, así como una arritmia (no
especificada en este momento). En los exámenes de laboratorio se revela
positividad para el anticuerpo antinuclear. ¿Cuál de los siguientes medicamentos
es el que está tomando este paciente con mayor probabilidad?"
b} Procainamida
9. Se requiere de su presencia en la sala de parto debido a que se espera el
nacimiento de un masculino de 37 semanas de gestación. Observa los
procedimientos del obstetra, no se sucitan complicaciones y recibe un recién
nacido masculino. Inicia la exploración física de éste y observa movimiento activo,
frecuencia cardiaca de 139 lpm, buen tono muscular, flexión de las cuatro
extremidades, acrocianosis y llanto vigoroso.
9a. De acuerdo con las características del recién nacido, ¿qué puntaje de Apgar
obtiene?
e} 8 puntos
9b. Al recibir a un recién nacido vigoroso como en el caso descrito, ¿cuál de los
siguientes pasos se debe realizar primero?
a} Verificar que la vía aérea esté permeable, secar, y estimular al recién nacido
9c. En caso de no contar con la información de la edad gestacional del recién
nacido descrito anteriormente, ¿cuál de los siguientes signos clínicos le indicaría
mejor que se trata de un recién nacido a término?
c} Tejido mamario palpable
9d. El neonato pasa a cuidados en cuneros y se comienza el protocolo de recién
nacido por las enfermeras, el cual incluye escrutinio de hipoglucemia. En el
neonato a término asintomático, ¿a qué nivel de glucosa debe iniciarse la
evaluación y tratamiento de hipoglucemia?
c} 45 mg/dl
9e. Después de una semana de nacido, el paciente es llevado a consulta de
revisión. ¿Cuál de las siguientes opciones es la correcta respecto al peso de un
recién nacido en la primera semana de vida?
d} Perder aproximadamente 5 a 10% del peso al nacimiento
1. Paciente masculino de 63 años acude a consulta por pérdida de 18 kg de peso
(> 20% peso habitual) y fiebre vespertina de cinco meses de evolución. Dentro de
sus antecedentes tiene un índice tabáquico de cinco. A la exploración física se
observa febril, adelgazado, disneico, frecuencia cardiaca de 100 latidos por
minuto, Tensión arterial de 100/70 mm Hg sin ortostatismo, hepatomegalia. Sin
otras lesiones.
El diagnóstico de tuberculosis se hace por:
d} Estudio bacteriológico
2. Femenino de 54 años de edad acude al servicio de urgencias por presentar
fiebre, ictericia, acolia y coluria además de dolor tipo cólico en hipocondrio
derecho. El diagnóstico más probable en esta paciente es:
a} Colangitis por obstrucción
3. Paciente femenino de 25 años de edad la cual acude a consulta por padecer
dispareunia, fiebre,1 dolor abdominal en hipogastrio principalmente y escasa
secreción vaginal maloliente y sanguinolenta. Entre los antecedentes de
importancia la paciente menciona que es sexualmente activa desde los 14 años de
edad, con múltiples parejas sexuales, y no siempre utiliza protección contra
enfermedades de transmisión sexual. A la exploración física se presenta dolor a la
movilización cervical. Se sospecha enfermedad pélvica inflamatoria. ¿Cuál de los
siguientes organismos se relaciona con esta enfermedad?"
a} Neisseria gonorrhoeae
5. ¿Cuál o cuáles de las siguientes aseveraciones en relación con la
miocardiopatía hipertrófica son las más correctas?
a} Hipertrofia asimétrica, de mayor intensidad en la región septal"

6. Se trata de paciente femenino pretérmino que al nacer manifestó cianosis y


frecuencia cardiaca de 45 latidos por minuto. Se inició
reanimación neonatal procediéndose a ventilación con presión positiva y se
reevaluó después de 30 segundos encontrándose con frecuencia cardiaca de 59
latidos por minuto y coloración rosada generalizada. Señale cuál de los siguientes
es el siguiente paso a seguir de acuerdo al manejo durante la reanimación
neonatal:
e} Dar compresiones torácicas externas

7. Paciente femenino de 71 años de edad que se presenta con hipercolesterolemia


que es controlada con dieta. En esta ocasión la paciente se
"presenta con rigidez y dolor alrededor de las articulaciones de los hombros y
caderas. La paciente menciona que al despertar siempre le cuesta trabajo salir de
la cama. A la exploración física de las articulaciones no se encuentra ninguna
anormalidad, sólo disminución en la fuerza muscular. ¿Cuál de las siguientes
opciones es el mejor estudio que se le debe realizar a esta paciente?"
d} Medición de la velocidad de sedimentación eritrocitaria
1O. Paciente masculino de 29 años de edad que acude al servicio de urgencia por
presentar disnea en reposo. Menciona que necesita utilizar
"tres ó cuatro almohadas al dormir para evitar la disnea. Recientemente el
paciente ha notado que se le hinchan los pies. Entre los antecedentes el paciente
niega el consumo de alcohol, hipertensión o enfermedad arterial coronaria. Hace
una semana tuvo un cuadro de infección de vías respiratorias. A la exploración
física la presión es de 115/76 mm Hg y la frecuencia cardiaca de 105
latidos/minuto. La presión venosa yugular es de 14 mm Hg. Existen estertores en
ambos campos pulmonares que se encuentran hasta las escápulas. A la
auscultación cardiaca no se encuentran soplos, pero es posible identificar el tercer
y cuarto ruidos cardiacos. En las extremidades inferiores tiene edema 2++. El
electrocardiograma no muestra cambios en el segmento ST o en las ondas T. La
radiografía de tórax muestra datos en relación a edema pulmonar y aumento del
tamaño de la silueta cardiaca. Existe elevación leve la fracción MB de la creatinina
cinasa. ¿Cuál de las siguientes"
opciones representa la principal causa de los síntomas de este paciente?
d} Miocardiopatía dilatada idiopática
1. ¿Cuál de los siguientes agentes fibrinolíticos tiene una vida media prolongada,
lo cual permite su administración en forma de bolo?"
c} Tenecteplasa
2. ¿Cuál de los siguientes medicamentos se relaciona con el desarrollo de
leucopenia y exantema eritematoso?
e} Procainamida
3. ¿Cuál de las siguientes descripciones corresponde con los nódulos de Osler?
b} Lesiones pequeñas, dolorosas, elevadas, de color rojo o púrpura, que se
presentan en las falanges terminales de las manos"
4. Paciente femenino de 28 años de edad que acude a consulta por disnea y
malestar general. Al interrogatorio refiere edema de extremidades inferiores,
ortopnea y disnea paroxística nocturna; menciona que ha tenido fiebre intermitente
y escalofríos. La presión arterial es de 150/60 mm Hg y la frecuencia cardiaca de
99 latidos/minuto. La presión venosa yugular se encuentra elevada. Los pulsos de
las arterias femorales y carótida son saltones con un colapso brusco. A la
auscultación se encuentra un soplo 11/IV sistólico de eyección y un soplo 111/IV
diastólico en decrescendo a lo largo del borde esternal izquierdo que aumenta de
intensidad durante el empuñamiento. Se puede percibir un tercer ruido cardiaco.
Existen estertores bilaterales y edema de las extremidades inferiores. No hay
datos de endocarditis infecciosa. No hay datos que sugieran endocarditis
infecciosa y el electrocardiograma no es diagnóstico. La radiografía de tórax
demuestra cardiomegal ia moderada y edema pulmonar intersticial. ¿Cuál sería el
estudio diagnóstico más adecuado en este caso?
b} Ecocardiografía transtorácica
5. Paciente masculino de 20 años que pertenece al ejército acude a urgencias por
presentar tos no productiva desde hace una semana asociada con fiebre, disnea y
dolor torácico al respirar profundamente. Las radiografías PA y lateral de tórax
muestran un infiltrado intersticial bilateral. Usted diagnostica neumonía atípica
adquirida en la comunidad.
¿Cuá es el antibiótico de elección para el tratamiento de la neumonía atípica
adquirida en la comunidad?
c} Azitromicina
6. Masculino de 64 años, consulta por debilidad muscular, fatiga y dolor leve en
hipocondrio derecho. Refiere ser fumador de larga evolución. Menciona que la
fatiga ha ido en aumento de manera paulatina y actualmente se fatiga al realizar
actividades de mínimo esfuerzo, pero no en reposo. A la exploración se registra
TA: 100/90, FC: 95 lpm, ingurgitación yugular leve, estertores finos bibasales,
precordio rítmico, con S3, hepatomegalia, frialdad en extremidades inferiores,
pulso poplíteo disminuido y pulso pedio ausente.
6a. De acuerdo con las características de este caso, ¿cuál es el estudio que
ayudaría a confirmar el diagnóstico y la causa?
a} Ecocarcliografía
6b. ¿Cuál de los siguientes medicamentos ha demostrado disminuir la mortalidad
en insuficiencia cardiaca?
b} Carvedilol
6c. ¿Cuál de las siguientes medidas terapéuticas generales no está indicada en la
insuficiencia cardiaca?
d} Restricción de sodio a 2 g/día en pacientes asintomáticos
6d. De acuerdo con el caso clínico, ¿en cuál estadio de insuficiencia cardiaca de la
AHA/ACC (American Heart Association/American College of Cardiology) se
encuentra el paciente?
c} C
6e. Según la clasificación de la New York Heart Association, ¿en qué clase
funcional se encuentra el paciente?
c} Clase 111
7. Se presenta a consulta neurológica un paciente del sexo femenino de 55 años
de edad la cual presenta alteraciones de la coordinación y marcha tambaleante. A
la exploración física la paciente presenta nistagmo, disartria, y alteraciones en la
prueba dedo-nariz. Los estudios de laboratorio muestran elevación de los
anticuerpos anti-Yo. Se ordena una resonancia magnética la cual revela atrofia de
ambos lóbulos del cerebelo. ¿Cuál es la causa que con mayor probabilidad está
ocasionando la sintomatología de la paciente?
c} Cáncer de glándula mamaria
8. Paciente del sexo femenino de 45 años de edad que se presenta al Servicio de
Urgencias por presentar diarrea con moco y sanguinolenta. Al interrogatorio la
paciente niega cualquier antecedente que pudiese explicar su padecimiento
actual. A la exploración física se encuentra que en las piernas la paciente tiene
lesiones ulceradas con bordes violáceos; al tacto rectal se evidencia la presencia
de sangre en recto. La biopsia de colon revela inflamación en la mucosa y
submucosa. ¿Cuál de los siguientes medicamentos es el más adecuado para el
tratamiento de esta paciente?
b} Sulfasalazina
9. ¿Cuál de las siguientes condiciones ocasiona una disminución de la
osmolaridad urinaria y un incremento del flujo urinario sin cambiar la fracción
excretada de sodio ni la depuración osmolar?
c} Administración de litio
1O. Paciente masculino de 60 años de edad con diagnóstico reciente de
hipertensión arterial sistólica aislada. Después de tres semanas de tratamiento la
presión arterial del paciente se ha reducido a los valores esperados, sin embargo
el paciente menciona que presenta edema tibial bilateraly cuadros de
enrojecimiento facial. ¿Cuál de los siguientes medicamentos es el que más
probablemente está consumiendo este paciente?
a} Amlodipino
11. Paciente femenina de 30 años, acude por astenia y mialgia de varias semanas
de evolución que han progresado en intensidad; recientemente inicia con pérdida
de peso, náusea, dolor abdominal y deseo de consumir sal. TA: 100/60 mm Hg,
FC: 65 lpm; se observa hiperpigmentación en nudillos y pliegues palmares. Labs:
glucosa 64 mg/dl, Na: 132 mmol/L, K: 5.5 mmol/L, pH: 7.32, PaCO: 30 mm Hg,
HC03: 15 mEq/L.
11a. ¿Cuál es el diagnóstico más probable en esta paciente?
a} Insuficiencia suprarrenal primaria
11b. ¿Cuál es la causa más frecuente del padecimiento de esta paciente?
b} Adrenalitis autoinmune
11c. ¿Cuál es el estudio indicado para iniciar el protocolo diagnóstico y el
resultado esperado en la insuficiencia suprarrenal primaria?
a} Cortisol plasmático disminuido, ACTH elevada
11d. Una vez diagnosticada la insuficiencia suprarrenal primaria, ¿cuál es el
siguiente paso para demostrar su causa?
d} Autoanticuerpos suprarrenales y tomografía suprarrenal
11e. ¿Cuál es el tratamiento indicado en la insuficiencia suprarrenal primaria?
a} Hidrocortisona + fludrocortisona
12. ¿Cuál de los siguientes efectos adversos corresponde a la administración de
mercaptopurina?
e} Dolor abdominal eictericia 13.
Paciente femenino de 87 años consulta por molestias al comer y dolor en la boca
del estómago; reporta que al ingerir alimento siente que le queman el estómago y
que tomar una pastilla para la gastritis le confiere alivio. Tiene antecedentes de
artritis desde hace 5 años, por lo que toma una pastilla de 500mg de ASA al día,
más dos de 275mg de naproxeno sódico/300mg de paracetamol. Además, refiere
haber bajado de peso. TA: 140/85, FC: 85x', FR: 18x', Temp: 37°C.
13a. ¿Cuáles su sospecha diagnóstica?
c} Enfermedad ácido péptica
13b. ¿Cuál es la causa más común de sangrado del tubo digestivo alto?
a} Enfermedad ácido péptica
13c. En esta paciente el factor de riesgo más importante es:
e} Uso de AINE
13d. ¿Cuál complicación de esta enfermedad representa mayor peligro (mayor
mortalidad)?
a} Perforación gástrica
13e. El tratamiento más adecuado para esta paciente es:
d} Omeprazol + inhibidores de la COX-2
14. Paciente femenino de 50 años, fumadora actual, en tratamiento de reemplazo
hormonal, se presenta a la sala de urgencias con dolor pleurítico y disnea. Ésta es
la segunda vez que acude al hospital en la última semana; hace cuatro días
acudió por una neumonía lobar de localización inferior en el pulmón derecho, la
cual fue diagnosticada utilizando radiografía de tórax. En esta ocasión a la
exploración física la pantorrilla izquierda de la paciente se presenta con edema y
eritema. Debido a estos datos, se sospecha de embolia pulmonar como la causa
del padecimiento actual de la paciente. ¿Cuál de los siguientes métodos
diagnósticos es el más adecuado en este caso?
e} Tomografía computarizada espiral contrastada del tórax
15. Paciente femenina de 65 años de edad. Consulta por cefalea y disnea de
esfuerzo. Al interrogatorio refiere dispepsia de larga evolución. A la exploración
física se registra TA: 120/80, FC: 11O lpm, palidez de tegumentos, disminución de
la sensibilidad vibratoria en miembros inferiores. Laboratorios: Hb: 5.5 g/dl, VCM:
120 ft, reticulocitos: 2%, LDH aumentada, leucocitos: 3 000, plaquetas: 85 000.
15a. Señale la causa más probable de la anemia en esta paciente:
b} Deficiencia de vitamina 812
15b. ¿Cómo se espera que sean los valores de ácido metilmalónico sérico y la
homocisteína total en un paciente con deficiencia de vitamina 812?
d} Ácido metilmalónico aumentado y homocisteína aumentada
15c. ¿Cuál es el estudio indicado para confirmar el diagnóstico de anemia
perniciosa?
a} Prueba de Schilling
15d. ¿En qué región del sistema gastrointestinal se absorbe la vitamina 812?
d} Íleon
15e. Se confirma el diagnóstico de anemia perniciosa. ¿Cuál es el tratamiento
indicado en el paciente?
c} Vitamina 812 parenteral
16. ¿Cuál de los siguientes enunciados es correcto respecto a la granulomatosis
de Wegener?
b} Los pacientes con enfermedad sinusal limitada se pueden beneficiar del
tratamiento crónico con trimetoprim/sulfametoxazol
17. Paciente masculino de 44 años de edad cuyo conteo más reciente de linfocitos
T CD4+ es de 55 células/mm3. Se decide iniciar tratamiento indicado en este
paciente; sin embargo, después de varias semanas se presenta con anemia
megaloblástica. ¿Cuál de los siguientes fármacos
es el que más probablemente está causando este cuadro?
e} Trimetoprim-sulfametoxazol
18. ¿Cuál de los siguientes antiarrítmicos disminuye la fase O del potencial de
acción?
c} Propafenona
19. Paciente masculino de 45 años de edad con antecedente de alcoholismo
crónico es llevado al servicio de urgencias posterior a sufrir un leve traumatismo al
ser golpeado por un automóvil. Una vez en el hospital el paciente se diagnostica
con una fractura de cadera. Se decide realizar reparación quirúrgica. A pesar que
durante el posquirúrgico inmediato no suceden eventualidades, al cuarto día de
hospitalización el paciente desarrolla mialgias y debilidad. Los estudios de
laboratorio revelan elevación de la creatinina sérica, con hematuria sin glóbulos
rojos.
¿Cuál de los siguientes estudios de laboratorio revelará con mayor probabilidad la
causa de los síntomas de este paciente?
b} Medición de los niveles séricos de fosfato
20. Paciente femenino de 45 años de edad con antecedente de cáncer de
glándula mamaria. La paciente fue diagnosticada hace dos meses y se le realizó
una mastectomía radical con extirpación de ganglios cervicales. Posteriormente
fue programada para administración de quimioterapia. Posterior a la
administración de quimioterapia presenta náusea y vómito incontrolables. ¿Cuál
de los siguientes medicamentos es el más adecuado utilizar en este contexto para
disminuir la náusea y el vómito?
a} Ondansetrón
21. ¿Cuál de las siguientes condiciones se asocia con disminución de la
capacidad pulmonar total, incremento del volumen residual, disminución de la
presión inspiratoria máxima y normalidad del índice fracción espirada de volumen
en el primer segundo/capacidad vital forzada?
d} Miastenia grave
22. Paciente femenino de 60 años de edad la cual es ingresada por cuarta ocasión
durante los últimos tres meses por presentar palpitaciones y disnea secundaria a
una fibrilación auricular con respuesta ventricular rápida. El equipo médico tratante
está considerando administrar amiodarona por tiempo indefinido para evitar este
tipo de ataques. ¿Cuál de los siguientes estudios es recomendable realizar antes
de administrar este medicamento?
a} Pruebas de función tiroidea
23. Paciente femenino de 19 años de edad que se presenta con menorragia desde
su menarca y recientemente ha sido hospitalizada por padecer anemia grave
(hemoglobina 6 g/dl). Los estudios de laboratorio demuestran niveles normales del
conteo plaquetario, tiempo de protrombina y del índice de internacional
normalizado. Entre las anormalidades se encuentran prolongación leve del tiempo
de tromboplastina parcial y del tiempo de sangrado. ¿Cuál es el diagnóstico más
probable en esta paciente?
e} Enfermedad de von Willebrand
24. ¿Cuál de los siguientes trastornos representa una contraindicación para la
administración de concentrados plaquetarios?
e} Púrpura trombocitopénica trombótica
25. ¿Cuál de los siguientes fármacos se asocia con la vasodilatación de la
circulación esplácnica?
e} Dosis bajas de dopamina (2 a 4 µg/kg/min}
26. ¿Cuál de los siguientes medicamentos es una benzodiacepina de larga
duración de acción?
a} Clorodiazepóxido
27. Paciente masculino de 65 años de edad con diagnóstico de diabetes mellitus
tipo 2 y enfisema con antecedente de tabaquismo intenso. El paciente se presenta
al servicio de urgencias por un cuadro neumónico ocasionado por Streptococcus
pneumoniae. El paciente recibe tratamiento antibiótico adecuado y es egresado
sin eventualidades. Al momento del egreso se le recomienda abandonar el hábito
tabáquico.
¿Cuál de las siguientes medidas también se debe realizar en relación a este
paciente previamente a su egreso?
O c} Administrar vacuna contra la influenza y el neumococo
28. Paciente masculino de 31 años de edad que se presenta a consulta por tener
una tumoración en el lado derecho del cuerpo, la cual ha aumentado
considerablemente de tamaño desde que apareció hace tres meses. Entre los
antecedentes de importancia, en este paciente se encuentran tabaquismo intenso
desde los 15 años de edad y consumo moderado de alcohol, llegando a la
embriaguez una vez por mes. Se realiza una biopsia de la lesión, la cual revela
una translocación t(14;18). ¿Cuál de los siguientes productos génicos es el que
corresponde a la translocación cromosómica que presenta este paciente?
a} Sobrexpresión de Bcl2
29. Paciente masculino de 45 años de edad sin ningún antecedente de
enfermedad hepática o digestiva. Durante una exploración física de rutina se
encuentra que tiene una tumoración abdominal en el cuadrante superior derecho y
que existe dolor a la palpación de ésta. Se realiza un ultrasonido el cual revela
hepatomegalia, así como la presencia de múltiples lesiones quísticas hepáticas. Al
interrogatorio el paciente comenta que hace algunas semanas viajó a visitar un
familiar en una granja, y tuvo contacto con perros y ovejas. ¿Cuál de los siguientes
agentes es el que con mayor probabilidad está ocasionando el cuadro de este
paciente?
d} Echinococcus granulosus
30. ¿Cuál es el mecanismo de acción por el que la selegelina puede mejorar la
sintomatología de la enfermedad de Parkinson?
b} Inhibición de la degradación de dopamina por la monoaminooxidasa tipo B
31. ¿Cuál de los siguientes medicamentos se asocia con síndrome nefrótico como
efecto adverso?
c} Sales de oro
32. ¿Cuál de los siguientes trastornos se caracteriza por niveles en rangos
normales de hormona luteinizante y testosterona, aumento de la hormona
foliculoestimulante en combinación con un bajo conteo de espermatozoides?
c} Criptorquidia
33. Paciente de 46 años con diagnóstico de varios años de esquizofrenia. El
médico tratante decide iniciar un nuevo antipsicótico. Dos semanas después del
cambio de antipsicóticos el paciente se presenta a consulta de urgencia con un
cuadro neumónico grave. La citometría hemática revela reducción del conteo de
neutrófilos, basófilos y eosinófilos. ¿Cuál de los siguientes fármacos es el que con
mayor probabilidad pudo haber causado este trastorno?
b} Clozapina
34. Paciente femenina de 35 años, ama de casa, inicia hace 2 meses con malestar
general, fatiga, artralgia en ambos carpos y articulaciones interfalángicas distales,
y rigidez matutina. Dactilitis en el 2do dedo de la mano derecha. Lesiones:
hiperqueratosis y onicólisis en uñas de las manos de más de un año de evolución.
Dolor a la palpación de la articulación sacroilíaca izquierda.
34a. Señale el diagnóstico más probable con base en la clínica de la paciente:
b} Artritis psoriásica
34b. ¿Cuál de los siguientes signos clínicos es característico de la artritis
psoriásica?:
a} Acortamiento de los dedos
34c. Hallazgo radiográfico encontrado en la artritis psoriásica.
d}Imagen de ""lápiz en copa"" en articulaciones interfalángicas"
34d. El diagnóstico de la artritis psoriásica se realiza con base en:
c} La clínica
34e. Tratamiento indicado en esta paciente para evitar la progresión de la
enfermedad.
a} AntiTNF
35. ¿Cuál de los siguientes enunciados respecto a la recurrencia de la infección
por C/ostridium difficile es correcta?
b} Las recurrencias de las infecciones por C/ostridium difícil e se asocian con
complicaciones serias
36. Se presenta a consulta externa paciente del sexo femenino de 20 años de
edad por presentar resequedad y ulceraciones en la piel que cubre las
prominencias óseas, principalmente las rodillas y los codos. Entre los hallazgos de
importancia a la exploración física se pueden observar placas pequeñas grisáceas
en la conjuntiva. ¿Cuál de los siguientes diagnósticos es el más probable en esta
paciente?
b} Deficiencia de vitamina A
37. Paciente femenino de 15 años de edad estudiante de bachillerato que se
presenta a consulta. La paciente presenta antecedente de asma leve intermitente
bajo tratamiento adecuado; sin embargo, desde hace cuatro días ha presentado
fiebre, tos, malestar general y mialgias intensas. Al interrogatorio menciona que
varios profesores y estudiantes de su centro de estudios presentan sintomatología
similar. Al revisar su expediente, el médico observa que esta paciente no recibió
su última dosis de inmunoprofilaxis contra la infección del virus de la influenza.
¿Cuál de los siguientes tratamientos es el más adecuado en ella?
b} Tratamiento sintomático basado en antihistamínicos
38. ¿Cuál de los siguientes organelos se encuentra ausente en los eritrocitos
maduros, y por esta razón éstos son incapaces de sintetizar el grupo heme de la
hemoglobina, a pesar de que se les inyecte el mRNA correspondiente?
e} Mitocondria
39. Paciente femenino de 19 años de edad acude a consulta por tener desde hace
varias semanas fatiga, palpitaciones y palidez. A la exploración física muestra un
síndrome anémico, caracterizado por palidez de mucosas y tegumentos,
taquicardia y soplo sistólico. Los exámenes de laboratorio confirman anemia
megaloblástica. La paciente menciona que está consumiendo antibióticos para
una infección de las vías urinarias. Debido a que en el pasado tuvo una infección
de vías urinarias y recibió una receta con el medicamento adecuado, consume el
mismo medicamento cada vez que tiene una infección de vías urinarias. ¿Cuál de
los siguientes medicamentos es el que con mayor probabilidad está consumiendo?
e} Trimetoprim-sulfametoxazol
40. En un paciente que se presenta con ""pie diabético"", ¿cuál de los siguientes
diagnósticos diferenciales de osteomielitis es el más difícil de descartar a pesar de
la realización de complejos estudios de imagen, como tomografía computarizada,
resonancia magnética, ultrasonido y medicina nuclear ósea?
b} Pie de Charcot
41. ¿Cuál de las siguientes vitaminas contribuye a la carboxilación de residuos de
glutamina de proteínas en el hígado?
o a} Vitamina K
42. ¿Cuál de los siguientes medicamentos es el tratamiento de elección en el
contexto de una intoxicación por propranolol?
c} Glucagon
43a. Con base en el interrogatorio de la paciente, el diagnóstico presuntivo es:
c} Parálisis facial derecha periférica
43b. El término como se le conoce a la parálisis periférica del nervio facial es:
e} Bell
43c. ¿Cuál es el tratamiento para la parálisis facial periférica idiopática?
e} Corticoides
43d. ¿Qué otros cuidados se deben mantener con el paciente que presenta
parálisis facial periférica?:
d} Cuidados de córnea y rehabilitación
43e. El síndrome de Ramsay Hunt se produce por:
b) Varicela zoster
44. ¿Cuál de las siguientes condiciones incrementa el volumen residual del
pulmón?
e} Enfisema pulmonar
45. ¿Cuál de las siguientes anormalidades de la coagulación tiene la prevalencia
más alta en pacientes con tromboembol ismo venoso?
a} Mutación en elfactor V de Leiden
46. Paciente femenino de 75 años con diagnóstico de DM2 desde hace 15
años(en tratamiento con metformina e insulina), hipertensa desde los 45 años de
edad (en tratamiento con nifedipino e hidralazina), fumadora desde hace 45 años
(cinco cigarrillos diarios),acude a consulta por sus propios medios (caminando)
debido a que sufrió el día de ayer una caída desde su propia altura, de sentón, en
el baño. Según refiere, la caída la relaciona con una pérdida reciente de fuerza en
las piernas; también se queja de dolor en la región de las nalgas. No presenta
limitaciones del movimiento y comenta que ha perdido altura de manera reciente.
La EF: 1.58m, 65kg; TA: 140/80; Temp: 36.5ºC. A la exploración se registra dolor
en la zona de las nalgas, movilidad y fuerza de los miembros pélvicos sin
alteraciones, reflejo anal y continencia fecal y urinaria conservadas, así como dolor
a la palpación de la columna lumbar.
46a. ¿Cuál de los siguientes estudios sería de mayor utilidad diagnóstica?
d} Radiografías simples de columna vertebral
46b. En el estudio anterior se evidencia disminución de la altura anterior de los
cuerpos vertebrales T2 y T3, además de una xifosis prominente. De acuerdo con
esto, ¿cuál sería su diagnóstico?
b} Fractura vertebral por compresión
46c. Por el tipo de fractura y los antecedentes, usted solicita una densitometría
ósea, la cual se la reportan con un valor de T=-2.8. ¿Cuál sería su diagnóstico?
c} Osteoporosis
46d. ¿Cuál es un factor de riesgo modificable para osteoporosis?
b} Tabaquismo
47. ¿Cuál de los siguientes hallazgos histopatológicos óseos son característicos
de la deficiencia de vitamina D?
e} Acumulación de matriz extracelular osteoide alrededor de las trabéculas
48. ¿Con cuál de las siguientes condiciones se asocia la enfermedad celiaca?
d} Dermatitis herpetiforme
49. ¿Cuál de los siguientes virus que ocasionan hepatitis requiere coinfección con
el virus de la hepatitis B para establecerse en el hígado?
d} Virus de la hepatitis D
50. Paciente masculino de 30 años de edad el cual se diagnostica recientemente
con diabetes mellitus tipo 2. Para iniciar el tratamiento se inicia un plan de ejercicio
y dieta; sin embargo, el paciente no tiene apego a estas medidas por lo que se
decide iniciar medicamentos hipoglucemiantes. La principal preocupación es que
debido a las actividades profesionales del paciente, éste suele tener periodos no
planeados y prolongados de ayuno, lo cual podría ocasionar episodios de
hipoglucemia en caso de iniciar con fármacos antidiabéticos. ¿Cuál de los
siguientes fármacos predispone al mayor riesgo de hipoglucemia?
b} Gliburida
51. ¿Cuál de las siguientes características está ausente en la insuficiencia
suprarrenal secundaria?
b} Hiperpotasemia
52. Paciente masculino de 55 años de edad el cual se presenta a consulta por
ardor gástrico desde hace cuatro semanas. Tiene antecedente de enfermedad
acidopéptica (úlceras gástricas), así como diabetes de cuatro años de diagnóstico.
El paciente menciona que al mismo tiempo que inició con el malestar gástrico
empezó a presentar saciedad temprana, pérdida de peso, fatiga, sudación cuando
duerme y episodios febriles. Al paciente se le realiza una prueba de urea en
aliento, la cual resulta positiva. ¿Cuál de las siguientes condiciones es la que
ocasiona directamente y con mayor probabilidad todos los síntomas de este
paciente?
b} Linfoma MALToma
53. ¿Cuál de los siguientes hallazgos se encontraría en un paciente son síndrome
de Bernard-Soulier?
b} Tiempo de sangrado prolongado
54. Paciente masculino de 65 años de edad en posquirúrgico mediato de
colecistectomía electiva. Posterior a la administración de un medicamento, el
paciente presenta eritema facial, diaforesis y náusea con pupilas mióticas pero con
adecuada respuesta a la estimulación luminosa; su presión arterial es de 100/75
mm Hg y la frecuencia cardiaca de 50 latidos/minuto. ¿Cuál de las siguientes
opciones representa con mayor probabilidad la indicación del medicamento que le
administró la enfermera a este paciente?"
e} Atonía vesical
55. Se presenta a la sala de urgencias un paciente masculino de 75 años de edad
que fue encontrado desmayado en el baño por sus familiares. Sus hijas refieren
que el paciente es fumador y que previamente se le habían diagnosticado niveles
elevados de colesterol. A la exploración física el paciente presenta presión arterial
de 180/100 y frecuencia cardiaca de 90 latidos por minuto. Cuando se trata de
interrogar al paciente, éste muestra una expresión de confusión; cuando intenta
comunicarse, su habla es extremadamente fluida pero sin propósitos informativos,
sin patrones gramaticales correctos ni significado. ¿Cuál es el sitio lesionado más
probable en este paciente?
a} En la mitad posterior de la circunvolución temporal superior y en la parte
adyacente de la circunvolución temporal media izquierda
56. Paciente masculino de 59 años de edad que acude a evaluación por padecer
disnea de medianos esfuerzos y dolor torácico. En el expediente clínico no hay
ningún antecedente de importancia para este padecimiento. A la exploración física
los pulsos de las carótidas se encuentran retrasados respecto al ciclo cardiaco, así
como disminuidos en intensidad. El punto de máxima intensidad cardiaco se
muestra hiperdinámico, pero sin desplazamiento. A la auscultación se encuentra
un soplo en crescendo-decrescendo a lo largo del borde esternal izquierdo con
máxima intensidad en telediástole. El componente aórtico del segundo ruido
cardiaco se encuentra ausente. En el electrocardiograma se encuentran datos en
relación a hipertrofia ventricular izquierda. La radiografía de tórax no muestra
incremento del tamaño de la silueta cardiaca y campos pulmonares sin
alteraciones. ¿Cuál de los siguientes estudios es el más adecuado para confirmar
el diagnóstico de este paciente?
c} Ecocardiografía
57. ¿Cuál de las siguientes complicaciones del infarto miocárdico se presenta
como dolor torácico recurrente dos días después del infarto?
d} Pericarditis
58. Con relación a la nefropatía por gota:
d} Con el tratamiento adecuado, se mantiene o mejora la función renal en 80% de
los casos
59. Un hombre de 80 años de edad es admitido en un hospital para someterse a
cirugía electiva de reemplazo de rodilla. En su historia médica se incluye
osteoartritis, hipertensión y enfermedad coronaria, habiendo sufrido un infarto de
miocardio con colocación de stent hace siete años. Sus medicamentos incluyen
hidroclorotiazida, metoprolol, aspirina (la cual suspendió una semana previa a la
cirugía) y ocasionalmente paracetamol. Niega uso de alcohol, aunque refiere
tabaquismo de 30 paquetes/año. La cirugía se realiza con éxito, sin embargo,
llaman en la noche porque el paciente se encuentra diaforético. Signos vitales
incluyen FC 102/min FR 25/min, T 36.0 C, PA 142/86 mm Hg. Saturación de
oxígeno es de 90%. Se encuentra consciente, aunque al interrogatorio no está
orientado en tiempo o espacio, se tarda en contestar y no pone completa atención.
Según lo observado en el expediente médico el paciente actualmente vive con su
esposa, con la que lleva casado 43 años. Es capaz de realizar todas las
actividades básicas de vida diaria, y antes de la cirugía, según la hija que se
encuentra presente, no tenía problemas de orientación o memoria.
59a. ¿Cuál es el diagnóstico más probable?
c} Delirio
59b. ¿Cuál de estos estudios de laboratorio es innnecesario para este paciente?
d} Amonio
59c. Los estudios de laboratorio que se realizaron para el paciente resultaron sin
anormalidades. Al interrogatorio con la esposa presente se refiere que el paciente
ha tenido cierto grado de deterioro en los últimos meses. La esposa ha tenido que
ayudarle a controlar sus medicinas, pues el paciente olvida tomarlas o se
confunde de horario. Además, ha tenido dificultad para reconocer a sus amistades
y en dos ocasiones ha requerido la ayuda de un vecino para regresar a casa de su
caminata diaria. La esposa refiere, sin embargo, que aún recuerda sin problema
eventos que ocurrieron mucho tiempo atrás. El paciente se mantiene interesado
en sus actividades diarias y disfruta pasar el tiempo con su familia. Niega
comportamiento extraño, desinhibido, hipersexualidad, pérdida de peso o
temblores de las extremidades. Se ordena una TAC de cráneo, que revela atrofia
cortical difusa. Se sospecha que el paciente tiene algún tipo de demencia
subyacente. ¿Cuál sería el tipo más probable en este paciente?
e} Enfermedad de Alzheimer
59d. El paciente se recupera del episodio de delirio y se le realiza un Mini-Mental,
con resultado de 23/30, falla principalmente en el área de memoria y de
orientación. ¿Qué tratamiento es recomendable para retrasar la progresión de la
enfermedad?
a} Donepezil
60. Paciente masculino de 70 años con antecedentes de hipertensión arterial
acude a urgencias por presentar inicio súbito de dolor intenso en la región
interescapular, desgarrador que se extiende al epigastrio. A la exploración física el
paciente se encuentra taquicárdico y diaforético con presión arterial de 170/102
mm Hg. La radiografía PA de tórax muestra ensanchamiento mediastinal y la
tomografía axial computarizada confirma una disección de la aorta descendente
¿Cuál es el tratamiento inicial más adecuado para este paciente?
b} Nitroprusiato de sodio
61. Se trata de una mujer de 45 años, diabética (tipo 1) insulinodependiente de 12
años de evolución con control metabólico irregular, se presenta a la consulta
quejándose de halos visuales posterior a un descontrol metabólico de 330 mg/dL.
En el examen oftalmológico reporta agudeza visual de ojo derecho (OD) 20/40 que
no mejora con refracción y en ojo izquierdo (01) 20/20, reflejos pupilares normales,
presión intraocular normal en ambos ojos, prueba de ishihara normal en ambos
ojos. En el fondo de ojo derecho se observa papila hiperémica y elevada, con
dilatación de la microvasculatura, no había datos de retinopatía diabética; en ojo
izquierdo se observa papila definida, excavación 20, sin datos de retinopatía
diabética. Se realiza campo visual de Goldman que reporta sólo un aumento de la
mancha ciega de ojo derecho, el ojo izquierdo está sin alteraciones. Ella ya traía
un estudio de TAC que no reportaba alteraciones neurológicas. Se realiza estudio
de FAG en el que se observa mínima fuga a nivel del disco óptico de ojo derecho,
sin datos de fuga por neovascularización. De los siguientes enunciados todos son
falsos, EXCEPTO:"
d} La mayoría de los casos no necesita tratamiento específico
62. Niña de cinco años de edad, que acude a guardería, presenta una dermatosis
diseminada que afecta palmas y plantas, caracterizada por máculas, pápulas y
vesículas ovaladas rodeadas de un halo eritematoso. Además, muestra enantema
en lengua y paladar, caracterizada por vesículas sobre una base eritematosa y
exulceraciones dolorosas. Tres días previos al inicio de la dermatosis presentó
fiebre y ataque al estado general. Dos de sus compañeros de la guardería tienen
lesiones similares, informa la madre, El diagnóstico más probable es:"
e} Enfermedad mano-pie-boca
63. Paciente femenino de 71 años que acude a urgencias por náusea y vómito.
Refiere que ha vomitado en cuatro ocasiones, siendo de contenido alimenticio a
simplemente un líquido claro amarillento. En sus antecedentes lo único que refiere
esosteoartritis, tratada con paracetamol de 750 mg. Al IPAS menciona que
últimamente ha requerido tomar más de este medicamento debido a que su
osteoartritis la ha molestado mucho. Cuenta que ha requerido tomarlo en seis
ocasiones en las últimas 24 horas. La hija, que acompaña a la paciente, sin
embargo cree que lo tomó en más ocasiones. Niega fiebre, dolor abdominal,
diarrea o hematoquezia. A la exploración física sus signos vitales incluyen FC
113/min, FR 19/min,T 37.1 C y PA 115/68 mm Hg. No hay anormalidades a la
exploración física. A continuación se muestran estudios de laboratorio:"
"1
Examen Resultado"
Leucocitos 8 700/mm;j
Eritrocitos 4 300 000/mm;j
Hb 1 12.0 g/dL
HCT 1 36.3%
VCM 1 75 mm;j
MHC 1 25 pg
CMHC 1 32 g/dL
RDW 13%
Plaquetas 270 000/mm;j
"Glucosa 92 mg/dL
1"
"Creatinina 1.0 mg/dL
1"
BUN 1 11 mg/dL
Albúmina 4.3 mg/dL
Prot. totales 6.1 mg/dL
AST 1 30 Ul/L
ALT 27 Ul/L
"Fosfatasa alcalina, 104 U/L"
Bilirrubina 0.7 mg/dL
¿Cuál de las siguientes aseveraciones es correcta respecto a la intoxicación con
paracetamol?
c} El riesgo de muerte por falla hepática es prácticamente nulo si se administra N-
acetilcisteína en las primeras 8 horas posterior a la sobredosis
64. Paciente masculino de 63 años de edad que acude a consulta por dolor
torácico en varias ocasiones, el cual es prolongado y continuo. El paciente
describe el dolor como una sensación ardorosa, que ocurre más frecuente durante
la noche; a veces el paciente se despierta con disnea después de que ya había
conciliado el sueño. Aunque estos episodios no han ocurrido durante el ejercicio,
el paciente evita cualquier actividad física extenuante como medida preventiva. A
pesar de que en el pasado se le recetó nitroglicerina, el paciente no ha adquirido
el medicamento. En una ocasión, un amigo de él le administró nitroglicerina
durante uno de estos episodios de dolor retroesternal, ocasionando desaparición
del dolor. El día de ayer, tuvo una riña con uno de sus hijos, lo cual le ocasiono
dolor torácico de gran intensidad, el cual mejoró cuando se sentó. ¿Cuál de las
siguientes opciones es con mayor probabilidad el diagnóstico de este paciente?
d} Reflujo esofágico y espasmos
65. Paciente masculino de 29 años, carnicero. Acude a urgencias por presentar
tos persistente sin expectoración de seis meses de evolución, acompañada de
dolor retrosternal relacionado con los accesos de tos. Niega disnea, hemoptisis,
fiebre o afectación del estado general. La exploración física reveló presencia de
adenopatías submandibulares e inguinales, siendo el resto normal. La radiografía
de tórax mostró infiltrados alveolares bilaterales en campos superiores y
adenopatías en hilio izquierdo. La baciloscopia del esputo en tres determinaciones
fue negativa para BAAR, al igual que los cultivos para Mycobacteríum
tuberculosis. El Mantoux también fue negativo. Se realizo una fibrobroncoscopia,
con exploración macroscópica normal y la anatomía patológica de la biopsia
bronquial reveló granulomas no necrosantes compatibles con sarcoidosis. Se
inició tratamiento con corticoesteroides orales en dosis descendentes, con buena
respuesta clínica y radiológica. En este caso se confundió el diagnóstico de
tuberculosis con sarcoidosis. ¿Cuáles son los otros diagnósticos diferenciales que
hay que tener en cuenta ante la sospecha de tuberculosis?
d} Bronquiectasias, linfoma, vasculitis, infección pulmonar por hongos, cáncer
pulmonar, sarcoidosis
66. Paciente masculino de 60 años originario de una zona endémica de
tuberculosis, proveniente de zona de bajos recursos. Ha trabajo en una fundidora
por varios años. Dentro de sus antecedentes personales patológicos padece
enfermedad pulmonar obstructiva crónica, ha recibido corticosteroides los últimos
seis meses. Ingreso a piso hace una semana evolucionando mal, con
empeoramiento de la falla respiratoria, ha requerido ventilación mecánica asistida
por una posible neumonía.
66a. El tubo endotraqueal se debería enviar para:
d} Tinción de Ziehl-Neelsen
66b. Si la tinción anterior fue positiva para muchos organismos que se tiñen de
rojo, ¿cuál sería el régimen terapéutico adecuado?
a} Rifampicina,isoniazida, vitamina 86, pirazinamida y etambutol
67. Paciente masculino de 35 años con antecedente de uso de drogas
intravenosas acude a urgencias por presentar fiebre y escalofríos de una semana
de evolución y disnea progresiva. Hace dos días presentó parestesias en el brazo
izquierdo con debilidad de inicio súbito que duró aproximadamente 3 h y se
resolvió de manera espontánea. Sus signos vitales son temperatura de 38.5 ºC,
pulso de 120 latidos por minuto, frecuencia respiratoria de 24 respiraciones por
minuto, presión arterial de 116/76 mm Hg. A la exploración física se ausculta soplo
sistólico 3/6 en el borde inferior esternal derecho y campos pulmonares bien
ventilados. Usted diagnostica endocarditis infecciosa y procede a tomar
hemocultivos previo a iniciar tratamiento antibiótico empírico.
¿Cuál es el agente etiológico más común en usuarios de drogas intravenosas y
que debe ser cubierto por el esquema empírico de antibióticos?
b} Staphy/ococcus aureus
68b. El tratamiento inicial de este paciente debe incluir:
b} Captopril sublingual
68d. Una emergencia hipertensiva debe ser manejada con:
a} Nitroprusiato de sodio intravenoso
69. Paciente masculino de 87 años que acude a consulta por fatiga. Refiere que
desde hace seis meses ha sentido gradualmente menor energía, y al inicio no le
había dado importancia. En sus antecedentes dice tener enfermedad de Crohn,
habiéndose sometido a resección intestinal de íleon y parte del colon hace cinco
años. Cuenta que inició hace unos meses con una terapia de medicina alternativa,
en donde dejó de ingerir productos animales. Niega actualmente síntomas de
diarrea, distensión abdominal, fiebre o sangrado. Lo que sí refiere es inestabilidad,
a veces perdiendo el equilibrio sin sentirse mareado. Sus signos vitales incluyen
FC 100/min, FR 17/min,T 37.2 C y PA 134/88 mm Hg. Se auscultan S1 y S2 sin
agregados y campos pulmonares con murmullo vesicular. No hay anormalidades a
la exploración abdomina En la neurológica se observa disminución a la
sensibilidad de la vibración en extremidades inferiores. La prueba de Romberg es
positiva. Se muestran a continuación los estudios de laboratorio."
Examen Resultado
Leucocitos 9 900/mm;j
Eritrocitos 14 390 000/mm;j
Hb 9.5 g/dL
HCT 29.6%
VCM 110 mm;j
MHC 28 pg
CMHC 33 g/dL
RDW 14%
Plaquetas 300 000/mm;j
¿Cuál es la causa más probable del padecimiento de este paciente?
d} Deficiencia de cobalamina
70. Paciente masculino de 84 años que acude a consulta por dolor abdominal.
Refiere al dolor de tipo punzante, localizado en el hipogastrio. En su historial
médico hay hipertensión, enfermedad coronaria e hiperlipidemias. Sus
medicamentos incluyen metoprolol, lisinopril, aspirina y simvastatina. Al IPAS
refiere dolor ""de los huesos"" en las últimas semanas, principalmente de las
extremidades y la espalda. También al interrogatorio menciona que no ha ido a
orinar en 24 horas. A la exploración física sus signos vitales incluyen FC 104/min,
FR 18/min, T 36.7 C y PA 145/80 mm Hg. No hay anormalidades a la exploración
torácica. En el reconocimiento abdominal se palpa una masa en el hipogastrio,
dolorosa altacto, timpánica a la percusión. Hay edema 1+ de extremidades
inferiores. Al tacto rectal se siente próstata agrandada, con una nodulación. Se
realiza una cateterización, y el examen de orina se observa sin anormalidades. A
continuación, se muestran estudios de laboratorio:"
Examen esultado
Sangre
Glucosa 93 mg/dL
Creatinina 2.1 mg/dL
BUN 40 mg/dL
Albúmina 4.5 mg/dL
Proteínas totales 6.3 mg/dL
Na 143 mEq/L
K 4.8 mEq/L
CI 103 mEq/L
HC03 23 mg/dL
¿Cuál es la causa más probable de la falla renal de este paciente?
e} Nefropatía obstructiva
71. Lo siguiente es cierto sobre la erisipela:
e} Los márgenes de la lesión están bien definidos y sonindurados
72. ¿Qué vasculitis se encuentra relacionada con una respuesta inmunitaria
aberrante a la hepatitis C crónica?
d} Crioglobulinemia mixta esencial
73. De los siguientes microorganismos, ¿en cuál NO se ha demostrado una
relación importante con algún tipo de cáncer?
d} Virus herpes simple tipo 1
74. Femenino de 69 años de edad, se presenta al servicio de urgencias por
padecimiento de 6 h de evolución caracterizado por dolor intenso en 2° dedo del
pie izquierdo, hiperestesia y amoratamiento del mismo. Antecedentes: diabetes
mellitus de 15 años de evolución tratada con metformina + glibenclamida,
hipertensión arterial sistémica de 15 años de evolución controlada con losartán y
metoprolol, tabaquismo crónico intenso desde su juventud hasta la actualidad a
razón de 2 cajetillas por día, claudicación intermitente a más de 300 m.
Exploración física: obesa bien hidratada orientada en las tres esferas, abdomen
blando depresible no doloroso con soplo holosistólico en mesogastrio, pulsos
femorales con soplos bilaterales, buena intensidad, pulsos poplíteos, tibiales
anteriores y posteriores disminuidos, en pie izquierdo se observa 2° dedo con
cianosis intensa, hipotermia de todo el dedo e hiperalgesia.
74a. El mecanismo fisiopatológico responsable del cuadro clínico de esta paciente
es:
d} Ateroembolismo distal
74b. La medida terapéutica indicada en esta paciente es:
d} Manejo conservador e iniciar estudio de patología aórtica
74c. El mecanismo fisiopatológico responsable del cuadro clínico de esta paciente
es:
d} Ulceración y embolización de placa de ateroma
74d. Debido a los antecedentes de la paciente, se decide que es necesario
descartar la presencia de aneurisma de la aorta abdominal. ¿Cuál es el estudio de
elección para corroborar este diagnóstico?
c} Angiotomografía
74e. Según los resultados de los estudios realizados, ¿cuál sería el tratamiento
médico de elección para disminuir la claudicación de la paciente?
d} Medidas de higiene arterial + cilostazol
75. Paciente masculino de 69 años de edad que acude al servicio de urgencias
por presentar disnea. Tiene antecedente de tabaquismo intenso desde los 20 años
de edad. Al interrogatorio menciona que ha perdido alrededor de 9 kg de manera
no intencionada en el último mes y que desde hace tres meses ha tenido tos
crónica. A la exploración física la presión arterial es 88/66 mm Hg, la frecuencia
cardiaca es de 109 latidos/minuto. Existe distensión de las venas yugulares. La
auscultación de los campos pulmonares revela disminución de los sonidos
respiratorios en el pulmón derecho. Los ruidos cardiacos se encuentran
notablemente disminuidos. La radiografía de tórax muestra crecimiento de la
silueta cardiaca y una tumoración en el hilio derecho. ¿Cuál de los siguientes
hallazgos electrocardiográficos se esperaría observar en este paciente?
c} Alternancia eléctrica

75.1. Paciente masculino de 69 años de edad que acude al servicio de urgencias


por presentar disnea. Tiene antecedente de tabaquismo intenso desde los 20 años
de edad. Al interrogatorio menciona que ha perdido alrededor de 9 kg de manera
no intencionada en el último mes y que desde hace tres meses ha tenido tos
crónica. A la exploración física la presión arterial es 88/66 mm Hg, la frecuencia
cardiaca es de 109 latidos/minuto. Existe distensión de las venas yugulares. La
auscultación de los campos pulmonares revela disminución de los sonidos
respiratorios en el pulmón derecho. Los ruidos cardiacos se encuentran
notablemente disminuidos. La radiografía de tórax muestra crecimiento de la
silueta cardiaca y una tumoración en el hilio derecho. ¿Cuál de los siguientes
hallazgos se esperaria encontrar en este paciente?
e) Ninguna de las opciones anteriores se esperaria encontrar en este paciente

76. ¿Cuál es la producción normal de cortisol al día?


c} 8 a 25 mg en 24 horas
77. ¿Por qué los pacientes con insuficiencia suprarrenal de origen central no
cursan con desequilibrio hidroelectrolítico?
c} No se observa hipopotasemia ni deshidratación
78. Paciente femenina de 65 años de edad. Nivel socioeconómico bajo.
Antecedente de hipertensión arterial sin tratamiento. Acude a consulta por fatiga y
malestar general de 3 meses de evolución. TA: 140/90, palidez de tegumentos,
edema. Laboratorio reporta BUN elevado, creatinina 2.8 mg/dl, anemia
normocítica-normocrómica. Riñones pequeños por ultrasonido. Filtrado glomerular
por creatinina sérica estimado en 35 mUmin/1.73 m2 .
78a. Según la clasificación KDOQI de la enfermedad renal crónica, ¿en qué
estadio se encuentra la paciente?
b} 111
78b. Señale la causa principal de enfermedad renal crónica:
a} Nefropatía diabética
78c. Señale las alteraciones electrolíticas y metabólicas con las que podría cursar
la paciente:
a} Hiperpotasiemia, acidosis metabólica, hiperfosfatemia, hipocalciemia"
78d. ¿Cuál es la principal causa de muerte en la insuficiencia renal crónica?
c} Anomalías cardiovasculares
78e. ¿Cuál de las siguientes opciones no corresponde a una indicación de
diálisis?:
a} Edema
79. Masculino de 45 años de edad, con antecedente de fiebre reumática. Acude a
consulta por fatiga de varios meses de evolución y reciente inicio de palpitaciones
y disnea de esfuerzo. A la exploración física se registra TA: 120/80, FC: 88 lpm,
pulso irregular. Campos pulmonares sin ruidos agregados. Choque de la punta
hiperdinámico. Se ausculta un soplo holosistólico intenso descendente irradiado a
axila, S1 ausente.
79a. ¿Cuál es la patología más probable en este paciente?
a} Insuficiencia mitral
79b. ¿Cuál es el mecanismo fisiopatológico más común de la insuficiencia mitral?
c} Prolapso de la válvula mitral
79c. ¿Cuál es el estudio indicado para continuar con el abordaje diagnóstico y
terapéutico en este paciente?
c} Ecocardiograma
79d. Se realiza un ecocardiograma confirmando insuficiencia mitral con FE de
40%, ¿cuál es el tratamiento indicado?
b} Tratamiento quirúrgico conservador
79e. ¿Cuál de los siguientes hallazgos en la exploración nos orienta más hacia el
diagnóstico de insuficiencia mitral?:
c} Aumento de la intensidad del soplo al apretar los puños
80. Una mujer de 65 años de edad acude a consulta para revisión médica general.
En su historia clínica refiere hipotiroidismo, tratado con levotiroxina, y asma leve
intermitente, que maneja con albuterol inhalado. Es viuda y actualmente vive sola.
Niega cualquier síntoma al IPAS. A la exploración física sus signos vitales son FC
88/min, FR 16/min, T 37.0 C y PA 142/80 mm Hg. En las extremidades superiores
se observan nódulos de Heberden y de Bouchard. A la auscultación cardiaca se
escuchan S1 y S2 sin agregados. Los campos pulmonares se auscultan con
murmullo vesicular. ¿Cuál sería el siguiente paso en el manejo de esta paciente?
d} Citar en dos semanas para volver a tomar la presión arterial
81. Masculino de 32 años, llevado a servicio de urgencias presentando: colapso
pulmonar completo con desplazamiento de mediastino, disminución del retorno
venoso e inestabilidad hemodinámica. Los datos clínicos de este paciente
corroboran el diagnóstico de:
d} Neumotórax a tensión
82. ¿Cuál de los siguientes anticonvulsivantes tiene una alta tasa de asociación
con el síndrome de Stevens-Johnson?
b} Etosuximida
83. Acude un masculino de 70 años a consulta de control, con antecedentes de
tabaquismo, etilismo, diabetes, hipertensión, hiperlipidemia y angina de pecho,
toma metformina, lisinopril, metoprolol, atorvastatina, nitroglicerina y aspirina.
Puede llevar a cabo todas sus actividades básicas sin problema. Se puso la
vacuna del neumococo hace cinco años, la de herpes zoster hace dos años y se
aplica la vacuna de influenza anualmente. Niega cualquier síntoma, excepto por
ocasionales dolores articulares. Su angina de pecho ha estado estable, sin
disminución de su capacidad física o aumento de frecuencia de sus síntomas. Sus
signos vitales son FC 76/min, FR 16/min, T 37.1 C y PA 128/82. A la auscultación
torácica es normal, en el abdominal se palpa una masa pulsante en mesogastrio, y
se ausculta un soplo que coincide con la sístole. Se confirma por ultrasonografía
un diámetro de 3.5 cm ¿Qué atención se le dará a este paciente?"
c} Observación para reevaluación en seis meses
84. Masculino de 22 años presenta síndrome nefrítico, la presencia de anticuerpos
anticitoplásmicos de neutrófilos con patrón citoplasmático (cANCA) positivos, nos
orienta hacia el diagnóstico de:
a} Granulomatosis de Wegener
85. ¿Cuál de los siguientes fármacos puede producir ototoxicidad?
b} Cisplatino
86. Se presenta al servicio de urgencias un paciente masculino de 22 años de
edad, el cual es traído por su novia. Ella refiere que habían estado en una fiesta
tipo rave y que al parecer el paciente ingirió algún tipo de droga. El paciente
presenta midriasis, taquicardia, hipertensión e incremento de la tensión de los
músculos mandibulares. ¿Cuál de las siguientes opciones es la droga que más
probablemente consumió este paciente?
a} Éxtasis
87. Una mujer de 65 años acude a consulta por lumbalgia. En sus antecedentes
patológicos refiere haber padecido osteoartritis, depresión y cáncer de mama,
diagnosticado como carcinoma invasivo ductal, tratado con lumpectomía y
disección axilar, quimioterapia y radioterapia hace seis años. Refiere que el último
mes ha padecido un dolor en la espalda baja, que se irradia hacia el pie izquierdo.
Niega debilidad, cambios de la sensibilidad, parestesias o incontinencia urinaria.
Niega pérdida de peso. A la exploración física, sus signos vitales incluyen FC
82/min, FR 18/min, T 36.2 C y PA 132/78 mm Hg. Se detecta un signo de Lasegue
positivo al elevar la pierna izquierda. El resto de la exploración neurológica se
reporta sin anormalidades. La radiografía revela leve osteofitosis. ¿Cuál sería el
siguiente paso en el manejo de la paciente?
b} Obtener resonancia magnética
88. ¿Cuál de los siguientes marcadores tumorales se eleva en los cánceres
ováricos?
a} CA-125
89. Paciente masculino de nueve años de edad que refiere desde hace una
semana malestar faríngeo, así como cefalea y dolor desde hace un día en la
región de la cadera izquierda. En la radiografía de cadera no se observan
anormalidades, sin embargo un aspirado guiado por ultrasonido demuestra la
presencia de un líquido estéril. Inmediatamente después de la aspiración los
síntomas mejoran, a pesar de que marcadores de inflamación se encuentran
elevados (velocidad de sedimentación eritrocitaria de 60 mm durante una hora y
proteína C reactiva de 77 mg/L). Se realiza diagnóstico de artritis viral y es dado
de alta. Tres días después el paciente regresa por dolor en ambos tobillos,
febrícula y eritema en tórax. A la exploración física presenta un soplo sistólico
grado 2/6 en la región de la base. Un ecocardiograma demuestra derrame
pericárdico, únicamente con electrocardiograma normal. En cuanto a estudios de
laboratorio presenta un título de antiestreptolisina O de 1200 unidades Todd y de
antiDNAsa B de 1 920 unidades Todd, así como cultivo faríngeo positivo para
estreptococo hemolítico del grupo A.
89a. ¿Cuáles son los criterios mayores para el diagnóstico de fiebre reumática?
b} Cultivo faríngeo positivo para estreptococo del grupo A o prueba de antígeno
rápida, o elevación de los títulos de anticuerpos contra componentes
estreptocócicos
89b. ¿Cuál es el tratamiento más adecuado en este paciente?
a} Penicilina benzatínica y salicilatos orales en altas dosis
89c. En una evaluación posterior, se determina que el paciente no presenta daño
valvular cardiaco. ¿Cuál es la duración recomendada del tratamiento profiláctico?
b} Diez años o hasta la edad adulta (la que sea de mayor duración)
90. Paciente del sexo femenino de 25 años de edad que acude a consulta médica
porque desde hace tres meses la paciente presenta fatiga, lo cual ha repercutido
en su desempeño académico. Al interrogatorio la paciente describe que ha perdido
aproximadamente 8 kg en dos meses. A la exploración física la paciente presenta
adenomegalias en la región posterior del triángulo cervical y en la axila izquierda.
Se realiza una biometría hemática, la cual sólo es relevante por conteo leucocitaria
cercano al rango superior de normalidad. El médico decide realizar una biopsia de
ganglio linfático; los resultados de la biopsia revelan la sobrexpresión de la
proteína antiapoptótica bcl-2. ¿Cuál es la translocación que más frecuentemente
se presenta en este tipo de neoplasia?
d} t(14:18}
91. Una mujer de 65 años acude a consulta para revisión general. Es una persona
sana, refiriendo sólo dolores articulares intermitentes e hipotiroidismo. Sus
medicamentos incluyen levotiroxina y ocasionalmente paracetamol. Refiere
tabaquismo de 20 paquetes/año. Niega cualquier síntoma al IPAS. Sus signos
vitales incluyen FC 82, FR 16, T 37.3 C y PA 120/67. Su preocupación principal es
la osteoporosis. ¿Cuál de las siguientes situaciones es indicación de densitometría
ósea?
e} Mujeres que han tenido terapia de reemplazo por periodos prolongados

92. Femenino de 43 años es ingresada al servicio de Medicina Interna por


cetoacidosis. Posterior a su recuperación metabólica inicia con fiebre, cefalea,
dolor facial, disminución del nivel de conciencia y enrojecimiento nasal con lesión
negruzca en fosa nasal derecha. Antecedentes: portadora de DM tipo 2y HTAS.
La entidad causal de la sintomatología de esta paciente es:
b} Infección por Mucor
93. ¿Cuál de las siguientes opciones es una manifestación de la deficiencia de
cobalamina?
b} Manifestaciones neuropsiquiátricas
94. Paciente femenino de 31 años de edad sin antecedentes cardiovasculares,
presenta cuadro de 2 meses de evolución que consiste en fiebre, pérdida de peso,
artralgias y anorexia. Es atendida en Urgencias, donde se le diagnostica EVC
isquémico. Al tomar la TA se encuentra una discrepancia entre la TA del brazo
derecho y la del izquierdo, con disminución marcada de pulsos en miembro
superior derecho. En la biometría hemática se registra ligera anemia normocítica
normocrómica y elevación de VSG.
94a. El diagnóstico más probable es:
b} Arteritis de Takayasu
94b. Aparte de la aorta, ¿cuál es la arteria que se afecta con mayor frecuencia en
este padecimiento?
d} Subclavia
94c. ¿Cuál de los siguientes no es un criterio de clasificación de la enfermedad de
la que trata el caso clínico?
c} Livedo reticular
94d. Si se tomara una biopsia de la zona afectada, ¿qué reportaría el patólogo?
d}Infiltrado celular con granulomas y células gigantes
94e. Esta enfermedad tiene tres fases, ¿cuáles son las características de la
segunda fase?
b} Disminución y discrepancia de los pulsos en las extremidades, equimosis,
claudicación, hipertensión renovascular, síncope neurogénico
95. Un paciente masculino de 15 años de edad acude a consulta pediátrica de
seguimiento debido a su padecimiento actual, el cual le produce retraso mental
leve. A la exploración física el paciente presenta un nevo pigmentado de forma
oval en la región lumbar, así como angiofibromas en la frente, los cuales la madre
dice que se han presentado desde el nacimiento. En el expediente se menciona
que en el pasado el paciente tuvo un rabdomioma cardiaco, el cual se encuentra
en regresión. ¿Cuál de las siguientes enfermedades incide con mayor riesgo de
presentar este paciente?
c} Epilepsia
96. ¿Cuál de las siguientes neoplasias malignas se asocia con policitemia como
síndrome paraneoplásico?
d} Carcinoma de células renales
97. ¿Cuál de las siguientes alteraciones electrolíticas se asocia con la aparición de
la onda J en el electrocardiograma?
b} Hipercalcemia
98. Paciente masculino de 19 años de edad que se presenta a consulta después
de haber notado la aparición de una úlcera no dolorosa en el pene posterior a una
relación sexual sin protección. ¿Cuál de los siguientes métodos es el mejor para
realizar el diagnóstico del agente etiológico del paciente?
b} Toma de exudado de la úlcera y observación bajo microscopio de campo
oscuro
99. Paciente masculino de 18 años de edad que conduce su automóvil en la
madrugada de regreso a casa. Durante el trayecto sufre colisión con otro
automóvil, cuyo conductor manejaba en estado de ebriedad. El paciente refiere
que sólo sufrió un impacto en la cabeza, sin pérdida del estado de alerta y que
tiene leve cefalea. Sin embargo, el personal paramédico insiste en llevarlo a un
hospital para evaluación adicional. Una vez en el hospital el paciente relata lo
sucedido cuando de repente se muestra somnoliento y después con pérdida del
estado de alerta.
¿Cuál es el diagnóstico más probable?
e} Hemorragia epidural
1OO. ¿Cuál es la prevalencia en población general de la artritis reumatoide?
e} 0.5 a 1%
101. ¿Cuál es el mecanismo de acción de las quinolonas?
d}Inhibición de la síntesis del DNA
r e} Ninguna de las anteriores
102. ¿A partir de qué momento se debe iniciar el tratamiento con FARME
(fármacos modificadores de la enfermedad) en pacientes con artritis
reumatoide?
a} Desde el momento del diagnóstico
103. Paciente de 59 años de edad que acude al servicio de urgencias por dolor
torácico asociado con disnea y náusea; el paciente tiene dificultad para describir
exactamente el tipo de dolor. Él estaba bajo control por hipertensión y no se le
conoce ningún otro antecedente de importancia. Los signos vitales al momento del
ingreso son: frecuencia cardiaca de 60 latidos/min, presión arterial 140/115 mm
Hg, frecuencia respiratoria de 15 respiraciones por minuto. A la auscultación se
puede detectar un soplo diastólico. La exploración física no se pudo completar
adecuadamente ya que el paciente desarrolla un edema de pulmón agudo, con
aumento en intensidad del soplo diastólico. Se realiza manejo agudo del paciente
y estabilización. Se solicita radiografía de tórax portátil y un ecocardiograma en la
cama del paciente que demuestran la presencia de una raíz aórtica dilatada,
asociada con insuficiencia aórtica moderada. Se realiza una tomografía
computarizada de urgencia la cual confirma la presencia de una disección aórtica
tipo A que involucra la válvula aórtica y ocluye el ostium de la arteria coronaria
derecha.
¿Cuál de los siguientes hallazgos a la exploración física debe crear la sospecha
de disección aórtica aguda?
a} Dolor torácico anterior
104. Paciente masculino de 64 años de edad se presenta en Urgencias por
presentar desde hace 9 semanas fiebre, confusión mental, anorexia y diaforesis
nocturna. Como antecedente de importancia refiere que se le realizó un recambio
valvular hace varios años; sin embargo, desconoce los detalles. A la exploración
física se notan petequias en conjuntiva y paladar, hemorragias retinianas, y
máculas hemorrágicas en palmas de las manos. A la auscultación cardiaca se
detecta un soplo de insuficiencia mitral.
104a. ¿Qué microorganismo es el que tiene mayor probabilidad de estar
involucrado en este caso?
b} S. viridans
104b. ¿Cuál se los siguientes NO es un criterio mayor de Duke?
e} Fiebre
104c. ¿Cuál es la válvula más afectada en pacientes adictos a drogas
intravenosas?
c} Tricúspide
104d. Si el resultado del hemocultivo es positivo para Streptococcus bovís, ¿qué
patología se descartaría de manera obligada?
c} Carcinoma de colon
104e. ¿Cuál es el antibiótico que se utiliza como profilaxis en procedimientos
dentales o de otra índole para evitar la endocarditis infecciosa?
a} Amoxicilina
105. La amantadina es un fármaco activo contra la influenza tipo A (pero no contra
la influenza tipo B), no obstante, recientemente ha surgido un alto grado de
resistencia a este fármaco. ¿Cuál de los siguientes enunciados se asocia mejor
con el mecanismo de acción de la amantadina?
b} Inhibición de la descapsidación
106. Paciente de 45 años de edad sin antecedentes de importancia, ingresa al
servicio de urgencias por deterioro neurológico, estado de choque y acidosis
láctica de brecha aniónica aumentada. A la exploración clínica neurológica se
encuentra estado de coma con flacidez y pérdida de los reflejos, pupilas isocóricas
y de lenta respuesta a la luz. La acidosis aniónica continuó empeorando y alcanzó
niveles de ácido láctico de 4.6 mmol/L y brecha osmolar de 11.35, con glucosa,
urea, creatinina y electrólitos normales. Al lavado gástrico se obtiene un material
rojizo. Se solicita un estudio de orina para buscar intencionadamente cristales de
oxalato de calcio. En caso de encontrar cristales de oxalato de calcio, ¿cuál es la
sustancia que más probablemente ingirió este paciente?
c} Etilenglicol
107. ¿Cuál de las siguientes condiciones es más probable que presente un título
de inhibición de hemaglutinina > 1:20?
a} Inmunidad contra el sarampión
108. Paciente masculino de 20 meses de edad se presenta al servicio de
urgencias con múltiples fracturas y hematomas, así como inflamación articular. La
exploración física revela múltiples petequias. En las radiografías se puede
observar engrosamiento de metáfisis y fragmentación alrededor de las epífisis. Los
estudios de laboratorio son característicos por un conteo de glóbulos rojos de 3
500 células/mm3, leucocitos de 2 500 células/mm3, volumen corpuscular medio de
70 fL, y concentración media de hemoglobina de 30%. ¿Cuál es el diagnóstico
más probable en este paciente?
b} Escorbuto
109. Paciente femenino de 23 años de edad la cual se presenta al consultorio por
presentar desde hace varias semanas secreción vaginal maloliente. La paciente
inició hace dos meses su vida sexual activa y está preocupada de que haya
adquirido una enfermedad de transmisión sexual. Al interrogatorio, menciona que
su pareja sexual no tiene ninguna sintomatología, y el método anticonceptivo que
utilizan son los anticonceptivos orales que una amiga le recomendó. A la
exploración física se observa una secreción blanquecina con olor a ""pescado"".
La medición del pH de la secreción es de 5.5 y la observación al microscopio
revela la presencia de cocobacilos cubiertos de células del epitelio vaginal. ¿Cuál
es el tratamiento más adecuado en esta paciente?
d} Metronidazol 500 mg vía oral dos veces al día durante siete días
11O. Paciente masculino de 34 años de edad que se presenta con tuberculosis
pulmonar aguda con presencia de bacilos ácido alcohol
resistentes en el esputo. ¿Cuál es el tratamiento más adecuado para este
paciente?
d} lsoniazida, rifampicina, pirazinamida, etambutol"
111. Un paciente de 60 años se presenta con diagnóstico reciente de cáncer de
mama en actual tratamiento con ciclofosfamida, metotrexato y 5-fluorouracilo. El
motivo de consulta es por sintomatología sugerente de cistitis ¿Cuál es el origen
de la mayoría de sus actuales síntomas?
d} Toxicidad por acroleína
112. Se presenta un masculino de 88 años con antecedente de hipertensión
arterial de 12 años de evolución en tratamiento irregular con diltiazem y diuréticos.
El principal motivo de consulta es disnea de inicio subagudo. En los cinco meses
previos ha sido hospitalizado varias veces. A la exploración física presenta PA de
230/11O mm Hg, FC 120 lpm, FR 28 rpm, temperatura de 36.5 ºC y saturación de
89%. La presión venosa yugular es de 14 cm H20, con pulsos hiperdinámicos. La
auscultación pulmonar revela estertores bilaterales. La auscultación cardiaca
indica presencia de tercer y cuarto ruidos cardiacos. Las extremidades muestran
un edema con Godete hasta los tobillos. El electrocardiograma tiene datos en
relación con hipertrofia ventricular izquierda y la radiografía de tórax demuestra un
corazón de tamaño normal con cambios moderados intersticiales consistentes con
edema pulmonar. Se realiza un ecocardiograma que revela hipertrofia ventricular
izquierda con conservación de la función sistólica ventricular izquierda. ¿Cuál es el
diagnóstico más probable en este paciente?
b} Insuficiencia cardiaca diastólica
113. Paciente femenina de 42 años refiere que inicia con fiebre y somnolencia
hace 2 semanas, pero que no presenta molestias de ningún tipo. Al interrogatorio
niega molestias faríngeas, urinarias, lesiones en piel o algo que haga sospechar
un origen infeccioso. A la exploración física, llama la atención que presenta
equimosis en piernas. Los resultados de la biometría hemática son:
• Hemoglobina 9.2 g/dL
• Hematocrito 28%
• Leucocitos 7 000 por microlitro
• Plaquetas 56 por microlitro
• Reticulocitos presentes
Usted decide realizar un Coombs, el cual se reporta negativo.
113a. ¿Cuál sería su impresión diagnóstica?
c} Púrpura trombocitopénica trombótica
113b. ¿Cuál es la fisiopatología de la enfermedad?
a} AutoanticuerposlgG que inhibenla actividad de ADAMTS 13, por lo que no se
separan los multímeros de vWF
113c. ¿Cuál es la clínica típica de la enfermedad?
d} Hay una pentada clásica: trombocitopenia, anemia hemolítica microangiopática,
alteraciones del estado mental, insuficiencia renal y fiebre
113d. Usted decide realizar un frotis para tener mayor certeza de su diagnóstico.
¿Qué esperaría ver en el frotis?
a} Esquistocitos
113e. ¿Cuál es el tratamiento ideal para esta paciente?
c} Plasmaféresis urgente y glucocorticoides
114. Paciente masculino de 39 años de edad que presenta pápulas pruríticas en el
cuerpo del pene y en el borde de la boca desde hace aproximadamente 20 días.
Al interrogatorio el paciente refiere que a pesar de que tiene varias parejas
heterosexuales, siempre utiliza condón y no realiza sexo oral. No existen
antecedentes personales o familiares de importancia. A la exploración física se
encuentran pápulas en el pene, en la mucosa oral, así como en las muñecas y
región anterior de la pierna. En la boca las lesiones son parecidas a estrías tipo
""encaje"" Utilizando únicamente la información previamente descrita, responda
las siguientes preguntas:
114a. ¿Cuál de las siguientes opciones diagnósticas es la más adecuada en este
paciente?
c} Biopsia + panel viral de hepatitis
114b. ¿Cuál de las siguientes opciones representa el agente etiológico más
frecuentemente asociado con esta condición dermatológica?
o a} Virus de la hepatitis e
114c. ¿Cuál de las siguientes opciones es el tratamiento más adecuado para la
condición dermatológica paciente?
a} Crema de dipropionato de betametasona
115. ¿Cuál de los siguientes fármacos interfiere con el funcionamiento de los
microtúbulos?
e} Griseofulvina
116a. Los datos clínicos de esta paciente corroboran el diagnóstico:
c} Necrosis tubular aguda
116b. La aparición de cilindros hemáticos en el sedimento urinario en esta
paciente corroboran:
d} Daño glomerular severo
117. ¿Cuál de las siguientes aseveraciones respecto del control respiratorio es
correcta?
c} Los quimiorreceptores periféricos estimulanla respiración cuando la presión
parcial de oxígeno cae por debajo de 60 mm Hg
118. Paciente femenino de 21 años, estudiante de preparatoria, no fumadora,
sana, con historia de un año de tos que ocurre esporádicamente o después de
exponerse a polvo o hacer ejercicio, asociada a opresión torácica y que se
autolimita en unas horas. Los síntomas son peores por la mañana y los presenta
menos de una vez a la semana durante el día y no más de dos veces al mes por la
noche. Acudió con un médico, quien le recetó omeprazol 20 mg cada 24 h, sin
mejoría. Llega a consulta con unas pruebas de función respiratorias normales y
una radiografía de tórax normal."
118a. ¿Qué prueba sería de utilidad para guiar el diagnóstico?
b} Test de metacolina
118b. ¿En qué clasificación de asma está la paciente?
a}Intermitente
118c. ¿Cuál de los siguientes es el responsable de mayor relevancia en la
fisiopatología del asma?
b} Leucotrieno C4
118d. ¿Cuál es el tratamiento indicado para la paciente?
c} Beta agonista de acción corta de rescate
118e. ¿Cuál de los siguientes tratamientos tiene una muy buena respuesta en
asma inducida por ejercicio?
a} Cromolín
119. ¿En cuál de las siguientes situaciones se establece con certeza el
diagnóstico de diabetes mellitus?
e} Concentraciones plasmáticas sucesivas de glucosa en ayuno de 147, 165 y 152
mg/dl en una mujer asintomática"
120. ¿A qué se refiere el ""efecto del alba""?
b} Hiperglucemia matutina causada por efectividad reducida de la insulina en ese
momento
121. La principal e inicial medida terapéutica del estado hiperglucémico
hiperosmolar es:
b} La rehidratación
122. Un paciente de 10 años de edad inhala un cacahuate, el cual se encuentra
obstruyendo uno de sus bronquios principales. Se obtiene una radiografía la cual
revela desviación del mediastino hacia el lado de la obstrucción. ¿Cuál de las
siguientes opciones es la más adecuada para describir las alteraciones
pulmonares que sufre este paciente?
a} Atelectasias de absorción
123. Hombre de 38 años acude por una dermatosis bilateral y simétrica
diseminada a ambas plantas de los pies, a nivel de los primeros metatarsianos de
cada lado. Se caracteriza por dos úlceras, la del pie derecho de 3 cm de diámetro
de forma circular, bien delimitada con bordes queratósicos y en el lecho de la
herida se observa necrosis en 50% y el porcentaje restante corresponde a fibrina,
con eritema perilesional de 2 cm aproximadamente. La úlcera del pie izquierdo
está cubierta por una callosidad, la cual se desbrida y se encuentra úlcera de 2 cm
de diámetro de forma circular, bien delimitada con 70% de granulación y 30% de
fibrina. El paciente tiene diagnóstico de diabetes tipo 2 de larga evolución
controlada, es deportista, refiere que no se dio cuenta cuándo inició con las
lesiones. El diagnóstico es pie diabético. La conducta a seguir es:
e} Todas las respuestas anteriores son correctas
124. Hombre de 38 años de edad. Acude a revisión oftalmológica general. Se
refiere asintomático. Sin antecedentes personales patológicos ni oftalmológicos. A
la exploración oftalmológica muestra agudeza visual de 20/20 en ambos ojos. El
globo ocular sin alteraciones. A la exploración de los párpados, se ubica una
lesión en el borde palpebral inferior derecho ligeramente elevada y pigmentada.
Se hace el diagnóstico de nevo pigmentado. ¿En qué estructura anatómica del
párpado se encuentra localizada la lesión?"
e} Lámina anterior delpárpado
125. A la exploración del fondo de ojo de una paciente de 16 años se observa
disminución focal y generalizada de las arteriolas. Al continuar con la exploración
se encuentran hemorragias en la retina izquierda, así como esclerosis. Al
interrogatorio la paciente niega cualquier alteración visual. Debido a estos
hallazgos, se decide tomar la presión arterial, la cual resulta en 190/103 mm Hg.
¿Cuál de los siguientes hallazgos es más probable que también se presente en
este caso?
a} Cefalea
126. Paciente masculino de 65 años de edad con factores de riesgo
cardiovascular se presenta a la sala de urgencias con hemiplejía derecha, así
como parálisis facial derecha con preservación de los movimientos de la mitad
derecha de la frente. El paciente también muestra dificultad para la comunicación
oral. El paciente se encuentra alerta con frecuencia cardiaca de 88 latidos por
minuto y presión arterial de 175/115 mm Hg, sin dificultad ventilatoria y con una
cifra de glucosa capilar de 190 mg/dL. Los familiares refieren que el cuadro inició
hace aproximadamente 1 hora. Una tomografía computarizada cerebral resulta
normal. ¿Cuál de los siguientes medicamentos tiene lugar en el tratamiento de
este paciente?
c} Aspirina
127. Si en un paciente con hipertiroidismo clínico y bioquímico el gammagrama
tiroideo con 1311 muestra captación del radiofármaco disminuida o ausente, el
diagnóstico es:
b} Tiroiditis subaguda

1. Paciente con una tumoración de 3 mm que le produce dolor intenso. El tumor se


encuentra por debajo de la uña del dedo índice de la mano izquierda. Se someterá a un
procedimiento quirúrgico para extirpar esta tumoración. ¿Cuál de los siguientes nervios
debe bloquearse para generar anestesia para la extirpación de este tumor?
d} Mediano
2. Paciente masculino de 77 años de edad que es llevado al servicio de urgencias por sus
hijos debido a que se encontraba con alteración del estado de alerta y diciendo que
estaba en la casa del presidente de la nación. A la exploración física se presenta con una
presión arterial de 80/60 mm Hg, y frecuencia cardiaca de 120 latidos/minuto, así como
con una temperatura de 39 ºC. Los hemocultivos son positivos para Escherichia coli. ¿Cuál
es el origen más frecuente de la bacteriemia en este paciente?
c} Vías urinarias

3. Paciente masculino de 18 años de edad que presenta episodios recurrentes en los que
aparece exantema vesicular en zonas expuestas a la luz. Durante los estudios de
laboratorio se descubre elevación de las porfirinas plasmáticas. ¿Cuálde las siguientes
enzimas es la que se encuentra con mayor probabilidad afectada en este paciente?
a} Uroporfirinógeno descarboxilasa

4. ¿Cuál de los siguientes enunciados es correcto respecto de la enfermedad celiaca?


a} En caso de alta sospecha de enfermedad celiaca, una biopsia de intestino delgado, los
marcadores serológicos como los anticuerpos antigliadina,antiendomisio y
antitransglutaminasa tisular pueden confirmar el diagnóstico

5. ¿Cuál de los siguientes nervios se puede lesionar en caso de una fractura del cuello
quirúrgico del húmero?
a} Nervio axilar

6. ¿Cuál de las siguientes funciones es la que corresponde a las ondas lentas en el tracto
gastrointestinal?
d} Despolarización de las células de músculo liso

7. Posterior a una infección de vías aéreas superiores, un paciente de sexo masculino de


seis años de edad desarrolla lesiones tipo púrpura en los glúteos y en las superficies
extensoras de las extremidades superiores e inferiores. Simultáneamente con la aparición
de estas lesiones el paciente inicia con dolor abdominal,vómito y artralgias. En una prueba
cualitativa de orina se puede detectar hematuria. ¿Cuál de las siguientes características se
encontraría en una biopsia renal en este paciente?
e} Depósito de lgA en las regiones mesangiales

8. ¿Cuál de las siguientes lesiones se asocia con más frecuencia a necrosis avascular?
a} Fractura de escafoides

9. Paciente masculino de 22 años de edad, el cuala pesar de apego al tratamiento, no ha


logrado controlar su cuadro asmático. El médico decide agregar zileutón al esquema.
¿Cuálde los siguientes estudios debe realizarse antes de iniciar tratamiento con este
medicamento?
d} Pruebas de función hepática
1O. ¿Cuál de los siguientes enunciados respecto a la pericarditis urémica es correcto?
a} Actualmente existen procedimientos quirúrgicos indicados en caso de derrame
pericárdico que ocasiona compromiso hemodinámico

11. Paciente masculino de 45 años de edad que se encuentra en este momento en fase de
rehabilitación por alcoholismo, el cual inició desde los 19 años de edad. En esta ocasión el
paciente acude a discutir los resultados de una biopsia hepática que se solicitó
previamente. A la exploración física presenta esplenomegalia y distensión abdominal. El
reporte histopatológico de la biopsia indica presencia de fibrosis difusa y regeneración
nodular. ¿Cuál de las siguientes complicaciones puede presentar este paciente a pesar de
que detenga el consumo de alcohol?
e} Carcinoma hepatocelular

12. Paciente masculino de 55 años de edad que a la exploración neurológica presenta


cierre palpebral a la estimulación de la córnea derecha, pero ausencia de cierre palpebral
consensual en el ojo izquierdo. ¿Cuál de los siguientes hallazgos será posible encontrar
también en este paciente?
c} Hiperacusia del oído izquierdo

13. Paciente masculino de 11 años de edad es sometido a una craneotomía para


extracción de craneofaringioma. En la cirugía recibió halotano; al concluir ésta fue
extubado sin ninguna eventualidad. Durante el periodo de recuperación, la enfermera
colocó al paciente una vía intravenosa e infusión de solución de dextrosa a 5% en un
tercio de solución salina normal a una velocidad de 125 mUh. Posteriormente, la
enfermera llama al médico tratante debido a que el paciente presenta disminución del
estado de alerta. Los resultados de laboratorio más reciente muestran la concentración
sérica de sodio de 157 mEq/L,osmolaridad de 320 mOsm/L, PaC02 x de 28 mm Hg,
bicarbonato de 22 mEq/L y pH de 7.3, así como incremento en el balance positivo de
líquidos. ¿Cuál de los siguientes diagnósticos es el más probable en este caso?
b} Lesión a la hipófisis posterior
14. Paciente del sexo femenino de 28 años de edad que acude a consulta por infertilidad.
A pesar que ha intentado relaciones sexuales regulares sin protección anticonceptiva
durante los últimos dos años, no ha logrado embarazarse. Al interrogatorio la paciente
menciona que tiene menstruaciones muy irregulares, las cuales ocurren cada dos a tres
meses, e inclusive,cada cinco meses. A la exploración física es evidente que presenta
hirsutismo y obesidad. Se decide iniciar tratamiento farmacológico, lo cual incrementa sus
niveles de progesterona en sangre; la realización de una biopsia de endometrio revela
cambios secretores posteriores al inicio del medicamento. ¿Cuálde los siguientes
medicamentos fue el que se le administró con mayor probabilidad a esta paciente?
d} Antagonistas de estrógenos

15. Paciente femenino de cuatro años que acude a consulta por odinofagia de tres días de
evolución. El día de hoy inició de forma súbita con fiebre,afonía, dificultad respiratoria y
estridor. En el examen físico se observa paciente taquipneica, con intensa sialorrea,tiraje
intercostal y subcostal, retracción xifoidea, en la auscultación se percibe estridor referido,
sin crepitantes ni sibilancias. ¿Cuál es su diagnóstico?
e} Epiglotitis aguda

16. Mujer de 24 años de edad que presenta tumoración en fosa iliaca derecha dolorosa.
Desde hace seis meses presenta palpitaciones, caída de cabello, diarrea e hiperreflexia. Se
diagnostica tumor de ovario benigno. Se observan tejidos de las tres capas germinativas.
¿Qué es característico de los tumores germinales de ovario?
a} El gonadoblastoma se relaciona a síndrome de Swyer

17. ¿Cuál de los siguientes ha demostrado ser uno de los predictores más importantes de
hiperuricemia en diversos estudios epidemiológicos?
d} Indice de masa corporal

18. Una paciente de 28 años de edad, sin antecedentes relevantes, cursa 33 y 2/7
semanas de gestación (sdg} por fecha de última menstruación (FUM), primigesta, hasta
ahora embarazo normoevolutivo. Acude a consulta por presentar cefalea intensa de 30
a45 minutos de evolución acompañada de epigastralgia,se toman signos vitales en reposo,
en ambos brazos, encontrando hipertensión arterial sistémica con tensión arterial media
(TAM) de 130 mmHg, FC 90 lpm, FR 20 rpm, temp. 36.6ºC, PA 170/11O mm Hg, peso 65
kg, talla 1.68 m.
18a. Con los datos obtenidos el diagnóstico de sospecha clínica es:
c} Preeclampsia severa

18b. Para protocolizar a la paciente usted solicita:


a} Biometría hemática, química sanguínea, PFH,TTPa,INR y examen general de orina
18c. Una vez monitoreada la paciente,establecido un acceso vascular periférico y en
reposo, ¿Qué tratamiento antihipertensivo inicia?
c} Hidralazina

18d. Con el control de la crisis hipertensiva y remisión sintomática, usted decide:


b} Hospitalización e iniciar maduradores pulmonares

18e.Algunas pacientes requieren ingresar a la Unidad de Cuidados Intensivos, ¿cuál de los


siguientes no es una indicación para manejo en hospitalización?
d} Puerperio fisiológico

19. Se trata de un paciente masculino de 34 años de edad, pescador, que fue rescatado de
un naufragio hace 5 semanas; acude a consulta por presentar insomnio, pesadillas
recurrentes y repetitivas con sensación de ahogamiento, nerviosismo y crisis de pánico,
refiere que los síntomas se acentúan cuando va a trabajar y esto limita su actividad
laboral. Niega ingesta de alcohol, tabaco o drogas. La exploración física es normal. FC 75
lpm, FR 18 rpm, temp. 36.5ºC, PA 125/80 mm Hg, peso 75 kg, talla 1.70 m.
19a. Con los datos proporcionados por el paciente, usted considera que está cursando
con:
b} Trastorno de estrés postraumático

19b. Para realizar el abordaje,usted debe descartar los diagnósticos dieferenciales, ¿cuál
de los siguientes no es un diagnóstico diferencial del trastorno de estrés postraumático?
c} Trastorno de conversión

19c. Establecido el diagnóstico usted decide que la psicoterapia debe ser:


c} Terapia cognitiva conductual

19d. El tratamiento farmacológico indicado en este problema es:


a} lnhibidores selectivos de la recaptura de serotonina

19e.Los siguientes factores son de buen pronóstico, excepto:


d} Evento traumático causado por humanos (violación,guerra...)

20. Paciente obeso de 45 años acude a urgencias por presentar dolor en la rodilla derecha
de inicio súbito que lo despertó esta mañana y ha ido aumentando rápidamente de
intensidad hasta dificultarle la deambulación. A la exploración física se observa rodilla
derecha con aumento de volumen, aumento local de la temperatura, hiperemia,
induración y dolor extremo a la movilización pasiva y activa. El paciente refiere que nunca
había experimentado un dolor similar,es sexualmente activo con dos parejas
heterosexuales, bebe en exceso cada fin de semana y fuma una cajetilla de cigarrillos
diario.
20a. ¿Cuál es el estudio paraclínico más importante para establecer el diagnóstico?
d} Artrocentesis
20b. ¿Cuál de los siguientes hallazgos confirma el diagnóstico de artritis gotosa?
b} La presencia de cristales con forma de aguja que se tornan azules con la luz polarizada

21.Se trata de un paciente masculino de 27 años de edad, sin antecedentes relevantes,


vive en una región tropical y es obrero en una construcción; refiere que hace 2 a 3
semanas presenta lesiones eritematosas, maceración, muy pruriginosas a nivel interdigital
del pie izquierdo, refiere uso de calzado cerrado; a la exploración física se encuentra lesión
de eritema, con bordes bien delimitados, datos de queratinización por el rascado, así
como descamación de predominio interdigital. FC 75 lpm, FR 16 rpm, temp. 36.5 ºC, PA
120/70 mm Hg, peso 62 kg, talla 1.68 m.
21a. Con los datos clínicos y la exploración física,¿qué diagnóstico integra?
a} Dermatofitosis
21b. Según las características de este paciente ¿cuál es el agente causal más común?
d} Tricophyton rubrum

21c. ¿Cuál es el método diagnóstico de elección?


b} Microscopia con hidróxico de potasio en escamas

21d. Una vez determinado el diagnóstico ¿qué tratamiento inicia?


c} Ketoconazol

21e.Una vez establecido el tratamiento ¿qué medida preventiva no da al paciente?


d} Inmersión de pies en agua de Alibour

22. Mujer de 60 años, consulta por aumento de volumen en cuello. Al interrogatorio


refiere fatiga y estreñimiento hace varias semanas, con aumento leve de peso. TA: 120/90,
FC: 65 x', Temp: 36.5 ºC. A la exploración del cuello se detecta bocio irregular e indoloro,
edema pretibial leve sin fóvea, y frialdad de extremidades. Sus medicamentos incluyen
ramipril y metformina. Perfil tiroideo: TSH elevada, T4 libre baja,anticuerpos
antiperoxidasa tifoidea positivos.
22a. Con la clínica y los laboratorios mencionados, usted hace el diagnóstico de:
O a} Enfennedad de Graves
 c} Hipotiroidismo autoinmune

22b. ¿Cuál es la base fisiopatológica de la enfermedad de este paciente?


a} Infiltración de linfocitos T CD4 y CDS activados y linfocitos B en la tiroides

22c. ¿Cuál de los siguientes enunciados es correcto?


d} La T3 es la hormona tiroidea más potente

22d. ¿Cuál es el tratamiento indicado en este paciente?


a} Levotiroxina
22e.¿Cuál de los siguientes enunciados es correcto respecto al tratamiento restitutivo con
levotiroxina en pacientes hipotiroideos?:
b} El objetivo del tratamiento es lograr una concentración normal de TSH

23. Paciente masculino de 75 años con antecedente de hipertensión arterial sistémica y


diabetes mellitus tipo 2 quien acude alservicio de urgencias por presentar hemorragia
nasal de aproximadamente 150 ml 3h previas a su ingreso hospitalario, sin causa
aparente; paciente niega trauma previo. A su ingreso presencia de agitación, náusea y
vomito con restos hemáticos, Glasgow de 15 puntos, taquicárdico, a nivel pulmonar sin
presencia de agregados, abdomen y extremidades sin datos patológicos. FC 112 lpm, FR
19 rpm, temp. 37 ºC, PA 140/80 mm Hg, peso 87 kg, talla 1.60 m.
23a. El diagnóstico que se integra es:
a} Epistaxis anterior
23b. Según los datos clínicos y la exploración física el paciente se encuentra con un
episodio de epistaxis anterior,en este paciente el factor de riesgo más importante para la
hemorragia es:
c} Hipertensión arterial

23c. Una vez efectuado el diagnóstico el procedimiento terapéutico que se aplica es:
d} Taponamiento nasal anterior y posterior

23d. Una vez que realiza el tratamiento la hemorragia cede adecuadamente por lo cual
usted recomienda al paciente:
b} Retiro del taponamiento en 5 días

23e.¿Qué medidas higiénico-dietéticas debe prevenir el paciente?


d} Manipular el tapón nasal

24. Paciente femenino de siete años de edad acude a consulta por diplopía que ha
empeorado en los últimos días. A la exploración oftalmológica demuestra parálisis del VI
nervio craneal. ¿Cuál de los siguientes padecimientos es el que con mayor probabilidad
tiene la paciente?
a} Glioma del puente

25. Lactante de tres meses de vida, con antecedente de retraso en la expulsión de


meconio, que ha presentado cuadros de estreñimiento alternando con episodios de
diarrea. Presenta datos de deshidratación y retraso en el crecimiento para su edad.
Indique cuál de los siguientes estudios proporciona el diagnóstico definitivo de la
enfermedad del paciente.
c} Biopsia de recto

26. Paciente masculino de 48 años de edad, sedentario, con circunferencia abdominal de


108 cm, hipertensión arterial e hipertrigliceridemia. Acude a consulta de control. Se
reporta asintomático. TA: 130/80 mm Hg, hiperpigmentación en cuello y axilas.
Laboratorios: glucosa en ayuno 154 mg/dl, HbA1c: 7.0%, colesterol LDL: 115 mg/dl,
HDL:48 mg/dl,triglicéridos: 300 mg/dl.
26a. ¿Cuál es el diagnóstico en este paciente?
c} Síndrome metabólico cumpliendo 4 criterios

26b. De acuerdo con el diagnóstico de diabetes mellitus tipo 2 en el paciente, señale el


enunciado correcto:
b} Se debe repetir la prueba para el diagnóstico definitivo

26c. Señale el enunciado correcto respecto a las complicaciones en la DM2:


a} La nefropatía es una complicación microangiopática

26d. En relación con el tratamiento en este paciente,señale el enunciado incorrecto:


a} La presión arterial se encuentra dentro de la meta para DM

27. Paciente femenino recién nacida producto de un embarazo prematuro de 32 semanas


que exhibe ictericia, exantema, rinitis persistente, anemia, linfadenopatía generalizada y
anormalidades óseas visibles en las radiografías. ¿Cuál de los siguientes factores es el más
importante durante el interrogatorio de la madre?"
b} Antecedentes sexuales

28. Lactante de siete meses de edad que se presenta con diarrea y vómito. La madre
menciona que hace cinco días inició consumo de leche de fórmula;previamente sólo había
recibido leche materna. De acuerdo al diagnóstico del paciente, ¿qué componentes de la
leche son los responsables del cuadro clínico?
a} Beta lactoglobulina y caseína

29. Niño de tres años de edad con probable reflujo vesicoureteral, se solicita cistografía
ureteral miccional y se observa reflujo bilateral hacia uréteres dilatados y tortuosos con
dilatación de pelvis y cálices renales. De acuerdo con la clasificación de reflujo
vesicoureteral del paciente,¿qué abordaje terapéutico es más adecuado?
d} Cirugía antirreflujo

30. Paciente masculino de 47 años de edad con antecedente de depresión en tratamiento


farmacológico no especificado, abandonado hace 6 meses. Acude traido por paramédicos
al encontrarse en su domicilio inconciente, se identifica una caja de paracetamol vacía. A
su ingreso con Glasgow de 13 puntos,ictérico, afebril,diaforético, taquicárdico, murmullo
vesicular conservado. Abdomen blando depresible no doloroso a la palpacion superficial
no profunda, presencia de hepatomegalia. Extremidades sin edema, pulsos distales
presentes, llenado capilar 2 segundos. FC 125 lpm, FR 25 rpm, temp 36.5 ºC, PA"
90/60 mm Hg, peso 87 kg, talla 1.62 m.
30a. ¿Cuál es su impresión diagnóstica?
b} Intoxicación por paracetamol
30b. ¿Cuál es el metabolito tóxico del paracetamol?
b} Acetil parabenzoquinoneimina

30c. ¿Cuál es la dosis tóxica de paracetamol en adultos?


c} 7g en 24h

30d. ¿Cuál es la etapa de intoxicación en la que se encuentra nuestro paciente?


b} Estadio II

30e.¿Cuál es el antídoto y su dosis?


a} N acetilcisteína 140mg/kg inicial 70mg/kg cada 4h, 17 tomas
31. Paciente femenina de 18 años de edad. Sin antecedentes patológicos. Estudiante.
Consulta por cefalea bilateral opresiva de 36 h de evolución, intermitente, la mayor parte
del día, y dificultad para conciliar el sueño, sin embargo el dolor no la despierta una vez
dormida; lo anterior se acompaña de náusea sin llegar al vómito. TA: 11ono mm Hg, FC: 75
x', FR: 14 x', Temp: 36.5 ºC. Exploración neurológica sin alteraciones. No ha consumido
medicamentos y se presenta sin"
laboratorios.
31a. Por la semiología del dolor,usted hace el diagnóstico presuntivo de:
b} Cefalea tensional

31c. ¿Cuál de los siguientes enunciados es cierto respecto a la cefalea tensional?:


b} Es más frecuente en mujeres

31d. ¿Cuál es el tratamiento para disminuir la sintomatología en esta paciente?


c} AINE

31e.En relación con la fisiopatología de la cefalea tensional, señale la respuesta correcta:


a} No se conoce en detalle los aspectos fisiopatológicos

32. Masculino de 20 años es enviado al servicio de urgencias acompañado por sus


familiares, portador de diabetes mellitus desde los 13 años contratamiento irregular con
dieta e insulina de acción intermedia. Refieren los familiares que en últimas fechas ha
presentado astenia, adinamia, anorexia, náusea y vómito de contenido gastrobiliar;
poliuria,dolor abdominal difuso de tipo punzante localizado en mesogastrio. En los dos
últimos días ha estado ingiriendo bebidas alcohólicas y suspendió las inyecciones de
insulina. Se inicia el ingreso del paciente en estado estuporoso evidente.
32a. Los hallazgos de laboratorio en este paciente son:
b} Hiponatriemia

32b. El estado metabólico causal de la sintomatología de este paciente es:


b} Cetoacidosis diabética
32c. Es el fenómeno fisiopatológico que suele presentarse en estos casos:
b} Exceso de glucagón en plasma

33. Masculino de 30 años que consulta por dolor en el centro de la cara,el cual se
acompaña de obstrucción nasal, rinorrea mucopurulenta, descarga retronasal, hiposmia,
halitosis, tos y ataque al estado general, de 7 días de evolución. TA: 140/85, FC: 85 lpm,
FR: 18x ', Temp: 38ºC. A la exploración física se observa edema e hiperemia de la mucosa
nasal, rinorrea purulenta y cornetes inferiores hipertróficos, así como descarga retronasal
amarillenta.
33a. ¿Cuálsería su sospecha diagnóstica?
c} Sinusitis

33b. ¿Cómo se clasifica esta enfermedad?


c} Aguda, subaguda y crónica

33c. Causamás común de este padecimiento:


c} Streptococcus pneumoniae

33d. Estándar de oro para el diagnóstico:


a} TC

33e.El tratamiento más adecuado para el paciente en cuestión es:


d} Amoxicilina

34. Paciente femenino de 22 años, con antecedente de 2 embarazos y 2 partos, trastorno


bipolar, con un hijo de seis años que moja la cama y un recién nacido de 15 días es traída a
urgencias tras ingerir múltiples pastillas en un intento de suicidio. Se encuentra letárgica,
con frecuencia cardiaca de 120 latidos por minuto, presión arterial 90/60 mm Hg y
empieza a convulsionar súbitamente. El monitor cardiaco muestra ensanchamiento
progresivo del QRS y prolongación del intervalo QT. Si los síntomas del paciente se deben
a intoxicación por un medicamento, ¿cuál es el fármaco que la paciente ingirió?
a} lmipramina

35. Angélica,de 24 años de edad, refiere inicio de vida sexual activa hace 5 años y utiliza
anticonceptivo de barrera,su ciclo menstrual es irregular y no recuerda su fecha de última
menstruación, niega otros antecedentes; acude por dolor abdominal, que inició hace 8
horas, localizado en la fosa iliaca derecha, tipo cólico, intensidad 8/10 que baja hasta
3/1O, se automedicó butilhiosina con mejoría parcial,se agregó anorexia,
adinamia,escalofrío, náusea y vómito, con intolerancia a la vía oral, niega otra
sintomatología. A la exploración presenta disminución de la peristalsis, dolor a la
palpación de fosa iliaca derecha,con signo de von Blumberg positivo, resto negativos. FC
95 lpm, FR 22 rpm,temp. 37.5 ºC, PA 130/65 mm Hg, peso 55 kg, talla 1.60 m.
35a. Usted considera que se trata de una apendicitis aguda, ¿cuálde los siguientes es el
diagnóstico diferenciala descartar?
b} Embarazo ectópico

35b. ¿Qué estudio de laboratorio solicita para descartar el embarazo ectópico?


c} Niveles de HGC-P

35c. ¿Qué estudio de imagen es de primera elección para confirmar apendicitis aguda?
c} Tomografía simple

35d. Se planea egresar al paciente a piso de Cirugía General después de la


apendicectomía. ¿Cuál es el tratamiento antimicrobiano de primera elección para los
pacientes con apendicitis aguda perforada?
a} Piperacilina-tazobactam

35e.Los laboratorios reportaron leucocitosis de 18,300 / mm3 con desviación a la


izquierda, con los datos clínicos y paraclínicos obtenidos ¿cuántos puntos de la
clasificación de Alvarado tiene la paciente?
c} 9 puntos

36. Una mujer de 38 años de edad con antecedente de control prenatal inadecuado trae a
consulta una niña de 3900 g de peso. En la exploración física se identifica hipotonía, cara
plana,hendiduras palpebrales sesgadas y ascendentes, piel redundante en la
nuca,clinodactilia y braquidactilia del quinto dedo, y pliegue transverso palmar.
36a. ¿Cuál es el diagnóstico más probable de esta paciente?
c} Síndrome de Down

36b. ¿Cuál es la incidencia del trastorno que presenta la paciente?


a} 1/600 a 800

36c. Los pacientes con este trastorno tienen más riesgo de:
d} Malformaciones cardiacas

36e.¿Cuál es la cardiopatía más común en pacientes con este diagnóstico?


b} Defectos auriculoventriculares

37. Renata es una paciente de 27 años de edad que acude al servicio de Urgencias por
presentar pérdida súbita de la agudeza visual de forma completa con dolor retroocular,
que se acompaña de debilidad de las extremidades inferiores, misma que limita la
deambulación. Al interrogatorio dirigido niega otros antecedentes, alergias, infecciones y
vacunación reciente. La exploración visual revela alteraciones en la campimetría por
confrontación, el fondo de ojo con borramiento de los bordes y eritema de la papila; la
exploración neurológica es normal. FC 75 lpm, FR 18 rpm,temp. 36.5 ºC, PA 110/65 mm
Hg, peso 58 kg, talla 1.67 m.
37a. Con base en el cuadro clínico, ¿cuál es su diagnóstico presuntivo?
d} Eclerosis múltiple
37b. Según las manifestaciones clínicas pueden seguir uno de los cuatro patrones
característicos, ¿cuál es el más común?
a} Brote-remisión

37c. Para establecer el diagnóstico de la paciente usted solicita:


c} Resonancia magnética de cráneo

37d. ¿Cuál de los siguientes es un diagnóstico diferencial de la esclerosis múltiple?


a} Vasculitis

37e.¿Cuál es el tratamiento de elección para brotes comunes?


d} Metilprednisolona

38. Paciente femenino de 67 años consulta por disminución en la agudeza visual. Tiene
antecedentes de DM2 y HTA desde hace 15 años, por lo que toma metformina 500mg
cada 12h y captopril 50 mg cada 12 h; señala que ha notado una pérdida mayor desde que
le indicaron insulina, la cual utiliza irregularmente.Al realizarle examen de fondo de ojo,
usted observa microaneurismas y algunos exudados de color amarillo.
38a. ¿Cuálsería su sospecha diagnóstica?
c} Retinopatía diabética

38c. Por los hallazgos en el examen de fondo de ojo, usted clasificaría la retinopatía
diabéticade esta paciente en:
c} No proliferativa moderada

38d. ¿Cuál de las siguientes condiciones sería indicativa de tratamiento con


fotocoagulación en la paciente?
a} Edema macular

38e.¿Cuáles son las principales causas de disminución en la agudeza visual dela paciente?
d} Edema macular,hemorragia vítrea y desprendimiento de retina

39. De las siguientes vacunas. ¿Cuál es de virus vivos atenuados?


d} Sarampión/parotiditis/rubéola

40. Durante el estrés agudo se pierde inicialmente la siguiente función suprarrenal:


a} Variabilidad circadiana y pulsatilidad de ACTH

41.Hormonas reguladoras del metabolismo de calcio y fósforo:


a} PTH,vitamina D,calcitonina
42. Un hombre de 68 años que se encuentra hospitalizado por sufrir un infarto anterior.
Como tratamiento se le realizó un cateterismo y se le colocó un stent. Llaman durante la
noche porque el paciente se queja de disnea. A la exploración física sus signos vitales
incluyen FC 52/min, FR 21/min, T 37.3 C y PA 108/69 mm Hg, con saturación de oxígeno
de 88%. A la auscultación se escucha S1 y S2 sin agregados. Los campos pulmonares se
auscultan con estertores crepitantes. Se le realiza un trazo de electrocardiograma, que se
muestra a continuación:

¿Cuál de las siguientes opciones representa la conducta más adecuada?


c} Colocación de marcapaso transcutáneo

43. Un hombre de 71 años acude a consulta por lumbalgia. Refiere que el dolor inició hace
cuatro meses, localizado en el área central de la espalda baja. En su historia clínica refiere
hipertensión tratada con hidroclorotiazida y osteoartritis. Al interrogatorio refiere sentirse
un poco más cansado de lo normal, y dolor intermitente de sus extremidades. Niega
incontinencia urinaria, parestesias, o cambios de la sensibilidad. A la exploración física se
observan nódulos de Heberdeen y de Bouchard. No hay dolor a la palpación de la columna
dorsal o el resto de la espalda. La sensibilidad perianal está intacta, así como el resto de la
exploración neurológica. En la radiografía se observa osteofitosis de la columna vertebral,
sin otras anormalidades aparentes. Se realizan estudios de laboratorio, que se muestran a
continuación:
¿Cuál de estos estudios diagnósticos estaría indicado en este paciente?
a} Electroforesis de proteínas séricas

44. Un hombre de 81 años acude a consulta por "ronchas". Él es residente de un asilo y se


ha reportado que estas lesiones han ocurrido recientemente en otros residentes. Los
antecedentes patológicos del paciente incluyen hipertensión, hiperlipidemia,enfermedad
coronaria y enfermedad de Alzheimer. El paciente no es capaz de responder el IPAS,
aunque la enfermera que lo acompaña niega fiebre, o síntomas de infección de las vías
respiratorias superiores. A la exploración física sus signos vitales incluyen FC 81/min, FR
16/min, T 37.2C y PA 130/76 mm Hg. Se observan en los brazos múltiples lesiones
papulares en superficies extensoras e interdigitales, con excoriaciones circundantes. ¿Cuál
sería el manejo de este paciente?
a} lvermectina dosis única a él y a todos los residentes y trabajadores del asilo

45. Paciente masculino de 67 años que se presenta al consultorio, su preocupación


principal es que un amigo acaba de morir de un infarto cardiaco y quiere hacer todo lo
que pueda para prevenirlo. Su padre murió de un infarto agudo de miocardio a los 74
años. Niega alguna enfermedad. Refiere tabaquismo de 24 paquetes/año. A la exploración
física sus signos vitales incluyen FC 72/min, FR 16/min,T 37.2 C y PA 126/78 mm Hg.
Refiere que nunca ha tenido una lectura de presión arterial arriba de 130/80 mm Hg en
sus mediciones previas. Su última medición de HDL fue de 50 mg/dl.A la exploración física,
se observa un paciente delgado, y no se detectan anormalidades. ¿Cuál sería la meta de
LDL para este paciente?
a} LDL < o igual 130 mg/dl

46. Paciente masculino de 23 años de edad que es llevado al hospital después de que en
una cantina se involucró en una riña en la que sufrió una herida por arma punzocortante
en la parte superolateral de la pared torácica izquierda a nivel de la tercera costilla. El
paciente presenta sangrado leve y ausencia de dificultad respiratoria. A la exploración
neurológica se revela que el borde medial de la escápula en el lado izquierdo se encuentra
desviado cuando se levanta el brazo; entre otras anormalidades el brazo no puede ser
abducido más allá de la posición horizontal. ¿Cuál es el nervio lesionado en este paciente?
d} Torácico largo

47. Paciente de 70 años con antecedente de enfermedad pulmonar obstructiva crónica


grave con administración de oxígeno en casa, se presenta a la sala de urgencias por
antecedente de tres días de disnea progresiva y tos productiva purulenta. Jamás ha sido
hospitalizado por una exacerbación de su enfermedad de base. A la inspección se tiene un
paciente anciano con dificultad respiratoria moderada a grave, que le impide pronunciar
oraciones completas con obnubilación. La exploración torácica revela campos pulmonares
con disminución de los ruidos respiratorios y sibilancias espiratorias. El médico interno
toma una gasometría la cual revela pH de 7.25, PaC02 de 70, Pa02 de 50. La radiografía de
tórax demuestra hiperinflación de los campos pulmonares sin infiltrados o neumotórax. Se
inicia tratamiento con oxígeno (mascarilla tipo Venturi con Fi02 de 0.50),
broncodilatadores, antibióticos y corticosteroides. Después de 50 minutos, el paciente se
presenta con mayor deterioro del estado de conciencia y la gasometría actual revela pH
de 7.15, PaC02 de 100 y Pa02 de 70. ¿Cuál es el mejor tratamiento en este paciente?
c} Intubar al paciente e iniciar ventilación mecánica

48. ¿Cuál de las siguientes estructuras forma el límite lateral y medial del triá ngulo
femoral?
d} Músculo aductor largo y sartorio

49. Paciente masculino de 29 años de edad con diagnóstico de infección por el virus de
inmunodeficiencia humana,se presenta al servicio de urgencias con una nodulación
indolora sobre la piel de la articulación del tobillo. Debido al elevado costo del tratamiento
antiviral, el paciente ha decidido suspender su medicación desde hace algunos meses
previos. A la exploración física el paciente se presenta afebrily sin ninguna otra lesión en el
resto del cuerpo. El conteo de linfocitos T CD4+ es de 20 células/mm3. El paciente solicita
que se le atienda rápido pues ha olvidado alimentar al gato que recientemente adquirió.
Se decide obtener una biopsia de la región la cual demuestra proliferación lobular de
vasos sanguíneos con crecimiento de las células endoteliales y un infiltrado crónico
inflamatorio. La tinción de Gram revela la presencia de bacilos gramnegativos. ¿Cuál de los
siguientes fármacos es la mejor opción en este paciente?
a} Macrólidos

50. Paciente de 65 años de edad que se presenta con disnea de inicio súbito y taquicardia.
El día anterior al inicio de la sintomatología el paciente regresó de Europa en viaje de 12
horas en avión. A la exploración física el paciente se presenta con ansiedad y dificultad
respiratoria; la presión arterial es de 145/84 mm Hg. Se solicita un electrocardiograma que
muestra taquicardia sinusal y desviación del eje hacia la derecha. La radiografía de tórax
no muestra anormalidades. La gasometría arterial presenta hipoxemia leve con alcalosis
respiratoria. Debido a la sospecha clínica se solicita un estudio de perfusión-ventilación
pulmonar que revela la presencia de una región no concordante en el lóbulo inferior del
pulmón derecho. ¿Cuál de los siguientes tratamientos NO es recomendado en este
paciente?
b} Activador delplasminógeno tisular intravenoso 100 mg cada 2 horas, seguido de dosis
de mantenimiento de heparina no fraccionada intravenosa

51. Paciente masculino de 35 años de edad que regresa de viaje de luna de miel en África.
A su regreso el paciente presenta cefalea y fiebre por lo que acude a servicio médico. El
paciente es diagnosticado con infección por Plasmodium falciparum. En este momento el
paciente presenta un hematocrito de 18%, hiperbilirrubinemia de 9 mg/dL y creatinina
sérica de 2.7 mg/dL. El paciente es ingresado con monitorización de la función renal y
administración de fenobarbital como profilaxis de crisis convulsivas. ¿Cuál de los
siguientes antibióticos es el más adecuado para este paciente?
e} Artesunato

52. En varias ocasiones la hipotermia grave no es diagnosticada durante la exploración


física inicial. La temperatura se puede reportar como el límite inferior del termómetro, lo
cual puede ser de tan sólo unos cuantos grados centígrados por debajo de lo normal.
¿Cuál de los siguientes es suficientemente adecuado para indicar el diagnóstico de
hipotermia?

d} Electrocardiograma

53. ¿Cuál es el principal efecto secundario de la administración de abacavir en pacientes


con infección del virus de la inmunodeficiencia humana?
b} Reacción de hipersensibilidad severa

54. ¿Cuál de los siguientes hallazgos se puede observar en un paciente con lesión del
nervio safeno?
b} Alteración de la sensibilidad enla porción medial de la pierna

55. Se presenta a consulta paciente femenino de 14 años por amenorrea primaria y


ausencia de desarrollo glandular mamario. La madre refiere que el único antecedente de
importancia es disminución de la audición que requirió uso de un dispositivo especial. A
pesar de que la paciente fue producto de un embarazo normal y parto eutócico, la madre
recuerda que los médicos le dijeron que su hija estaba un poco "hinchada" al nacer. A la
exploración física la paciente muestra signos vitales normales, mide 1.50 m y pesa 60 kg. A
la inspección, se encuentra implantación baja de orejas y cuello alado. La exploración
cardiovascular revela un soplo sistólico grado 11/IV,con pezones extremadamente
separados y glándulas mamarias Tanner l. La exploración pélvica revela ausencia de vello
púbico y genitales externos en fase prepuberal. Se puede observar mediante un espéculo
la presencia de cuello uterino (cervix). ¿Cuáles otros hallazgos son probables de identificar
en esta paciente?
a} Anomalías renales, linfedema de extremidades, anomalías cardiovasculares (en especial
coartación de aorta}

56. ¿Cuál de los siguientes medicamentos se utiliza en el tratamiento de la esclerosis


múltiple?
b} lnterferón beta

57. Paciente masculino de seis años de edad es llevado a consulta por sus padres debido a
un cuadro que inicia como un aparente resfriado con rinorrea, pero en este momento el
paciente presenta disnea intensa. Los padres informan que el paciente muestra notable
mejoría durante la mañana pero los síntomas empeoran conforme se acerca la noche. A la
exploración física se pueden auscultar estertores y sibilancias pero es imposible localizar
los ruidos cardiacos. La radiografía de tórax revela hiperinsuflación pulmonar,
bronquiectasias e inversión izquierda/derecha de varias estructuras del sistema
respiratorio. ¿Cuál de las siguientes complicaciones puede presentar este paciente?
e} Infertilidad

58. Durante la etapa fetal, ¿en cuál de las siguientes estructuras el porcentaje de
saturación de hemoglobina fetal es menor?
a} Ductus arteriosus

59. Paciente femenino de 28 años con diagnóstico de síndrome de Wolff-Parkinson-White


acude a urgencias por presentar palpitaciones de inicio súbito hace 45 minutos. A la
exploración física se encuentra angustiada y destaca ritmo irregular. El electrocardiograma
muestra fibrilación auricular con respuesta ventricular rápida con una frecuencia de 170
latidos por minuto.
¿Cuál es el tratamiento más adecuado para la paciente?
c} Procainamida

60. Paciente del género masculino de 35 años de edad con antecedente de infección por
el virus de la inmunodeficiencia humana (diagnosticado desde hace 3 años). El paciente no
desea la administración de antirretrovirales, pues cree que su enfermedad es un "invento
del gobierno y de las compañías farmacéuticas". Actualmente, en una revisión de rutina, el
paciente refiere una "erupción" que apareció hace 24 horas en la región lumbar y en la
región posterior de la pierna derecha. El paciente refiere las lesiones como dolorosas (tipo
"ardor"). Entre la información obtenida en el interrogatorio, el paciente mencionó que fue
ascendido en su trabajo recientemente, pero que esto lo ha estresado demasiado pues
ahora tiene mayores responsabilidades. A la inspección de la lesión se observa un
exantema formado por vesículas sobre una base eritematosa ("gotas de rocío sobre
pétalos de rosa roja").
Utilizando únicamente la información previamente descrita, responda las siguientes
preguntas:
60a. ¿Cuál de las siguientes opciones representa la etiología más probable del exantema
en este paciente?
d} Virus herpes humano 3

60b. ¿Cuál de las siguientes opciones representa la conducta inmediata más adecuada
para este paciente?
a} Administración de aciclovir oral por 7-10 días

60c. ¿Cuál es la categoría clínica de infección por el virus de la inmunodeficiencia humana


en que se encuentra este paciente?
b} Categoría B

60d. Después de solicitar estudios de laboratorio, resulta que el paciente presenta una
carga viral de 15,000 copias/mL y un conteo de células T CD4+ de 349/mcL. ¿Cuál de las
siguientes opciones representa el mejor tratamiento a seguir?
a}Iniciar tratamiento antirretroviral,pues el conteo de linfocitos T CD4+lo indica

60e. ¿Cuál de las siguientes opciones representa un desenlace esperado del tratamiento
antirretroviral para considerar éste como exitoso?
a} Reducción de porlo menos 10 veces (1logaritmo} en los niveles de RNA del virus dentro
de 1-2 meses siguientes del inicio del tratamiento, y eventualmente una reducción a < 50
copias/ml

61. Paciente masculino de 1O años de edad el cual es llevado a la sala de urgencias por la
presencia de adenopatía axilar dolorosa. A la exploración física se encuentra una serie de
rasguños en el antebrazo. Se decide realizar una aspiración del nódulo linfático, el cual
produce un material purulento que es enviado a evaluación en el laboratorio. El reporte
revela la presencia de un bacilo altamente pleomórfico. ¿Cuál es con mayor probabilidad
el agente causal de este padecimiento?
d} Bartonella henselae

62. Paciente femenino de 55 años de edad que acude a consulta por ictericia. Al
interrogatorio dirigido la paciente no revela información de utilidad que oriente al
diagnóstico. A la exploración física presenta signo de Murphy negativo, pero con vesícula
biliar palpable. ¿Cuál es el diagnóstico más probable en esta paciente?
c} Cáncer de páncreas

63. Se presenta a consulta geriátrica paciente de 80 años de edad, quien es llevado por su
esposa debido a que lo ha notado más "olvidadizo" que lo usual. Al interrogar al paciente
acerca de su memoria, éste refiere que fue obligado por su esposa para acudir a consulta,
ya que él mismo no hubiera acudido. La esposa ha mencionado que el problema de
memoria de su esposo ha empeorado desde los últimos meses. Los antecedentes sólo son
de importancia por hipertensión de cinco años de diagnóstico. A la exploración
neurológica el paciente muestra deficiencias en la memoria de corto plazo, pero no hay
problemas en cuanto al funcionamiento sensitivo y motor,sin ataxia. Se decide obtener
una tomografía computarizada de la cabeza la cual muestra atrofia cerebral. Es claro que
el paciente presenta un síndrome demencial. ¿Cuál de las siguientes causas reversibles de
demencia es probable que presente el paciente?
e} Hipotiroidismo

64. Mujer gestante secundigesta con antecedente de parto pretérmino a la semana 35,
acude a consulta en la semana 32 de embarazo por la percepción de contracciones. A la
exploración física se detecta cuello uterino permeable, borramiento de 20%, frecuencia
cardiaca fetal de 145 latidos por minuto y contracciones uterinas cada 6 minutos. Se
realiza un ultrasonido, se observa un cuello uterino de 3.5 cm de longitud, fibronectina
negativa. ¿Qué medicamento utilizaría para incrementar la maduración pulmonar del
producto?
a} Betametasona 12 mg cada 24 horas, dos dosis

65. Recién nacido de término de 1O días de vida que se observa con párpados y labios
tumefactos, hipertelorismo, macroglosia, ictericia desde el nacimiento, letargia,fontanela
anterior abombada. ¿Qué resultados en las pruebas de tamizaje y confirmatorias apoyan
el diagnóstico de la enfermedad que sospecha?
a} Niveles de TSH > 25 microunidades internacionales en sangre de talón,TSH sérica >5
microunidades internacionales y T4 sérica > O.7 nanogramos por decilitro

66. Embarazada que se encuentra en primer periodo del trabajo de parto. Contracciones
uterinas cada 3 min con 50 s de duración, con 8 cm de dilatación, 90% de borramiento,
cuello uterino blando, anterior, con membranas íntegras. ¿En qué momento se inicia la
fase activa del primer periodo de trabajo de parto?"
d} Cuando elcuello uterino tiene una dilatación de 3 cm

67. Llega a consulta de urgencias paciente embarazada de 37 semanas de gestación con


cefalea, acúfenos, presión arterial de 180/100, anasarca, ROT normales y proteinuria de 4
g. ¿Qué tratamiento se le brindaría?
b} Control de la presión arterial einducción de parto

68. Acude a urgencias, paciente femenino de 23 años de edad con diagnóstico de


embarazo de 37 semanas de gestación. Presenta dermatosis generalizada, que respeta
palmas y plantas, caracterizada por vesículas sobre fondo eritematoso, pústulas y costras;
así como prurito. Producto sin alteraciones aparentes. ¿Qué acción tomaría?
d} Gammaglobulina específica

69. Acude a consulta paciente femenino de 20 años de edad por amenorrea. Refiere que sí
ha presentado menstruaciones en el pasado. No se observan alteraciones en la
talla,fenotipo ni cardiacas, hay vagina corta. Se realiza un cariotipo encontrando 46XX, los
ovarios se encuentran normales. ¿Cuál de las siguientes es una causa de amenorrea
secundaria?
c} Síndrome de Asherman

70. Varón de 40 años que presenta una dermatosis localizada en la región


perianal,constituida por una neoformacion exofítica en forma de coliflor, de aspecto
vegetante que usted diagnostica como condiloma acuminado. ¿Cuáles son los serotipos
más frecuentes en este padecimiento?
b} 6 y 11

71. ¿En qué pacientes estaría contraindicado el uso de insulina en un esquema intensivo
de tratamiento?
a} Pacientes con retinopatía progresiva

72. El tiempo de duración de acción promedio de la insulina NPH es de:

d} 12 a 15 horas

73. Paciente femenino de 11 años de edad acude por presencia de masa eritematosa
dolorosa en región nasal del párpado inferior del ojo derecho desde hace tres días que ha
ido en aumento. La madre refiere que había presentado cuadros previos semejantes y que
había resuelto con tratamiento no especificado. Desde lactante presentaba descarga de
secreción mucopurulenta a través del punto lagrimal del mismo ojo. A la exploración se
observa eritema cutáneo, edema,se palpa indurada por debajo del tendón cantal medio y
con presión leve sobre el saco lagrimal se obtiene material mucopurulento amarillo-
verdoso. Se observa conjuntiva sin hiperemia, córnea transparente, sin alteraciones de la
motilidad ocular. ¿Cuál es el diagnóstico más probable?
c} Dacriocistitis

74. Varón de 40 años de edad acude por una dermatosis bilateral y asimétrica diseminada
a ambos pies y ambas manos, afectando planta de pie izquierdo y dorso de 1 dedo, dorso
de pie derecho y de 3 y 4 dedos, cara dorsal de 3 dedo de la mano derecha y cara lateral y
5 dedo de la mano izquierda. Se caracteriza por ampollas de contenido líquido claro,
algunas de las cuales están exulceradas. El padecimiento inició hace un mes y medio con
aparición de ampollas de forma espontánea en manos y pies. El paciente niega
antecedente de traumatismos; sin embargo, es diabético tipo 2 de larga evolución, tiene
amputación del 2 dedo del pie izquierdo por osteomielitis y control de la enfermedad
irregular. El diagnóstico es:

d} Bullosis diabeticorum

75. Mujer de 70 años de edad padece diabetes mellitus tipo 2 complicada con retinopatía
proliferativa, neuropatía sensitiva distal e insuficiencia renal crónica, acude a consulta de
revisión. A la exploración física muestra presión arterial 160/95 mm Hg, frecuencia
respiratoria 24 resp/min, ictericia generalizada y debilidad muscular; datos de laboratorio
con anemia normocítica normocrómica (Hb 1O g/dl, VCM 90 fl,HCM 30 pg), creatinina 4
mg/dl, BUN 60 mg/dl, hiperpotasemia, hiperfosfatemia, hipocalcemia (7 mg/dl) y acidosis
metabólica de brecha aniónica elevada (pH 7.32, HC03 18 mEq/L, PaC02 33 mm Hg). Su
depuración de creatinina por Cockcroft-Gault es de 40 mUmin."

75c. La anemia normocítica normocrómica en esta paciente es secundaria a:

c} Déficit de eritropoyetina

76. Se presenta a la sala de urgencias una paciente de 70 años por dolor punzante
epigástrico y hematemesis con sangre rojo rutilante. Al interrogatorio la paciente expresa
que desde hace varios años ha presentado pirosis y ocasionalmente consume antiácidos
sólo cuando el dolor es insoportable. Una vez estabilizada la paciente se realiza
esofagoduodenoscopia, la cual revela una úlcera sangrante en la porción posterior del
duodeno aproximadamente 3 cm distales al píloro. ¿Cuál de las siguientes fuentes es la
arteria más probablemente afectada?

e} Arteria gastroduodenal

77. En un paciente de cinco años de edad se solicita examen general de orina, en el que se
encuentran proteínas. Como parte del seguimiento se decide cuantificar la pérdida
proteínica por orina, lo cual resulta en una proteinuria mayor a 2 mg/dl con sedimento
lipoideo e hipoalbuminemia de 3 mg/dl. ¿Cuál es el hallazgo histológico más probable que
esté presente en una biopsia renal en este paciente?

b} Desaparición delos procesos periféricos de los podocitos observados en microscopia


electrónica

78. ¿Cuál de los siguientes estudios diagnósticos puede ser de utilidad para el diagnóstico
de la seudohiponatremia?
a} Medición de la osmolalidad sérica

79. Un paciente se presenta con pérdida de la sensibilidad al tacto en la cara, pero con
conservación de la sensibilidad dolorosa y térmica, así como ausencia de debilidad de los
músculos de la masticación. ¿Cuál es la estructura más probablemente afectada?
d} Núcleo principal deltrigémino

80. ¿Cuál de los siguientes diagnósticos es más probable en un paciente con hipertensión
arterial, hipopotasemia, alcalosis metabólica y disminución de las concentraciones de
renina y aldosterona?
b} Síndrome de Liddle

81. Paciente masculino de 20 años de edad acude a la sala de urgencias por presentar una
herida por arma de fuego en el epigastrio. Al parecer no hay orificio de salida. A la
palpación, refiere intenso dolor abdominal. Es programado para exploración quirúrgica, y
antes es llevado a tomar una tomografía computarizada. El estudio de imagen demuestra
que la bala lesionó el páncreas. ¿Cuál de los siguientes enunciados es el más adecuado
según la lesión del paciente?
a} La principal causa de muerte en este paciente es la hemorragia asociada a lesiones
vasculares mayores

82. Paciente femenino de 55 años de edad con una úlcera en el tercer dedo de la mano
izquierda. Según refiere, la úlcera se presentó hace siete días y no es dolorosa. Decidió
automedicarse utilizando un antibacteriano tópico e hidrocortisona en crema. Al
interrogatorio no logra recordar algún traumatismo que explique esta lesión.
Latemperatura de la paciente es de 39° C con adenomegalia epitroclear y axilar izquierda.
La úlcera es de alrededor de 3 cm en la cara dorsal del tercer dedo de la mano izquierda y
se encuentra cubierta por una escara negra y rodeada por edema. ¿Cuál de los siguientes
diagnósticos es el más probable en esta paciente?
b} Ántrax cutáneo

83. Paciente femenino de siete años de edad acude al servicio de urgencias por presentar
fatiga y palidez. La paciente había sido tratada hace poco por infección de vías aéreas
superiores. En esta ocasión, la exploración física revela esplenomegalia y equimosis en las
extremidades superiores. Los estudios de laboratorio demuestran un conteo de leucocitos
de 11 000 células/mm3, hemoglobina de 6 g/dL, y conteo plaquetario de 40 000
plaquetas/mm3. La prueba de Coombs es positiva. ¿Cuál es el diagnóstico más probable
en esta paciente?"
c} Síndrome de Evans

84. Un hombre de 74 años de edad acude a consulta por síntomas urinarios que se han
vuelto molestos. En sus antecedentes familiares aparece que su padre murió de cáncer de
colon a los 68 años y su madre falleció en un accidente automovilístico a los 50. En su
historial médico niega cualquier padecimiento, hace 1O años que no veía a un médico. El
paciente vive con su esposa y es capaz de realizar todas las actividades básicas de vida
diaria. Refiere tabaquismo de 35 paquetes/año, y toma un vaso de tequila al día. Niega
hacer ejercicio. Al IPAS refiere que durante el mes pasado ha tenido frecuencia, teniendo
que orinar cada dos horas; aproximadamente la mitad de las veces que va, de urgencia la
mitad del tiempo, nicturia menos de una vez en cinco y chorro débil menos de la mitad del
tiempo. Niega tenesmo vesical, intermitencia, y necesidad de pujar. Refiere que en los
últimos seis meses ha perdido siete kg, sin tener un cambio en sus hábitos alimentarios.
Además refiere sentirse más cansado en general, aunque niega disnea o dolor torácico. A
la exploración física sus signos vitales incluyen FC 86/min, FR 15/min,T 36.8 C y PA 135/87
mm Hg. En el tacto rectal se palpa una próstata agrandada, sin nódulos, cambio de textura
e indolora. Entre los estudios de laboratorio que se piden se le solicita un examen de
orina, que se muestra a continuación:

84a. ¿Cuál sería el siguiente paso en el diagnóstico de este paciente?

d} Cistoscopia

84b. Se realizan los estudios necesarios y se descarta la presencia de cáncer renal o del
tracto urinario. Se lleva a cabo la medición de un antígeno prostático, que resulta ser de
2.0 ng/ml. La glucosa sérica con glucometría capilar es de 98 mg/dl. ¿Cuál sería el
siguiente paso en el manejo de este paciente?
b} Iniciar tratamiento con finasterida

84c. ¿Cuál es el mecanismo de acción de la finasterida?


c}Inhibición dela 5-alfa reductasa

84d. ¿En qué parte de la glándula se desarrolla más comúnmente el cáncer de próstata?
a} Zona periférica

85. Paciente femenino de 58 años de edad acude a consulta por pirosis, fatiga, pérdida de
peso, palpitaciones, diarrea,eritema facial súbito y edema de tobillos desde hace seis
meses. Menciona que ha presentado desde hace varias semanas sensación de plenitud en
el cuello. No hay antecedentes de importancia, pues siempre ha sido sana. Al
interrogatorio niega el consumo de medicamentos. La exploración física revela pulsos
irregulares, distensión venosa yugular y un soplo holosistólico con variación de su
intensidad respecto a la respiración, y edema periférico. ¿Cuál es el diagnóstico más
probable de esta paciente?
c} Síndrome carcinoide
86. ¿Cuál de los siguientes calendarios de administración de la vacuna DPT es el más
correcto según el esquema de vacunación de México?
c} A los dos, cuatro, seis y 18 meses de edad y a los cuatro años de edad

87. Paciente masculino de 19 años de edad acude a consulta de rutina, ya que lo exige el
comité organizador del torneo mundial de canotaje al cual va a asistir próximamente. Se
muestra preocupado porque hace dos meses, durante un entrenamiento, casi se desmaya
y cae al agua. Ha estado entrenando desde hace tres años, y desde entonces refiere que
ha presentado casos aislados de palpitaciones. En los antecedentes no hay historia de
hipertensión o algún familiar que haya sufrido muerte súbita o enfermedad arterial
coronaria prematura. A la exploración física la presión cardiaca es de 119/75 mm Hg, y la
frecuencia cardiaca de 55 latidos/min;es posible percibir un soplo sistólico áspero grado
2/4, en crescendo-decrescendo en el borde esternal izquierdo, el cual se vuelve más
intenso cuando el paciente adquiere la posición de pie. ¿Cuál de los siguientes enunciados
es el más correcto respecto al estado de salud de este paciente?

b} La hipertrofia persistente del ventrículo izquierdo demostrada por ecocardiografía


después de varios meses de que se ha abandonado elejercicio es consistente con el
diagnóstico de miocardiopatía hipertrófica

88. ¿Cuál de las siguientes opciones representa al agente infeccioso más frecuente en
pacientes de cuatro meses de edad con secreciones nasales copiosas, sibilancias y tos, en
especial en la época de invierno?
d} Virus sincitial respiratorio

89. ¿Cuál de las siguientes enfermedades se caracteriza por hipertensión pulmonar?


a} Esclerodermia

90. ¿Cuál de los siguientes medicamentos se relaciona con trombocitopenia y


microtrombosis?
a} Heparina

91. ¿Cuál de las siguientes descripciones corresponde con las hemorragias en astilla?
d} Lesiones lineares o enforma de llama

92. Una mujer de 40 años de edad acude para extirpación de "verrugas" en el cuello
porque le duelen cuando se pone una cadena. Tiene una dermatosis localizada,
bilateral,simétrica que afecta cuello y axilas. Se caracteriza por múltiples neoformaciones
exofíticas, pediculadas, bien delimitadas, lisas, del color de la piel. Le molestan con cuellos
altos de la ropa y cadenas y refiere aumento después de un embarazo. Ella tiene
sobrepeso y resistencia a la insulina.
92a. ¿Cuál es el diagnóstico más probable?
d} Fibromas blandos

92b. ¿A cuál enfermedad sistémica se relacionan los fibromas blandos?


e} Diabetes mellitus o síndrome metabólico

92c. ¿Cuál es la causa aceptada de los fibromas blandos?


c} Sobrepeso

92d. ¿Cuál es el tratamiento de elección para los fibromas blandos?


d} Rasurado

92e. ¿Cuál es el tratamiento preventivo de los fibromas blandos?


e} Ninguno

93. Paciente femenino de 31 años acude a consulta y comenta que fue diagnosticada con
lupus eritematoso sistémico (LES) hace 2 años. Inició con eritema malar, dolor articular y
síntomas neuropsiquiátricos. Además, se le realizaron estudios en donde se encontraron
ANA positivos; actualmente está controlada. Acude a consulta pues desea embarazarse y
quiere tener mayor información al respecto. No tiene otros antecdents de importancia. A
la exploración física registra FC: 67, FR: 20, Temp: 36 ºC; el resto sin alteraciones.
93a. ¿Cuál de los siguientes planteamientos es incorrecto en cuanto al embarazo y el LES?

b} El LES es contraindicación absoluta de embarazo

93b. En cuanto a la nefritis lúpica, ¿cuál es el subtipo más común en las pacientes con LES?
c} Proliferativa difusa

93c. En el LES, ¿qué anticuerpos se asocian a enfermedad renal?


c} DNA doble cadena

93d. ¿Cuál es el principal efecto adverso de la hidroxicloroquina?


a} Daño en retina

93e. Es un criterio de clasificación de LES:


b} Linfopenia {menor a 1500/mm3}

94. La Comisión Nacional de los Derechos Humanos, específicamente en el Programa de


Asuntos de la Mujer y de igualdad entre mujeres y hombres, publica los delitos de
hostigamiento sexual, abuso sexual,estupro y violación, de los cuales es preciso saber:
94a. El abuso sexual por medio de la violencia física o moral y si la víctima fuera la esposa
c} 8 a 20 años de prisión

94b. Se equipara a la violación, al que sin consentimiento de una persona o con el


consentimiento de un o una menor de 14 años introduzca uno o más dedos o un objeto de
cualquier naturaleza en la región anal o vaginal,a esto se le llama:"
b} Violación impropia

95. Ante un niño de cinco años que presentó síndrome anticolinérgico por intoxicación
medicamentosa, ¿cuál de las siguientes NO forma parte del síndrome de toxicidad de los
antihistamínicos?
e) Diarrea

96. Paciente femenino de 52 años con obesidad mórbida acude a urgencias por presentar
edema del miembro pélvico izquierdo de tres días de evolución que ha ido aumentando
progresivamente. La paciente refiere que hace tres días comenzó con fiebre y después
notó edema y eritema en el pie izquierdo que ha ido ascendiendo hasta cubrir toda la
extremidad. El día de ayer aparecieron grandes ampollas que al romperse dejan salir
líquido claro. A la exploración física usted nota temperatura de 38.5 ºC, el miembro
pélvico izquierdo se observa con edema 4+ con godete, toda la extremidad es dolorosa a
la palpación y está cubierta por placas eritematosas de bordes elevados y bien definidos,
calientes al tacto con ampollas y vesículas de las que sale un líquido claro. Los pulsos son
2/4 en ambas extremidades.
¿Cuál es el diagnóstico más probable?
d} Erisipela

97. Acude a Urgencias un paciente masculino de 35 años, sin antecedentes de


importancia,con fiebre de 3 días de evolución, cefalea, fotosensibilidad y somnolencia. A
la exploración física se nota rigidez de cuello, así como signo de Kernig y Brudzinski
positivos. Los signos vitales son temperatura de 39 ºC, FC: 98, FR: 25, TA: 110/70 mm Hg.
Se toma la decisión de internarlo para estudiarlo.
97a. ¿Cuál es el agente etiológico más frecuente de meningitis en pacientes de esta edad?
a} S. pneumoniae

97b. ¿Cuál de las siguientes condiciones NO es indicación para realizar una tomografía de
cráneo antes de una punción lumbar?
c} Fiebre

97c. Usted lleva a cabo una punción lumbar, por medio de la cual encuentra los siguientes
datos en el LCR: presión de apertura elevada, 9 000 PMN, glucosa 20 mg/dL, proteínas de
1 000 mg/dL, lo cual lo lleva a sospechar de una meningitis:"
b} Bacteriana
97d. El médico de base no va a llegar hasta dentro de 8 h,así que usted tiene que decidir el
tratamiento. ¿Qué tratamiento administraría?
d} Ceftriaxona más vancomicina más dexametasona

97e. Si se tratara de un paciente con meningitis meningocócica, ¿cuál es el tratamiento


profiláctico para los contactos cercanos?
b} Rifampicina

98. Mujer de 20 años de edad quien presenta desde hace 1O años cuadros remitentes en
zonas de exposición solar caracterizados por pápulas eritematosas, costras hemáticas,
exulceraciones y zonas de liquenificación,algunas manchas y cicatrices residuales. Además
presenta conjuntivitis y seudopterigión. Refiere que la enfermedad empeora en épocas de
verano y le provoca prurito intenso que no mejora con antihistamínicos.
98a. ¿Cuál es su diagnóstico?
e} Prúrigo actínico

98b. ¿Cuál es el tratamiento de elección del prúrigo actínico?


c} Talidomida

99. Acude a consulta paciente masculino de 67 años, fumador,por un cuadro de malestar


general y dolor en columna vertebral que inició hace 3 meses y ha ido progresando. A la
exploración física solamente llama la atención el dolor difuso en la columna vertebral. En
los laboratorios se encuentra anemia normocítica normocrómica, plaquetas 115
000/µL,VSG 120, creatinina de 2.8 mg/dL, proteínas totales 8.5 g/dL e hipoalbuminemia
de 2.1 g/dL, orina con proteinuria de 4 g/L e hipercalciemia de 11.2 mg/dL.
99a. De acuerdo con el caso clínico, ¿cuál es su sospecha diagnóstica?
a} Neoplasia maligna de células plasmáticas

99c. Para estadificar la enfermedad se emplean los siguientes criterios, según Durie-
Salmon:
b} Hb, Ca, lesiones líticas en hueso, lgG olgA o cadenas ligeras en orina

99d. ¿Cuál es el tratamiento de un paciente con esta enfermedad que se encuentra


asintomático o en estadio I?
d} No se da tratamiento

99e. ¿Cuál de los siguientes enunciados no es verdadero?


b} El pico monoclonal en la mayoría de los casos es de lgA
1OO. Paciente masculino de 65 años con diagnóstico de envío de enfermedad ampollosa
no especificada presenta dermatosis diseminada que afecta tronco y mucosa oral y
conjuntiva!, además del ano, caracterizada por ampollas tensas, eritema y descamación
en encías. En la valoración oftalmológica se encuentra conjuntivitis inespecífica,triquiasis y
entropión que se acompañan de disminución en la agudeza visual. En mucosa anal
presenta atrofia y erosiones. Refiere cinco años de evolución y ha recibido múltiples
tratamientos tópicos a base de antibióticos y corticoesteroides así como neosporina
oftálmica. Mencione cuál de las siguientes afirmaciones es correcta.
c} Afecta hasta en 85% la mucosa oral además de afección oftálmica

101. Paciente masculino de 70 años, con antecedente de hipertensión arterial de 5 años


de evolución controlada con captopril 25 mg, es fumador desde hace 20 años a razón de
una cajetilla diaria y fue diagnosticado con enfermedad pulmonar obstructiva crónica hace
2 años. Acude a Urgencias debido a que ha aumentado bastante la tos y presenta un leve
aumento del esputo, así como disnea de esfuerzos mínimos. La radiografía muestra datos
típicos de enfisema pulmonar.
101a. ¿Qué tratamiento le daría a este paciente en Urgencias para mejorar su estado
actual?
c} Salbutamol y bromuro de ipratropio

101c. Su paciente presenta una FEV1 de 48%. ¿En qué estadio de GOLD está?
c} Severo

101d. De acuerdo con el estadio de GOLD en el que se encuentra su paciente, ¿cuál es el


tratamiento crónico indicado para él?
a} Tiotropio diariamente y un esteroideinhalado en exacerbaciones

101e. ¿Cuál de las siguientes es una indicación para la utilización de oxígeno


suplementario en casa?
b} Saturación de 02 menor a 89%

102. Masculino de 2 días VEU. Producto G:I, madre 24 años sana, control prenatal
irregular. Dos US gestacionales normales. Obtenido por parto eutócico a las 37 sdg.
Exploración: peso 2 150 g, Talla 46 cm, PC 37 cm Apgar 7/8. Fontanela anterior 5 x 3 cm.
Borde hepático palpable 4-4-4 cm bajo borde costal, esplenomegalia 3 cm bajo borde
costal. Adenopatías palpables de 0.5 cm submaxilares, axilares e inguinales.
La entidad patológica causal de la sintomatología de este paciente es:
d} Toxoplasmosis
103. Paciente femenino de seis años de edad con bajo rendimiento escolar se presenta a
la consulta pediátrica. La maestra informa que con frecuencia la niña tiene periodos de
inatención en los que se queda mirando fijamente al pizarrón o cualquier otro punto con
un parpadeo rápido. ¿Cuál es el medicamento más adecuado para la condición que
presenta esta paciente?
d} Etosuxamida

104. ¿En el medio hospitalario, las infecciones producidas por gramnegativos productores
de betalactamasas de amplio espectro son más frecuentes de ocurrir posteriormente al
uso frecuente de cuál de la siguiente clase de antibióticos?
a} Cefalosporinas de tercera generación

105. Acude a consulta un paciente masculino de 52 años por un dolor intenso


periorbitario unilateral,acompañado de inyección conjuntival, lagrimeo y rinorrea.
Comenta que esto le sucede dos veces al día, aprox. desde hace 2 semanas, e incluso en
una ocasión lo despertó en la madrugada. Como antecedente de importancia tuvo un
infarto agudo de miocardio hace 2 años con aplicación de stent. Toma diariamente
aspirina 85 mg y clopidogrel. A la exploración física, los signos vitales son FC: 70, FR: 18,
TA: 130/85 mm Hg, Temp: 36 ºC. El resto de la exploración es normal.
105a. ¿Cuál de los siguientes es el tratamiento más efectivo para este paciente?

d} Prednisolona e inhalación de oxígeno

105b. ¿Cuál es de los siguientes datos no sirven como indicadores para tomar una
neuroimagen en el caso de cefalea?
c} La visión de escotomas

105c. En cuanto a la cefalea tensional,¿cuál de los siguientes medicamentos se utiliza


como profilaxis?
a} Amitriptilina

105d. En cuanto a la cefalea migrañosa, ¿en qué casos se deben de indicar medicamentos
profilácticos?
d} Todos los anteriores

105e. ¿A qué se le conoce como estatus migrañoso?


a} Cuando las manifestaciones clínicas duran más de 72 hy no responden a tratamiento
habitual

106. ¿En cuál de los siguientes escenarios NO se indica la colonoscopia?


e} Paciente femenino de 64 años de edad con colitis ulcerativa desde hace 25 años y que
se presenta en este momento con megacolon tóxico

107. ¿Cuál de los siguientes fármacos antibióticos ejerce su efecto mediante la inhibición
de la formación de puentes cruzados en la pared
celular de peptidoglucano?
b} Penicilina
108. ¿Cuál de las siguientes aseveraciones respecto a la vacuna del virus del papiloma
humano es correcta?
e} Las mujeres vacunadas conla vacuna contra la infección delvirus del papiloma humano
deben continuar acudiendo anualmente para la realización de citología y tinción de
Papanicolau

109. Un hombre de de 71 años de edad acude a consulta porque presenta dolor en la


región glútea izquierda que se irradia hacia la porción lateral del muslo y el dorso del pie.
El paciente niega la presencia de debilidad. Al interrogatorio menciona que el dolor
empeora cuando inclina el cuerpo hacia delante, por ejemplo, cuando intenta amarrar las
agujetas de sus zapatos. No existen alteraciones de la función vesical o rectal anal. A la
exploración física no existen anormalidades en la marcha. Cuando eleva su miembro
inferior en completa extensión se puede reproducir el dolor.Al continuar con la evaluación
es evidente que el extensor largo del primer dedo izquierdo se encuentra parético en
comparación con el lado contralateral. Los reflejos patelares y del tendón de Aquiles no
muestran anormalidades. ¿Cuál de los siguientes diagnósticos es el más probable en este
paciente?
b} Radiculopatía a nivel de L5

11O. Se presenta al departamento de urgencias un paciente de 19 años de edad que sufrió


un accidente automovilístico. El paciente se encuentra hipotenso e hiperventilando. Los
estudios de laboratorio revelan que el paciente sufre una acidosis metabólica con
incremento de la brecha aniónica y aumento del lactato plasmático. ¿Cuál de los
siguientes eventos a nivel bioquímico explica de mejor manera el cuadro de
este paciente?
e} Disminución dela actividad dela enzima piruvato deshidrogenasa

111. ¿Cuál de los siguientes hallazgos NO se observa en la enfermedad de Wilson?


a} Elevación de ceruplasmina
112. ¿Cuál de los siguientes medicamentos presenta un riesgo incrementado de
desarrollar colitis seudomembranosa?
b} Clindamicina

113. ¿Cuál de los siguientes antimicóticos NO es efectivo en el tratamiento de la fungemia


por Gandida albicans?
b} Terbinafina

114. Según la clasificación de fracturas del platillo tibial de Schatzker, ¿cuál de las
siguientes opciones representa una fractura bicondilar?
e} Tipo V

115. Se presenta a consulta un lactante con evidente retraso mental. Durante la


exploración física se encuentra que el paciente presenta hepatomegalia. Por sospecha
clínica se solicita a la enfermera que mida las concentraciones de glucosa capilar,las cuales
resultan disminuidas. Se solicita una biopsia hepática la cual demuestra acumulación de
materialtipo dextrinas en el citoplasma de los hepatocitos con ausencia de infiltración
grasa. ¿Cuál de las siguientes enzimas es la que con mayor probabilidad se encuentra
deficiente en este paciente?
c} Amilo-1,6-glucosidasa

116. Paciente masculino de 58 años de edad acude a consulta externa por saciedad
temprana y pérdida de peso inexplicado (cerca de 14 kg en ocho meses). Cuenta con
antecedente de hipertensión arterial (controlada con inhibidores de la enzima
convertidora de angiotensina) y diabetes mellitus tipo 2 de 12 años de diagnóstico
(controlada con insulina). El paciente ya había acudido con otro médico, debido al temor
de padecer cáncer, sin embargo, a pesar de que se le realizaron varios estudios no se le
diagnosticó ningún tipo de cáncer. Debido al antecedente endocrinológico se decide
realizar una prueba de vaciado gástrico, la cual demuestra lentitud del vaciamiento
gástrico. ¿Cuál de los siguientes efectos adversos es el que puede presentarse con más
probabilidad con el tratamiento que se le debe aplicar al paciente?
d} Trastornos extrapiramidales

117. Cual de las siguientes aseveraciones respecto a la policitemia vera es el mas


correcto?
e)La elevacion de los niveles de eritropoyetina excluyente el diagnostico de policitemia
vera
118. Cual de los sigueintes organismos se asocia con zonas deserticas y se presentan como
esferulas con endosporas en la biopsia de tehidos infectados?
b)Coccidioides immitis

119. Cua de las siguientes alteraciones es la unica que se asocia a ateroesclerosis


prematura?
b)Homocistinuria

120. Cual de los siguientes esquemas de tromboembolia venosa es el mas adecuado en


pacientes de alto riesgo para desarrollar esta?
c) Administracion subcutanea de heparina de bajo peso molecular

1. Paciente femenino de 32 años de edad, quien tiene como antecedentes relevantes crisis
convulsivas por cisticercosis, previamente tratada con valproato; hace 4 semanas se hizo
cambio por carbamazepina. Hace 3 días inicia con malestar general, astenia, adinamia,
hiporexia y hepatalgia, el día de hoy inicia con eritema morbiliforme generalizado, motivo
por el cual acude a consulta. FC 95 lpm, FR 18 rpm, temp. 38.6ºC, PA 110/70, peso 58 kg,
talla 1.68 m.
1a. Con los datos clínicos anteriores, usted realiza el diagnóstico de:
b} Síndrome de DRESS

1b. Estadísticamente, ¿qué órgano se afecta con mayor frecuencia en el síndrome de


DRESS?
b} Hígado

1c. Usted solicita una biometría hemática que reporta eosinofilia > 1.5 x 109 L, los
laboratorios que solicita reportan elevación de TGO, TGP y DHL, con tiempos de
coagulación prolongados. Usted solicita una biopsia de piel,¿qué hallazgos espera que se
reporten?
a} Paraqueratosis, acantosis local e infiltrado linfocitario con epidermotropismo

1d. Se debe iniciar el tratamiento, empezando por retirar el fármaco que provocó el
síndrome. Usted inicia el tratamiento con:
d} Esteroides

1e. Se han documentado diversos datos de severidad del síndrome de DRESS, ¿cuál se
asociacon mayor mortalidad?
b} Necrosis hepática

2. Paciente masculino de 31 años de edad con antecedente de dos semanas de fiebre, a lo


cual se agregó hace una semana distensión abdominal,estreñimiento y dolor epigástrico.
Entre otras molestias también refiere cefalea, escalofríos, mialgias y malestar general. La
exploración física revela la presencia de un "exantema rosado"(exantema maculopapular
color salmón que blanquea a la presión), principalmente en el tórax, así como dolor
abdominal generalizado a la palpación y esplenomegalia. Presenta temperatura de 38.9
ºC, frecuencia cardiaca de 45 latidos por minuto, frecuencia respiratoria de 20
respiraciones por minuto, presión arterial de 100/65 mm Hg. Los estudios de laboratorio
básicos son relevantes por leucopenia y neutropenia. Se realiza un hemocultivo el cual
resulta positivo para Salmonella typhi. ¿Cuál es el tratamiento antibiótico más efectivo en
cepas no provenientes de brotes resistentes?
a} Fluoroquinolonas

3. Paciente masculino de 36 años, arqueólogo, diabético. Estuvo en contacto con


pacientes con tuberculosis en su último viaje a Zimbabwe. Comenzó con fiebre, tos
productiva con estrías sanguinolentas y ataque al estado general. Durante su traslado
presentó sudores nocturnos. La radiografía de tórax revela infiltrados nodulares en lóbulo
superior,con una lesión cavitada que está drenando a través de un bronquio. PPD
negativo. Se inicia tratamiento antituberculoso. Acude a consulta a ajuste de
hipoglucemiantes orales debido a que su glucosa sérica se elevo casi el doble de lo que
manejaba hace tres meses. Refiere que ha respetado su dieta y ha estado tomando su
medicación. Pocas semanas después del inicio del tratamiento antituberculoso el paciente
reporto disminución de tos e incremento de energía.
¿Cuál es la explicación de la hiperglucemia en este paciente?
b} Rifampicina

4. Paciente masculino de nueve años de edad con antecedente de otorrea izquierda


intermitente de un año de evolución, febril y con dolor retroocular ipsolateral. Otoscopia
izquierda, membrana timpánica con perforación de 80%, otorrea purulenta y fétida.
Movilidad de ojo izquierdo limitado en la abducción. El diagnóstico más probable es:"
b} Apicitis petrosa

5. Masculino de 40 años acude a consulta por presentar desde hace 5 meses dolor
retroesternal y molestias al ingerir alimento. Dichas molestias son de carácter
intermitente y el dolor a veces se irradia a la mandíbula. No refiere pirosis ni episodios de
regurgitación.
5a. ¿Cuál es su sospecha diagnóstica?
d} Espasmo esofágico difuso

5b. Usted solicita una manometría.¿Qué resultado apoyaría su sospecha diagnóstica?


a} >20% de las contracciones delos dos terciosinferiores son peristálticas, de amplitud
variable (amplitud media >30 mm Hg}

5c. La imagen característica en el trago de bario es:


b} Esófago en sacacorchos
5d. ¿Cuál de las siguientes opciones no es parte del manejo de estetrastorno?
d} Funduplicatura

5e. En caso de que el tratamiento conservador fracasara, ¿cuál otro no estaría indicado?
d} Funduplicatura

6. Paciente femenina de 22 años. Inicia con fatiga, malestar general y mialgia de una
semana de duración; posteriormente aparece fiebre y faringitis, por lo que acude a
consultar. A la exploración se registra TA: 125/85 mm Hg, FC: 88 lpm,Temp: 37.8 ºC.
Faringe hiperémica con hipertrofia amigdalina y exudado blancuzco bilateral, adenopatías
cervicales posteriores, y esplenomegalia. Biometría hemática: Hb: 13 mg/dl, Htc: 39%,
VCM: 88, leucocitos totales: 16 000, linfocitos: 7 200, plaquetas: 127 000, ALT: 187 U/L,
AST 250 U/L, BT: 1.8 mg/dl.Anticuerpo heterófilo positivo.
6a. El diagnóstico que se integra por la clínica y el laboratorio es:
b} Mononucleosis infecciosa

6b. ¿Cuál es el microorganismo causal de la enfermedad de esta paciente?


d} Virus de Epstein-Barr

6c. ¿Cuál es el porcentaje de linfocitos atípicos que se espera encontrar en la enfermedad


que cursa el paciente?
c} > 10%

6d. ¿Cuál es el tratamiento indicado en esta paciente?


d} Medidas de sostén y reposo

6e. En relación con la fisiopatología de la mononucleosis infecciosa, señale el enunciado


correcto:
a} La rotura esplénica es una complicación dela enfermedad

7. Hombre de 35 años de edad, fumador y bebedor social desde los 17 años, consulta por
haber observado sangre fresca en el retrete y en el papel higiénico tras obrar; no refiere
dolor ni cambios en sus hábitos intestinales, así como tampoco sensación de masas o
tumoración. A la exploración física se observa región anal sin sangre ni tumoraciones. TA:
120/SOmm Hg, FC: 70x', Temp: 36.5ºC.
7a. Su principal sospecha diagnóstica es:
c} Hemorroides

7b. ¿Cuál es la causa de este padecimiento?


b} Dilatación anormal delas anastomosis arteriovenosas
7c. Es la referencia para la clasificación de esta enfermedad.
b} Línea dentada

7d. El primer paso dentro del tratamiento es:


a} Fibra en la dieta y ablandadores de heces

7e. La escleroterapia y la ligadura con banda elástica están contraindicadas en:


d} Hemorroides externas

8. Paciente masculino de 28 años de edad, cuyo padre padece artritis psoriásica, inicia
padecimiento hace 4 meses con dolor lumbar que disminuye por las tardes, acompañado
de rigidez matutina, dolor glúteo bilateral y recientemente gonartralgia izquierda difusa al
subir escaleras. A la exploración física refiere dolor a la palpación de articulaciones
sacroilíacas, apófisis espinosas lumbares, y a la flexión del muslo izquierdo. VSG elevado,
factor reumatoide negativo.
8a. Señale el diagnóstico más probable en este paciente:
d} Espondilitis anquilosante

8b. ¿Cuál es la manifestación extraarticular oftalmológica característica en la espondilitis


anquilosante?
b} Uveítis anterior

8c. De acuerdo con los hallazgos radiográficos, señale la respuesta correcta:


a} Los cuerpos vertebrales "cuadrados" son un signo radiológico de espondilitis
anquilosante"

8d. Se considera limitación en la movilidad de la columna lumbar cuando el resultado de la


prueba modificada de Schober es:
c} < 5 cm

8e. Respecto al tratamiento indicado en este paciente, señale el enunciado correcto:


d} Debe recibir AINE y un programa de rehabilitación

9. Paciente femenino de 50 años es llevada al servicio de urgencias por presentar disnea


de una semana de evolución que ha progresado hasta ser de pequeños esfuerzos y se
acompaña en ocasiones de palpitaciones y dolor torácico que dura pocos minutos. La
disnea fue máxima esta mañana, asociada con dolor retroesternal 7/1O que aumenta con
la respiración. La paciente no tiene historia de tabaquismo, niega cirugías recientes. Su
esposo refiere que la ha visto cansada y que ya casi no se levanta de la cama desde que le
colocaron una férula en el tobillo izquierdo debido a que sufrió un esguince hace dos
semanas. A la exploración física es evidente la dificultad respiratoria con uso de músculos
accesorios, signos vitales con frecuencia respiratoria de 26, frecuencia cardiaca 105 lpm,
presión arterial 85/50 mm Hg, temperatura 37.5 ºC. Se aprecia ingurgitación yugular de 8
cm, taquicardia, S1 y S2 sin soplos ni ruidos accesorios, campos pulmonares bien
ventilados y edema de miembros pélvicos, especialmente del lado izquierdo, con dolor a
la palpación de la pantorrilla izquierda.
La sospecha clínica de que esta paciente presente una embolia pulmonar es:
d} Alta

1O. Paciente femenino de cinco años es traída a urgencias por sus padres quienes la
encontraron jugando en un campo que había sido regado con pesticidas. La paciente
presentó inicialmente abundante salivación y lagrimeo, vómito, diarrea e incontinencia
urinaria seguida de confusión y contracciones musculares generalizadas. A la exploración
física se encuentra letárgica, diaforética, con pupilas mióticas y fasciculaciones."
Usted de inmediato sospecha intoxicación por organofosforados.
¿Cuál es el medicamento adecuado para tratar los efectos nicotínicos en esta paciente?
e} Pralidoxima

11. Masculino de 14 años, sin antecedentes patológicos,es traído al hospital por sus
padres tras habersufrido una lesión mientras jugaba futbol americano. El paciente
comenta que al ser tacleado cayó sobre el hombro derecho e inmediatamente sintió dolor
en la región (EVA 10/1O); dicho dolor ha ido en aumentoy le impide mover la extremidad
superior derecha. A la exploración usted comprueba que a la movilización de dicha
extremidad el dolor es insoportable y observa que el hombro derecho está descendido
con respecto al izquierdo; al palpar la zona superior derecha del tórax, usted nota edema,
crepitación y movilidad anormal en la región.
11a. ¿Cuálsería su sospecha diagnóstica?
a} Fractura clavicular

11b. ¿Qué estudio solicitaría para confirmar su diagnóstico?


a} Radiografía de tórax anteroposterior

11c. ¿Cómo se clasifica esta lesión?


c} Terciolateral,tercio medio o tercio medial

11d. El tratamiento clásico de esta lesión es:


a} Vendaje o dispositivo en "8"

11e. Fractura más frecuente durante el trabajo de parto:


c} Clavicular
12. Paciente femenina de 68 años de edad, ingresa a Urgencias por disfasia motora
mientras comía y que posteriormente evolucionó a afasia. Sus medicamentos incluyen
amlodipino, aspirina,atorvastati na. A la exploración física se encuentra consciente y
reactiva,con hemiparesia e hemihipoestesia faciobraquial derecha. ECG: frecuencia
ventricular 160 x', irregular,QRS angosto y ondas f.
12a. Señale el territorio vascular donde se ubica la oclusión.
b} Arteria cerebral media

12b. ¿Cuál es el origen fisiopatológico más probable del cuadro de la paciente?


c} Embolia

12c. Principal factor de riesgo para la enfermedad vascular cerebral.


a} Hipertensión arterial

12d. ¿Cuál de las siguientes opciones es una contraindicación para la administración de rt-
PA en el EVC isquémico?
d} Síntomas que mejoran con rapidez

12e. ¿Cuánto tiempo es el periodo de ventana para iniciar un tratamiento efectivo con rt-
PA en el EVC isquémico?
a} 3 h

13. Masculino de 11 años de edad es traído a consulta por su madre, la cual comenta que
la maestra del niño le ha reportado que lo nota distraído en las clases y que, por otra
parte, en la casa ve la TV muy de cerca;además de que por la tarde trae los ojos muy rojos.
Durante la entrevista usted se da cuenta de que el niño entrecierra mucho los ojos para
observar los objetos en la pared. Al interrogar al niño, éste comenta que le cuesta trabajo
ver de lejos, por lo que se acerca para ver bien,y que después de ver mucho tiempo la TV
le da dolor de cabeza.
13a. ¿Cuál sería su diagnóstico para este paciente?
b} Miopía

13c. ¿Cómo se define un dioptrio?


c} Toda superficie que separa dos medios con distinto índice de refracción

13d. Del sistema óptico del ojo, ¿cuál estructura es la más potente?
b} Córnea

13e. El tratamiento de primera elección para este paciente sería:


c} Lentes divergentes

14. ¿Cuál de las siguientes condiciones se relaciona con la presencia de macrocitosis en el


frotis de sangre periférica?
b} Hipotiroidismo

15. Paciente masculino de 84 años de edad que se presenta al servicio de urgencias por
presentar de manera súbita cefalea y fatiga,así como pérdida de la visión. A la exploración
física el paciente sólo puede percibir ciertos destellos luminosos en el derecho, mientras
que en el ojo izquierdo presenta una agudeza visual de 20/20. A la exploración del reflejo
luminoso oculomotor se puede evidenciar un defecto aferente pupilar en la pupila
derecha (pupila de Marcus Gunn). A la oftalmoscopia directa se observa una retina pálida
con una mancha roja brillante en la fóvea. ¿Cuál es el diagnóstico más probable en este
paciente?

b} Oclusión de la arteria central de la retina

16. ¿Cuál de los siguientes cambios neuropatológicos se observa en el síndrome de


Marchiafava-Bignami?

d} Desmielinización del cuerpo calloso yla comisura anterior

17. Paciente masculino de 65 años de edad actualmente bajo tratamiento de depresión


mayor con sertralina, diabetes mellitus con hipoglucemiantes orales, e hipertensión con
metoprolol. ¿Cuál de los siguientes efectos adversos secundarios al tratamiento
antidepresivo es más probable que este paciente presente?
a} Disfunción eréctil

18. ¿Cuál de los siguientes enunciados es incorrecto respecto a las esotropías en lactantes
menores de seis meses?

d} El tratamiento quirúrgico debe posponerse hasta la

19. ¿Cuál de los siguientes efectos adversos corresponde a la administración de


vincristina?
a} Alteraciones de la sensibilidad en las regiones distales de las extremidades

20. ¿Cuál de las siguientes proteínas del virus del papiloma humano produce la neoplasia
intraepitelial cervical?
c} E6

21. Paciente masculino de 32 años de edad con diagnóstico de infección por el virus de la
inmunodeficiencia humana y síndrome de inmunodeficiencia adquirida. El paciente está
recibiendo actualmente altas dosis de ganciclovir debido a la presencia de colitis
hemorrágica.
¿Cuál sería el efecto secundario que más probablemente presentaría este paciente si se le
administra concomitantemente zidovudina?
c} Neutropenia
22. Paciente masculino de ocho años de edad que acude al servicio de urgencias llevado
por su madre. Ésta se muestra muy preocupada porque hace poco su hijo tuvo una
aparente crisis convulsiva. Reporta que abruptamente el niño perdió el estado de
alerta,cayó al piso y empezó a "agitarse". Al interrogatorio la madre menciona que en el
pasado su hijo había presentado episodios en los que mira fijamente un punto en el
espacio, en varias ocasiones durante el mismo día. A la exploración física el paciente se
presenta desorientado y somnoliento. El médico de guardia decide ingresarlo para
observación. Después de varias horas el paciente recupera un estado normal de alerta,
por lo que es egresado con indicaciones especiales y una consulta a neurología pediátrica.
¿Cuál de los siguientes agentes farmacológicos es el más adecuado en el tratamiento a
largo plazo para este paciente?
b} Ácido valproico

23. Paciente femenino de 61 años de edad que acude a la sala de urgencias por la
presencia de hemoptisis en varias ocasiones durante las últimas 48 horas. Reporta que
entre todos los episodios es probable que haya expulsado una cantidad de sangre similar
al contenido de una pequeña cuchara. Entre los antecedentes de importancia se
encuentra el de tabaquismo desde los 18 años de edad con frecuencia de una cajetilla
diaria. La radiografía de tórax muestra infiltrados difusos bilaterales de predominio
inferior. Los estudios de laboratorio reportan un hematocrito de 27% y creatinina sérica
de 4.9 mg/dL;el examen general de orina demuestra cilindros eritrocitarios y proteinuria.
¿Cuál de los siguientes anticuerpos se encuentra con mayor probabilidad en esta
paciente?
e} Anticuerpos contra la membrana basal glomerular

24. ¿Cuál de los siguientes fármacos puede empeorar la oftalmopatía infiltrativa de la


enfermedad de Graves?
b} Yodo radiactivo

25. Paciente masculino de 25 años de edad que se presenta al consultorio para una
consulta de rutina,debido a que esto es un requisito en su trabajo. Al interrogatorio el
paciente comenta que es aparentemente sano, no presenta ninguna enfermedad ni fuma.
Refiere que realiza ejercicio aeróbico cinco veces a la semana. Entre los antecedentes
heredofamiliares se niega la presencia de familiares con diabetes o cáncer de colon. Al
continuar el interrogatorio el paciente menciona que es homosexual y tiene una pareja
estable desde hace tres años. Durante las relaciones homosexuales utiliza condón. El
paciente presenta los resultados de una prueba negativa de infección por el virus de
inmunodeficiencia humana que se realizó hace seis meses. ¿Cuál de las siguientes
conductas es la más adecuada en este paciente?

c} Administrar vacuna contra la hepatitis


26. ¿Cuál de los siguientes mecanismos de acción es el que mejor describe el del
oseltamivir?
e}Inhibición dela liberación de partículas virales

27. Paciente masculino de 12 años de edad que es llevado por sus padres a consulta
pediátrica. El paciente se unió hace un mes a un equipo de fútbol,y desde entonces ha
padecido dolor de rodillas. Debido a que se le realizó un examen físico antes de ingresar al
equipo la madre puede comentar con confianza que el hijo ha perdido exactamente 4 kg
desde entonces. Por indicaciones del entrenador del equipo se le tomó al paciente una
radiografía de rodilla, la cual presentan los padres ahora al médico. En la radiografía se
observa una lesión densamente esclerótica en la porción distal del fémur la cual se
extiende desde el disco de crecimiento hacia la diáfisis. Se observa que el periostio se
encuentra elevado, formando una angulación con la corteza ósea. Los tejidos blandos
alrededor de la lesión se observan como "rayos de sol".
¿Cuál de los siguientes diagnósticos es el más probable en este paciente?
b} Osteosarcoma

28. De acuerdo con los criterios CURB-65, ¿cuál de los siguientes pacientes debe ser
ingresado para tratamiento de neumonía adquirida en la comunidad?
c} Paciente masculino de 45 años de edad sin alteraciones del estado de alerta, nitrógeno
ureico en sangre de 25 mg/dl, frecuencia respiratoria de 31respiraciones/min,y presión
arterial de 110/75 mm Hg

29.¿Cuál de los siguientes fármacos se utiliza en eltratamiento del tabaquismo?


a} Bupropiona

30. Paciente del sexo femenino de 40 años de edad que es llevada al servicio de urgencias
por presentar parálisis de la mitad izquierda de la cara. La paciente notó la evidente
simetría al despertar esta mañana. Al interrogatorio la paciente y familiares mencionan
que es una persona aparentemente sana sin ningún antecedente de infecciones virales o
exantemas recientes. A la exploración física se puede corroborar la asimetría facial,con
parálisis del lado izquierdo; es incapaz de cerrar el ojo izquierdo, y cuando trata de hacerlo
el globo ocular desvía la mirada hacia arriba. Al continuar con la exploración física se
corrobora incapacidad para fruncir la frente del lado izquierdo y mostrar los dientes. ¿Cuál
es el manejo más adecuado en esta paciente?"
c} Administración de aciclovir y corticoesteroides

31. ¿Cuál de los siguientes diuréticos es capaz de ocasionar edema pulmonar?


c} Manito!
32. ¿Cuál de los siguientes diagnósticos se presenta con un reporte de biopsia de ganglios
linfáticos que menciona expansión de los folículos linfoides, preservación de la
arquitectura ganglionar con linfocitos atípicos en la zona paracortical?
d} Infección aguda por el virus de Epstein-Barr

33. Paciente femenino de 42 años de edad que se presenta con adenomegalia cervical
única sin fiebre, sudación nocturna y pérdida de peso. Se realiza una biopsia del ganglio
linfático la cual revela presencia de linfocitos pequeños y pocas células de Reed-Sternberg.
Se efectúa una resonancia magnética para etapificar a esta paciente; el estudio de imagen
indica enfermedad limitada a sólo un grupo cervical. ¿Cuál sería la clasificación adecuada
de esta paciente?
a} Linfoma de Hodgkin etapa IA

34. Paciente femenino de 36 meses de edad que se presenta con fiebre de 38 ºC y dolor
en ambos oídos. Los padres mencionan que esto ha ocurrido en varias ocasiones, y que
casi siempre le recetan dos medicamentos, los cuales no recuerdan en este momento. A la
exploración física la paciente se encuentra alerta, con agitación secundaria al
dolor,congestión nasal y tos. Las membranas timpánicas se observan eritematosas, con
movimiento positivo a la insuflación. El timpanograma demuestra una curva "picuda".
¿Cuál es el tratamiento más adecuado en esta paciente?
c} Administración de paracetamol

35. Femenino de 40 años que se encuentra en evaluación por infecciones de las vías
urinarias recurrentes desde hace varios años, junto con hematuria microscópica. A la
exploración física no se encuentran datos de utilidad para este padecimiento. Los estudios
de laboratorio muestran normalidad de los niveles de creatinina y nitrógeno ureico en
sangre. Se solicita un ultrasonido el cual muestra crecimiento irregular de los conductos
papilares de ambos riñones, así como múltiples quistes medulares; el tamaño de ambos
riñones es normal. ¿Cuál de los siguientes diagnósticos es el más probable en este
paciente?
b} Riñón esponjoso medular

36. Paciente masculino de 40 años de edad que se presenta a hospitalización por fiebre
elevada y persistente, así como hemorragia de encías y fatiga. En un frotis de sangre
periférica se observan abundantes mieloblastos con bastones de Auer. El paciente
desarrolla trastornos de la coagulación correspondientes a la coagulación intravascular
diseminada. ¿Cuál de las siguientes alteraciones cromosómicas es la que presentan con
mayor probabilidad las células anormales de este paciente?
c} t(15;17}

37. ¿Cuál de los siguientes trastornos del sueño es el más común en la población general?
d}Insomnio
38. Paciente masculino de 59 años de edad con antecedente de enfermedad acidopéptica
y reflujo gastroesofágico. Debido a que presenta insuficiencia cardiaca y fibrilación se
encuentra bajo tratamiento con digoxina. El paciente decide iniciar por sí mismo
tratamiento con antiácidos de hidróxido de aluminio. ¿Cuál de las siguientes alteraciones
electrolíticas puede ser ocasionada por el antiácido y con alta
morbilidad en este paciente?
d} Hipopotasemia

39. Paciente femenino de 85 años de edad que se presenta con alteración del estado de
alerta, desorientación,combativa,con pupilas midriáticas no reactivas a la luz, taquicardia
sinusal, piel seca, hipertermia, constipación y retención urinaria. Todos estos síntomas
aparecieron después que por error de una estudiante de enfermería se le administró una
dosis excesivamente alta de atropina. ¿Cuál de los siguientes medicamentos es el mejor
antídoto en esta paciente?
a} Fisostigmina

40. Paciente masculino de 24 horas de vida producto de una paciente de 42 años de edad.
A la exploración física el neonato se presenta con microcefalia, implantación baja de
orejas, occipital prominente y micrognatia. ¿Cuál de las siguientes anormalidades en el
cariotipo es la que se puede encontrar con más probabilidad en este paciente?"
b} Trisomía 18

41. Paciente masculino de 65 años se presenta en la madrugada al servicio de urgencias


por presentar disnea en reposo y edema remitente de miembros inferiores pero que va en
ascenso. Cuenta con antecedente de hipertensión y diabetes mellitus tipo 2, así como de
tres infartos miocárdicos, de los cuales dos de ellos no pudieron ser trombolizados porque
el paciente llegó fuera de ventana terapéutica. Hace dos días presentó un cuadro de
infección de vías aéreas superiores, que ha progresado hasta convertirse en una
neumonía incipiente. ¿Cuál de las siguientes combinaciones de fármacos, a pesar de ser
efectiva en este paciente, también puede resultar peligrosa?
a} Digoxina + furosemida

42. ¿Cuál de los siguientes antiarrítmicos tiene su principal mecanismo de acción en


células de despolarización automática y de lenta conducción?
e} Esmolol

43. ¿Cuál de los siguientes medicamentos se asocia con hiperpotasemia como efecto
adverso?
e} Ciclosporina
44. Paciente masculino de dos días de vida el cual presenta acidosis, vómito, hipotonía y
alteraciones neurológicas. Se solicitan estudios de laboratorio los cuales revelan niveles
elevados de lactato y alanina. ¿Cuál de las siguientes enzimas es la que con mayor
probabilidad se encuentra deficiente en este paciente?
b} Piruvato deshidrogenasa

45. ¿Cuál de los siguientes mecanismos de acción corresponde al de las estatinas?


b}Inhibición dela enzima hidroxi-metil-glutaril-coenzima A

46. Sujeto femenino de 60 años que llega a la consulta por episodio de litiasis. Sin
antecedentes heredofamiliares de importancia. Refiere historia de varios episodios de
pielonefritis aguda que han requerido hospitalización. Trae consigo una urografía
excretora,que revela captación del medio de contraste únicamente por el riñón derecho
en fase tardía, captación por el riñón izquierdo normal. Asimismo cuenta con un
renograma que cuantifica la función renal del riñón derecho en 15%, frente a 85% del
izquierdo. ¿Cuál es la actitud terapéutica que se debe tomar?
d} Nefrectomía

47. Una mujer de 18 años de edad se presenta a consulta externa de Obstetricia en su 18


semana de gestación. Hace tres años, la paciente fue diagnosticada con depresión
moderada y se le recetó paroxetina 20 mg/día, la cual continuó tomando hasta hace
aproximadamente seis meses, debido a que consideró que no necesitaba el medicamento.
La paciente informa que recientemente no ha dormido "en semanas", y lo atribuye al
embarazo. Al interrogatorio menciona que también se ha mostrado recientemente
letárgica, sin ideación suicida y que desea alimentar a su futuro hijo con leche de su propio
seno. ¿Cuál de los siguientes esquemas de tratamiento es el más adecuado para el manejo
de depresión de esta paciente?
a} Sertralina 50 mg al día, con consulta psiquiátrica de seguimiento en un mes"

48. ¿Cuál de las siguientes cadenas de globinas se produce exclusivamente durante la


etapa embrionaria?
e} Zeta globina

49. Paciente femenino de 82 años presenta historia de frecuencia urinaria y urgencia con
restos de moderadas cantidades de orina, la cual comienza después de un evento vascular
cerebral tres meses antes. Sin antecedentes familiares de importancia. A la exploración
física nada relevante. Orina residual 35 ml. La biometría hemática y el examen general de
orina son normales. El tratamiento que más probablemente mejore los síntomas de la
paciente es:
a} Oxibutina

50. Paciente femenino de 34 años de edad. Inicia su padecimiento tres días previos a su
ingreso con dolor en región dorsal, progresivo, de tipo punzante, intenso, de predominio
nocturno con irradiación hacia región anterior del tórax, se administran analgésicos y
antiinflamatorios sin mejoría, después se agrega retención urinaria aguda, debilidad
muscular de los miembros inferiores, parestesias de los miembros pélvicos y en región
torácica anterior. Evolucionando con pérdida progresiva de la fuerza en miembros
inferiores y disminución de la sensibilidad hasta la región torácica. Presenta dolor a la
palpación en región dorsal a nivel de T3-T4. Extremidades superiores con fuerza 5/5,
inferior derecha 3/5, inferior izquierda 1/5, reflejos osteotendinosos de +++ en
extremidades inferiores y ++ en extremidades superiores, con respuesta plantar extensora
bilateral. Nivel sensitivo de hipoestesia en T4, incontinencia urinaria y pérdida en el tono
del esfínter anal. Se realizaron exámenes de laboratorio de rutina en rangos normales. Se
programa para tomografía computarizada simple y contrastada de columna dorsal con
foco en T3- T4, sin encontrar lesiones espinales ni lesiones intrarraquídeas o
intramedulares. Se realiza IRM de columna dorsal donde se encuentra una lesión quística
a nivel de T1 con reforzamiento anular y una imagen hiperdensa dentro de la lesión con
edema perilesional compatible con cisticerco intramedular. En relación a la fisiología de la
micción y la incontinencia urinaria todo es característico EXCEPTO:

d} Laincontinencia urinaria aguda tiene un inicio brusco, generalmente en relación con


una enfermedad aguda o problema iatrogénico y no remite tras resolverse la enfermedad
o el problema de medicación.Se cura aproximadamente 10%

51. Paciente masculino de 68 años, con antecedentes de diabetes mellitus tipo 2 de 6 años
de evolución (controlada con metformina 850 mg por la noche) e hipertensión de 2 años
de evolución (en tratamiento con captopril 25 mg por las mañanas), inicia su cuadro hace
3 h con hemiparesia corporal izquierda y afasia. A la exploración física se reporta FC: 67,
FR: 21,TA: 160/75 mm Hg, Temp: 36 ºC."
51a. ¿Cuál es el diagnóstico más probable?
a} EVC isquémico

51b. De acuerdo con el cuadro clínico del paciente, ¿cuál es la arteria que probablemente
se encuentre afectada?
d} Cerebral media

51c. ¿Qué estudio le mandaría hacer primero al paciente en Urgencias?


a} TC

51d. En este caso, ¿cuál es la ventana terapéutica para iniciar trombólisis?


b) 4.5 h

51e. ¿Cuál de las siguientes no es una contraindicación para trombólisis?


d} TA sistólica menor a 90 mm Hg
52. Paciente masculino de 55 años de edad con diagnóstico de bronquitis crónica desde
hace cinco años, acude al Servicio de Urgencias por presentar disnea, tos con
expectoración purulenta y fiebre, a la exploración física se encuentra con frecuencia
cardiaca de 120 lpm, frecuencia respiratoria de 36 rpm, presión arterial de 80/50 mm Hg,
temperatura 39.3 C, somnoliento y con uso de músculos accesorios de la respiración, en
su gasometría arterial con Pa02 39 mm Hg, de acuerdo a la gravedad de la exacerbación
que presenta este paciente, ¿qué microorganismos deben ser cubiertos con el
tratamiento antibiótico empírico?
c} S.pneumoniae, H. influenzae, M. catarrhalis, Klebsiella pneumoniae, E. coli, Proteus y
Pseudomonas aeruginosa

53. Paciente femenino de tres años de edad con cuadro de tres días de evolución con
irritabilidad, malestar general, palidez, evacuaciones diarreicas sanguinolentas y oliguria.
En la exploración se observa palidez, somnolencia y petequias diseminadas. La biometría
hemática muestra leucocitosis con neutrofilia, trombocitopenia y anemia. Existe elevación
considerable de la creatinina sérica. Los tiempos de coagulación son normales, así como la
concentración sérica de fibrinógeno.
De acuerdo con la patología que presenta, señale la opción menos indicada.
b} El diagnóstico más probable es coagulaciónintravascular diseminada

54. Masculino de 65 años presenta dolor, calor y tumefacción en la rodilla derecha.


El hallazgo más útil para establecer el diagnóstico de seudogota en este paciente es:
d} Calcio en elmenisco en una radiografía dela rodilla afectada

55. Niño de tres años de edad sin antecedentes de importancia. Desde hace varias
semanas los padres notaron edema facial que después se generalizó. El niño ha estado
hipoactivo con malestar general. Signos vitales normales. El examen físico confirma la
presencia de edema generalizado. Los estudios de laboratorio revelan albúmina 2.3 g por
decilitro y el examen general de orina muestra proteinuria.
Ante la sospecha de un síndrome nefrótico en este paciente, ¿cuál es la enfermedad que
con mayor frecuencia produce este cuadro clínico en"
niños?
b} Enfermedad de cambios mínimos

56. Paciente femenino de 57 años, con antecedente de hipertensión y osteoporosis en


tratamiento. Es traída a Urgencias en una ambulancia por un choque automovilístico, ella
refiere haberse golpeado el abdomen con el volante del automóvil. Al realizarse estudios
de imagen llama la atención de manera incidental una masa abdominal de más de 7 cm de
diámetro que parece ser originada en el riñón, pero que no lo sobrepasa. Se sospecha
cáncer renal. Al interrogatorio la paciente refiere que en los últimos meses ha presentado
dolor en el flanco derecho y hematuria.
56a. ¿Cuál es el principalfactor de riesgo modificable para presentar cáncer renal?
b} Tabaquismo
56b. ¿Cuál es el subtipo histológico más común de cáncer renal?
a} Células claras

56c. ¿Qué caracteriza al síndrome de von Hippel-Lindau?


c} Cáncer renal,hemangioblastomas en SNC y feocromocitoma

56d. ¿Cuál es el tratamiento ideal para la paciente?


a) Nefrectomía

56e. ¿En qué clasificación del TNM en cuanto al tumor se encuentra la paciente?
c} T2

57. Paciente de 43 años se presenta con parálisis facial bilateral,así como parálisis motora
arrefléxica en ambos miembros inferiores, aunque también presenta leve debilidad en los
miembros superiores. Al interrogatorio el paciente refiere dolor difuso que inicia en cuello
y se extiende hacia el resto de la espalda, así como antecedente de infección
gastrointestinal en la semana previa. ¿Cuál es el diagnóstico más probable?
c} Síndrome de Guillain-Barré

58. Paciente masculino de 29 años de edad que acude al servicio de urgencia por
presentar disnea en reposo. Menciona que necesita utilizar tres o cuatro almohadas al
dormir para evitar la disnea. Recientemente el paciente ha notado que se le hinchan los
pies. Entre los antecedentes el paciente niega el consumo de alcohol, hipertensión o
enfermedad arterial coronaria. Hace una semana tuvo un cuadro de infección de vías
respiratorias. A la exploración física la presión es de 115/76 mm Hg y la frecuencia
cardiaca de 105 latidos/minuto. La presión venosa yugular es de 14 mm Hg. Existen
estertores en ambos campos pulmonares que se encuentran hasta las escápulas. A la
auscultación cardiaca no se encuentran soplos, pero es posible identificar el tercer y
cuarto ruido cardiaco. En las extremidades inferiores tiene edema 2++. El
electrocardiograma no muestra cambios en el segmento ST o en las ondas T. La
radiografía de tórax muestra datos en relación a edema pulmonar y aumento del tamaño
de la silueta cardiaca. Existe elevación leve la fracción MB de la creatinina cinasa. ¿Cuál es
la probabilidad de que este paciente se recupere de este evento?
e} 50%

59. Una mujer de 40 años de edad acude para revisión de su lunar que ha tenido toda la
vida, pero que ha cambiado en los últimos tres meses. Tiene una dermatosis localizada,
unilateral que afecta la espalda. Se caracteriza por una neoformación aplanada, mal
delimitada, de color marrón oscuro, claro y aéreas blanquecinas. Es asintomático. ¿Cuál es
el diagnóstico más probable?
e} Nevo melanocítico displásico
60. Usted recibe para consulta a paciente masculino de 2 años y medio de edad. La madre
refiere fiebre de alto grado de 5 días de evolución, mal estado general e irritabilidad. A la
exploración física resalta la presencia de un exantema maculopapuloso discreto en tronco
e hiperemia conjuntiva! bilateral sin secreción,además de observarse enrojecimiento
bucal con lengua aframbuesada e hiperemia faríngea sin exudados en amígdalas;también
se palpan adenopatías laterocervicales de alrededor de 1.5 cm de diámetro.
60a. Con base en los datos clínicos del paciente, ¿cuál es el diagnóstico probable?
b} Enfermedad de Kawasaki

60b. ¿Cuál es la causa de la enfermedadde Kawasaki?


a} Se desconoce

60e. ¿Cuál es el seguimiento de un paciente con esta enfermedad en caso de no presentar


alteración coronaria?
a} Ecocardiograma ala semana 2 de la realización del diagnóstico, sin ASA después de 6 a 8
semanas de la enfermedad

61. ¿Cuál de las siguientes articulaciones es la más afectada en la enfermedad por


depósito de cristales de fosfato cálcico básico?
a} Hombro

62. Masculino de 12 años acude a consulta externa por presentar fiebre de 8 h de


evolución, malestar general,erupción vesicular generalizada la cual inicia en tronco, se
extiende a cara y miembros pélvicos,acompañada de prurito.A la exploración física se
encuentran lesiones vesiculares en tronco yen cara y extremidades de tipo máculo-
papular.
¿Cuál es el diagnóstico más probable en este paciente?
a} Varicela

63. Masculino de 30 años. Acude a consulta externa por antecedentes: frecuente irritación
conjuntiva!, comezón en el borde palpebral, aparición habitual de orzuelos y
enrojecimiento crónico de la conjuntiva. Exploración física: observa, entre las pestañas y al
tallarlas, descamación en forma de caspa,escasa secreción en los ángulos e hiperemia
tarsal y conjuntiva! bulbar leves.
En este paciente el diagnóstico más probable es:
b} Blefaroconjuntivitis

64. Un paciente diabético de 60 años de edad se presenta a la consulta externa debido a


fiebre de moderada intensidad, secreción nasal y cefalea de una semana de evolución. El
paciente tiene antecedente de descontrol glucémico frecuente. A pesar de tratar con
antihistamínicos comunes, el paciente no ha presentado mejoría, por lo que el médico
decide evacuar los senos maxilares y etmoidales y enviarlos a estudio patológico. Mientras
se espera el reporte de patología, el estado neurológico del paciente empeora y una
tomografía computarizada evidencia una masa en el lóbulo frontal. ¿Cuál es el agente
etiológico que más probablemente reporte el estudio de secreción sinusal?"
e} Mucor y Rhizopus

65. ¿Cuál de los siguientes tipos de catarata es posible que se presente en un paciente de
un mes de edad?
e} Catarata polar anterior

66. Paciente femenino de 71 años de edad se presenta a consulta por pérdida progresiva
de la movilidad del hombro derecho. Menciona que hace algunos años sufrió una caída
sobre su hombro; durante la evaluación médica en dicha ocasión se le dijo que no sufrió
ninguna fractura. La paciente puede flexionar el hombro hasta 90 grados, abducirlo a 80
grados, rotarlo internamente lo suficiente como para tocar la vértebra L5 con su mano
derecha. Las radiografías demuestran migración superior del húmero, esclerosis
subcondral de la porción superior del húmero y la porción inferior del acromion, y
disminución del espacio glenohumeral. ¿Cuál es el diagnóstico más probable en esta
paciente?
b} Artropatía por desgarro del manguito rotador

67. ¿Cuál de los siguientes enunciados es correcto respecto a las hipospadias?


a} Las hipospadias se asocian frecuentemente con curvatura ventral delpene

68. Paciente femenino de ocho años de edad, con esquema de vacunación incompleto,
con historia de cefalea y malestar general hace una semana. A partir de ayer nota
aumento de volumen en región mandibular bilateral, más acentuada del lado derecho. En
la exploración se observa tumefacción mandibular bilateral, mayor en lado derecho,
desplazamiento de ambos lóbulos de las orejas hacia arriba, con dolor a la palpación. No
se encuentran adenomegalias ni alteraciones en mucosa orofaríngea.
68a. ¿Cuál es la causa más probable de esta infección?
d} Paramyxovirus

68b. ¿Cuál es la estructura que se afecta con mayor frecuencia?


c} Glándula parótida

68c. ¿Cuál de las siguientes NO es una complicación de esta infección?


a}Insuficiencia renal

69. Recién nacido de término, con antecedente de distrés respiratorio después del
nacimiento, resultó sin complicaciones. A los 13 días de vida inicia con distensión
abdominal importante, intolerancia a los alimentos con vómito y evacuaciones
hemorrágicas abundantes. Se solicita una radiografía abdominal, en la que se encuentra
neumatosis intestinal.
69a. En relación con el diagnóstico que sospecha, todas las siguientes condiciones se
asocian a esta patología, EXCEPTO una, indique cuál:
e} Fibrosis quística

69b. Se decide iniciar manejo médico. Todas las opciones que se indican son correctas
EXCEPTO una, indique cuál:
b} Alimentación entera!

69c. ¿Cuál de las siguientes situaciones justificaría el manejo quirúrgico de este paciente?
b} Neumoperitoneo

69d. Posterior a manejo quirúrgico el paciente egresa y continúa sin complicaciones los
primeros meses de vida. A los cuatro meses, inicia con distensión abdominal y vómitos
persistentes. De acuerdo con los antecedentes, ¿qué complicación puede esperarse en
este paciente?
a} Estenosis colónica

70. Se presenta paciente gesta 1, para O, aborto O, cesárea O, en trabajo de parto desde
hace 4 horas con aproximadamente tres contracciones cada 1O minutos con duración de
30 segundos. Se puede palpar un cuello casi borrado en su totalidad, con cerca de 4 cm de
dilatación con membranas rotas y presentación cefálica. La paciente acude con pulso
regular y frecuencia de 80 latidos por minuto sin dolor abdominal, ha acudido a control
obstétrico desde el principio y recuerda con precisión la fecha de su última menstruación,
determinándose que se trata de un embarazo de 38 semanas de gestación. El examen
general de orina y la biometría hemática son normales y la amniocentesis demuestra un
índice lecitina/esfingomielina > 2. ¿Cuál es la conducta a seguir?

b} Manejo expectante y espera por dilatación completa

71. Paciente masculino de 86 años, con antecedentes de fibrilación auricular, acude a


urgencias por presentar disnea progresiva de dos meses de evolución, acompañada de
astenia, adinamia y pérdida de peso no cuantificada. A la exploración física se encuentra
afebrily se sospecha derrame pleural derecho. La radiografía de tórax confirma el derrame
pleural y un ensanchamiento mediastínico sin afectación de parénquima pulmonar. Los
análisis de sangre no muestran alteraciones. La toracocentesis mostró un líquido espeso
de aspecto lechoso con 1400 leucocitos/mm3 con 10% de neutrófilos y 90% de linfocitos,
proteínas 49 g/L, LDH 262 U/L, glucosa 10.2 mmol/L, adenosina deaminasa 16
U/L,colesterol 1.75 mmol/L y triglicéridos 12.8 mmol/L. La citología y el cultivo fueron
negativos. La intradermorreacción de Mantoux fue negativa. Latomografía
toracoabdominal mostró múltiples masas adenopáticas en el mediastino y derrame
pleural bilateral de predominio derecho. Se realizó una punción-aspiración con aguja fina
de la masa mediastínica. Los resultados de patología mostraron lesiones granulomatosas
crónicas compatibles con tuberculosis tanto en pleura como en pulmón.
En relación a la tuberculosis miliar, señale la aseveración falsa:

b} Los tubérculos coroideos son frecuentes

72. Paciente masculino de 46 años acude a consulta por haber presentado esputo
hemoptoico que se autolimitó en cuatro días. Dentro de sus antecedentes destaca el
diagnóstico de tuberculosis pulmonar a los 26 años que se trató de manera
correcta,quedando lesión cavitaria residual de pared fina.
En la tuberculosis pulmonar la radiografía de tórax es útil porque:
b} Complementa el diagnóstico de tuberculosis

73. Mujer de 48 años de edad con menorragia secundaria a miomatosis uterina ha iniciado
tratamiento hormonal con mejoría del sangrado; sin embargo, ha persistido con astenia y
cefalea, por lo que se decide realizarle una biometría hemática, de la se obtienen los
siguientes valores: hemoglobina 9.5 g/dL, Htc: 28.5%, VCM: 72 fL, HCM 23 pg, plaquetas:
400 000 x mm3, leucocitos: 5 200 cel/mm3, reticulocitos: 1%, TA: 120/80, FC 11O x min,
FR: 18 x min.
73a. La causa probable de la anemia en la paciente es:
c} Pérdida crónica de hierro

73b. El estudio indicado para continuar con el diagnóstico del paciente es:
c} Ferritina sérica

73c. Al realizar un perfil de hierro, los datos que se esperan encontrar en esta paciente
son:
a} Ferritina baja, transferrina alta, saturación de transferrina baja

73d. El tratamiento indicado para este paciente es:


d} Hierro ferroso junto con alimentos o medicamentos que disminuyan elpH
gastrointestinal,como ácido ascórbico por 3 a 6 meses después de corregir la hemoglobina

74. Paciente masculino de 89 años que acude a urgencias por dolor abdominal. El dolor
inició hace dos días, y ha ido empeorando. Lo refiere en epigastrio, con irradiación a la
espalda y además fiebre. Niega cambios en la orina o las evacuaciones. En sus
antecedentes se reporta enfermedad coronaria habiéndose sometido a dos cirugías de
bypass coronario, insuficiencia cardiaca, hipertensión, diabetes, insuficiencia renal crónica
e hiperlipidemia. Su nivel de actividad actual es muy limitado, pudiendo dar sólo unos
pasos sin sentir disnea. A la exploración física sus signos vitales incluyen FC 112/min, FR
23/min, T 38.7 C y PA 88/62 mm Hg. A la exploración cardiaca se ausculta un S3, y a la
exploración pulmonar estertores leves. A la exploración abdominal hay dolor a la
palpación en epigastrio e hipocondrio derechos, con signo de Murphy positivo. Se realiza
un ultrasonido revelando litos en la vesícula biliar, con la pared engrosada. Al preguntar al
paciente sobre medidas extraordinarias, refiere que desea que se realicen todas las de
resucitación en caso de ser necesario. ¿Qué tratamiento le daría a este paciente?
a} Colecistostomía

75. Paciente femenino de 66 años que es llevada a urgencias por presentar hemiplejía. Su
esposo refiere que inició hace dos horas, sin actividad o síntomas específicos previos. En
sus antecedentes se menciona hipertensión, diabetes y enfermedad coronaria (con un
infarto agudo de miocardio hace 1O años). Cuenta que ha recibido terapia de reemplazo
hormonal en los últimos 13 años posterior a una histerosalpingooforectomía. El IPAS es
difícil porque la paciente es incapaz de responder.A la exploración física sus signos vitales
incluyen FC 102/min, FR 19/min, T 37.2 C y PA 187/89 mm Hg. A la exploración
neurológica se detecta franca hemiparesia del lado derecho. Se observa una región
isquémica en lóbulo anterior izquierdo. ¿Cuál sería una contraindicación para terapia
trombolítica en esta paciente?
d} Presión arterial

76. Se agrega ampicilina al tratamiento empírico de una meningitis cuando se sospecha


que el agente causal puede ser:
b} Listeria monocytogenes

77. El siguiente es un método adecuado para el diagnóstico de la fiebre tifoidea:

b} Mielocultivo enla segunda semana

78. En sesión clínica de hospital, se revisó un caso de hepatitis B en masculino de 35 años,


soltero, quien tenía como factores de riesgo: múltiples parejas sexuales, un tatuaje y
haber recibido tratamiento de acupuntura. Refiere sólo algunos síntomas generales como
náusea, astenia,adinamia. Por laboratorio se haconfirmado la infección.
78a. Sobre esta enfermedad es importante que conozcas la definición operativa de caso
probable de hepatitis virales:
a} Toda persona conictericia y/o elevación sérica delas pruebas de funcionamiento
hepático (transaminasas glutámico oxalacéticas [TGO])

78b. De acuerdo al Sistema Nacional de Vigilancia Epidemiológica, los casos probables de


hepatitis deben ser:
b} Notificados en su totalidad semanalmente en elInforme Semanal de Casos Nuevos de
Enfermedad

79. Paciente femenino de 22 años de edad se diagnosticó con un trastorno hemorrágico


desde la adolescencia. Entre los principales síntomas, la paciente informa hemorragias al
cepillado de los dientes, episodios de menorragia y anemia grave. El mismo cuadro
presentaron la madre y abuela de esta paciente. Los estudios de laboratorio revelan un
conteo plaquetario de 50 OOO/mm3, incremento en el tiempo de sangrado, pero con
normalidad del tiempo de protrombina y el tiempo de tromboplastina parcial. ¿Cuál es el
diagnóstico más probable en esta paciente?
d} Enfermedad de Bernard Soulier
80. Paciente que se presenta con poliuria, disminución de la osmolalidad y gravedad
específica urinaria. A pesar de la administración de vasopresina, la osmolalidad urinaria no
se ve incrementada. ¿Cuál de los siguientes diagnósticos es el más probable en esta
paciente?
b} Diabetes insípida nefrogénica

81. Femeninode 17 años es traída al servicio de urgencias por sus familiares, quienes
refieren que la encontraron excitada y se les informó que fue expuesta a una sustancia
tóxica. Exploración física: taquicardica, hipertensión arterial,taquipnea y midriasis. La
sintomatología de esta paciente es consecuencia de la administración de:
d} Cocaína

82. Paciente bajo tratamiento por diagnóstico de neumonía adquirida en la comunidad.


Recientemente el paciente desarrolló fiebre, exantema, disuria y urgencia urinaria. Los
resultados del examen general de orina revelan gravedad específica de 1.001 con
hematuria, eosinófilos y proteinuria. La biopsia renal demuestra destrucción de las
estructuras tubulointersticiales con infiltración de leucocitos, eosinófilos y linfocitos con
necrosis papilar. ¿Cuál es el agente causal de este cuadro?
e} Administración de antibióticos

83. Se presenta a consulta una paciente de 17 años de edad la cual es llevada por su
madre porque ha notado que se induce el vómito después de cada alimento. Debido a la
condición de la paciente, el médico decide internarla para realizar una evaluación integral.
La paciente se muestra cooperativa y decidida a recibir tratamiento. Los primeros estudios
de laboratorio demuestran potasio sérico de 3.3 mEq/L, un nivel de cloro de 94 mEq/L y
un nivel de nitrógeno ureico en sangre de 22 mg/dL. ¿Cuál de los siguientes
medicamentos se encuentra contraindicado en esta paciente?
e} Bupropión

84. ¿Cuál de los siguientes anticonvulsivantes tiene un alto grado de asociación con el
desarrollo de litiasis de vías urinarias?
e} Topiromato
85. Es llevado al servicio de urgencias lactante menor que se muestra en la fotografía.
La complicación más común en este paciente es:
b) Candidosis

86. Paciente femenino de 70 años de edad que ha permanecido los últimos 25 años de su
vida en su departamento y tan sólo en contadas ocasiones ha salido a la calle. La paciente
tiene una hermana la cual refiere que desde que ambas eran jóvenes la paciente era una
persona muy tranquila y en raras ocasiones se le conocieron amistades. Los vecinos del
edificio le compran comida y artículos de limpieza con frecuencia y los dejan en la puerta
de su departamento, aunque recientemente ha hecho uso de los servicios a domicilio de
internet. ¿Cuál es el diagnóstico más probable en esta paciente?
b} Trastorno esquizoide de la personalidad

87. ¿Cuál de las siguientes neoplasias del sistema nervioso central en adultos es
considerado el tumor cerebral primario más frecuente?
a} Glioblastoma multiforme

88. ¿Cuál de los siguientes hallazgos se asocia con carcinoma de células pequeñas de
pulmón?
b} Síndrome de Eaton-Lambert

89. ¿Cuál de las siguientes enfermedades se debe a una deficiencia de la hipoxantina


guanina fosforribosiltransferasa?
c} Síndrome de Lesch-Nyhan
90. ¿Cuál de las siguientes translocaciones cromosómicas se asocia con el linfoma de
células del manto?
e} t(11:14}

91. Femenino de 24 años, cursa con embarazo de 37 semanas atendida en sala de


urgencias por presentar pérdida del estado de alerta posterior a crisis convulsivas tónico-
clónicas. Antecedentes: G-2,signos vitales con TA 170/120mm Hg, Fe 95x', reflejos
osteotendinosos aumentados, FCF de 132x' y edema importante de miembros inferiores.
La entidad patológica causal de la sintomatología de esta paciente es:
a} Eclampsia

92. Ingresa al servicio de urgencias paciente femenino de 76 años de edad con


antecedente de hipertensión de 23 años de diagnóstico, diabetes de 15 años de
diagnóstico y dislipidemia de reciente diagnóstico. Al momento del ingreso la paciente
presenta un episodio de emesis, síndrome de Horner del lado izquierdo y pérdida de la
sensibilidad facial del lado izquierdo, sin alteración de la sensibilidad del lado derecho del
cuerpo. ¿Cuál de las siguientes arterias es la que más probablemente está ocluida?
a} Arteria cerebelar posteroinferior izquierda

93. Se presenta a consulta pediátrica paciente masculino de cinco años de edad. Éste es
hijo único de padres sanos, producto de un embarazo a término y parto eutócico. La
madre refiere que desde las primeras semanas de haber nacido presentó un exantema
eritematoso en las regiones expuestas al sol, las cuales después se volvieron color oscuro
y se generalizaron, con la aparición posterior de lesiones malignas tumorales. A la
exploración física el paciente muestra un exantema generalizado que respeta axilas y
genitales externos, caracterizada por aumento de la pigmentación, entremezclada con
áreas de hipopigmentación y regiones con atrofia. Se observa también escamas y
telangiectasias generalizadas. Existen múltiples cicatrices en antebrazo, las cuales son
secuelas de extirpación de carcinomas epidermoides.
¿Cuál de los siguientes hallazgos es más probable que se ubiquen en este paciente?
e} Dímero de timina en el DNA

94. ¿Cuál es el mecanismo por el que se produce la diarrea en infecciones intestinales por
Escherichia coli enterotoxigénica?
d}Incremento de las concentraciones intracelulares de AMP cíclico

95. Paciente femenino de 6 años de edad que presenta lesiones dérmicas, máculas y
pápulas, que evolucionan a ampollas en horas. La madre informa que su hija padeció
infección de vías respiratorias superiores hace 2 semanas.
¿Cuál es el diagnóstico en este caso?
c} Varicela
96. Paciente de 65 años de edad que se presenta al servicio de urgencias con fiebre alta,
mareo y disnea. A la exploración física el paciente se presenta taquicárdico, taquipneico y
con insuficiencia mitral. Se realiza un ecocardiograma el cual confirma la presencia de
insuficiencia mitral. Se decide tomar hemocultivos e iniciar tratamiento con vancomicina y
gentamicina mientras se esperan los resultados del cultivo. Dos días después el cultivo
resulta positivo para Streptococcusbovis. ¿Cuál de las siguientes opciones es la más
adecuada en el manejo de este paciente?
c} Solicitar una colonoscopia

97. ¿Cuál de las siguientes alteraciones electrolíticas se asocia con ensanchamiento del
complejo QRS?
d} Hiperpotasemia

98. Paciente masculino de 29 años de edad que sufre una caída con lesión en la pierna
izquierda durante un partido de futbol americano. En el servicio de urgencias se le toma
una radiografía de la pierna izquierda la cual demuestra fractura del cuello del peroné.
¿Cuál de los siguientes hallazgos puede ser el más probable que se presente en este
paciente?
a} Alteración dela sensibilidad en el dorso del pie

99. Paciente femenino de 28 años de edad con neumonía atípica por Mycop/asma
pneumoniae. Al momento más agudo de su enfermedad la paciente presenta
hemoglobina de 1O mg/dL y elevación de los niveles de lactato deshidrogenasa. La
paciente se trata con macrólidos y regresa a consulta tres meses después. Los estudios de
laboratorio de control demuestran ausencia de la anemia o de la elevación de lactato
deshidrogenasa. ¿Cuál es la mejor explicación a este fenómeno?
c} Normalización de la respuesta inmune

1OO. ¿Cuál de los siguientes estudios de laboratorio corresponde a un paciente con


intoxicación por vitamina D?
b} Calcio elevado, fosfato elevado, fosfatasa alcalina normal

101. ¿Qué porcentaje de pacientes tiene síntomas mandibulares (artritis de la articulación


temporomandi bular) en algún momento de la evolución de artritis reumatoide?
c} 55%

102. ¿Cuál de las siguientes condiciones se asocia con elevación de la presión capilar
pulmonar de enclavamiento?
b}Insuficiencia ventricularizquierda con hipertensión pulmonar secundaria
103. Paciente masculino de 80 años de edad que acude a consulta externa llevado por su
hija. El paciente enviudó hace cuatro años, y desde entonces vive con su hija. Refiere que
ha presentado frecuencia y urgencia urinarias, así como nocturia desde la muerte de su
esposa, pero en los últimos meses el problema ha empeorado. El paciente refiere que
presenta dificultad para iniciar la micción. Comunica que la próxima semana acudirá con el
ortopedista porque desde hace varias semanas ha presentado dolor lumbar que no
mejora con reposo. El paciente también refiere pérdida de peso en los últimos cuatro
meses. ¿Cuál es el medicamento más adecuado para el padecimiento principal de este
paciente?
d} Flutamida

104. Mujer de 26 años de edad presenta desde hace 20 días artritis de las articulaciones
del hombro derecho, ambas rodillas y 2ª y 3ª metacarpofalángicas izquierdas, con
incapacidad para realizar sus actividades cotidianas; por 36 horas requirió apoyo por parte
de sus familiares para realizar actividades cotidianas, como aseo personal. Comenta una
rigidez articular matutina de 120 minutos, niega cualquier otra sintomatología. A la
exploración física: se corrobora artritis, en clase funcional 111, sin gran afección sistémica.
Un médico le recetó diclofenaco 100 mg cada 12 h con mejoría parcial. Usted decide
realizar artrocentesis de rodilla derecha y encuentra un líquido sinovial turbio con
viscosidad baja, leucocitos de 5000/µL con predominio de polimorfonucleares, aumento
de proteínas y ausencia de cristales y de bacterias.
Una semana después, la paciente regresa con la misma sintomatología. Le presenta una
BH con leucocitos de 1O 000, trombocitosis 678 000, hemoglobina 11 g, Hto 33%, PCR 10 y
VSG 45 mm3/h.
104a. Su primera sospecha diagnóstica sería:

c} Artritis reumatoide

104b. ¿Qué estudio solicitaría para corroborar su diagnóstico?


d} Factor reumatóide y Anti-CCP (antipéptidos cíclicos citrulinados}

104c. ¿Qué tratamiento iniciaría usted?


d}Iniciar con un esquema corto y a dosis bajas de prednisona más metotrexato semanal e
hidroxicloroquina

104d. ¿Qué vigilaría durante la administración de metotrexato?


a} La medula ósea yla función hepática
104e. El paciente tiene muchas dudas, ha escuchado sobre el riesgo de enfermedades
neoplásicas asociadas a su enfermedad, y le cuestiona a usted sobre ese supuesto riesgo.
Usted:
b} Le explica sobre elriesgo de cáncer hematológico

105. ¿Cuál de los siguientes eventos es el menos probable que se presente durante un
ataque agudo de asma?
b} La presión pleural durante lainspiración se vuelve más negativa

106. ¿Cuál de los siguientes vasos venosos se encarga de conducir sangre de la circulación
porta en el sistema cava a pesar de la obstrucción de la vena porta (p. ej., durante cirrosis
hepática)?
e} Venas ácigos y hemiácigos

107. Los pacientes con cáncer de colon no polipósico hereditario presentan genes con
inestabilidad de microsatélites, lo que significa que contienen múltiples regiones con
fragmentos de DNA de cadena única y de estructura anormal. ¿Cuál de las siguientes es el
tipo de proteína anormal en estos pacientes?
a} Enzimas de reparación de apareamiento incorrecto

108. Paciente masculino de 68 años de edad con diagnóstico de mieloma múltiple desde
hace seis meses, acude al servicio de urgencias por presentar tos con expectoración
purulenta y fiebre, a la exploración física se encuentra con frecuencia respiratoria de 24
rpm, saturando a 88% al aire ambiental, en los campos pulmonares se auscultan
estertores en base derecha, la placa de tórax muestra una zona de infiltrado, este tipo de
pacientes suele infectarse por los siguientes microorganismos:
a} Streptococcus pneumoniae, Staphy/ococcus aureus y Klebsiella pneumoniae

109. Llega paciente masculino de 8 años de edad a Urgencias por dolor escrotal izquierdo
que inició de manera repentina con evolución de 6 h, el cual es muy intenso y ha sido
progresivo. A la exploración física se encuentra paciente ansioso, poco cooperador,con
edema importante en el testículo doloroso y con pérdida del reflejo cremastérico.
109a. Con base en las características del paciente, ¿cuál es su sospecha diagnóstica?
c} Torsión testicular

109b. De acuerdo con su sospecha clínica, ¿qué hallazgos esperaría encontrar durante la
exploración física con transiluminación?
c} Aumento del tamaño deltestículo e hidrocele

109c. En relacióncon su sospecha clínica, ¿qué hallazgos espera encontrara! realizar una
ecografía doppler?"
a} Ausencia o disminución del flujo en el testículo afectado
109d. Si la exploración física y la ecografía concuerdan con los hallazgos esperados de su
sospecha clínica, ¿cuál es el tratamiento indicado?
b} Cirugía inmediata

109e. En conclusión, ¿cuál de las siguientes opciones menciona las características que se
presentan en el padecimiento de este paciente?
a} Edad neonato/puberal,dolor deinicio súbito muyintenso, posición elevada u horizontal
deltestículo, reflejo cremastérico ausente, signo de Prehn negativo

11O. Ante el caso de un recién nacido postérmino, con presencia de líquido amniótico con
meconio, que manifestó apnea y cianosis al nacimiento, ¿qué situación de las siguientes
justificaría el uso de adrenalina para reanimarlo?
d} Frecuencia cardiaca menor a 60latidos por minuto posterior a ventilación con presión
positiva y compresiones torácicas externas

111. Paciente femenino de 40 años de edad con diagnóstico de embarazo de 14 semanas


de gestación. Como antecedentes de importancia tiene los siguientes: 2 embarazos, en los
cuales no hubo complicaciones, los dos fueron partos. Los productos sin alteraciones.
Acude por estar preocupada, ya que sabe que los embarazos a edades mayores se
relacionan con alteraciones en los niños. ¿En qué momentos de la meiosis se realizan las
dos detenciones en la ovogénesis?
b} Diplotena y metafase 11

112. Te encuentras explorando a una paciente embarazada con 33 semanas de gestación,


con fondo uterino de 26 cm desde sínfisis de pubis, situación transversa y decides
auscultar foco fetal. ¿A partir de qué semana de gestación se puede percibir el latido fetal
a través del Pinard?
d} 20 semanas de gestación

113. Paciente masculino de 2 años y medio es llevado a consulta por irritabilidad


persistente. Al interrogara los padres, se conoce presencia de irritabilidad, vómito bilioso y
cólicos de 24 h de evolución, junto con distensión abdominal. A la exploración física se
observa paciente irritable, deshidratación leve, abdomen ligeramente distendido con
molestias a la palpación y ruidos peristálticos de lucha. Signos vitales dentro de los
límites normales.
113a. Con base en las características del paciente, ¿cuál sería su principal sospecha
clínica?
a} Malrotación intestinal

113b. ¿Qué método diagnóstico es el más adecuado para confirmar su sospecha clínica?
d} Serie gastroduodenal
113c. De acuerdo con su sospecha clínica, ¿cuál es la razón por la cual se desarrolló el
padecimiento?
d} Alteración durante el desarrollo fetal

113d. Si su sospecha clínica se confirma, ¿cuál es el tratamiento indicado?"

c} Reparación quirúrgica

113e. De acuerdo con su sospecha clínica, ¿cuál es la complicación que podría presentar
este paciente?
a} Síndrome de intestino corto

114. Paciente masculino de 4 años de edad ingresa al departamento de Urgencias por


presentar accesos de tos y cianosis con periodos de apnea. Los padres mencionan que
previo dicho cuadro el niño se encontraba perfectamente bien. A la exploración física se
encuentra paciente masculino muy ansioso, con paroxismos de tos, cianosis peribucal,
babeo y estridor inspiratorio. Se aspiran secreciones abundantes y se oxigena
adecuadamente.
114a. Con base en las características del paciente, ¿cuál es el diagnóstico más probable?
d} Cuerpo extraño

114b. De acuerdo con su sospecha clínica,¿cuál es la causa de este padecimiento?


c} Cuerpo extraño en vía aérea

114c. En relación con su sospecha clínica, ¿qué hallazgos encontraría en la radiografía?


c} Enfisema obstructivo

114d. Con base en su sospecha clínica, ¿qué tratamiento le indicaría al paciente?


a} Broncoscopia

114e. De acuerdo con su sospecha clínica,¿cuál es la complicación que se puede presentar


en el paciente?
b} Estenosis bronquial o neumonías recidivantes

115. De acuerdo con la clasificación de la Universidad de Texas, una úlcera


superficial,infectada y no isquémica en el pie de un paciente diabético corresponde a:
a} 1B

116. Es el primer fármaco de elección en el tratamiento de la diabetes tipo 2 y obesidad.


d} Metformina
117. Prepúber de 12 años de edad con verrugas plantares.
117a. ¿Cuál es el agente causal de esta infección?
a} Virus delpapiloma humano

117b. ¿Cuál es el serotipo más prevalente en este tipo de verrugas?


a} 1

117c. ¿Qué le explicarías al paciente con respecto a la evolución y respuesta al


tratamiento?
e} Las verrugas pueden llegar ainvolucionar espontáneamente

117d. El paciente decide tratarse ya que le molesta mucho y no le gusta. De los siguientes,
¿cuál es una opción de tratamiento indicada?
b} Crioterapia

118. Paciente masculino de cuatro meses de edad que acude a consulta para aplicación de
vacuna en centro de salud de comunidad rural. Es producto de madre de 25 años, G2, P2,
que cursó con infección de vías urinarias recurrente en el tercer trimestre; fue obtenido
por parto eutócico a las 37 SDG, con apgar desconocido, lloró y respiró al nacer,no estuvo
en incubadora. La madre inició control prenatal hasta el tercer trimestre. El peso al nacer
fue de 2 800 g y talla de 50 cm. A la exploración física se encuentra peso de 3 900 g y talla
de 52 cm, íntegro, con sostén cefálico, no fija mirada, ausencia de reflejo rojo en ambos
ojos, se observa opacidad blanquecina agudeza visual de pupila en ambos ojos. Ruidos
cardiopulmonares normales, exploración abdominal normal,sin alteraciones morfológicas
aparentes. La madre no sabe especificar los medicamentos que tomó durante el embarazo
y no cuenta con inmunización de toxoide tetánico. Se desconocen antecedentes familiares
de importancia en línea materna o patena. ¿Cuál es el diagnóstico más probable en este
paciente?
b} Catarata congénita

119. Mujer de 89 años de edad, sin antecedentes heredofamiliares de importancia, y


como único antecedente personal patológico refiere diagnóstico de catarata hace cinco
años en ojo derecho. Acude a consulta por presentar dolor ocular y enrojecimiento
intensos en el ojo derecho de dos días de evolución. Su agudeza visual advierte
movimientos de mano a 30 cm en el ojo derecho y 20/400 de ojo izquierdo. La exploración
oftalmológica del ojo derecho muestra córnea con pérdida de la transparencia edematosa
y presión de 60 mm Hg, sin posibilidad de observar la cámara anterior. El ojo izquierdo
presenta catarata pardusca, pero aparentemente la cámara anterior en profunda y normal
con presión de 18 mm Hg. La córnea derecha se aclara con glicerina tópica y se observan
partículas que flotan en cámara anterior,con presencia de catarata morganiana. La
gonioscopia revela ángulos cerrados en ojo derecho y abiertos en ojo izquierdo. No es
posible visualizar en fondo de ojo en ninguno de los dos ojos. La paciente niega
antecedente de cuadro uveítico. Al realizar una ecografía modo B de ambos ojos, sólo
revela catarata significativa sin desprendimiento de retina o tumor intraocular. ¿Cuál es el
diagnóstico más probable?
e} Glaucoma facolítico

120. Paciente masculino de 4 años de edad se presenta al departamento de Urgencias por


dolor abdominal. En el interrogatorio se refiere cuadro viral hace 2 semanas, dolor
abdominal de pocas horas de evolución, vómito, oliguria y erupción cutánea extensa. En la
exploración física se encuentra un exantema maculopapular en las nalgas y extremidades
inferiores que no blanquea con la digitopresión. La biometría hemática es normal, pero el
examen general de orina muestra hematuria.
120a. Con base en las características del paciente, ¿cuál es el diagnóstico más probable?
d} Púrpura de Henoch-Schonlein

120b. ¿Qué hallazgos en los exámenes de laboratorio apoyan su sospecha diagnóstica?


b} Biometría hemática normal,sangre oculta en heces

120c. De acuerdo con su sospecha clínica,¿cuál es la complicación que se puede presentar


a largo plazo?
c}Invaginación intestinal

120d. Con base en su sospecha clínica, ¿cuál es el tratamiento indicado?


a} AINE

120e. ¿Qué aspectodel cuadro clínico del paciente es relevante mencionar a los padres?
d} Hematuria microscópica

121. Paciente femenino de 19 años de edad, con embarazo de 12 semanas de gestación


por fecha de última menstruación, acude a urgencias por presentar hemorragia genital
persistente, con dolor en hipogastrio tipo cólico, sin irradiaciones que ha ido aumentando
en intensidad y frecuencia. A la exploración física: frecuencia cardiaca 105 latidos por
minuto, frecuencia respiratoria 20 respiraciones por minuto, temperatura 37 C. Abdomen
blando, depresible, doloroso a la palpación del hipogastrio, sin datos de irritación
peritoneal. Se observa hemorragia genitaly se realiza tacto vaginal encontrando cuello
uterino central,con 4 cm de dilatación y borramiento de 30%. ¿Cuál es su impresión
diagnóstica?
c} Aborto en evolución
122. Paciente femenino de 19 años de edad que fue sometida a evacuación por succión y
curetaje de una mola hidatiforme. Previo a este procedimiento presentaba un nivel de
hormona gonadotropina coriónica humana de 129 000 mUl/mL y un tamaño uterino de 23
SDG. Después de 14 días posteriores al procedimiento se encuentran niveles de hormona
gonadotropina coriónica humana de 25 000 mUl/mL. A la cuarta semana el valor es de 33
000 mUl/mL. ¿Cuál es el siguiente paso más importante en el manejo de esta paciente?
d} Tomografía computarizada de la cabeza, tórax, abdomen y pélvica

123. Paciente masculino de 2 años de edad es llevado a consulta de rutina. Se mencionan


como antecedentes que nació a las 37 semanas de gestación, fue alimentado de manera
exclusiva mediante seno materno durante el primer año de vida y que actualmente no es
bueno comiendo. Su peso y estatura se encuentran en el percentil 5. A la exploración
física se observa que sus piernas están algo arqueadas y el hueso frontal es muy
prominente. No tiene dientes y todavía no camina por sí solo.
123a. Con base en las características del paciente, ¿cuál es el diagnóstico más probable?
b} Deficiencia de vitamina D

123b. En relación con su sospecha clínica,¿cuál de las siguientes opciones representa una
característica particular del padecimiento?
a} Extremos distales ensanchados con forma de copa y con desgaste en la radiografía

123c. Respecto a su sospecha clínica, ¿qué función tiene en el cuerpo la vitamina que se
encuentra con deficiencia en el paciente?"
c} Depósito de calcio

123d. De acuerdo con su sospecha diagnóstica, ¿qué tratamiento I indicaría al paciente?


d} Suplemento de vitamina O

123e. Referente a su sospecha clínica, ¿cuál es el mecanismo de prvención que se debió


realizar para evitar esta patología?
b} Exposición al sol

124. Paciente femenino de 48 años de edad acude a consulta por presentar dolor en la
porcion lateral de la rodilla. Durante los ultimos meses ha notado un nodulo de alrededor
de 1 cm en la region lateral de la rodilla, justo donde refiere el dolor. A la exploracion
fisica el nodulo se puede desplazar y al parecer de consistencia quistica. A pesar de que
otro medico interno manejo conservativo no quirurgico, la sintomatologia no mejora.
¿Cuál es el mejor tratamiento para este paciente?

c)Menisectomia artroscopica y descomprension del quiste

125. ¿Cuál de los sigientes opciones es la causa mas probable de dolor de rodilla en
pacientes que juegan con frecuencia baloncesto?
c)Tenidonosis patelar

126. Una paciente de 35 años sin antecedentes previos de importancia ingresa a la sala de
urgencias. El motivo es por dolor abdominal de aprooximadamente 24 horas de evolucion,
el cual inicio en la region periumbilical pero recientemente ha migrado hacia el
hipocondrio derecho. La paciente refiere nausea y vomito en tres ocasiones. A la
exploracion fisica, se detectan mucosass deshidratadas asi como dolor abdominal
neutrofilos. ¿Cuáles son los nervios que conducen la sensacion dolorosa periumbilical en
esta paciente?
b)Nervio esplacnico menor

1. La siguiente es una enfermedad definitoria de SIDA:


e} Carcinoma cervicalinvasor

2. La forma de Entamoeba histolytica que infecta al ser humano al ser ingerida es:
c} Quistes

3. Paciente masculino de 59 años de edad que se ha sometido a un transplante ortotópico


de corazón hace seis años por miocardiopatía isquémica. Ha tenido tres episodios de
rechazo, los cuales han sido tratados con dosis intravenosas de metilprednisolona.
¿Cuál(es) de las siguientes opciones representa(n) la(s) complicación(es) que puede sufrir
este paciente?
e} Todas las opciones anteriores representan posibles complicaciones en esta paciente

4. La madre de un recién nacido de una semana de vida,con antecedente de parto vaginal


instrumentado, le pregunta sobre una "depresión" que sintió en el cráneo del neonato. Al
explorarlo, usted palpa la fontanela anterior. De acuerdo con su impresión diagnóstica,
¿qué información le indicaría a la madre del neonato?
d} Es normal y desaparecerá entre los nueve y 18 meses de edad cuando se cierren las
suturas craneales

5. Masculino de 66 años acude a consulta externa refiriendo: cefalea aguda posterior a


esfuerzo físico acompañada de vómito, posteriormente se presenta pérdida del estado de
alerta. Antecedentes: diabético, hipertenso sin control médico adecuado.
Con los datos anteriores el diagnóstico que integra es:
c} Hemorragia subaracnoidea

6. ¿Cuál de los siguientes anticonvulsivantes tiene un alto grado de asociación con


aumento de la masa corporal?
e} Ácido valproico
7. Femenino de 35 años, acude al servicio de urgencias por referir fiebre desde hace más
de tres días, dolor en epigastrio así como cansancio leve. Antecedentes: de trasplante
renal hace mes y medio, asimismo trae consigo los siguientes resultados de laboratorio:
leucopenia (2400/mm3), transaminasas (ALT 75 Ul/L;AST 89 Ul/L)
En este paciente el diagnóstico más probable es:
b} Infección por Citomegalovirus

8. Paciente de 38 años de edad que se presenta a consulta ginecológica por haber


detectado una masa en el pecho durante su autoexploración mensual. La exploración
física revela una tumoración en el cuadrante superior externo de la mama izquierda. La
paciente había sido diagnosticada previamente con mastopatía fibroquística y no presenta
antecedentes de cáncer de mama en la familia. Se solicita una mastografía la cual resulta
negativa. Se solicita un ultrasonido cuya conclusión es una masa de carácter sospechoso,
sin proporcionar más información de utilidad. La paciente se somete a una biopsia por
aspiración de aguja fina, cuyo reporte patológico es no concluyente. ¿Cuál de las
siguientes opciones representa el manejo más adecuado en esta paciente?
c} Biopsia excisional

9. ¿Cuál de las siguientes neoplasias del sistema nervioso central en pacientes pediátricos
es la más frecuente en la región de la fosa posterior?
e} Astrocitoma pilocítico

1O. Femenino de 54 años de edad acude al servicio de urgencias por presentar fiebre,
ictericia, acolia y coluria además de dolor tipo cólico en hipocondrio derecho. El
diagnóstico más probable en esta paciente es:
a} Colangitis por obstrucción

11. ¿Cuál de los siguientes hallazgos son característicos del síndrome carcinoide?
a} Rubor facial

12. ¿Cuál de las siguientes enfermedades ocasiona aterosclerosis prematura?


b} Homocistinuria

13. ¿Cuál de las siguientes translocaciones cromosómicas se asocia con la leucemia


promielocítica?
c} t(15:17}

14. Masculino que se presenta a consulta externa, refiere que su pareja ha sido
diagnosticada con infección de virus del papiloma.
La forma más frecuente de la infección por papilomavirus en varones es:
c} Subclínica en genitales
15. Paciente de 22 años de edad con antecedente de asma y pólipos nasales se presenta al
servicio de urgencias con dificultad respiratoria posterior a la ingesta de dos tabletas de
aspirina. ¿Cuál de las siguientes opciones representa el mecanismo a través del cual esta
paciente presentó un ataque agudo?
e} Producción incrementada de leucotrienos

16. Paciente masculino de 22 años de edad con antecedente de feocromocitoma


extirpado hace ocho años, acude a consulta de endocrinología por presentar palpitaciones
en reposo asociadas con sudación de manos en paroxismos, documentando hipertensión
arterial. En esta ocasión el paciente refiere parestesias y disestesias en tercero, cuarto y
quinto dedos de la mano izquierda, con disminución de la sensibilidad, acompañado de
inestabilidad de la marcha. Se realizan estudios de laboratorio e imagen, los cuales
reportan niveles elevados de metanefrinas séricas y urinarias, así como imágenes
compatibles con hemangioblastoma cerebelar y una segunda lesión en médula espinal,
por la cual fue sometido a intervención quirúrgica sin complicaciones. El paciente también
presenta disminución de la agudeza visual secundaria a la presencia de un angioma
retiniano. ¿Cuál de los siguientes hallazgos es más probable que se
observen en este paciente?
d} Desprendimiento de retina

17. ¿Cuál es la principal función del toxoide diftérico en la vacuna contra la infección por
Haemophí/us influenzae tipo b?
d}Incrementar la inmunogenicidad de la vacuna

18. Masculino de 73 años llevado a servicio de urgencias. Antecedente: acude a una cena
familiar de celebración y pasa una gran noche. Al dejar el restaurante sufre un colapso. La
entidad causal de la sintomatología de este paciente es:
d} Síncope posprandial

19. ¿Cuál de las siguientes alteraciones electrolíticas se asocia con una prolongación del
potencial de reposo de los cardiomiocitos e inactivación de los canales de sodio?
d} Hiperpotasemia

20. Paciente femenino de 25 años de edad la cual acude a consulta por padecer
dispareunia, fiebre,1 dolor abdominal en hipogastrio principalmente y escasa secreción
vaginal maloliente y sa nguinolenta. Entre los antecedentes de importancia la paciente
menciona que es sexualmente activa desde los 14 años de edad, con múltiples parejas
sexuales, y no siempre utiliza protección contra enfermedades de transmisión sexual. A la
exploración física se presenta dolor a la movilización cervical. Se sospecha enfermedad
pélvica inflamatoria. ¿Cuál de los siguientes organismos se relaciona con esta
enfermedad?
a} Neisseria gonorrhoeae
21. ¿Cuál de los siguientes estudios de laboratorio corresponde a un paciente con
osteoporosis?
a} Calcio normal,fosfato normal,fosfatasa alcalina normal

22. ¿Qué patrón de presentación es el más común en la artritis reumatoide?


d}Insidioso

23. ¿Cuál o cuáles de las siguientes aseveraciones en relación con la miocardiopatía


hipertrófica son las más correctas?
a} Hipertrofia asimétrica, de mayorintensidad enla región septal

24. Paciente masculino de 20 años de edad que hace cuatro semanas tuvo una relación
sexual homosexual sin protección. El día anterior el paciente recibió la noticia de que
dicho compañero sexual era portador del virus de la inmunodeficiencia humana. ¿Cuál de
las siguientes opciones representa más correctamente los resultados de laboratorio en
caso de que este individuo se encuentre infectado en este momento?
a} Disminución en el número de linfocitos T CD4+ e incremento en los niveles del antígeno
p24

25. ¿Cuál es el índice radiológico más utilizado en estudios clínicos, para valorar el daño
articular en pacientes con artritis reumatoide?
e} b, c y d son correctas

26. Paciente masculino de 22 años de edad se presenta a la consulta externa. Desde hace
cuatro años se le diagnósticó asma; en esta ocasión acude debido a que al parecer la
presencia de sibilancias y disnea se ha hecho cada vez más frecuente. El paciente informa
que presenta sintomatología alrededor de tres veces por semana y que durante el último
mes se despertó tres noches debido a ataques de disnea. Refiere que los síntomas
mejoran cada vez que inhala el agonista beta adrenérgico. ¿Cuál de las siguientes
aseveraciones describe la mejor conducta terapéutica en este paciente?
c} Iniciar tratamiento con dosis bajas de corticosteroides inhalados

27. ¿Cuál de los siguientes tipos de folículos se caracteriza por localización estroma! y la
presencia de un antro?
b} Folículo secundario

28. ¿Cuál de las siguientes estructuras podría resultar dañada en caso de una lesión
penetrante por arma punzocortante en el quinto espacio intercostal al nivel de la línea
media axilar del lado derecho?
a} Hígado
29. ¿Cuál de las siguientes afirmaciones NO es correcta respecto a la intoxicación con
monóxido de carbono?
c} El uso de oxígeno hiperbárico se recomienda cuando los niveles de
carboxihemoglobina se encuentran por arriba de 15 %

30. Paciente de cinco años de edad que acude a consulta pediátrica por presentar desde
hace una semana una tumoración cervical. La madre refiere que la aparición de esta
tumoración coincide con una infección respiratoria de vías aéreas superiores. A la
exploración física se encuentra una tumoración en la línea media del cuello de
aproximadamente 2.5 cm por encima del cartílago tiroideo. Cuando se le solicita al
paciente que protruya la le ngua, la masa se eleva. ¿Cuál es el diagnóstico más probable
en este paciente?
c} Quiste del conducto tirogloso

31. Se trata de paciente femenino pretérmino que al nacer manifestó cianosis y frecuencia
cardiaca de 45 latidos por minuto. Se inició reanimación neonatal procediéndose a
ventilación con presión positiva y se reevaluó después de 30 segundos encontrándose con
frecuencia cardiaca de 59 latidos por minuto y coloración rosada generalizada. Señale cuál
de los siguientes es el siguiente paso a seguir de acuerdo al manejo durante la
reanimación neonatal:
e} Dar compresiones torácicas externas

32. Llega una mujer de 31 años a Urgencias de Ginecoobstetricia con embarazo de 37


semanas de gestación, gesta 3 cesárea 2, el lapso entre los tres embarazos fue de 10 a 12
meses. Acude porque hace 2 horas inició con contracciones, tres cada 1O minutos, con
duración de 50 segundos, pero que súbitamente presentó dolor suprapúbico intenso.
Refiere salida de tapón mucoso y después hemorragia transvaginal leve. A la exploración
física se encuentra FC 98 lpm, FR 19 rpm, Temperatura 37 ºC, PA 125/80 mm Hg. Se
observa angustiada, pálida y diaforética. Cardiopulmonar sin compromiso. Abdomen con
útero gestante de 33 cm de longitud desde sínfisis de pubis, situación
longitudinal,presentación pélvica, posición izquierda, se palpan con facilidad las partes
fetales, frecuencia cardiaca fetal de 130 latidos por minuto y no se percibe actividad
uterina. ¿Cuál es el factor más importante
para desarrollar la patología que tiene esta paciente?
a} Cicatriz uterina previa

33. Paciente femenino de 18 años de edad acude a consulta acompañada de su madre.


Ésta refiere que desde hace dos semanas su hija ha presentado náusea y en dos ocasiones
ha llegado al vómito de contenido alimentario. Al interrogatorio sin la madre la paciente
refiere astenia, adinamia, disuria, polaquiuria, urgencia miccional, con retraso en la
menstruación de 4 semanas. Refiere actividad sexual. El abdomen se encuentra blando,
depresible, no doloroso y a la exploración vaginal se observa la mucosa vaginal de color
rojo violáceo. ¿Cuál de los siguientes NO es un dato de probabilidad de embarazo?"
b} Red venosa de Haller
34. Acude a consulta paciente femenino de 37 semanas de gestación, gesta 3 partos 2, no
presenta datos de vasoespasmo y el embarazo ha sido normoevolutivo. A la exploración
física: frecuencia cardiaca, respiratoria, temperatura y presión arterial sin alteraciones.
Para conocer la situación fetal,¿que maniobra de Leopold realizaría?
b} Segunda maniobra de Leopold

35. Llega a consulta paciente primigesta de 24 años de edad con 34 semanas de


embarazo, por iniciar contracciones hace 2 horas. No refiere salida de tapón mucoso pero
sí sangrado transvaginal de moderada cantidad. Sin datos de vasoespasmo. Fondo uterino
de 29 cm,frecuencia cardiaca fetal de 150 latidos por minuto, contracciones cada 5
minutos con una duración de 40 segundos. Cuello uterino dehiscente, central, firme,
borramiento de 20%. Usted piensa que la administración de un supositorio de
indometacina no es adecuada en el paciente. ¿Cuál de los siguientes es un efecto adverso
de la indometacina?
c} Hemorragia intraventricular,cierre del conducto arterioso y enterocolitis necrosante

36. Neonato en su segunda semana de vida extrauterina, producto de primera gestación


de 30 semanas de edad gestacional, nacimiento por parto vaginal eutócico, con rotura
prematura de membranas (de 24 horas), que pesó 2 200 gramos. Entre sus
comorbilidades presenta onfalitis desde la primera semana de vida. Actualmente presenta
frecuencia cardiaca de 190 latidos por minuto, con frecuencia respiratoria de 55
respiraciones por minuto y temperatura de 39 ºC, con 42 000 leucocitos en la biometría
hemática.
¿Cuál de los siguientes agentes etiológicos es la causa más frecuente de sepsis neonatal de
inicio tardío?
d} Staphylococcus epidermidis

37. ¿Cuál es el valor del sodio corregido en un paciente que llega al servicio de urgencias
con Na en 120 y una glucosa en 853?
c} 132

38. ¿Cuál de los siguientes marcadores tumorales es el más adecuado para detectar
enfermedad recurrente posterior al tratamiento del cáncer testicular no seminomatoso?
d} Alfa fetoproteína

39. ¿Cuál de las siguientes opciones representa correctamente el uso de la hemoglobina


glucosilada?
a} Monitoreo y eficacia delapego al tratamiento de diabetes
40. Varón de 50 años de edad trasplantado renal,que presenta múltiples verrugas
vulgares. Una de ellas, en la cara extensora del antebrazo, se observa más queratósica,
eritematosa, ulcerada y sangrante. Acude con usted porque recuerda que le dijeron que
estas lesiones podrían malignizarse. ¿Cuál es el cáncer de piel que se relaciona más a"
menudo con infección por VPH en la piel?
a} Carcinoma epidermoide

41. Paciente femenino de 30 años de edad, sin antecedentes de importancia,acude al


servicio de urgencias por presentar fatiga, disnea de medianos esfuerzos y dolor lumbar. A
la exploración física se encuentra ictérica y con frecuencia cardiaca de 100 lpm, en sus
resultados de laboratorio destaca hemoglobina 8 mg/dl,VCM 90 fl,HCM 30 pg, leucocitos
5.0 1 OOO/mm3 y plaquetas 400 1 OOO/mm3, creatinina 0.6 mg/dl, bilirrubina total 5
mg/dl, bilirrubina indirecta de 4 mg/dl.
41a. El diagnóstico más probable es:
d} Anemia hemolítica

41b. Con el fin de confirmar el diagnóstico de sospecha, se debe solicitar:

d} Medición de reticulocitos, lactato deshidrogenasa, haptoglobina y prueba de Coombs"

41c. Le reportan hierro sérico 100 µg/dl (normal), ferritina 150 µg/dl (normal), transferrina
300 µg/dl (normal), saturación de transferrina 45% (normal), ácido fólico 1O ng/ml
(normal), vitamina 812 1 100 pg/ml (normal), reticulocitos 3%, lactato deshidrogenasa 250
mg/dl (elevada), haptoglobina 30 mg/dl (normal), Coombs positivo para lgG y negativo
para complemento, en el frotis de sangre periférica se observan microesferocitos; con
base en estos hallazgos, ¿Cuál es el tratamiento de elección?"
r a} Esplenectomía
b} Corticoesteroides

42. Es un análogo de insulina con pH ácido:


e} Glargina

43. Se trata de paciente femenino de 54 años de edad, enviada a revisión oftalmológica de


fondo de ojo por su endocrinólogo ya que es diabética de cinco años de evolución,
actualmente con adecuado control metabólico mediante el uso de hipoglucemiantes
orales. Niega sintomatología oftalmológica. A la exploración se observa agudeza visual de
201200 en ambos ojos que mejora a 20120 con refracción de +2.50 Dp en ambos ojos. A la
biomicroscopia se encuentra córnea transparente, cámara anterior poco profunda, pupila
fotorreactiva, sin rubeosis y cristalino transparente. Presión intraocular de 16 mm Hg y
ángulos cerrados en ambos ojos. El residente de primer año decide dilatarla para realizar
la fundoscopia empleando tropicamida con fenilefrina cada 1O minutos por tres dosis. A
los 30 minutos la paciente inicia con dolor ocular bilateral intenso, disminución brusca de
la agudeza visual,ojo rojo, además de náusa y vómito. Se toma nuevamente la presión
encontrándose de 34 en AO. Se realizan iridotomías y la presión se ubica en 17 mm Hg
empleando posterior al procedimiento acetato de prednisolona a 1% en dosis reductivas,
timolol cada 12 horas y brimonidina cada 8 h. ¿Cuál es el diagnóstico más probable?"
a} Cierre agudo del ángulo

44. Paciente de 20 años de edad que durante un partido de fútbol sufre una lesión en la
rodilla izquierda por una mala caída. Se queja que no puede extender por completo la
rodilla. A la exploración física existe derrame sinovial mínimo y dolor en la región medial
de la rodilla lesionada. No hay evidencia de laxitud ya sea con aplicación de presión
anteroposterior, lateral o medial. Se solicita una resonancia magnética la cual revela un
desgarro del menisco medial. El paciente es la estrella del equipo de fútbol, el cual tiene
altas probabilidades de convertirse en el campeón nacional. Sólo tres partidos separan al
equipo del campeonato. ¿Cuál de las siguientes opciones representa de mejor forma el
tratamiento de este paciente?

b} Reparación artroscópica delmenisco

45. Paciente masculino de 29 años de edad con dolor bilateral en las rodillas. Al realizar la
historia clínica menciona que es jugador semiprofesional de baloncesto. ¿Cuál es la causa
más probable del dolor de rodillas en este paciente?
e} Tendinosis patelar

46. Paciente masculino de 31 años de edad con antecedente de dos semanas de fiebre, a
lo cual se agregó hace una semana distensión abdominal,estreñimiento y dolor
epigástrico. Entre otras molestias también refiere cefalea, escalofríos (calosfríos), mialgias
y malestar general. La exploración física revela la presencia de un "exantema rosado"
(exantema maculopapular color salmón que blanquea a la presión), principalmente en el
tórax, así como dolor abdominal generalizado a la palpación y esplenomegalia. Presenta
temperatura de 38.9 ºC, frecuencia cardiaca de 45 latidos por minuto, frecuencia
respiratoria de 20 respiraciones por minuto, presión arterial de 100/65 mm Hg. Los
estudios de laboratorio básicos son relevantes por leucopenia y neutropenia. ¿Cuál es la
prueba diagnóstica con mayor sensibilidad a pesar de tratamiento antibiótico?
c} Cultivo de médula ósea

47. El tratamiento inicial para cáncer diferenciado de tiroides (papilar,folicular) es:


c} Quirúrgico

48. Se presenta paciente masculino de 78 años a la sala de urgencias por dolor abdominal
punzante de inicio súbito y de 2 horas de evolución acompañado de vómito de contenido
gástrico. El paciente localiza el dolor en la abdominal, lumbar con irradiación hacia la ingle.
Refiere que en la mañana de hoy el dolor fue tan intenso que le ocasionó síncope. Entre
los antecedentes de importancia el paciente es fumador (20 cigarros/día) desde los 20
años de edad, enfermedad pulmonar obstructiva crónica de cinco años de diagnóstico,
hipertensión arterial sistémica de 1O años de diagnóstico y angina estable desde hace
ocho meses. Se registran frecuencia cardiaca de 140 latidos/min, presión arterial de 90/50
mm Hg, frecuencia respiratoria de 32 respiraciones/min y normotermia. A la exploración
física es posible palpar una masa abdominal pulsátil por lo que se sospecha rotura
aneurismática. Se solicita ultrasonido abdominal el cual reporta líquido en la cavidad
peritoneal y aumento del diámetro de la aorta abdominal. El paciente se somete a cirugía
de reparación aneurismática. Posterior a la cirugía el paciente presenta sintomatología
atribuida a la oclusión temporal de las ramas que dan origen a la arteria espinal anterior.
¿Qué sintomatología neurológica presentará este paciente?

d} Paresia espástica con hiperreflexia y pérdida de la sensación dolorosa y térmica en


ambas extremidades inferiores; conservación de la sensibilidad vibratoria y propiocepción

49. Un paciente de 70 años posterior a un infarto cerebral presenta dificultad para


entender el lenguaje hablado, así como incapacidad para reconocer objetos colocados en
su lado derecho. ¿Cuál de las siguientes arterias es la que más probablemente sufrió
trombosis en este paciente?
d} Arteria cerebral mediaizquierda en su división inferior

50. Un paciente de 40 años de edad acude a consulta por presentar dolor agudo en la
región inguinal posterior a un día que levantó múltiples costales de cemento. El paciente
es diagnosticado con una hernia de disco. ¿Cuál de los siguientes nervios espinales es más
probable que cause el dolor en este paciente?
a} L1

51.Acude a la sala de urgencias un paciente del sexo femenino de tres semanas de vida
por presentar vómito durante las últimas 24 horas así como distensión abdominal. A la
exploración física, se confirma la distensión abdominal, sin palpar ninguna estructura
sobresaliente. Los estudios de electrólitos séricos demuestran una concentración de sodio
de 131 mEq/L, potasio 2.9 mEq/L,cloro 85 mEq/L, dióxido de carbono 38 mmol/L. ¿Cuál es
el diagnóstico más probable de esta paciente?"
a} Rotación del intestino delgado alrededor de la arteria mesentérica superior

52. Masculino de 40 años con antecedente de diabetes mellitus tipo 2 e insuficiencia renal
terminal con tratamiento sustitutivo con hemodiálisis, se presenta al Servicio de Urgencias
por debilidad. Menciona que no acudió a su última sesión de diálisis, y que desde hace dos
días no se administra insulina debido a que está esperando recibir su salario. Al
interrogatorio, el único síntoma es la debilidad. A la exploración física no se encuentran
anomalías relevantes. Los estudios de laboratorio revelan una concentración de sodio de
125 mEq/L, potasio de 5.2 mEq/L, cloro de 104 mEq/L, bicarbonato de 18 mEq/L. La
glucosa sérica es de 700 mg/dL, y la osmolalidad sérica de 31O mOsm/kg. Una radiografía
de tórax revela campos pulmonares normales, y el electrocardiograma es normal. ¿Cuál
de las siguientes opciones es la más adecuada respecto al siguiente paso terapéutico a
seguir en este paciente?
b} Administración de insulina intravenosa
53. Paciente femenino de 64 años refiere historia de tos y fatiga de cinco semanas.
Reportó que su tos era en un principio seca, pero se volvió productiva en las últimas dos
semanas. Refiere escalofríos, sudores nocturnos y pérdida de peso de 13 kg en los últimos
dos meses. Ha viajado por Latinoamérica, tiene antecedentes de consumo de alcohol y
diabetes tipo 2 para lo que toma glibenclamida. A la exploración física se auscultan
disminuidos los ruidos respiratorios en la zona superior y media de los campos
pulmonares. No se encuentra hepatomegalia ni esplenomegalia. La radiografía de tórax
mostró infiltrado fibronodular en lóbulo superior derecho con una cavitación pequeña. Se
tomaron muestras de esputo y se encontró bacilo ácido-alcohol resistente. Le dieron
diagnóstico de tuberculosis pulmonar; mientras los cultivos están pendientes, se inició
tratamiento con isoniazida, rifampicina, pirazinamida y etambutol. De manera simultánea
desarrolló parestesias en ambos miembros inferiores, comenzando por los pies, y terminó
con un intenso dolor.
¿Cuál es la causa más probable de la neuropatía?
a} lsoniazida

54. Masculino de tres años y medio es referido al servicio para valoración por masa y
distención abdominal.A la exploración física lo encuentra con buen estado general y
hemihipertrofia corporal;se corrobora una masa abdominal quese envía para estudio
urográfico con contraste, el cual muestra una masa voluminosa sin calcificación en el riñón
izquierdo que distorsiona el sistema pielocalicial,la ecografía abdominal determina el
carácter sólido de la masa,así como la existencia de trombosis en la vena renal.
54a. Con base en las características del paciente, ¿cuál es el diagnóstico más probable?
b} Tumor de Wilms

54b. De acuerdo con su sospecha clínica, ¿qué características histológicas esperaría


observar si solicita una biopsia?
a} Estroma, blastema y epitelio sinectopia nianaplasia

54c. En caso de confirmarse su sospecha diagnóstica, ¿cuáles son los sitios más comunes
de enfermedad metastásica?
c} Pulmón e hígado

54e. Con base en las características del tumor reportado en los estudios de imagen y en
una biopsia con histología favorable, ¿cuál es el tratamiento indicado?
a} Quimioterapia y cirugía

55. Mujer de 60 años, ama de casa, obesa, sin otros antecedentes de importancia,desde
hace dos años presenta dolor en las muñecas y rodillas de forma ocasional, que se
exacerba con la actividad física, y disminuye con la ingesta de analgésicos como el
paracetamol. Ha tenido rigidez articular matutina de 30 minutos o menos. Así mismo
desde hace tres meses, después de una caída de su misma altura, presentó de forma
súbita artritis de rodilla derecha a lo que se agrego en un periodo de dos semanas artritis
de la muñeca del mismo lado, que remitieron parcialmente con la ingesta de diclofenaco.
En esta ocasión, el motivo de la consulta es nuevamente artritis de la rodilla derecha
desde hace 24 horas, a pesar de tomar tratamiento antiinflamatorio. A la exploración
física se encuentra con dolor en muñeca derecha, artritis de rodilla derecha. Sus exámenes
de laboratorio muestran una BH con leucocitosis de 13 000 y predominio de neutrófilos,
VSG de 40 y ácido úrico de 5 mg/dl. En la radiografía de rodilla derecha se observan
calcificaciones de aspecto lineal que siguen el contorno del cartílago.
55a. ¿Cuál es el diagnóstico más probable?
e} Artropatía por cristales de pirofosfato cálcico

55b. ¿Cual es la conducta inmediata a seguir?


e} Punción de rodillainflamada para búsqueda de cristales einfiltración intra articular de
glucocorticoides

55c. ¿Qué otros exámenes de laboratorio solicitaría para completar el protocolo de


estudio?
a) B y C son correctas

55d. ¿Qué estudios de gabinete se pueden utilizar como parte del protocolo de estudio de
este paciente?
e} Todaslas anteriores

56. Paciente masculino de 27 años acude a urgencias por presentar dolor y deformidad del
antebrazo derecho posterior a sufrir caída sobre su mano derecha extendida. A la
exploración física es notoria la imposibilidad del paciente de hacer una pinza aproximando
el pulgar y el índice. La radiografía muestra fractura transversa distal del radio con
desplazamiento de la articulación radioulnar distal. Usted le informa al paciente que esta
fractura requiere tratamiento quirúrgico en todos los casos.
¿Cuál es el diagnóstico?
c} Fractura de Galeazzi

57. Paciente femenino de 42 años de edad, fumadora, que acude por embarazo de 14 SDG
porque "escuchó en la televisión que embarazos de mujeres grandes tienen riesgos. Sin
antecedentes heredofamiliares de importancia. G2 PO AO C1. El primer embarazo fue
normoevolutivo sin alteraciones y la cesárea se realizó por sufrimiento fetal. Este es un
embarazo no planeado. No toma fumarato ferroso ni ácido fálico, aunque refiere que hace
seis meses tomaba vitamina A "para mejorar su vista". Ya recibió su primer dosis de
toxoide tetánico y se le realizó un examen general de orina que no reporta alteraciones.
Usted decide practicarle un USG.
57a. ¿Qué datos ecográficos no harían sospechar de una cromosomopatía?
a} Gastrosquisis
57b. ¿Qué datos de laboratorio no nos harían pensar en un síndrome de Down?
e} iAFP

57c. Sobre métodos de diagnóstico, señale la relación incorrecta.


c} Biopsia corial:a las 6 SDG

58. Paciente masculino de 6 años de edad, sin antecedentes médicos de importancia,


inició su padecimiento 24 h antes de su ingreso. Comenzó con tos metálica,sin otros
síntomas; 12 h después ésta se intensificó y apareció dificultad respiratoria. Fue entonces
que se le llevó a un centro médico en donde se le inició tratamiento con oxígeno y
micronebulizaciones de adrenalina,sin que haya habido mejoraalguna. Posteriormente
apareció estridor inspiratorio y sialorrea. Al realizar revisión de la vía aérea se observa
epiglotis enrojecida,oscura y edematosa.
58a. Con base en las características del paciente, ¿cuál es el diagnóstico más probable?
b} Epiglotitis

58b. De acuerdo con su sospecha clínica,¿cuál es la causa de este padecimiento?


a} Haemophilus influenzae

58c. En relación con su sospecha clínica, ¿qué hallazgos encontraría en la radiografía?

d} Signo del pulgar

58d. De acuerdo con su sospecha clínica, ¿qué tratamiento le indicaría al paciente?


c}Intubación endotraqueal y antibióticos

58e. Con base en su sospecha clínica,¿cuál es la complicación que se puede presentar en


el paciente?
c} Neumonía, linfadenitis cervical o meningitis

59. Paciente masculino de 56 años es traído con antecedente de haber ingerido cinco
botellas de vino el día anterior, acude a urgencias por presentar dolor epigástrico
constante de intensidad 10/1O que se irradia hacia la espalda como si "lo atravesara". El
dolor inició de manera súbita hace 8 horas seguido de náusea y vómito. A la exploración
física se observa al paciente recostado en posición de gatillo, deshidratado, se encuentra
hipotenso y taquicárdico. Presenta dolor a la palpación del epigastrio y ausencia de ruidos
abdominales. Los estudios de laboratorio muestran hematocrito elevado, leucocitosis
moderada, amilasa sérica 300 U/L, lipasa sérica 1 000 U/L con una relación amilasa/lipasa
> 3. Usted realiza el diagnóstico de pancreatitis aguda de etiología alcohólica e inicia
tratamiento médico. ¿Cuál de las siguientes condiciones está asociada con un mal
pronóstico, en los controles de 48 horas?
d} Elevación del BUN mayor a 5 mg/100 ml
60. Paciente femenino de 12 años es traída a la sala de urgencias de un hospital en Baja
California por su madre una hora después de haber sido mordida por una viuda negra. La
paciente se encuentra ansiosa, refiere dolor intenso en la pierna derecha, sitio en donde
fue mordida por la araña y parestesias en todo el miembro pélvico derecho. Sus signos
vitales se encuentran dentro de parámetros normales. ¿De qué manera actúa el veneno
de la viuda negra?
a} Liberación de adrenalina y acetilcolina en la sinapsis

61. Se presenta a consulta un paciente masculino de 8 años de edad por dolor en la pierna
derecha. En el interrogatorio semenciona dolor progresivo de 4 meses de evolución, sin
referir ningún antecedente traumático. El dolor es intenso y regularmente lo despierta por
la noche.En la exploración física no hay datos relevantes; la palpación y arcos de
movimientos no producen dolor, pero a la marcha se muestra una cojera casi
imperceptible.
61a. Con base en las características del paciente, ¿cuál es el diagnóstico más probable?
b} Osteoma osteoide

61b. De acuerdo con su sospecha clínica,¿qué hallazgos esperaría encontraren una


radiografía?
d} Zona radiolúcida central

61c. En relación con su sospecha clínica, ¿cuál es la fisiopatología de este padecimiento?


c} Proliferación de osteoblastos inmaduros

61d. En relación con su sospecha clínica, ¿cuál es el tratamiento que recomendaría?


a} Resección en bloque o ablación percutánea

61e. Con base en su sospecha clínica, ¿cuál es la complicación más importante que se
puede presentar?
b} Recurrencia

63. ¿Cuál de los siguientes procesos produce en la radiografía destrucción ósea,tejidos


blandos en forma de ""rayo de sol"", y elevación del periostio?
b} Osteosarcoma

64. Paciente femenino de 49 años de edad acude a consulta por sentirse fatigada. Niega la
existencia de fiebre, vómito o diarrea. A la exploración física se encuentra presión arterial
de 88/55 mm Hg y es evidente la deshidratación intensa de mucosas. Los estudios de
laboratorio muestran una concentración sérica de sodio de 129 mEq/L y un nivel de
potasio de 5.5 mEq/L. ¿Cuál de los siguientes diagnósticos es el más probable en esta
paciente?
e} Enfermedad de Addison
65. ¿Cuál de los siguientes fármacos antihipertensivos produce hipotensión ortostática en
su primera dosis?
e} Doxazosin

66. ¿Cuál de los siguientes síndromes se ha asociado con el carcinoma de células renales?
a} Miopatía con o sin neuropatía

67. Paciente masculino de 66 años de edad con cuadros de angina de pecho autolimitados
que ocurren típicamente bajo estrés emocional o incremento en la actividad física. Cada
vez que se presenta uno de estos episodios el paciente ingiere una tableta sublingual de
nitroglicerina. ¿Cuál de los siguientes mecanismos de acción es el que mejor describe el de
la nitroglicerina?
c} Disminución delvolumen ventricular izquierdo

68. ¿Cuál de los siguientes tipos de pericarditis es el que más frecuentemente se asocia a
procesos inmunológicos?
c} Pericarditis serosa

69. Acude una mujer de 56 años de edad al Servicio de Urgencias, con desarrollo de
cianosis de las extremidades y las mucosas, se refiere sin antecedentes personales
patológicos, y ha estado tomando fenazopiridina para una infección urinaria. A la
exploración física se encuentra con FC 88x', FR 20x', PA 130/70 mm Hg, Tem 35.5ºC y Sp02
de 98%. ¿Cuál de los siguientes diagnósticos es el más probable que sea el causal de esta
condición?
c} Metahemoglobinemia

70. ¿Cuál de los siguientes fármacos es el más adecuado para el tratamiento de


hipertiroidismo durante la lactancia?
c} Propiltiouracilo

71. Paciente masculino de 54 años de edad que se somete a una colonoscopia para
detección temprana de cáncer. Entre los antecedentes de importancia este paciente no
cuenta con historia familiar de cáncer de colon o sintomatología relacionada a alguna
enfermedad gastroenterológica. Durante el estudio se encuentra un solo pólipo de 0.7 cm
en la unión rectosigmoidea; se realiza polipectomía. El reporte histopatológico menciona
la presencia de un adenoma tubular con displasia de bajo grado. ¿Cuál de las siguientes
conductas es la más adecuada en este paciente?
e} Solicitar colonoscopia en cinco años
72. ¿Cuál de las siguientes conductas terapéuticas es la más adecuada en el fenómeno
"encendido/apagado"de la enfermedad de Parkinson en pacientes tratados con
levodopa/carbidopa?
a} Continuar con la misma dosis de levodopa/carbidopa

73. Paciente femenino de 30 años de edad conocida por el servicio de gastroenterología


por presentar colitis ulcerativa desde hace tres años. En esta ocasión la paciente acude
por la presencia de ictericia de leve intensidad, prurito generalizado y fatiga. En los
estudios de laboratorio se observa elevación intensa de la fosfatasa alcalina. Se realiza una
colangiopancreatografía endoscópica retrógrada la cual demuestra irregularidades difusas
de las vías biliares intra y extrahepáticas consistente con estenosis multifocales de corta
longitud, anulares, que delimitan zonas de conductos biliares normales o dilatados, dando
lugar a un aspecto "en rosario" del sistema biliar.
b} Colangitis esclerosante primaria

74b. Con la sospecha clínica usted solicita:


b} Radiografía anteroposterior y lateral de antebrazo

74c. Realizado el diagnóstico y clasificación de la fractura, ¿cuándo no considera que


requiere manejo quirúrgico?
d} Fractura aislada de cúbito no desplazada

74d. Para poder referir a este paciente con un especialista en Traumatología y Ortopedia
¿qué tratamiento decide?:
b} Colocar férula braquipalmar

74e. Posterior al manejo quirúrgico y egreso hospitalario, usted recomienda:


c} Rehabilitación física

75. Paciente femenino de 30 años de edad con disnea y dolor torácico subesternal al
momento de realizar actividades físicas intensas. Entre los estudios de laboratorio se
encuentra una gasometría con pH de 7.48, P02 x de 80 mm Hg y PC03 x de 31; un
electrocardiograma presenta desviación del eje cardiaco hacia la derecha; la radiografía de
tórax se muestra normal sin infiltrados parenquimatosos ni crecimientos de estructuras
vasculares; un estudio de medicina nuclear perfusorio-ventilatorio de pulmón muestra
sólo un defecto que clasifica como de baja probabilidad para embolia pulmonar. ¿Cuál de
los siguientes estudios diagnósticos es el más adecuado en esta paciente?
a} Cateterismo de cavidades cardiacas derechas

76. ¿Cuál de los siguientes fármacos se asocia con el desarrollo de priapismo?


b} Trazodona

77. Paciente masculino de 35 años de edad que se presenta a consulta con un cuadro de
congestión nasal. Entre los antecedentes de importancia se menciona que hace
aproximadamente dos meses padeció una infección de vías aéreas superiores con
rinorrea, congestión nasal y tos. Desde entonces, el paciente ha continuado presentando
congestión nasal intensa sin presencia de secreción. Otro médico le recomendó utilizar
alfa-1 agonistas tópicos (oximetazolina) dos veces al día desde hace mes y medio. A la
exploración física se encuentra dolor leve a la percusión de ambos senos maxilares. Las
narinas están eritematosas e inflamadas sin secreción purulenta; no es posible observar
ninguna obstrucción sinusal. ¿Cuál de los siguientes tratamientos es el más probable en
este paciente?
c} Suspender eluso de alfa-1 agonista tópica

78. Paciente masculino de 18 años de edad que sufre un accidente por impacto de gran
velocidad cuando viajaba en su motocicleta en estado de ebriedad. Posterior a su llegada
al hospital,el paciente desarrolla taquipnea intensa y disminución considerable de la
saturación de oxígeno; el paciente muere por edema pulmonar. ¿Cuál de los siguientes
medicamentos pudo haber contribuido a ocasionar este cuadro?
c} Manito!

79. Estudiante de medicina de sexo masculino de 22 años de edad que acude al


consultorio de su profesor de hematología. El sujeto se siente preocupado porque
recientemente ha presentado fatiga, fiebre recurrente de predominio nocturno, así como
adenomegalias cervicales. Está preocupado porque hace poco ha estado rotando por el
servicio de neumología y ha convivido con muchos pacientes con tuberculosis. A pesar de
que el profesor de hematología no parece preocupado por el diagnóstico de tuberculosis,
decide tomar una biopsia de ganglio cervical y ordenar estudios de laboratorio. El reporte
de la biopsia menciona que existe expansión de los folículos linfoides con preservación de
la arquitectura normal; existen linfocitos atípicos en las zonas paracorticales. ¿Cuál de los
siguientes diagnósticos es el más probable en este paciente?
d} Infección aguda por el virus de Epstein-Barr
80. Paciente que presenta un estado de acidosis respiratoria crónica. ¿Cuál de los
siguientes estudios es el más adecuado para distinguir entre una hipoventilación mediada
por el sistema nervioso central o debido a alteraciones en la vía aérea/parénquima
pulmonar?
e} Medición delgradiente alveoloarterial de oxígeno

81. Paciente del sexo femenino de 12 años de edad que acude a consulta externa de
pediatría. La madre menciona que la niña ha estado cojeando desde hace varias semanas
y refiere dolor intenso en la rodilla izquierda. Al interrogatorio la madre dice que la
paciente se ha encontrado afebril,no recuerda golpes o traumatismos dirigidos a las
extremidades, ni antecedente de enfermedades recientes. A la exploración física es
evidente que la paciente no puede rotar internamente la pierna afectada. ¿Cuál de los
siguientes diagnósticos es el más probable en esta paciente?
a} Epifisiólisis de cabeza femoral

82. Masculino de 76 años que es llevado por sus familiares al neurólogo, pues presenta
alteraciones importantes en la memoria; en las últimas semanas ya no ha podido salir de
su casa porque no recuerda cómo regresar. Al interrogatorio dirigido sólo refiere
estreñimiento de varios años de evolución. Exploración física: piel seca y fría, tiroides
pequeña pero aumentada de consistencia, ruidos cardiacos rítmicos con frecuencia de
48x' y reflejos con la fase de relajación lenta.
82a. En este paciente el diagnóstico mas probable es:
b} Hipotiroidismo primario

82b. La alteración metabólica en este paciente es:


a} T3 y T4 bajas, TSH alta

82c. El mecanismo fisiopatológico causal del cuadro clínico de este paciente es:
d} Autoinmune

83. Femenino de 22 años de edad la cual se presenta a consulta de evaluación obstétrica


por encontrarse en el segundo trimestre de su primer embarazo. La paciente menciona
que recientemente ha padecido disuria,frecuencia y urgencia urinarias desde hace 48
horas. Los cultivos de orina son positivos para Escherichia coli por lo que se decide tratar a
la paciente con un esquema de nitrofurantoína durante siete días. Dos semanas después
se repite el urocultivo, una vez que la paciente se encuentra asintomática; el resultado de
éste es aún positivo para Escherichia coli. ¿Cuál de las siguientes conductas terapéuticas
es la más adecuada en esta paciente?
O a}Iniciar otro esquema de administración de nitrofurantoína y ordenarla realización de
cultivos subsecuentes para monitorizar porla presencia de bacteriuria asintomática
84. Paciente del sexo femenino de 20 años de edad se presenta a consulta ginecológica
por amenorrea primaria. A la exploración física la paciente tiene una estatura de 145 cm e
hipodesarrollo de los caracteres sexuales secundarios. La paciente trae consigo un
ultrasonido que ordenó otro médico, el cual revela la presencia de ovarios; sin embargo
éstos son pequeños y elongados. ¿Cuál de los siguientes mecanismos de enfermedad es el
que mejor explica el padecimiento de esta paciente?
d} Error en las primeras mitosis deldesarrollo

85. ¿Cuál de las siguientes características puede diferenciar de mejor manera a la


enfermedad de Crohn de la colitis ulcerativa?
O b} Granulomas no caseificantes

86. Paciente femenino de 20 años de edad estudiante de medicina la cual presenta bajo
rendimiento académico. Atribuye este bajo rendimiento a una sensación de tensión
generalizada la mayor parte del tiempo. La paciente dice que siempre está preocupada
por diversas situaciones, la mayoría aquéllas que no puede ni siquiera controlar.
Durante la entrevista es evidente que la paciente no sufre depresión. La exploración física
es normal. Se inicia tratamiento con buspirona; sin embargo una semana después la
paciente se presenta sin consulta diciendo que el medicamento no funciona y que
continúa con la misma sintomatología. ¿Cuál de las siguientes opciones terapéuticas
es la más adecuada para esta paciente?
d} Solicitar a la paciente que continúe con eltratamiento actual

87. Paciente femenino de 71 años de edad que se presenta con hipercolesterolemia que
es controlada con dieta. En esta ocasión la paciente se presenta con rigidez y dolor
alrededor de las articulaciones de los hombros y caderas. La paciente menciona que al
despertar siempre le cuesta trabajo salir de la cama. A la exploración física de las
articulaciones no se encuentra ninguna anormalidad, sólo disminución en la fuerza
muscular. ¿Cuál de las siguientes opciones es el mejor estudio que se le debe realizar a
esta paciente?
d} Medición de la velocidad de sedimentación eritrocitaria

88. Paciente femenino de 40 años de edad, indígena, que se presenta a la consulta del
centro de salud y lleva a un neonato de 20 días de vida. La paciente menciona que su hijo
ha estado flácido desde hace varios días y no acepta el alimento. El recién nacido fue
producto de un parto asistido por "partera"en la comunidad indígena de donde proviene
esta paciente. A la exploración física el paciente presenta ictericia leve, crecimiento de la
lengua, hipotonía generalizada y hernia umbilical. ¿Cuál de los siguientes es el diagnóstico
más probable en este paciente?
a} Hipotiroidismo
89. ¿Cuál de los siguientes antiarrítmicos prolonga el potencial de acción y disminución del
flujo de salida celular de potasio, sin afectar los canales de sodio ni de potasio?
a} lbutilide

90. ¿Cuál de los siguientes medicamentos se asocia con hipopotasemia como efecto
adverso?
d} Furosemida

91. Una resonancia magnética cerebral revela la presencia de tumoraciones bilaterales en


el lóbulo occipital de un paciente de 21 años con pérdida progresiva de la visión. La
biopsia de músculo esquelético revela la presencia de fibras musculares "rasgadas". Entre
los antecedentes de importancia se encuentra que el paciente tiene una madre de 50 años
de edad, la cual presenta de manera intermitente debilidad muscular y niveles elevados
de lactato sérico. Uno de los hermanos de su mamá desarrolló hemiplejía a la edad de 30
años. Si todos estos miembros de la familia, incluyendo el paciente presentan la misma
mutación genética, ¿cuál es el término adecuado para expresar la variabilidad en la
presentación clínica?
a} Heteroplasmia

92. ¿Cuál de los siguientes mecanismos de acción corresponde al de la colestiramina y el


colestipol?
a} Unión y excreción de las secreciones biliares

93. Paciente recién nacido con cianosis inmediata al parto. La realización de un


ecocardiograma revela la presencia de persistencia del tronco arterioso. ¿Cuál de los
siguientes defectos es probable que también se encuentre?
b} Comunicacióninterventricular de tipo membranosa

94. Paciente masculino de 30 años de edad que se presenta al servicio de urgencias por
presentar fiebre elevada, escalofríos, dolor en espalda torácica y lumbar,así como
malestar general. En el pasado este paciente había sido hospitalizado debido al uso de
drogas intravenosas; en esta ocasión el paciente reporta que ha consumido este tipo de
drogas en cuatro ocasiones en el último mes, la última hace una semana. A la exploración
física el paciente presenta fiebre de 38.9 ºC, frecuencia cardiaca de 109 latidos/minuto,
presión arterial de 120/75 mm Hg, y saturación de oxígeno de 98% con administración de
oxígeno de 2 L por cánula nasal. Una radiografía de tórax demuestra la presencia de un
gran número de infiltrados nodulares periféricos; estos hallazgos son corroborados por
una tomografía computarizada. Se toman hemocultivos, cuyos resultados todavía no se
encuentran disponibles. ¿Cuántos criterios menores de Duke para endocarditis presenta
este paciente?
d} 3
95. Un bebé de cinco meses de edad es llevado por sus padres a revisión por antecedente
de 48 horas con tos y secreción nasal. A la exploración física el paciente presenta
temperatura de 37.2 C, frecuencia cardiaca de 120 latidos por minuto y frecuencia
respiratoria de 35 respiraciones por minuto. La frecuencia cardiaca y el ritmo son
regulares, sin presencia de soplos. El paciente presenta leve aleteo nasal y ausencia de
retracciones intercostales. Los sonidos respiratorios demuestran esfuerzo respiratorio, y
en ocasiones se puede percibir sibilancias. Las membranas timpánicas son normales. De
las siguientes conductas, ¿cuál describe el siguiente paso?
d} Evaluar el nivelde oxigenación y capacidad para tolerarla vía oral

96. Paciente masculino de cinco meses de edad el cual presenta anemia hemolítica. Al
frotis de sangre periférica se observan eritrocitos con agregados insolubles de
subunidades de hemoglobina. Al interrogatorio el paciente fue producto de un embarazo
a término y parto eutócico; abuelos paternos y maternos son de origen griego. ¿Cuál de
las siguientes moléculas fue la principal responsable del adecuado desarrollo intrauterino
de este lactante?
c} Gamma globina

97. Paciente masculino de 50 años de edad, acude a consulta por disminución súbita de la
agudeza visual; al interrogatorio refiere que cuenta con el diagnóstico de miopía hace 15
años, se realizó cirugía refractiva corneal hace 1 año, niega haber recibido traumatismos
recientes en la región facial; la sintomatología inició de forma aguda con fotopsias y un
escotoma temporal superior del ojo derecho. FC 75 lpm, FR 18 rpm, PA 130/70 mm Hg,
peso 71 kg, talla 1.72 m.
97a. Con los datos obtenidos en el interrogatorio, usted sospecha de:
c} Desprendimiento de retina regmatógeno

97b. Para confirmar su diagnóstico usted realiza una exploración de fondo de ojo
esperando encontrar:
d} Velo grisáceo suspendido en el vítreo

97c. El tratamiento de elección en este paciente es:


c} Vitrectomía

97d. Para la vigilancia del paciente, ¿cuál de los siguientes es la complicación más común
del manejo?
b} Membrana epirretiniana

97e. Este paciente puede presentar desprendimiento de retina en el otro ojo, ¿cuál es el
tratamiento profiláctico?
b} Fotocoagulación con láser
98. Paciente masculino de 55 años de edad es llevado por su esposa al servicio de
urgencias por presentar melena desde hace tres días. El paciente niega la presencia de
dolor abdominal. A la exploración física se encuentra frecuencia cardiaca de 11O
latidos/min e hipotensión ortostática significativa. A la inspección se encuentra atrofia
bilateral temporal, conjuntivas pálidas, angiomas en araña en la parte superior del tórax,
atrofia muscular, hepatoesplenomegal ia;a la auscultación se encuentra hiperactividad de
los movimientos intestinales; a la palpación el abdomen no es doloroso. Durante la
exploración física el paciente evacuó heces de características melénicas. Se decide colocar
una sonda nasogástrica la cual aspira material gástrico en posos de café. Los estudios de
laboratorio son característicos por un hematocrito de 0.26. ¿Cuál de las siguientes
conductas es la más adecuada de manera inmediata para este paciente después de su
estabilización hemodinámica?
a} Realización de panendoscopia

99. ¿Cuál de los siguientes fármacos antibióticos ejerce su efecto mediante la inhibición de
la síntesis de proteínas en bacterias alterando la función de la subunidad SOS ribosomal?
a} Clindamicina

1OO. Paciente femenino de 22 años de edad que se presenta al consultorio universitario


por haber presentado desde hace 24 horas fiebre de 38.5 ºC, ardor faríngeo, disfagia, así
como lesiones papulovesiculares grisáceas en el paladar blando, úvula y tonsilas sin
presencia de gingivitis. ¿Cuál es el agente infeccioso que con mayor probabilidad ocasiona
estas lesiones?
d} Coxsackievirus

101. Paciente masculino de 60 años de edad con discapacidad neurológica, tiene síndrome
de von Hippel-Lindau es parapléjico, con vejiga e intestino neurogénicos. Se detectó como
hallazgo en una radiografía simple de abdomen un cálculo grande bilateral en astas de
venado. ¿Cuál es la composición más probable del cálculo?
b} Estruvita

102. ¿Cuál de los siguientes enunciados es el que más adecuadamente representa las
características de la fractura de Monteggia?
a} Fractura proximal del cúbito yluxación dela cabeza delradio

103. Paciente masculino de dos semanas de edad que se presenta a la consulta pediátrica
por presentar vómito y letargo. A la exploración física el paciente presenta hipertonicidad
y rigidez muscular.Al interrogatorio se revela que la madre ha notado un olor a "azúcar
quemada" cada vez que cambia los pañales del paciente. ¿Cuál de los siguientes
aminoácidos debe ser restringido en la alimentación de este paciente?
b} Leucina
104. Un niño de 12 años de edad es llevado al consultorio por sus padres por presentar
dolor durante deambulación y dolor localizado en la ingle y rodilla izquierdas. Al
interrogatorio el paciente menciona que el dolor es constante, sin alteraciones en la
sensibilidad. Al parecer no existió ningún evento que se haya relacionado con el inicio de
esta sintomatología, sin embargo el paciente menciona que el dolor se incrementa
durante los entrenamientos de fútbol. Cuando se rota externamente la articulación de la
cadera del paciente, éste flexiona la cadera. ¿Cuál es el diagnóstico más probable en este
paciente?
a} Epifisiólisis de cabeza femoral

105. Paciente masculino de 4 horas de vida, el cual presenta letargo, vómito y taquipnea.
Los estudios de laboratorio revelan la presencia de acidosis metabólica con brecha
aniónica incrementada, cetosis e hipoglucemia, así como elevación del ácido
metilmalónico. ¿Cuál de las siguientes reacciones se encuentra deficiente en este
paciente?
c} lsomerización

106. Es llevado al servicio de urgencias lactante con facies grotesca, con macroglosia,
edema palpebral, mínima hipotonía, con hernia umbilical, la madre refiere constipación La
alteración genéticaimplicada en este caso es:
d} Hipotiroidismo congénito

107. Se trata de una paciente de 75 años de edad, que vive en un medio rural e ingresa al
Servicio de Urgencias, fue encontrada por sus familiares en su casa somnolienta y refieren
que durante el trayecto presentó crisis convulsivas tónico-clónico generalizadas, en su
habitación tiene un calentador por combustión encendido. A su ingreso a Urgencias con
escala de coma de Glasgow de 12 puntos (M 6, O 4, V 2), se le observa disneica, con
rubicundez facial y presenta nueva crisis convulsiva, que se mitiga con benzodiazepinas
intravenosas. FC 11O lpm, FR 24 rpm, temp.35.6 ºC, PA 90/50 mm Hg, peso 68 kg, talla
1.52 m.
107a. Con los datos obtenidos del interrogatorio y exploración, ¿qué diagnóstico
sospecha?
b} Intoxicación por monóxido de carbono

107b. Para confirmar el diagnóstico usted solicita uno de los siguientes estudios:
e} Gasometría arterial

107c. Basado en los datos clínicos de la paciente, ¿qué porcentaje de COHb% espera
encontrar?
e} De 40 a 50%

107d. Establecido el diagnóstico, usted decide iniciar el siguiente tratamiento:


a} Oxígeno por mascarilla con reservorio
107e. ¿Cuál de las siguientes medidas no está indicada en el tratamiento secundario de
este paciente?
d} Anticonvulsivos

108. Hombre de 35 años de edad, fumador y bebedor social desde los 17 años, consulta
por tener dificultad para ingerir alimentos. El paciente refiere que un médico le realizó
estudios y le diagnosticóacalasia.
108a. La causa de la acalasia es:
c} Pérdida de neuronas del plexo mientérico

108b. Dentro del tratamiento farmacológico de la acalasia se encuentran indicados:


b} Nitritos y antagonistas de calcio

108c. Es el manejo quirúrgico de elección para la acalasia:


b} Miotomíalaparoscópica

108d. Se considera una alternativa en el manejo de la acalasia para quienes se encuentra


contraindicada la cirugía.
a} Dilataciones esofágicas y toxina botulínica

108e. La eficacia de las dilataciones neumáticas es de:


d) 80%

109. Niña de cinco años con antecedente de impétigo hace 1O días. Desde el día de ayer
ha presentado edema que inició en región periorbitaria y el día de hoy es generalizado.
Niega otros síntomas. Los signos vitales son normales. El examen general de orina reveló
hematuria microscópica. En relación a la patología que sospecha, señale la opción
FALSA.
a} El periodo de latencia entre lainfección y elinicio de glomerulonefritis es de 120 días

11O. Paciente masculino de 65 años de edad con factores de riesgo cardiovascular


(obesidad, tabaquismo, hiperlipidemia,antecedentes familiares, diabetes mellitus tipo 2),
así como gota y antecedente de enfermedad ulceropéptica. ¿Cuál de los siguientes
fármacos es el menos indicado?
a} Niacina

111. Paciente masculino de 29 años de edad que acude al servicio de urgencia por
presentar disnea en reposo. Menciona que necesita utilizar tres ó cuatro almohadas al
dormir para evitar la disnea. Recientemente el paciente ha notado que se le hinchan los
pies. Entre los antecedentes el paciente niega el consumo de alcohol, hipertensión o
enfermedad arterial coronaria. Hace una semana tuvo un cuadro de infección de vías
respiratorias. A la exploración física la presión es de 115/76 mm Hg y la frecuencia
cardiaca de 105 latidos/minuto. La presión venosa yugular es de 14 mm Hg. Existen
estertores en ambos campos pulmonares que se encuentran hasta las escápulas. A la
auscultación cardiaca no se encuentran soplos, pero es posible identificar el tercer y
cuarto ruidos cardiacos. En las extremidades inferiores tiene edema 2++. El
electrocardiograma no muestra cambios en el segmento ST o en las ondas T. La
radiografía de tórax muestra datos en relación a edema pulmonar y aumento del tamaño
de la silueta cardiaca. Existe elevación leve la fracción MB de la creatinina cinasa. ¿Cuál de
las siguientes opciones representa la principal causa de los síntomas de este paciente?
d} Miocardiopatía dilatada idiopática

112. Femenino de 25 años de edad se presenta a consulta con historia de diarrea de más
de 6 semanas de evolución, la cual se acompaña de distensión abdominal.Refiere haber
observado en algunas ocasiones sangre en las heces. Además, tiene historia de anemia ya
resuelta y comenta que los cereales le causan diarrea desde que era niña. A la exploración
física usted observa a la paciente pálida, con apariencia de cansancio y en el área de los
codos múltiples ronchas y ampollas que parecen muy pruriginosas.
112a. ¿Cuál es el síndrome clínico principal en esta paciente?
d} Diarreico crónico

112b. Su principal sospecha diagnóstica es:


a} Enfermedad celíaca

112c. Según su sospecha diagnóstica,¿cuál sería el mecanismo fisiopatológico?

c} No digestión de gliadina e inmunológico

112e. El tratamiento para esta paciente es:


c} Dieta libre de gluten

113. Paciente femenina de 58 años de edad, ama de casa, acude por gonartralgia al subir
escaleras y dolor en la base del primer dedo de la mano al coser desde hace varios meses.
Últimamente inició con rigidez matutina de 30 min de duración. A la exploración física
muestra dolor a la flexión de la articulación trapeciometacarpiana,engrosamiento nodular
en articulaciones interfalángicas distales de la mano, dolor a la flexión de la rodilla con
crepitación ósea.
113a. Señale el diagnóstico clínico más probable:
b} Osteoartritis

113b. Hallazgo radiológico esperado en la rodilla de la paciente.


d} Disminución irregular del espacio articular + esclerosis subcondral + osteofitos
113c. ¿Cómo se conoce al nódulo en las articulaciones interfalángicas distales que
presenta la paciente?
a} Nódulo de Heberden

113d. Señale el factor de riesgo más importante para la osteoartrosis:


a} Edad

113e. Tratamiento indicado:


d} Paracetamol + reducción dela carga articular

114. Paciente masculino de 25 años, floricultor,originario y residente de Cuautla Morelos,


cuenta con antecedente de rinitis alérgica detectada durante la infancia,así como con
alergia a la penicilina. PA: inicia en la infancia, con cuadros intermitentes de congestión
nasal,que se han intensificado desde hace 2 años,y consisten en obstrucción nasal
bilateral de predominio izquierdo, rinorrea anterior y posterior de color amarillento,
hiposmia, sensación de oídos tapados, prurito nasal, estornudos en salva y pesantez de la
cara.
114a. ¿Cuál sería su diagnóstico?
c} Rinosinusitis crónica

114b. La transiluminación sirve para valorar:


e} Senos maxilares y frontales

114c. Agentes etiológicos más frecuentes de esta enfermedad.


b} S.aureus, estafilococos coagulasa negativos y anaerobios

114d. Agente principal de la sinusitis no bacteriana, no viral.


c} Aspergillus fumigatus

114e. Complicación que se observa principalmente en diabéticos.


d} Mucormicosis

115. Masculino de 40 años de edad, trabajador de la construcción, fumador y bebedor


social, sin antecedentes de hipertensión ni diabetes,acude a consulta por presentar desde
hace un par de semanas adormecimiento de la cara dorsal del dedo medio y de la parte
externa del antebrazo derecho, además de notar que le cuesta más trabajo cargar objetos
con este mismo brazo. Al realizar la exploración física se encuentrapérdida de la
sensibilidad en la región afectada, así como pérdida del reflejo tricipital.
115a. ¿A qué nivel se integra el reflejo bicipital?
b} C5-C6
115c. La prueba de imagen con mayor utilidad es:
c} Resonancia magnética

115e. La causa más común de radiculopatía en mayores de 55 años.


c} Espondilosis

116. Masculino de 45 años de edad, ingresa a urgencias por crisis convulsivas. El familiar
refiere que mientras se encontraba comiendo presentó desviación tanto ocular como de
la cabeza hacia la derecha, seguida de pérdida súbita del conocimiento, durante lo cual se
observaron movimientos tónico-clónicos de las cuatro extremidades de unos 3 min de
duración, con incontinencia vesicaly estado confusional de 30 min posterior al episodio.
No cuenta con antecedentes patológicos.
116a. ¿Cuál es el término correcto del episodio que presentó el paciente?
d} Crisis parcial secundariamente generalizada

116b. Estudio a realizar en este paciente para el diagnóstico etiológico.


c} TC cerebral

116c. ¿Cuál es la causa más frecuente de crisis convulsiva de reciente inicio en adultos
mayores?
b} Enfermedad cerebrovascular

116d. ¿Cuál de los siguientes fármacos antiepilépticos no ejerce su acción mediante


inhibición de los canales de Na?
a} Benzodiacepinas

116e. ¿Cuál es el primer paso del tratamiento farmacológico del estado epiléptico
convulsivo generalizado?
e} Diazepam

117. Mujer de 58 años, acude por cuadro de poliartritis simétrica que afecta hombros,
codos, muñecas, IFP, IFD, rodillas, tarsos y tobillos, de cuatro años de evolución; los
cuadros se presentan una vez al mes, y desde hace cuatro meses de forma continua. Tiene
historia de uso crónico de glucocorticoides desde hace cuatro años que inicio la
enfermedad, en la actualidad se encuentra en clase funcional IV, poliartritis, múltiples
nódulos en interfalángico proximales, y bursas olecraneanas, con obesidad, hiperglucemia
e hipertensión arterial,sus exámenes de laboratorio con glucosa 120, Acido úrico de 9,
triglicéridos 300, Cr 1.4, BH 1O 000 leucocitos, diferencial normal, VSG 40. Las radiografías
de manos y pies muestran erosiones en sacabocado en 2ª IFP derecha y 1ª MTF bilateral,el
espacio articular está conservado. El diagnóstico que sospecha es:
a} Gota
118. Se presenta masculino de 50 años de edad a su consultorio portando fotografía de
ultrasonografía renal, usted observa una masa bien delimitada, en la que se interrumpe el
contorno renal, sin presencia de ecos en su interior y refuerzo posterior, El diagnóstico
más probable en este paciente es:
d} Quiste renal simple

119. Masculino de 12 años de edad es traído a consulta por su mamá, la cual señala que el
niño presenta dolores durante la noche en la parte superior externa de la pierna
izquierda.Los síntomas iniciaron hace una semana, y el dolor cede o disminuye de
intensidad con la toma de paracetamol o aspirina. A la exploración física se observa un
ligero edema en la parte superoexterna de la pierna izquierda, y al palpar el trocánter
mayor del fémur el paciente refiere dolor intenso. Temp: 36.5ºC, TA: 125/75mm Hg, FC:
70 lpm. No hay antecedentes recientes de infecciones ni traumatismos.
119a. El diagnóstico más probable es:

b} Tumor óseo

119b. En frecuencia, ¿cuál es el segundo tumor maligno óseo?


e} Osteosarcoma

119c. Es el método diagnóstico de primera línea.


b} Radiografías

119d. Características radiológicas del osteoma osteoide.


d} Nido central redondeado radiolúcido y rodeado por una zona esclerótica

120. Después de un intento de suicidio, una mujer de 35 años de edad es llevada por sus
familiares al servicio de urgencias. La paciente refiere que sólo tomó todas las píldoras
encontradas en una botella, sin embargo no recuerda el nombre del medicamento y la
botella no es hallada por los familiares. Al momento de la exploración física la paciente se
encuentra asintomática. Después de varias horas bajo observación se decide egresar a la
paciente. Tres días después regresa con datos de ictericia,coluria y dolor abdominal
localizado en el cuadrante superior derecho. Las enzimas hepáticas se encuentran
elevadas. ¿Cuál es el medicamento que con mayor probabilidad ingirió la paciente?
a} Paracetamol

121. Paciente masculino de cuatro años de edad que es llevado a consulta por talla baja.
121a. ¿Cuál de las siguientes corresponde a la definición de talla baja?
b} Talla por debajo de 2 desviaciones estándar de la talla media para su edad

121b. ¿Qué es VERDADERO en relación al retraso constitucional del crecimiento?


b} La velocidad de crecimiento no es normal

121c. En el contexto de un déficit de hormona del crecimiento, ¿cuál de los siguientes


estudios NO es útil para confirmar el diagnóstico?
c} Niveles de insulina

122. Paciente femenino de 35 años acude a Urgencias por haber sufrido un accidente
mientras realizaba labores del hogar, durante lo cual agarró un bote de plástico con un
líquido desconocido, que se le resbalo y le salpicó dentro de los ojos.Inmediatamente
despuésde esto presentó un cuadro de disminución de la agudeza visual, fotofobia intensa
y dolor importante. Al realizar el examen del ojo se observan lesiones con bordes bien
delimitados, edema corneal y conjuntiva hiperémica.
122a. El diagnósticomás probable es:
d} Quemadura por ácido

122b. La primera medida que se toma en un accidente de este tipo es:


c} Lavado con agua o sueroinmediato

122d. El tratamiento médico debe de ser:


c} Pilocarpina tópica airrigación corneal y fondos de saco con solución salina durante
30min o 1L mínimo a ocluir el ojo

122e. ¿Qué agentes provocan coagulación de proteínas?


c} Fuego directo y ácidos

123. Paciente masculino de 52 años de edad con antecedente de alcoholismo, acude a


consulta debido a que desde hace 48 horas presenta dolor localizado en la cadera derecha
que inició mientras podaba el pasto de su casa. Al interrogatorio el paciente menciona que
desde hace cerca de ocho semanas había sufrido dolor intermitente en la cadera derecha.
A la exploración física el paciente se encuentra afebrily con limitación significativa en la
rotación de la cadera, con una contractura en flexión de 20 grados y marcha antiálgica. Los
estudios de laboratorio indican niveles normales de proteína C reactiva y leucocitos; por
otro lado la velocidad de sedimentación eritrocitaria está ligeramente elevada (21 mm/h).
La radiografía de la región muestra colapso de la cabeza femoral y cambios acetabulares.
¿Cuál de los siguientes diagnósticos es el más probable en este paciente?
c} Osteonecrosis de cabeza femoral

124. Paciente femenino de cinco años que inició hace siete días con cuadro consistente en
fiebre y dolor abdominal los dos primeros días, seguido de aparición de lesiones
maculopapulares y vesículas en tronco y cara, altamente pruriginosas. En la exploración se
encuentran signos vitales normales, mucosa oral seca, múltiples vesículas rodeadas por
eritema en frente, nariz y mejillas, así como en tronco; presencia de maculopápulas
eritematosas, vesículas y pústulas con costras en el tronco. ¿Cuál de los siguientes
fármacos estaría contraindicado de forma absoluta en este caso?
b} Ácido acetilsalicílico

125. Paciente masculino de cinco años de edad, presenta traumatismo nasal de 2 h de


evolución al caerse de un columpio; se acompaña de epistaxis y edema importante que no
permite valorar la pirámide nasal. No se palpa crepitación de huesos propios, rinoscopia
sin alteración septal. La siguiente conducta sería:
b} Controlar la epistaxis, antiinflamatorios y revalorar en 72 h

126. Paciente femenino de 31 años de edad con gesta 2, para 2, sexualmente activa y con
uso de dispositivo intrauterino desde hace cuatro años. El motivo de la consulta es por
dolor abdominal intenso y pélvico bilateral que aparece desde hace tres días, con
sensación de pesantez pélvica. La sintomatología inició al día siguiente después de su
última menstruación. El día de hoy la paciente ha presentado náusea y cefalea. A la
exploración física la paciente se encuentra levemente taquicárdica, presión arterial dentro
de rangos normales, temperatura de 39.5 ºC. Reporta dolor a la palpación de la región
abdominal pélvica. La exploración ginecológica revela secreción vaginal purulenta y dolor
extremo a la movilización cervical;no se palpan masas en anexos. La biometría indica
ligera anemia y leucocitosis de 14 000 células/µL. El ultrasonido transvaginal demuestra
septaciones de la pared tubaria (signo de la rueda dentada). Se realiza prueba de
embarazo en orina la cual resulta negativa. ¿Cuál es la conducta terapéutica
inmediata más adecuada?
e}Ingreso hospitalario y administración de clindamicina 900 mg intravenosa tres veces
al día en combinación con gentamicina 2 mg/kg intravenosa y posteriormente 1.5 mg/kg
intravenosa cada 8 horas.
127. Paciente masculino de 42 años de edad actualmente hospitalizado por evento
vascular cerebral. Se interconsulta por presentar rinorrea purulenta, fiebre y halitosis. A la
exploración física se encuentra bajo intubación orotraqueal y con una sonda nasogástrica
en fosa nasal derecha; presenta una deflexión septal izquierda contactante y salida de
material purulento por ambos meatos medios.
127a. ¿Cuál es el germen más frecuente que ocasiona el cuadro del paciente?
a} Proteus

127b. Cuatro días después el paciente presenta proptosis y quemosis izquierdas,


limitación en la aducción del ojo izquierdo y disminución en la agudeza visual izquierda.
¿Cuál es el diagnóstico del paciente?
e} Celulitis orbitaria

127c. ¿Cuál es el tratamiento para esta complicación?


a} Cirugía endoscópica funcional en caso de no mejorar con antibióticos,irrigación nasal y
descongestionantes

128. Acude a consulta una madre con su hijo de 7 meses de edad. La madre refiere que el
menor comenzó con irritabilidad y fiebre desde hace 3 días. Menciona que durante este
tiempo administró un medicamento no especificado y que la fiebre no cedió. El motivo de
la consulta es porque dice que la fiebre ha desaparecido, pero que el niño inició con unas
manchas rojas por todo el cuerpo. A la exploración física encuentra paciente masculino
irritable, sin fiebre, con faringe y membranas timpánicas no hiperémicas, no observa
exudados en faringe ni líquido detrás de la membrana timpánica. Observa un exantema
maculopapular en tronco, cuello, cara y extremidades, no pruriginosas y sin descamación.
El aspecto general del menor es bueno.
128a. De acuerdo con las características del paciente, ¿cuál es el diagnóstico más
probable?
c} Roséola

128b. Con base en la sospecha clínica, ¿cuál es el agente etiológico del padecimiento del
paciente?
b} HHV-6

128c. ¿Cómo se comprueba la sospecha de diagnósticode este paciente?


b} Clínico

128d. ¿Cuál es una complicación probableenpacientes con roséola?


a} Conwlsiones febriles

128e. ¿Cuál es el tratamiento indicado para este paciente?


b} Paracetamol
129. Paciente femenino de 69 años que acude a consulta porque ha manchado su ropa
interior con sangre. Al interrogarla de manera apenada refiere que ha ocurrido en dos a
tres ocasiones todas relacionadas con el coito. Refiere tener relaciones sexuales sólo con
su esposo, con el que lleva casada 42 años. Su fecha de última menstruación fue hace 20
años. Menciona además dispareunia, refiriéndolo como ardor poscoital. Niega
disuria,fiebre o exudados purulentos. A la exploración pélvica se observa una mucosa
delgada y friable, con un poco de petequias y eritema,con muy pocos pliegues. Se observa
leucorrea serosa con un pH de 6. Un frotis muestra la presencia de leucocitos y
disminución de la población de lactobacilos. ¿Qué tratamiento le daría a esta paciente?
d} Estrógenos tópicos vaginales

130. Paciente masculino de 55 años, con DM 2 diagnosticada hace un año. Acude al


servicio de urgencias con historia de compromiso del estado general progresivo en los
últimos tres meses, pérdida de peso, y sudoración nocturna. La semana previa al ingreso
se agregó dolor en flanco y fosa ilíaca izquierda, fiebre de 38.5 ºC y vómito escaso. Al
examen físico se observa caquéctico, pálido, febril,taquicárdico, destacando muget en la
cavidad bucal. Se realiza tomografía de abdomen,que revela múltiples adenopatías
hipodensas retroperitoneales y a nivel de la raíz del mesenterio. Se confirma
inmunodepresión asociada a VIH. Después se realiza punción de adenopatías
obteniéndose baciloscopia (+) para tuberculosis. La biopsia mostró abundantes bacilos
intracelulares.
Respecto a las distintas formas clínicas de tuberculosis, señale la aseveración falsa:
e} La afección intestinal más frecuente es duodeno-yeyunal

131. Paciente masculino de 63 años acude a consulta por pérdida de 18 kg de peso (> 20%
peso habitual) y fiebre vespertina de cinco meses de evolución. Dentro de sus
antecedentes tiene un índice tabáquico de cinco. A la exploración física se observa febril,
adelgazado, disneico, frecuencia cardiaca de 100 latidos por minuto, Tensión arterial de
100/70 mm Hg sin ortostatismo, hepatomegalia. Sin otras lesiones.
El diagnóstico de tuberculosis se hace por:
d} Estudio bacteriológico

132. Paciente femenino de 55 años con antecedentes de diabetes mellitus descontrolada


es traída a urgencias en ambulancia por presentar disuria y polaquiuria asociada con dolor
en el flanco izquierdo de cuatro días de evolución, agregándose malestar general,fiebre y
escalofríos desde hace dos días. Sus signos vitales son temperatura de 39 ºC, frecuencia
cardiaca de 100 latidos por minuto, presión arterial 90/60 mm Hg y frecuencia respiratoria
de 18 respiraciones por minuto. La biometría hemática muestra leucocitos de 1O 000
cél/mm3 con 20% de bandas, plaquetas 130 OOO/mm3, Usted diagnostica un síndrome
de respuesta inflamatoria sistémica y mantiene una alta sospecha clínica de pielonefritis
con urosepsis.
Los siguientes son criterios del síndrome de respuesta inflamatoria sistémica, excepto:
c} Presión arterial < 110/60 mm Hg
1. Mujer de 20 años de edad quien presenta desde hace 1O años cuadros remitentes en
zonas de exposición solar caracterizados por pápulas eritematosas, costras hemáticas,
exulceraciones y zonas de liquenificación,algunas manchas y cicatrices residuales. Además
presenta conjuntivitis y seudopterigión. Refiere que la enfermedad empeora en épocas de
verano y le provoca prurito intenso que no mejora con antihistamínicos.
1a. ¿Cuál es su diagnóstico?
e} Prúrigo actínico

1b. ¿Cuál es el tratamiento de elección del prúrigo actínico?


e} Talidomida

2. ¿Cuál de los siguientes tipos de pericarditis es el que más frecuentemente se asocia a


neoplasias malignas?
e} Pericarditis hemorrágica

3. ¿Cuál de los siguientes hallazgos es correcto en relación al diagnóstico de la


enfermedad pulmonar obstructiva crónica?
e} La limitación al flujo de aire no es reversible en pacientes con enfermedad pulmonar
obstructiva crónica

4. ¿Cuál de los siguientes fármacos es el más adecuado para el tratamiento de la


oftalmopatía infiltrativa de la enfermedad de Graves?
a} Esteroides

5. Paciente masculino de 70 años con diagnóstico previo a enfermedad de Parkinson


actualmente en tratamiento con levodopa, en combinación con carbidopa. El paciente se
queja que aun cuando tiene apego al tratamiento farmacológico presenta periodos de
intensa restricción motora, mientras que existen otros de libre movilidad. ¿Cuál de las
siguientes opciones es la mejor conducta terapéutica para este paciente?
a} Continuar con la misma dosis de levodopa/carbidopa

6. Se presenta a consulta pediátrica un paciente de sexo masculino de 11 años de edad. La


madre lleva a su hijo a consulta porque ha notado "que tiene la "cara hinchada" y está
preocupada que esto se deba a algún tipo de "alergia" o "intoxicación". Al interrogatorio
el niño parece no tener molestias, más que un cuadro de infección reciente de vías aéreas
superiores. No existe ningún otro antecedente de importancia en este paciente. A la
exploración física se presenta normotenso, con edema periorbital y edema en ambas
extremidades. Se realiza una prueba rápida cualitativa de orina, la cual muestra
proteinuria extensa; al ver este resultado se decide solicitar estudios de laboratorio en
sangre y orina. Los estudios de sangre revelan una concentración de albúmina de 2.4 g/dL,
colesterol LDL de 300 mg/dL; el estudio de orina de 24 horas reveló una cantidad de
proteína excretada de 4 g. ¿Cuál de los siguientes hallazgos son los más probables que se
presenten en la biopsia renal de este paciente?
a} Apariencia normal a la microscopia de luz
7. Paciente masculino de 40 años de edad el cual se presenta al consultorio del médico
laboral porque desde las últimas 12 horas ha presentado una erección constante. Esta es
la primera ocasión que esto le ocurre. Al interrogatorio el paciente menciona que se
encuentra en tratamiento farmacológico por depresión mayor con insomnio. ¿Cuál es el
medicamento que más probablemente está causando la sintomatología de este paciente?
b} Trazodona

8. Paciente del sexo femenino de 39 años de edad sin ningún antecedente de importancia
se presenta a consulta con tos con esputo amarillento y espeso desde hace 72 horas. La
paciente menciona que también tiene rinorrea de apariencia transparente, ardor de
garganta, mialgias y cefaleas leves desde hace tres días previos a que iniciara la tos. La
semana pasada acudió a la misma torre de consultorios médicos porque sus dos hijos, uno
de ocho años y otro de cinco años tenían sintomatología similar. La paciente menciona
que la tos es más intensa cuando se encuentra realizando actividad física, y desaparece
durante la noche. Al interrogatorio se niega la presencia de fiebre, escalofríos, secreción
sinusal o dolor torácico. A la exploración física los signos vitales se muestran estables; en
ambas bases pulmonares se pueden percibir sibilancias, y se corrobora la presencia de tos
con esputo espeso y amarillento. A pesar que ha tratado con antigripales usuales, éstos
han tenido poco impacto en la sintomatología. ¿Cuál de las siguientes conductas es la más
adecuada en esta paciente?
c} Iniciar tratamiento con agonistas beta-2 adrenérgicos inhalados y antitusígenos

9. Paciente agricultor de 53 años de edad que acude al servicio médico local por presentar
confusión, calambres musculares y dificultad para respirar. Los antecedentes médicos son
irrelevantes. A la exploración física el paciente se observa diaforético, con bradicardia y
sibilancias espiratorias. Las pupilas se encuentran mióticas, pero con respuesta a la
estimulación luminosa. Posterior a la administración de atropina los síntomas del paciente
mejoran considerablemente. ¿Cuál de los siguientes eventos tiene el paciente aún riesgo
de presentar?
d} Parálisis muscular

1O. Estudiante de medicina de sexo masculino de 22 años de edad que acude al


consultorio de su profesor de hematología. El sujeto se siente preocupado porque
recientemente ha presentado fatiga, fiebre recurrente de predominio nocturno, así como
adenomegalias cervicales. Está preocupado porque hace poco ha estado rotando por el
servicio de neumología y ha convivido con muchos pacientes con tuberculosis. A pesar de
que el profesor de hematología no parece preocupado por el diagnóstico de tuberculosis,
decide tomar una biopsia de ganglio cervicaly ordenar estudios de laboratorio. El reporte
de la biopsia menciona que existe expansión de los folículos linfoides con preservación de
la arquitectura normal;existen linfocitos atípicos en las zonas paracorticales. ¿Cuál de los
siguientes diagnósticos es el más probable en este paciente?
d} Infección aguda por el virus de Epstein-Barr
11. Paciente masculino de 33 años de edad se encuentra hospitalizado por sobredosis de
cocaína. Ha presentado hipertermia, taquicardia y urgencia hipertensiva. Desarrolla
episodios breves de taquicardia ventricular sin pérdida del estado de alerta; pero estos
episodios se están volviendo más frecuentes y de mayor duración. ¿Cuál de los siguientes
tratamientos es el más adecuado en este paciente?
a} Administración de propranolol

12. ¿Cuál de las siguientes condiciones es contraindicación absoluta para el uso de


anticonceptivos orales?
a} Cefalea migrañosa con datos de focalización neurológica

13. Masculino de 2 días VEU. Producto G:I, madre 24 años sana, control prenatal irregular.
Dos US gestacionales normales. Obtenido por parto eutócico a las 37 sdg. Exploración:
peso 2 150 g, Talla 46 cm, PC 37 cm Apgar 7/8. Fontanela anterior 5 x 3 cm. Borde
hepático palpable 4-4-4 cm bajo borde costal, esplenomegalia 3 cm bajo borde costal.
Adenopatías palpables de 0.5 cm submaxilares, axilares e inguinales.
La entidad patológica causal de la sintomatología de este paciente es:
d} Toxoplasmosis

14. Masculino de 26 años en estudio diagnóstico por deglución dolorosa y dolor torácico
retroesternal. A excepción de estos síntomas el paciente se encuentra sano. Los síntomas
iniciaron hace dos días y han progresado (menciona que hasta la deglución de la saliva es
dolorosa). Al interrogatorio niega náusea,vómito, hematemesis o disnea. Se encuentra
bajo un esquema de dos semanas de doxiciclina para tratamiento del acné. A la
exploración física el paciente no presenta alguna alteración evidente, aun a la exploración
de la orofaringe. ¿Cuál de las siguientes medidas terapéuticas/diagnósticas es el siguiente
paso en este paciente?

c} Suspender la administración de doxiciclina

15. Una pareja de padres sanos lleva a consulta pediátrica a su hijo varón de dos años de
edad el cual presenta retraso mental y un exantema eccematoso. Es inevitable percibir el
olor a moho que desprende el niño. ¿Cuál es la probabilidad de que un segundo hijo de
esta misma pareja presente el mismo cuadro?
a} 1/4

16. ¿Cuál es la conducta más adecuada en un paciente que tiene evidencia de infección
reciente por tuberculosis (prueba de PPD positiva, cuando se tenía evidencia de
negatividad en el pasado)?
d} Solicitar una radiografía de tórax
17. Los estudios de laboratorio de un paciente masculino de 35 años de edad muestran
anormalidades de la función hepática. Entre los antecedentes de importancia se
encuentra el consumo de drogas intravenosas. El panel serológico de hepatitis B presenta
los siguientes resultados:
- Antígeno de envoltura del virus de la hepatitis B negativo
- Anticuerpo contra el antígeno de envoltura del virus de la hepatitis B negativo
- Anticuerpo contra el core del virus de la hepatitis B positivo
- Antígeno de superficie del virus de la hepatitis B postivio
- Anticuerpo contra el antígeno de superficie del virus de la hepatitis B negativo

¿Cuál de los siguientes enunciados es el más correcto en este caso?


c} Estado crónico portador delvirus de la hepatitis B con un bajo grado deinfectividad

18. ¿Cuál de las siguientes interacciones farmacológicas incrementa el efecto


anticoagulante de la warfarina?
a} Cefalosporinas de tercera generación

19. Paciente masculino de 22 años de edad que se presenta a consulta porque tuvo hace
siete días un episodio de otitis externa derecha. Entre los antecedentes de importancia se
encuentra el hecho de que este paciente es nadador. Para su cuadro de otitis externa se le
recomendó suspender sus actividades de natación y administración de una solución ótica
de ácido acético a 2%. A pesar que el paciente tuvo apego al tratamiento, los síntomas no
mejoraron. A la exploración del conducto auditivo externo derecho se revela exudado en
cantidad moderada y eritema. ¿Cuál es el mejor tratamiento que se debe administrar a
este paciente en este momento?
b} Administración de fluoroquinolona en combinación con esteroides tópicos

20. ¿Cuál de las siguientes alteraciones se presenta en el síndrome de X frágil?


c} Macroorquidismo, mandíbula y orejas grandes

21. Paciente femenino de 34 años que acude a urgencias por presentar dolor abdominal
intenso de 8 h de evolución que la despertó mientras dormía,acompañado de anorexia,
náusea, vómito y escalofríos. El dolor es mayor en el hipocondrio derecho, es constante,
se irradia a la escápula derecha y mejoró transitoriamente con el vómito. La paciente
utiliza anticonceptivos orales desde hace cinco años, niega consumo de alcohol y drogas. A
la exploración física se encuentra anictérica, levemente taquicárdica, temperatura de 38
ºC, presión arterial 115/80 mm Hg, presenta distensión abdominal, dolor a la palpación en
el cuadrante superior derecho, interrupción de la inspiración al palpar debajo del reborde
costal derecho y resistencia muscular. Los estudios de laboratorio muestran leucocitosis
de 13 000 cél/mm3, elevación discreta de transaminasas, fosfatasa alcalina y amilasa,
bilirrubina total en límites normales.
21a. ¿Cuál es el diagnóstico más probable?
d} Colecistitis aguda

21b. ¿Cuál es el estudio diagnóstico más adecuado en este momento?


b} Ultrasonido de hígado y vías biliares

21c. ¿Cuál es el manejo adecuado en el servicio de urgencias?


a} Hospitalizar,ayuno, sonda nasogástrica, hidratación y analgésicos IV, antibióticos
empíricos, consultar al servicio de cirugía

22. ¿Cuál es el mecanismo de acción por el que la bromocriptina puede mejorar la


sintomatología de la enfermedad de Parkinson?
e} Agonista sobrelos receptores dopaminérgicos

23. ¿Cuál de los siguientes fármacos con actividad analgésica tiene la menor actividad
antiplaquetaria?
b} Rofecoxib

24. ¿Cuál de los siguientes cambios se observa en el caso de diarreas acuosas por
organismos enterotoxigénicos?
e} Ausencia de cambios en el volumen del espaciointracelular

25. ¿Cuál de los siguientes medicamentos se asocia con hipopotasemia como efecto
adverso?
d} Hidroclorotiazida

26. ¿Cuál de los siguientes trastornos se caracteriza por niveles incrementados de la


hormona luteinizante y foliculoestimulante, con niveles disminuidos de testosterona y
espermatozoides?
e} Síndrome de Klinefelter

27. Paciente femenino de seis años de edad con bajo rendimiento escolar se presenta a la
consulta pediátrica. La maestra informa que con frecuencia la niña tiene periodos de
inatención en los que se queda mirando fijamente al pizarrón o cualquier otro punto con
un parpadeo rápido. ¿Cuál es el medicamento más adecuado para la condición que
presenta esta paciente?
d} Etosuxamida

28. ¿En el medio hospitalario, las infecciones producidas por gramnegativos productores
de betalactamasas de amplio espectro son más frecuentes de ocurrir posteriormente al
uso frecuente de cuál de la siguiente clase de antibióticos?
a} Cefalosporinas de tercera generación
29. Paciente masculino de 45 años, sin antecedentes familiares de importancia,
enfermedad de Crohn en tratamiento. Acude a urgencias en un grito de dolor, muy
inquieto, presenta dolor de inicio súbito en fosa lumbar derecha, intensidad 10/1O, que se
irradia a genitales y se acompaña de diaforesis, náusea y vómito, no se alivia con ninguna
posición. Presenta fiebre de 38 C. La radiografía simple de abdomen muestra un lito
radiopaco en el uréter derecho y como hallazgo importante agenesia de riñón izquierdo.
El laboratorio muestra hiperoxaluria e hipocalciuria que sugieren la presencia de lito de
oxalato de calcio con cólico renal complicado. De las siguientes señale la indicación de
cirugía como primera elección:
c} Litiasis de estruvita localizada en cavidades caliceales

30. Se presenta a la consulta externa una niña de tres años de edad, acompañada de su
madre, por la presencia de eccema y una ""bolita"" en la porción medial de su muslo
izquierdo. La lesión a la que se refiere la madre de la paciente apareció aproximadamente
hace una semana y desde entonces ha crecido progresivamente hasta encontrarse el día
de hoy eritematosa y dolorosa. A la exploración física se puede observar piel seca y
evidencia de escoriación en la fosa antecubital y las rodillas. Se puede encontrar un
nódulo necrótico de aproximadamente 1 cm, con drenaje opaco, en la porción medial del
muslo izquierdo. Alrededor del nódulo la piel se encuentra inflamada. ¿Cuál es el manejo
más adecuado en esta paciente?
c} Cultivo del líquido de drenaje, seguido de tratamiento con cefalexina oral

31. ¿En cuál de los siguientes escenarios NO se indica la colonoscopia?


e} Paciente femenino de 64 años de edad con colitis ulcerativa desde hace 25 años y que
se presenta en este momento con megacolon tóxico

32. ¿Cuál de los siguientes fármacos antibióticos ejerce su efecto mediante la inhibición de
la formación de puentes cruzados en la pared
celular de peptidoglucano?
O b} Penicilina

33. Recién nacido de 39.5 semanas de gestación, obtenido por cesárea,presenta dificultad
respiratoria durante las primeras horas de vida, con frecuencias respiratorias de 90
respiraciones por minuto y cianosis que mejora con oxígeno. Los campos pulmonares
están bien ventilados. La radiografía de tórax muestra líneas de líquido en las cisuras y
marcas vasculares prominentes.
33a. Con base en las características del paciente, ¿cuál es su sospecha diagnóstica?
a} Taquipnea transitoria del recién nacido
33b. ¿Cuál es la fisiopatología que ocasiona la taquipnea transitoria del recién nacido?
b} Lenta absorción dellíquido de los pulmones fetales

34. ¿Cuál de las siguientes aseveraciones respecto a la vacuna del virus del papiloma
humano es correcta?
e} Las mujeres vacunadas conla vacuna contra la infección delvirus del papiloma humano
deben continuar acudiendo anualmente para la realización de citología y tinción de
Papanicolau

35. Paciente masculino de 27 años es referido para evaluación de hematuria. El paciente


ha notado hematuria sin dolor dos o tres veces al año, los últimos seis años, la cual remitió
espontáneamente a los dos días. En esta ocasión ha aparecido sangre rojo brillante en su
orina los pasados ocho días. Niega haber tenido un traumatismo en sus riñones o
cualquier otra enfermedad reciente. Aunque tiene un hermano menor con anemia de
células falciformes, el paciente ha estado libre de cualquier síntoma de esta enfermedad y
no hay antecedentes heredofamiliares de enfermedad renal incluyendo infecciones o
litos. El examen general de orina muestra orina de rojo a rosa con numerosos eritrocitos,
sin proteinuria detectable por tira reactiva. ¿Cuál es el diagnóstico más probable?
e} Rasgo de células falciformes

36. Un hombre de de 71 años de edad acude a consulta porque presenta dolor en la


región glútea izquierda que se irradia hacia la porción lateral del muslo y el dorso del pie.
El paciente niega la presencia de debilidad. Al interrogatorio menciona que el dolor
empeora cuando inclina el cuerpo hacia delante, por ejemplo, cuando intenta amarrar las
agujetas de sus zapatos. No existen alteraciones de la función vesical o rectal anal. A la
exploración física no existen anormalidades en la marcha. Cuando eleva su miembro
inferior en completa extensión se puede reproducir el dolor.Al continuar con la evaluación
es evidente que el extensor largo del primer dedo izquierdo se encuentra parético en
comparación con el lado contralateral. Los reflejos patelares y del tendón de Aquiles no
muestran anormalidades. ¿Cuál de los siguientes diagnósticos es el más probable en este
paciente?
b} Radiculopatía a nivel de L5

37. Se presenta al departamento de urgencias un paciente de 19 años de edad que sufrió


un accidente automovilístico. El paciente se encuentra hipotenso e hiperventilando. Los
estudios de laboratorio revelan que el paciente sufre una acidosis metabólica con
incremento de la brecha aniónica y aumento del lactato plasmático. ¿Cuál de los
siguientes eventos a nivel bioquímico explica de mejor manera el cuadro de
este paciente?
e} Disminución dela actividad dela enzima piruvato deshidrogenasa

38. ¿Cuál de los siguientes hallazgos NO se observa en la enfermedad de Wilson?


a} Elevación de ceruplasmina
39. Paciente femenino de 50 años, con pérdidas involuntarias de orina de dos años de
evolución. Antecedente obstétrico de cuatro partos e histerectomía total con
ooforectomía bilateral,a los 40 años. No ha tomado terapia de sustitución hormonal. Fue
operada hace un año de incontinencia urinaria de esfuerzo por técnica de Marshall-
Marchetti-Krantz y al cabo de 15 días de operada, volvió a presentar los episodios de
incontinencia o pérdida involuntaria de orina. En la actualidad, consulta por presentar
infecciones urinarias sintomáticas a repetición de dos años de evolución, con episodios de
pérdidas urinarias involuntarias desde aproximadamente un año de evolución, las cuales
las presenta al querer orinar o cuando hace algún tipo de esfuerzo (levantar pesos, reírse,
toser) y se han intensificado en los últimos meses al punto de afectar negativamente su
calidad de vida. Presenta en la actualidad flaccidez importante de los músculos del piso
pélvico, con la presencia de cistocele grado 1 y alteración importante de los ángulos de
continencia (ángulos uretrovesical posterior y ángulo uretrovertical), y presenta en el
estudio urodinámico hiperactividad vesical pero con contractilidad deficiente. Al
realizarle determinación de orina residual,ésta es significativa con más de 100 mL. Y el
cuestionario de síntomas de impacto de su incontinencia urinaria tiene 26 puntos
(puntuación grave).
En relación a este caso es cierto que:
d} Durante la exploración física el médico debe reproducir laincontinencia urinaria y
observarla, examinando al paciente en posición de pie.

40. Según la clasificación de fracturas del platillo tibial de Schatzker, ¿cuál de las siguientes
opciones representa una fractura bicondilar?
e} Tipo V

41. Se presenta a consulta un lactante con evidente retraso mental. Durante la


exploración física se encuentra que el paciente presenta hepatomegalia. Por sospecha
clínica se solicita a la enfermera que mida las concentraciones de glucosa capilar,las cuales
resultan disminuidas. Se solicita una biopsia hepática la cual demuestra acumulación de
materialtipo dextrinas en el citoplasma de los hepatocitos con ausencia de infiltración
grasa. ¿Cuál de las siguientes enzimas es la que con mayor probabilidad se encuentra
deficiente en este paciente?
c} Amilo-1,6-glucosidasa

42. Paciente masculino de 20 años de edad que acude a consulta para obtener un
certificado de buena salud y poder ingresar a un gimnasio. A la auscultación cardiaca es
posible percibir un soplo en el segundo espacio intercostal izquierdo, en el borde esternal.
El soplo se describe en crescendo-decrescendo, se presenta durante toda la sístole y
diástole, con máxima intensidad que coincide con el segundo ruido cardíaco. Al rotar la
cabeza del paciente o al cambiar la posición de ésta el soplo no cambia. ¿Cuál de las
siguientes opciones describe de mejor forma el soplo que presenta este paciente?
a} Escontinuo debido a un cortocircuito congénito, probablemente persistencia del
conducto arterioso
43. Femeninode 27 años es valorada en sala de urgencias por presentar actividad uterina
irregular y salida abrupta de líquido vaginal.
Antecedentes: cursa con 34 semanas de embarazo.
La entidad nosológica responsable de la sintomatología de esta paciente es:
c} Infección de vías urinarias

44. Lactante de 3 meses, nacido a término y sin antecedentes de interés, fue ingresado
por presentar rectorragia de aparición brusca. Al ingreso presentaba palidez generalizada
y datos de choque. A la exploración física se percibe abdomen blando, depresible y
ligeramente distendido. El tacto rectal demostró un gran coágulo de sangre en el ámpula
recta. En el hemograma se reportó una hemoglobina de 8.1 g/dl, por lo que fue
transfundido.
44a. Con base en las características del paciente, ¿cuál sería su principal sospecha clínica?
c} Divertículo de Meckel

44b. ¿Qué método diagnóstico es el más adecuado para confirmarsu sospecha clínica?
b} Gammagrafía

44c. En relación con su sospecha clínica, ¿cuál es la fisiopatología de este padecimiento?


a} Falla de lainvolución delconducto onfalomesentérico

44d. Si su sospecha clínica se confirma,¿cuál es el tratamiento indicado?


b} Extirpación quirúrgica

44e. De acuerdo con su sospecha clínica, ¿qué complicación es una posibilidad para este
paciente?
d} Fuga dela anastomosis

45. Niño de siete años con antecedente de faringitis bacteriana que recibió manejo
antibiótico hace dos semanas y se resolvió por completo. Desde el día de ayer ha
presentado hematuria y oliguria. Además de esto, los padres refieren edema periorbitario.
El paciente niega otros síntomas. En el examen físico se encuentra con palidez y edema
periorbitario, el resto de la exploración es normal.

¿Cuál es el diagnóstico más probable?


c} Glomerulonefritis posestreptocócica

46. Paciente femenino de 24 años de edad con diagnóstico de bulimia que ha requerido
ingreso a la sala de urgencias en varias ocasiones. En esta ocasión se presenta con fatiga,
mialgias, debilidad muscular de las extremidades inferiores, así como diarrea intensa. El
electrocardiograma demuestra inversión de la onda T,presencia de una onda U
prominente, así como depresión del segmento ST y prolongación del intervalo QU. ¿Cuál
es el origen de la sintomatología en esta paciente?
d} Hipopotasemia
47. Paciente femenino de cuatro años que acude a consulta por odinofagia de tres días de
evolución. El día de hoy inició de forma súbita con fiebre, afonía, dificultad respiratoria y
estridor. En el examen físico se observa paciente taquipneica, con intensa sialorrea,tiraje
intercostal y subcostal, retracción xifoidea, en la auscultación se percibe estridor referido,
sin crepitantes ni sibilancias. ¿Cuál es su diagnóstico?
e} Epiglotitis aguda

48. Paciente femenino de 64 años de edad que acude al servicio de urgencias por dolor
torácico. La paciente es conocida por el servicio ya que hace cuatro días sufrió una caída
sobre la cadera derecha, lo cual requirió inmovilización por dolor de la región. Hace 24
horas inició con dolor y edema en la pierna derecha. No cuenta con ningún otro
antecedente de importancia y sus único medicamento actualmente es una combinación
de estrógenos y progesteronas como tratamiento de reemplazo hormonal. ¿Cuál de los
siguientes estudios de diagnóstico NO es de utilidad cuando se utiliza de forma única para
realizar el diagnóstico definitivo del padecimiento de esta paciente?
b} Dímero O

49. Un paciente masculino de 7 meses de edad es llevado al departamento de Urgencias


por su madre a las 3:00am. En el interrogatorio la madre menciona que varios miembros
de la familiar han tenido ""resfriados"" en la última semana. El padecimiento del menor
inició con tos y coriza hace 3 días, y posteriormente la tos se volvió ""de perro"". En la
exploración física se observa paciente masculino irritable, con faringe eritematosa y
estridor inspiratorio. Sus signos vitales son Temp: 38ºC, FR: 55 rpm y FC: 140 lpm."
49a. Con base en las características del paciente, ¿cuál es el diagnóstico más probable?
a} Síndrome de croup

49b. De acuerdo con su sospecha clínica,¿cuál es la causa de este padecimiento?


d} Virus dela parainfluenza

49c. De acuerdo con su sospecha clínica, ¿qué hallazgos encontraría en la radiografía?


b} Signo del campanario

49d. Con base en su sospecha clínica, ¿qué tratamiento le indicaría al paciente?


b} Nebulización con epinefrina y corticoesteroides

49e. En relación con su sospecha clínica,¿cuál es la complicación que se puede presentar


en el paciente?
a}Infección en oído medio
50. Paciente masculino de 17 años se presenta a consulta por dolor en el brazo izquierdo
de aproximadamente 3 meses de evolución,y presencia de fiebre en el último mes. En la
exploración física se observa edema del tejido blando sobre los dos tercios superiores del
húmero izquierdo, condolor leve a la palpación; asimismo,se confirma la presencia de
fiebre.
50a. Con base en las características del paciente, ¿cuál es el diagnóstico más probable?
a} Sarcoma de Ewing

50b. De acuerdo con su sospecha clínica,¿qué hallazgos esperaría encontraren una


radiografía?
b} Capas de cebolla

50c. En relación con su sospecha clínica, ¿cuál es la fisiopatología de este padecimiento?


d} Translocación cromosómica

50d. Con base en su sospecha clínica, ¿cuál es el tratamiento que recomendaría?


c} Quimioterapia, radioterapia o cirugía

50e. De acuerdo con su sospecha clínica, ¿cuál es la complicación más importante que se
puede presentar?
a} Fractura patológica

51. ¿Cuál de los siguientes ha demostrado ser uno de los predictores más importantes de
hiperuricemia en diversos estudios epidemiológicos?
d} Índice de masa corporal

52. Se presenta paciente masculino de 14 años de edad con peso de 58 kg, asintomático y
sin antecedentes patológicos relevantes en el interrogatorio. Consulta por un registro de
hipertensión arterial (156/95 mm Hg) detectado en un control de rutina que se realizó en
su colegio previo al inicio de la temporada de actividades deportivas de este año.En la
exploración física no se encuentran anormalidades, únicamente se identifica presión de
150/100 mm Hg en ambos miembros superiores.
52a. Con base en las características del paciente, ¿cuál es el diagnóstico más probable?
b} Coartación de la aorta

52b. De acuerdo con su sospecha clínica, ¿con qué problema médico se ha asociado este
diagnóstico?
b} Síndrome de Turner

52c. En relación con su sospecha clínica, ¿qué hallazgos esperaría encontrar en la


radiografía de tórax?
c} Muescas enlos bordesinferiores delas costillas
52d. De acuerdo con su sospecha clínica, ¿cuál es el tratamiento indicado en este
paciente?
a} Anastomosis término-terminal

52e. Respecto a su sospecha clínica, ¿cuál es la complicación que se puede presentar a


largo plazo?
b} Hipertensión sistémica y disfunción miocárdica

53. Una paciente de 28 años de edad, sin antecedentes relevantes, cursa 33 y 217
semanas de gestación (sdg) por fecha de última menstruación (FUM), primigesta, hasta
ahora embarazo normoevolutivo. Acude a consulta por presentar cefalea intensa de 30
a45 minutos de evolución acompañada de epigastralgia, se toman signos vitales en
reposo, en ambos brazos, encontrando hipertensión arterial sistémica con tensión arterial
media (TAM) de 130 mmHg, FC 90 lpm, FR 20 rpm,temp. 36.6ºC, PA 170/11O mm Hg,
peso 65 kg, talla 1.68 m.
53a. Con los datos obtenidos el diagnóstico de sospecha clínica es:

c} Preeclampsia severa

53b. Para protocolizar a la paciente usted solicita:


a} Biometría hemática, química sanguínea, PFH,TTPa, INR y examen general de orina

53c. Una vez monitoreada la paciente, establecido un acceso vascular periférico y en


reposo, ¿Qué tratamiento antihipertensivo inicia?"
e} Hidralazina

53d. Con el control de la crisis hipertensiva y remisión sintomática, usted decide:


b} Hospitalización e iniciar maduradores pulmonares

53e. Algunas pacientes requieren ingresar a la Unidad de Cuidados Intensivos, ¿cuál de los
siguientes no es una indicación para manejo en hospitalización?
d} Puerperio fisiológico

54. Se trata de un paciente masculino de 34 años de edad, pescador, que fue rescatado de
un naufragio hace 5 semanas; acude a consulta por presentar insomnio, pesadillas
recurrentes y repetitivas con sensación de ahogamiento, nerviosismo y crisis de pánico,
refiere que los síntomas se acentúan cuando va a trabajar y esto limita su actividad
laboral. Niega ingesta de alcohol,tabaco o drogas. La exploración física es normal. FC 75
lpm, FR 18 rpm, temp. 36.5ºC, PA 125/80 mm Hg, peso 75 kg, talla 1.70 m.
54a. Con los datos proporcionados por el paciente, usted considera que está cursando
con:
b} Trastorno de estrés postraumático
54b. Para realizar el abordaje, usted debe descartar los diagnósticos dieferenciales, ¿cuál
de los siguientes no es un diagnóstico diferencial del trastorno de estrés postraumático?
c} Trastorno de conversión

54c. Establecido el diagnóstico usted decide que la psicoterapia debe ser:


c} Terapia cognitiva conductual

54d. El tratamiento farmacológico indicado en este problema es:


a}lnhibidores selectivos dela recaptura de serotonina

54e. Los siguientes factores son de buen pronóstico, excepto:"


d} Evento traumático causado por humanos (violación,guerra...)

55. Los pacientes con una fractura de cadera y que acuden a tiempo al hospital, tienen un
riesgo de 6% de trombosis venosa profunda. ¿Cuál es el riesgo de trombosis venosa
profunda para los pacientes que tardan más de 48 horas en ser hospitalizados?
c} 55%

56. Se trata de un paciente masculino de 27 años de edad, sin antecedentes relevantes,


vive en una región tropical y es obrero en una construcción; refiere que hace 2 a 3
semanas presenta lesiones eritematosas, maceración, muy pruriginosas a nivel interdigital
del pie izquierdo, refiere uso de calzado cerrado; a la exploración física se encuentra lesión
de eritema, con bordes bien delimitados, datos de queratinización por el rascado, así
como descamación de predominio interdigital. FC 75 lpm, FR 16 rpm, temp. 36.5 ºC, PA
120/70 mm Hg, peso 62 kg, talla 1.68 m.
56a. Con los datos clínicos y la exploración física, ¿qué diagnóstico integra?
a} Dermatofitosis

56b. Según las características de este paciente ¿cuál es el agente causal más común?
d} Tricophyton rubrum

56c. ¿Cuál es el método diagnóstico de elección?


b} Microscopia con hidróxico de potasio en escamas

56d. Una vez determinado el diagnóstico ¿qué tratamiento inicia?


e} Ketoconazol

56e. Una vez establecido el tratamiento ¿qué medida preventiva no da al paciente?


d}Inmersión de pies en agua de Alibour
57. Un recién nacido es evaluado después del nacimiento y se encuentran los siguientes
hallazgos en el examen físico: brazo izquierdo en aducción y rotación interna, reflejo de
Moro ausente y reflejo de presión palmar presente en mano izquierda. ¿Cuál de las
siguientes condiciones NO es factor de riesgo para la lesión que tiene el paciente?
d} Prematurez

58. Paciente femenino de 20 días de vida, recibe traumatismo nasal al caerse de los brazos
de su madre. Se observa edema importante de dorso nasal, no se palpa crepitación de
huesos propios; en la rinoscopia se observa deflexión septal obstructiva. La siguiente
conducta sería:
c} Reducción cerrada

59. Niño de cuantro años que refiere dolor intenso en codo derecho. En el examen físico
se encuentra limitación a flexión, pronación completa del antebrazo y dolor intenso sobre
la cabeza radial, sin datos de inflamación a este nivel. Limita el uso del brazo derecho
soportando su peso con el brazo izquierdo. Se obtiene una radiografía del codo que
aparece normal.
Se realiza maniobra de supinación completa del codo y se lleva de flexión completa a
extensión completa; con lo cual se palpa un clic a nivel de la cabeza radial y a los 15
minutos el paciente ya moviliza el brazo derecho de forma adecuada y niega dolor.
59a. ¿Cuál es el diagnóstico más probable?
a} Subluxación de la cabeza radial

59b. ¿Cuál es el mecanismo de trauma de la lesión que tiene el paciente?


e} Tracción axial delbrazo

59c. ¿Cuál es el tratamiento más adecuado?


b} Maniobra de reducción manual

60. Paciente femenino de 31 años de edad con gesta 2, para 2, sexualmente activa y con
uso de dispositivo intrauterino desde hace cuatro años. El motivo de la consulta es por
dolor abdominal y pélvico bilateral que aparece desde hace tres días, con sensación de
pesantez pélvica. La sintomatología inició al día siguiente después de su última
menstruación. El día de hoy la paciente ha presentado náusea y cefalea. A la exploración
física la paciente se encuentra levemente taquicárdica, presión arterial dentro de rangos
normales, temperatura de 39 ºC. Reporta dolor a la palpación de la región abdominal
pélvica. La exploración ginecológica revela secreción vaginal purulenta y dolor extremo a
la movilización cervical; no se palpan masas en anexos. La biometría indica ligera anemia y
leucocitosis de 14 000 células/µL. El ultrasonido transvaginal demuestra septaciones de la
pared tubaria (signo de la rueda dentada). Se realiza prueba de embarazo en orina la cual
resulta negativa. ¿Cuál es el diagnóstico más probable?
a} Peritonitis-salpingitis aguda
61. Masculino de 40 años de edad acude al servicio de Urgencias por haber evacuado en la
madrugada y el día de hoy cinco veces. Refiere que las evacuaciones fueron de
consistencia líquida,y que ha presentado dolor tipo cólico, así comotres vómitos de
importante cantidad y contenido líquido. Este padecimiento lo relaciona el paciente con
haber ingerido esquites de un puesto fuera del metro el día de ayer por la noche. Niega
haberse automedicado. A la exploración, usted lo encuentra con franco dolor abdominal y
con temperatura de 38.SºC.
61a. ¿Cuál es el síndrome clínico principal en este paciente?
c} Diarreico agudo

61b. El principal origen de este síndrome es:


c}Infeccioso en 90% de las veces

61c. ¿Cuáles son los principales agentes causales de este cuadro (diarrea adquirida en la
comunidad)?
e} Salmonella, Campy/obacter, Shigella, E. Co/i y Cryptosporidium

61d. Por el antecedente de ingesta de esquites (mayonesa), usted piensa que el


responsable es S. aureus.¿Cuál es el mecanismo de daño de este agente?
b} Neurotoxina preformada

61e. El primer paso en el tratamiento de este paciente es:


c} Hidratación

62. Paciente masculino de 35 años de edad con diagnóstico reciente de leucemia


linfoblástica aguda, acude al servicio de urgencias por presentar náusea, vómito, diarrea,
debilidad muscular, calambres y parestesias. Los resultados de laboratorio revelan
hiperpotasemia, hipocalcemia, hiperfosfatemia e hiperuricemia, por lo que se le
diagnostica síndrome de lisis tumoral.
62a. ¿A que medida terapéutica se ha asociado este síndrome?
 e} Todas las anteriores

62b. ¿Cuáles son los factores de riesgo para presentar síndrome de lisis tumoral?
e} Todas las anteriores

62c. Las neoplasias con mayor riesgo de presentar síndrome de lisis tumoral son:
a} Leucemia linfoblástica aguda, linfoma de Burkitt y cáncer de mama

62d. El tratamiento inicial en el síndrome de lisis tumoral consiste en:


c} Hidratación intravenosa con solución salinaisotónica a 2 500 a 3 000 ml/m2/24 h (
63. Masculino de 20 años es enviado al servicio de urgencias acompañado por sus
familiares, portador de diabetes mellitus desde los 13 años contratamiento irregular con
dieta e insulina de acción intermedia. Refieren los familiares que en últimas fechas ha
presentado astenia, adinamia, anorexia, náusea y vómito de contenido gastrobiliar;
poliuria, dolor abdominal difuso de tipo punzante localizado en mesogastrio. En los dos
últimos días ha estado ingiriendo bebidas alcohólicas y suspendió las inyecciones de
insulina. Se inicia el ingreso del paciente en estado estuporoso evidente.
63a. Los hallazgos de laboratorio en este paciente son:
b} Hiponatriemia

63b. El estado metabólico causal de la sintomatología de este paciente es:


b} Cetoacidosis diabética

63c. Es el fenómeno fisiopatológico que suele presentarse en estos casos:

b} Exceso de glucagón en plasma

64. Masculino de 30 años que consulta por dolor en el centro de la cara,el cual se
acompaña de obstrucción nasal, rinorrea mucopurulenta, descarga retronasal, hiposmia,
halitosis, tos y ataque al estado general,de 7 días de evolución. TA: 140/85, FC: 85 lpm, FR:
18x ', Temp: 38ºC. A la exploración física se observa edema e hiperemia de la mucosa
nasal, rinorrea purulenta y cornetes inferiores hipertróficos, así como descarga retronasal
amarillenta.
64a. ¿Cuálsería su sospecha diagnóstica?

e} Sinusitis

64b. ¿Cómo se clasifica esta enfermedad?


c} Aguda, subaguda y crónica

64c. Causamás común de este padecimiento:


c} Streptococcus pneumoniae

64d. Estándar de oro para el diagnóstico:


a} TC

64e. El tratamiento más adecuado para el paciente en cuestión es:


d} Amoxicilina
65. Paciente femenino de 22 años, con antecedente de 2 embarazos y 2 partos, trastorno
bipolar, con un hijo de seis años que moja la cama y un recién nacido de 15 días es traída a
urgencias tras ingerir múltiples pastillas en un intento de suicidio. Se encuentra
letárgica,con frecuencia cardiaca de 120 latidos por minuto, presión arterial 90/60 mm Hg
y empieza a convulsionar súbitamente. El monitor cardiaco muestra ensanchamiento
progresivo del QRS y prolongación del intervalo QT. Si los síntomas del paciente se deben
a intoxicación por un medicamento, ¿cuál es el fármaco que la paciente ingirió?"
a} lmipramina

66. Masculino de 65 años de edad acude a consulta por haber observado sangre en sus
heces el día de ayer. Comenta que desde hace tiempo presenta cambios en la consistencia
de las heces y le llama la atención que haya bajado de peso en los últimos días. Como
antecedentes de importancia, tiene un hermano que falleció hace ya 1O años a causa de
cáncer, al parecer de colon. No se sabe enfermo de nada ni toma medicamentos; su dieta
es mayormente carne y acostumbra beber vino diario. TA: 130/80, FC: 95 lpm, FR: 23x ',
Temp: 37.5ºC."
66a. ¿Qué marcador tumoral esperaría encontrar elevado?
b} Antígeno carcinoembrionario

66b. Lugar más frecuente de tumores del intestino grueso.


c} Sigmoides

66d. La preparación colónica para cirugía se debe hacer con:


d} Polietilenglicol

66e. ¿Cuáles de éstos son factores de riesgo para cáncer de colon?


d} a y c

67. Paciente femenina de 68 años cursa posoperatorio de colectomía por cáncer de colon.
Inicia con disnea súbita. TA: 70/50 mm Hg, FC: 108 lpm, Sa02: 93%, ruidos cardiacos
rítmicos con soplo diastólico de predominio en segundo espacio paraesternal izquierdo.
Campos pulmonares con discreta hipoventilación paraescapular derecha. Se realiza
ecocardiografía que reporta hipocinesia de los segmentos medio y basal de la pared libre
del VD.
67a. Según la Escala de Wells, ¿cuál es la probabilidad clínica de embolia pulmonar de la
paciente?
d} Alta

67b. Estudio indicado para continuar con el diagnóstico de la paciente.

e}Angiotomografía
67c. Hallazgo electrocardiográfico que se esperaría encontrar en la paciente.
a} Taquicardia sinusal

67d. Tratamiento indicado en esta paciente.


e} Embolectomía

67e. En relación con el tratamiento farmacológico en la embolia pulmonar,señale la


respuesta correcta:
a} El periodo de ventana parala fibrinólisis es de 2 semanas.

68. Joven de 23 años acude a consulta por presentar desde el día de ayer dolor y aumento
de tamaño en la rodilla derecha, sin antecedente de traumatismo. El dolor le impide
mover libremente la extremidad derecha. A la exploración física se le encuentra afebril,
con aumento de la temperatura y rubor en la zona afectada.
68a. ¿Cuál sería su impresión diagnóstica?
b} Artritis séptica gonocócica

68b. El agente más común de esta entidad en menores de 2 meses es:


c} S. aureus

68c. ¿Cuál de las siguientes opciones NO es considerada un factor de riesgo?


b} Múltiples parejas sexuales

68d. Método diagnóstico de elección.


c} Punción sinovial y análisis de líquido sinovial (gram y cultivo}

68e. El tratamiento de elección para este paciente es:


d} Ceftriaxona con tratamiento ala pareja

69. Paciente femenino de 67 años consulta por disminución en la agudeza visual. Tiene
antecedentes de DM2 y HTA desde hace 15 años, por lo que toma metformina 500mg
cada 12h y captopril 50 mg cada 12 h; señala que ha notado una pérdida mayor desde que
le indicaron insulina, la cual utiliza irregularmente. Al realizarle examen de fondo de ojo,
usted observa microaneurismas y algunos exudados de color amarillo.
69a. ¿Cuálsería su sospecha diagnóstica?
c} Retinopatía diabética

69c. Por los hallazgos en el examen de fondo de ojo, usted clasificaría la retinopatía
diabéticade esta paciente en:
c} No proliferativa moderada
69d. ¿Cuál de las siguientes condiciones sería indicativa de tratamiento con
fotocoagulación en la paciente?
a} Edema macular

69e. ¿Cuáles son las principales causas de disminución en la agudeza visual dela paciente?
d} Edema macular,hemorragia vítrea y desprendimiento de retina

70. En un paciente con DM mal controlada,que presenta fiebre, presión arterial de 92/60,
frecuencia cardiaca de 104, leucocitos de 15 000, hemoglobina de 11.8 y plaquetas de 65
000, ¿cuál es un criterio de sepsis grave?
d} La trombocitopenia

71. De las siguientes vacunas. ¿Cuál es de virus vivos atenuados?


d} Sarampión/parotiditis/rubéola

72. Durante el estrés agudo se pierde inicialmente la siguiente función suprarrenal:


a} Variabilidad circadiana y pulsatilidad de ACTH

73. Paciente femenino de 35 años de edad, G1 PO CO AO con embarazo de 32 SDG, con


diagnóstico de preeclampsia. Es llevada por su esposo a urgencias porque desde hace 2
horas inició con astenia, adinamia, palidez y cefalea holocraneana pulsátil con intensidad
de 9/1O; y epistaxis que no cesaba, posteriormente se encontraba somnolienta. El esposo
niega que presentara acúfenos y fosfenos. A la exploración física, presenta TA 135/85, FC
115, FR 20, T 38 C, Sa02 94%, somnolienta,con palidez de mucosas y tegumentos, sin
epistaxis, con máculas violáceas en tronco, abdomen y extremidades, que no desaparecen
a la digitopresión, con debilidad generalizada. Campos pulmonares bien ventilados, sin
agregados, ruidos cardiacos rítmicos, adecuada intensidad, sin soplos. Abdomen globoso,
fondo uterino a 31 cm de la sínfisis del pubis, situación longitudinal,presentación cefálica,
posición derecha, FCF 160. Giordano negativo. Laboratorios: Hb 9 g/dL, Hto 27%, VCM 90
fL,CMHC 32%, leucocitos 11 OOO/mm3, bandas 5%, plaquetas 50 OOO/mm3, reticulocitos
3%, en el frotis periférico se observan esquistocitos, glucosa 118 mg/dL, BUN 50 mg/dL, Cr
2 mg/dL, Na 135 mg/dL, K 4 mg/dL, CI 100 mg/dL, DHL 400 mg/dL,TP 15 segundos (s),"
"TTPa 40 s, ALT 60 U/L,AST 50 U/L, BT 3 mg/dL, BD O.7 mg/dL, haptoglobina indetectable.
EGO: color oscuro, proteinuria ++, densidad 1.01O, glucosa, cetonas negativo,
hemoglobina positivo, leucocitos 5 por campo.
73a. ¿Cuál es el diagnóstico más probable de esta paciente?
d} PTT

73b. ¿Cuál es la característica más apropiada de esta patología?

c} Alteración de ADAMTS13
73c. Se realiza un registro cardiotocográfico el cual reporta una FCF de 164 lpm, con
variabilidad de 5 a 1O, con ascensos ausentes y desaceleraciones tardías o DIP tipo 11.
¿Cuál de los siguientes parámetros no es parte del perfil biofísico modificado?
e} Antropometría

73d. Se realiza un USG Doppler observando alteraciones en las resistencias de la arteria


cerebral media fetal. ¿Qué dato ultrasonográfico fetal es ominoso y se asocia a muerte
fetal en 50%?
e}Inversión del flujo telediastólico

74. Ante un recién nacido de 16 días de vida que presenta distensión abdominal y
evacuaciones sanguinolentas, se solicita radiografía abdominal observándose neumatosis
intestinal y patrón en miga de pan. De acuerdo al diagnóstico que usted sospecha, ¿cuál
de las siguientes opciones terapéuticas está indicada?"
d} Antibióticos

75. Masculino de 72 años de edad es recibido en la sala de urgencias de la unidad


hospitalaria donde usted trabaja con Dx. de infarto agudo de miocardio.
El marcador de mayor sensibilidad para confirmar el diagnóstico de este paciente es:
c} Troponina

76. Paciente femenino de 30 años de edad. Antecedente materno de hipoacusia. Refiere


hipoacusia derecha progresiva de cuatro años de evolución acompañada de acúfeno,
niega antecedente de trauma o infección en el oído. Diapasones: Weber derecho, Rinne
invertido derecho, positivo izquierdo.
76a. El diagnóstico más probable es:
c} Otoesclerosis

76b. El dato clínico en la exploración física que apoyaría el diagnóstico sería:


c} Aumento de la vascularidad en promontorio

76c. La fisiopatología se debe principalmente a:


c) Remodelación ósea de la cápsula ótica

76d. El tratamiento consiste en:


c} Estapedectomía

77. Masculino de 42 años se encuentra en sala de hospitalización. Antecedentes: refiere


malos hábitos alimentarios, comenta que con frecuencia consume alimentos en la vía
pública. Exploracion física: fiebre no cuantificada, edema facial, fotofobia y mioartralgias.
Con los siguientes datos clínicos el diagnóstico de este paciente es:
b} Trichinellosis
78. ¿Cuál es el nombre de la enfermedad producida por deficiencia de la enzima alfa-
cetoácido deshidrogenasa?
d} Enfermedad de orina de jarabe de maple

79. ¿Cuál de las siguientes translocaciones cromosómicas se asocia con el linfoma de


Burkitt?

b} t(8:14}

80. Masculino de 6 años es llevado al servicio de urgencias por salida de áscaris por el ano.
Exploración física: se detecta dificultad y sibilancia respiratoria,abdomen con
hepatomegalia y distensión abdominal,se confirma el diagnóstico por USG. El tratamiento
de elección en este paciente es:
d} Albendazol vía oral

81. Un paciente informa que al mirarse en el espejo ha notado que el lado izquierdo de su
cara parece ""caerse"". También menciona que necesita utilizar el teléfono sólo del lado
derecho, ya que no puede oír por el lado izquierdo. A la exploración física presenta
sordera neurosensorial en el oído izquierdo y no puede cerrar los párpados del lado
izquierdo. Cuando se le solicita al paciente que tome un vaso de agua,el líquido escurre
por la comisura izquierda de la boca. El paciente presenta incapacidad para fruncir la
mitad izquierda de la frente. ¿Cuál es la localización anatómica más probable de la lesión
en este paciente?"
c} Meato auditivo interno

82. Paciente masculino de 45 años de edad, trabajador de construcción,el cual es llevado


al servicio de urgencias por la presencia de contracciones musculares paroxísticas
involuntarias, las cuales afectan principalmente mandíbula, cuello y tórax. ¿En dónde se
encuentra principalmente la sustancia tóxica que está ocasionando la sintomatología del
paciente?
b} Médula espinal

83. Femenino de 39 años acude a servicio de consulta externa refiriendo antecedentes de


amenorrea secundaria de 2 años y medio de evolución. Portadora de resultados de
laboratorio reportando: prolactina 150 ng/mL (normal hasta 20 ng/mL), resonacia
magnética detecta macrotumor de 2.8 cm de diámetro con expansión lateral izquierda.
¿Cuál es el tratamiento más adecuado para

b} Tx. médico con agonistas dopaminérgicos


84. Paciente de 35 años de edad que es admitido al servicio de urgencias durante el
invierno debido a disnea. El paciente reporta que se encontraba solo en su casa
trabajando en la reparación de su automóvil cuando súbitamente inició con disnea. A la
exploración física el paciente presenta una frecuencia respiratoria de 30
respiraciones/minuto, con una coloración rosada en la cara; el paciente se muestra
ansioso y con dificultad respiratoria evidente. La radiografía de tórax no demuestra alguna
anormalidad. La saturación de oxígeno es de 100%. Se realiza un electrocardiograma el
cual solo revela taquicardia sinusal. ¿Cuál es el diagnóstico más probable en este
paciente?
e} Intoxicación por monóxido de carbono

85. ¿Cuál de los siguientes es un factor de riesgo para el desarrollo de linfoma gástrico?
a}Infección por Helicobacter py/ori

86. Paciente masculino de 1O años de edad que se presenta con tres días con fiebre, dolor
abdominal y diarrea. Los padres del paciente mencionan que el perro de la familia
también ha presentado diarrea. El paciente no tiene antecedente alguno de importancia y
al parecer su esquema de vacunación está actualizado hasta el día del padecimiento
actual. La realización de un estudio coproparasitoscópico no reveló presencia de huevos o
parásitos. ¿Cuál de los siguientes organismos es con mayor probabilidad el agente causal
de diarrea y fiebre en este paciente?
b} Campy/obacter

87. Paciente de 22 años de edad que acude a la consulta externa debido a que necesita
una evaluación de rutina para obtener un certificado médico. El paciente no refiere ni
nguna molestia y la exploración física es irrelevante. Dentro de los estudios de rutina,
como única anormalidad se encontró elevación de la bilirrubina (2.6 mg/dL). ¿Cuál de las
siguientes opciones representa la causa más probable de la hiperbilirrubinemia en este
paciente?
e} Síndrome de Gilbert

88. ¿Cuál es el marcador serológico más específico de artritis reumatoide y que es capaz
de diferenciar la artritis reumatoide de otras enfermedades?
c} Anticuerpos antipéptido cíclico citrulinado (anti-CCP

89. ¿Cuál de las siguientes condiciones se asocia con un primer ruido con intensidad
incrementada?
d} Síndrome de Wolff-Parkinson-White

90. Paciente del sexo masculino de tres semanas de edad el cual se presenta al servicio de
urgencias traído por su madre con fiebre alta, disminución del consumo de leche materna
y llanto durante las últimas 6 horas. Como parte de los procedimientos rutinarios del
hospital se decide tomar una punción lumbar la cual revela una meningitis bacteriana.
¿Cuál es la causa más frecuente de meningitis bacteriana en pacientes dentro de este
grupo de edad?
d} Estreptococo del grupo B, Escherichia coli y Listeria

91. ¿Cuál es el porcentaje de compromiso renal en artritis reumatoide?


a} De 1.5 a 9%

92. Paciente de 39 años de edad se presenta con un derrame pleural transudativo en el


hemitórax derecho. ¿Cuál de los siguientes hallazgos clínicos es menos probable que se
observe en este paciente?
c} Hiperresonante ala percusión del hemitóraxizquierdo

93. Paciente masculino de 23 años de edad que es llevado al hospital después que en una
cantina se involucró en una riña en la que sufrió una herida por arma punzocortante en el
lado superolateral de la pared torácica izquierda a nivel de la tercera costilla. El paciente
presenta sangrado leve y ausencia de dificultad respiratoria. A la exploración neurológica
se revela que el borde medial de la escápula en el lado izquierdo se encuentra desviado
cuando se levanta el brazo; entre otras anormalidades el brazo no puede ser abducido
más allá de la posición horizontal. ¿Cuál de los siguientes músculos es el que con mayor
probabilidad se encuentra lesionado en este paciente?

b} Serrato anterior

94. Paciente masculino de 23 años de edad que es llevado al hospital después de que en
una cantina se involucró en una riña en la que sufrió una herida por arma punzocortante
en la parte superolateral de la pared torácica izquierda a nivel de la tercera costilla. El
paciente presenta sangrado leve y ausencia de dificultad respiratoria. A la exploración
neurológica se revela que el borde medial de la escápula en el lado izquierdo se encuentra
desviado cuando se levanta el brazo; entre otras anormalidades el brazo no puede ser
abducido más allá de la posición horizontal. ¿Cuál es el nervio lesionado en este paciente?

d} Torácico largo

95. Paciente de 70 años con antecedente de enfermedad pulmonar obstructiva crónica


grave con administración de oxígeno en casa, se presenta a la sala de urgencias por
antecedente de tres días de disnea progresiva y tos productiva purulenta. Jamás ha sido
hospitalizado por una exacerbación de su enfermedad de base. A la inspección se tiene un
paciente anciano con dificultad respiratoria moderada a grave, que le impide pronunciar
oraciones completas con obnubilación. La exploración torácica revela campos pulmonares
con disminución de los ruidos respiratorios y sibilancias espiratorias. El médico interno
toma una gasometría la cual revela pH de 7.25, PaC02 de 70, Pa02 de 50. La radiografía de
tórax demuestra hiperinflación de los campos pulmonares sin infiltrados o neumotórax. Se
inicia tratamiento con oxígeno (mascarilla tipo Venturi con Fi02 de 0.50),
broncodilatadores, antibióticos y corticosteroides. Después de 50 minutos, el paciente se
presenta con mayor deterioro del estado de conciencia y la gasometría actual revela pH
de 7.15, PaC02 de 100 y Pa02 de 70. ¿Cuál es el mejor tratamiento en este paciente?

c} Intubar al paciente e iniciar ventilación mecánica

96. ¿Cuál de las siguientes estructuras forma el límite lateral y medial del triá ngulo
femoral?
d} Músculo aductor largo y sartorio

97. Durante la atención del recién nacido es fundamental estar preparado para casos en
que se presente depresión que amerite maniobras de reanimación. En el caso de un recién
nacido de 30 semanas de edad gestacional,que respiró espontáneamente al nacer pero
muestra signos de dificultad respiratoria y flacidez muscular,los procedimientos INICIALES
a realizar como parte de la reanimación consisten en:

d} Evitar pérdida de calor,extender el cuello, aspirar secreciones, secar y estimular


táctilmente

98. Paciente femenino de 18 años acude a consulta por embarazo sin conocer las semanas
de gestación. Al interrogatorio no refiere datos de importancia. Usted decide calcular las
semanas de gestación y la fecha probable de parto, sabiendo que el primer día de la
última menstruación fue el 18 de marzo de 2008 y hoy es septiembre 14 de 2008. ¿En qué
semana de gestación está y cuál es la probable fecha de parto?

c} 25 de diciembre de 2008 y 25 semanas de

99. Mujer gestante secundigesta con antecedente de parto pretérmino a la semana 35,
acude a consulta en la semana 32 de embarazo por la percepción de contracciones. A la
exploración física se detecta cuello uterino permeable, borramiento de 20%, frecuencia
cardiaca fetal de 145 latidos por minuto y contracciones uterinas cada 6 minutos. Se
realiza un ultrasonido, se observa un cuello uterino de 3.5 cm de longitud, fibronectina
negativa. ¿Qué medicamento utilizaría para incrementar la maduración pulmonar del
producto?
a} Betametasona 12 mg cada 24 horas, dos dosis
1OO. Recién nacido de término de 1O días de vida que se observa con párpados y labios
tumefactos, hipertelorismo, macroglosia, ictericia desde el nacimiento, letargia,fontanela
anterior abombada. ¿Qué resultados en las pruebas de tamizaje y confirmatorias apoyan
el diagnóstico de la enfermedad que sospecha?

a} Niveles de TSH > 25 microunidades internacionales en sangre de talón,TSH sérica >5


microunidades internacionales y T4 sérica > O.7 nanogramos por decilitro

101. Embarazada que se encuentra en primer periodo del trabajo de parto. Contracciones
uterinas cada 3 min con 50 s de duración, con 8 cm de dilatación, 90% de borramiento,
cuello uterino blando, anterior, con membranas íntegras. ¿En qué momento se inicia la
fase activa del primer periodo de trabajo de parto?

d} Cuando elcuello uterino tiene una dilatación de 3 cm

102. Hombre de 24 años, que regresó hace cuatro días de un viaje al estado de Veracruz,
inició con datos de irritación ocular. Después, de manera súbita presentó fiebre, cefalea,
dolor retroocular y faríngeo, así como mialgias y artralgias graves. Refiere "dolor
insoportable en la espalda". A la exploración física, se encuentra presión arterial de
124/82, taquicardia (98 latidos/min), frecuencia respiratoria de 20, temperatura de 39.5 C,
un exantema macular en tronco y extremidades y adenopatías submandibulares, poco
dolorosas.
102a. ¿Cuál es el diagnóstico más probable?
b} Dengue

102b. El agente causal de esta enfermedad es:

e} Un flavivirus

102c. El vector que transmite este microorganismo es:

a} Aedes aegypti

102d. El tratamiento indicado en este caso es:


a} Tratamiento de soporte, conlíquidos y medidas sintomáticas

102e. Con respecto a esta enfermedad, es cierto que:


b) El vector se encuentra cerca de agua almacenada (jarrones,llantas viejas,etc.)

103. ¿Cuál de los siguientes factores conlleva un pronóstico desfavorable para la aparición
o (progresión) de insuficiencia renal sintomática en una mujer de 29 años con DM tipo 1
desde los 14 años?
a} Albuminuria de 0.12 a 0.17 g/día en tres ocasiones diferentes

104. Varón de 40 años que presenta una dermatosis localizada en la región


perianal,constituida por una neoformacion exofítica en forma de coliflor, de aspecto
vegetante que usted diagnostica como condiloma acuminado. ¿Cuáles son los serotipos
más frecuentes en este padecimiento?

b} 6 y 11

105. ¿En qué pacientes estaría contraindicado el uso de insulina en un esquema intensivo
de tratamiento?
a} Pacientes con retinopatía progresiva

106. El tiempo de duración de acción promedio de la insulina NPH es de:

d} 12 a 15 horas

107. Varón de 57años de edad que refiere como único antecedente personal patológico
de importancia hipermetropía de +5.00 Dp, migraña con aura, acude a consulta por
presentar dolor de cabeza tipo pulsátil, especialmente a nivel de ojo derecho y visión
borrosa intermitente desde hace dos días, en los que ve luces con halos de colores. La
mañana de hoy tuvo además náusea y vómito. Tips: las molestias comenzaron después de
acudir al cine. A la exploración oftalmológica, el ojo derecho del paciente con
enrojecimiento, pupila fija y dilatada, no cuenta sus dedos a corta distancia pero percibe
luz y el movimiento de su mano frente a ese ojo.
¿Cuál es el diagnóstico más probable?

a} Glaucoma agudo de ángulo estrecho

108. Paciente femenino de 28 años de edad con diagnóstico de embarazo de 30 semanas


de gestación. Acude a urgencias llevada por su madre, porque presenta hemorragia
transvaginal intensa color rojo marrón, la cual no ha cedido; refiere astenia, adinamia,
anorexia, náusea y mareo y dolor tipo cólico en el abdomen,el cual ha ido incrementando
en intensidad y se irradia a región lumbar; comenta que no percibe movimientos fetales
desde que inició el sangrado hace 2 horas. Como antecedentes de importancia: gesta 3,
partos 1,cesárea O, aborto 1. Refiere que hace un mes fue diagnosticada de preeclampsia.
A la exploración física se encuentra: frecuencia cardiaca: 112 latidos por minuto,
frecuencia respiratoria: 18 respiraciones por minuto, temperatura 37 C. Se observa pálida,
diaforética, cardiopulmonar sin alteraciones. Abdomen globoso a expensas de útero
gestante hipertónico. Fondo uterino doloroso a la palpación, con 32 cm de distancia desde
sínfisis del pubis, situación longitudinal,presentación cefálica, dorso a la izquierda,
frecuencia cardiaca fetal de 120 latidos por minuto.

108a. Esta enfermedad se caracteriza por no presentar uno de los siguientes factores de
riesgo. ¿Cuál es?.

b} Cicatrices uterinas previas


108b. ¿Cuál NO es una complicación de este tipo de patología?
d}Insuficiencia cardiaca

108c. ¿Cuál es la causa por la que puede haber hemorragia oculta en esta patología?
e} Todaslas anteriores

109. Paciente masculino de 29 años de edad con dolor bilateral en las rodillas. Al realizar
la historia clínica menciona que todos los días trota una distancia de 8 a 1O km. ¿Cuál es la
segunda causa más probable del dolor de rodillas en corredores?
b} Síndrome de la banda iliotibial

110. En condiciones normales, qué porcentaje de T4 libre y T4 unido a proteínas se


encuentra al medir una determinación de T4 total?
c} 99% unido a proteínas/1% libre

111. Paciente masculino de 31 años de edad con antecedente de dos semanas de fiebre, a
lo cual se agregó hace una semana distensión abdominal,estreñimiento y dolor
epigástrico. Entre otras molestias también refiere cefalea, escalofríos (calosfríos), mialgias
y malestar general. La exploración física revela la presencia de un ""exantema rosado""
(exantema maculopapular color salmón que blanquea a la presión), principalmente en el
tórax, así como dolor abdominal generalizado a la palpación y esplenomegalia. Presenta
temperatura de 38.9 ºC, frecuencia cardiaca de 45 latidos por minuto, frecuencia
respiratoria de 20 respiraciones por minuto, presión arterial de 100/65 mm Hg. Los
estudios de laboratorio básicos son relevantes por leucopenia y neutropenia. ¿Cuál es el
sitio más frecuente de perforación en este paciente?
d} lleon

Paciente masculino de 31 años de edad con antecedente de dos semanas de fiebre, a lo


cual se agregó hace una semana distensión abdominal,estreñimiento y dolor epigástrico.
Entre otras molestias también refiere cefalea, escalofríos (calosfríos), mialgias y malestar
general. La exploración física revela la presencia de un ""exantema rosado"" (exantema
maculopapular color salmón que blanquea a la presión), principalmente en el tórax, así
como dolor abdominal generalizado a la palpación y esplenomegalia. Presenta
temperatura de 38.9 ºC, frecuencia cardiaca de 45 latidos por minuto, frecuencia
respiratoria de 20 respiraciones por minuto, presión arterial de 100/65 mm Hg. Los
estudios de laboratorio básicos son relevantes por leucopenia y neutropenia.
¿Cuál es la prueba diagnóstica con mayor sensibilidad a pesar de tratamiento antibiótico?
b) Cultivo de médula ósea

112. El tratamiento inicial del hipertiroidismo ocasionado por una tiroiditis subaguda
consiste en:
a} Antiinflamatorios no esteroideos y betabloqueadores

113. Paciente masculino de 81 años de edad se presenta a la consulta externa por


segunda ocasión en 15 días por tos productiva. Entre los antecedentes de importancia se
destaca el consumo de aproximadamente 20 cigarros al día durante 50 años; a pesar de
que se le ofreció consejo antitabaco en varias ocasiones, siempre fue rechazado. Durante
24 años el paciente sufrió exposición laboral a diferentes tóxicos. Diagnóstico desde hace
cuatro años de enfermedad pulmonar obstructiva crónica. Menciona que recientemente
ha presentado pérdida de peso y disnea en reposo. A la exploración física revela
frecuencia cardiaca de 11O latidos por minuto y datos en relación con fibrilación auricular,
presión arterial de 155/70. Se solicita radiografía de tórax la cual demuestra una imagen
de consolidación mal definida en el lóbulo pulmonar medio, así como infiltrado difuso
bilateral. Posteriormente, se solicita tomografía computarizada de tórax la cual revela
enfisema centroacinar,fibrosis (parénquima afectado y engrosamiento de septos), así
como masa en lóbulos pulmonares medio e inferior derecho de 9 x 7 x 6 cm, así como
múltiples adenopatías. ¿Cuál es la estirpe histológica más frecuente del cáncer de
pulmón?
d} Adenocarcinoma

114. Una paciente de 45 años es llevada a la sala de urgencias debido a que ha tropezado
y se ha torcido el tobillo al utilizar zapatos de tacones. ¿Cuál de los siguientes ligamentos
es el que más probablemente se encuentra afectado en esta paciente?
d} Ligamento lateral
115. Un paciente de 40 años de edad, arquitecto sin antecedentes médicos de
importancia, acude a la consulta por dolor lumbar intenso que inicio súbitamente cuando
intentó cargar unos costales con cemento. El paciente describe el dolor como punzante
con radiación hacia la pierna y pie izquierdos. Al interrogatorio el paciente niega la
presencia de incontinencia fecal o urinaria. Se realiza la prueba de Lasegue la cual produce
dolor al realizar una flexión de la cadera de aproximadamente 50º con rodilla en
extensión. El reflejo del tobillo es de menor intensidad en el lado izquierdo que en el lado
derecho. Existe disminución de la sensibilidad en la porción lateral del pie izquierdo. ¿Cuál
de los siguientes hallazgos es más probable de encontrar también durante la exploración
física?
c} Debilidad para realizar flexión plantar del pie izquierdo

116. Se presenta a la sala de urgencias una paciente de 70 años por dolor punzante
epigástrico y hematemesis con sangre rojo rutilante. Al interrogatorio la paciente expresa
que desde hace varios años ha presentado pirosis y ocasionalmente consume antiácidos
sólo cuando el dolor es insoportable. Una vez estabilizada la paciente se realiza
esofagoduodenoscopia, la cual revela una úlcera sangrante en la porción posterior del
duodeno aproximadamente 3 cm distales al píloro. ¿Cuál de las siguientes fuentes es la
arteria más probablemente afectada?
e} Arteria gastroduodenal

117. En un paciente de cinco años de edad se solicita examen general de orina, en el que
se encuentran proteínas. Como parte del seguimiento se decide cuantificar la pérdida
proteínica por orina, lo cual resulta en una proteinuria mayor a 2 mg/dl con sedimento
lipoideo e hipoalbuminemia de 3 mg/dl. ¿Cuál es el hallazgo histológico más probable que
esté presente en una biopsia renal en este paciente?
b} Desaparición delos procesos periféricos de los podocitos observados en microscopia
electrónica

118. ¿Cuál de los siguientes estudios diagnósticos puede ser de utilidad para el diagnóstico
de la seudohiponatremia?
a} Medición de la osmolalidad sérica

119. Paciente femenino de 50 años, enfermera, la prueba de Mantoux siempre ha


resultado negativa año tras año. Este año la induración fue de 16 mm. Esta asintomática y
su radiografía de tórax son normales ¿Cuál sería la conducta más adecuada a seguir?
b} Profilaxis conisoniazida a 300 mg/día durante nueve meses

120. ¿Cuál de los siguientes diagnósticos es más probable en un paciente con hipertensión
arterial, hipopotasemia,alcalosis metabólica y disminución de las concentraciones de
renina y aldosterona?
b} Síndrome de Liddle
121. Paciente femenino de 58 años de edad acude a consulta por pirosis, fatiga, pérdida
de peso, palpitaciones, diarrea, eritema facial súbito y edema de tobillos desde hace seis
meses. Menciona que ha presentado desde hace varias semanas sensación de plenitud en
el cuello. No hay antecedentes de importancia, pues siempre ha sido sana. Al
interrogatorio niega el consumo de medicamentos. La exploración física revela pulsos
irregulares, distensión venosa yugular y un soplo holosistólico con variación de su
intensidad respecto a la respiración, y edema periférico. ¿Cuál es el diagnóstico más
probable de esta paciente?
c} Síndrome carcinoide

122. Femenino de 60 años de edad, que padece diabetes desde hace 10 años y lleva
tratamiento con metformina, consulta por episodios de reflujo y dolor al ingerir cualquier
tipo de alimento. La paciente comenta que sus molestias comenzaron hace una semana,
niega sensación de cuerpo extraño y no presenta cambios en la voz.
122a. El estudio inicial para este caso sería:
a} Esofagografía con bario

122b. Dentro de las causas de este trastorno se encuentran:


a} Pérdida de cuerpos neuronales, anticuerpos antimientéricos, disminución delpéptido
intestinal vasoactivo

122c. Se considera tratamiento de primera elección en mayores de 50 años.


c} Dilataciones endoscópicas y toxina botulínica

122d. La administración repetida de la toxina botulínica llega a ocasionar:


a} Fibrosis

122e. Son complicaciones de las dilataciones neumáticas:


c} Perforación y hemorragia

123. Paciente femenino de 52 años con obesidad morbida acude a urgencias por
presentar edema del miembro pelvico izquierdo de tres dias de evolucion que ha ido
aumentando progresivamente. La paciente refiere que hace tres dias comenzo con fiebre
despues noto edema y eritema en el pie izquierdo que ha ido ascendiendo hasta cubrir
toda la extremidad. El dia de ayer aparecio grandes ampollas que al romperse dejan salir
liquido claro. A la exploracion fisica usted nota temperatura de 38.5C, el miembro pelvico
izquierdo se observan con edema 4+ con godete, toda la extremidad es dolorsa a la
palpacion y esta cubierta por placas eritematosas de bordes elevados y bien definidos,
calientes al tacto con ampollas y vesiculas de las que sale un liquido claro. Los pulsos son
2/4 en ambas extremidades.
¿Cuál es el diagnostico mas probable?
d)Erisipela

1. ¿Cuál de los siguientes hallazgos se puede observar en un paciente con lesión del nervio
safena?
b} Alteración de la sensibilidad enla porción medial de la pierna

2. Se presenta a consulta paciente femenino de 14 años por amenorrea primaria y


ausencia de desarrollo glandular mamario. La madre refiere que el único antecedente de
importancia es disminución de la audición que requirió uso de un dispositivo especial. A
pesar de que la paciente fue producto de un embarazo normal y parto eutócico, la madre
recuerda que los médicos le dijeron que su hija estaba un poco "hinchada" al nacer. A la
exploración física la paciente muestra signos vitales normales, mide 1.50 m y pesa 60 kg. A
la inspección, se encuentra implantación baja de orejas y cuello alado. La exploración
cardiovascular revela un soplo sistólico grado 11/IV,con pezones extremadamente
separados y glándulas mamarias Tanner l. La exploración pélvica revela ausencia de vello
púbico y genitales externos en fase prepuberal. Se puede observar mediante un espéculo
la presencia de cuello uterino (cervix). ¿Cuáles otros hallazgos son probables de identificar
en esta paciente?
a} Anomalías renales, linfedema de extremidades, anomalías cardiovasculares (en
especial coartación de aorta

3. ¿Cuál de los siguientes medicamentos se utiliza en el tratamiento de la esclerosis


múltiple?
b} lnterferón beta

4. Paciente masculino de seis años de edad es llevado a consulta por sus padres debido a
un cuadro que inicia como un aparente resfriado con rinorrea, pero en este momento el
paciente presenta disnea intensa. Los padres informan que el paciente muestra notable
mejoría durante la mañana pero los síntomas empeoran conforme se acerca la noche. A la
exploración física se pueden auscultar estertores y sibilancias pero es imposible localizar
los ruidos cardiacos. La radiografía de tórax revela hiperinsuflación pulmonar,
bronquiectasias e inversión izquierda/derecha de varias estructuras del sistema
respiratorio. ¿Cuál de las siguientes complicaciones puede presentar este paciente?
e} Infertilidad

5. Durante la etapa fetal, ¿en cuál de las siguientes estructuras el porcentaje de saturación
de hemoglobina fetal es menor?
a} Ductus arteriosus
6. Paciente femenino de 28 años con diagnóstico de síndrome de Wolff-Parkinson-White
acude a urgencias por presentar palpitaciones de inicio súbito hace 45 minutos. A la
exploración física se encuentra angustiada y destaca ritmo irregular. El electrocardiograma
muestra fibrilación auricular con respuesta ventricular rápida con una frecuencia de 170
latidos por minuto.
¿Cuál es el tratamiento más adecuado para la paciente?
e} Procainamida

7. Paciente del género masculino de 35 años de edad con antecedente de infección por el
virus de la inmunodeficiencia humana (diagnosticado desde hace 3 años). El paciente no
desea la administración de antirretrovirales, pues cree que su enfermedad es un "invento
del gobierno y de las compañías farmacéuticas". Actualmente, en una revisión de rutina, el
paciente refiere una "erupción" que apareció hace 24 horas en la región lumbar y en la
región posterior de la pierna derecha. El paciente refiere las lesiones como dolorosas (tipo
"ardor"). Entre la información obtenida en el interrogatorio, el paciente mencionó que fue
ascendido en su trabajo recientemente, pero que esto lo ha estresado demasiado pues
ahora tiene mayores responsabilidades. A la inspección de la lesión se observa un
exantema formado por vesículas sobre una base eritematosa ("gotas de rocío sobre
pétalos de rosa roja").Utilizando únicamente la información previamente descrita,
responda las siguientes preguntas:
7a. ¿Cuál de las siguientes opciones representa la etiología más probable del exantema en
este paciente?
d} Virus herpes humano 3

7b. ¿Cuál de las siguientes opciones representa la conducta inmediata más adecuada para
este paciente?
a} Administración de aciclovir oral por 7-10 días

7c. ¿Cuál es la categoría clínica de infección por el virus de la inmunodeficiencia humana


en que se encuentra este paciente?
b} Categoría B

7d. Después de solicitar estudios de laboratorio, resulta que el paciente presenta una
carga viral de 15,000 copias/ml y un conteo de células T CD4+ de 349/mcl. ¿Cuál de las
siguientes opciones representa el mejor tratamiento a seguir?
a}Iniciar tratamiento antirretroviral,pues el conteo de linfocitos T CD4+lo indica

7e. ¿Cuál de las siguientes opciones representa un desenlace esperado del tratamiento
antirretroviral para considerar éste como exitoso?
a} Reducción de porlo menos 10 veces (1logaritmo} en los niveles de RNA del virus
dentro de 1-2 meses siguientes del inicio del tratamiento, y eventualmente una reducción
a < 50 copias/ml
8. Paciente masculino de 1O años de edad el cual es llevado a la sala de urgencias por la
presencia de adenopatía axilar dolorosa. A la exploración física se encuentra una serie de
rasguños en el antebrazo. Se decide realizar una aspiración del nódulo linfático, el cual
produce un material purulento que es enviado a evaluación en el laboratorio. El reporte
revela la presencia de un bacilo altamente pleomórfico. ¿Cuál es con mayor probabilidad
el agente causal de este padecimiento?
d} Bartonella hense/ae

9. Paciente femenino de 55 años de edad que acude a consulta por ictericia. Al


interrogatorio dirigido la paciente no revela información de utilidad que oriente al
diagnóstico. A la exploración física presenta signo de Murphy negativo, pero con vesícula
biliar palpable. ¿Cuál es el diagnóstico más probable en esta paciente?
c} Cáncer de páncreas

1O. Se presenta a consulta geriátrica paciente de 80 años de edad, quien es llevado por su
esposa debido a que lo ha notado más "olvidadizo" que lo usual. Al interrogar al paciente
acerca de su memoria, éste refiere que fue obligado por su esposa para acudir a consulta,
ya que él mismo no hubiera acudido. La esposa ha mencionado que el problema de
memoria de su esposo ha empeorado desde los últimos meses. Los antecedentes sólo son
de importancia por hipertensión de cinco años de diagnóstico. A la exploración
neurológica el paciente muestra deficiencias en la memoria de corto plazo, pero no hay
problemas en cuanto al funcionamiento sensitivo y motor,sin ataxia. Se decide obtener
una tomografía computarizada de la cabeza la cual muestra atrofia cerebral. Es claro que
el paciente presenta un síndrome demencial. ¿Cuál de las siguientes causas reversibles de
demencia es probable que presente el paciente?
e} Hipotiroidismo

11. Paciente del sexo femenino de 20 años de edad que consulta por amenorrea primaria.
Durante el interrogatorio la paciente menciona que ocasionalmente desde la niñez ha
presentado cuadros ocasionales de náusea,vómito y debilidad generalizada. La
exploración física revela hipertensión arterial (180/95 mm Hg). Se solicitan estudios de
laboratorio los cuales muestran hipopotasemia inversa. ¿Cuál de las siguientes
deficiencias enzimáticas con mayor probabilidad presenta esta paciente?

O b} Deficiencia de 17 alfa hidroxilasa

12. Paciente masculino de 50 años de edad que mantiene un matrimonio desde los
últimos 1O años, pero recientemente ha expresado que su esposa ha sido infiel desde el
primer día de matrimonio a pesar de que ella lo ha negado rotundamente. Su problema de
desconfianza se ha extendido al resto de sus relaciones y menciona que la mayoría de las
personas están listas para traicionarlo. Cuando acude a consulta psiquiátrica el paciente se
presenta defensivo a cada comentario del médico. ¿Cuál es el trastorno de la personalidad
que más probablemente
presenta este paciente?
c} Paranoide

13. ¿Cuál de los siguientes anticonvulsivantes produce sedación e inducción del citocromo
P450?
c} Fenobarbital

14. ¿Cuál de los siguientes fármacos puede producir neuropatía periférica?


O c} Paclitaxel

15. Acude a consulta de urgencias un paciente del sexo femenino de 38 años de edad sin
antecedentes de importancia. La paciente refiere que tuvo una riña con uno de sus
hermanos; posteriormente cuando trató de salir de inmediato de la casa, ella fue incapaz
de abrir la puerta por un "ataque" de debilidad en el brazo derecho. A la exploración física
la paciente presenta una fuerza muscular en el brazo derecho 0/5 sin alteraciones de los
reflejos de estiramiento en la misma extremidad. ¿Cuál es el diagnóstico más probable en
esta paciente?
a} Trastorno conversivo

16. ¿Cuál de las siguientes neoplasias del sistema nervioso central en pacientes
pediátricos se asocia con el síndrome de von Hippel-Lindau?
b} Hemangioblastoma

17. ¿Cuál de los siguientes marcadores tumorales se eleva en el carcinoma colorrectal?


b} Antígeno carcinoembrionario

18. Se presenta una paciente de 22 años de edad con su primer embarazo. La paciente se
muestra renuente a abstenerse del consumo de alcohol durante el resto de su embarazo.
En el caso de una mujer embarazada que bebe entre 60 y 120 mlde alcohol a 80%, ¿cuál
es el riesgo de presentar un neonato con síndrome alcohólico fetal?

b} 10 %

19. ¿Cuál de las siguientes translocaciones cromosómicas se asocia con el linfoma


folicular?
d} t(14:18}

20. Paciente masculino de 30 años de edad acude por poliuria y polidipsia. El paciente
menciona que recientemente ha notado un incremento en el tono de la pigmentación de
su piel a pesar de que no se ha expuesto más tiempo del normal a la radiación solar.
Expresa que por las mañanas presenta edema de las extremidades inferiores y necesita
dormir con dos o tres almohadas para evitar la sensación de disnea nocturna. ¿Cuál es el
diagnóstico más probable en este paciente?
d} Hemocromatosis

21. Paciente masculino de 27 años de edad acude a consulta por dolor de 72 horas de
evolución en el muslo izquierdo. Menciona que el dolor se ha acompañado de un
crecimiento en la región, el cual rápidamente ha aumentado de tamaño. Al revisar el
expediente clínico, se revela que el paciente ha acudido en tres ocasiones en los últimos
cuatro meses por lesiones similares, las cuales describe como "picaduras de insectos" en
los glúteos y abdomen. A la exploración física el paciente se encuentra afebril y
normotenso. En la zona dolorosa el paciente tiene una masa fluctuante de 3 cm rodeada
de un anillo eritematoso de 4 cm. ¿Cuál de las siguientes opciones es la más adecuada
para este paciente?
b} Realizar incisión y drenaje, obtener cultivos y prescribir clindamicina oral

22. ¿Cuál de las siguientes alteraciones electrolíticas se asocia con la aparición de la onda
de Osborn en el electrocardiograma?
b} Hipercalcemia

23. Acude a urgencias, paciente femenino de 23 años de edad con diagnóstico de


embarazo de 37 semanas de gestación. Presenta dermatosis generalizada, que respeta
palmas y plantas, caracterizada por vesículas sobre fondo eritematoso, pústulas y costras;
así como prurito. Producto sin alteraciones aparentes. ¿Qué acción tomaría?
d} Gammaglobulina específica

24. Mencione cuál de los siguientes factores está más asociado con el riesgo de padecer
artritis reumatoide:
e} Multifactorial

25. Acude a consulta paciente femenino de 20 años de edad por amenorrea. Refiere que sí
ha presentado menstruaciones en el pasado. No se observan alteraciones en la
talla,fenotipo ni cardiacas, hay vagina corta. Se realiza un cariotipo encontrando 46XX, los
ovarios se encuentran normales. ¿Cuál de las siguientes es una causa de amenorrea
secundaria?
c} Síndrome de Asherman

26. La investigación epidemiológica reconoce situaciones de alerta que demandan


investigación de campo, identifica principios, métodos y procedimientos básicos en el
estudio de brotes y establece bases prácticas para la investigación de campo en los niveles
de salud local. Además en la promoción de la salud se suma la expansión de la vigilancia,
prevención y control de problemas de salud que incluyen no sólo enfermedades
transmisibles, sino estilos de vida, factores de riesgo y desórdenes genéticos, eventos de
salud ocupacional, riesgos ambientales, discapacidad y enfermedades crónicas, entre
otros.
26a. Si en tu comunidad diagnosticas un cuadro de rubéola en menor de 2 años, estás
ante una alerta epidemiológica y debes actuar de manera rápida y eficiente para
interrumpir la infección. Para esta situación la Salud Pública cuenta con dos métodos de
investigación:
a} Epidemiología descriptiva y analítica

26b. La investigación epidemiológica de campo aplica métodos de las ciencias básicas,


estadísticas y sociales que incluye la investigación de brotes como una tifoidea y suele
aplicar un diseño descriptivo. Algunas de sus características son:
b} Ágil,riguroso, técnicamente sencillo, ofrece respuestas urgentes ante epidemia local

26c. Al aumento inusual en el número de casos relacionados epidemiológicamente, de


aparición súbita y diseminación localizada en un espacio específico, como pudiera ser una
intoxicación alimentaria se le llama:
d} Brote

26d. ¿Cuál es la utilidad de tener el canal endémico actualizado como el de los casos de
influenza en adultos mayores?
a} Para emitir una alerta epidemiológica

26e. En Salud Pública, al indicador que mide la severidad de una enfermedad como sería
para la diabetes tipo 2 se le conoce como:
d} Tasa de Letalidad

26f. Para confirmar un brote se requiere verificar el diagnóstico a través de la historia


clínica y exámenes de laboratorio de los casos notificados. El siguiente paso es el trabajo
de campo y el tercero es la definición operacional de caso que reúne diferentes criterios.
Menciona los criterios epidemiológicos.
c} Distribución,tiempo, espacio y persona, periodo de incubación y de exposición al
probable caso índice, casos secundarios y fuente común,tiempo de exposición y área
geográfica

26g. El diseño epidemiológico de tipo analítico más empleado durante la investigación de


una alerta epidemiológica, por ejemplo de un caso de hepatitis A en niños, es el estudio
de caso-control que investiga a dos grupos de la comunidad: con la enfermedad parte del
brote (casos) y sin la enfermedad (controles). El análisis que explora la significancia entre
exposición y enfermedad se determina con:
a} Chi-cuadrado

27. Paciente femenino de 27 años de edad, que presenta a nivel genital cuatro úlceras
blandas y dolorosas, con inflamación perilesional. Ganglios en ingle derecha dolorosos y
ulcerados que según la paciente aparecieron cinco días después de la primera úlcera.
Antes de la aparición de la primera úlcera tuvo contacto sexual con una nueva pareja.
27a. ¿Cuál es el agente etiológico de esta patología?
b} Haemophilus ducreyi
27b. ¿Cuál es el tratamiento indicado para este microorganismo?
c} Ceftriaxona

28. Paciente femenino de 11 años de edad acude por presencia de masa eritematosa
dolorosa en región nasal del párpado inferior del ojo derecho desde hace tres días que ha
ido en aumento. La madre refiere que había presentado cuadros previos semejantes y que
había resuelto con tratamiento no especificado. Desde lactante presentaba descarga de
secreción mucopurulenta a través del punto lagrimal del mismo ojo. A la exploración se
observa eritema cutáneo, edema,se palpa indurada por debajo del tendón cantal medio y
con presión leve sobre el saco lagrimal se obtiene material mucopurulento amarillo-
verdoso. Se observa conjuntiva sin hiperemia, córnea transparente, sin alteraciones de la
motilidad ocular. ¿Cuál es el diagnóstico más probable?
e} Dacriocistitis

29. Mujer de 70 años de edad padece diabetes mellitus tipo 2 complicada con retinopatía
proliferativa, neuropatía sensitiva distal e insuficiencia renal crónica, acude a consulta de
revisión. A la exploración física muestra presión arterial 160/95 mm Hg, frecuencia
respiratoria 24 resp/min, ictericia generalizada y debilidad muscular; datos de laboratorio
con anemia normocítica normocrómica (Hb 1O g/dl, VCM 90 fl,HCM 30 pg), creatinina 4
mg/dl, BUN 60 mg/dl, hiperpotasemia, hiperfosfatemia, hipocalcemia (7 mg/dl) y acidosis
metabólica de brecha aniónica elevada (pH 7.32, HC03 18 mEq/L, PaC02 33 mm Hg). Su
depuración de creatinina por Cockcroft-Gault es de 40 mUmin."
29c. La anemia normocítica normocrómica en esta paciente es secundaria a:
c} Déficit de eritropoyetina

30. Paciente masculino de 20 años de edad acude a la sala de urgencias por presentar una
herida por arma de fuego en el epigastrio. Al parecer no hay orificio de salida. A la
palpación, refiere intenso dolor abdominal. Es programado para exploración quirúrgica, y
antes es llevado a tomar una tomografía computarizada. El estudio de imagen demuestra
que la bala lesionó el páncreas. ¿Cuál de los siguientes enunciados es el más adecuado
según la lesión del paciente?
a} La principal causa de muerte en este paciente es la hemorragia asociada a lesiones
vasculares mayores

31. Paciente masculino de 40 años, sin antecedentes de importancia, acude después de


trauma contuso con pelota de squash en ojo derecho de 3 horas de evolución. A la
exploración oftalmológica se encuentra agudeza visual de 20/400 observando hiperemia
conjuntiva!, inyección ciliar, córnea transparente, cámara anterior formada con
celularidad leve y un nivel hemático de 15% de la cámara anterior,pupila normorrefléctica,
presión intraocular dentro de límites normales, cristalino transparente, retina aplicada y
vítreo sin opacidades. ¿Cuál sería el tratamiento inicial para el hifema traumático en este
paciente?
a} Esteroides y ciclopléjicos tópicos

32. Paciente femenino de 55 años de edad con una úlcera en el tercer dedo de la mano
izquierda. Según refiere, la úlcera se presentó hace siete días y no es dolorosa. Decidió
automedicarse utilizando un antibacteriano tópico e hidrocortisona en crema. Al
interrogatorio no logra recordar algún traumatismo que explique esta lesión. La
temperatura de la paciente es de 39° C con adenomegal ia epitroclear y axilar izquierda. La
úlcera es de alrededor de 3 cm en la cara dorsal del tercer dedo de la mano izquierda y se
encuentra cubierta por una escara negra y rodeada por edema. ¿Cuál de los siguientes
diagnósticos es el más probable en esta paciente?
b} Ántrax cutáneo

33. Paciente femenino de siete años de edad acude al servicio de urgencias por presentar
fatiga y palidez. La paciente había sido tratada hace poco por infección de vías aéreas
superiores. En esta ocasión, la exploración física revela esplenomegalia y equimosis en las
extremidades superiores. Los estudios de laboratorio demuestran un conteo de leucocitos
de 11 000 células/mm3, hemoglobina de 6 g/dL, y conteo plaquetario de 40 000
plaquetas/mm3. La prueba de Coombs es positiva. ¿Cuál es el diagnóstico más probable
en esta paciente?
c} Síndrome de Evans

34. ¿Cuál de los siguientes calendarios de administración de la vacuna DPT es el más


correcto según el esquema de vacunación de México?
c} A los dos, cuatro, seis y 18 meses de edad y a los cuatro años de edad

35. ¿Cuál de las siguientes opciones representa al agente infeccioso más frecuente en
pacientes de cuatro meses de edad con secreciones nasales copiosas, sibilancias y tos, en
especial en la época de invierno?
d} Virus sincitial respiratorio

36. ¿Cuál de las siguientes enfermedades se caracteriza por hipertensión pulmonar?


a} Esclerodermia

37. Paciente femenino de 75 años de edad que se encuentra en la actualidad en


tratamiento por infección de vías urinarias con trimetoprimsulfametoxazol. Después de
una semana de haber completado el tratamiento, acude al servicio de urgencias por la
aparición de vesículas en el abdomen, las cuales evolucionan a bulas flácidas y erosiones
de la piel del tórax, abdomen,espalda y muslos. En la mucosa oral también tiene
erosiones. La biopsia demuestra separación intraepidérmica. ¿Cuál es el diagnóstico más
probable en esta paciente?
b} Pénfigo vulgar

38. ¿Cuál de los siguientes medicamentos se relaciona con trombocitopenia y


microtrombosis?
a} Heparina

39. ¿Cuál de las siguientes descripciones corresponde con las hemorragias en astilla?
d} Lesiones lineares o enforma de llama

40. ¿Cuál es la causa más probable del dolor de rodillas en corredores?


a} Síndrome patelofemoral

41. Una mujer de 40 años de edad acude para extirpación de "verrugas" en el cuello
porque le duelen cuando se pone una cadena. Tiene una dermatosis localizada,
bilateral,simétrica que afecta cuello y axilas. Se caracteriza por múltiples neoformaciones
exofíticas, pediculadas, bien delimitadas, lisas, del color de la piel. Le molestan con cuellos
altos de la ropa y cadenas y refiere aumento después de un embarazo. Ella tiene
sobrepeso y resistencia a la insulina.
41a. ¿Cuál es el diagnóstico más probable?
d} Fibromas blandos

41b. ¿A cuál enfermedad sistémica se relacionan los fibromas blandos?


e} Diabetes mellitus o síndrome metabólico

41c. ¿Cuál es la causa aceptada de los fibromas blandos?


c} Sobrepeso

41d. ¿Cuál es el tratamiento de elección para los fibromas blandos?


d} Rasurado

41e. ¿Cuál es el tratamiento preventivo de los fibromas blandos?


e} Ninguno

42. Paciente masculino de 25 años de edad, con antecedentes familiares por línea
materna de hemorragias. Acude a consultar por hematomas que aparecieron en ambos
brazos tras golpes mínimos desde la infancia. Ha recibido anteriormente tratamiento con
desmopresina por hemofilia tipo A sin resultados.
42a. Señale el tratamiento indicado en este paciente:
a} Factor VIII recombinante

42b. Tipo de herencia de la enfermedad con la que cursa el paciente.


c} Ligada al cromosoma X
42c. ¿Cuál de las siguientes manifestaciones hemorrágicas no forma parte de la clínica en
los pacientes con hemofilia?:
b} Petequias

42d. ¿Cuál de las siguientes alteraciones en el laboratorio se encuentran en los pacientes


con hemofilia?
b} Tiempo tromboplastina parcial alargado-tiempo de protrombina normal
42e. Señale el enunciado correcto respecto a la terapia anticoagulante:
d} El control dela medicación anticoagulante oral se realiza con elTP

43. Paciente femenino de 31 años acude a consulta y comenta que fue diagnosticada con
lupus eritematoso sistémico (LES) hace 2 años. Inició con eritema malar, dolor articular y
síntomas neuropsiquiátricos. Además, se le realizaron estudios en donde se encontraron
ANA positivos; actualmente está controlada. Acude a consulta pues desea embarazarse y
quiere tener mayor información al respecto. No tiene otros antecedentes de importancia.
A la exploración física registra FC: 67, FR: 20, Temp: 36 ºC; el resto sin alteraciones.
43a. ¿Cuál de los siguientes planteamientos es incorrecto en cuanto al embarazo y el LES?
b} El LES es contraindicación absoluta de embarazo

43b. En cuanto a la nefritis lúpica, ¿cuál es el subtipo más común en las pacientes con LES?
c} Proliferativa difusa

43c. En el LES, ¿qué anticuerpos se asocian a enfermedad renal?


c} DNA doble cadena

43d. ¿Cuál es el principal efecto adverso de la hidroxicloroquina?


a} Daño en retina

43e. Es un criterio de clasificación de LES:


b} Linfopenia (menor a 1500/mm3}

44. La Comisión Nacional de los Derechos Humanos, específicamente en el Programa de


Asuntos de la Mujer y de igualdad entre mujeres y hombres, publica los delitos de
hostigamiento sexual, abuso sexual,estupro y violación, de los cuales es preciso saber:
44a. El abuso sexual por medio de la violencia física o moral y si la víctima fuera la esposa
o concubina, se impondrá una pena de:
c} 8 a 20 años de prisión

44b. Se equipara a la violación, al que sin consentimiento de una persona o con el


consentimiento de un o una menor de 14 años introduzca uno o más dedos o un objeto de
cualquier naturaleza en la región anal o vaginal,a esto se le llama:
b} Violación impropia
45. Ante un niño de cinco años que presentó síndrome anticolinérgico por intoxicación
medicamentosa, ¿cuál de las siguientes NO forma parte del síndrome de toxicidad de los
antihistamínicos?
e} Diarrea

46. Paciente masculino de 65 años de edad, con antecedente de tabaquismo, refiere


padecer dificultad para respirar al caminar por las mañanas desde hace 3 años, por lo que
mantiene una vida sedentaria. Acude a consulta por tos intermitente desde hace 3 meses.
Niega dolor torácico. A la auscultación se detecta disminución del murmullo vesicular. La
radiografía de tórax reporta aplanamiento diafragmático, disminución del trama vascular y
silueta cardiaca alargada.
46a. ¿Cuál es el punto de corte en la espirometría para el diagnóstico de EPOC?
b} FEV1/FVC < 70%

46b. Esquema farmacológico indicado en este paciente.


a} Tiotropio + salbutamol/ipratropio

46c. ¿Cuáles son las medidas que han demostrado disminuir la mortalidad en la EPOC?
b} Abandono deltabaco-oxigenoterapia

46d. Señale los criterios de Winnipeg que definen exacerbación de la EPOC:


d} Aumento de disnea + aumento de expectoración + purulencia delesputo

46e. ¿Cuál es el tratamiento indicado ante una exacerbación de la EPOC de causa


infecciosa?
c} Amoxicilina + ácido clavulánico

47. Acude a Urgencias un paciente masculino de 35 años, sin antecedentes de


importancia,con fiebre de 3 días de evolución, cefalea, fotosensibilidad y somnolencia. A
la exploración física se nota rigidez de cuello, así como signo de Kernig y Brudzinski
positivos. Los signos vitales son temperatura de 39 ºC, FC: 98, FR: 25, TA: 110/70 mm Hg.
Se toma la decisión de internarlo para estudiarlo.
47a. ¿Cuál es el agente etiológico más frecuente de meningitis en pacientes de esta edad?
a} S. pneumoniae

47b. ¿Cuál de las siguientes condiciones NO es indicación para realizar una tomografía de
cráneo antes de una punción lumbar?
c} Fiebre

47c. Usted lleva a cabo una punción lumbar, por medio de la cual encuentra los siguientes
datos en el LCR: presión de apertura elevada, 9 000 PMN, glucosa 20 mg/dL, proteínas de
1 000 mg/dL, lo cual lo lleva a sospechar de una meningitis:
b} Bacteriana

47d. El médico de base no va a llegar hasta dentro de 8 h,así que usted tiene que decidir el
tratamiento. ¿Qué tratamiento administraría?
d} Ceftriaxona más vancomicina más dexametasona

47e. Si se tratara de un paciente con meningitis meningocócica, ¿cuál es el tratamiento


profiláctico para los contactos cercanos?
b} Rifampicina

48. Acude a consulta paciente masculino de 67 años, fumador,por un cuadro de malestar


general y dolor en columna vertebral que inició hace 3 meses y ha ido progresando. A la
exploración física solamente llama la atención el dolor difuso en la columna vertebral. En
los laboratorios se encuentra anemia normocítica normocrómica, plaquetas 115
000/µL,VSG 120, creatinina de 2.8 mg/dL, proteínas totales 8.5 g/dL e hipoalbuminemia
de 2.1 g/dL, orina con proteinuria de 4 g/L e hipercalciemia de 11.2 mg/dL.
48a. De acuerdo con el caso clínico, ¿cuál es su sospecha diagnóstica?
a} Neoplasia maligna de células plasmáticas

48c. Para estadificar la enfermedad se emplean los siguientes criterios, según Durie-
Salmon:
b} Hb, Ca, lesiones líticas en hueso, lgG olgA o cadenas ligeras en orina

48d. ¿Cuál es el tratamiento de un paciente con esta enfermedad que se encuentra


asintomático o en estadio I?
d} No se da tratamiento

48e. ¿Cuál de los siguientes enunciados no es verdadero?


b} El pico monoclonal en la mayoría de los casos es delgA

49. Paciente masculino de 65 años con diagnóstico de envío de enfermedad ampollosa no


especificada presenta dermatosis diseminada que afecta tronco y mucosa oraly
conjuntiva!, además del ano, caracterizada por ampollas tensas, eritema y descamación
en encías. En la valoración oftalmológica se encuentra conjuntivitis inespecífica,triquiasis y
entropión que se acompañan de disminución en la agudeza visual. En mucosa anal
presenta atrofia y erosiones. Refiere cinco años de evolución y ha recibido múltiples
tratamientos tópicos a base de antibióticos y corticoesteroides así como neosporina
oftálmica. Mencione cuál de las siguientes afirmaciones es correcta.
c} Afecta hasta en 85% la mucosa oral además de afección oftálmica

50. Paciente masculino de 70 años, con antecedente de hipertensión arterial de 5 años de


evolución controlada con captopril 25 mg, es fumador desde hace 20 años a razón de una
cajetilla diaria y fue diagnosticado con enfermedad pulmonar obstructiva crónica hace 2
años. Acude a Urgencias debido a que ha aumentado bastante la tos y presenta un leve
aumento del esputo, así como disnea de esfuerzos mínimos. La radiografía muestra datos
típicos de enfisema pulmonar.
50a. ¿Qué tratamiento le daría a este paciente en Urgencias para mejorar su estado
actual?
e} Salbutamol y bromuro de ipratropio

50c. Su paciente presenta una FEV1 de 48%. ¿En qué estadio de GOLD está?
c} Severo

50d. De acuerdo con el estadio de GOLD en el que se encuentra su paciente, ¿cuál es el


tratamiento crónico indicado para él?
a} Tiotropio diariamente y un esteroideinhalado en exacerbaciones

50e. ¿Cuál de las siguientes es una indicación para la utilización de oxígeno suplementario
en casa?
b} Saturación de 02 menor a 89%

51.Acude a consulta un paciente masculino de 52 años por un dolor intenso periorbitario


unilateral, acompañado de inyección conjuntival, lagrimeo y rinorrea. Comenta que esto le
sucede dos veces al día, aprox. desde hace 2 semanas, e incluso en una ocasión lo
despertó en la madrugada. Como antecedente de importancia tuvo un infarto agudo de
miocardio hace 2 años con aplicación de stent. Toma diariamente aspirina 85 mg y
clopidogrel. A la exploración física, los signos vitales son FC: 70, FR: 18, TA: 130/85 mm Hg,
Temp: 36 ºC. El resto de la exploración es normal.
51a. ¿Cuál de los siguientes es el tratamiento más efectivo para este paciente?
d} Prednisolona e inhalación de oxígeno

51b. ¿Cuál es de los siguientes datos no sirven como indicadores para tomar una
neuroimagen en el caso de cefalea?
c} La visión de escotomas

51c. En cuanto a la cefalea tensional, ¿cuál de los siguientes medicamentos se utiliza como
profilaxis?
a} Amitriptilina

51d. En cuanto a la cefalea migrañosa, ¿en qué casos se deben de indicar medicamentos
profilácticos?
d} Todos los anteriores

51e. ¿A qué se le conoce como estatus migrañoso?


a} Cuando las manifestaciones clínicas duran más de 72 hy no responden a tratamiento
habitual
52. Paciente masculino de 58 años de edad acude a consulta externa por saciedad
temprana y pérdida de peso inexplicado (cerca de 14 kg en ocho meses). Cuenta con
antecedente de hipertensión arterial (controlada con inhibidores de la enzima
convertidora de angiotensina) y diabetes mellitus tipo 2 de 12 años de diagnóstico
(controlada con insulina). El paciente ya había acudido con otro médico, debido al temor
de padecer cáncer, sin embargo, a pesar de que se le realizaron varios estudios no se le
diagnosticó ningún tipo de cáncer. Debido al antecedente endocrinológico se decide
realizar una prueba de vaciado gástrico, la cual demuestra lentitud del vaciamiento
gástrico. ¿Cuál de los siguientes efectos adversos es el que puede presentarse con más
probabilidad con el tratamiento que se le debe aplicar al paciente?
d} Trastornos extrapiramidales

53. ¿Cuál de las siguientes aseveraciones respecto a la policitemia vera es el más correcto?
e} La elevación de los niveles de eritropoyetina excluye el diagnóstico de policitemia vera

54. ¿Cuál de los siguientes organismos se asocia con zonas desérticas y se presenta como
esférulas con endosporas en la biopsia de tejidos infectados?
b} Coccidioides immitis

55. ¿Cuál de los siguientes esquemas profilácticos de tromboembolia venosa es el más


adecuado en pacientes de alto riesgo para desarrollar ésta?
c} Administración subcutánea de heparina de bajo peso molecular

56. Paciente con una tumoración de 3 mm que le produce dolor intenso. El tumor se
encuentra por debajo de la uña del dedo índice de la mano izquierda. Se someterá a un
procedimiento quirúrgico para extirpar esta tumoración. ¿Cuál de los siguientes nervios
debe bloquearse para generar anestesia para la extirpación de este tumor?
d} Mediano

57. Paciente masculino de 77 años de edad que es llevado al servicio de urgencias por sus
hijos debido a que se encontraba con alteración del estado de alerta y diciendo que
estaba en la casa del presidente de la nación. A la exploración física se presenta con una
presión arterial de 80/60 mm Hg, y frecuencia cardiaca de 120 latidos/minuto, así como
con una temperatura de 39 ºC. Los hemocultivos son positivos para Escherichía colí. ¿Cuál
es el origen más frecuente de la bacteriemia en este paciente?
c} Vías urinarias
58. Paciente masculino de 18 años de edad que presenta episodios recurrentes en los que
aparece exantema vesicular en zonas expuestas a la
luz. Durante los estudios de laboratorio se descubre elevación de las porfirinas
plasmáticas. ¿Cuál de las siguientes enzimas es la que se encuentra con mayor
probabilidad afectada en este paciente?
a} Uroporfirinógeno descarboxilasa

59. ¿Cuál de los siguientes enunciados es correcto respecto de la enfermedad celiaca?


a} En caso de alta sospecha de enfermedad celiaca, una biopsia de intestino delgado, los
marcadores serológicos como los anticuerpos antigliadina, antiendomisio y
antitransglutaminasa tisular pueden confirmar eldiagnóstico

60. ¿Cuál de los siguientes nervios se puede lesionar en caso de una fractura del cuello
quirúrgico del húmero?
a} Nervio axilar

61. Paciente masculino de 13 meses de edad con historia de fiebre de tres días de
evolución (temperatura máxima 39.8 C) que se resolvió y después apareció rash en
tronco, diseminándose después a brazos, cuello y cara. En la exploración se encuentran
lesiones maculopapulares rosadas, no confluentes, en cara, cuello, brazos y tronco.
¿Cuál es el agente etiológico más probable?
e} Herpes virus tipo 6

62. Paciente masculino de 73 años, diabético de 25 años de evolución (en control con
metformina 850 mg por la noche) e hipertenso de 10 años de evolución (en control con
captopril 25 mg por la mañana), acude a consulta para control de un problema de
insuficiencia renal crónica. Comenta que se siente en general cansado y con prurito en
todo el cuerpo. A la exploración física sus signos vitales son FC: 67, TA: 150/90 mm Hg, FR:
18, Temp: 36ºC; el resto de la exploración normal. Al calcular su tasa de filtración
glomerular usted obtiene un resultado de 29 mUmin/1.73 m2 .
62a. ¿En qué estadio de insuficiencia renal crónica se encuentra este paciente?
c} Estadio 4

62b. ¿Cuál es la definición correcta de insuficiencia renal crónica?


c} Tasa de filtración glomerular menor a 60 ml/min/1.73 m2 o daño renal anatómico o
funcional por 3 meses o más

62c. ¿Cuál es la causa principal de insuficiencia renal crónica?


b} Diabetes mellitus

62d. ¿Cuál es la meta de presión arterial en un paciente con insuficiencia renal crónica?
b} Menor a 130/80 mm Hg
62e. ¿En qué momento se recomienda suspender el uso de metformina en un paciente
diabético con insuficiencia renal?
b} Cuando la creatinina sérica sea mayor a 1.4 mg/dl en hombres y 1.5 mg/d en mujeres

63. ¿Cuál de las siguientes funciones es la que corresponde a las ondas lentas en el tracto
gastrointestinal?
d} Despolarización de las células de músculo liso

64. Posterior a una infección de vías aéreas superiores, un paciente de sexo masculino de
seis años de edad desarrolla lesiones tipo púrpura en los glúteos y en las superficies
extensoras de las extremidades superiores e inferiores. Simultáneamente con la aparición
de estas lesiones el paciente inicia con dolor abdominal, vómito y artralgias. En una
prueba cualitativa de orina se puede detectar hematuria. ¿Cuál de las siguientes
características se encontraría en una biopsia renal en este paciente?
e} Depósito de lgA en las regiones mesangiales

65. ¿Cuál de las siguientes lesiones se asocia con más frecuencia a necrosis avascular?
a} Fractura de escafoides

66. Paciente masculino de 22 años de edad, el cual a pesar de apego al tratamiento, no ha


logrado controlar su cuadro asmático. El médico decide agregar zileutón al esquema.
¿Cuál de los siguientes estudios debe realizarse antes de iniciar tratamiento con este
medicamento?
d} Pruebas de función hepática

67. ¿Cuál de los siguientes enunciados respecto a la pericarditis urémica es correcto?


a} Actualmente existen procedimientos quirúrgicos indicados en caso de derrame
pericárdico que ocasiona compromiso hemodinámico

68. Paciente masculino de 45 años de edad que se encuentra en este momento en fase de
rehabilitación por alcoholismo, el cual inició desde los 19 años de edad. En esta ocasión el
paciente acude a discutir los resultados de una biopsia hepática que se solicitó
previamente. A la exploración física presenta esplenomegal ia y distensión abdominal. El
reporte histopatológico de la biopsia indica presencia de fibrosis difusa y regeneración
nodular. ¿Cuál de las siguientes complicaciones puede presentar este paciente a pesar de
que detenga el consumo de alcohol?
e} Carcinoma hepatocelular

69. Paciente masculino de 55 años de edad que a la exploración neurológica presenta


cierre palpebral a la estimulación de la córnea derecha, pero ausencia de cierre palpebral
consensual en el ojo izquierdo. ¿Cuál de los siguientes hallazgos será posible encontrar
también en este paciente?
c} Hiperacusia del oído izquierdo
70. Paciente masculino de 11 años de edad es sometido a una craneotomía para
extracción de craneofaringioma. En la cirugía recibió halotano; al concluir ésta fue
extubado sin ninguna eventualidad. Durante el periodo de recuperación, la enfermera
colocó al paciente una vía intravenosa e infusión de solución de dextrosa a 5% en un
tercio de solución salina normal a una velocidad de 125 mUh. Posteriormente, la
enfermera llama al médico tratante debido a que el paciente presenta disminución del
estado de alerta. Los resultados de laboratorio más reciente muestran la concentración
sérica de sodio de 157 mEq/L, osmolaridad de 320 mOsm/L, PaC02 x de 28 mm Hg,
bicarbonato de 22 mEq/L y pH de 7.3, así como incremento en el balance positivo de
líquidos. ¿Cuál de los siguientes diagnósticos es el más probable en este caso?
b} Lesión a la hipófisis posterior

71. ¿Cuál de los siguientes factores de la coagulación es dependiente de vitamina K?


a} Factor 11, VII,IX y X

72. Paciente del sexo femenino de 28 años de edad que acude a consulta por infertilidad.
A pesar que ha intentado relaciones sexuales regulares sin protección anticonceptiva
durante los últimos dos años, no ha logrado embarazarse. Al interrogatorio la paciente
menciona que tiene menstruaciones muy irregulares, las cuales ocurren cada dos a tres
meses, e inclusive, cada cinco meses. A la exploración física es evidente que presenta
hirsutismo y obesidad. Se decide iniciar tratamiento farmacológico, lo cual incrementa sus
niveles de progesterona en sangre; la realización de una biopsia de endometrio revela
cambios secretores posteriores al inicio del medicamento. ¿Cuál de los siguientes
medicamentos fue el que se le administró con mayor probabilidad a esta paciente?
d} Antagonistas de estrógenos

73. Paciente masculino de 33 años de edad con antecedente de alcoholismo crónico se


presenta con confusión,alteración de la marcha y nistagmo. Posterior a la administración
de tiamina el paciente muestra mejoría significativa. ¿Cuál de los siguientes cambios
neuropatológicos presentó este paciente?
a} Necrosis hemorrágica de los cuerpos mamilares y dela materia gris periacueductal

74. Paciente masculino de 45 años de edad el cual se presenta con fatiga, palidez de piel y
mucosas, y soplo sistólico. En los estudios de laboratorio se presenta un hematocrito de
27%, volumen corpuscular medio de 80 fl,concentración corpuscular media de
hemoglobina de 25 pg/célula y reticulocitos de 4.5%, así como una concentración de
ferritina de 340 µg/L. El frotis de sangre periférica demuestra macrocitos policromatófilos.
¿Cuál es el diagnóstico más probable en este paciente?
a} Disfunción de médula ósea

75. ¿Cuál de los siguientes antimicóticos ejerce su efecto sobre los microtúbulos de las
células micóticas?
c} Griseofulvina
76. Paciente femenino de 45 años de edad con antecedente de cáncer de glándula
mamaria. La paciente fue diagnosticada hace dos meses y se le realizó una mastectomía
radical con extirpación de ganglios cervicales. Posteriormente fue programada para
administración de quimioterapia. Posterior a la administración de quimioterapia la
paciente presenta náusea y vómito incontrolable por lo que se administra un fármaco
para disminuir esta sintomatología, Aunque el medicamento disminuyó la náusea y el
vómito, también ocasionó constipación. ¿Cuál de los siguientes fármacos es el que más
probablemente se le administró a esta paciente?
a} Un antagonista de los receptores de serotonina 5HT3

77. Paciente masculino de 70 años con dolor en hipocondrio derecho desde hace 1O
horas. A la exploración física se presenta ictérico y con fiebre. Los estudios de laboratorio
revelan elevación leve de las transaminasas, hiperbilirrubinemia (6 mg/dL) y leucocitosis
(16 000 células/mm3). ¿Cuál de las siguientes conductas es la más adecuada en este
paciente?
c} Solicitar una colangiopancreatografía endoscópica retrógrada

78. ¿Cuál de los siguientes fármacos antihipertensivos es adecuado para el manejo de


hipertensión arterial sistólica aislada en pacientes no diabéticos?
a} Amlodipino

79. ¿Cuál de las siguientes opciones representa la lesión glomerular intrínseca primaria
más frecuente?
b} Nefropatía por lgA

80. Paciente masculino de 66 años de edad que acude a consulta por presentar desde
hace varias semanas febrícula, poliartralgias, y exantema eritematoso localizado
principalmente en la cara, aunque hace poco tiempo se ha observado también en la
región superior del tórax. Al interrogatorio el paciente menciona que recientemente sufrió
un infarto agudo al miocardio, así como una arritmia (no especificada en este momento).
En los exámenes de laboratorio se revela positividad para el anticuerpo antinuclear. ¿Cuál
de los siguientes medicamentos es el que está tomando este paciente con mayor
probabilidad?
b} Procainamida

81. Paciente masculino de 70 años de edad conocido por el servicio de cardiología porque
acude desde hace tres años por cuadros de angina estable. En esta ocasión asiste al
servicio de urgencias por presentar dolor torácico intenso. La enfermera toma un
electrocardiograma, el cual revela elevación del segmento ST; las pruebas enzimáticas
rápidas determinan elevación de la creatinincinasa-MB. El paciente es estabilizado y
admitido a la unidad coronaria. Varios días después de este evento presenta de nuevo
dolor torácico. A la exploración física presenta fiebre, frote pericárdico intenso, estertores
pulmonares y edema de extremidades inferiores. ¿Cuál de los siguientes diagnósticos es el
más probable en este paciente?
d} Pericarditis fibrinoide

82. ¿De los métodos diagnósticos presentados a continuación, cuál es el más adecuado
para realizar el diagnóstico de embolia pulmonar en un paciente con un proceso
neumónico subyacente?
e} Tomografía computarizada espiral contrastada deltórax
83. Paciente masculino de 44 años de edad que se presenta a consulta por intensos
dolores cólicos, constipación, irritabilidad y cefalea. El paciente trabaja en una fábrica de
pintura en aerosol. Presenta unos estudios de laboratorio que el médico laboral le solicitó;
la principal característica de éstos es una anemia microcítica. Al revisar el expediente
médico el paciente ha presentado episodios depresivos mayores en el pasado. ¿Cuál de
las siguientes sustancias es con mayor probabilidad la causante la sintomatología de este
sujeto?
e} Plomo

84. Se requiere de su presencia en la sala de parto debido a que se espera el nacimiento


de un masculino de 37 semanas de gestación. Observa los procedimientos del obstetra, no
se sucitan complicaciones y recibe un recién nacido masculino. Inicia la exploración física
de éste y observa movimiento activo, frecuencia cardiaca de 139 lpm, buen tono
muscular, flexión de las cuatro extremidades, acrocianosis y llanto vigoroso.
84a. De acuerdo con las características del recién nacido, ¿qué puntaje de Apgar obtiene?
c} 8 puntos

84b. Al recibir a un recién nacido vigoroso como en el caso descrito, ¿cuál de los siguientes
pasos se debe realizar primero?
a} Verificar que la vía aérea esté permeable, secar,y estimular al recién nacido

84c. En caso de no contar con la información de la edad gestacional del recién nacido
descrito anteriormente, ¿cuál de los siguientes signos clínicos le indicaríamejorque se
trata de un recién nacido a término?
c} Tejido mamario palpable

84d. El neonato pasa a cuidados en cuneros y se comienza el protocolo de recién nacido


por las enfermeras, el cual incluye escrutinio de hipoglucemia. En el neonato a término
asintomático, ¿a qué nivel de glucosa debe iniciarse la evaluación y tratamiento de
hipoglucemia?
c} 45 mg/dl

84e. Después de una semana de nacido, el paciente es llevado a consulta de revisión.


¿Cuál de las siguientes opciones es la correcta respecto al peso de un recién nacido en la
primera semana de vida?
d} Perder aproximadamente 5 a 10% delpeso al nacimiento
85. Paciente masculino de 50 años de edad con esófago de Barrett que se presenta con
gastroenterólogo para endoscopia de seguimiento. Se toman múltiples biopsias y se
envían a estudio histopatológico. El reporte revela la presencia de displasia de bajo grado.
¿Cuál de las siguientes conductas clínicas es la más adecuada?
a} Repetir endoscopia en 6 y 12 meses

86. Mujer de 24 años de edad que presenta tumoración en fosa ilíaca derecha dolorosa.
Desde hace seis meses presenta palpitaciones, caída de cabello, diarrea e hiperreflexia. Se
diagnostica tumor de ovario benigno. Se observan tejidos de las tres capas germinativas.
¿Qué es característico de los tumores germinales de ovario?
a} El gonadoblastoma se relaciona a síndrome de Swyer

87. Femenino de 62 años llevado a sala de urgencias por experimentar episodios


frecuentes de ptosis, diplopía y fatiga generalizada. EF: se encuentra timo palpable y
parálisis del nervio oculomotor que se corrige transitoriamente al administrar edrofonio.
La entidad nosológica responsable de la sintomatología de este paciente es:
a} Miastenia grave

88. Un paciente bajo tratamiento antipsicótico por esquizofrenia acude a la consulta por
exceso de rigidez muscular, hipertermia y disminución de la sudación. A su ingreso el
paciente presenta cambios bruscos en las cifras tensionales, acompañados de episodios
de taquicardia y bradicardia. ¿Cuál de los siguientes medicamentos se debe administrar
inmediatamente a este paciente?
d} Dantroleno

89. Mujer de 60 años, consulta por aumento de volumen en cuello. Al interrogatorio


refiere fatiga y estreñimiento hace varias semanas, con aumento leve de peso. TA: 120/90,
FC: 65 x', Temp: 36.5 ºC. A la exploración del cuello se detecta bocio irregular e indoloro,
edema pretibial leve sin fóvea, y frialdad de extremidades. Sus medicamentos incluyen
ramipril y metformina. Perfil tiroideo: TSH elevada, T4 libre baja, anticuerpos
antiperoxidasa tifoidea positivos.
89a. Con la clínica y los laboratorios mencionados, usted hace el diagnóstico de:
c} Hipotiroidismo autoinmune

89b. ¿Cuál es la base fisiopatológica de la enfermedad de este paciente?


a}Infiltración de linfocitos T CD4 y CDS activados y linfocitos B enla tiroides

89c. ¿Cuál de los siguientes enunciados es correcto?


d} La T3 es la hormona tiroidea más potente

89d. ¿Cuál es el tratamiento indicado en este paciente?


a} Levotiroxina
89e. ¿Cuál de los siguientes enunciados es correcto respecto altratamiento restitutivo con
levotiroxina en pacientes hipotiroideos?:
b} El objetivo del tratamiento es lograr una concentración normal de TSH

90. Neonato de seis horas de vida cuya madre fue positiva para antígeno de superficie de
hepatitits B (HBsAg). ¿Cuál es la conducta más adecuada en relación a profilaxis contra
hepatitis B?
c} Administrar la primera dosis de vacuna anti-hepatitis B e inmunoglobulina de hepatitis
B antes de las primeras 12 horas de vida

91. Paciente femenino de siete años de edad acude a consulta por diplopía que ha
empeorado en los últimos días. A la exploración oftalmológica demuestra parálisis del VI
nervio craneal. ¿Cuál de los siguientes padecimientos es el que con mayor probabilidad
tiene la paciente?
a} Glioma del puente

92. Paciente masculino de 48 años de edad, sedentario, con circunferencia abdominal de


108 cm, hipertensión arterial e hipertrigliceridemia. Acude a consulta de control. Se
reporta asintomático. TA: 130/80 mm Hg, hiperpigmentación en cuello y axilas.
Laboratorios: glucosa en ayuno 154 mg/dL, HbA1c: 7.0%, colesterol LDL: 115 mg/dL, HDL:
48 mg/dL,triglicéridos: 300 mg/dL.
92a. ¿Cuál es el diagnóstico en este paciente?
c} Síndrome metabólico cumpliendo 4 criterios

92b. De acuerdo con el diagnóstico de diabetes mellitus tipo 2 en el paciente, señale el


enunciado correcto:
b} Se debe repetir la prueba para el diagnóstico definitivo

92c. Señale el enunciado correcto respecto a las complicaciones en la DM2:


a} La nefropatía es una complicación microangiopática

92d. En relación con el tratamiento en este paciente, señale el enunciado incorrecto:


a} La presión arterial se encuentra dentro dela meta para DM

93. Paciente femenino recién nacida producto de un embarazo prematuro de 32 semanas


que exhibe ictericia,exantema, rinitis persistente, anemia, linfadenopatía generalizada y
anormalidades óseas visibles en las radiografías. ¿Cuál de los siguientes factores es el más
importante durante el interrogatorio de la madre?
b} Antecedentes sexuales

94. Lactante de siete meses de edad que se presenta con diarrea y vómito. La madre
menciona que hace cinco días inició consumo de leche de fórmula; previamente sólo
había recibido leche materna. De acuerdo al diagnóstico del paciente, ¿qué componentes
de la leche son los responsables del cuadro clínico?
a} Beta lactoglobulina y caseína

95. ¿Cuál de los siguientes tipos de catarata se asocia con la enfermedad de Wilson?
b} Catarata de girasol

96. Paciente masculino de 72 años de edad acude a consulta por mareo y ojo rojo crónico
con disminución de la agudeza visual. Se realiza un angiograma, el cual demuestra retraso
en la visualización de los vasos retinianos con hemorragias en el polo posterior y periferia.
¿Cuál es la condición con más probabilidad de ser responsable de este cuadro?
a} Estenosis carotídea

97. Niño de tres años de edad con probable reflujo vesicoureteral, se solicita cistografía
ureteral miccional y se observa reflujo bilateral hacia uréteres dilatados y tortuosos con
dilatación de pelvis y cálices renales. De acuerdo con la clasificación de reflujo
vesicoureteral del paciente, ¿qué abordaje terapéutico es más adecuado?
d} Cirugía antirreflujo

98. Paciente masculino de 47 años de edad con antecedente de depresión en tratamiento


farmacológico no especificado, abandonado hace 6 meses. Acude traído por paramédicos
al encontrarse en su domicilio inconciente, se identifica una caja de paracetamol vacía. A
su ingreso con Glasgow de 13 puntos,ictérico, afebril, diaforético, taquicárdico, murmullo
vesicular conservado. Abdomen blando depresible no doloroso a la palpacion superficial
no profunda, presencia de hepatomegalia. Extremidades sin edema, pulsos distales
presentes, llenado capilar 2 segundos. FC 125 lpm, FR 25 rpm, temp 36.5 ºC, PA 90/60 mm
Hg, peso 87 kg, talla 1.62 m.
98a. ¿Cuál es su impresión diagnóstica?
b} Intoxicación por paracetamol

98b.¿Cuál es el metabolito tóxico del paracetamol?


b) Acetil parabenzoquinoneimina

98c. ¿Cuál es la dosis tóxica de paracetamol en adultos?


c} 7g en 24h

98d. ¿Cuál es la etapa de intoxicación en la que se encuentra nuestro paciente?


b} Estadio II

98e. ¿Cuál es el antídoto y su dosis?


a} N acetil cisteína 140mg/kg inicial 70mg/kg cada 4h,17 tomas
99. Paciente femenina de 18 años de edad. Sin antecedentes patológicos. Estudiante.
Consulta por cefalea bilateral opresiva de 36 h de evolución, intermitente, la mayor parte
del día, y dificultad para conciliar el sueño, sin embargo el dolor no la despierta una vez
dormida; lo anterior se acompaña de náusea sin llegar al vómito. TA: 110/70 mm Hg, FC:
75 x', FR: 14 x', Temp: 36.5 ºC. Exploración neurológica sin alteraciones. No ha consumido
medicamentos y se presenta sin laboratorios.
99a. Por la semiología del dolor,usted hace el diagnóstico presuntivo de:
b} Cefalea tensional

99c. ¿Cuál de los siguientes enunciados es cierto respecto a la cefalea tensional?


b} Es más frecuente en mujeres

99d. ¿Cuál es el tratamiento para disminuir la sintomatología en esta paciente?


c} AINE

99e. En relación con la fisiopatología de la cefalea tensional,señale la respuesta correcta:


a} No se conoce en detallelos aspectos fisiopatológicos

1OO. Paciente femenino de 88 años de edad es hospitalizada por una caída en su casa
debido a tropezón. No puede caminar o soportar peso sobre su extremidad inferior
derecha. Las radiografías revelan que tiene una fractura desplazada del cuello del
fémur,sin demostrar algún otro hallazgo de importancia. Al interrogario niega cualquier
otro traumatismo en la región afectada. Posterior a la estabilización de esta paciente,
¿cuál es el tratamiento más óptimo para su tipo de fractura?
d} Hemiartroplastia

101.Angélica, de 24 años de edad, refiere inicio de vida sexual activa hace 5 años y utiliza
anticonceptivo de barrera, su ciclo menstrual es irregular y no recuerda su fecha de última
menstruación, niega otros antecedentes; acude por dolor abdominal, que inició hace 8
horas, localizado en la fosa ilíaca derecha, tipo cólico, intensidad 8/1O que baja hasta
3/1O, se automedicó butilhiosina con mejoría parcial,se agregó anorexia,adinamia,
escalofrío, náusea y vómito, con intolerancia a la vía oral, niega otra sintomatología. A la
exploración presenta disminución de la peristalsis, dolor a la palpación de fosa ilíaca
derecha, con signo de von Blumberg positivo, resto negativos. FC 95 lpm, FR 22 rpm,
temp. 37.5 ºC, PA 130/65 mm Hg, peso 55 kg, talla 1.60 m.
101a. Usted considera que se trata de una apendicitis aguda, ¿cuál de los siguientes es el
diagnóstico diferencial a descartar?
b} Embarazo ectópico

101b. ¿Qué estudio de laboratorio solicita para descartar el embarazo ectópico?


c} Niveles de HGC-P

101c. ¿Qué estudio de imagen es de primera elección para confirmar apendicitis aguda?
c} Tomografía simple
101d. Se planea egresar al paciente a piso de Cirugía General después de la
apendicectomía. ¿Cuál es el tratamiento antimicrobiano de primera elección para los
pacientes con apendicitis aguda perforada?
a} Piperacilina-tazobactam

101e. Los laboratorios reportaron leucocitosis de 18,300 / mm3 con desviación a la


izquierda,con los datos clínicos y paraclínicos obtenidos ¿cuántos puntos de la clasificación
de Alvarado tiene la paciente?
c} 9 puntos

102. Paciente femenino de 33 años de edad que acude a consulta por presentar debilidad
y parestesias en los primeros tres dedos de la mano derecha. La paciente informa también
dolor localizado a la articulación de la muñeca. Durante el interrogatorio acepta que los
síntomas son peores cuando escribe en la computadora. Lo más probable es que la
paciente presente un síndrome del túnel del carpo. ¿Cuál de las siguientes pruebas es la
más sensible para realizar este diagnóstico?
b} Prueba de compresión

103. Masculino de 55 años de edad, fumador,dislipidémico, refiere dolor precordial


opresivo irradiado a mandíbula al ejercicio que desaparece con el reposo a los 5 min; lleva
2 meses con la misma sintomatología. TA: 145/95, FC: 88 lpm. Ruidos cardiacos rítmicos
sin soplos. Sin laboratorios.
103a. ¿Cuál es el diagnóstico clínico de este paciente?
a} Angina de pecho estable

103b. ¿En qué clase funcional se encuentra el paciente según la clasificación de la New
York Heart Association?
b} Clase II

103c. ¿Cuál es el estudio para continuar con el diagnóstico del paciente?

c} Prueba de esfuerzo

103d. Estándar de oro para la detección de cardiopatía isquémica:


c} Coronariografía

103e. ¿Cuál de los siguientes esquemas terapéuticos es el más indicado para el paciente?
a} Aspirina + estatina + IECA + betabloqueador

104. Paciente masculino de tres años de edad, con antecedente de otalgia y otorrea
derecha, desde los dos años de edad, en cuatro ocasiones las cuales remitieron con
manejo antimicrobiano. Refiere la madre cuadro de vías respiratorias superiores de 48 h
de evolución; acude por otalgia derecha. Exploración física. Temperatura 38 ºC. Otoscopia
derecha con membrana timpánica hiperémica, no móvil. Otoscopia izquierda con
membrana timpánica opaca y no móvil. Cavidad oral con amígdalas grado 3, crípticas sin
exudados. Cuello sin adenomegalias.
104a. El diagnóstico más probable es:
e} Otitis media aguda recurrente

104b. El microorganismo con más frecuencia implicado en esta patología es


c} Streptococcus pneumoniae

104c. El tratamiento consiste en:


c} Amoxicilina

104. Paciente masculino de tres años de edad, con antecedente de otalgia y otorrea
derecha, desde los dos años de edad, en cuatro ocasiones las cuales remitieron con
manejo antimicrobiano. Refiere la madre cuadro de vías respiratorias superiores de 48 h
de evolución; acude por otalgia derecha. Exploración física. Temperatura 38 ºC. Otoscopia
derecha con membrana timpánica hiperémica, no móvil. Otoscopia izquierda con
membrana timpánica opaca y no móvil. Cavidad oral con amígdalas grado 3, crípticas sin
exudados. Cuello sin adenomegalias.
104d. Dos meses después el paciente es llevado a consulta porque presenta sensación de
plenitud aural y la madre refiere que escucha el televisor con un volumen más alto del
habitual. En la otoscopia se observan las membranas timpánicas íntegras, opacas y no
móviles. Usted solicita una impedanciometría que reporta disminución en la complianza
de la membrana timpánica. La conducta siguiente sería:
a} Manejo expectante

105. Renata es una paciente de 27 años de edad que acude al servicio de Urgencias por
presentar pérdida súbita de la agudeza visual de forma completa con dolor
retroocular,que se acompaña de debilidad de las extremidades inferiores, misma que
limita la deambulación. Al interrogatorio dirigido niega otros antecedentes, alergias,
infecciones y vacunación reciente. La exploración visual revela alteraciones en la
campimetría por confrontación,el fondo de ojo con borramiento de los bordes y eritema
de la papila; la exploración neurológica es normal. FC 75 lpm, FR 18 rpm, temp. 36.5 ºC,
PA 110/65 mm Hg, peso 58 kg, talla 1.67 m.
105a. Con base en el cuadro clínico, ¿cuál es su diagnóstico presuntivo?
d} Eclerosis múltiple

105b. Según las manifestaciones clínicas pueden seguir uno de los cuatro patrones
característicos, ¿cuál es el más común?
a} Brote-remisión

105c. Para establecer el diagnóstico de la paciente usted solicita:


c} Resonancia magnética de cráneo
105d. ¿Cuál de los siguientes es un diagnóstico diferencial de la esclerosis múltiple?
a} Vasculitis

105e. ¿Cuál es el tratamiento de elección para brotes comunes?


d} Metilprednisolona
106. Paciente masculino de 70 años que acude a consulta por problemas de visión. Refiere
que en los últimos meses ha notado visión borrosa "justo en el frente" de su campo
visual,que no se puede corregir con sus lentes. Este problema le afecta ambos ojos. En sus
antecedentes patológicos menciona hipertensión,tratada con amlodipino. Refiere
tabaquismo de 20 paquetes/año. Al mostrarle la cartilla de Amsler dice notar un área
borrosa y asimétrica cerca del centro. A la oftalmoscopia se observan depósitos de drusen
en la mácula y áreas de depigmentación. No hay datos de neovascularización. ¿Cuál sería
el tratamiento indicado en este paciente?

e} Observación

107. Un hombre de 68 años que se encuentra hospitalizado por sufrir un infarto anterior.
Como tratamiento se le realizó un cateterismo y se le colocó un stent. Llaman durante la
noche porque el paciente se queja de disnea. A la exploración física sus signos vitales
incluyen FC 52/min, FR 21/min,T 37.3 C y PA 108/69 mm Hg, con saturación de oxígeno de
88%. A la auscultación se escucha S1 y S2 sin agregados. Los campos pulmonares se
auscultan con estertores crepitante Se le realiza un trazo de electrocardiograma, que se
muestra a continuación:

¿Cuál de las siguientes opciones representa la conducta más adecuada?


c} Colocación de marcapaso transcutáneo

108. Un hombre de 71 años acude a consulta por lumbalgia. Refiere que el dolor inició
hace cuatro meses, localizado en el área central de la espalda baja. En su historia clínica
refiere hipertensión tratada con hidroclorotiazida y osteoartritis. Al interrogatorio refiere
sentirse un poco más cansado de lo normal, y dolor intermitente de sus extremidades.
Niega incontinencia urinaria, parestesias, o cambios de la sensibilidad. A la exploración
física se observan nódulos de Heberdeen y de Bouchard. No hay dolor a la palpación de la
columna dorsal o el resto de la espalda. La sensibilidad perianal está intacta, así como el
resto de la exploración neurológica. En la radiografía se observa osteofitosis de la columna
vertebral, sin otras anormalidades aparentes. Se realizan estudios de laboratorio, que se
muestran a continuación:

¿Cuál de estos estudios diagnósticos estaría indicado en este paciente?


a} Electroforesis de proteínas séricas

109. Un hombre de 81 años acude a consulta por "ronchas". Él es residente de un asilo y


se ha reportado que estas lesiones han ocurrido recientemente en otros residentes. Los
antecedentes patológicos del paciente incluyen hipertensión, hiperlipidemia,enfermedad
coronaria y enfermedad de Alzheimer. El paciente no es capaz de responder el IPAS,
aunque la enfermera que lo acompaña niega fiebre, o síntomas de infección de las vías
respiratorias superiores. A la exploración física sus signos vitales incluyen FC 81/min, FR
16/min, T 37.2C y PA 130/76 mm Hg. Se observan en los brazos múltiples lesiones
papulares en superficies extensoras e interdigitales, con excoriaciones circundantes. ¿Cuál
sería el manejo de este paciente?
a} lvermectina dosis única a él y a todos los residentes y trabajadores del asilo

11O. Paciente masculino de 67 años que se presenta al consultorio, su preocupación


principal es que un amigo acaba de morir de un infarto cardiaco y quiere hacer todo lo
que pueda para prevenirlo. Su padre murió de un infarto agudo de miocardio a los 74
años. Niega alguna enfermedad. Refiere tabaquismo de 24 paquetes/año. A la exploración
física sus signos vitales incluyen FC 72/min, FR 16/min,T 37.2 C y PA 126/78 mm Hg.
Refiere que nunca ha tenido una lectura de presión arterial arriba de 130/80 mm Hg en
sus mediciones previas. Su última medición de HDL fue de 50 mg/dl.A la exploración física,
se observa un paciente delgado, y no se detectan anormalidades. ¿Cuál sería la meta de
LDL para este paciente?
a} LDL s 130 mg/dl
111. Paciente femenino de 67 años que acude a urgencias por náusea y vómito. Refiere
que el padecimiento inició hace tres días, y ocurre de manera intermitente. Niega fiebre o
hematemesis; también diarrea, refiriendo que no ha tenido evacuaciones en cinco días. En
sus antecedentes patológicos tiene historia de colelitiasis diagnosticada hace dos meses,
pero negó tratamiento quirúrgico. A la exploración física sus signos vitales incluyen FC
104/min, FR 19/min, T 38.3 C y PA 113/70 mm Hg. No hay anormalidades a la exploración
torácica. En la búsqueda abdominal se observa un abdomen distendido, timpánico, con
dolor difuso a la palpación y peristalsis aumentada. ¿Cuál es el estudio inicial más
apropiado para esta paciente?

d} Radiografía simple de abdomen

112. Paciente masculino de 67 años que acude a consulta por fiebre y disnea. Refiere que
inició hace dos días, con fiebre y tos, pero actualmente tiene dificultad para respirar.
También cuenta que expectora flema amarillenta. Cuenta con tabaquismo de 24
paquetes/año e ingerir tres cervezas diarias. Niega antecedentes médicos de importancia.
El único medicamento que toma es aspirina. A la exploración física sus signos vitales son
FC 101/min, FR 21/min,T 38.6 C, PA 133/80 mm Hg. A la exploración física se auscultan
estertores crepitantes. En una placa de tórax se muestra una consolidación en el lóbulo
medio del pulmón derecho. ¿Cuál es la etiología más común de este padecimiento en
pacientes de este grupo de edad?
d} Streptococcus pneumoniae

113. Paciente femenino de 73 años que acude a urgencias por cefalea. Inició hace unas
horas, pero ha ido aumentando en intensidad y no mejoró con dos tabletas de
paracetamol de 500 mg. El dolor es unilateral,y cuando señala donde coloca su mano
frente a su ojo del lado derecho. También refiere algo de visión borrosa y náusea. Niega
que el dolor hubiera ocurrido antes de esa manera. Al IPAS niega vómito, fotofobia y
sonofobia. Al examen de agudeza visual no puede identificar números ni letras, sólo
movimientos de la mano. Se observa inyección conjuntiva! ocular y una pupila dilatada, no
reactiva a la luz. Sus signos vitales incluyen FC 103/min, FR 18/min, T 37.1 C, PA 134/79
mm Hg. El resto de los pares craneales, incluyendo los movimientos oculares, está intacto.
No hay anormalidades en el resto de la exploración neurológica. ¿Cuál sería el siguiente
paso en el manejo de este paciente?
d} Medición de la presión intraocular

114. ¿Cuál es la vasculitis a la que el siguiente enunciado hace referencia?: conocida


también como angeítis y granulomatosis alérgica, caracterizada por la presencia de asma,
eosinofilia tisular y periférica, formación de granulomas extravasculares y vasculitis de
varios órganos y sistemas.
a} Síndrome de Churg-Strauss

115. Paciente femenino de 7 meses de edad es traída a consulta. La madre refiere que ha
presentado cuadros frecuentes de vómito, movimientos no coordinados y crisis
convulsivas; menciona que el cabello se le ha vuelto más claro en comparación al resto de
sus hermanos. En la exploración física se identifica piel seborreica, microcefalia, maxilar
prominente, dientes espaciados y retraso en el crecimiento.
115a. De acuerdo con las características clínicas, ¿cuál es el trastorno más probable de
esta paciente?
d} Fenilcetonuria

115b. ¿Con cuál de los siguientes exámenes de laboratorio confirmaría el diagnóstico?


a} Niveles de fenilalanina en plasma

115c. Con base en su sospecha clínica, ¿qué tratamiento indicaría?


c} Dieta libre de fenilalanina y cofactor tetrahidrobiopterina

115d. ¿Qué tipo de transmisión tiene este padecimiento?


b} Rasgo autosómico recesivo

116. Paciente masculino de 35 años de edad que regresa de viaje de luna de miel en
África. A su regreso el paciente presenta cefalea y fiebre por lo que acude a servicio
médico. El paciente es diagnosticado con infección por Plasmodium falciparum. En este
momento el paciente presenta un hematocrito de 18%, hiperbilirrubinemia de 9 mg/dL y
creatinina sérica de 2.7 mg/dL. El paciente es ingresado con monitorización de la función
renal y administración de fenobarbital como profilaxis de crisis convulsivas. ¿Cuál de los
siguientes antibióticos es el más adecuado para este paciente?
e} Artesunato

117. En varias ocasiones la hipotermia grave no es diagnosticada durante la exploración


física inicial. La temperatura se puede reportar como el límite inferior del termómetro, lo
cual puede ser de tan sólo unos cuantos grados centígrados por debajo de lo normal.
¿Cuál de los siguientes es suficientemente adecuado para indicar el diagnóstico de
hipotermia?
d} Electrocardiograma

118. ¿Cuál es el principal efecto secundario de la administración de abacavir en pacientes


con infección del virus de la inmunodeficiencia humana?
b} Reacción de hipersensibilidad severa
1. Paciente del sexo masculino de 49 años de edad en quien se encuentra como hallazgo
no intencionado en una radiografía de abdomen una masa calcificada, irregular y de gran
tamaño con múltiples proyecciones en la pelvis renal. ¿Cuál de los siguientes hallazgos es
el más probable de encontrar en los estudios de laboratorio de este paciente?
e} Exceso de amoniaco en orina

2. Neonato de tres días de vida con primer tono cardiaco fuerte, soplo sistólico de
hiperflujo pulmonar y desdoblamiento fijo del segundo tono. Se sospecha de una
comunicación interauricular. Respecto a esta condición, señale cuál de las siguientes es
FALSA.
d} El foramen oval permeable es una variante normal sin repercusiones fisiopatológicas

3. ¿Cuál de las siguientes formas de porfiria se caracteriza por herencia autosómica


recesiva?
a} Deficiencia de 5-ALA dehidratasa

4. Paciente masculino de 70 años de edad que es llevado al servicio de urgencia por


presentar desde hace 72 horas secreción purulenta,dolor y eritema intenso en el ojo
izquierdo. El paciente menciona que hace algunas semanas se le realizó una cirugía de
catarata. La agudeza visual de este paciente es 20/200 en el ojo izquierdo. A la inspección
el ojo se presenta intensamente eritematoso con edema de los tejidos perioculares, así
como presencia de un líquido lechoso en el interior del ojo que se acumula en la cámara
anterior. ¿Cuál de los siguientes diagnósticos es el más probable en este paciente?
d} Endoftalmitis bacteriana

5. ¿Cuál de los siguientes antimicóticos depende de la incorporación de ergosterol en la


membrana del eucarionte para ser efectivo?
a} Anfotericina B

6. Se presenta al servicio de urgencias una paciente del sexo femenino de 52 años de


edad, conocida por el servicio de reumatología por diagnóstico de artritis reumatoide de
hace varios años. La paciente acude porque presenta dolor epigástrico intenso tipo
ardoroso, náusea y vómito. La paciente menciona que decidió acudir porque el vómito no
ha cedido en las últimas 24 horas, después que consumió dos copas de vino durante la
cena. En la última ocasión el vómito fue sanguinolento. Se decide realizar una endoscopia,
la cual revela una gastritis aguda. ¿Cuál de las siguientes condiciones explica de mejor
manera la causa de la gastritis aguda en esta paciente?
c} Antiinflamatorios no
7. Paciente masculino de 77 años de edad se presenta a consulta quirúrgica con
diagnóstico de diverticulosis, pero con antecedente de diverticulitis en varias ocasiones.
Se le somete a una resección sigmoidea electiva. Después de la cirugía la enfermera coloca
al paciente una infusión hecha de 500 mL de solución glucosada a 5% y 500 mL de solución
salina a una velocidad de 80 mUmin y registra volúmenes urinarios de 30 a 45 mUh. No
obstante, durante el segundo y el tercer días la cuantificación de orina disminuye a 1O a
15 mUh, a pesar de incrementar la infusión de soluciones intravenosas. Los estudios de
laboratorio muestran una acidosis (pH de 7.25), con leve hipocapnia (PaC02 x de 33 mm
Hg), disminución del bicarbonato sérico (16 mEq/L), hiperpotasemia (5.9 mEq/L),
elevación de creatinina (5.9 mg/dL) y del nitrógeno ureico en sangre (77 mg/dL); la orina
tiene una concentración de sodio de 90 mEq/L. ¿Cuál de los siguientes diagnósticos es el
más probable en este caso?
a} Insuficiencia renal aguda

8. Paciente femenino de 14 años de edad padece tos crónica productiva,con aspecto


purulento, de aproximadamente 5 años de evolución, así como múltiples episodios de
neumopatía aguda por infecciones. De hecho hace un mes sufrió una nueva exacerbación
bronquial asociada a fiebre recurrente, malestar general,intolerancia al ejercicio y pérdida
de peso progresiva. En la exploración física se registraTemp: 38ºC, taquipnea en reposo y
saturación de 88% con aire ambiental,además de observarse desnutrida, con
crepitaciones basales bilaterales.En los estudios complementarios resalta: lgG muy baja,
lgA baja, lgM normal,citometría de flujo con linfocitos B normales.
8a. Con base en las características de la paciente, ¿cuál es el diagnóstico más probable?
b} Inmunodeficiencia común variable

8b. ¿Qué hallazgos apoyan su diagnóstico en los estudios complementarios?


a} Niveles bajos delgG e lgA y linfocitos B normales

8c. De acuerdo con su sospecha clínica,¿cuál es la alteración genética asociada?


b} No se ha identificado

8d. En relación con su sospecha clínica, ¿cuál es el tratamiento que estaría indicado en
esta paciente?
c) lnmunoglobulina intravenosa y evaluación de alteraciones autoinmunes

8e. De acuerdo con su sospecha clínica, ¿cuál es la complicación que se podría presentar
en la paciente?
d} Linfoma
9. Paciente masculino de 43 años de edad que se encuentra hospitalizado por presentar
celulitis de la extremidad inferior derecha. El paciente se mantuvo sin tratamiento durante
dos semanas hasta que decidió acudir al servicio de urgencias. Al momento del ingreso la
concentración de creatinina era de 1.2 mg/dl. Después de aproximadamente ocho días de
tratamiento con nafcilina, clindamicina y gentamicina la concentración de creatinina es de
3.5 mg/dl. La realización de un ultrasonido renal no revela anormalidades como
disminución del tamaño renal o hidronefrosis. Al revisar el expediente clínico es evidente
que el paciente presenta hipertensión (170/100 mm Hg), disminución del gasto urinario
(20 mUh). Los estudios de rutina revelan la presencia de hematuria y proteinuria,así como
la existencia de cilindros eritrocitarios y epiteliales.
¿Cuál es el diagnóstico más probable en este paciente?
b} Glomerulonefritis secundaria a la infección cutánea

1O. Se presenta a la consulta externa de pediatría un paciente del sexo masculino de 18


meses de edad para evaluación de rutina. Según la información proporcionada por los
padres y registrada en el expediente clínico, el paciente ha sido saludable desde el
nacimiento, y sus vacunas están actualizadas. La medición de la talla, peso y circunferencia
cefálica se encuentran en el percentil 50. A la exploración física es evidente que el
paciente ya puede caminar y correr; la madre menciona que ha iniciado a subir escaleras.
Al interrogatorio la madre niega que el niño salte o lance una pelota. El paciente es capaz
de sostener objetos con dedos y pulgares; cuando se le entregan cubos, puede agruparlos,
pero no puede crear una torre de dos cubos. ¿Cuál de las siguientes opciones es la más
correcta respecto al desarrollo motor de este paciente?

b} Desarrollo motor grueso normal pero con retraso del desarrollo motor fino

11. Paciente masculino de 70 años de edad que sufre una parálisis del miembro pélvico
izquierdo posterior a un infarto cerebral. A la exploración física revela respuesta flexora
para el reflejo de Babinski del lado derecho, pero respuesta extensora en el lado
izquierdo. La sensibilidad de la pierna izquierda se encuentra disminuida. ¿Cuál de las
siguientes arterias es la que con mayor probabilidad sufrió la obstrucción?
c} Arteria cerebral anterior

12. Paciente masculino de 55 años de edad con antecedente de consumo de 45 paquetes


de cigarros al año, se presenta a consulta por dolor en la región del hombro derecho y
debilidad en la mano derecha,así como constricción de la pupila derecha y ptosis del
mismo lado. ¿Cuál es el siguiente paso para realizar la confirmación diagnóstica?

c} Biopsia guiada por tomografía computarizada

13. ¿Cuál de los siguientes efectos adversos se relacionan principalmetne con la


administración de nevirapina y efavirenz?
e} Síndrome de Stevens-Johnson
14. Paciente masculino de 44 años de edad cuyo conteo más reciente de linfocitos T CD4+
es de 55 células/mm3. Se decide iniciar tratamiento indicado en este paciente; sin
embargo después de varias semanas el sujeto se presenta con hiperglucemia. ¿Cuál de los
siguientes fármacos el que más probablemente está causando este cuadro?
c} lndinavir

15. Durante un programa de revisión médica general en una compañía constructora se


detecta a un paciente masculino de 30 años de edad que presenta un soplo de eyección
mesosistólico en el foco pulmonar y a lo largo del borde esternal izquierdo. A la
exploración cardiovascular se detecta un latido intenso proveniente del lado derecho del
corazón con desdoblamiento fijo del segundo ruido cardiaco. Se solicita un
electrocardiograma el cual presenta desviación hacia la derecha del eje cardiaco; la
radiografía de tórax demuestra crecimiento del ventrículo y aurícula derechos. ¿Cuál de
los siguientes diagnósticos es el más probable en este paciente?
e} Comunicación interauricular

16. Paciente masculino de 31 años de edad se presenta a consulta por dolor faríngeo y
fiebre moderada. ¿Cuáles de los siguientes hallazgos apoyan el diagnóstico de faringitis
estreptocócica en este paciente?
a} Linfadenopatía cervical dolorosa y exudados faríngeos

17. ¿Cuál de los siguientes antiarrítmicos actúa directamente en los canales de calcio?
b} Verapamil

18. Paciente femenino de 50 años de edad con antecedente desde hace 1O años de
diabetes mellitus tipo 2. En esta ocasión se presenta por padecer dolor lumbar desde hace
varias semanas. Debido a que a su esposo se le recetó previamente indometacina 25 mg
tres veces al día, la paciente decidió automedicarse y utilizar el mismo esquema para el
dolor lumbar. Los resultados de los estudios de laboratorio de sangre son los siguientes:
sodio 136 mEq/L, potasio 6.2 mEq/L, cloro 106 mEq/L, bicarbonato 18 mEq/L, nitrógeno
ureico 55 mg/dl,creatinina sérica 5.3 mg/dl. Al comparar estos resultados con los
registrados en el expediente de hace seis meses es evidente un aumento considerable en
la concentración de creatinina en sangre (incremento de 2 mg/dl). ¿Cuál de los siguientes
enunciados son correctos respecto a la nefritis intersticial secundaria a la administración
de antiinflamatorios no esteroideos?
a} La hematuria se presenta frecuentemente enla nefritisintersticial por
antiinflamatorios no esteroideos
19. Paciente masculino de 70 años que se presenta al consultorio por presentar desde
hace aproximadamente cuatro meses dificultad para iniciar la micción, así como goteo. El
paciente decidió acudir hasta ahora porque antes los síntomas no eran tan intensos como
ahora, los cuales están afectando intensamente su calidad de vida. Se decide iniciar
tratamiento con finasterida. ¿Cuál de los siguientes enunciados describe de mejor forma
el mecanismo de acción de la finasterida?
a} Disminución dela reducción periférica de andrógenos

20.¿Cuálde las siguientes lesiones se observa como componente del síndrome de


Gardner?
d} Osteocondroma

21. ¿Cuál de las siguientes condiciones se asocia con disminución de la capacidad


pulmonar total, disminución del volumen corriente, disminución del volumen residual e
incremento del índice fracción espirada de volumen en el primer segundo/capacidad
vitalforzada?
c} Fibrosis pulmonar idiopática

22. Paciente femenino de 85 años de edad la cual presenta diagnóstico de diabetes


mellitus tipo 2 desde hace 20 años. Actualmente la paciente padece insuficiencia renal
crónica sin necesidad de tratamiento sustitutivo. Desde hace 24 horas presenta disnea,
edema periférico, fiebre que va en aumento, disuria y frecuencia urinaria. El equipo
médico tratante le administra gentamicina como tratamiento de su proceso infeccioso.
Afortunadamente la paciente mejora y es egresada,sin embargo ésta sufre disminución
significativa de la audición. ¿Cuál de los siguientes medicamentos es más probable que se
le haya administrado a esta paciente durante su hospitalización y que sea responsable de
la sordera?
d} Ácido etacrínico

23. Paciente masculino de 68 años de edad que se presenta a consulta por presentar
disfagia desde hace seis meses así como pérdida no intencionada de aproximadamente 1O
kg. Entre los antecedentes de importancia se encuentran tabaquismo intenso desde los 20
años de edad, consumo diario de bebidas alcohólicas y sintomatología de reflujo
gastroesofágico no tratada desde hace varios años. A la inspección se encuentra un
paciente obeso (a pesar de la pérdida no intencional de peso). Al revisar el expediente
médico se reporta que en el pasado este paciente sufrió acalasia. En este momento se
sospecha de la presencia de cáncer esofágico. ¿Cuál de los siguientes factores de riesgo es
más importante para la presentación de un cáncer esofágico de células escamosas sobre
un adenocarcinoma?
d} Antecedente de acalasia
24. En una población "X" existe una frecuencia de portadores de la fibrosis quística de
1/30 habitantes. Por otro lado en la población "Y" existe una frecuencia de portadores de
fibrosis quística de 1/100 habitantes. ¿Cuál es la probabilidad que existe de tener un hijo
con la enfermedad de fibrosis quística en caso de que su madre sea de la población "X" y
su padre de la población "Y"?
a} 1/12 000

25. ¿Cuál de los siguientes trastornos de la hemostasia es ocasionado por una deficiente
adhesión plaquetaria al subendotelio, la cual puede ser tratada mediante la
administración de crioprecipitados?
b} Enfermedad de von Willebrand

26. ¿Cuál de los siguientes fármacos se asocia con los receptores alfa-1, alfa-2 y beta-1
adrenérgicos, pero no con los beta-2 adrenérgicos?
b} Norepinefrina

27. Paciente de 79 años de edad el cual fue diagnosticado hace cinco años con
enfermedad de Parkinson. En esta ocasión el paciente acude debido a que ha estado
presentando insomnio desde hace tres meses, lo cual ha deteriorado considerablemente
su calidad de vida. ¿Cuál de los siguientes medicamentos no se debería administrar en
este paciente debido a que incrementaría demasiado su riesgo de caídas?
a} Clorodiazepóxido

28. Femenino de 55 años, llevada al departamento de Urgencias por presentar alteración


del estado mental caracterizado por somnolencia, confusión y letargo. Los familiares
mencionan que recientemente había estado consumiendo volúmenes elevados de agua y
con incremento de los volúmenes urinarios. Entre los antecedentes de importancia se
sabe que la paciente padece un trastorno esquizoafectivo y retraso mental leve. Entre los
medicamentos que consume actualmente se encuentran divalproex, litio, risperidona,
mesilato de benztropina, quetiapina y clonazepam. En este momento la paciente está
consciente y alerta. A la exploración física se encuentra taquicardia de 105 latidos/minuto
y mucosas deshidratadas. Los estudios de laboratorio muestran una concentración de
sodio de 147 mmol/L y glucosa en ayunas de 82 mg/dL. La osmolalidad sérica es de 304
mmol/kg. ¿Cuál de los siguientes exámenes es el más adecuado para realizar el
diagnóstico de esta paciente?
d} Examen de deprivación hídrica

29. Paciente recién nacido de 5 horas de vida, producto de una gestación de 28 semanas.
El paciente muere a la sexta hora de vida. En la autopsia se reporta fenotipo caracterizado
por aplanamiento facialy exceso de piel en la parte posterior de cuello, así como atresia
duodenal y comunicación interventricular. ¿Cuál de las siguientes alteraciones
cromosómicas es la más probable que presente este paciente?
a} Trisomía 21
30. ¿Cuál es el mecanismo de acción por el que la amantidina puede mejorar la
sintomatología de la enfermedad de Parkinson?
a} Estimulación de la liberación endógena de dopamina

31. Masculino de 20 días de nacido es llevado a consulta por secreción en los ojos y fiebre.
Como antecedente, la madre refiere mal control prenatal, parto eutócico a las 38 semanas
de gestación y Apgar de 8/1O. En la exploración física resalta la secreción purulenta
espesa en ambos ojos,locual dificultasu apertura y evaluación (véase Figura), y la
conjuntiva se encuentra muy hiperémica. Además, después de revisar los ojos usted
realiza una exploración completa y encuentra a la auscultación torácica estertores
bilaterales y taquipnea. Confirma la presencia de fiebre. El frotis del exudado reporta
polimorfonucleares y abundantes bacterias.

31a. Con base en las características del paciente, ¿cuál es el diagnóstico más probable?
b} Oftalmía neonatal

31b. De acuerdo con su sospecha clínica, ¿cuál es el tratamiento que indicaría?


a} Antibióticos sistémicos

31c. En relación con su sospecha clínica, ¿cuál es la causa más frecuente?


c} Chlamydia trachomatis

31d. De acuerdo con su sospecha clínica,¿cuál es la complicación que se puede presentar


en este paciente?
a} Neumonía

31e. En conclusión, ¿cuál de los siguientes conjuntos de signos y síntomas orientó su


sospecha diagnóstica?
a} Exudado mucopurulento, neutrófilos en frotis y estertores pulmonares bilaterales
32. Paciente de 15 años de edad que se presenta con temblor y marcha inestable. Los
estudios de laboratorio son reveladores por la presencia de elevación de las enzimas
hepáticas y disminución de la concentración de ceruloplasmina. ¿Cuál de los siguientes
medicamentos es el más adecuado para este paciente?
a} Penicilamina

33. ¿Cuál de los siguientes medicamentos se asocia con diabetes insípida nefrogénica
como efecto adverso?
b} Litio

34. ¿Cuál de los siguientes trastornos se caracteriza por niveles en rangos normales de
hormona luteinizante, foliculoestimulante y testosterona en combinación con un bajo
conteo de espermatozoides?
b} Obstrucción de la salida dellíquido seminal

35. Paciente femenino de 67 años de edad con neumonía adquirida en la comunidad por
Klebsiella pneumoniae. A pesar de que se ha iniciado tratamiento con cefotaxima, la
paciente no muestra mejoría y los hemocultivos siguen resultando positivos para el
organismo causal. Se decide administrar un segundo medicamento. Cuarenta y ocho horas
después de iniciar el segundo medicamento los estudios de laboratorio de la paciente son
los siguientes: sodio 138 mEq/L, potasio 4 mEq/L, cloro 100 mEq/L, bicarbonato 25 mEq/L,
nitrógeno ureico en sangre 49 mg/dL, creatinina 5.5 mg/dL. ¿Cuál de los siguientes
antibióticos fue el que con mayor probabilidad se le administró a esta paciente?
b} Tobramicina

36. ¿Cuál de los siguientes antiarrítmicos ejerce su efecto farmacológico únicamente


durante la fase O del potencial de acción?
d} Flecainamida

37. Masculino de 18 años acude a consulta por cuadro de 7 horas, dolor abdominal en
cuadrante inferior derecho. En la nota médica no se especifica el tipo, intensidad,
irradiaciones, con que se aumenta o disminuye el dolor. Se prescribió analgésico en caso
de continuar igual. Regresó a las 6 horas con más dolor y por ello lo internan. En el
expediente se reportó abdomen blando, depresible, hiperestesia en hipogastrio, rebote
negativo, peristaltismo audible. La ultrasonografía abdominal con discreta ectasia piélica y
ureteral. Se concluyó pielonefritis derecha agudizada y datos de colon espástico. BH con
12300 leucos, bandas 11%, examen general de orina normal. Se estableció diagnóstico de
colitisvs. apendicitis. Se dejó hospitalizado e inició tratamiento con gentamicina,
análgesico y dieta líquida. Cuatro horas después, fiebre de 39.9ºC. Continúa refiriendo
dolor abdominal pero no se reporta en el expediente, 12 h después se agregan
evacuaciones diarreicas, sin mayor semiología. Se agregó antiespasmódico y antidiarreico.
Dos días después permanece afebril, sin datos de irritación peritoneal por lo que fue dado
de alta con los mismos medicamentos y dieta blanda. A las 24 h de haberse egresado,
continuó con mismo cuadro y reapareció la fiebre. Al cuarto día de esto fue con otro
médico y a través de TAC se estableció el diagnóstico de apendicitis complicada con
absceso pélvico. Se informó al paciente y familiares y se intervino quirúrgicamente. El
hallazgo quirúrgico fue absceso pélvico de 300 ml, de material purulento, apéndice
perforado en su base, sellada por ciego y vejiga. El paciente evolucionó hacia la mejoría y
fue dado de alta días después.
37a. Para la correcta aplicación de la NOM-004 del expediente clínico, se toman en cuenta
los principios éticos y científicos que orientan la práctica médica, especialmente en la
libertad prescriptiva, en beneficio del usuario y es de observancia obligatoria para:
d} Los sectores público, social y privado, incluidos los consultorios

37b. De acuerdo con lo normado para el expediente clínico, el documento inicial que en
este caso no se menciona es:
b} La historia clínica

37c. El padecimiento actual,la semiología de cada síntoma, la exploración física, los


resultados previos y actuales de laboratorio, los diagnósticos o problemas clínicos, el
pronóstico e indicaciones terapéuticas era necesario haberlos anotado en:
c} La historia clínica

37d. De acuerdo con la NOM-004 del expediente clínico las notas de: interconsulta,
referencia/traslado, notas médicas de urgencia, preoperatoria, preanestesia y
posoperatoria son responsabilidad exclusiva de:
c} Los médicos tratantes

37e. En el presente caso, el expediente clínico tuvo múltiples deficiencias que denotan la
falta de habilidad clínica de los médicos. Cuál será una recomendación que recibirá esta
institución para que mejore la calidad de la atención medica.
b} La aplicación delas Guías de Práctica Clínica

38. ¿Cuál de las siguientes enzimas requiere de la vitamina K como coenzima?


d} Gammaglutamil carboxilasa

39. Paciente masculino de 30 años que acude a la consulta por presentar dolor tipo cólico
de inicio agudo en el ángulo costovertebral izquierdo que irradia al escroto, además
presenta hematuria. En la radiografía simple de abdomen se visualiza una piedra en el
uréter izquierdo. Posteriormente el paciente expulsa el lito espontáneamente; se
encuentra que está compuesto de oxalato de calcio. Por lo que la etiología más probable
es la hipercalciuria idiopática. ¿Cuál de los siguientes enunciados es correcto?
b} Los litos radiopacos están compuestos de oxalato de calcio
40. Acude a consulta un hombre de 55 años por dolor profundo en la región cervical que
se extiende hacia la escápula derecha desde hace aproximadamente cinco meses. Al
interrogatorio el paciente menciona que el dolor inició después de que sufrió un accidente
en su automóvil. A la exploración física presenta normalidad en fuerza muscular y
sensibilidad. A pesar de que el paciente ha utilizado varios analgésicos, ninguno ha sido de
utilidad. ¿Cuál de las siguientes opciones es el diagnóstico más probable?
b} Enfermedad cervical derecha interapofisiaria

41. Paciente femenino de 87 años. Cuenta con antecedentes personales de enfermedad


de Alzheimer,fibrilación auricular, accidente cerebrovascular e incontinencia urinaria. La
paciente está consciente pero desorientada temporoespacial mente. Presenta leve
desnutrición, bien hidratada, con palidez cutánea, mucosa y úlceras por decúbito en sacro
y talones por la inmovilidad, así como incontinencia urinaria.¿Cuál de las siguiente no es
una causa de incontinencia aguda o persistente?
c} Ácido acetilsalicílico

42. Paciente femenino de 47 años. Fue operada de apendicitis aguda, rinoplastia,


amigdalectomía y colecistectomía. Le realizaron cesáreas a los 24 y 27 años. A los 32 años
de edad, le efectuaron la tercera cesárea con anestesia epidural,a nivel del espacio
intervertebral L3-L4; se administraron 15 mL de lidocaína a 2% durante el procedimiento.
La paciente presentó después en forma súbita una cefalea intensa de tipo opresivo y
global,asociada a sensación de "descarga eléctrica" en la columna lumbar con irradiación a
la nuca y la cabeza que obligaron a finalizar la intervención con anestesia general. En el
puerperio presentó incontinencia vesical.
Señale la aseveración falsa en relación con las diferencias en el diagnóstico entre las
incontinencias urinarias de tipo permanente:

e} La hiperactividad deldetrusor se debe ala enfermedad dela médula espinal,neuropatía


autonómica dela diabetes, alcoholismo, deficiencia de vitamina 812, Parkinson,tabes
dorsalis u obstrucción crónica del esfínter

43. Paciente de sexo femenino de 13 años de edad acude por epistaxis de repetición de
tipo severo(que requerirían atención en el departamento de Urgencias) y sangrado
menstrual profuso. En los exámenes de laboratorio se encuentra biometría hemática con
plaquetas normales, tiempo de protrombina normal,tiempo de tromboplastina
prolongado.
43a. Con base en las características dela paciente, ¿cuál es el diagnóstico más probable?
c} Enfermedad de von Willebrand
43b. ¿Qué hallazgos en los exámenes de laboratorio apoyan su sospecha diagnóstica?
a} PT normal,PTT prolongado, actividad de vWF disminuida, actividad de factor VIII
disminuida

43c. De acuerdo con su sospecha clínica,¿cuál es la complicación que se puede presentar a


largo plazo?
d} Anemia

43d. Con base en su sospecha clínica, ¿cuál es el tratamiento indicado?


d} Desmopresina

43e. ¿Qué aspecto del cuadro clínico dela paciente es relevante mencionar a los padres?
c} Expectativa de vida normal

44. Niño de 12 años con antecedente de dermatitis atópica que presenta exacerbación del
cuadro desde hace unos meses. ¿Cuál de los siguientes signos espera usted encontrar?
c} Pliegue de Dennie-Morgan y signo de Hertoghes

45. Se presenta ante usted una señora llevando una fotografía del tórax de su hijo para
preguntarle qué es lo que tiene.
45a. Con la siguiente imagen usted hace el diagnóstico de:
c} Pitiriasis versicolor

45b. Localizaciones anatómicas más frecuentes de la entidad patológica que presenta este
paciente.
a} Pecho y espalda
46. Un paciente de tres meses es llevado a la consulta pediátrica por presentar eritema y
edema en la zona del pañal con un margen definido en descamación que involucra la zona
perigenital y perianal,así como las regiones internas de los muslos. El paciente presenta
lesiones pustulares en el área cubierta por el pañal.

Fuente: Wolff K, Johnson RA, Suurmond D. Fitzpatrick's Color Atlas &"


Synopsis of Clinical Dermatology. 5th ed. http://www.accessmedicine.com
"Copyright © The McGraw-Hill Companies, lnc. All rights reservad."
46a. ¿Cuál es el diagnóstico más probable?
a} Dermatitis del pañal

46b. ¿Cuál es la conducta terapéutica más adecuada?


e} Nistatina tópica

47. Ramiro es paciente masculino de 74 años de edad, traído al servicio de Urgencias por
presentar síncope en su domicilio, su hija refiere que no mostró otros síntomas pero que
hace 8 años inició con hipertensión arterial sistémica con mal apego a tratamiento, hace 4
años tuvo un infarto agudo de miocardio que fue manejado con cateterismo y colocación
de stent en 2 arterias coronarias, desde entonces toma ácido acetilsalicílico 100 mg cada
24 horas diario. Ingresa a Urgencias con escala de coma de Glasgow de 12 puntos, ruidos
cardiacos con soplo sugestivo de estenosis aórtica, pulsos débiles y filiformes, llenado
capilar de 5 segundos. FC 30 lpm, FR 12 rpm, T 35.SºC, TA 80/40 mm Hg, peso 70 kg, talla
1.72 m, IMC 23.6 kg/m2.
47a. ¿Cuál es el diagnóstico más probable?
d} Bradicardia extrema, probable bloqueo AV
47. Ramiro es paciente masculino de 74 años de edad, traído al servicio de Urgencias por
presentar síncope en su domicilio, su hija refiere que no mostró otros síntomas pero que
hace 8 años inició con hipertensión arterial sistémica con mal apego a tratamiento, hace 4
años tuvo un infarto agudo de miocardio que fue manejado con cateterismo y colocación
de stent en 2 arterias coronarias, desde entonces toma ácido acetilsalicílico 100 mg cada
24 horas diario. Ingresa a Urgencias con escala de coma de Glasgow de 12 puntos, ruidos
cardiacos con soplo sugestivo de estenosis aórtica, pulsos débiles y filiformes, llenado
capilar de 5 segundos. FC 30 lpm, FR 12 rpm, T 35.SºC, TA 80/40 mm Hg, peso 70 kg, talla
1.72 m, IMC 23.6 kg/m2.
47b. El abordaje inicial del paciente requiere de un ECG para determinar el tipo de
bloqueo, aquí se presenta un ECG en D-1 y V-1:

Electrocardiograma (ECG) que muestra bloqueo AV de tercer grado.


-Señale el tipo de bloqueo AV que se observa en el trazo.
b} Bloqueo AV de tercer grado

47c. Ya que se trata de una emergencia,se monitorea al paciente con ECG continuo,
pulsioximetría, se debe colocar oxígeno complementario, tener cerca el carro rojo, una
vena permeable (o en su defecto un catéter venoso central) e iniciar el manejo.
¿Cuál es el fármaco de elección para el manejo inicial?
b} Atropina 1 mg IV

47d. El paciente continúa estable, con los datos de laboratorio obtenidos (encontrando los
siguientes datos: Biometría: Leucocitos 8.7, Neutrófilos 6.7, Linfocitos 2.1,Plaquetas
178000, Hemoglobina 13.2, Hematocrito 40.1;Química sanguínea: Glucosa 117, BUN 176,
Creatinina 7.4;Electrolitos: Na 137, K 6.1, CI 103;Enzimas cardiacas: prueba rápida
cualitativa negativa) usted decide que el mejor manejo en este momento es:
a} Diurético de asa, nebulización con salbutamol,solución polarizante

47e. Después de la adminitración del tratamiento, el bloqueo AV remite, y es momento de


pensar en el siguiente paso, lo más prudente será hacer una interconsulta a Nefrología
para el manejo integral de la lesión renal. ¿Qué fármaco recomendaría para manejo
ambulatorio?
a} Furosemida
48. Paciente masculino de 59 años de edad que acude a evaluación por sufrir disnea de
medianos esfuerzos y dolor torácico. En el expediente clínico no hay ningún antecedente
de importancia para este padecimiento. A la exploración física los pulsos de las carótidas
se encuentran retrasados respecto al ciclo cardiaco, así como disminuidos en intensidad.
El punto de máxima intensidad cardiaco se muestra hiperdinámico, pero sin
desplazamiento. A la auscultación se encuentra un soplo en crescendo-decrescendo a lo
largo del borde esternal izquierdo con máxima intensidad en telediástole. El componente
aórtico del segundo ruido cardiaco se encuentra ausente. En el electrocardiograma se
encuentran datos en relación a hipertrofia ventricular izquierda. La radiografía de tórax no
muestra incremento del tamaño de la silueta cardiaca y campos pulmonares sin
alteraciones. ¿Cuál de las siguientes opciones parece ser la causa más probable del cuadro
clínico de este paciente?
d} Válvula aórtica bivalva

49. ¿Cuál de las siguientes complicaciones del infarto miocárdico se presenta entre uno y
cinco días posteriores al infarto con disociación electromecánica?
c} Rotura dela pared libre ventricular izquierda

50. Con relación a la gota poliarticular simétrica:


d} Pueden cursar con factor reumatoide positivo a títulos bajos y en mujeres
posmenopáusicas

51. Un paciente infantil de 6 años de edad se presenta con pápulas de color carne sobre la
mano que no le producen prurito. La exploración física revela lesiones de
aproximadamente 4 mm de diámetro con umbilicación central. Se observa un halo
alrededor de las lesiones al someterlas a regresión.
¿Cuál de los siguientes es el diagnóstico más probable?
b} Molusco contagioso

52. Lactante de seis meses de edad que ha recibido, de acuerdo con el esquema de
vacunación 2007, la vacuna de poliovirus inactivos. En relación al motivo por el cual se ha
sustituido la vacuna oral de poliovirus (Sabin) por la vacuna de poliovirus inactivos,
indique la opción CORRECTA.
a} Por elriesgo de desarrollar poliomielitis paralítica asociada a la vacuna de poliovirus
oral

53. Un paciente femenino de 29 años de edad con antecedentes personales de


nefrolitiasis e hipercolesterolemia se ingresa al servicio de urgencias con una clínica de
cinco días de evolución de fiebre alta (39 ºC), acompañada de escalofríos (calosfríos),
cefalea retroorbitral, fotofobia, artralgias y exantema en ambas manos y abdomen. A su
ingreso el exantema había desaparecido. Faringe eritematosa y dolor a la palpación
profunda en epigastrio. En la sala de urgencias la paciente presenta alteración del estado
de alerta y posteriormente convulsiones tónico clónicas generalizadas las cuales son
controladas con medidas farmacológicas.
53a. ¿Cuál de las siguientes opciones es el abordaje inmediato más adecuado?
d} Inicio de tratamiento antibiótico empírico

53b. El equipo médico solicita una tomografía axial computarizada. Debido a que el
estudio de imagen es normal, se realiza una punción lumbar. En caso de que se trate de
una meningitis viral,¿cuáles serían los hallazgos más comunes en el líquido
cefalorraquídeo extraído?
b} Pleocitosis a base de células mononucleares, niveles de proteínas normales o elevados,
niveles de glucosa normales o disminuidos

53c. ¿Cuál es el mejor estudio diagnóstico para realizar el diagnóstico de meningitis y


encefalitis virales?
a} Amplificación delmaterial genético viral por medio de reacción en cadena dela
polimerasa

54. Paciente masculino de 30 años de edad, fumador, presenta malestar desde hace 3
meses, localizado en la parte superior del abdomen. Refieredicho malestar como ardoroso
y hace énfasis en que la molestia coincide con la ingesta de alimentos y se calma después
de un par de horas, o bien, cuando toma algo para la gastritis.
54a. La principal sospecha diagnóstica es:
d} Úlcera gástrica

54b. ¿Cuál es el estudio inicial de elección?


a} Endoscopia

54c. ¿Cuál es el estudio no invasivo de elección?


a} Tránsito de bario

54d. ¿Cuál es la complicación más frecuente de este padecimiento?


c} Hemorragia

54e. Principal factor patogénico (bacteriano) de H. pylori.


d} Ureasa

55. Paciente masculino de 25 años de edad, inicia padecimiento hace 4 semanas con un
cuadro de rinofaringitis que evolucionó a la curación sin complicaciones; a los 1O días
presenta malestar general,fiebre de 38 ºC y dolor retroesternal intermitente que ha
disminuido en frecuencia e intensidad. Actualmente se encuentra afebril,con ruidos
cardiacos frecuentes, ritmo e intensidad normal, sin ruidos agregados. Se realiza un ECG
que reporta inversión difusa de la onda T. Marcadores de inflamación normales."
55a. Señale el enunciado que corresponda a una manifestación electrocardiográfica
presente en la pericarditis:
b} Inversión de la onda T

55b. ¿En qué fase electrocardiográfica de la pericarditis se encuentra inversión de la onda


T?
c} 3

55c. Señale la opción que corresponde a la tríada de Beck:


d} Distensión venosa yugular + ruidos cardiacos disminuidos o ausentes + hipotensión

55d. ¿Cuánto debe disminuir la presión arterial sistólica a la inspiración para definir el
pulso paradójico?
b} > 10 mm Hg

55e. ¿Cuál es la conducta terapéutica a seguir en este paciente?


b} Observación

56. Masculino de 55 años con diagnóstico de ERGE desde hace 15 años, en tratamiento
con IBP; tambiénpadece diabetes desde hace 1º años, la cual se controla con metformina
e insulina 10 UI al día. Acude a consulta por sentir una bolita en la ingle derecha; comenta
que apareció hace un mes, pero como no le dolía y era bastante pequeña, no le dio
importancia.Hace una semana notó que la bolita había aumentado de tamaño y se
abultaba aúnmásal toser. EF: 1.68m, TA: 140/80, Temp: 36.5ºC. A la exploración se
encuentra tranquilo, consciente, orientado, sin hiperalgesia ni hiperbaralgesia, ruidos
intestinales de adecuada intensidad y frecuencia; se observa tumoración de 2 x 2cm en
región inguinal derecha, sin aumento de temperatura ni eritema.
56a. ¿A través de qué estructura se produce el defecto anatómico de este paciente?
d} Orificio miopectíneo de Fruchaud

56c. Tipo de hernias más comunes en hombres y mujeres, respectivamente:"


c}Inguinal indirecta y femoral

56d. El tratamiento para este paciente debe de ser:


b} Quirúrgico

56e. ¿En qué porcentaje se presentan las hernias posincisionales después de una cirugía
abdominal?
b} 10%

57. Paciente femenina de 20 años de edad, sin antecedentes patológicos, inicia hace 2
meses con fatiga y fiebre sin cuantificar, artralgia en ambas manos y rodillas, y aparición
de exantema elevado en V del escote. Hace una semana comienza con disnea de esfuerzo
que ha empeorado paulatinamente; también reporta palpitación y mareo. TA: 150/95, FC:
115 lpm, FR: 19 x min, Temp: 37.8 ºC, palidez generalizada, ictericia en escleras y mucosas,
sinovitis en ambos carpos, campos pulmonares y precordio sin alteraciones, miembros
inferiores con edema ++. Laboratorios: Hb: 6.5 g/dL, VCM: 11O ft, leucocitos: 3 000,
plaquetas: 100 000, reticulocitos: 8%, BT: 5.8, 81: 4.9, DHL: 1000. EGO reporta proteinuria,
hematuria y cilindros granulosos. Se sospecha LES.
57a. Señale los anticuerpos que por su alta sensibilidad se utilizan para realizar el
diagnóstico de LES:
a} Anticuerpos antinucleares

57b. Señale la clase de nefropatía lúpica más frecuente:


c} ClaseIV: nefritis proliferativa difusa

57c. En relación con las manifestaciones diseminadas de LES, señale la respuesta correcta:
c} La endocarditis de tipo fibrinosa asociada alupus se conoce como endocarditis de
Libman-Sacks

57d. Se realiza una biopsia renal que reporta nefritis lúpica proliferativa difusa. Con base
en esto, señale el tratamiento indicado:
d} Ciclofosfamida + glucocorticoide

57e. ¿Cuál es el autoanticuerpo que se relaciona con el lupus inducido por fármacos?
d} antihistona

58. Paciente masculino de 12 años de edad acude a consulta por dolor intenso en la rodilla
izquierda posterior a una lesión en un partido de fútbol hace ocho semanas. Refiere
edema a lo largo de la porción posterior de la rodilla afectada y en toda la pierna del lado
izquierdo. El dolor se incrementa cuando se involucra en actividades que incluyen
levantamiento de peso. Al interrogatorio niega fiebre, sudación nocturna o pérdida de
peso reciente. Al revisar el expediente, se encuentra que en el pasado padeció un
retinoblastoma, el cual fue tratado con defecto visual residual mínimo. Se solicita un
escaneo óseo con tecnecio, el cual muestra una lesión localizada en la epífisis distal del
fémur izquierdo. Después se solicita una resonancia magnética que demuestra que las
estructuras neurovasculares no se encuentran involucradas. ¿Cuál de las siguientes
opciones terapéuticas es la más adecuada para este paciente?
c} Resección extensa y quimioterapia

59. Paciente femenino de 33 años, sin antecedentes de importancia, acude a consulta por
haber presentado trauma con la uña de su hija de cinco meses de edad en ojo derecho,
posterior al cual refiere dolor,fotofobia, ojo rojo y abundante lagrimeo. A la exploración
oftalmológica se ubica hiperemia conjuntiva!, inyección ciliar, córnea con área de pérdida
de continuidad en el epitelio de 4 x 3.5 mm, cámara anterior formada sin celularidad,
pupila normorrefléctica, cristalino transparente, vítreo transparente, retina aplicada.
59a. ¿Con qué tinción superficial se establece el diagnóstico de defecto epitelial corneal?
d} Fluoresceína
59b.¿Cuálserá el tratamiento del trauma ocular con desepitelización cornea!?
e} A y B son correctas

60. Paciente masculino de 48 años de edad es traído a Urgencias por haber sufrido una
caída durante una competencia de motociclismo.EI accidente sucedió luego de perder el
control de la moto al dar una vuelta muy cerrada, lo que ocasionó que cayera de costado
sobre el pavimento; comenta que su reacción fue caer con la mano,para aminorar la
fuerza del impacto. En el lugar del accidente fue auxiliado por paramédicos, los cuales
descartaron lesiones cervicales, y al ver deformidad en el antebrazo derecho lo
trasladaron al servicio. Al realizar la exploración física se observa una deformidad en la
muñeca con hematoma, así como mucho dolor a la palpación,especialmente en la región
de la tabaquera anatómica,e imposibilidad para la movilización No se sabe enfermo de
nada; no fuma ni toma. EF: 1.80m; 95kg; TA: 140/85; Temp: 36.5ºC.
60a. ¿Cuál de las siguientes lesiones involucra únicamente una fractura del radio distal?
c} Goyrand-Smith

60b. ¿Qué estudios indicaria para corroborar su diagnostico?


b)Rayos X, placa antero-posterior y lateral

60c. En el estudio solicitado se reporta una fractura distal del radio, con una desviacion del
fragmento distal en direccion dorsal y radial, ademas de una fractura proximal en el hueso
semilunar; por la anterior. ¿Cuál lesion es posible que presenta este paciente?
c)Pouteau-Colles

60d. Con respecto a la fractura del escafoides, ¿cuál es la complicación de mayor


preocupación?
b} Necrosis avascular

60e. ¿Cuál manejo sería el indicado?


c} Reducción cerrada einmovilización

61. ¿Cuál de las siguientes opciones es la más correcta respecto al éxito del trasplante de
páncreas en pacientes con diabetes tipo 1?
e} Este tratamiento produce mantenimiento de niveles normales de glucosa sérica

62. Paciente masculino de 28 años de edad, refiere traumatismo nasal de cuatro días de
evolución. En la exploración física se observa laterorrinea izquierda menor a 50%. Refiere
obstrucción nasal izquierda. La siguiente conducta sería:
c} Reducción cerrada
63. Femenino de 55 años de edad acude a consulta por presentar desde hace 5 días
lagrimeo excesivo del ojo izquierdo, asociado con inflamación y dolor en el lado interno
del mismo. Comenta que desde hace una semana despierta con lagaña muy amarillenta,
la cual le cuesta trabajo quitar. No refiere fotofobia ni cambios en la agudeza visual. La piel
del ángulo interno del ojo afectado se nota hiperémica, y al ejercer presión sobre esta
zona se nota que despierta dolor y se corrobora la tumoración.
63a. ¿Cuál es su sospecha diagnóstica?
d} Dacriocistitis

63c. La prueba donde se instila fluoresceína en el ojo y se le pide al paciente que suene
(sople) su nariz, después de 5 min es:
b} Jones 1

63d. ¿Cuál de las siguientes opciones NO es parte del manejo de este padecimiento?
d} Esteroides sistémicos

63e. Con respecto a la dacrioestenosis, es cierto que:

d} a y b

64. Paciente masculino de 66 años que acude a consulta para preguntar sobre la vacuna
del neumococo. Él es actualmente un empresario retirado. Niega antecedentes
patológicos de importancia. Refiere haber requerido una colecistectomía por colecistitis
hace 20 años y una apendectomía cuando era joven. Menciona tabaquismo, alrededor de
uno o dos cigarros diarios desde que tenía 21 años. Niega consumo de alcohol. Dice correr
30 minutos diarios. Su última colonoscopia fue hace seis años y su último antígeno
prostático hace un año fue de 1.O ng/ml. Niega tos, fiebre, hemoptisis o pérdida de peso.
¿Cuál de las siguientes aseveraciones sobre el uso de la vacuna del neumococo en adultos
es falsa?
b} Se utiliza la vacuna con siete serotipos

65. ¿Cuál de los siguientes enunciados NO es correcto respecto de la exploración física de


la estenosis aórtica?
e} La administración de nitratos disminuiría laintensidad del soplo

66. Paciente masculino de 64 años acude a la consulta por presentar cuadro de fiebre, tos
y pérdida de peso de dos meses de evolución. En la radiografía de tórax se observa
infiltrado cavitado en segmento apical de lóbulo inferior izquierdo. Mantoux de 3 mm. Se
observan bacilos ácido-alcohol resistentes en esputo.
¿Cuál es la aseveración correcta?
a} Probable tuberculosis pulmonar,se debe iniciar tratamiento conisoniazida, rifampicina
y pirazinamida
67. Paciente masculino de 37 años VIH-positivo acude al servicio de urgencias por
presentar fatiga, pérdida de peso, fiebre y empeoramiento progresivo de la tos de seis
semanas de duración. Asimismo refiere diaforesis nocturna, COMBE-, hace tres años que
le diagnosticaron la infección de VIH, su última cuenta de CD4 fue de 150. Signos vitales:
fiebre 39.5 ºC. En cuanto a la exploración física se encuentra delgado, letárgico, muget
oral, linfadenopatía cervical moderada bilateral, estertores bilaterales audibles en ápices.
Cultivo de esputo positivo para bacilo ácido resistente, PPD y anergia (secundaria a la
inmunosupresión), cultivos que muestran Mycobacterium tuberculosis. Radiografía de
tórax en la que se observa complejo primario de Ghon y consolidación y cavitación en
lóbulo superior derecho, adenopatía traqueal derecha.
El paciente está en tratamiento con antirretrovirales, ¿cuál debe ser el tratamiento a
seguir para la tuberculosis?
d} Rifabutina, isoniazida, pirazinamida por dos meses, después lsoniazida y rifabutina por
siete meses

68. Paciente masculino de 53 años, con antecedente de cáncer


broncopulmonar,actualmente en tratamiento con quimioterapia, acude a Urgencias una
semana después de su quimioterapia por fatiga extrema; refiere que no le ha sido posible
comer bien ni ingerir líquidos, aparte de que el día de ayer vomitó varias veces y ha
presentado diarrea. Ha tomado múltiples analgésicos por cefalea. A la exploración física el
paciente se observa somnoliento, pero con signos vitales normales: FC 68, FR 21, TA
120/70 mm Hg, 36ºC. Se le toman unos electrolitos séricos, los cuales se reportan con
sodio y potasio normales, pero con un BUN de 78 mg/dl y creatinina de 3.7 mg/dl.
68a. ¿Cuál es la causa de la insuficiencia renal del paciente?
d} Hipovolemia por quimioterapia y baja ingesta de líquidos

68b. ¿Cómo se encuentra la relación BUN/Cr en una insuficiencia renal prerrenal?


d} Mayor a 20

68c. ¿En dónde se encuentran cilindros granulosos o "muddy brown"?


c} Necrosis tubular aguda

68d. ¿Cuál de las siguientes no es una indicación de diálisis en un paciente con


insuficiencia renal aguda?
d} Hipocalciemia severa

68e. ¿Qué representa un FENa menor a 1%?

a}Insuficiencia renal prerrenal


69. Paciente masculino de 56 años comienza con tos, hemoptisis, pérdida de peso y
disnea, por lo que acude a consulta. Tiene como antecedentes tabaquismo (a razón de 20
cigarrillos diarios desde hace 20 años), hipertensión (controlada con nifedipino) y padece
enfermedad ácido péptica (para lo cual toma inhibidores de bomba de protones). A la
exploración física se registra FC: 67, FR: 22, Temp: 36 ºC y TA: 130/75 mm Hg. Trae consigo
unos estudios de laboratorio en donde llama la atención hipercalciemia.
69a. ¿Qué subtipo de cáncer broncopulmonar sospecharía usted?
b} Escamoso

69b. ¿Cuál es el cáncer broncogénico más agresivo?


d} Células pequeñas

69c. En relación con el tipo histológico que se sospecha, ¿qué sería común que le
sucediera a este paciente debido a la localización del tumor?
c} Síndrome de vena cava superior

69d. ¿Cuál de los siguientes síndromes paraneoplásicos se asocia al adenocarcinoma?

 b} Osteoartropatía

69e. ¿Cuál es el tratamiento del cáncer broncogénico limitado de células pequeñas?


c} Quimioterapia más radioterapia

70. Paciente masculino de 77 años que acude a consulta por dolor epigástrico. Refiere al
dolor como de tipo sordo, de intensidad 5/1O, que lo ha molestado los últimos meses.
Además refiere que ha perdido 1O kg de peso, sin cambiar hábitos alimenticios. En sus
antecedentes patológicos se refieren múltiples episodios de pancreatitis. En sus APNP el
paciente comenta que es alcohólico de muchos años actualmente rehabilitado, y que al
momento fuma tres a cuatro cigarrillos al día. Al IPAS refiere también que con frecuencia
presenta heces de olor fétido, que curiosamente flotan. Niega la presencia de fiebre,
náusea, vómito, hematoquezia y melena. A la exploración física sus signos vitales son FC
87/min, FR 16/min,T 37.0 C, PA 123/78 mm Hg. No hay anormalidades a la auscultación
cardiaca y pulmonar.A la exploración abdominal hay un dolor leve a la palpación en
epigastrio y mesogastrio. Se ordena una tomografía computarizada y en la consulta de
seguimiento se describe un páncreas reducido de tamaño con calcificaciones. ¿Qué
tratamiento le daría a este paciente?

b} Reemplazo de enzimas pancreáticas

71. Se atiende a un paciente masculino de 70 años que refiere debilidad. Cuenta que
desde el día de ayer se ha sentido más cansado y mantenido completamente en cama.
Actualmente acaba de iniciar quimioterapia y radioterapia, con diagnóstico de linfoma no
Hodgkin. Niega otra sintomatología en el IPAS. A la exploración física sus signos vitales
incluyen FC 101/min, FR 19/min, T 38.0 C y PA 145/82 mm Hg. No se detectan
anormalidades a la exploración física. Se realiza un electrocardiograma, que muestra
ondas T hiperagudas. Se ordenan estudios de laboratorio, que se muestran a
continuación:

¿Cuál es la causa más probable de la hiperpotasemia de este paciente?


d} Lisis de células tumorales

72. Paciente femenino de 66 años que acude a consulta por edema de miembros
inferiores. Refiere que esto inició hace tres días, y ha ido empeorando. Sus antecedentes
patológicos incluyen hipertensión, enfermedad coronaria y estenosis de la arteria renal.
Los medicamentos que ha tomado por varios años son metoprolol, aspirina y
atorvastatina e hidroclorotiazida, aunque recientemente le cambiaron este último por
otro para controlar su presión. A la exploración física sus signos vitales incluyen FC
89/min, FR 17/min, T 37.1 C y PA 150/80 mm Hg. A la auscultación cardiaca se escuchan
S1 y S2 sin agregados, y los campos pulmonares con murmullo vesicular sin agregados. No
hay distensión venosa yugular u organomegalia abdominal. Se palpa edema 2+ en
extremidades inferiores. A continuación se muestran estudios de laboratorio:
¿Cuál sería el medicamento que inició recientemente más probable?

b} Lisinopril

73. ¿Cuál de los siguientes fenómenos relacionados a endocarditis NO tiene una etiología
inmunológica?
e} Manchas de Janeway

74. ¿Cuál es el efecto adverso más grave de los bolos de glucocorticoides?


a} Arritmias cardiacas

75. ¿Cuál de los siguientes hallazgos NO esperaría encontrar en alguien que tenga fiebre
tifoidea?

b} Taquicardia asociada ala fiebre

76. Un recién nacido de 42 semanas presenta al nacimiento salida de líquido amniótico


con meconio y en las primeras horas de vida inicia con dificultad respiratoria y taquipnea.
La radiografía de tórax muestra infiltrados en parche con diafragmas aplanados e
hiperinsuflación pulmonar. En relación con la patología del paciente, señale la respuesta
FALSA:
c} El manejo con surfactante está contraindicado en estos pacientes

77. Un neonato de 20 días de vida es llevado a consulta por presentar masa abdominal. En
el examen físico se observa una masa a nivel de la cicatriz umbilical,sin cambios en las
características de la piel suprayacente, se palpa masa de consistencia blanda, depresible,
de aproximadamente 1 cm de diámetro, no es dolorosa en apariencia. ¿Cuál es el
diagnóstico más probable?
d} Hernia umbilical
78. Paciente masculino de 70 años de edad que se presenta a consulta debido a dificultad
para orinar desde hace aproximadamente una semana. La fuerza del chorro urinario se ha
reducido, sin embargo no hay dificultad para iniciar la micción ni dolor. ¿Cuál de las
siguientes condiciones con mayor probabilidad es la causante de la sintomatología de este
paciente?
e} Disminución de la contractilidad del detrusor

79. Los resultados de laboratorio de un paciente revelan incremento en el tiempo de


sangrado y en el tiempo de tromboplastina parcial, pero con normalidad del conteo
plaquetario y el tiempo de trombina. ¿Cuál es el diagnóstico más probable?
c} Enfermedad de von Willebrand

80. ¿Cuál de los siguientes anticonvulsivantes se asocia con el desarrollo de


hepatotoxicidad y agranulocitosis?
a} Carbamacepina

81. Masculino de 5 años llevado a consulta por presentar exantema maculopapular de


inicio en cara, posteriormente en tronco, extremidades y nalgas, con adenopatía cervical y
retroauricular, refiere la madre que ha presentado fiebre no cuantificada y catarro. El
diagnóstico más probable de este paciente es:
b} Rubéola

82. ¿Cuál de los siguientes fármacos puede producir miocardiopatías?


a} Doxorrubicina

83. Un paciente de 64 años de edad se presenta al departamento de urgencias debido a


que su esposa lo encontró en su cama haciendo comentarios incoherentes. El paciente
menciona que se encuentra en la cámara de diputados y necesita hablar con el
presidente; cada vez que su esposa trataba de dirigirse hacia él, éste se mostraba agresivo
y combativo. En el servicio de urgencias el personal médico es incapaz de obtener una
historia clínica o de completar la exploración física debido a la agitación que presenta el
paciente. Éste se encuentra taquicárdico, con presión de 90/50 mm Hg y temperatura de
40 ºC. El único antecedente de importancia es un pequeño resfriado desde hace tres días;
además, el paciente jamás ha presentado antecedentes psiquiátricos. ¿Cuál de los
siguientes estudios es el más adecuado para determinar la causa del presente trastorno
psiquiátrico del paciente?
d} Hemocultivo, urocultivo y cultivo de esputo

84. Paciente masculino de 44 años de edad que se presenta a consulta externa por
presentar desde hace varios meses calambres frecuentes. A la exploración física presenta
una presión arterial de 200/100 mm Hg. Los estudios de laboratorio revelan un nivel de
sodio de 154 mEq/L, y potasio de 3 mEq/L, así como bicarbonato de 33 mEq/L. ¿Cuál de
los siguientes diagnósticos es el más adecuado para este paciente?
b} Síndrome de Conn

85. Femenino de19 años, acude al servicio médico por referir una secreción transvaginal
bastante líquida, de baja viscosidad, maloliente de color amarillo y gris, espumoso.
Antecedentes: G1 P1.
El agente patógeno causal de la sintomatología de esta paciente es:
c} Tricomonas

86. ¿Cuál de los siguientes marcadores tumorales se eleva en los cánceres testiculares
seminomatosos?
c} P-hormona gonadotropina coriónica

87. Paciente masculino de 56 años de edad se presenta con pérdida de peso, esteatorrea y
malabsorción. Se decide realizar una tomografía computarizada de abdomen la cual revela
una tumoración en la cabeza del páncreas. La biopsia de la lesión indica inflamación y
atrofia de los acinos pancreáticos con un incremento en el depósito de colágena;sin
embargo, no se encuentran datos relacionados con malignidad. ¿Cuál es la causa más
común de esta descripción histológica?
e} Alcoholismo crónico

88. ¿Cuál de las siguientes neoplasias en adultos se presenta principalmente en pacientes


con neurofibromatosis?
c} Schwannoma

89. Masculino de 32 años que acude al servicio de consulta externa, presenta prurito y
lagrimeo en ambos ojos, hiperemia conjuntival, fotofobia, exudado más o menos viscoso y
formación papilar en la conjuntiva tarsal. El diagnóstico más probable en este paciente es:
b} Conjuntivitis alérgica

90. ¿Cuál de los siguientes hallazgos se asocia con el glucagonoma pancreático?


e} Eritema migratorio necrolítico
91. Se presenta a consulta un paciente del sexo masculino de cuatro meses de edad. Los
padres llevan al paciente porque presenta secreción acuosa por la nariz y tos desde las
últimas 72 horas. A la exploración física el paciente muestra retracción intercostal y
estertores bilaterales; la radiografía muestra infiltrados difusos. La enfermera reporta una
frecuencia respiratoria de 45 respiraciones por minuto y una frecuencia cardiaca de 130
latidos por minuto. Entre los antecedentes de importancia la madre menciona que el niño
fue producto de un embarazo a término y parto eutócico. ¿Cuál de las siguientes opciones
representa la principal vía por la que este paciente adquirió esta infección?
a} Durante elparto

92. ¿Cuál de las siguientes enfermedades se debe a una acumulación de aminoácidos


ramificados (isoleucina, valina, leucina)?
d} Enfermedad de orina de jarabe de maple

93. ¿Cuál de las siguientes neoplasias malignas se asocia con síndrome de secreción
inadecuada de hormona antidiurética?
b} Neoplasias intracraneales

94. Una mujer de 24 años refiere que sus manos se tornan blancas y luego azules en el
frío. Como lo muestra la fotografía. El dato clínico que apoya el diagnóstico de esta
paciente es:
a} Engrosamiento cutáneo distal que se extiende en dirección proximal hasta las
articulaciones metacarpofalángicas

95. Paciente de 35 años de edad ingresa al servicio de urgencias por disnea. Se presenta
disneico con uso de músculos respiratorios, con hipoxia grave que requiere administración
de oxígeno a 100%. La radiografía de tórax muestra zonas de "vidrio despulido". ¿Cuál de
las siguientes condiciones es con mayor probabilidad la enfermedad de base responsable
del cuadro clínico en este paciente?
a} Infección por el virus de la inmunodeficiencia humana
96. ¿Cuál es la función de la vitamina deficiente en el síndrome de Hartnup?
b} Coenzima parala gliceraldehido-3-fosfato deshidrogenasa

97. Paciente del sexo femenino de 18 años de edad la cual se presenta al servicio de
urgencias por un cuadro de 24 horas de evolución de fiebre, vómito, diarrea y mialgias. A
la exploración física se encuentra una presión arterial de 90/60 mm Hg, y frecuencia
cardiaca de 125 latidos/minuto. A la inspección la paciente presenta eritrodermia y la
exploración pélvica revela un tampón en vagina. ¿Cuál de las siguientes células presenta
una activación excesiva como parte central de la fisiopatología de esta paciente?"
c} Macrófagos y linfocitos T

98. Masculino de 52 años acude a tu consultorio con resultado de laboratorio: título de


antígeno prostático de 72 ng/dl.
La siguiente medida para corroborar el diagnóstico de este paciente es:
d} Biopsia con aguja guiada por USG

99. ¿Cuál de las siguientes hormonas es la responsable del desarrollo de diabetes


gestacional?
e} Lactógeno placentario humano

1OO. ¿Cuál de las siguientes alteraciones electrolíticas se asocia con depresión del
segmento ST?
e} Hipopotasemia

101. ¿Cuál es el mecanismo de acción de las penicilinas?

c}Inhibición dela síntesis dela pared

102. ¿Cuál de las siguientes hormonas gastrointestinales se encarga de estimular la


secreción enzimática del páncreas, la contracción de la vesícula biliar y de inhibir la
secreción ácida gástrica?
e} Colecistocinina

103. ¿Cuáles son los fármacos que han demostrado modificar la evolución de la artritis
reumatoide?
c} Anti-TNF/lnmunosupresores

104. ¿En cuál de las siguientes condiciones se observa el signo de Kussmaul?


c} Tromboembolia pulmonar

105. ¿Cuál de las siguientes arterias participa en la irrigación de la vejiga?


c} Arteria iliaca interna
106. Paciente de 65 años de edad acude a la consulta externa por disnea progresiva y tos
crónica. Entre los antecedentes de importancia se sabe que el paciente trabajó durante
más de 30 años en una fundidora. Se realizan pruebas de función pulmonar,las cuales
muestran un patrón restrictivo; una radiografía de tórax muestra infiltrados
micronodulares en ambos ápices, con adenopatía hiliar calcificada. ¿Cuál es el diagnóstico
más probable?
a} Silicosis

107. La vancomicina es un antibiótico bactericida para la mayoría de los organismos


grampositivos, en particular estafilococos y estreptococos, y bacteriostático para la
mayoría de los enterococos. Este fármaco no se absorbe por vía oral,aunque se puede
utilizar para el tratamiento de la enterocolitis asociada con antibióticos. ¿Cuál de los
siguientes enunciados tiene mejor relación con el mecanismo de acción de la
vancomicina?
e} Inhibición de la pared celular

108. Masculino de 66 años que acude al servicio de urgencias por presentar cuadro de
dolor lumbar derecho y fiebre de 38 C. Sin antecedentes familiares, ni patológicos de
importancia. A la exploración física destaca dolor generalizado en el hemiabdomen
derecho, sin defensa ni datos de peritonismo. Giordano derecho.
108a. ¿Cuál de los siguientes procedimientos resultaría menos adecuado?
c} Realizar sondaje vesical para descartar uropatía obstructiva infravesical

108b. Tras las exploraciones adecuadas, se detecta una dilatación del sistema excretor
derecho causada por una litiasis radiolúcida localizada en el uréter derecho a nivel de L3.
En el transcurso de estos procedimientos, el paciente comienza a presentar hipotensión.
La biometría hemática revela leucocitos: 2.5/mm3, hemoglobina de 9 g/dl.
Lo siguiente que procedería sería:
e} Colocar una nefrostomía o un catéter doble J, una vez estabilizado elpaciente

109. Paciente femenino de 60 años de edad. Acude a consulta por presentar salida de
orina sin poder controlarla desde hace dos meses. Refiere que se presenta cuando tose y
se ríe. Tuvo ocho embarazos, todos nacieron vía vaginal. No recuerda si hubo
complicaciones durante el parto. ¿Cuál de los siguientes enunciados es correcto?
a} La incontinencia de esfuerzo puede ser causada por hipermovilidad uretral

11O. Femenino de 34 años con referencia por diagnóstico de lupus eritematoso


generalizado.
El dato clínico incluido en los criterios que corrobora el diagnóstico de este paciente es:
b} Fotosensibilidad
111. Paciente femenino de 25 años que acude a consulta por sospecha de embarazo,
refiere un retraso en el ciclo menstrual de 25 días, con clínica sugerente de embarazo. Se
solicita prueba de laboratorio. Usted sabe que las hormonas esteroideas son muy
importantes durante el embarazo. ¿Qué hormona se encarga de mantener el cuerpo
lúteo?

e} Gonadotropina coriónica humana

112. Ante una mujer con tipo sanguíneo AB-, cursando su segundo embarazo en la
semana 27 de gestación, con prueba de Coombs indirecto negativo, ¿cuál es la conducta
más adecuada?
c} Administrar anti-lgM en este momento y después del nacimiento

113. Estado de la República Mexicana con menor prevalencia de DM2 (ENSA 2000):
a} Guanajuato

114. ¿Cuál de las siguientes condiciones es la principal causante de los divertículos


uretrales?
d}Infecciones bacterianas

115. ¿Cuál de las siguientes opciones representa el criterio diagnóstico de intolerancia a la


glucosa?
c} Glucemia 2 horas poscarga de glucosa vía oral de 140 a 200 mg/dl

116. Niño de tres años de edad, acude por dermatosis diseminada, bilateral y asimétrica
que afecta axilas, ingle y la fosa poplítea derecha, constituida por pápulas umbilicadas, en
forma de domo, translúcidas y umbilidacas, bien delimitadas. Refiere haber visto una
desde hace un año y posteriormente le han ido saliendo más, existe el antecedente de
tener dermatitis atópica. ¿Cuál es el diagnóstico más probable?
a} Molusco contagioso

117. ¿A qué se refiere el "efecto Somogyi"?


a} Hiperglucemia matutina en respuesta a hipoglucemia en la madrugada,la cual causa
liberación de hormonas contrarreguladoras

118. Paciente masculino con antecedente de hipertensión manejado con metoprolol e


hidroclorotiazida. Recientemente se diagnosticó con osteoartritis, por lo que inició
tratamiento con indometacina. Refiere iniciar posteriormente con un cuadro diarreico
acompañado de malestar general e hipotensión ortostática. El laboratorio reporta BUN:
65 mg/dl, creatinina sérica: 2.8 mg/dl,sodio: 130 meq/L, potasio: 5.9 mEq/L. Se sospecha
lesión renal aguda.
118b. Señale el patrón de laboratorio que se espera encontrar al presentarse lesión renal
aguda prerrenal:
d} FE Na < 1%, OsmU > 500 mOsm, NaU < 20 mEq/L"

11Bc. Señale el enunciado correcto respecto a la relación BUN/creatinina en la lesión renal


aguda
a} Una relación > 20:1orienta a causa prerrenal

118d. La oliguria se define como:


d} < 400 mlde orina en 24 h

119. Se encuentra usted rotando en un hospital maternoinfantil y se presenta a consulta


una mujer de 22 años de edad con nueve semanas de amenorrea secundaria. Niega
antecedentes de interés. Se decide tomar fracción beta de hormona gonadotropina
coriónica humana, la cual presenta títulos elevados. Después se indica ultrasonografía
transvaginal y no se halla ocupación uterina, pero en tuba izquierda se encuentra imagen
de anillo. ¿Cuál de los siguientes no es un factor de riesgo para presentar esta
enfermedad?
a} Cesárea previa

120. Paciente masculino de 54 años de edad. Antecedente de fiebre reumática. Acude por
presentar palpitaciones y disnea al esfuerzo durante varios meses, lo cual ha empeorado
las últimas semanas, por lo que acude a consultar.TA: 150/60, FC: 100 lpm, Temp: 36.5 ºC.
Pulso fuerte y colapsante. Soplo diastólico decreciente que predomina en tercer espacio
intercostal borde paraesternal izquierdo. Se observan pulsaciones del lecho capilar de los
dedos.
120a. ¿Cuál es el diagnóstico clínico en este paciente?
c} Insuficiencia aórtica

120b. ¿Qué nombre reciben las pulsaciones del lecho capilar en los dedos que presenta el
paciente?
d} Signo de Quincke

120c. En la insuficiencia aórtica a menudo se ausculta un soplo mesodiastólico suave


localizado en la punta con carácter de retumbo. A este soplo se le conoce como:
b} Soplo de Austin-Flint

120d. El estudio de gabinete ideal para continuar con el estudio del paciente es:
c} Ecocardiograma

120e. ¿Cuál de los siguientes hallazgos en la exploración orientan más hacia el diagnóstico
de insuficiencia aórtica?
a} Aumento de laintensidad del soplo al apretar los puños

121. Paciente masculino de 50 años de edad acude al servicio de urgencias después de


haber padecido cuatro episodios de vómito sanguinolento. Jamás había presentado
episodios de hematemesis. A la exploración física presenta presión arterial de 100/70 mm
Hg en posición supina y de 70/45 mm Hg de pie, con una frecuencia cardiaca de 102
latidos/min, frecuencia respiratoria de 20 respiraciones/min,y saturación de oxígeno
cercana al 100%. A la inspección, está pálido, escleras ictéricas, telangiectasias en el
pecho. A la palpación se puede encontrar esplenomegalia, y a la percusión, el abdomen es
en su mayoría mate. Los estudios de laboratorio revelan hematocrito de 29%, tiempo de
protrombina de 33 segundos y un índice de normalización internacional de 3.3. El paciente
es estabilizado y se realiza una endoscopia, la cual revela múltiples várices grado 3 en el
esófago. Una de las várices tiene un tapón de fibrina sobre lo que parece ser una rotura de
la mucosa. ¿Cuál de las siguientes opciones es el mejor tratamiento en este paciente?

a} Ligadura de las várices

122. Paciente masculino de 65 años de edad con factores de riesgo cardiovascular se


presenta a la sala de urgencias con hemiplejía derecha, así como parálisis facial derecha
con preservación de los movimientos de la mitad derecha de la frente. El paciente
también muestra dificultad para la comunicación oral. El paciente se encuentra alerta con
frecuencia cardiaca de 88 latidos por minuto y presión arterial de 175/115 mm Hg, sin
dificultad ventilatoria y con una cifra de glucosa capilar de 105 mg/dL. Los familiares
refieren que el cuadro inició hace aproximadamente 1 hora. ¿Cuál es la conducta
inmediata más apropiada?"

a} Tomografía axial computarizada no contrastada

123. Es una causa de hipotirodismo central o secundario:

a} Síndrome de Sheehan

124. Acude a consulta de rutina un paciente masculino de 30 meses de edad. La madre


está preocupada porque ha observado que su hijo se muestra más cansado de lo normal',
presentando disnea en ocasiones cuando juega con otros niños de su misma edad. Al
interrogatorio dirigido, la madre niega cualquier antecedente pediátrico u obstétrico de
importancia. A la exploración física el paciente se presenta taquicárdico, con palidez de
mucosas, escleras ligeramente ictéricas, así como incremento en el diámetro abdominal a
expensas de una importante esplenomegalia. Se solicitan estudios de laboratorio los
cuales son relevantes por una anemia normocítica normocrómica. El frotis de sangre
periférica reporta reticulocitos espiculados. ¿Cuál es el diagnóstico más probable en este
paciente?
a} Deficiencia de piruvatocinasa

125. Un paciente de 55 años presenta dolor en ambas extremidades inferiores,


parestesias y alteraciones sensitivas altacto en las regiones plantares de ambos pies, y
nicturia con incontinencia en ocasiones. El paciente menciona que ha notado que
recientemente su marcha es inestable y se siente "inseguro" al caminar. ¿Cuál de las
siguientes características más probablemente también presenta este paciente?

a} Pupilas con reflejo de acomodación pero que no responden a la estimulaciónluminosa

126. Un paciente masculino de 75 años de edad con antecedentes de hipercolesterolemia,


hipertensión arterial de cinco años de evolución, diabetes mellitus tipo 2 de cinco años de
diagnóstico y fibrilación auricular se presenta a la sala de urgencias por dolor abdominal
de inicio súbito, acompañado de náusea, vómito y febrícula. Se realiza una angiografía la
cual revela un bloqueo completo de la arteria mesentérica superior,con permeabilidad del
tronco celiaco y la arteria mesentérica inferior. ¿Cuál de las siguientes estructuras
presentaría isquemia en este paciente?

a} Región cecal

127. Paciente masculino de 18 años, proveniente de una zona de pobreza extrema,


tabaquismo negativo. Ha presentado tos no productiva en un inicio que después se
convirtió en tos en accesos con expectoración amarillenta, densa y estrías sanguinolentas
durante más de 20 días, en realidad no le dio importancia, pero comenzó con disnea de
bajos esfuerzos, por lo que acude al servicio de urgencias. A la exploración física muestra
fiebre, desnutrición, estertores en ápices del pulmón derecho y disminución de los ruidos
respiratorios en base de pulmón derecho. Se toma radiografía de tórax en la que se
observa derrame pleural derecho y una cavitación con nivel hidroaéreo en ápices
pulmonares. Se diagnóstica pleuritis tuberculosa.

¿Cuál de las siguientes muestras de líquido pleural se asocia a tuberculosis pleural?

a}

128. Paciente masculino de 4 años de edad, se presenta con historia de estreñimiento


desde los 6 meses de nacido. Su familiar describe sus hábitos intestinales como ""pocos a
la semana""; ya con el interrogatorio dirigido, se habla de evacuaciones cada 3 a 4 días, de
características largas y de consistencia dura, no se mencionan otros síntomas. La
exploración física es normal;el tacto rectal revela una ámpula grande, tono del esfínter
pobre y presencia de heces en la ampolla rectal. 128a. Con base en las características
del paciente, ¿cuál es su sospecha clínica?
b} Estreñimiento funcional
128c. ¿Cuál es la causa más común de estreñimiento en los niños?

c} Retención voluntaria

128e. ¿Qué tratamiento indicaría si el paciente se presenta con impacción?

d} Todas las anteriores

129. Paciente femenino de 17 años es traída a urgencias por sufrir quemaduras en ambos
miembros inferiores al caer de rodillas en una tina con agua hirviendo. Usted observa que
únicamente la piel de la parte anterior y posterior de ambos muslos, piernas y pies
presenta quemaduras de segundo grado de espesor parcial, muy dolorosas, con eritema,
edema y ampollas. La enfermera le indica que la paciente pesa 50 kg, tiene ya insertadas
dos líneas periféricas de calibre 16 y espera su indicación para iniciar la reposición de
líquidos.
¿Cuál es el requerimiento de líquidos en las primeras 24 horas de esta paciente con
quemaduras?
c} 7.2 L de Ringer lactato

130. Masculino de 12 años de edad es llevado a consulta externa por presentar fiebre de 8
h de evolución, malestar generalizado y erupción generalizada en la piel. La madre
comenta que esta erupción inició en tronco y que posteriormente se extendió a cara y
miembros pélvicos. A la exploración física se observan lesiones vesiculosas de predominio
en tronco, mientras que en la cara y extremidades son escasas; dichas lesiones son de tipo
macular, papular y vesicular. El paciente refiere pruritointenso.
130a. Con baseen las características del paciente, ¿cuál es el diagnóstico más probable?

a} Varicela

130b. De acuerdo con su sospecha clínica, ¿cuál es el agente etiológico del padecimiento?

d} Varicela zóster
130c. En relación con su sospecha clínica, ¿qué complicación es factible que se presente?

b}Infección bacteriana secundaria

130d. De acuerdo con su sospecha clínica, ¿cuáles son los hallazgos que esperaría
encontrar en los exámenes complementarios?

d} Leucocitos normales

130e. Con base en su sospecha clínica, ¿cuál es el tratamiento indicado para este
paciente?
a} Paracetamol,difenhidramina, uñas cortas y antibióticos tópicos

131. Neonato de cuatro días de vida con aspecto de ambos pies como "zambos". ¿Cuáles
de los siguientes defectos integran las deformidades del pie equino varo?

1. Desviación medial del empeine con convexidad del borde lateral del pie
2. Flexión plantar del tobillo
3. Inclinación medial de la planta del pie
4. Elevación de la bóveda plantar
5. Aplanamiento del arco longitudinal medial al soportar peso
6. lncurvación medial de los metatarsianos.

e} 2, 3 y 6

132. ¿Cuál de las siguientes drogas produce un síndrome de abstinencia caracterizado por
ansiedad de rebote, crisis convulsivas, temblor e insomnio?

e} Benzodiacepinas

1.Cual es el paso limitante en la esteroidogénesis en pacientes con insuficiencia


suprarrenal?
a)La conversión de pregnenolona en progesterona

También podría gustarte